IMO Problem and Shortlist PDF
IMO Problem and Shortlist PDF
IMO Problem and Shortlist PDF
1959/1.
21n+4
Prove that the fraction 14n+3
is irreducible for every natural number n.
1959/2.
For what real values of x is
q √ q √
(x + 2x − 1) + (x − 2x − 1) = A,
√
given (a) A = 2, (b) A = 1, (c) A = 2, where only non-negative real
numbers are admitted for square roots?
1959/3.
Let a, b, c be real numbers. Consider the quadratic equation in cos x :
a cos2 x + b cos x + c = 0.
Using the numbers a, b, c, form a quadratic equation in cos 2x, whose roots
are the same as those of the original equation. Compare the equations in
cos x and cos 2x for a = 4, b = 2, c = −1.
1959/4.
Construct a right triangle with given hypotenuse c such that the median
drawn to the hypotenuse is the geometric mean of the two legs of the triangle.
1959/5.
An arbitrary point M is selected in the interior of the segment AB. The
squares AM CD and M BEF are constructed on the same side of AB, with
the segments AM and M B as their respective bases. The circles circum-
scribed about these squares, with centers P and Q, intersect at M and also
at another point N. Let N 0 denote the point of intersection of the straight
lines AF and BC.
(a) Prove that the points N and N 0 coincide.
(b) Prove that the straight lines M N pass through a fixed point S indepen-
dent of the choice of M.
(c) Find the locus of the midpoints of the segments P Q as M varies between
A and B.
1959/6.
Two planes, P and Q, intersect along the line p. The point A is given in the
plane P, and the point C in the plane Q; neither of these points lies on the
straight line p. Construct an isosceles trapezoid ABCD (with AB parallel to
CD) in which a circle can be inscribed, and with vertices B and D lying in
the planes P and Q respectively.
Second International Olympiad, 1960
1960/1.
Determine all three-digit numbers N having the property that N is divisible
by 11, and N/11 is equal to the sum of the squares of the digits of N.
1960/2.
For what values of the variable x does the following inequality hold:
4x2
√ < 2x + 9?
(1 − 1 + 2x)2
1960/3.
In a given right triangle ABC, the hypotenuse BC, of length a, is divided
into n equal parts (n an odd integer). Let α be the acute angle subtending,
from A, that segment which contains the midpoint of the hypotenuse. Let h
be the length of the altitude to the hypotenuse of the triangle. Prove:
4nh
tan α = 2 .
(n − 1)a
1960/4.
Construct triangle ABC, given ha , hb (the altitudes from A and B) and ma ,
the median from vertex A.
1960/5.
Consider the cube ABCDA0 B 0 C 0 D0 (with face ABCD directly above face
A0 B 0 C 0 D0 ).
(a) Find the locus of the midpoints of segments XY, where X is any point
of AC and Y is any point of B 0 D0 .
(b) Find the locus of points Z which lie on the segments XY of part (a) with
ZY = 2XZ.
1960/6.
Consider a cone of revolution with an inscribed sphere tangent to the base
of the cone. A cylinder is circumscribed about this sphere so
that one of its bases lies in the base of the cone. Let V1 be the volume of the
cone and V2 the volume of the cylinder.
(a) Prove that V1 6= V2 .
(b) Find the smallest number k for which V1 = kV2 , for this case, construct
the angle subtended by a diameter of the base of the cone at the vertex of
the cone.
1960/7.
An isosceles trapezoid with bases a and c and altitude h is given.
(a) On the axis of symmetry of this trapezoid, find all points P such that
both legs of the trapezoid subtend right angles at P.
(b) Calculate the distance of P from either base.
(c) Determine under what conditions such points P actually exist. (Discuss
various cases that might arise.)
Third International Olympiad, 1961
1961/1.
Solve the system of equations:
x+y+z = a
x + y 2 + z 2 = b2
2
xy = z 2
where a and b are constants. Give the conditions that a and b must satisfy
so that x, y, z (the solutions of the system) are distinct positive numbers.
1961/2. √
Let a, b, c be the sides of a triangle, and T its area. Prove: a2 +b2 +c2 ≥ 4 3T.
In what case does equality hold?
1961/3.
Solve the equation cosn x − sinn x = 1, where n is a natural number.
1961/4.
Consider triangle P1 P2 P3 and a point P within the triangle. Lines P1 P, P2 P, P3 P
intersect the opposite sides in points Q1 , Q2 , Q3 respectively. Prove that, of
the numbers
P1 P P2 P P3 P
, ,
P Q1 P Q2 P Q 3
at least one is ≤ 2 and at least one is ≥ 2.
1961/5.
Construct triangle ABC if AC = b, AB = c and 6 AM B = ω, where M is
the midpoint of segment BC and ω < 90◦ . Prove that a
solution exists if and only if
ω
b tan ≤ c < b.
2
In what case does the equality hold?
1961/6.
Consider a plane ε and three non-collinear points A, B, C on the same side of
ε; suppose the plane determined by these three points is not parallel to ε. In
plane a take three arbitrary points A0 , B 0 , C 0 . Let L, M, N be the midpoints
of segments AA0 , BB 0 , CC 0 ; let G be the centroid of triangle LM N. (We will
not consider positions of the points A0 , B 0 , C 0 such that the points L, M, N
do not form a triangle.) What is the locus of point G as A0 , B 0 , C 0 range
independently over the plane ε?
Fourth International Olympiad, 1962
1962/1.
Find the smallest natural number n which has the following properties:
(a) Its decimal representation has 6 as the last digit.
(b) If the last digit 6 is erased and placed in front of the remaining digits,
the resulting number is four times as large as the original number n.
1962/2.
Determine all real numbers x which satisfy the inequality:
√ √ 1
3−x− x+1> .
2
1962/3.
Consider the cube ABCDA0 B 0 C 0 D0 (ABCD and A0 B 0 C 0 D0 are the upper and
lower bases, respectively, and edges AA0 , BB 0 , CC 0 , DD0 are parallel). The
point X moves at constant speed along the perimeter of the square ABCD
in the direction ABCDA, and the point Y moves at the same rate along
the perimeter of the square B 0 C 0 CB in the direction B 0 C 0 CBB 0 . Points X
and Y begin their motion at the same instant from the starting positions A
and B 0 , respectively. Determine and draw the locus of the midpoints of the
segments XY.
1962/4.
Solve the equation cos2 x + cos2 2x + cos2 3x = 1.
1962/5.
On the circle K there are given three distinct points A, B, C. Construct (using
only straightedge and compasses) a fourth point D on K such that a circle
can be inscribed in the quadrilateral thus obtained.
1962/6.
Consider an isosceles triangle. Let r be the radius of its circumscribed circle
and ρ the radius of its inscribed circle. Prove that the distance d between
the centers of these two circles is
q
d= r(r − 2ρ).
1962/7.
The tetrahedron SABC has the following property: there exist five spheres,
each tangent to the edges SA, SB, SC, BCCA, AB, or to their extensions.
(a) Prove that the tetrahedron SABC is regular.
(b) Prove conversely that for every regular tetrahedron five such spheres
exist.
Fifth International Olympiad, 1963
1963/1.
Find all real roots of the equation
q √
x2 − p + 2 x2 − 1 = x,
1963/2.
Point A and segment BC are given. Determine the locus of points in space
which are vertices of right angles with one side passing through A, and the
other side intersecting the segment BC.
1963/3.
In an n-gon all of whose interior angles are equal, the lengths of consecutive
sides satisfy the relation
a1 ≥ a2 ≥ · · · ≥ an .
Prove that a1 = a2 = · · · = an .
1963/4.
Find all solutions x1 , x2 , x3 , x4 , x5 of the system
x5 + x2 = yx1
x1 + x3 = yx2
x2 + x4 = yx3
x3 + x5 = yx4
x4 + x1 = yx5 ,
where y is a parameter.
1963/5.
Prove that cos π7 − cos 2π
7
+ cos 3π
7
= 12 .
1963/6.
Five students, A, B, C, D, E, took part in a contest. One prediction was that
the contestants would finish in the order ABCDE. This prediction was very
poor. In fact no contestant finished in the position predicted, and no two
contestants predicted to finish consecutively actually did so. A second pre-
diction had the contestants finishing in the order DAECB. This prediction
was better. Exactly two of the contestants finished in the places predicted,
and two disjoint pairs of students predicted to finish consecutively actually
did so. Determine the order in which the contestants finished.
Sixth International Olympiad, 1964
1964/1.
(a) Find all positive integers n for which 2n − 1 is divisible by 7.
(b) Prove that there is no positive integer n for which 2n + 1 is divisible by
7.
1964/2.
Suppose a, b, c are the sides of a triangle. Prove that
a2 (b + c − a) + b2 (c + a − b) + c2 (a + b − c) ≤ 3abc.
1964/3.
A circle is inscribed in triangle ABC with sides a, b, c. Tangents to the circle
parallel to the sides of the triangle are constructed. Each of these tangents
cuts off a triangle from ∆ABC. In each of these triangles, a circle is inscribed.
Find the sum of the areas of all four inscribed circles (in terms of a, b, c).
1964/4.
Seventeen people correspond by mail with one another - each one with all
the rest. In their letters only three different topics are discussed. Each pair
of correspondents deals with only one of these topics. Prove that there are
at least three people who write to each other about the same topic.
1964/5.
Suppose five points in a plane are situated so that no two of the straight lines
joining them are parallel, perpendicular, or coincident. From each point per-
pendiculars are drawn to all the lines joining the other four points. Determine
the maximum number of intersections that these perpendiculars can have.
1964/6.
In tetrahedron ABCD, vertex D is connected with D0 the centroid of ∆ABC.
Lines parallel to DD0 are drawn through A, B and C. These lines intersect the
planes BCD, CAD and ABD in points A1 , B1 and C1 , respectively. Prove
that the volume of ABCD is one third the volume of A1 B1 C1 D0 . Is the result
true if point D0 is selected anywhere within ∆ABC?
Seventh Internatioaal Olympiad, 1965
1965/1.
Determine all values x in the interval 0 ≤ x ≤ 2π which satisfy the inequality
¯√ √ ¯ √
2 cos x ≤ ¯¯ 1 + sin 2x − 1 − sin 2x¯¯ ≤ 2.
1965/2.
Consider the system of equations
1965/3.
Given the tetrahedron ABCD whose edges AB and CD have lengths a and
b respectively. The distance between the skew lines AB and CD is d, and
the angle between them is ω. Tetrahedron ABCD is divided into two solids
by plane ε, parallel to lines AB and CD. The ratio of the distances of ε from
AB and CD is equal to k. Compute the ratio of the volumes of the two solids
obtained.
1965/4.
Find all sets of four real numbers x1 , x2 , x3 , x4 such that the sum of any one
and the product of the other three is equal to 2.
1965/5.
Consider ∆OAB with acute angle AOB. Through a point M 6= O perpendic-
ulars are drawn to OA and OB, the feet of which are P and Q respectively.
The point of intersection of the altitudes of ∆OP Q is H. What is the locus
of H if M is permitted to range over (a) the side AB, (b) the interior of
∆OAB?
1965/6.
In a plane a set of n points (n ≥ 3) is given. Each pair of points is connected
by a segment. Let d be the length of the longest of these segments. We define
a diameter of the set to be any connecting segment of length d. Prove that
the number of diameters of the given set is at most n.
Eighth International Olympiad, 1966
1966/1.
In a mathematical contest, three problems, A, B, C were posed. Among the
participants there were 25 students who solved at least one problem each.
Of all the contestants who did not solve problem A, the number who solved
B was twice the number who solved C. The number of students who solved
only problem A was one more than the number of students who solved A
and at least one other problem. Of all students who solved just one problem,
half did not solve problem A. How many students solved only problem B?
1966/2.
Let a, b, c be the lengths of the sides of a triangle, and α, β, γ, respectively,
the angles opposite these sides. Prove that if
γ
a + b = tan (a tan α + b tan β),
2
the triangle is isosceles.
1966/3.
Prove: The sum of the distances of the vertices of a regular tetrahedron from
the center of its circumscribed sphere is less than the sum of the distances of
these vertices from any other point in space.
1966/4.
Prove that for every natural number n, and for every real number x 6=
kπ/2t (t = 0, 1, ..., n; k any integer)
1 1 1
+ + ··· + n
= cot x − cot 2n x.
sin 2x sin 4x sin 2 x
1966/5.
Solve the system of equations
1967/3.
Let k, m, n be natural numbers such that m + k + 1 is a prime greater than
n + 1. Let cs = s(s + 1). Prove that the product
(cm+1 − ck )(cm+2 − ck ) · · · (cm+n − ck )
is divisible by the product c1 c2 · · · cn .
1967/4.
Let A0 B0 C0 and A1 B1 C1 be any two acute-angled triangles. Consider all
triangles ABC that are similar to ∆A1 B1 C1 (so that vertices A1 , B1 , C1 cor-
respond to vertices A, B, C, respectively) and circumscribed about triangle
A0 B0 C0 (where A0 lies on BC, B0 on CA, and AC0 on AB). Of all such
possible triangles, determine the one with maximum area, and construct it.
1967/5.
Consider the sequence {cn }, where
c1 = a1 + a2 + · · · + a8
c2 = a21 + a22 + · · · + a28
···
cn = an1 + an2 + · · · + an8
···
in which a1 , a2 , · · · , a8 are real numbers not all equal to zero. Suppose that
an infinite number of terms of the sequence {cn } are equal to zero. Find all
natural numbers n for which cn = 0.
1967/6.
In a sports contest, there were m medals awarded on n successive days (n >
1). On the first day, one medal and 1/7 of the remaining m − 1 medals
were awarded. On the second day, two medals and 1/7 of the now remaining
medals were awarded; and so on. On the n-th and last day, the remaining n
medals were awarded. How many days did the contest last, and how many
medals were awarded altogether?
Tenth International Olympiad, 1968
1968/1.
Prove that there is one and only one triangle whose side lengths are consec-
utive integers, and one of whose angles is twice as large as another.
1968/2.
Find all natural numbers x such that the product of their digits (in decimal
notation) is equal to x2 − 10x − 22.
1968/3.
Consider the system of equations
ax21 + bx1 + c = x2
ax22 + bx2 + c = x3
···
2
axn−1 + bxn−1 + c = xn
ax2n + bxn + c = x1 ,
with unknowns x1 , x2 , · · · , xn , where a, b, c are real and a 6= 0. Let ∆ =
(b − 1)2 − 4ac. Prove that for this system
(a) if ∆ < 0, there is no solution,
(b) if ∆ = 0, there is exactly one solution,
(c) if ∆ > 0, there is more than one solution.
1968/4.
Prove that in every tetrahedron there is a vertex such that the three edges
meeting there have lengths which are the sides of a triangle.
1968/5.
Let f be a real-valued function defined for all real numbers x such that, for
some positive constant a, the equation
1 q
f (x + a) = + f (x) − [f (x)]2
2
holds for all x.
(a) Prove that the function f is periodic (i.e., there exists a positive number
b such that f (x + b) = f (x) for all x).
(b) For a = 1, give an example of a non-constant function with the required
properties.
1968/6.
For every natural number n, evaluate the sum
∞
" # · ¸ · ¸ " #
X n + 2k n+1 n+2 n + 2k
= + + ··· + + ···
k=0 2k+1 2 4 2k+1
(The symbol [x] denotes the greatest integer not exceeding x.)
Eleventh International Olympiad, 1969
1969/1.
Prove that there are infinitely many natural numbers a with the following
property: the number z = n4 + a is not prime for any natura1 number n.
1969/2.
Let a1 , a2 , · · · , an be real constants, x a real variable, and
1 1
f (x) = cos(a1 + x) + cos(a2 + x) + cos(a3 + x)
2 4
1
+··· + cos(an + x).
2n−1
Given that f (x1 ) = f (x2 ) = 0, prove that x2 − x1 = mπ for some integer m.
1969/3.
For each value of k = 1, 2, 3, 4, 5, find necessary and sufficient conditions on
the number a > 0 so that there exists a tetrahedron with k edges of length
a, and the remaining 6 − k edges of length 1.
1969/4.
A semicircular arc γ is drawn on AB as diameter. C is a point on γ other
than A and B, and D is the foot of the perpendicular from C to AB. We
consider three circles, γ1 , γ2 , γ3 , all tangent to the line AB. Of these, γ1 is
inscribed in ∆ABC, while γ2 and γ3 are both tangent to CD and to γ, one on
each side of CD. Prove that γ1 , γ2 and γ3 have a second tangent in common.
1969/5.
Given n > 4 points³ in ´the plane such that no three are collinear. Prove that
there are at least n−3
2
convex quadrilaterals whose vertices are four of the
given points.
1969/6.
Prove that for all real numbers x1 , x2 , y1 , y2 , z1 , z2 , with x1 > 0, x2 > 0, x1 y1 −
z12 > 0, x2 y2 − z22 > 0, the inequality
8 1 1
2 ≤ +
(x1 + x2 ) (y1 + y2 ) − (z1 + z2 ) x1 y1 − z1 x2 y2 − z22
2
1970/6.
In a plane there are 100 points, no three of which are collinear. Consider all
possible triangles having these points as vertices. Prove that no more than
70% of these triangles are acute-angled.
Thirteenth International Olympiad, 1971
1971/1.
Prove that the following assertion is true for n = 3 and n = 5, and that it is
false for every other natural number n > 2 :
If a1 , a2 , ..., an are arbitrary real numbers, then
(a1 − a2 )(a1 − a3 ) · · · (a1 − an ) + (a2 − a1 )(a2 − a3 ) · · · (a2 − an )
+ · · · + (an − a1 )(an − a2 ) · · · (an − an−1 ) ≥ 0
1971/2.
Consider a convex polyhedron P1 with nine vertices A1 A2 , ..., A9 ; let Pi be
the polyhedron obtained from P1 by a translation that moves vertex A1 to
Ai (i = 2, 3, ..., 9). Prove that at least two of the polyhedra P1 , P2 , ..., P9 have
an interior point in common.
1971/3.
Prove that the set of integers of the form 2k − 3(k = 2, 3, ...) contains an
infinite subset in which every two members are relatively prime.
1971/4.
All the faces of tetrahedron ABCD are acute-angled triangles. We consider
all closed polygonal paths of the form XY ZT X defined as follows: X is a
point on edge AB distinct from A and B; similarly, Y, Z, T are interior points
of edges BCCD, DA, respectively. Prove:
(a) If 6 DAB + 6 BCD 6= 6 CDA + 6 ABC, then among the polygonal paths,
there is none of minimal length.
(b) If 6 DAB + 6 BCD = 6 CDA + 6 ABC, then there are infinitely many
shortest polygonal paths, their common length being 2AC sin(α/2), where
α = 6 BAC + 6 CAD + 6 DAB.
1971/5.
Prove that for every natural number m, there exists a finite set S of points
in a plane with the following property: For every point A in S, there are
exactly m points in S which are at unit distance from A.
1971/6.
Let A = (aij )(i, j = 1, 2, ..., n) be a square matrix whose elements are non-
negative integers. Suppose that whenever an element aij = 0, the sum of the
elements in the ith row and the jth column is ≥ n. Prove that the sum of
all the elements of the matrix is ≥ n2 /2.
Fourteenth International Olympiad, 1972
1972/1.
Prove that from a set of ten distinct two-digit numbers (in the decimal sys-
tem), it is possible to select two disjoint subsets whose members have the
same sum.
1972/2.
Prove that if n ≥ 4, every quadrilateral that can be inscribed in a circle can
be dissected into n quadrilaterals each of which is inscribable in a circle.
1972/3.
Let m and n be arbitrary non-negative integers. Prove that
(2m)!(2n)!
m0n!(m + n)!
is an integer. (0! = 1.)
1972/4.
Find all solutions (x1 , x2 , x3 , x4 , x5 ) of the system of inequalities
(x21 − x3 x5 )(x22 − x3 x5 ) ≤ 0
(x22 − x4 x1 )(x23 − x4 x1 ) ≤ 0
(x23 − x5 x2 )(x24 − x5 x2 ) ≤ 0
(x24 − x1 x3 )(x25 − x1 x3 ) ≤ 0
(x25 − x2 x4 )(x21 − x2 x4 ) ≤ 0
where x1 , x2 , x3 , x4 , x5 are positive real numbers.
1972/5.
Let f and g be real-valued functions defined for all real values of x and y,
and satisfying the equation
f (x + y) + f (x − y) = 2f (x)g(y)
for all x, y. Prove that if f (x) is not identically zero, and if |f (x)| ≤ 1 for all
x, then |g(y)| ≤ 1 for all y.
1972/6.
Given four distinct parallel planes, prove that there exists a regular tetrahe-
dron with a vertex on each plane.
Fifteenth International Olympiad, 1973
1973/1.
−−→ −−→ −−→
Point O lies on line g; OP1 , OP2 , ..., OPn are unit vectors such that points
P1 , P2 , ..., Pn all lie in a plane containing g and on one side of g. Prove that
if n is odd, ¯−−→ −−→
¯ −−→¯¯
¯OP1 + OP2 + · · · + OPn ¯ ≥ 1
¯−−→¯ −−→
¯ ¯
Here ¯OM ¯ denotes the length of vector OM .
1973/2.
Determine whether or not there exists a finite set M of points in space not
lying in the same plane such that, for any two points A and B of M, one can
select two other points C and D of M so that lines AB and CD are parallel
and not coincident.
1973/3.
Let a and b be real numbers for which the equation
x4 + ax3 + bx2 + ax + 1 = 0
has at least one real solution. For all such pairs (a, b), find the minimum
value of a2 + b2 .
1973/4.
A soldier needs to check on the presence of mines in a region having the
shape of an equilateral triangle. The radius of action of his detector is equal
to half the altitude of the triangle. The soldier leaves from one vertex of the
triangle. What path shouid he follow in order to travel the least possible
distance and still accomplish his mission?
1973/5.
G is a set of non-constant functions of the real variable x of the form
f (x) = ax + b, a and b are real numbers,
and G has the following properties:
(a) If f and g are in G, then g ◦ f is in G; here (g ◦ f )(x) = g[f (x)].
(b) If f is in G, then its inverse f −1 is in G; here the inverse of f (x) = ax + b
is f −1 (x) = (x − b)/a.
(c) For every f in G, there exists a real number xf such that f (xf ) = xf .
Prove that there exists a real number k such that f (k) = k for all f in G.
1973/6.
Let a1 , a2 , ..., an be n positive numbers, and let q be a given real number such
that 0 < q < 1. Find n numbers b1 , b2 , ..., bn for which
(a) ak < bk for k = 1, 2, · · · , n,
(b) q < bk+1
bk
< 1q for k = 1, 2, ..., n − 1,
(c) b1 + b2 + · · · + bn < 1+q1−q
(a1 + a2 + · · · + an ).
Sixteenth International Olympiad, 1974
1974/1.
Three players A, B and C play the following game: On each of three cards
an integer is written. These three numbers p, q, r satisfy 0 < p < q < r. The
three cards are shuffled and one is dealt to each player. Each then receives
the number of counters indicated by the card he holds. Then the cards are
shuffled again; the counters remain with the players.
This process (shuffling, dealing, giving out counters) takes place for at least
two rounds. After the last round, A has 20 counters in all, B has 10 and C
has 9. At the last round B received r counters. Who received q counters on
the first round?
1974/2.
In the triangle ABC, prove that there is a point D on side AB such that CD
is the geometric mean of AD and DB if and only if
C
sin A sin B ≤ sin2 .
2
1974/3. ³ ´
Pn 2n+1
Prove that the number k=0 2k+1
23k is not divisible by 5 for any integer
n ≥ 0.
1974/4.
Consider decompositions of an 8 × 8 chessboard into p non-overlapping rect-
angles subject to the following conditions:
(i) Each rectangle has as many white squares as black squares.
(ii) If ai is the number of white squares in the i-th rectangle, then a1 < a2 <
· · · < ap . Find the maximum value of p for which such a decomposition is
possible. For this value of p, determine all possible sequences a1 , a2 , · · · , ap .
1974/5.
Determine all possible values of
a b c d
S= + + +
a+b+d a+b+c b+c+d a+c+d
where a, b, c, d are arbitrary positive numbers.
1974/6.
Let P be a non-constant polynomial with integer coefficients. If n(P ) is
the number of distinct integers k such that (P (k))2 = 1, prove that n(P ) −
deg(P ) ≤ 2, where deg(P ) denotes the degree of the polynomial P.
Seventeenth International Olympiad, 1975
1975/1.
Let xi , yi (i = 1, 2, ..., n) be real numbers such that
x1 ≥ x2 ≥ · · · ≥ xn and y1 ≥ y2 ≥ · · · ≥ yn .
1975/2.
Let a1 , a2 , a3, · · · be an infinite increasing sequence of positive integers. Prove
that for every p ≥ 1 there are infinitely many am which can be written in
the form
am = xap + yaq
with x, y positive integers and q > p.
1975/3.
On the sides of an arbitrary triangle ABC, triangles ABR, BCP, CAQ are
constructed externally with 6 CBP = 6 CAQ = 45◦ , 6 BCP = 6 ACQ =
30◦ , 6 ABR = 6 BAR = 15◦ . Prove that 6 QRP = 90◦ and QR = RP.
1975/4.
When 44444444 is written in decimal notation, the sum of its digits is A. Let
B be the sum of the digits of A. Find the sum of the digits of B. (A and B
are written in decimal notation.)
1975/5.
Determine, with proof, whether or not one can find 1975 points on the cir-
cumference of a circle with unit radius such that the distance between any
two of them is a rational number.
1975/6.
Find all polynomials P, in two variables, with the following properties:
(i) for a positive integer n and all real t, x, y
P (b + c, a) + P (c + a, b) + P (a + b, c) = 0,
(iii) P (1, 0) = 1.
Eighteenth International Olympiad, 1976
1976/1.
In a plane convex quadrilateral of area 32, the sum of the lengths of two
opposite sides and one diagonal is 16. Determine all possible lengths of the
other diagonal.
1976/2.
Let P1 (x) = x2 − 2 and Pj (x) = P1 (Pj−1 (x)) for j = 2, 3, · · ·. Show that,
for any positive integer n, the roots of the equation Pn (x) = x are real and
distinct.
1976/3.
A rectangular box can be filled completely with unit cubes. If one places as
many cubes as possible, each with volume 2, in the box, so that their edges
are parallel to the edges of the box, one can fill exactly 40% of the box.
Determine the possible dimensions of all such boxes.
1976/4.
Determine, with proof, the largest number which is the product of positive
integers whose sum is 1976.
1976/5.
Consider the system of p equations in q = 2p unknowns x1 , x2 , · · · , xq :
with every coefficient aij member of the set {−1, 0, 1}. Prove that the system
has a solution (x1 , x2 , · · · , xq ) such that
(a) all xj (j = 1, 2, ..., q) are integers,
(b) there is at least one value of j for which xj 6= 0,
(c) |xj | ≤ q(j = 1, 2, ..., q).
1976/6.
A sequence {un } is defined by
1978/6. An international society has its members from six different countries.
The list of members contains 1978 names, numbered 1, 2, ..., 1978. Prove that
there is at least one member whose number is the sum of the numbers of two
members from his own country, or twice as large as the number of one member
from his own country.
Twenty-first International Olympiad, 1979
1979/1. Let p and q be natural numbers such that
p 1 1 1 1 1
= 1 − + − + ··· − + .
q 2 3 4 1318 1319
Prove that p is divisible by 1979.
1979/2. A prism with pentagons A1 A2 A3 A4 A5 and B1 B2 B3 B4 B5 as top and
bottom faces is given. Each side of the two pentagons and each of the line-
segments Ai Bj for all i, j = 1, ..., 5, is colored either red or green. Every
triangle whose vertices are vertices of the prism and whose sides have all
been colored has two sides of a different color. Show that all 10 sides of the
top and bottom faces are the same color.
1979/3. Two circles in a plane intersect. Let A be one of the points of
intersection. Starting simultaneously from A two points move with constant
speeds, each point travelling along its own circle in the same sense. The two
points return to A simultaneously after one revolution. Prove that there is
a fixed point P in the plane such that, at any time, the distances from P to
the moving points are equal.
1979/4. Given a plane π, a point P in this plane and a point Q not in π,
find all points R in π such that the ratio (QP + P A)/QR is a maximum.
1979/5. Find all real numbers a for which there exist non-negative real
numbers x1 , x2 , x3 , x4 , x5 satisfying the relations
5
X 5
X 5
X
kxk = a, k 3 xk = a2 , k 5 xk = a 3 .
k=1 k=1 k=1
f (m + n) − f (m) − f (n) = 0 or 1
(a) Prove that for every such sequence, there is an n ≥ 1 such that
x20 x21 x2
+ + · · · + n−1 ≥ 3.999.
x1 x2 xn
(b) Find such a sequence for which
x3 − 3xy 2 + y 3 = n
has a solution in integers (x, y), then it has at least three such solutions.
Show that the equation has no solutions in integers when n = 2891.
1982/5. The diagonals AC and CE of the regular hexagon ABCDEF are
divided by the inner points M and N , respectively, so that
AM CN
= = r.
AC CE
Determine r if B, M, and N are collinear.
1982/6. Let S be a square with sides of length 100, and let L be a path
within S which does not meet itself and which is composed of line segments
A0 A1 , A1 A2 , · · · , An−1 An with A0 6= An . Suppose that for every point P of
the boundary of S there is a point of L at a distance from P not greater than
1/2. Prove that there are two points X and Y in L such that the distance
between X and Y is not greater than 1, and the length of that part of L
which lies between X and Y is not smaller than 198.
Twenty-fourth International Olympiad, 1983
1983/1. Find all functions f defined on the set of positive real numbers which
take positive real values and satisfy the conditions:
(i) f (xf (y)) = yf (x) for all positive x, y;
(ii) f (x) → 0 as x → ∞.
1983/2. Let A be one of the two distinct points of intersection of two unequal
coplanar circles C1 and C2 with centers O1 and O2 , respectively. One of the
common tangents to the circles touches C1 at P1 and C2 at P2 , while the
other touches C1 at Q1 and C2 at Q2 . Let M1 be the midpoint of P1 Q1 ,and
M2 be the midpoint of P2 Q2 . Prove that 6 O1 AO2 = 6 M1 AM2 .
1983/3. Let a, b and c be positive integers, no two of which have a common
divisor greater than 1. Show that 2abc − ab − bc − ca is the largest integer
which cannot be expressed in the form xbc + yca + zab,where x, y and z are
non-negative integers.
1983/4. Let ABC be an equilateral triangle and E the set of all points
contained in the three segments AB, BC and CA (including A, B and C).
Determine whether, for every partition of E into two disjoint subsets, at least
one of the two subsets contains the vertices of a right-angled triangle. Justify
your answer.
1983/5. Is it possible to choose 1983 distinct positive integers, all less than
or equal to 105 , no three of which are consecutive terms of an arithmetic
progression? Justify your answer.
1983/6. Let a, b and c be the lengths of the sides of a triangle. Prove that
1985/4. Given a set M of 1985 distinct positive integers, none of which has
a prime divisor greater than 26. Prove that M contains at least one subset
of four distinct elements whose product is the fourth power of an integer.
1985/5. A circle with center O passes through the vertices A and C of triangle
ABC and intersects the segments AB and BC again at distinct points K and
N, respectively. The circumscribed circles of the triangles ABC and EBN
intersect at exactly two distinct points B and M. Prove that angle OM B is
a right angle.
1985/6. For every real number x1 , construct the sequence x1 , x2 , ... by setting
µ ¶
1
xn+1 = xn xn + for each n ≥ 1.
n
Prove that there exists exactly one value of x1 for which
for every n.
27th International Mathematical Olympiad
Warsaw, Poland
Day I
July 9, 1986
1. Let d be any positive integer not equal to 2, 5, or 13. Show that one can find
distinct a, b in the set {2, 5, 13, d} such that ab − 1 is not a perfect square.
5. Find all functions f , defined on the non-negative real numbers and taking non-
negative real values, such that:
6. One is given a finite set of points in the plane, each point having integer coor-
dinates. Is it always possible to color some of the points in the set red and the
remaining points white in such a way that for any straight line L parallel to
either one of the coordinate axes the difference (in absolute value) between the
numbers of white point and red points on L is not greater than 1?
28th International Mathematical Olympiad
Havana, Cuba
Day I
July 10, 1987
1. Let pn (k) be the number of permutations of the set {1, . . . , n}, n ≥ 1, which
have exactly k fixed points. Prove that
n
X
k · pn (k) = n!.
k=0
4. Prove that there is no function f from the set of non-negative integers into itself
such that f (f (n)) = n + 1987 for every n.
1. Consider two coplanar circles of radii R and r (R > r) with the same center.
Let P be a fixed point on the smaller circle and B a variable point on the larger
circle. The line BP meets the larger circle again at C. The perpendicular l to
BP at P meets the smaller circle again at A. (If l is tangent to the circle at P
then A = P .)
For which values of n can one assign to every element of B one of the numbers
0 and 1 in such a way that Ai has 0 assigned to exactly n of its elements?
f (1) = 1, f (3) = 3,
f (2n) = f (n),
f (4n + 1) = 2f (2n + 1) − f (n),
f (4n + 3) = 3f (2n + 1) − 2f (n),
a2 + b2
ab + 1
is the square of an integer.
30th International Mathematical Olympiad
Braunschweig, Germany
Day I
1. Prove that the set {1, 2, . . . , 1989} can be expressed as the disjoint union of
subsets Ai (i = 1, 2, . . . , 117) such that:
(i) The area of the triangle A0 B0 C0 is twice the area of the hexagon AC1 BA1 CB1 .
(ii) The area of the triangle A0 B0 C0 is at least four times the area of the
triangle ABC.
3. Let n and k be positive integers and let S be a set of n points in the plane such
that
Prove that:
1 √
k< + 2n.
2
30th International Mathematical Olympiad
Braunschweig, Germany
Day II
4. Let ABCD be a convex quadrilateral such that the sides AB, AD, BC satisfy
AB = AD + BC. There exists a point P inside the quadrilateral at a distance
h from the line CD such that AP = h + AD and BP = h + BC. Show that:
1 1 1
√ ≥√ +√ .
h AD BC
5. Prove that for each positive integer n there exist n consecutive positive integers
none of which is an integral power of a prime number.
6. Prove that there exists a convex 1990-gon with the following two properties:
a2 − a1 = a3 − a2 = · · · = ak − ak−1 > 0,
3. Let S = {1, 2, 3, . . . , 280}. Find the smallest integer n such that each n-
element subset of S contains five numbers which are pairwise relatively
prime.
2. Let R denote the set of all real numbers. Find all functions f : R → R
such that
³ ´
f x2 + f (y) = y + (f (x))2 for all x, y ∈ R.
3. Consider nine points in space, no four of which are coplanar. Each pair
of points is joined by an edge (that is, a line segment) and each edge is
either colored blue or red or left uncolored. Find the smallest value of
n such that whenever exactly n edges are colored, the set of colored
edges necessarily contains a triangle all of whose edges have the same
color.
where |A| denotes the number of elements in the finite set |A|. (Note:
The orthogonal projection of a point onto a plane is the foot of the
perpendicular from that point to the plane.)
3. For each positive integer n, S(n) is defined to be the greatest integer
such that, for every positive integer k ≤ S(n), n2 can be written as
the sum of k positive squares.
1. For three points P, Q, R in the plane, we define m(P QR) as the min-
imum length of the three altitudes of 4P QR. (If the points are
collinear, we set m(P QR) = 0.)
Prove that for points A, B, C, X in the plane,
(a) There is a positive integer M (n) such that after M (n) steps all
the lamps are on again;
(b) If n = 2k , we can take M (n) = n2 − 1;
(c) If n = 2k + 1, we can take M (n) = n2 − n + 1.
The 35th International Mathematical Olympiad (July 13-14,
1994, Hong Kong)
1. Let m and n be positive integers. Let a1 , a2 , . . . , am be distinct elements
of {1, 2, . . . , n} such that whenever ai + aj ≤ n for some i, j, 1 ≤ i ≤ j ≤ m,
there exists k, 1 ≤ k ≤ m, with ai + aj = ak . Prove that
a1 + a2 + · · · + am n+1
≥ .
m 2
2. ABC is an isosceles triangle with AB = AC. Suppose that
1. M is the midpoint of BC and O is the point on the line AM such that
OB is perpendicular to AB;
2. Q is an arbitrary point on the segment BC different from B and C;
3. E lies on the line AB and F lies on the line AC such that E, Q, F are
distinct and collinear.
Prove that OQ is perpendicular to EF if and only if QE = QF .
3. For any positive integer k, let f (k) be the number of elements in the set
{k + 1, k + 2, . . . , 2k} whose base 2 representation has precisely three 1s.
• (a) Prove that, for each positive integer m, there exists at least one
positive integer k such that f (k) = m.
• (b) Determine all positive integers m for which there exists exactly one
k with f (k) = m.
4. Determine all ordered pairs (m, n) of positive integers such that
n3 + 1
mn − 1
is an integer.
5. Let S be the set of real numbers strictly greater than −1. Find all
functions f : S → S satisfying the two conditions:
1. f (x + f (y) + xf (y)) = y + f (x) + yf (x) for all x and y in S;
f (x)
2. x
is strictly increasing on each of the intervals −1 < x < 0 and 0 < x.
6. Show that there exists a set A of positive integers with the following
property: For any infinite set S of primes there exist two positive integers
m ∈ A and n ∈/ A each of which is a product of k distinct elements of S for
some k ≥ 2.
36th International Mathematical Olympiad
First Day - Toronto - July 19, 1995
Time Limit: 4 12 hours
1. Find the maximum value of x0 for which there exists a sequence x0 , x1 . . . , x1995
of positive reals with x0 = x1995 , such that for i = 1, . . . , 1995,
2 1
xi−1 + = 2xi + .
xi−1 xi
6 AP B − 6 ACB = 6 AP C − 6 ABC.
3. Let S denote the set of nonnegative integers. Find all functions f from
S to itself such that
(a) x0 = xn = 0.
(b) For each i with 1 ≤ i ≤ n, either xi − xi−1 = p or xi − xi−1 = −q.
Show that there exist indices i < j with (i, j) 6= (0, n), such that
xi = xj .
38th International Mathematical Olympiad
Mar del Plata, Argentina
Day I
July 24, 1997
1. In the plane the points with integer coordinates are the vertices of unit squares.
The squares are colored alternately black and white (as on a chessboard).
For any pair of positive integers m and n, consider a right-angled triangle whose
vertices have integer coordinates and whose legs, of lengths m and n, lie along
edges of the squares.
Let S1 be the total area of the black part of the triangle and S2 be the total
area of the white part. Let
f (m, n) = |S1 − S2 |.
(a) Calculate f (m, n) for all positive integers m and n which are either both
even or both odd.
(b) Prove that f (m, n) ≤ 21 max{m, n} for all m and n.
(c) Show that there is no constant C such that f (m, n) < C for all m and n.
2. The angle at A is the smallest angle of triangle ABC. The points B and C
divide the circumcircle of the triangle into two arcs. Let U be an interior point
of the arc between B and C which does not contain A. The perpendicular
bisectors of AB and AC meet the line AU at V and W , respectively. The lines
BV and CW meet at T . Show that
AU = T B + T C.
|x1 + x2 + · · · + xn | = 1
and
n+1
|xi | ≤ i = 1, 2, . . . , n.
2
Show that there exists a permutation y1 , y2 , . . . , yn of x1 , x2 , . . . , xn such that
n+1
|y1 + 2y2 + · · · + nyn | ≤ .
2
38th International Mathematical Olympiad
Mar del Plata, Argentina
Day II
July 25, 1997
6. For each positive integer n , let f (n) denote the number of ways of representing
n as a sum of powers of 2 with nonnegative integer exponents. Representations
which differ only in the ordering of their summands are considered to be the
same. For instance, f (4) = 4, because the number 4 can be represented in the
following four ways:
4; 2 + 2; 2 + 1 + 1; 1 + 1 + 1 + 1.
3. For any positive integer n, let d(n) denote the number of positive divisors
of n (including 1 and n itself). Determine all positive integers k such that
d(n2 )/d(n) = k for some n.
39th International Mathematical Olympiad
Taipei, Taiwai
Day II
July 16, 1998
4. Determine all pairs (a, b) of positive integers such that ab2 + b + 7 divides
a2 b + a + b.
5. Let I be the incenter of triangle ABC. Let the incircle of ABC touch the sides
BC, CA, and AB at K, L, and M , respectively. The line through B parallel
to M K meets the lines LM and LK at R and S, respectively. Prove that angle
RIS is acute.
6. Consider all functions f from the set N of all positive integers into itself sat-
isfying f (t2 f (s)) = s(f (t))2 for all s and t in N . Determine the least possible
value of f (1998).
40th International Mathematical Olympiad
Bucharest
Day I
July 16, 1999
1. Determine all finite sets S of at least three points in the plane which satisfy the
following condition:
p is a prime,
n not exceeded 2p, and
(p − 1)n + 1 is divisible by np−1 .
5. Two circles G1 and G2 are contained inside the circle G, and are tangent to G
at the distinct points M and N , respectively. G1 passes through the center of
G2 . The line passing through the two points of intersection of G1 and G2 meets
G at A and B. The lines M A and M B meet G1 at C and D, respectively.
Prove that CD is tangent to G2 .
Problem 4. 100 cards are numbered 1 to 100 (each card different) and
placed in 3 boxes (at least one card in each box). How many ways can this
be done so that if two boxes are selected and a card is taken from each, then
the knowledge of their sum alone is always sufficient to identify the third
box?
1
42nd International Mathematical Olympiad
Problems
Each problem is worth seven points.
Problem 1
Let ABC be an acute-angled triangle with circumcentre O . Let P on BC be the foot of the altitude from A.
Problem 2
Prove that
a b c
1
a2 8bc b2 8ca c2 8ab
for all positive real numbers a, b and c .
Problem 3
Problem 4
Let n be an odd integer greater than 1, and let k1 , k2 , …, kn be given integers. For each of the n permutations
a a1 , a2 , …, an of 1, 2, …, n , let
n
Sa ki ai .
i1
Prove that there are two permutations b and c, b c, such that n is a divisor of Sb
Sc.
http://imo.wolfram.com/
2 IMO 2001 Competition Problems
Problem 5
In a triangle ABC , let AP bisect BAC , with P on BC , and let BQ bisect ABC , with Q on CA.
Problem 6
ac bd b d a
cb d
a c.
Prove that ab cd is not prime.
http://imo.wolfram.com/
43rd IMO 2002
Problem 3. Find all pairs of integers m > 2, n > 2 such that there are
infinitely many positive integers k for which k n + k 2 − 1 divides k m + k − 1.
Problem 4. The positive divisors of the integer n > 1 are d1 < d2 < . . . <
dk , so that d1 = 1, dk = n. Let d = d1 d2 + d2 d3 + · · · + dk−1 dk . Show that
d < n2 and find all n for which d divides n2 .
Problem 5. Find all real-valued functions on the reals such that (f (x) +
f (y))((f (u) + f (v)) = f (xu − yv) + f (xv + yu) for all x, y, u, v.
Problem 6. n > 2 circles of radius 1 are drawn in the plane so that no line
meets more than two of the circles. Their centers are O1 , O2 , · · · , On . Show
that i<j 1/Oi Oj ≤ (n − 1)π/4.
P
1
44th IMO 2003
Problem 1. S is the set {1, 2, 3, . . . , 1000000}. Show that for any subset A
of S with 101 elements we can find 100 distinct elements xi of S, such that
the sets {a + xi |a ∈ A} are all pairwise disjoint.
m2
Problem 2. Find all pairs (m, n) of positive integers such that 2mn2 −n3 +1
is a positive integer.
Problem 3. A convex hexagon has the property √that for any pair of opposite
sides the distance between their midpoints is 3/2 times the sum of their
lengths Show that all the hexagon’s angles are equal.
Problem 6. Show that for each prime p, there exists a prime q such that
np − p is not divisible by q for any positive integer n.
1
45rd IMO 2004
Problem 2. Find all polynomials f with real coefficients such that for all
reals a,b,c such that ab + bc + ca = 0 we have the following relations
f (a − b) + f (b − c) + f (c − a) = 2f (a + b + c).
Determine all m×n rectangles that can be covered without gaps and without
overlaps with hooks such that
1
Problem 6. We call a positive integer alternating if every two consecutive
digits in its decimal representation are of different parity.
Find all positive integers n such that n has a multiple which is alternating.
2
46rd IMO 2005
Problem 4. Determine all positive integers relatively prime to all the terms
of the infinite sequence
an = 2n + 3n + 6n − 1, n ≥ 1.
1
day: 1
language: English
12 July 2006
Problem 1. Let ABC be a triangle with incentre I. A point P in the interior of the
triangle satisfies
6 P BA + 6 P CA = 6 P BC + 6 P CB.
Problem 3. Determine the least real number M such that the inequality
2
ab(a2 − b2 ) + bc(b2 − c2 ) + ca(c2 − a2 ) ≤ M (a2 + b2 + c2 )
language: English
13 July 2006
1 + 2x + 22x+1 = y 2 .
Problem 5. Let P (x) be a polynomial of degree n > 1 with integer coefficients and let
k be a positive integer. Consider the polynomial Q(x) = P (P (. . . P (P (x)) . . .)), where P
occurs k times. Prove that there are at most n integers t such that Q(t) = t.
Problem 6. Assign to each side b of a convex polygon P the maximum area of a triangle
that has b as a side and is contained in P . Show that the sum of the areas assigned to
the sides of P is at least twice the area of P .
di = max{aj : 1 ≤ j ≤ i} − min{aj : i ≤ j ≤ n}
and let
d = max{di : 1 ≤ i ≤ n}.
d
max{|xi − ai | : 1 ≤ i ≤ n} ≥ . (∗)
2
(b) Show that there are real numbers x1 ≤ x2 ≤ · · · ≤ xn such that equality holds
in (∗).
Problem 4. In triangle ABC the bisector of angle BCA intersects the circumcircle
again at R, the perpendicular bisector of BC at P , and the perpendicular bisector of AC
at Q. The midpoint of BC is K and the midpoint of AC is L. Prove that the triangles
RP K and RQL have the same area.
Problem 5. Let a and b be positive integers. Show that if 4ab − 1 divides (4a2 − 1)2 ,
then a = b.
Problem 1. An acute-angled triangle ABC has orthocentre H. The circle passing through H with
centre the midpoint of BC intersects the line BC at A1 and A2 . Similarly, the circle passing through
H with centre the midpoint of CA intersects the line CA at B1 and B2 , and the circle passing through
H with centre the midpoint of AB intersects the line AB at C1 and C2 . Show that A1 , A2 , B1 , B2 ,
C1 , C2 lie on a circle.
x2 y2 z2
+ + ≥1
(x − 1)2 (y − 1)2 (z − 1)2
for all real numbers x, y, z, each different from 1, and satisfying xyz = 1.
(b) Prove that equality holds above for infinitely many triples of rational numbers x, y, z, each
different from 1, and satisfying xyz = 1.
2
Problem 3. Prove that there exist √ infinitely many positive integers n such that n + 1 has a prime
divisor which is greater than 2n + 2n.
Problem 4. Find all functions f : (0, ∞) → (0, ∞) (so, f is a function from the positive real
numbers to the positive real numbers) such that
2 2
f (w) + f (x) w 2 + x2
=
f (y 2 ) + f (z 2 ) y2 + z2
Problem 5. Let n and k be positive integers with k ≥ n and k − n an even number. Let 2n lamps
labelled 1, 2, . . . , 2n be given, each of which can be either on or off. Initially all the lamps are off.
We consider sequences of steps: at each step one of the lamps is switched (from on to off or from off
to on).
Let N be the number of such sequences consisting of k steps and resulting in the state where
lamps 1 through n are all on, and lamps n + 1 through 2n are all off.
Let M be the number of such sequences consisting of k steps, resulting in the state where lamps
1 through n are all on, and lamps n + 1 through 2n are all off, but where none of the lamps n + 1
through 2n is ever switched on.
Determine the ratio N/M .
Problem 6. Let ABCD be a convex quadrilateral with |BA| 6= |BC|. Denote the incircles of
triangles ABC and ADC by ω1 and ω2 respectively. Suppose that there exists a circle ω tangent to
the ray BA beyond A and to the ray BC beyond C, which is also tangent to the lines AD and CD.
Prove that the common external tangents of ω1 and ω2 intersect on ω.
Day: 1
Problem 1. Let n be a positive integer and let a1 , . . . , ak (k ≥ 2) be distinct integers in the set
{1, . . . , n} such that n divides ai (ai+1 −1) for i = 1, . . . , k −1. Prove that n does not divide ak (a1 −1).
Problem 2. Let ABC be a triangle with circumcentre O. The points P and Q are interior points
of the sides CA and AB, respectively. Let K, L and M be the midpoints of the segments BP , CQ
and P Q, respectively, and let Γ be the circle passing through K, L and M . Suppose that the line
P Q is tangent to the circle Γ. Prove that OP = OQ.
are both arithmetic progressions. Prove that the sequence s1 , s2 , s3 , . . . is itself an arithmetic pro-
gression.
Day: 2
Problem 4. Let ABC be a triangle with AB = AC. The angle bisectors of 6 CAB and 6 ABC
meet the sides BC and CA at D and E, respectively. Let K be the incentre of triangle ADC.
Suppose that 6 BEK = 45◦ . Find all possible values of 6 CAB.
Problem 5. Determine all functions f from the set of positive integers to the set of positive integers
such that, for all positive integers a and b, there exists a non-degenerate triangle with sides of lengths
Day: 1
holds for all x, y ∈ R. (Here bzc denotes the greatest integer less than or equal to z.)
Problem 2. Let I be the incentre of triangle ABC and let Γ be its circumcircle. Let the line AI
˙ and F a point on the side BC such that
intersect Γ again at D. Let E be a point on the arc BDC
Finally, let G be the midpoint of the segment IF . Prove that the lines DG and EI intersect on Γ.
Problem 3. Let N be the set of positive integers. Determine all functions g : N → N such that
Ä äÄ ä
g(m) + n m + g(n)
Day: 2
Problem 4. Let P be a point inside the triangle ABC. The lines AP , BP and CP intersect the
circumcircle Γ of triangle ABC again at the points K, L and M respectively. The tangent to Γ at C
intersects the line AB at S. Suppose that SC = SP . Prove that M K = M L.
Problem 5. In each of six boxes B1 , B2 , B3 , B4 , B5 , B6 there is initially one coin. There are two
types of operation allowed:
Type 1: Choose a nonempty box Bj with 1 ≤ j ≤ 5. Remove one coin from Bj and add two
coins to Bj+1 .
Type 2: Choose a nonempty box Bk with 1 ≤ k ≤ 4. Remove one coin from Bk and exchange
the contents of (possibly empty) boxes Bk+1 and Bk+2 .
Determine whether there is a finite sequence of such operations that results in boxes B1 , B2 , B3 , B4 , B5
2010 c c
being empty and box B6 containing exactly 20102010 coins. (Note that ab = a(b ) .)
Problem 6. Let a1 , a2 , a3 , . . . be a sequence of positive real numbers. Suppose that for some
positive integer s, we have
an = max{ak + an−k | 1 ≤ k ≤ n − 1}
for all n > s. Prove that there exist positive integers ` and N , with ` ≤ s and such that an = a` +an−`
for all n ≥ N .
Day: 1
Problem 1. Given any set A = {a1 , a2 , a3 , a4 } of four distinct positive integers, we denote the sum
a1 + a2 + a3 + a4 by sA . Let nA denote the number of pairs (i, j) with 1 ≤ i < j ≤ 4 for which ai + aj
divides sA . Find all sets A of four distinct positive integers which achieve the largest possible value
of nA .
Problem 2. Let S be a finite set of at least two points in the plane. Assume that no three points
of S are collinear. A windmill is a process that starts with a line ` going through a single point
P ∈ S. The line rotates clockwise about the pivot P until the first time that the line meets some
other point belonging to S. This point, Q, takes over as the new pivot, and the line now rotates
clockwise about Q, until it next meets a point of S. This process continues indefinitely.
Show that we can choose a point P in S and a line ` going through P such that the resulting windmill
uses each point of S as a pivot infinitely many times.
Problem 3. Let f : R → R be a real-valued function defined on the set of real numbers that
satisfies
f (x + y) ≤ yf (x) + f (f (x))
for all real numbers x and y. Prove that f (x) = 0 for all x ≤ 0.
Day: 2
Problem 4. Let n > 0 be an integer. We are given a balance and n weights of weight 20 ,
2 , . . . , 2 . We are to place each of the n weights on the balance, one after another, in such a
1 n−1
way that the right pan is never heavier than the left pan. At each step we choose one of the weights
that has not yet been placed on the balance, and place it on either the left pan or the right pan,
until all of the weights have been placed.
Determine the number of ways in which this can be done.
Problem 5. Let f be a function from the set of integers to the set of positive integers. Suppose
that, for any two integers m and n, the dierence f (m) − f (n) is divisible by f (m − n). Prove that,
for all integers m and n with f (m) ≤ f (n), the number f (n) is divisible by f (m).
Problem 6. Let ABC be an acute triangle with circumcircle Γ. Let ` be a tangent line to Γ, and
let `a , `b and `c be the lines obtained by reecting ` in the lines BC , CA and AB , respectively. Show
that the circumcircle of the triangle determined by the lines `a , `b and `c is tangent to the circle Γ.
Day: 1
Problem 1. Given triangle ABC the point J is the centre of the excircle opposite the vertex A.
This excircle is tangent to the side BC at M , and to the lines AB and AC at K and L, respectively.
The lines LM and BJ meet at F , and the lines KM and CJ meet at G. Let S be the point of
intersection of the lines AF and BC, and let T be the point of intersection of the lines AG and BC.
Prove that M is the midpoint of ST .
(The excircle of ABC opposite the vertex A is the circle that is tangent to the line segment BC,
to the ray AB beyond B, and to the ray AC beyond C.)
Problem 2. Let n ≥ 3 be an integer, and let a2 , a3 , . . . , an be positive real numbers such that
a2 a3 · · · an = 1. Prove that
(1 + a2 )2 (1 + a3 )3 · · · (1 + an )n > nn .
Problem 3. The liar’s guessing game is a game played between two players A and B. The rules
of the game depend on two positive integers k and n which are known to both players.
At the start of the game A chooses integers x and N with 1 ≤ x ≤ N . Player A keeps x secret,
and truthfully tells N to player B. Player B now tries to obtain information about x by asking player
A questions as follows: each question consists of B specifying an arbitrary set S of positive integers
(possibly one specified in some previous question), and asking A whether x belongs to S. Player
B may ask as many such questions as he wishes. After each question, player A must immediately
answer it with yes or no, but is allowed to lie as many times as she wants; the only restriction is
that, among any k + 1 consecutive answers, at least one answer must be truthful.
After B has asked as many questions as he wants, he must specify a set X of at most n positive
integers. If x belongs to X, then B wins; otherwise, he loses. Prove that:
2. For all sufficiently large k, there exists an integer n ≥ 1.99k such that B cannot guarantee a
win.
Day: 2
Problem 4. Find all functions f : Z → Z such that, for all integers a, b, c that satisfy a + b + c = 0,
the following equality holds:
Problem 5. Let ABC ∠BCA = 90◦ , and let D be the foot of the altitude from
be a triangle with
C . Let X be a point in the interior of the segment CD. Let K be the point on the segment AX
such that BK = BC . Similarly, let L be the point on the segment BX such that AL = AC . Let M
be the point of intersection of AL and BK .
Show that M K = M L.
Problem 6. Find all positive integers n for which there exist non-negative integers a1 , a2 , . . . , an
such that
1 1 1 1 2 n
+ + · · · + an = a1 + a2 + · · · + an = 1.
2a1 2a2 2 3 3 3
Problem 1. Prove that for any pair of positive integers k and n, there exist k positive integers
m1 , m2 , . . . , mk (not necessarily different) such that
2k − 1
1 1 1
1+ = 1+ 1+ ··· 1 + .
n m1 m2 mk
Problem 2. A configuration of 4027 points in the plane is called Colombian if it consists of 2013 red
points and 2014 blue points, and no three of the points of the configuration are collinear. By drawing
some lines, the plane is divided into several regions. An arrangement of lines is good for a Colombian
configuration if the following two conditions are satisfied:
Find the least value of k such that for any Colombian configuration of 4027 points, there is a good
arrangement of k lines.
Problem 3. Let the excircle of triangle ABC opposite the vertex A be tangent to the side BC at the
point A1 . Define the points B1 on CA and C1 on AB analogously, using the excircles opposite B and
C, respectively. Suppose that the circumcentre of triangle A1 B1 C1 lies on the circumcircle of triangle
ABC. Prove that triangle ABC is right-angled.
The excircle of triangle ABC opposite the vertex A is the circle that is tangent to the line segment
BC, to the ray AB beyond B, and to the ray AC beyond C. The excircles opposite B and C are similarly
defined.
Problem 4. Let ABC be an acute-angled triangle with orthocentre H, and let W be a point on the
side BC, lying strictly between B and C. The points M and N are the feet of the altitudes from B and
C, respectively. Denote by ω1 the circumcircle of BW N , and let X be the point on ω1 such that W X
is a diameter of ω1 . Analogously, denote by ω2 the circumcircle of CW M , and let Y be the point on ω2
such that W Y is a diameter of ω2 . Prove that X, Y and H are collinear.
Problem 5. Let Q>0 be the set of positive rational numbers. Let f : Q>0 → R be a function satisfying
the following three conditions:
Problem 6. Let n ≥ 3 be an integer, and consider a circle with n + 1 equally spaced points marked
on it. Consider all labellings of these points with the numbers 0, 1, . . . , n such that each label is used
exactly once; two such labellings are considered to be the same if one can be obtained from the other
by a rotation of the circle. A labelling is called beautiful if, for any four labels a < b < c < d with
a + d = b + c, the chord joining the points labelled a and d does not intersect the chord joining the
points labelled b and c.
Let M be the number of beautiful labellings, and let N be the number of ordered pairs (x, y) of
positive integers such that x + y ≤ n and gcd(x, y) = 1. Prove that
M = N + 1.
Problem 1. Let a0 < a1 < a2 < · · · be an innite sequence of positive integers. Prove that there
exists a unique integer n ≥ 1 such that
a0 + a1 + · · · + an
an < ≤ an+1 .
n
Problem 3. Convex quadrilateral ABCD has ∠ABC = ∠CDA = 90◦ . Point H is the foot of the
perpendicular from A to BD. Points S and T lie on sides AB and AD, respectively, such that H
lies inside triangle SCT and
Problem 4. Points P and Q lie on side BC of acute-angled triangle ABC so that ∠P AB = ∠BCA
and ∠CAQ = ∠ABC . Points M and N lie on lines AP and AQ, respectively, such that P is the
midpoint of AM , and Q is the midpoint of AN . Prove that lines BM and CN intersect on the
circumcircle of triangle ABC .
Problem 5. For each positive integer n, the Bank of Cape Town issues coins of denomination n1 .
Given a nite collection of such coins (of not necessarily dierent denominations) with total value at
most 99 + 21 , prove that it is possible to split this collection into 100 or fewer groups, such that each
group has total value at most 1.
Problem 6. A set of lines in the plane is in general position if no two are parallel and no three
pass through the same point. A set of lines in general position cuts the plane into regions, some of
which have nite area; we call these its nite regions. Prove that√for all suciently large n, in any
set of n lines in general position it is possible to colour at least n of the lines blue in such a way
that none of its nite regions has a completely blue boundary.
√ √
Note: Results with n replaced by c n will be awarded points depending on the value of the
constant c.
Pr♦❜❧❡♠ ✶✳ ❲❡ s❛② t❤❛t ❛ ✜♥✐t❡ s❡t S ♦❢ ♣♦✐♥ts ✐♥ t❤❡ ♣❧❛♥❡ ✐s ❜❛❧❛♥❝❡❞ ✐❢✱ ❢♦r ❛♥② t✇♦ ❞✐✛❡r❡♥t
♣♦✐♥ts A ❛♥❞ B ✐♥ S ✱ t❤❡r❡ ✐s ❛ ♣♦✐♥t C ✐♥ S s✉❝❤ t❤❛t AC = BC ✳ ❲❡ s❛② t❤❛t S ✐s ❝❡♥tr❡✲❢r❡❡ ✐❢
❢♦r ❛♥② t❤r❡❡ ❞✐✛❡r❡♥t ♣♦✐♥ts A✱ B ❛♥❞ C ✐♥ S ✱ t❤❡r❡ ✐s ♥♦ ♣♦✐♥t P ✐♥ S s✉❝❤ t❤❛t P A = P B = P C ✳
✭❛✮ ❙❤♦✇ t❤❛t ❢♦r ❛❧❧ ✐♥t❡❣❡rs n > 3✱ t❤❡r❡ ❡①✐sts ❛ ❜❛❧❛♥❝❡❞ s❡t ❝♦♥s✐st✐♥❣ ♦❢ n ♣♦✐♥ts✳
✭❜✮ ❉❡t❡r♠✐♥❡ ❛❧❧ ✐♥t❡❣❡rs n > 3 ❢♦r ✇❤✐❝❤ t❤❡r❡ ❡①✐sts ❛ ❜❛❧❛♥❝❡❞ ❝❡♥tr❡✲❢r❡❡ s❡t ❝♦♥s✐st✐♥❣ ♦❢ n
♣♦✐♥ts✳
Pr♦❜❧❡♠ ✷✳ ❉❡t❡r♠✐♥❡ ❛❧❧ tr✐♣❧❡s (a, b, c) ♦❢ ♣♦s✐t✐✈❡ ✐♥t❡❣❡rs s✉❝❤ t❤❛t ❡❛❝❤ ♦❢ t❤❡ ♥✉♠❜❡rs
ab − c, bc − a, ca − b
✐s ❛ ♣♦✇❡r ♦❢ 2✳
✭❆ ♣♦✇❡r ♦❢ 2 ✐s ❛♥ ✐♥t❡❣❡r ♦❢ t❤❡ ❢♦r♠ 2n ✱ ✇❤❡r❡ n ✮
✐s ❛ ♥♦♥✲♥❡❣❛t✐✈❡ ✐♥t❡❣❡r✳
Pr♦❜❧❡♠ ✸✳ ▲❡t ABC ❜❡ ❛♥ ❛❝✉t❡ tr✐❛♥❣❧❡ ✇✐t❤ AB > AC ✳ ▲❡t Γ ❜❡ ✐ts ❝✐r❝✉♠❝✐r❝❧❡✱ H ✐ts
♦rt❤♦❝❡♥tr❡✱ ❛♥❞ F t❤❡ ❢♦♦t ♦❢ t❤❡ ❛❧t✐t✉❞❡ ❢r♦♠ A✳ ▲❡t M ❜❡ t❤❡ ♠✐❞♣♦✐♥t ♦❢ BC ✳ ▲❡t Q ❜❡ t❤❡
♣♦✐♥t ♦♥ Γ s✉❝❤ t❤❛t ∠HQA = 90◦ ✱ ❛♥❞ ❧❡t K ❜❡ t❤❡ ♣♦✐♥t ♦♥ Γ s✉❝❤ t❤❛t ∠HKQ = 90◦ ✳ ❆ss✉♠❡
t❤❛t t❤❡ ♣♦✐♥ts A✱ B ✱ C ✱ K ❛♥❞ Q ❛r❡ ❛❧❧ ❞✐✛❡r❡♥t✱ ❛♥❞ ❧✐❡ ♦♥ Γ ✐♥ t❤✐s ♦r❞❡r✳
Pr♦✈❡ t❤❛t t❤❡ ❝✐r❝✉♠❝✐r❝❧❡s ♦❢ tr✐❛♥❣❧❡s KQH ❛♥❞ F KM ❛r❡ t❛♥❣❡♥t t♦ ❡❛❝❤ ♦t❤❡r✳
Pr♦❜❧❡♠ ✹✳ ❚r✐❛♥❣❧❡ ABC ❤❛s ❝✐r❝✉♠❝✐r❝❧❡ Ω ❛♥❞ ❝✐r❝✉♠❝❡♥tr❡ O✳ ❆ ❝✐r❝❧❡ Γ ✇✐t❤ ❝❡♥tr❡ A
✐♥t❡rs❡❝ts t❤❡ s❡❣♠❡♥t BC ❛t ♣♦✐♥ts D ❛♥❞ E ✱ s✉❝❤ t❤❛t B ✱ D✱ E ❛♥❞ C ❛r❡ ❛❧❧ ❞✐✛❡r❡♥t ❛♥❞ ❧✐❡
♦♥ ❧✐♥❡ BC ✐♥ t❤✐s ♦r❞❡r✳ ▲❡t F ❛♥❞ G ❜❡ t❤❡ ♣♦✐♥ts ♦❢ ✐♥t❡rs❡❝t✐♦♥ ♦❢ Γ ❛♥❞ Ω✱ s✉❝❤ t❤❛t A✱ F ✱
B ✱ C ❛♥❞ G ❧✐❡ ♦♥ Ω ✐♥ t❤✐s ♦r❞❡r✳ ▲❡t K ❜❡ t❤❡ s❡❝♦♥❞ ♣♦✐♥t ♦❢ ✐♥t❡rs❡❝t✐♦♥ ♦❢ t❤❡ ❝✐r❝✉♠❝✐r❝❧❡ ♦❢
tr✐❛♥❣❧❡ BDF ❛♥❞ t❤❡ s❡❣♠❡♥t AB ✳ ▲❡t L ❜❡ t❤❡ s❡❝♦♥❞ ♣♦✐♥t ♦❢ ✐♥t❡rs❡❝t✐♦♥ ♦❢ t❤❡ ❝✐r❝✉♠❝✐r❝❧❡ ♦❢
tr✐❛♥❣❧❡ CGE ❛♥❞ t❤❡ s❡❣♠❡♥t CA✳
❙✉♣♣♦s❡ t❤❛t t❤❡ ❧✐♥❡s F K ❛♥❞ GL ❛r❡ ❞✐✛❡r❡♥t ❛♥❞ ✐♥t❡rs❡❝t ❛t t❤❡ ♣♦✐♥t X ✳ Pr♦✈❡ t❤❛t X ❧✐❡s ♦♥
t❤❡ ❧✐♥❡ AO✳
Pr♦❜❧❡♠ ✺✳ ▲❡t R ❜❡ t❤❡ s❡t ♦❢ r❡❛❧ ♥✉♠❜❡rs✳ ❉❡t❡r♠✐♥❡ ❛❧❧ ❢✉♥❝t✐♦♥s f : R → R s❛t✐s❢②✐♥❣ t❤❡
❡q✉❛t✐♦♥
f x + f (x + y) + f (xy) = x + f (x + y) + yf (x)
❢♦r ❛❧❧ r❡❛❧ ♥✉♠❜❡rs x ❛♥❞ y ✳
Pr♦❜❧❡♠ ✶✳ ❚r✐❛♥❣❧❡ BCF ❤❛s ❛ r✐❣❤t ❛♥❣❧❡ ❛t B ✳ ▲❡t A ❜❡ t❤❡ ♣♦✐♥t ♦♥ ❧✐♥❡ CF s✉❝❤ t❤❛t
F A = F B ❛♥❞ F ❧✐❡s ❜❡t✇❡❡♥ A ❛♥❞ C ✳ P♦✐♥t D ✐s ❝❤♦s❡♥ s✉❝❤ t❤❛t DA = DC ❛♥❞ AC ✐s t❤❡
❜✐s❡❝t♦r ♦❢ ∠DAB ✳ P♦✐♥t E ✐s ❝❤♦s❡♥ s✉❝❤ t❤❛t EA = ED ❛♥❞ AD ✐s t❤❡ ❜✐s❡❝t♦r ♦❢ ∠EAC ✳ ▲❡t M
❜❡ t❤❡ ♠✐❞♣♦✐♥t ♦❢ CF ✳ ▲❡t X ❜❡ t❤❡ ♣♦✐♥t s✉❝❤ t❤❛t AM XE ✐s ❛ ♣❛r❛❧❧❡❧♦❣r❛♠ ✭✇❤❡r❡ AM k EX
❛♥❞ AE k M X ✮✳ Pr♦✈❡ t❤❛t ❧✐♥❡s BD✱ F X ✱ ❛♥❞ M E ❛r❡ ❝♦♥❝✉rr❡♥t✳
Pr♦❜❧❡♠ ✷✳ ❋✐♥❞ ❛❧❧ ♣♦s✐t✐✈❡ ✐♥t❡❣❡rs n ❢♦r ✇❤✐❝❤ ❡❛❝❤ ❝❡❧❧ ♦❢ ❛♥ n × n t❛❜❧❡ ❝❛♥ ❜❡ ✜❧❧❡❞ ✇✐t❤ ♦♥❡
♦❢ t❤❡ ❧❡tt❡rs ■✱ ▼ ❛♥❞ ❖ ✐♥ s✉❝❤ ❛ ✇❛② t❤❛t✿
• ✐♥ ❡❛❝❤ r♦✇ ❛♥❞ ❡❛❝❤ ❝♦❧✉♠♥✱ ♦♥❡ t❤✐r❞ ♦❢ t❤❡ ❡♥tr✐❡s ❛r❡ ■✱ ♦♥❡ t❤✐r❞ ❛r❡ ▼ ❛♥❞ ♦♥❡ t❤✐r❞ ❛r❡
❖ ❀ ❛♥❞
• ✐♥ ❛♥② ❞✐❛❣♦♥❛❧✱ ✐❢ t❤❡ ♥✉♠❜❡r ♦❢ ❡♥tr✐❡s ♦♥ t❤❡ ❞✐❛❣♦♥❛❧ ✐s ❛ ♠✉❧t✐♣❧❡ ♦❢ t❤r❡❡✱ t❤❡♥ ♦♥❡ t❤✐r❞
♦❢ t❤❡ ❡♥tr✐❡s ❛r❡ ■✱ ♦♥❡ t❤✐r❞ ❛r❡ ▼ ❛♥❞ ♦♥❡ t❤✐r❞ ❛r❡ ❖✳
◆♦t❡✿ ❚❤❡ r♦✇s ❛♥❞ ❝♦❧✉♠♥s ♦❢ ❛♥ n × n t❛❜❧❡ ❛r❡ ❡❛❝❤ ❧❛❜❡❧❧❡❞ 1 t♦ n ✐♥ ❛ ♥❛t✉r❛❧ ♦r❞❡r✳ ❚❤✉s
❡❛❝❤ ❝❡❧❧ ❝♦rr❡s♣♦♥❞s t♦ ❛ ♣❛✐r ♦❢ ♣♦s✐t✐✈❡ ✐♥t❡❣❡rs (i, j) ✇✐t❤ 1 6 i, j 6 n✳ ❋♦r n > 1✱ t❤❡ t❛❜❧❡ ❤❛s
4n − 2 ❞✐❛❣♦♥❛❧s ♦❢ t✇♦ t②♣❡s✳ ❆ ❞✐❛❣♦♥❛❧ ♦❢ t❤❡ ✜rst t②♣❡ ❝♦♥s✐sts ♦❢ ❛❧❧ ❝❡❧❧s (i, j) ❢♦r ✇❤✐❝❤ i + j ✐s
❛ ❝♦♥st❛♥t✱ ❛♥❞ ❛ ❞✐❛❣♦♥❛❧ ♦❢ t❤❡ s❡❝♦♥❞ t②♣❡ ❝♦♥s✐sts ♦❢ ❛❧❧ ❝❡❧❧s (i, j) ❢♦r ✇❤✐❝❤ i − j ✐s ❛ ❝♦♥st❛♥t✳
Pr♦❜❧❡♠ ✹✳ ❆ s❡t ♦❢ ♣♦s✐t✐✈❡ ✐♥t❡❣❡rs ✐s ❝❛❧❧❡❞ ❢r❛❣r❛♥t ✐❢ ✐t ❝♦♥t❛✐♥s ❛t ❧❡❛st t✇♦ ❡❧❡♠❡♥ts ❛♥❞
❡❛❝❤ ♦❢ ✐ts ❡❧❡♠❡♥ts ❤❛s ❛ ♣r✐♠❡ ❢❛❝t♦r ✐♥ ❝♦♠♠♦♥ ✇✐t❤ ❛t ❧❡❛st ♦♥❡ ♦❢ t❤❡ ♦t❤❡r ❡❧❡♠❡♥ts✳ ▲❡t
P (n) = n2 + n + 1✳ ❲❤❛t ✐s t❤❡ ❧❡❛st ♣♦ss✐❜❧❡ ✈❛❧✉❡ ♦❢ t❤❡ ♣♦s✐t✐✈❡ ✐♥t❡❣❡r b s✉❝❤ t❤❛t t❤❡r❡ ❡①✐sts ❛
♥♦♥✲♥❡❣❛t✐✈❡ ✐♥t❡❣❡r a ❢♦r ✇❤✐❝❤ t❤❡ s❡t
{P (a + 1), P (a + 2), . . . , P (a + b)}
✐s ❢r❛❣r❛♥t❄
✐s ✇r✐tt❡♥ ♦♥ t❤❡ ❜♦❛r❞✱ ✇✐t❤ ✷✵✶✻ ❧✐♥❡❛r ❢❛❝t♦rs ♦♥ ❡❛❝❤ s✐❞❡✳ ❲❤❛t ✐s t❤❡ ❧❡❛st ♣♦ss✐❜❧❡ ✈❛❧✉❡ ♦❢ k ❢♦r
✇❤✐❝❤ ✐t ✐s ♣♦ss✐❜❧❡ t♦ ❡r❛s❡ ❡①❛❝t❧② k ♦❢ t❤❡s❡ ✹✵✸✷ ❧✐♥❡❛r ❢❛❝t♦rs s♦ t❤❛t ❛t ❧❡❛st ♦♥❡ ❢❛❝t♦r r❡♠❛✐♥s
♦♥ ❡❛❝❤ s✐❞❡ ❛♥❞ t❤❡ r❡s✉❧t✐♥❣ ❡q✉❛t✐♦♥ ❤❛s ♥♦ r❡❛❧ s♦❧✉t✐♦♥s❄
Pr♦❜❧❡♠ ✻✳ ❚❤❡r❡ ❛r❡ n > 2 ❧✐♥❡ s❡❣♠❡♥ts ✐♥ t❤❡ ♣❧❛♥❡ s✉❝❤ t❤❛t ❡✈❡r② t✇♦ s❡❣♠❡♥ts ❝r♦ss✱ ❛♥❞
♥♦ t❤r❡❡ s❡❣♠❡♥ts ♠❡❡t ❛t ❛ ♣♦✐♥t✳ ●❡♦✛ ❤❛s t♦ ❝❤♦♦s❡ ❛♥ ❡♥❞♣♦✐♥t ♦❢ ❡❛❝❤ s❡❣♠❡♥t ❛♥❞ ♣❧❛❝❡ ❛
❢r♦❣ ♦♥ ✐t✱ ❢❛❝✐♥❣ t❤❡ ♦t❤❡r ❡♥❞♣♦✐♥t✳ ❚❤❡♥ ❤❡ ✇✐❧❧ ❝❧❛♣ ❤✐s ❤❛♥❞s n − 1 t✐♠❡s✳ ❊✈❡r② t✐♠❡ ❤❡ ❝❧❛♣s✱
❡❛❝❤ ❢r♦❣ ✇✐❧❧ ✐♠♠❡❞✐❛t❡❧② ❥✉♠♣ ❢♦r✇❛r❞ t♦ t❤❡ ♥❡①t ✐♥t❡rs❡❝t✐♦♥ ♣♦✐♥t ♦♥ ✐ts s❡❣♠❡♥t✳ ❋r♦❣s ♥❡✈❡r
❝❤❛♥❣❡ t❤❡ ❞✐r❡❝t✐♦♥ ♦❢ t❤❡✐r ❥✉♠♣s✳ ●❡♦✛ ✇✐s❤❡s t♦ ♣❧❛❝❡ t❤❡ ❢r♦❣s ✐♥ s✉❝❤ ❛ ✇❛② t❤❛t ♥♦ t✇♦ ♦❢
t❤❡♠ ✇✐❧❧ ❡✈❡r ♦❝❝✉♣② t❤❡ s❛♠❡ ✐♥t❡rs❡❝t✐♦♥ ♣♦✐♥t ❛t t❤❡ s❛♠❡ t✐♠❡✳
✭❛✮ Pr♦✈❡ t❤❛t ●❡♦✛ ❝❛♥ ❛❧✇❛②s ❢✉❧✜❧ ❤✐s ✇✐s❤ ✐❢ n ✐s ♦❞❞✳
✭❜✮ Pr♦✈❡ t❤❛t ●❡♦✛ ❝❛♥ ♥❡✈❡r ❢✉❧✜❧ ❤✐s ✇✐s❤ ✐❢ n ✐s ❡✈❡♥✳
Determine all values of a0 for which there is a number A such that an = A for infinitely many values
of n.
Problem 2. Let R be the set of real numbers. Determine all functions f : R → R such that, for
all real numbers x and y,
f (f (x)f (y)) + f (x + y) = f (xy).
Problem 3. A hunter and an invisible rabbit play a game in the Euclidean plane. The rabbit’s
starting point, A0 , and the hunter’s starting point, B0 , are the same. After n − 1 rounds of the game,
the rabbit is at point An−1 and the hunter is at point Bn−1 . In the nth round of the game, three
things occur in order.
(i) The rabbit moves invisibly to a point An such that the distance between An−1 and An is
exactly 1.
(ii) A tracking device reports a point Pn to the hunter. The only guarantee provided by the tracking
device to the hunter is that the distance between Pn and An is at most 1.
(iii) The hunter moves visibly to a point Bn such that the distance between Bn−1 and Bn is
exactly 1.
Is it always possible, no matter how the rabbit moves, and no matter what points are reported
by the tracking device, for the hunter to choose her moves so that after 109 rounds she can ensure
that the distance between her and the rabbit is at most 100?
Problem 4. Let R and S be different points on a circle Ω such that RS is not a diameter. Let `
be the tangent line to Ω at R. Point T is such that S is the midpoint of the line segment RT . Point
J is chosen on the shorter arc RS of Ω so that the circumcircle Γ of triangle JST intersects ` at two
distinct points. Let A be the common point of Γ and ` that is closer to R. Line AJ meets Ω again
at K. Prove that the line KT is tangent to Γ.
(2) no one stands between the third and fourth tallest players,
..
.
Problem 6. An ordered pair (x, y) of integers is a primitive point if the greatest common divisor
of x and y is 1. Given a finite set S of primitive points, prove that there exist a positive integer n
and integers a0 , a1 , . . . , an such that, for each (x, y) in S, we have:
Problem 1. Let Γ be the circumcircle of acute-angled triangle ABC. Points D and E lie on
segments AB and AC, respectively, such that AD = AE. The perpendicular bisectors of BD and
CE intersect the minor arcs AB and AC of Γ at points F and G, respectively. Prove that the lines
DE and F G are parallel (or are the same line).
Problem 2. Find all integers n ≥ 3 for which there exist real numbers a1 , a2 , . . . , an+2 , such that
an+1 = a1 and an+2 = a2 , and
ai ai+1 + 1 = ai+2
for i = 1, 2, . . . , n.
Problem 3. An anti-Pascal triangle is an equilateral triangular array of numbers such that, except
for the numbers in the bottom row, each number is the absolute value of the difference of the two
numbers immediately below it. For example, the following array is an anti-Pascal triangle with four
rows which contains every integer from 1 to 10.
4
2 6
5 7 1
8 3 10 9
Does there exist an anti-Pascal triangle with 2018 rows which contains every integer from 1 to
1 + 2 + · · · + 2018?
Problem 4. A site is any point (x, y) in the plane such that x and y are both positive integers less
than or equal to 20.
Initially, each of the 400 sites is unoccupied. Amy and Ben take turns placing stones with Amy
going first. On her turn, Amy places a new red stone on an√unoccupied site such that the distance
between any two sites occupied by red stones is not equal to 5. On his turn, Ben places a new blue
stone on any unoccupied site. (A site occupied by a blue stone is allowed to be at any distance from
any other occupied site.) They stop as soon as a player cannot place a stone.
Find the greatest K such that Amy can ensure that she places at least K red stones, no matter
how Ben places his blue stones.
Problem 1. Let Z be the set of integers. Determine all functions f : Z → Z such that, for all
integers a and b,
f (2a) + 2f (b) = f (f (a + b)).
Problem 2. In triangle ABC, point A1 lies on side BC and point B1 lies on side AC. Let P and Q
be points on segments AA1 and BB1 , respectively, such that P Q is parallel to AB. Let P1 be a point
on line P B1 , such that B1 lies strictly between P and P1 , and ∠P P1 C = ∠BAC. Similarly, let Q1
be a point on line QA1 , such that A1 lies strictly between Q and Q1 , and ∠CQ1 Q = ∠CBA.
Prove that points P , Q, P1 , and Q1 are concyclic.
Problem 3. A social network has 2019 users, some pairs of whom are friends. Whenever user A
is friends with user B, user B is also friends with user A. Events of the following kind may happen
repeatedly, one at a time:
Three users A, B, and C such that A is friends with both B and C, but B and C are
not friends, change their friendship statuses such that B and C are now friends, but A is
no longer friends with B, and no longer friends with C. All other friendship statuses are
unchanged.
Initially, 1010 users have 1009 friends each, and 1009 users have 1010 friends each. Prove that there
exists a sequence of such events after which each user is friends with at most one other user.
Problem 5. The Bank of Bath issues coins with an H on one side and a T on the other. Harry has
n of these coins arranged in a line from left to right. He repeatedly performs the following operation:
if there are exactly k > 0 coins showing H, then he turns over the k th coin from the left; otherwise,
all coins show T and he stops. For example, if n = 3 the process starting with the configuration
T HT would be T HT → HHT → HT T → T T T , which stops after three operations.
(a) Show that, for each initial configuration, Harry stops after a finite number of operations.
(b) For each initial configuration C, let L(C) be the number of operations before Harry stops. For
example, L(T HT ) = 3 and L(T T T ) = 0. Determine the average value of L(C) over all 2n
possible initial configurations C.
Problem 6. Let I be the incentre of acute triangle ABC with AB 6= AC. The incircle ω of ABC is
tangent to sides BC, CA, and AB at D, E, and F , respectively. The line through D perpendicular
to EF meets ω again at R. Line AR meets ω again at P . The circumcircles of triangles P CE
and P BF meet again at Q.
Prove that lines DI and P Q meet on the line through A perpendicular to AI.
Algebra 7
Problem A1 . . . . . . . . . . . . . . . . . . . . . . . . . . . . . . . . . . . . . . 7
Problem A2 . . . . . . . . . . . . . . . . . . . . . . . . . . . . . . . . . . . . . . 9
Problem A3 . . . . . . . . . . . . . . . . . . . . . . . . . . . . . . . . . . . . . . 10
Problem A4 . . . . . . . . . . . . . . . . . . . . . . . . . . . . . . . . . . . . . . 13
Problem A5 . . . . . . . . . . . . . . . . . . . . . . . . . . . . . . . . . . . . . . 15
Problem A6 . . . . . . . . . . . . . . . . . . . . . . . . . . . . . . . . . . . . . . 17
Combinatorics 19
Problem C1 . . . . . . . . . . . . . . . . . . . . . . . . . . . . . . . . . . . . . . 19
Problem C2 . . . . . . . . . . . . . . . . . . . . . . . . . . . . . . . . . . . . . . 21
Problem C3 . . . . . . . . . . . . . . . . . . . . . . . . . . . . . . . . . . . . . . 23
Problem C4 . . . . . . . . . . . . . . . . . . . . . . . . . . . . . . . . . . . . . . 25
Problem C5 . . . . . . . . . . . . . . . . . . . . . . . . . . . . . . . . . . . . . . 27
Problem C6 . . . . . . . . . . . . . . . . . . . . . . . . . . . . . . . . . . . . . . 29
Problem C7 . . . . . . . . . . . . . . . . . . . . . . . . . . . . . . . . . . . . . . 31
Geometry 35
Problem G1 . . . . . . . . . . . . . . . . . . . . . . . . . . . . . . . . . . . . . . 35
Problem G2 . . . . . . . . . . . . . . . . . . . . . . . . . . . . . . . . . . . . . . 36
Problem G3 . . . . . . . . . . . . . . . . . . . . . . . . . . . . . . . . . . . . . . 38
Problem G4 . . . . . . . . . . . . . . . . . . . . . . . . . . . . . . . . . . . . . . 39
Problem G5 . . . . . . . . . . . . . . . . . . . . . . . . . . . . . . . . . . . . . . 40
Problem G6 . . . . . . . . . . . . . . . . . . . . . . . . . . . . . . . . . . . . . . 42
Problem G7 . . . . . . . . . . . . . . . . . . . . . . . . . . . . . . . . . . . . . . 45
Problem G8 . . . . . . . . . . . . . . . . . . . . . . . . . . . . . . . . . . . . . . 46
Problem G9 . . . . . . . . . . . . . . . . . . . . . . . . . . . . . . . . . . . . . . 48
Problem G10 . . . . . . . . . . . . . . . . . . . . . . . . . . . . . . . . . . . . . 51
Number Theory 55
Problem N1 . . . . . . . . . . . . . . . . . . . . . . . . . . . . . . . . . . . . . . 55
Problem N2 . . . . . . . . . . . . . . . . . . . . . . . . . . . . . . . . . . . . . . 56
Problem N3 . . . . . . . . . . . . . . . . . . . . . . . . . . . . . . . . . . . . . . 57
Problem N4 . . . . . . . . . . . . . . . . . . . . . . . . . . . . . . . . . . . . . . 58
Problem N5 . . . . . . . . . . . . . . . . . . . . . . . . . . . . . . . . . . . . . . 59
Problem N6 . . . . . . . . . . . . . . . . . . . . . . . . . . . . . . . . . . . . . . 60
Problem N7 . . . . . . . . . . . . . . . . . . . . . . . . . . . . . . . . . . . . . . 63
3
Contributing Countries
Andrej Bauer
Robert Geretschläger
Géza Kós
Marcin Kuczma
Svetoslav Savchev
Algebra
here a0 is an arbitrary real number, bai c denotes the greatest integer not exceeding ai , and
hai i = ai − bai c. Prove that ai = ai+2 for i sufficiently large.
(Estonia)
Solution. First note that if a0 ≥ 0, then all ai ≥ 0. For ai ≥ 1 we have (in view of hai i < 1
and bai c > 0)
bai+1 c ≤ ai+1 = bai c · hai i < bai c;
the sequence bai c is strictly decreasing as long as its terms are in [1, ∞). Eventually there
appears a number from the interval [0, 1) and all subsequent terms are 0.
Now pass to the more interesting situation where a0 < 0; then all ai ≤ 0. Suppose the
sequence never hits 0. Then we have bai c ≤ −1 for all i, and so
this means that the sequence bai c is nondecreasing. And since all its terms are integers from
(−∞, −1], this sequence must be constant from some term on:
c2
ai = for i ≥ i0 .
c−1
In the second case, c = −1, equations (1) and (2) say that
(
1 i−i0 ai0 for i = i0 , i0 + 2, i0 + 4, . . . ,
ai = − + (−1) bi0 =
2 1 − ai0 for i = i0 + 1, i0 + 3, i0 + 5, . . . .
Summarising, we see that (from some point on) the sequence (ai ) either is constant or takes
alternately two values from the interval (−1, 0). The result follows.
Comment. There is nothing mysterious in introducing the sequence (bi ). The sequence (ai ) arises by
iterating the function x 7→ cx − c2 whose unique fixed point is c2 /(c − 1).
9
Solution. The proof goes by induction. For n = 1 the formula yields a1 = 1/2. Take n ≥ 1,
assume a1 , . . . , an > 0 and write the recurrence formula for n and n + 1, respectively as
n n+1
X ak X ak
=0 and = 0.
k=0
n−k+1 k=0
n−k+2
Subtraction yields
n+1 n
X ak X ak
0 = (n + 2) − (n + 1)
k=0
n−k+2 k=0
n−k+1
n
X n+2 n+1
= (n + 2)an+1 + − ak .
k=0
n − k + 2 n − k + 1
The coefficients of a1 , , . . . , an are all positive. Therefore, a1 , . . . , an > 0 implies an+1 > 0.
Comment.
P Students familiar with the technique of generating functions will immediately recognise
an xn as the power series expansion of x/ ln(1 − x) (with value −1 at 0). But this can be a trap;
attempts along these lines lead to unpleasant differential equations and integrals hard to handle. Using
only tools from real analysis (e.g. computing the coefficients from the derivatives) seems very difficult.
On the other hand, the coefficients can be approached applying complex contour integrals and some
other techniques from complex analysis and an attractive formula can be obtained for the coefficients:
Z ∞
dx
an = (n ≥ 1)
1 xn π 2 + log2 (x − 1)
m < αx + βy < M
ϕn−1 − ψ n−1
cn = for n ≥ 0.
ϕ−ψ
Suppose that the numbers α and β have the stated property, for appropriately chosen m and M.
Since (cn , cn−1 ) ∈ S for each n, the expression
α β 1
αcn + βcn−1 = √ ϕn−1 − ψ n−1 + √ ϕn−2 − ψ n−2 = √ (αϕ + β)ϕn−2 − (αψ + β)ψ n−2
5 5 5
is bounded as n grows to infinity. Because ϕ > 1 and −1 < ψ < 0, this implies αϕ + β = 0.
To satisfy αϕ + β = 0, one can set for instance α = ψ, β = 1. We now find the required m
and M for this choice of α and β.
Note first that the above displayed equation gives cn ψ + cn−1 = ψ n−1 , n ≥ 1. In the sequel,
we denoteP the pairs in SP
by (aJ , bJ ), where J is a finitePsubset of the set N of positive integers
and aJ = j∈J cj , bJ = j∈J cj−1 . Since ψaJ + bJ = j∈J (cj ψ + cj−1 ), we obtain
X
ψaJ + bJ = ψ j−1 for each (aJ , bJ ) ∈ S. (1)
j∈J
Lemma. Let x, y be nonnegative integers such that −1 < ψx + y < ϕ. Then there exists a
subset J of N such that X
ψx + y = ψ j−1 (2)
j∈J
Proof. For x = y = 0 it suffices to choose the empty subset of N as J, so let at least one of x, y
be nonzero. There exist representations of ψx + y of the form
ψx + y = ψ i1 + · · · + ψ ik
where i1 ≤ · · · ≤ ik is a sequence of nonnegative integers, not necessarily distinct. For instance,
we can take x summands ψ 1 = ψ and y summands ψ 0 = 1. Consider all such representations
of minimum length k and focus on the ones for which i1 has the minimum possible value j1 .
Among them, consider the representations where i2 has the minimum possible value j2 . Upon
choosing jP3 , . . . , jk analogously, we obtain a sequence j1 ≤ · · · ≤ jk which clearly satisfies
ψx + y = kr=1 ψ jr . To prove the lemma, it suffices to show that j1 , . . . , jk are pairwise distinct.
Suppose on the contrary that jr = jr+1 for some r = 1, . . . , k − 1. Let us consider the
case jr ≥ 2 first. Observing that 2ψ 2 = 1 + ψ 3 , we replace jr and jr+1 by jr − 2 and jr + 1,
respectively. Since
ψ jr + ψ jr+1 = 2ψ jr = ψ jr −2 (1 + ψ 3 ) = ψ jr −2 + ψ jr +1 ,
the new sequence also represents ψx + y as needed, and the value of ir in it contradicts the
minimum choice of jr . P
Let jr = jr+1 = 0. Then the sum ψx + y = kr=1 ψ jr contains at least two summands equal
to ψ 0 = 1. On the other hand js 6= 1 for all s, because the equality 1 + ψ = ψ 2 implies that a
representation of minimum length cannot contain consecutive ir ’s. It follows that
k
X
ψx + y = ψ jr > 2 + ψ 3 + ψ 5 + ψ 7 + · · · = 2 − ψ 2 = ϕ,
r=1
ψx+y ψx+y−1
respectively; therefore it suffices to find an appropriate set for ψ or ψ , respectively.
ψx+y
Consider ψ . Knowing that
ψx + y
= x + (ψ − 1)y = ψy + (x − y),
ψ
ψx+y
let x0 = y, y 0 = x − y and test the induction hypothesis on these numbers. We require ψ ∈ (−1, ϕ)
which is equivalent to
ψx + y ∈ (ϕ · ψ, (−1) · ψ) = (−1, −ψ). (3)
Relation (3) implies y 0 = x − y ≥ −ψx − y > ψ > −1; therefore x0 , y 0 ≥ 0. Moreover, we have
3x0 + 2y 0 = 2x + y ≤ 32 n; therefore, if (3) holds then the induction applies: the numbers x0 , y 0 are
represented in the form as needed, hence x, y also.
Now consider ψx+y−1
ψ . Since
ψx + y − 1
= x + (ψ − 1)(y − 1) = ψ(y − 1) + (x − y + 1),
ψ
ψx+y−1
we set x0 = y − 1 and y 0 = x − y + 1. Again we require that ψ ∈ (−1, ϕ), i.e.
If (4) holds then y − 1 ≥ ψx + y − 1 > −1 and x − y + 1 ≥ −ψx − y + 1 > −ϕ + 1 > −1, therefore
x0 , y 0 ≥ 0. Moreover, 3x0 + 2y 0 = 2x + y − 1 < 23 n and the induction works.
Finally, (−1, −ψ) ∪ (0, ϕ) = (−1, ϕ) so at least one of (3) and (4) holds and the induction step is
justified.
13
And thus:
X ai aj X 1 (ai − aj )2 n − 1 1 X (ai − aj )2
L= = ai + aj − = ·S− . (2)
a + aj
i<j i i<j
4 ai + aj 4 4 i<j ai + aj
X
To represent R we express the sum ai aj in two ways; in the second transformation
i<j
identity (1) will be applied to the squares of the numbers ai :
X 1 2 X 2
S −
ai aj = ai ;
i<j
2 i
X 1 X 2 n−1 X 1X
ai aj = ai + a2j − (ai − aj )2 = · a2i − (ai − aj )2 .
i<j
2 i<j
2 i
2 i<j
Multiplying the first of these equalities by n − 1 and adding the second one we obtain
X n−1 2 1X
n ai aj = ·S − (ai − aj )2 .
i<j
2 2 i<j
Hence
n X n−1 1 X (ai − aj )2
R= ai aj = ·S− . (3)
2S i<j 4 4 i<j S
Now compare (2) and (3). Since S ≥ ai + aj for any i < j, the claim L ≥ R results.
14
!
1 XXX XXX XX
= ai ak + aj ak + 2ai aj
8S i
k i6=k j6=i k j6=k i6=j j6=i
!
1 XX XX XX
= (n − 1)ai ak + (n − 1)aj ak + 2ai aj
8S i
k i6=k k j6=k j6=i
n XX n X
= ai aj = ai aj .
4S i 2S i<j
j6=i
Comment.P Here is an P outline of another possible approach. Examine the function R − L subject to
constraints i ai = S, i<j ai aj = U for fixed constants S, U > 0 (which can jointly occur as values
of these symmetric forms). Suppose that among the numbers ai there are some three, say ak , al , am
such that ak < al ≤ am . Then it is possible to decrease the value of R − L by perturbing this triple so
that in the new triple a0k , a0l , a0m one has a0k = a0l ≤ a0m , without touching the remaining ai s and without
changing the values of S and U ; this requires some skill in algebraic manipulations. It follows that
the constrained minimum can be only attained for n − 1 of the ai s equal and a single one possibly
greater. In this case, R − L ≥ 0 holds almost trivially.
15
(Korea)
√ √ √ √
Solution 1.√ Note
√ first√that the√denominators
√ √ are all positive,
√ √ e.g. √ a + b > a + b > c.
Let x = b + c − a, y = c + a − b and z = a + b − c. Then
2 2 2
z+x x+y y+z x2 + xy + xz − yz 1
b+c−a= + − = = x2 − (x − y)(x − z)
2 2 2 2 2
and
√ r
b+c−a (x − y)(x − z) (x − y)(x − z)
√ √ √ = 1− 2
≤1− ,
b+ c− a 2x 4x2
√
applying 1 + 2u ≤ 1 + u in the last step. Similarly we obtain
√ √
c+a−b (z − x)(z − y) a+b−c (y − z)(y − x)
√ √ √ ≤ 1− and √ √ √ ≤1− .
c+ a− b 4z 2 a+ b− c 4y 2
(z − x)(z − y)
≥0
z2
and (1) follows.
which holds for all positive numbers x, y, z and real t; in our case t = −2. Case t > 0 is called Schur’s
inequality. More generally, if x ≤ y ≤ z are real numbers and p, q, r are nonnegative numbers such
that q ≤ p or q ≤ r then
Comment 2. One might also start using Cauchy–Schwarz’ inequality (or the root mean square
vs. arithmetic mean inequality) to the effect that
X √ 2
b+c−a X b+c−a
√ √ √ ≤3· √ √ √ 2 , (2)
b+ c− a b+ c− a
in cyclic sum notation. There are several ways to prove that the right-hand side of (2) never exceeds 9
(and this is just what we need). One of them is to introduce new variables x, y, z, as in Solution 1,
which upon some manipulation brings the problem again to inequality (1).
Alternatively, the claim that
P right-hand
P side of (2) is not greater than 9 can be expressed in terms
of the symmetric forms σ1 = x, σ2 = xy, σ3 = xyz equivalently as
which is a known
√ √ inequality.
√ A yet different method to deal with the right-hand expression in (2) is
to consider a, b, c as sides of a triangle. Through standard trigonometric formulas the problem
comes down to showing that
p2 ≤ 4R2 + 4Rr + 3r 2 , (4)
p, R and r standing for the semiperimeter, the circumradius and the inradius of that triangle. Again,
(4) is another known inequality. Note that the inequalities (1), (3), (4) are equivalent statements
about the same mathematical situation.
Solution 2. Due to the symmetry of variables, it can be assumed that a ≥ b ≥ c. We claim
that √ √ √
a+b−c b+c−a c+a−b
√ √ √ ≤ 1 and √ √ √ +√ √ √ ≤ 2.
a+ b− c b+ c− a c+ a− b
The first inequality follows from
√ √ (a + b − c) − a b−c √ √
a+b−c− a= √ √ ≤√ √ = b − c.
a+b−c+ a b+ c
√ √ √ √
For proving the second inequality, let p = a + b and q = a − b. Then a − b = pq and
the inequality becomes √ √
c − pq c + pq
√ + √ ≤ 2.
c−q c+q
√
From a ≥ b ≥ c we have p ≥ 2 c. Applying the Cauchy-Schwarz inequality,
√ √ 2
c − pq c + pq c − pq c + pq 1 1
√ + √ ≤ √ +√ √ +√
c−q c+q c−q c+q c−q c+q
√ √ √
2(c c − pq 2 ) 2 c c2 − cpq 2 c2 − 2cq 2
= · = 4 · ≤ 4 · ≤ 4.
c − q2 c − q2 (c − q 2 )2 (c − q 2 )2
17
since the cubic coefficient is b − c. The left-hand side of the proposed inequality can therefore
be written in the form
The problem comes down to finding the smallest number M that satisfies the inequality
Note that this expression is symmetric, and we can therefore assume a ≤ b ≤ c without loss of
generality. With this assumption,
2
(b − a) + (c − b) (c − a)2
|(a − b)(b − c)| = (b − a)(c − b) ≤ = , (2)
2 4
or equivalently,
3(c − a)2 ≤ 2 · [(b − a)2 + (c − b)2 + (c − a)2 ], (3)
again with equality only for 2b = a + c. From (2) and (3) we get
C1. We have n ≥ 2 lamps L1 , . . . , Ln in a row, each of them being either on or off . Every
second we simultaneously modify the state of each lamp as follows:
— if the lamp Li and its neighbours (only one neighbour for i = 1 or i = n, two neighbours for
other i) are in the same state, then Li is switched off;
— otherwise, Li is switched on.
Initially all the lamps are off except the leftmost one which is on.
(a) Prove that there are infinitely many integers n for which all the lamps will eventually
be off.
(b) Prove that there are infinitely many integers n for which the lamps will never be all off.
(France)
Solution. (a) Experiments with small n lead to the guess that every n of the form 2k should
be good. This is indeed the case, and more precisely: let Ak be the 2k ×2k matrix whose rows
represent the evolution of the system, with entries 0, 1 (for off and on respectively). The top
row shows the initial state [1, 0, 0, . . . , 0]; the bottom row shows the state after 2k − 1 steps.
The claim is that:
The bottom row of Ak is [1, 1, 1, . . . , 1].
This will of course suffice because one more move then produces [0, 0, 0, . . . , 0], as required.
The proof is by induction on k. The base k = 1 isobvious.Assume the claim to be true for a
Ak Ok
k ≥ 1 and write the matrix Ak+1 in the block form with four 2k ×2k matrices. After
Bk Ck
m steps, the last 1 in a row is at position m + 1. Therefore Ok is the zero matrix. According
to the induction hypothesis, the bottom row of [Ak Ok ] is [1, . . . , 1, 0, . . . , 0], with 2k ones and
2k zeros. The next row is thus
[0, . . . , 0, 1, 1, 0, . . . , 0]
| {z } | {z }
2k −1 2k −1
It is symmetric about its midpoint, and this symmetry is preserved in all subsequent rows
because the procedure described in the problem statement is left/right symmetric. Thus Bk is
the mirror image of Ck . In particular, the rightmost column of Bk is identical with the leftmost
column of Ck .
Imagine the matrix Ck in isolation from the rest of Ak+1 . Suppose it is subject to evolution
as defined in the problem: the first (leftmost) term in a row depends only on the two first terms
in the preceding row, according as they are equal or not. Now embed Ck again in Ak . The
‘leftmost’ terms in the rows of Ck now have neighbours on their left side—but these neighbours
are their exact copies. Consequently the actual evolution within Ck is the same, whether or not
Ck is considered as a piece of Ak+1 or in isolation. And since the top row of Ck is [1, 0, . . . , 0],
it follows that Ck is identical with Ak .
20
The bottom row of Ak is [1, 1, . . . , 1]; the same is the bottom row of Ck , hence also of Bk ,
which mirrors Ck . So the bottom row of Ak+1 consists of ones only and the induction is
complete.
(b) There are many ways to produce an infinite sequence of those n for which the state
[0, 0, . . . , 0] will never be achieved. As an example, consider n = 2k + 1 (for k ≥ 1). The
evolution of the system can be represented by a matrix A of width 2k + 1 with infinitely many
rows. The top 2k rows form the matrix Ak discussed above, with one column of zeros attached
at its right.
In the next row we then have the vector [0, 0, . . . , 0, 1, 1]. But this is just the second row of A
reversed. Subsequent rows will be mirror copies of the foregoing ones, starting from the second
one. So the configuration [1, 1, 0, . . . , 0, 0], i.e. the second row of A, will reappear. Further rows
will periodically repeat this pattern and there will be no row of zeros.
21
C2. A diagonal of a regular 2006-gon is called odd if its endpoints divide the boundary into
two parts, each composed of an odd number of sides. Sides are also regarded as odd diagonals.
Suppose the 2006-gon has been dissected into triangles by 2003 nonintersecting diagonals.
Find the maximum possible number of isosceles triangles with two odd sides.
(Serbia)
Solution 1. Call an isosceles triangle odd if it has two odd sides. Suppose we are given a
dissection as in the problem statement. A triangle in the dissection which is odd and isosceles
will be called iso-odd for brevity.
Lemma. Let AB be one of dissecting diagonals and let L be the shorter part of the boundary of
the 2006-gon with endpoints A, B. Suppose that L consists of n segments. Then the number
of iso-odd triangles with vertices on L does not exceed n/2.
Proof. This is obvious for n = 2. Take n with 2 < n ≤ 1003 and assume the claim to be true
for every L of length less than n. Let now L (endpoints A, B) consist of n segments. Let P Q
be the longest diagonal which is a side of an iso-odd triangle P QS with all vertices on L (if
there is no such triangle, there is nothing to prove). Every triangle whose vertices lie on L is
obtuse or right-angled; thus S is the summit of P QS. We may assume that the five points
A, P, S, Q, B lie on L in this order and partition L into four pieces LAP , LP S , LSQ , LQB (the
outer ones possibly reducing to a point).
By the definition of P Q, an iso-odd triangle cannot have vertices on both LAP and LQB .
Therefore every iso-odd triangle within L has all its vertices on just one of the four pieces.
Applying to each of these pieces the induction hypothesis and adding the four inequalities we
get that the number of iso-odd triangles within L other than P QS does not exceed n/2. And
since each of LP S , LSQ consists of an odd number of sides, the inequalities for these two pieces
are actually strict, leaving a 1/2 + 1/2 in excess. Hence the triangle P SQ is also covered by
the estimate n/2. This concludes the induction step and proves the lemma.
The remaining part of the solution in fact repeats the argument from the above proof.
Consider the longest dissecting diagonal XY . Let LXY be the shorter of the two parts of the
boundary with endpoints X, Y and let XY Z be the triangle in the dissection with vertex Z
not on LXY . Notice that XY Z is acute or right-angled, otherwise one of the segments XZ, Y Z
would be longer than XY . Denoting by LXZ , LY Z the two pieces defined by Z and applying
the lemma to each of LXY , LXZ , LY Z we infer that there are no more than 2006/2 iso-odd
triangles in all, unless XY Z is one of them. But in that case XZ and Y Z are odd diagonals
and the corresponding inequalities are strict. This shows that also in this case the total number
of iso-odd triangles in the dissection, including XY Z, is not greater than 1003.
This bound can be achieved. For this to happen, it just suffices to select a vertex of the
2006-gon and draw a broken line joining every second vertex, starting from the selected one.
Since 2006 is even, the line closes. This already gives us the required 1003 iso-odd triangles.
Then we can complete the triangulation in an arbitrary fashion.
22
Solution 2. Let the terms odd triangle and iso-odd triangle have the same meaning as in the
first solution.
Let ABC be an iso-odd triangle, with AB and BC odd sides. This means that there are
an odd number of sides of the 2006-gon between A and B and also between B and C. We say
that these sides belong to the iso-odd triangle ABC.
At least one side in each of these groups does not belong to any other iso-odd triangle.
This is so because any odd triangle whose vertices are among the points between A and B has
two sides of equal length and therefore has an even number of sides belonging to it in total.
Eliminating all sides belonging to any other iso-odd triangle in this area must therefore leave
one side that belongs to no other iso-odd triangle. Let us assign these two sides (one in each
group) to the triangle ABC.
To each iso-odd triangle we have thus assigned a pair of sides, with no two triangles sharing
an assigned side. It follows that at most 1003 iso-odd triangles can appear in the dissection.
This value can be attained, as shows the example from the first solution.
23
C3. Let S be a finite set of points in the plane such that no three of them are on a line. For
each convex polygon P whose vertices are in S, let a(P ) be the number of vertices of P , and
let b(P ) be the number of points of S which are outside P . Prove that for every real number x
X
xa(P ) (1 − x)b(P ) = 1,
P
where the sum is taken over all convex polygons with vertices in S.
NB. A line segment, a point and the empty set are considered as convex polygons of 2, 1
and 0 vertices, respectively.
(Colombia)
Solution 1. For each convex polygon P whose vertices are in S, let c(P ) be the number of
points of S which are inside P , so that a(P ) + b(P ) + c(P ) = n, the total number of points
in S. Denoting 1 − x by y,
c(P )
X
a(P ) b(P )
X
a(P ) b(P ) c(P )
XX c(P ) a(P )+i b(P )+c(P )−i
x y = x y (x + y) = x y .
P P P i=0
i
Solution 2. Apply induction on the number n of points. The case n = 0 is trivial. Let n > 0
and assume the statement for less than n points. Take a set S of n points.
Let C be the set of vertices of the convex hull of S, let m = |C|.
Let X ⊂ C be an arbitrary nonempty set. For any convex polygon P with vertices in the
set S \ X, we have b(P ) points of S outside P . Excluding the points of X — all outside P
— the set S \ X contains exactly b(P ) − |X| of them. Writing 1 − x = y, by the induction
hypothesis X
xa(P ) y b(P )−|X| = 1
P ⊂S\X
(where P ⊂ S \ X means that the vertices of P belong to the set S \ X). Therefore
X
xa(P ) y b(P ) = y |X| .
P ⊂S\X
All convex polygons appear at least once, except the convex hull C itself. The convex hull
adds xm . We can use the inclusion-exclusion principle to compute the sum of the other terms:
X m
X X X m
X X
a(P ) b(P ) k−1 a(P ) b(P )
x y = (−1) x y = (−1)k−1 yk
P 6=C k=1 |X|=k P ⊂S\X k=1 |X|=k
m
X
k−1 m k
= (−1) y = − (1 − y)m − 1 = 1 − xm
k=1
k
and then X X X
xa(P ) y b(P ) = + = xm + (1 − xm ) = 1.
P P =C P 6=C
25
C4. A cake has the form of an n × n square composed of n2 unit squares. Strawberries lie
on some of the unit squares so that each row or column contains exactly one strawberry; call
this arrangement A.
Let B be another such arrangement. Suppose that every grid rectangle with one vertex
at the top left corner of the cake contains no fewer strawberries of arrangement B than of
arrangement A. Prove that arrangement B can be obtained from A by performing a number
of switches, defined as follows:
A switch consists in selecting a grid rectangle with only two strawberries, situated at its
top right corner and bottom left corner, and moving these two strawberries to the other two
corners of that rectangle.
(Taiwan)
Solution. We use capital letters to denote unit squares; O is the top left corner square. For
any two squares X and Y let [XY ] be the smallest grid rectangle containing these two squares.
Strawberries lie on some squares in arrangement A. Put a plum on each square of the target
configuration B. For a square X denote by a(X) and b(X) respectively the number of straw-
berries and the number of plums in [OX]. By hypothesis a(X) ≤ b(X) for each X, with strict
inequality for some X (otherwise the two arrangements coincide and there is nothing to prove).
The idea is to show that by a legitimate switch one can obtain an arrangement A0 such that
X X
a(X) ≤ a0 (X) ≤ b(X) for each X; a(X) < a0 (X) (1)
X X
(with a0 (X) defined analogously to a(X); the sums range over all unit squares X). This will be
enough because the same reasoning
P thenP A0 , giving rise to a new arrangement A00 ,
applies to P
and so Pon (induction). Since a(X) < a (X) < a00 (X) < . . . and all these sums do not
0
exceed b(X), we eventually obtain a sum with all summands equal to the respective b(X)s;
all strawberries will meet with plums.
Consider the uppermost row in which the plum and the strawberry lie on different squares
P and S (respectively); clearly P must be situated left to S. In the column passing through P ,
let T be the top square and B the bottom square. The strawberry in that column lies below
the plum (because there is no plum in that column above P , and the positions of strawberries
and plums coincide everywhere above the row of P ). Hence there is at least one strawberry in
the region [BS] below [P S]. Let V be the position of the uppermost strawberry in that region.
O T
P U S
R
V W
B
26
Denote by W the square at the intersection of the row through V with the column through S
and let R be the square vertex-adjacent to W up-left. We claim that
This is so because if X ∈ [P R] then the portion of [OX] left to column [T B] contains at least
as many plums as strawberries (the hypothesis of the problem); in the portion above the row
through P and S we have perfect balance; and in the remaining portion, i.e. rectangle [P X]
we have a plum on square P and no strawberry at all.
Now we are able to perform the required switch. Let U be the square at the intersection
of the row through P with the column through V (some of P, U, R can coincide). We move
strawberries from squares S and V to squares U and W . Then
And since the rectangle [UR] is contained in [P R], we still have a0 (X) ≤ b(X) for all S, in view
of (2); conditions (1) are satisfied and the proof is complete.
27
C5. An (n, k)-tournament is a contest with n players held in k rounds such that:
(i) Each player plays in each round, and every two players meet at most once.
(ii) If player A meets player B in round i, player C meets player D in round i, and player A
meets player C in round j, then player B meets player D in round j.
Determine all pairs (n, k) for which there exists an (n, k)-tournament.
(Argentina)
Solution. For each k, denote by tk the unique integer such that 2tk −1 < k + 1 ≤ 2tk . We show
that an (n, k)-tournament exists if and only if 2tk divides n.
First we prove that if n = 2t for some t then there is an (n, k)-tournament for all k ≤ 2t − 1.
Let S be the set of 0−1 sequences with length t. We label the 2t players with the elements of S
in an arbitrary fashion (which is possible as there are exactly 2t sequences in S). Players are
identified with their labels in the construction below. If α, β ∈ S, let α + β ∈ S be the result
of the modulo 2 term-by-term addition of α and β (with rules 0 + 0 = 0, 0 + 1 = 1 + 0 = 1,
1 + 1 = 0; there is no carryover). For each i = 1, . . . , 2t − 1 let ω(i) ∈ S be the sequence of
base 2 digits of i, completed with leading zeros if necessary to achieve length t.
Now define a tournament with n = 2t players in k ≤ 2t − 1 rounds as follows: For all
i = 1, . . . , k, let player α meet player α + ω(i) in round i. The tournament is well-defined as
α + ω(i) ∈ S and α + ω(i) = β + ω(i) implies α = β; also [α + ω(i)] + ω(i) = α for each α ∈ S
(meaning that player α + ω(i) meets player α in round i, as needed). Each player plays in each
round. Next, every two players meet at most once (exactly once if k = 2t − 1), since ω(i) 6= ω(j)
if i 6= j. Thus condition (i) holds true, and condition (ii) is also easy to check.
Let player α meet player β in round i, player γ meet player δ in round i, and player α meet
player γ in round j. Then β = α + ω(i), δ = γ + ω(i) and γ = α + ω(j). By definition, β will
play in round j with
as required by (ii).
So there exists an (n, k)-tournament for pairs (n, k) such that n = 2t and k ≤ 2t − 1. The
same conclusion is straightforward for n of the form n = 2t s and k ≤ 2t − 1. Indeed, consider
s different (2t , k)-tournaments T1 , . . . , Ts , no two of them having players in common. Their
union can be regarded as a (2t s, k)-tournament T where each round is the union of the respective
rounds in T1 , . . . , Ts .
In summary, the condition that 2tk divides n is sufficient for an (n, k)-tournament to exist.
We prove that it is also necessary.
Consider an arbitrary (n, k)-tournament. Represent each player by a point and after each
round, join by an edge every two players who played in this round. Thus to a round i = 1, . . . , k
there corresponds a graph Gi . We say that player Q is an i-neighbour of player P if there is a
path of edges in Gi from P to Q; in other words, if there are players P = X1 , X2 , . . . , Xm = Q
such that player Xj meets player Xj+1 in one of the first i rounds, j = 1, 2 . . . , m−1. The set
of i-neighbours of a player will be called its i-component. Clearly two i-components are either
disjoint or coincide.
Hence after each round i the set of players is partitioned into pairwise disjoint i-components.
So, to achieve our goal, it suffices to show that all k-components have size divisible by 2tk .
To this end, let us see how the i-component Γ of a player A changes after round i+1.
Suppose that A meets player B with i-component ∆ in round i+1 (components Γ and ∆ are
28
not necessarily distinct). We claim that then in round i+1 each player from Γ meets a player
from ∆, and vice versa.
Indeed, let C be any player in Γ, and let C meet D in round i+1. Since C is an i-neighbour
of A, there is a sequence of players A = X1 , X2 , . . . , Xm = C such that Xj meets Xj+1 in one
of the first i rounds, j = 1, 2 . . . , m−1. Let Xj meet Yj in round i+1, for j = 1, 2 . . . , m; in
particular Y1 = B and Ym = D. Players Yj exists in view of condition (i). Suppose that Xj
and Xj+1 met in round r, where r ≤ i. Then condition (ii) implies that and Yj and Yj+1 met
in round r, too. Hence B = Y1 , Y2, . . . , Ym = D is a path in Gi from B to D. This is to say, D
is in the i-component ∆ of B, as claimed. By symmetry, each player from ∆ meets a player
from Γ in round i+1. It follows in particular that Γ and ∆ have the same cardinality.
It is straightforward now that the (i+1)-component of A is Γ ∪ ∆, the union of two sets
with the same size. Since Γ and ∆ are either disjoint or coincide, we have either |Γ ∪ ∆| = 2|Γ|
or |Γ ∪ ∆| = |Γ|; as usual, |· · ·| denotes the cardinality of a finite set.
Let Γ1 , . . . , Γk be the consecutive components of a given player A. We obtained that either
|Γi+1 | = 2|Γi| or |Γi+1 | = |Γi| for i = 1, . . . , k−1. Because |Γ1 | = 2, each |Γi | is a power of 2,
i = 1, . . . , k−1. In particular |Γk | = 2u for some u.
On the other hand, player A has played with k different opponents by (i). All of them
belong to Γk , therefore |Γk | ≥ k+1.
Thus 2u ≥ k+1, and since tk is the least integer satisfying 2tk ≥ k+1, we conclude that
u ≥ tk . So the size of each k-component is divisible by 2tk , which completes the argument.
29
C6. A holey triangle is an upward equilateral triangle of side length n with n upward unit
triangular holes cut out. A diamond is a 60◦ −120◦ unit rhombus. Prove that a holey triangle T
can be tiled with diamonds if and only if the following condition holds: Every upward equilateral
triangle of side length k in T contains at most k holes, for 1 ≤ k ≤ n.
(Colombia)
Solution. Let T be a holey triangle. The unit triangles in it will be called cells. We say simply
“triangle” instead of “upward equilateral triangle” and “size” instead of “side length.”
The necessity will be proven first. Assume that a holey triangle T can be tiled with diamonds
and consider such a tiling. Let T 0 be a triangle of size k in T containing h holes. Focus on the
diamonds which cover (one or two) cells in T 0 . Let them form a figure R. The boundary of T 0
consists of upward cells, so R is a triangle of size k with h upward holes cut out and possibly
some downward cells sticking out. Hence there are exactly (k 2 + k)/2 − h upward cells in R, and
at least (k 2 − k)/2 downward cells (not counting those sticking out). On the other hand each
diamond covers one upward and one downward cell, which implies (k 2 + k)/2 − h ≥ (k 2 − k)/2.
It follows that h ≤ k, as needed.
We pass on to the sufficiency. For brevity, let us say that a set of holes in a given triangle T
is spread out if every triangle of size k in T contains at most k holes. For any set S of spread
out holes, a triangle of size k will be called full of S if it contains exactly k holes of S. The
proof is based on the following observation.
Lemma. Let S be a set of spread out holes in T . Suppose that two triangles T 0 and T 00 are full
of S, and that they touch or intersect. Let T 0 + T 00 denote the smallest triangle in T containing
them. Then T 0 + T 00 is also full of S.
Proof. Let triangles T 0 , T 00 , T 0 ∩ T 00 and T 0 + T 00 have sizes a, b, c and d, and let them contain
a, b, x and y holes of S, respectively. (Note that T 0 ∩ T 00 could be a point, in which case c = 0.)
Since S is spread out, we have x ≤ c and y ≤ d. The geometric configuration of triangles
clearly satisfies a + b = c + d. Furthermore, a + b ≤ x + y, since a + b counts twice the holes in
T 0 ∩ T 00 . These conclusions imply x = c and y = d, as we wished to show.
Now let Tn be a holey triangle of size n, and let the set H of its holes be spread out. We
show by induction on n that Tn can be tiled with diamonds. The base n = 1 is trivial. Suppose
that n ≥ 2 and that the claim holds for holey triangles of size less than n.
Denote by B the bottom row of Tn and by T 0 the triangle formed by its top n − 1 rows.
There is at least one hole in B as T 0 contains at most n − 1 holes. If this hole is only one,
there is a unique way to tile B with diamonds. Also, T 0 contains exactly n − 1 holes, making
it a holey triangle of size n − 1, and these holes are spread out. Hence it remains to apply the
induction hypothesis.
So suppose that there are m ≥ 2 holes in B and label them a1 , . . . , am from left to right. Let
` be the line separating B from T 0 . For each i = 1, . . . , m − 1, pick an upward cell bi between ai
and ai+1 , with base on `. Place a diamond to cover bi and its lower neighbour, a downward
cell in B. The remaining part of B can be tiled uniquely with diamonds. Remove from Tn
row B and the cells b1 , . . . , bm−1 to obtain a holey triangle Tn−1 of size n − 1. The conclusion
will follow by induction if the choice of b1 , . . . , bm−1 guarantees that the following condition
is satisfied: If the holes a1 , . . . , am−1 are replaced by b1 , . . . , bm−1 then the new set of holes is
spread out again.
We show that such a choice is possible. The cells b1 , . . . , bm−1 can be defined one at a time
in this order, making sure that the above condition holds at each step. Thus it suffices to prove
that there is an appropriate choice for b1 , and we set a1 = u, a2 = v for clarity.
30
Let ∆ be the triangle of maximum size which is full of H, contains the top vertex of the
hole u, and has base on line `. Call ∆ the associate of u. Observe that ∆ does not touch v.
Indeed, if ∆ has size r then it contains r holes of Tn . Extending its slanted sides downwards
produces a triangle ∆0 of size r + 1 containing at least one more hole, namely u. Since there
are at most r + 1 holes in ∆0 , it cannot contain v. Consequently, ∆ does not contain the top
vertex of v.
Let w be the upward cell with base on ` which is to the right of ∆ and shares a common
vertex with it. The observation above shows that w is to the left of v. Note that w is not a
hole, or else ∆ could be extended to a larger triangle full of H.
We prove that if the hole u is replaced by w then the new set of holes is spread out again.
To verify this, we only need to check that if a triangle Γ in Tn contains w but not u then Γ is
not full of H. Suppose to the contrary that Γ is full of H. Consider the minimum triangle Γ+∆
containing Γ and the associate ∆ of u. Clearly Γ + ∆ is larger than ∆, because Γ contains w
but ∆ does not. Next, Γ + ∆ is full of H \ {u} by the lemma, since Γ and ∆ have a common
point and neither of them contains u.
Γ+∆
Γ
∆
w
u v
C7. Consider a convex polyhedron without parallel edges and without an edge parallel to
any face other than the two faces adjacent to it.
Call a pair of points of the polyhedron antipodal if there exist two parallel planes passing
through these points and such that the polyhedron is contained between these planes.
Let A be the number of antipodal pairs of vertices, and let B be the number of antipodal
pairs of midpoints of edges. Determine the difference A − B in terms of the numbers of vertices,
edges and faces.
(Japan)
Solution 1. Denote the polyhedron by Γ; let its vertices, edges and faces be V1 , V2 , . . . , Vn ,
E1 , E2 , . . . , Em and F1 , F2 , . . . , F` , respectively. Denote by Qi the midpoint of edge Ei .
Let S be the unit sphere, the set of all unit vectors in three-dimensional space. Map the
boundary elements of Γ to some objects on S as follows.
For a face Fi , let S + (Fi ) and S − (Fi ) be the unit normal vectors of face Fi , pointing outwards
from Γ and inwards to Γ, respectively. These points are diametrically opposite.
For an edge Ej , with neighbouring faces Fi1 and Fi2 , take all support planes of Γ (planes
which have a common point with Γ but do not intersect it) containing edge Ej , and let S + (Ej )
be the set of their outward normal vectors. The set S + (Ej ) is an arc of a great circle on S.
Arc S + (Ej ) is perpendicular to edge Ej and it connects points S + (Fi1 ) and S + (Fi2 ).
Define also the set of inward normal vectors S − (Ei ) which is the reflection of S + (Ei ) across
the origin.
For a vertex Vk , which is the common endpoint of edges Ej1 , . . . , Ejh and shared by faces
Fi1 , . . . , Fih , take all support planes of Γ through point Vk and let S + (Vk ) be the set of their out-
ward normal vectors. This is a region on S, a spherical polygon with vertices S + (Fi1 ), . . . , S + (Fih )
bounded by arcs S + (Ej1 ), . . . , S + (Ejh ). Let S − (Vk ) be the reflection of S + (Vk ), the set of inward
normal vectors.
Note that region S + (Vk ) is convex in the sense that it is the intersection of several half
spheres.
S + (Vk )
Vk
Ej
Fi S + (Fi ) S + (Ej )
Γ S
Lemma 1. For any 1 ≤ i, j ≤ n, regions S − (Vi ) and S + (Vj ) overlap if and only if vertices Vi
and Vj are antipodal.
Lemma 2. For any 1 ≤ i, j ≤ m, arcs S − (Ei ) and S + (Ej ) intersect if and only if the midpoints
Qi and Qj of edges Ei and Ej are antipodal.
Proof of lemma 1. First note that by properties (a,b,c) above, the two regions cannot share
only a single point or an arc. They are either disjoint or they overlap.
Assume that the two regions have a common interior point u. Let P1 and P2 be two parallel
support planes of Γ through points Vi and Vj , respectively, with normal vector u. By the
definition of regions S − (Vi ) and S + (Vj ), u is the inward normal vector of P1 and the outward
normal vector of P2 . Therefore polyhedron Γ lies between the two planes; vertices Vi and Vj
are antipodal.
To prove the opposite direction, assume that Vi and Vj are antipodal. Then there exist two
parallel support planes P1 and P2 through Vi and Vj , respectively, such that Γ is between them.
Let u be the inward normal vector of P1 ; then u is the outward normal vector of P2 , therefore
u ∈ S − (Vi ) ∩ S + (Vj ). The two regions have a common point, so they overlap.
Proof of lemma 2. Again, by properties (a,b) above, the endpoints of arc S − (Ei ) cannot belong
to S + (Ej ) and vice versa. The two arcs are either disjoint or intersecting.
Assume that arcs S − (Ei ) and S + (Ej ) intersect at point u. Let P1 and P2 be the two
support planes through edges Ei and Ej , respectively, with normal vector u. By the definition
of arcs S − (Ei ) and S + (Ej ), vector u points inwards from P1 and outwards from P2 . Therefore
Γ is between the planes. Since planes P1 and P2 pass through Qi and Qj , these points are
antipodal.
For the opposite direction, assume that points Qi and Qj are antipodal. Let P1 and P2
be two support planes through these points, respectively. An edge cannot intersect a support
plane, therefore Ei and Ej lie in the planes P1 and P2 , respectively. Let u be the inward normal
vector of P1 , which is also the outward normal vector of P2 . Then u ∈ S − (Ei ) ∩ S + (Ej ). So
the two arcs are not disjoint; they therefore intersect.
Now create a new decomposition of sphere S. Draw all arcs S + (Ei ) and S − (Ej ) on sphere S
and put a knot at each point where two arcs meet. We have ` knots at points S + (Fi ) and
another ` knots at points S − (Fi ), corresponding to the faces of Γ; by property (c) they are
different. We also have some pairs 1 ≤ i, j ≤ m where arcs S − (Ei ) and S + (Ej ) intersect. By
Lemma 2, each antipodal pair (Qi , Qj ) gives rise to two such intersections; hence, the number
of all intersections is 2B and we have 2` + 2B knots in all.
Each intersection knot splits two arcs, increasing the number of arcs by 2. Since we started
with 2m arcs, corresponding the edges of Γ, the number of the resulting curve segments is
2m + 4B.
The network of these curve segments divides the sphere into some “new” regions. Each new
region is the intersection of some overlapping sets S − (Vi ) and S + (Vj ). Due to the convexity,
the intersection of two overlapping regions is convex and thus contiguous. By Lemma 1, each
pair of overlapping regions corresponds to an antipodal vertex pair and each antipodal vertex
pair gives rise to two different overlaps, which are symmetric with respect to the origin. So the
number of new regions is 2A.
The result now follows from Euler’s polyhedron theorem. We have n + l = m + 2 and
(2` + 2B) + 2A = (2m + 4B) + 2,
therefore
A − B = m − ` + 1 = n − 1.
Therefore A − B is by one less than the number of vertices of Γ.
33
Solution 2. Use the same notations for the polyhedron and its vertices, edges and faces as
in Solution 1. We regard points as vectors starting from the origin. Polyhedron Γ is regarded
as a closed convex set, including its interior. In some cases the edges and faces of Γ are also
regarded as sets of points. The symbol ∂ denotes the boundary of the certain set; e.g. ∂Γ is
the surface of Γ.
Let ∆ = Γ − Γ = {U − V : U, V ∈ Γ} be the set of vectors between arbitrary points of
Γ. Then ∆, being the sum of two bounded convex sets, is also a bounded convex set and, by
construction, it is also centrally symmetric with respect to the origin. We will prove that ∆ is
also a polyhedron and express the numbers of its faces, edges and vertices in terms n, m, `, A
and B.
Lemma 1. For points U, V ∈ Γ, point W = U − V is a boundary point of ∆ if and only if U
and V are antipodal. Moreover, for each boundary point W ∈ ∂∆ there exists exactly one pair
of points U, V ∈ Γ such that W = U − V .
Proof. Assume first that U and V are antipodal points of Γ. Let parallel support planes
P1 and P2 pass through them such that Γ is in between. Consider plane P = P1 − U =
P2 − V . This plane separates the interiors of Γ − U and Γ − V . After reflecting one of the
sets, e.g. Γ − V , the sets Γ − U and −Γ + V lie in the same half space bounded by P . Then
(Γ − U) + (−Γ + V ) = ∆ − W lies in that half space, so 0 ∈ P is a boundary point of the set
∆ − W . Translating by W we obtain that W is a boundary point of ∆.
To prove the opposite direction, let W = U − V be a boundary point of ∆, and let Ψ =
(Γ − U) ∩ (Γ − V ). We claim that Ψ = {0}. Clearly Ψ is a bounded convex set and 0 ∈ Ψ. For
any two points X, Y ∈ Ψ, we have U +X, V +Y ∈ Γ and W +(X −Y ) = (U +X)−(V +Y ) ∈ ∆.
Since W is a boundary point of ∆, the vector X −Y cannot have the same direction as W . This
implies that the interior of Ψ is empty. Now suppose that Ψ contains a line segment S. Then
S + U and S + V are subsets of some faces or edges of Γ and these faces/edges are parallel to S.
In all cases, we find two faces, two edges, or a face and an edge which are parallel, contradicting
the conditions of the problem. Therefore, Ψ = {0} indeed.
Since Ψ = (Γ−U)∩(Γ−V ) consists of a single point, the interiors of bodies Γ−U and Γ−V
are disjoint and there exists a plane P which separates them. Let u be the normal vector of P
pointing into that half space bounded by P which contains Γ − U. Consider the planes P + U
and P + V ; they are support planes of Γ, passing through U and V , respectively. From plane
P + U, the vector u points into that half space which contains Γ. From plane P + V , vector
u points into the opposite half space containing Γ. Therefore, we found two proper support
through points U and V such that Γ is in between.
For the uniqueness part, assume that there exist points U1 , V1 ∈ Γ such that U1 −V1 = U −V .
The points U1 − U and V1 − V lie in the sets Γ − U and Γ − V separated by P . Since
U1 − U = V1 − V , this can happen only if both are in P ; but the only such point is 0. Therefore,
U1 − V1 = U − V implies U1 = U and V1 = V . The lemma is complete.
Lemma 2. Let U and V be two antipodal points and assume that plane P , passing through
0, separates the interiors of Γ − U and Γ − V . Let Ψ1 = (Γ − U) ∩ P and Ψ2 = (Γ − V ) ∩ P .
Then ∆ ∩ (P + U − V ) = Ψ1 − Ψ2 + U − V .
Proof. The sets Γ − U and −Γ + V lie in the same closed half space bounded by P . Therefore,
for any points X ∈ (Γ − U) and Y ∈ (−Γ + V ), we have X + Y ∈ P if and only if X, Y ∈ P .
Then
(∆ − (U − V )) ∩ P = (Γ − U) + (−Γ + V ) ∩ P = (Γ − U) ∩ P + (−Γ + V ) ∩ P = Ψ1 − Ψ2 .
Now classify the boundary points W = U − V of ∆, according to the types of points U and
V . In all cases we choose a plane P through 0 which separates the interiors of Γ − U and Γ − V .
We will use the notation Ψ1 = (Γ − U) ∩ P and Ψ2 = (Γ − V ) ∩ P as well.
Case 1: Both U and V are vertices of Γ. Bodies Γ − U and Γ − V have a common vertex
which is 0. Choose plane P in such a way that Ψ1 = Ψ2 = {0}. Then Lemma 2 yields
∆ ∩ (P + W ) = {W }. Therefore P + W is a support plane of ∆ such that they have only one
common point so no line segment exists on ∂∆ which would contain W in its interior.
Since this case occurs for antipodal vertex pairs and each pair is counted twice, the number
of such boundary points on ∆ is 2A.
Case 2: Point U is an interior point of an edge Ei and V is a vertex of Γ. Choose plane
P such that Ψ1 = Ei − U and Ψ2 = {0}. By Lemma 2, ∆ ∩ (P + W ) = Ei − V . Hence there
exists a line segment in ∂∆ which contains W in its interior, but there is no planar region in
∂∆ with the same property.
We obtain a similar result if V belongs to an edge of Γ and U is a vertex.
Case 3: Points U and V are interior points of edges Ei and Ej , respectively. Let P be the
plane of Ei − U and Ej − V . Then Ψ1 = Ei − U, Ψ2 = Ej − V and ∆ ∩ (P + W ) = Ei − Ej .
Therefore point W belongs to a parallelogram face on ∂∆.
The centre of the parallelogram is Qi − Qj , the vector between the midpoints. Therefore an
edge pair (Ei , Ej ) occurs if and only if Qi and Qj are antipodal which happens 2B times.
Case 4: Point U lies in the interior of a face Fi and V is a vertex of Γ. The only choice for
P is the plane of Fi − U. Then we have Ψ1 = Fi − U, Ψ2 = {0} and ∆ ∩ (P + W ) = Fi − V .
This is a planar face of ∂∆ which is congruent to Fi .
For each face Fi , the only possible vertex V is the farthest one from the plane of Fi .
If U is a vertex and V belongs to face Fi then we obtain the same way that W belongs to
a face −Fi + U which is also congruent to Fi . Therefore, each face of Γ has two copies on ∂∆,
a translated and a reflected copy.
Case 5: Point U belongs to a face Fi of Γ and point V belongs to an edge or a face G. In
this case objects Fi and G must be parallel which is not allowed.
Γ−V Γ−V Γ−V Γ−V
P P P P
0 0 0 0
Now all points in ∂∆ belong to some planar polygons (cases 3 and 4), finitely many line
segments (case 2) and points (case 1). Therefore ∆ is indeed a polyhedron. Now compute the
numbers of its vertices, edges and faces.
The vertices are obtained in case 1, their number is 2A.
Faces are obtained in cases 3 and 4. Case 3 generates 2B parallelogram faces. Case 4
generates 2` faces.
We compute the number of edges of ∆ from the degrees (number of sides) of faces of Γ. Let
di be the the degree of face Fi . The sum of degrees is twice as much as the number of edges, so
d1 +d2 +. . .+dl = 2m. The sum of degrees of faces of ∆ is 2B ·4+2(d1 +d2 +· · ·+dl ) = 8B +4m,
so the number of edges on ∆ is 4B + 2m.
Applying Euler’s polyhedron theorem on Γ and ∆, we have n+l = m+2 and 2A+(2B+2`) =
(4B + 2m) + 2. Then the conclusion follows:
A − B = m − ` + 1 = n − 1.
Geometry
G1. Let ABC be a triangle with incentre I. A point P in the interior of the triangle satisfies
∠P BA + ∠P CA = ∠P BC + ∠P CB.
Show that AP ≥ AI and that equality holds if and only if P coincides with I.
(Korea)
M
Ω
P ω
A B
Let Ω be the circumcircle of triangle ABC. It is a well-known fact that the centre of ω
is the midpoint M of the arc BC of Ω. This is also the point where the angle bisector AI
intersects Ω.
From triangle AP M we have
AP + P M ≥ AM = AI + IM = AI + P M.
Therefore AP ≥ AI. Equality holds if and only if P lies on the line segment AI, which occurs
if and only if P = I.
36
G2. Let ABCD be a trapezoid with parallel sides AB > CD. Points K and L lie on the
line segments AB and CD, respectively, so that AK/KB = DL/LC. Suppose that there are
points P and Q on the line segment KL satisfying
Solution 1. Because AB k CD, the relation AK/KB = DL/LC readily implies that the lines
AD, BC and KL have a common point S.
S
D L C
A K B
Consider the second intersection points X and Y of the line SK with the circles (ABP )
and (CDQ), respectively. Since AP BX is a cyclic quadrilateral and AB k CD, one has
This shows that BC is tangent to the circle (ABP ) at B. Likewise, BC is tangent to the
circle (CDQ) at C. Therefore SP · SX = SB 2 and SQ · SY = SC 2 .
Let h be the homothety with centre S and ratio SC/SB. Since h(B) = C, the above
conclusion about tangency implies that h takes circle (ABP ) to circle (CDQ). Also, h takes AB
to CD, and it easily follows that h(P ) = Y , h(X) = Q, yielding SP/SY = SB/SC = SX/SQ.
Equalities SP · SX = SB 2 and SQ/SX = SC/SB imply SP · SQ = SB · SC, which is
equivalent to P , Q, B and C being concyclic.
37
Solution 2. The case where P = Q is trivial. Thus assume that P and Q are two distinct
points. As in the first solution, notice that the lines AD, BC and KL concur at a point S.
S
D L C
P
E F
A K B
Let the lines AP and DQ meet at E, and let BP and CQ meet at F . Then ∠EP F = ∠BCD
and ∠F QE = ∠ABC by the condition of the problem. Since the angles BCD and ABC add
up to 180◦ , it follows that P EQF is a cyclic quadrilateral.
Applying Menelaus’ theorem, first to triangle ASP and line DQ and then to triangle BSP
and line CQ, we have
AD SQ P E BC SQ P F
· · = 1 and · · = 1.
DS QP EA CS QP F B
The first factors in these equations are equal, as ABkCD. Thus the last factors are also equal,
which implies that EF is parallel to AB and CD. Using this and the cyclicity of P EQF , we
obtain
∠BCD = ∠BCF + ∠F CD = ∠BCQ + ∠EF Q = ∠BCQ + ∠EP Q.
On the other hand,
and consequently ∠BCQ = ∠QP F . The latter angle either coincides with ∠QP B or is sup-
plementary to ∠QP B, depending on whether Q lies between K and P or not. In either case it
follows that P , Q, B and C are concyclic.
38
The diagonals BD and CE meet at P . Prove that the line AP bisects the side CD.
(USA)
Solution. Let the diagonals AC and BD meet at Q, the diagonals AD and CE meet at R,
and let the ray AP meet the side CD at M. We want to prove that CM = MD holds.
E
A
R
P
B D
Q
M
C
The idea is to show that Q and R divide AC and AD in the same ratio, or more precisely
AQ AR
= (1)
QC RD
(which is equivalent to QRkCD). The given angle equalities imply that the triangles ABC,
ACD and ADE are similar. We therefore have
AB AC AD
= = .
AC AD AE
Since ∠BAD = ∠BAC + ∠CAD = ∠CAD + ∠DAE = ∠CAE, it follows from AB/AC =
AD/AE that the triangles ABD and ACE are also similar. Their angle bisectors in A are AQ
and AR, respectively, so that
AB AQ
= .
AC AR
Because AB/AC = AC/AD, we obtain AQ/AR = AC/AD, which is equivalent to (1). Now
Ceva’s theorem for the triangle ACD yields
AQ CM DR
· · = 1.
QC MD RA
G4. A point D is chosen on the side AC of a triangle ABC with ∠C < ∠A < 90◦ in such
a way that BD = BA. The incircle of ABC is tangent to AB and AC at points K and L,
respectively. Let J be the incentre of triangle BCD. Prove that the line KL intersects the line
segment AJ at its midpoint.
(Russia)
Solution. Denote by P be the common point of AJ and KL. Let the parallel to KL through
J meet AC at M. Then P is the midpoint of AJ if and only if AM = 2 · AL, which we are
about to show.
B
J
P
C M T D L A
AM = AD + (BD + CD − BC)
= AD + AB + DC − BC
= AC + AB − BC
= 2 · AL,
G5. In triangle ABC, let J be the centre of the excircle tangent to side BC at A1 and to
the extensions of sides AC and AB at B1 and C1 , respectively. Suppose that the lines A1 B1
and AB are perpendicular and intersect at D. Let E be the foot of the perpendicular from C1
to line DJ. Determine the angles ∠BEA1 and ∠AEB1 .
(Greece)
B1
C K
J
A1 E
A D B C1
Since ∠CA1 J = ∠CB1 J = ∠CEJ = 90◦ , points A1 , B1 and E lie on the circle of diameter
CJ. Then ∠DBA1 = ∠A1 CJ = ∠DEA1 , which implies that quadrilateral BEA1 D is cyclic;
therefore ∠A1 EB = 90◦ .
Quadrilateral ADEB1 is also cyclic because ∠EB1 A = ∠EJC = ∠EDC1 , therefore we
obtain ∠AEB1 = ∠ADB = 90◦ .
B1
C
ω3 J
A1 E
ω2
A D B C1
ω1
41
B1
C J
ω
A1 O=E
A D B C1
In triangle B1 A1 J, line JC1 is the external bisector at vertex J. The point C1 is the
intersection of two external angle bisectors (at A1 and J) so C1 is the centre of the excircle ω,
tangent to side A1 J, and to the extension of B1 A1 at point D.
Now consider the similarity transform ϕ which moves B1 to A, A1 to B and J to C. This
similarity can be decomposed into a rotation by 90◦ around a certain point O and a homothety
from the same centre. This similarity moves point C1 (the centre of excircle ω) to J and moves
D (the point of tangency) to C1 .
Since the rotation angle is 90◦ , we have ∠XOϕ(X) = 90◦ for an arbitrary point X 6= O.
For X = D and X = C1 we obtain ∠DOC1 = ∠C1 OJ = 90◦ . Therefore O lies on line segment
DJ and C1 O is perpendicular to DJ. This means that O = E.
For X = A1 and X = B1 we obtain ∠A1 OB = ∠B1 OA = 90◦ , i.e.
∠BEA1 = ∠AEB1 = 90◦ .
Comment. Choosing X = J, it also follows that ∠JEC = 90◦ which proves that lines DJ and CC1
intersect at point E. However, this is true more generally, without the assumption that A1 B1 and
AB are perpendicular, because points C and D are conjugates with respect to the excircle. The last
observation could replace the first paragraph of Solution 1.
42
G6. Circles ω1 and ω2 with centres O1 and O2 are externally tangent at point D and
internally tangent to a circle ω at points E and F , respectively. Line t is the common tangent
of ω1 and ω2 at D. Let AB be the diameter of ω perpendicular to t, so that A, E and O1 are
on the same side of t. Prove that lines AO1 , BO2 , EF and t are concurrent.
(Brasil)
Solution 1. Point E is the centre of a homothety h which takes circle ω1 to circle ω. The
radii O1 D and OB of these circles are parallel as both are perpendicular to line t. Also, O1 D
and OB are on the same side of line EO, hence h takes O1 D to OB. Consequently, points E,
D and B are collinear. Likewise, points F , D and A are collinear as well.
Let lines AE and BF intersect at C. Since AF and BE are altitudes in triangle ABC, their
common point D is the orthocentre of this triangle. So CD is perpendicular to AB, implying
that C lies on line t. Note that triangle ABC is acute-angled. We mention the well-known fact
that triangles F EC and ABC are similar in ratio cos γ, where γ = ∠ACB. In addition, points
C, E, D and F lie on the circle with diameter CD.
C
γ
E
P
ω F
O1 D
N
O2
ω2
ω1 t
A V O K B
U L
Let P be the common point of lines EF and t. We are going to prove that P lies on
line AO1 . Denote by N the second common point of circle ω1 and AC; this is the point of ω1
diametrically opposite to D. By Menelaus’ theorem for triangle DCN, points A, O1 and P are
collinear if and only if
CA NO1 DP
· · = 1.
AN O1 D P C
Because NO1 = O1 D, this reduces to CA/AN = CP/P D. Let line t meet AB at K. Then
CA/AN = CK/KD, so it suffices to show that
CP CK
= . (1)
PD KD
To verify (1), consider the circumcircle Ω of triangle ABC. Draw its diameter CU through C,
and let CU meet AB at V . Extend CK to meet Ω at L. Since AB is parallel to UL, we have
∠ACU = ∠BCL. On the other hand ∠EF C = ∠BAC, ∠F EC = ∠ABC and EF/AB = cos γ,
as stated above. So reflection in the bisector of ∠ACB followed by a homothety with centre C
and ratio 1/ cos γ takes triangle F EC to triangle ABC. Consequently, this transformation
43
M
E
Q
F
O1
O2
D
ω2
ω1 t
A O B
Consider triangles ABC and O1 O2 M. Lines O1 O2 and AB are parallel, both of them being
perpendicular to line t. Next, MO1 is the line of centres of circles (CEF ) and ω1 whose common
chord is DE. Hence MO1 bisects ∠DME which is the external angle at M in the isosceles
triangle CEM. It follows that ∠DMO1 = ∠DCA, so that MO1 is parallel to AC. Likewise,
MO2 is parallel to BC.
Thus the respective sides of triangles ABC and O1 O2 M are parallel; in addition, these
triangles are not congruent. Hence there is a homothety taking ABC to O1 O2 M. The lines AO1 ,
BO2 and CM = t are concurrent at the centre Q of this homothety.
Finally, apply Pappus’ theorem to the triples of collinear points A, O, B and O2 , D, O1 .
The theorem implies that the points AD ∩ OO2 = F , AO1 ∩ BO2 = Q and OO1 ∩ BD = E are
collinear. In other words, line EF passes through the common point Q of AO1 , BO2 and t.
Comment. Relation (1) from Solution 1 expresses the well-known fact that points P and K are
harmonic conjugates with respect to points C and D. It is also easy to justify it by direct computation.
Denoting ∠CAB = α, ∠ABC = β, it is straightforward to obtain CP/P D = CK/KD = tan α tan β.
44
45
G7. In a triangle ABC, let Ma , Mb , Mc be respectively the midpoints of the sides BC, CA,
AB and Ta , Tb , Tc be the midpoints of the arcs BC, CA, AB of the circumcircle of ABC, not
containing the opposite vertices. For i ∈ {a, b, c}, let ωi be the circle with Mi Ti as diameter.
Let pi be the common external tangent to ωj , ωk ({i, j, k} = {a, b, c}) such that ωi lies on the
opposite side of pi than ωj , ωk do. Prove that the lines pa , pb , pc form a triangle similar to ABC
and find the ratio of similitude.
(Slovakia)
Solution. Let Ta Tb intersect circle ωb at Tb and U, and let Ta Tc intersect circle ωc at Tc and V .
Further, let UX be the tangent to ωb at U, with X on AC, and let V Y be the tangent to ωc
at V , with Y on AB. The homothety with centre Tb and ratio Tb Ta /Tb U maps the circle ωb
onto the circumcircle of ABC and the line UX onto the line tangent to the circumcircle at Ta ,
which is parallel to BC; thus UXkBC. The same is true of V Y , so that UXkBCkV Y .
Let Ta Tb cut AC at P and let Ta Tc cut AB at Q. The point X lies on the hypotenuse P Mb
of the right triangle P UMb and is equidistant from U and Mb . So X is the midpoint of Mb P .
Similarly Y is the midpoint of Mc Q.
Denote the incentre of triangle ABC as usual by I. It is a known fact that Ta I = Ta B
and Tc I = Tc B. Therefore the points B and I are symmetric across Ta Tc , and consequently
∠QIB = ∠QBI = ∠IBC. This implies that BC is parallel to the line IQ, and likewise, to IP .
In other words, P Q is the line parallel to BC passing through I.
ωb
Tb
ωc
Tc
Mc Mb
V U
Y X
I
Q P
Ma
B C
ωa
Ta
G8. Let ABCD be a convex quadrilateral. A circle passing through the points A and D
and a circle passing through the points B and C are externally tangent at a point P inside the
quadrilateral. Suppose that
Solution. We start with a preliminary observation. Let T be a point inside the quadrilateral
ABCD. Then:
Circles (BCT ) and (DAT ) are tangent at T
if and only if ∠ADT + ∠BCT = ∠AT B. (1)
Indeed, if the two circles touch each other then their common tangent at T intersects the
segment AB at a point Z, and so ∠ADT = ∠AT Z, ∠BCT = ∠BT Z, by the tangent-chord
theorem. Thus ∠ADT + ∠BCT = ∠AT Z + ∠BT Z = ∠AT B.
And conversely, if ∠ADT + ∠BCT = ∠AT B then one can draw from T a ray T Z with Z
on AB so that ∠ADT = ∠AT Z, ∠BCT = ∠BT Z. The first of these equalities implies that
T Z is tangent to the circle (DAT ); by the second equality, T Z is tangent to the circle (BCT ),
so the two circles are tangent at T .
D
C
Z
A
So the equivalence (1) is settled. It will be used later on. Now pass to the actual solution.
Its key idea is to introduce the circumcircles of triangles ABP and CDP and to consider their
second intersection Q (assume for the moment that they indeed meet at two distinct points P
and Q).
Since the point A lies outside the circle (BCP ), we have ∠BCP + ∠BAP < 180◦ . Therefore
the point C lies outside the circle (ABP ). Analogously, D also lies outside that circle. It follows
that P and Q lie on the same arc CD of the circle (BCP ).
C
P Q
B
A
47
By symmetry, P and Q lie on the same arc AB of the circle (ABP ). Thus the point Q lies
either inside the angle BP C or inside the angle AP D. Without loss of generality assume that
Q lies inside the angle BP C. Then
The last sum is equal to ∠AP B, according to the observation (1) applied to T = P . And
because ∠AP B = ∠AQB, we obtain
Applying now (1) to T = Q we conclude that the circles (BCQ) and (DAQ) are externally
tangent at Q. (We have assumed P 6= Q; but if P = Q then the last conclusion holds trivially.)
Finally consider the halfdiscs with diameters BC and DA constructed inwardly to the
quadrilateral ABCD. They have centres at M and N, the midpoints of BC and DA re-
spectively. In view of (2) and (3), these two halfdiscs lie entirely inside the circles (BQC)
and (AQD); and since these circles are tangent, the two halfdiscs cannot overlap. Hence
MN ≥ 21 BC + 21 DA.
−−→ −→ −−→
On the other hand, since MN = 21 (BA + CD ), we have MN ≤ 21 (AB + CD). Thus indeed
AB + CD ≥ BC + DA, as claimed.
48
G9. Points A1 , B1 , C1 are chosen on the sides BC, CA, AB of a triangle ABC, respectively.
The circumcircles of triangles AB1 C1 , BC1 A1 , CA1 B1 intersect the circumcircle of triangle
ABC again at points A2 , B2 , C2 , respectively (A2 6= A, B2 6= B, C2 6= C). Points A3 , B3 , C3 are
symmetric to A1 , B1 , C1 with respect to the midpoints of the sides BC, CA, AB respectively.
Prove that the triangles A2 B2 C2 and A3 B3 C3 are similar.
(Russia)
Solution. We will work with oriented angles between lines. For two straight lines `, m in the
plane, ∠(`, m) denotes the angle of counterclockwise rotation which transforms line ` into a
line parallel to m (the choice of the rotation centre is irrelevant). This is a signed quantity;
values differing by a multiple of π are identified, so that
If ` is the line through points K, L and m is the line through M, N, one writes ∠(KL, MN)
for ∠(`, m); the characters K, L are freely interchangeable; and so are M, N.
The counterpart of the classical theorem about cyclic quadrilaterals is the following:
If K, L, M, N are four noncollinear points in the plane then
Passing to the solution proper, we first show that the three circles (AB1 C1 ), (BC1 A1 ),
(CA1 B1 ) have a common point. So, let (AB1 C1 ) and (BC1 A1 ) intersect at the points C1
and P . Then by (1)
A A
A2 A2
ϕ
B4
B1
ϕ
C1
ϕ O 2ϕ
A1
B C B C
ϕ
C4
P P
C2 C2
B2 B2
A4
49
A
ϕ
C3 B4
B1 B4
A5
B3
ϕ
C1 A5 2ϕ
ϕ
O C4
B C
A1 A3 P
ϕ
C4
P B5
C5
ϕ
B5 A4
C5
A4
i.e., ∠(B4 C5 , C5 C1 ) = ϕ. This combined with ∠(C5 C1 , C1 A) = ∠(P C1 , AB) = ϕ (see (2)) proves
that the quadrilateral AB4 C5 C1 is an isosceles trapezoid with AC1 = B4 C5 .
Interchanging the roles of A and B we infer that also BC1 = A4 C5 . And since AC1 + BC1 =
AB = A4 B4 , it follows that the point C5 lies on the line segment A4 B4 and partitions it into
segments A4 C5 , B4 C5 of lengths BC1 (= AC3 ) and AC1 (= BC3 ). In other words, the rotation
which maps triangle A4 B4 C4 onto ABC carries C5 onto C3 . Likewise, it sends A5 to A3 and
B5 to B3 . So the triangles A3 B3 C3 and A5 B5 C5 are congruent. It now suffices to show that the
latter is similar to A2 B2 C2 .
Lines B4 C5 and P C5 coincide respectively with A4 B4 and P C1 . Thus by (4)
∠(B4 C5 , P C5 ) = ϕ.
Analogously (by cyclic shift) ϕ = ∠(C4 A5 , P A5), which rewrites as
ϕ = ∠(B4 A5 , P A5 ).
50
These relations imply that the points P, B4 , C5 , A5 are concyclic. Analogously, P, C4, A5 , B5
and P, A4, B5 , C5 are concyclic quadruples. Therefore
On the other hand, since the points A2 , B2 , C2 , A4 , B4 , C4 all lie on the circle (ABC), we have
G10. To each side a of a convex polygon we assign the maximum area of a triangle contained
in the polygon and having a as one of its sides. Show that the sum of the areas assigned to all
sides of the polygon is not less than twice the area of the polygon.
(Serbia)
Solution 1.
Lemma. Every convex (2n)-gon, of area S, has a side and a vertex that jointly span a triangle
of area not less than S/n.
Proof. By main diagonals of the (2n)-gon we shall mean those which partition the (2n)-gon
into two polygons with equally many sides. For any side b of the (2n)-gon denote by ∆b the
triangle ABP where A, B are the endpoints of b and P is the intersection point of the main
diagonals AA0 , BB 0 . We claim that the union of triangles ∆b , taken over all sides, covers the
whole polygon.
To show this, choose any side AB and consider the main diagonal AA0 as a directed segment.
Let X be any point in the polygon, not on any main diagonal. For definiteness, let X lie on the
left side of the ray AA0 . Consider the sequence of main diagonals AA0 , BB 0 , CC 0 , . . . , where
A, B, C, . . . are consecutive vertices, situated right to AA0 .
The n-th item in this sequence is the diagonal A0 A (i.e. AA0 reversed), having X on its
right side. So there are two successive vertices K, L in the sequence A, B, C, . . . before A0 such
that X still lies to the left of KK 0 but to the right of LL0 . And this means that X is in the
triangle ∆`0 , `0 = K 0 L0 . Analogous reasoning applies to points X on the right of AA0 (points
lying on main diagonals can be safely ignored). Thus indeed the triangles ∆b jointly cover the
whole polygon.
The sum of their areas is no less than S. So we can find two opposite sides, say b = AB
and b0 = A0 B 0 (with AA0 , BB 0 main diagonals) such that [∆b ] + [∆b0 ] ≥ S/n, where [· · · ] stands
for the area of a region. Let AA0 , BB 0 intersect at P ; assume without loss of generality that
P B ≥ P B 0 . Then
[W ] = ki · [T ] ≥ ki · S/n = qi · S > Si ,
Solution 2. As in the first solution, we allow again angles of size 180◦ at some vertices of the
convex polygons considered.
To each convex n-gon P = A1 A2 . . . An we assign a centrally symmetric convex (2n)-gon Q
−−−−→
with side vectors ±Ai Ai+1 , 1 ≤ i ≤ n. The construction is as follows. Attach the 2n vectors
−−−−→ −
→ − → −→
±Ai Ai+1 at a common origin and label them b1 , b2 , . . . , b2n in counterclockwise direction; the
−
→
choice of the first vector b1 is irrelevant. The order of labelling is well-defined if P has neither
parallel sides nor angles equal to 180◦ . Otherwise several collinear vectors with the same
−
→ −−→ −−→
direction are labelled consecutively bj , bj+1 , . . . , bj+r . One can assume that in such cases the
−
→ −−→ −−→ −−→ −
→
respective opposite vectors occur in the order −bj , −bj+1 , . . . , −bj+r , ensuring that bj+n = −bj
for j = 1, . . . , 2n. Indices are taken cyclically here and in similar situations below.
−−−−→ − →
Choose points B1 , B2 , . . . , B2n satisfying Bj Bj+1 = bj for j = 1, . . . , 2n. The polygonal line
P − → − →
Q = B1 B2 . . . B2n is closed, since 2n j=1 bj = 0 . Moreover, Q is a convex (2n)-gon due to the
−
→ −−−−→
arrangement of the vectors bj , possibly with 180◦ -angles. The side vectors of Q are ±Ai Ai+1 ,
−−−−→
1 ≤ i ≤ n. So in particular Q is centrally symmetric, because it contains as side vectors Ai Ai+1
−−−−→
and −Ai Ai+1 for each i = 1, . . . , n. Note that Bj Bj+1 and Bj+n Bj+n+1 are opposite sides of Q,
1 ≤ j ≤ n. We call Q the associate of P.
Let Si be the maximum area of a triangle with side Ai Ai+1 in P, 1 ≤ i ≤ n. We prove that
n
X
[B1 B2 . . . B2n ] = 2 Si (1)
i=1
and
[B1 B2 . . . B2n ] ≥ 4 [A1 A2 . . . An ] . (2)
It is clear that (1) and (2) imply the conclusion of the original problem.
Lemma. For a side Ai Ai+1 of P, let hi be the maximum distance from a point of P to line Ai Ai+1 ,
−−−−→ −−−−→
i = 1, . . . , n. Denote by Bj Bj+1 the side of Q such that Ai Ai+1 = Bj Bj+1 . Then the distance
between Bj Bj+1 and its opposite side in Q is equal to 2hi .
Proof. Choose a vertex Ak of P at distance hi from line Ai Ai+1 . Let u be the unit vector
perpendicular to Ai Ai+1 and pointing inside P. Denoting by x · y the dot product of vectors x
and y, we have
−−−→ −−−−→ −−−−−→ −−−−→ −−−−−→
h = u · Ai Ak = u · (Ai Ai+1 + · · · + Ak−1 Ak ) = u · (Ai Ai−1 + · · · + Ak+1 Ak ).
In Q, the distance Hi between the opposite sides Bj Bj+1 and Bj+n Bj+n+1 is given by
−−−−→ −−−−−−−−→ −
→ −−→ −−−−→
Hi = u · (Bj Bj+1 + · · · + Bj+n−1 Bj+n ) = u · (bj + bj+1 + · · · + bj+n−1 ).
−
→ −−→ −−−−→
The choice of vertex Ak implies that the n consecutive vectors bj , bj+1 , . . . , bj+n−1 are precisely
−−−−→ −−−−−→ −−−−→ −−−−−→
Ai Ai+1 , . . . , Ak−1 Ak and Ai Ai−1 , . . . , Ak+1Ak , taken in some order. This implies Hi = 2hi .
For a proof of (1), apply the lemma to each side of P. If O the centre of Q then, using the
notation of the lemma,
number ` of side directions of P, i. e. the number of pairwise nonparallel lines each containing
a side of P.
We choose to start the induction with ` = 1 as a base case, meaning that certain degen-
erate polygons are allowed. More exactly, we regard as degenerate convex polygons all closed
polygonal lines of the form X1 X2 . . . Xk Y1 Y2 . . . Ym X1 , where X1 , X2 , . . . , Xk are points in this
order on a line segment X1 Y1 , and so are Ym , Ym−1 , . . . , Y1 . The initial construction applies to
degenerate polygons; their associates are also degenerate, and the value of d is zero. For the
inductive step, consider a convex polygon P which determines ` side directions, assuming that
d(P) ≥ 0 for polygons with smaller values of `.
Suppose first that P has a pair of parallel sides, i. e. sides on distinct parallel lines. Let
Ai Ai+1 and Aj Aj+1 be such a pair, and let Ai Ai+1 ≤ Aj Aj+1. Remove from P the parallelo-
−−−−→ −−−−→
gram R determined by vectors Ai Ai+1 and Ai Aj+1 . Two polygons are obtained in this way.
−−−−→
Translating one of them by vector Ai Ai+1 yields a new convex polygon P 0 , of area [P] − [R]
and with value of ` not exceeding the one of P. The construction just described will be called
operation A.
Aj+1 Aj
P
R
Q
Ai Ai+1 Q0
P0
The associate of P 0 is obtained from Q upon decreasing the lengths of two opposite sides
by an amount of 2Ai Ai+1 . By the lemma, the distance between these opposite sides is twice
the distance between Ai Ai+1 and Aj Aj+1 . Thus operation A decreases [Q] by the area of a
parallelogram with base and respective altitude twice the ones of R, i. e. by 4[R]. Hence A
leaves the difference d(P) = [Q] − 4[P] unchanged.
Now, if P 0 also has a pair of parallel sides, apply operation A to it. Keep doing so with
the subsequent polygons obtained for as long as possible. Now, A decreases the number p of
pairs of parallel sides in P. Hence its repeated applications gradually reduce p to 0, and further
applications of A will be impossible after several steps. For clarity, let us denote by P again
the polygon obtained at that stage.
The inductive step is complete if P is degenerate. Otherwise ` > 1 and p = 0, i. e. there
are no parallel sides in P. Observe that then ` ≥ 3. Indeed, ` = 2 means that the vertices of P
all lie on the boundary of a parallelogram, implying p > 0. −→
Furthermore, since P has no parallel sides, consecutive collinear vectors in the sequence bk
(if any) correspond to consecutive 180◦ -angles in P. Removing the vertices of such angles, we
obtain a convex polygon with the same value of d(P).
In summary, if operation A is impossible for a nondegenerate polygon P, then ` ≥ 3. In
addition, one may assume that P has no angles of size 180◦ .
The last two conditions then also hold for the associate Q of P, and we perform the fol-
lowing construction. Since ` ≥ 3, there is a side Bj Bj+1 of Q such that the sum of the angles
at Bj and Bj+1 is greater than 180◦. (Such a side exists in each convex k-gon for k > 4.) Natu-
rally, Bj+n Bj+n+1 is a side with the same property. Extend the pairs of sides Bj−1 Bj , Bj+1 Bj+2
54
and Bj+n−1 Bj+n , Bj+n+1Bj+n+2 to meet at U and V , respectively. Let Q0 be the centrally sym-
metric convex 2(n+1)-gon obtained from Q by inserting U and V into the sequence B1 , . . . , B2n
as new vertices between Bj , Bj+1 and Bj+n , Bj+n+1, respectively. Informally, we adjoin to Q
the congruent triangles Bj Bj+1 U and Bj+n Bj+n+1V . Note that Bj , Bj+1 , Bj+n and Bj+n+1 are
kept as vertices of Q0 , although Bj Bj+1 and Bj+n Bj+n+1 are no longer its sides.
−−−−→ −−−−→ − →
Let Ai Ai+1 be the side of P such that Ai Ai+1 = Bj Bj+1 = bj . Consider the point W such
that triangle Ai Ai+1 W is congruent to triangle Bj Bj+1U and exterior to P. Insert W into the
sequence A1 , A2 , . . . , An as a new vertex between Ai and Ai+1 to obtain an (n+1)-gon P 0 . We
claim that P 0 is convex and its associate is Q0 .
Ai−1
Ai Bj Bj+n+1
W P V
U Q
(USA)
Solution. If (x, y) is a solution then obviously x ≥ 0 and (x, −y) is a solution too. For x = 0
we get the two solutions (0, 2) and (0, −2).
Now let (x, y) be a solution with x > 0; without loss of generality confine attention to y > 0.
The equation rewritten as
2x (1 + 2x+1 ) = (y − 1)(y + 1)
shows that the factors y − 1 and y + 1 are even, exactly one of them divisible by 4. Hence
x ≥ 3 and one of these factors is divisible by 2x−1 but not by 2x . So
or, equivalently
1 + 2x+1 = 2x−2 m2 + m.
Therefore
1 − m = 2x−2 (m2 − 8). (2)
For = 1 this yields m2 − 8 ≤ 0, i.e., m = 1, which fails to satisfy (2).
For = −1 equation (2) gives us
N2. For x ∈ (0, 1) let y ∈ (0, 1) be the number whose nth digit after the decimal point is the
(2n )th digit after the decimal point of x. Show that if x is rational then so is y.
(Canada)
Solution. Since x is rational, its digits repeat periodically starting at some point. We wish to
show that this is also true for the digits of y, implying that y is rational.
Let d be the length of the period of x and let d = 2u · v, where v is odd. There is a positive
integer w such that
2w ≡ 1 (mod v).
(For instance, one can choose w to be ϕ(v), the value of Euler’s function at v.) Therefore
2n+w = 2n · 2w ≡ 2n (mod v)
2n+w ≡ 2n ≡ 0 (mod 2u ).
2n+w ≡ 2n (mod d)
holds. Thus, for n sufficiently large, the 2n+w th digit of x is in the same spot in the cycle of x
as its 2n th digit, and so these digits are equal. Hence the (n + w)th digit of y is equal to its
nth digit. This means that the digits of y repeat periodically with period w from some point
on, as required.
57
Solution. Let g(n) = nf (n) for n ≥ 1 and g(0) = 0. We note that, for k = 1, . . . , n,
jnk n − 1
− =0
k k
Hence
meaning that
d(1) + d(2) + · · · + d(n)
f (n) = .
n
In other words, f (n) is equal to the arithmetic mean of d(1), d(2), . . . , d(n). In order to prove
the claims, it is therefore sufficient to show that d(n + 1) > f (n) and d(n + 1) < f (n) both
hold infinitely often.
We note that d(1) = 1. For n > 1, d(n) ≥ 2 holds, with equality if and only if n is prime.
Since f (6) = 7/3 > 2, it follows that f (n) > 2 holds for all n ≥ 6.
Since there are infinitely many primes, d(n + 1) = 2 holds for infinitely many values of n,
and for each such n ≥ 6 we have d(n + 1) = 2 < f (n). This proves claim (b).
To prove (a), notice that the sequence d(1), d(2), d(3), . . . is unbounded (e. g. d(2k ) = k + 1
for all k). Hence d(n + 1) > max{d(1), d(2), . . . , d(n)} for infinitely many n. For all such n, we
have d(n + 1) > f (n). This completes the solution.
58
N4. Let P be a polynomial of degree n > 1 with integer coefficients and let k be any positive
integer. Consider the polynomial Q(x) = P (P (. . . P (P (x)) . . .)), with k pairs of parentheses.
Prove that Q has no more than n integer fixed points, i.e. integers satisfying the equation
Q(x) = x.
(Romania)
Solution. The claim is obvious if every integer fixed point of Q is a fixed point of P itself.
For the sequel assume that this is not the case. Take any integer x0 such that Q(x0 ) = x0 ,
P (x0 ) 6= x0 and define inductively xi+1 = P (xi ) for i = 0, 1, 2, . . . ; then xk = x0 .
It is evident that
is a divisor of the next one; and since xk − xk+1 = x0 − x1 , all these differences have equal
absolute values. For xm = min(x1 , . . . , xk ) this means that xm−1 − xm = −(xm − xm+1 ). Thus
xm−1 = xm+1 (6= xm ). It follows that consecutive differences in the sequence (2) have opposite
signs. Consequently, x0 , x1 , x2 , . . . is an alternating sequence of two distinct values. In other
words, every integer fixed point of Q is a fixed point of the polynomial P (P (x)). Our task is
to prove that there are at most n such points.
Let a be one of them so that b = P (a) 6= a (we have assumed that such an a exists); then
a = P (b). Take any other integer fixed point α of P (P (x)) and let P (α) = β, so that P (β) = α;
the numbers α and β need not be distinct (α can be a fixed point of P ), but each of α, β is
different from each of a, b. Applying property (1) to the four pairs of integers (α, a), (β, b),
(α, b), (β, a) we get that the numbers α − a and β − b divide each other, and also α − b and
β − a divide each other. Consequently
Solution. The equation has no integer solutions. To show this, we first prove a lemma.
x7 − 1
Lemma. If x is an integer and p is a prime divisor of then either p ≡ 1 (mod 7) or p = 7.
x−1
Proof. Both x7 −1 and xp−1 −1 are divisible by p, by hypothesis and by Fermat’s little theorem,
respectively. Suppose that 7 does not divide p − 1. Then gcd(p−1, 7) = 1, so there exist integers
k and m such that 7k + (p − 1)m = 1. We therefore have
and so
x7 − 1
= 1 + x + · · · + x6 ≡ 7 (mod p).
x−1
It follows that p divides 7, hence p = 7 must hold if p ≡ 1 (mod 7) does not, as stated.
x7 − 1
The lemma shows that each positive divisor d of satisfies either d ≡ 0 (mod 7) or
x−1
d ≡ 1 (mod 7).
Now assume that (x, y) is an integer solution of the original equation. Notice that y − 1 > 0,
x7 − 1 x7 − 1
because > 0 for all x 6= 1. Since y − 1 divides = y 5 − 1, we have y ≡ 1 (mod 7)
x−1 x−1
or y ≡ 2 (mod 7) by the previous paragraph. In the first case, 1 + y + y 2 + y 3 + y 4 ≡ 5 (mod 7),
and in the second 1 + y + y 2 + y 3 + y 4 ≡ 3 (mod 7). Both possibilities contradict the fact that
x7 − 1
the positive divisor 1 + y + y 2 + y 3 + y 4 of is congruent to 0 or 1 modulo 7. So the given
x−1
equation has no integer solutions.
60
N6. Let a > b > 1 be relatively prime positive integers. Define the weight of an integer c,
denoted by w(c), to be the minimal possible value of |x| + |y| taken over all pairs of integers x
and y such that
ax + by = c.
An integer c is called a local champion if w(c) ≥ w(c ± a) and w(c) ≥ w(c ± b).
Find all local champions and determine their number.
(USA)
Solution. Call the pair of integers (x, y) a representation of c if ax + by = c and |x| + |y| has
the smallest possible value, i.e. |x| + |y| = w(c).
We characterise the local champions by the following three observations.
Lemma 1. If (x, y) a representation of a local champion c then xy < 0.
Proof. Suppose indirectly that x ≥ 0 and y ≥ 0 and consider the values w(c) and w(c + a). All
representations of the numbers c and c + a in the form au + bv can be written as
for all k. On the other hand, w(c + a) ≤ w(c), so there exists a k for which
Then
Comparing the first and the third expressions, we find k(a − b) + 1 ≤ 0 implying k < 0.
Comparing the second and fourth expressions, we get |x + 1 − kb| ≤ |x − kb|, therefore kb > x;
this is a contradiction.
If x, y ≤ 0 then we can switch to −c, −x and −y.
From this point, write c = ax − by instead of c = ax + by and consider only those cases
where x and y are nonzero and have the same sign. By Lemma 1, there is no loss of generality
in doing so.
Lemma 2. Let c = ax − by where |x| + |y| is minimal and x, y have
the same sign. The number
c is a local champion if and only if |x| < b and |x| + |y| = a+b
2
.
Proof. Without loss of generality we may assume x, y > 0.
The numbers c − a and c + b can be written as
and trivially w(c − a) ≤ (x − 1) + y < w(c) and w(c + b) ≤ x + (y − 1) < w(c) in all cases.
Now assume that c is a local champion and consider w(c + a). Since w(c + a) ≤ w(c), there
exists an integer k such that
This inequality cannot hold if k ≤ 0, therefore k > 0. We prove that we can choose k = 1.
Consider the function f (t) = |x + 1 − bt| + |y − at| − (x + y). This is a convex function and
we have f (0) = 1 and f (k) ≤ 0. By Jensen’s inequality, f (1) ≤ 1 − k1 f (0) + k1 f (k) < 1. But
f (1) is an integer. Therefore f (1) ≤ 0 and
|x + 1 − b| + |y − a| ≤ x + y.
x + y ≤ |x − b| + |y − a|.
(b − x − 1) + (a − y) ≤ x + y ≤ (b − x) + (a − y),
a+b−1 a+b
≤x+y ≤ .
2 2
Hence x + y = a+b2
.
To prove the opposite direction, assume 0 < x < b and x + y = a+b
2
. Since a > b, we also
have 0 < y < a. Then
w(c + a) ≤ |x + 1 − b| + |y − a| = a + b − 1 − (x + y) ≤ x + y = w(c)
and
w(c − b) ≤ |x − b| + |y + 1 − a| = a + b − 1 − (x + y) ≤ x + y = w(c)
therefore c is a local champion indeed.
Lemma 3. Let c = ax − by and assume that x and y have the same sign, |x| < b, |y| < a and
|x| + |y| = a+b
2
. Then w(c) = x + y.
Proof. By definition w(c) = min{|x − kb| + |y − ka| : k ∈ Z}. If k ≤ 0 then obviously
|x − kb| + |y − ka| ≥ x + y. If k ≥ 1 then
and
2c1 − 2c2 ≡ −2a (mod a + b).
The number a + b is odd and relatively prime to a, therefore the elements of C + and C − belong
to two different residue classes modulo a + b. Hence, the set C is the union of two disjoint
arithmetic progressions and the number of all local champions is 2(b − 1).
So the number of local champions is b − 1 if both a and b are odd and 2(b − 1) otherwise.
Comment. The original question, as stated by the proposer, was:
(a) Show that there exists only finitely many local champions;
(b) Show that there exists at least one local champion.
63
N7. Prove that, for every positive integer n, there exists an integer m such that 2m + m is
divisible by n.
(Estonia)
Solution. We will prove by induction on d that, for every positive integer N, there exist positive
integers b0 , b1 , . . . , bd−1 such that, for each i = 0, 1, 2, . . . , d − 1, we have bi > N and
respectively. The d sequences contain a0 d = a numbers altogether. We shall now prove that no
two of these numbers are congruent modulo a.
Suppose that
2bi + (bi + mk) ≡ 2bj + (bj + nk) (mod a) (3)
for some values of i, j ∈ {0, 1, . . . , d − 1} and m, n ∈ {0, 1, . . . , a0 − 1}. Since d is a divisor of a,
we also have
2bi + (bi + mk) ≡ 2bj + (bj + nk) (mod d).
Because d is a divisor of k and in view of (1), we obtain i ≡ j (mod d). As i, j ∈ {0, 1, . . . , d−1},
this just means that i = j. Substituting this into (3) yields mk ≡ nk (mod a). Therefore
mk 0 ≡ nk 0 (mod a0 ); and since a0 and k 0 are coprime, we get m ≡ n (mod a0 ). Hence also
m = n.
It follows that the a numbers that make up the d sequences (2) satisfy all the requirements;
they are certainly all greater than N because we chose each bi > max(2M , N). So the statement
holds for a, completing the induction.
64
48th International Mathematical Olympiad
Vietnam 2007
Algebra 7
Problem A1 . . . . . . . . . . . . . . . . . . . . . . . . . . . . . . . . . . . . . . 7
Problem A2 . . . . . . . . . . . . . . . . . . . . . . . . . . . . . . . . . . . . . . 10
Problem A3 . . . . . . . . . . . . . . . . . . . . . . . . . . . . . . . . . . . . . . 12
Problem A4 . . . . . . . . . . . . . . . . . . . . . . . . . . . . . . . . . . . . . . 14
Problem A5 . . . . . . . . . . . . . . . . . . . . . . . . . . . . . . . . . . . . . . 16
Problem A6 . . . . . . . . . . . . . . . . . . . . . . . . . . . . . . . . . . . . . . 20
Problem A7 . . . . . . . . . . . . . . . . . . . . . . . . . . . . . . . . . . . . . . 22
Combinatorics 25
Problem C1 . . . . . . . . . . . . . . . . . . . . . . . . . . . . . . . . . . . . . . 25
Problem C2 . . . . . . . . . . . . . . . . . . . . . . . . . . . . . . . . . . . . . . 28
Problem C3 . . . . . . . . . . . . . . . . . . . . . . . . . . . . . . . . . . . . . . 30
Problem C4 . . . . . . . . . . . . . . . . . . . . . . . . . . . . . . . . . . . . . . 31
Problem C5 . . . . . . . . . . . . . . . . . . . . . . . . . . . . . . . . . . . . . . 32
Problem C6 . . . . . . . . . . . . . . . . . . . . . . . . . . . . . . . . . . . . . . 34
Problem C7 . . . . . . . . . . . . . . . . . . . . . . . . . . . . . . . . . . . . . . 36
Problem C8 . . . . . . . . . . . . . . . . . . . . . . . . . . . . . . . . . . . . . . 37
Geometry 39
Problem G1 . . . . . . . . . . . . . . . . . . . . . . . . . . . . . . . . . . . . . . 39
Problem G2 . . . . . . . . . . . . . . . . . . . . . . . . . . . . . . . . . . . . . . 41
Problem G3 . . . . . . . . . . . . . . . . . . . . . . . . . . . . . . . . . . . . . . 42
Problem G4 . . . . . . . . . . . . . . . . . . . . . . . . . . . . . . . . . . . . . . 43
Problem G5 . . . . . . . . . . . . . . . . . . . . . . . . . . . . . . . . . . . . . . 44
Problem G6 . . . . . . . . . . . . . . . . . . . . . . . . . . . . . . . . . . . . . . 46
Problem G7 . . . . . . . . . . . . . . . . . . . . . . . . . . . . . . . . . . . . . . 49
Problem G8 . . . . . . . . . . . . . . . . . . . . . . . . . . . . . . . . . . . . . . 52
Number Theory 55
Problem N1 . . . . . . . . . . . . . . . . . . . . . . . . . . . . . . . . . . . . . . 55
Problem N2 . . . . . . . . . . . . . . . . . . . . . . . . . . . . . . . . . . . . . . 56
Problem N3 . . . . . . . . . . . . . . . . . . . . . . . . . . . . . . . . . . . . . . 57
Problem N4 . . . . . . . . . . . . . . . . . . . . . . . . . . . . . . . . . . . . . . 58
Problem N5 . . . . . . . . . . . . . . . . . . . . . . . . . . . . . . . . . . . . . . 60
Problem N6 . . . . . . . . . . . . . . . . . . . . . . . . . . . . . . . . . . . . . . 62
Problem N7 . . . . . . . . . . . . . . . . . . . . . . . . . . . . . . . . . . . . . . 63
Contributing Countries
Ha Huy Khoai
Ilya Bogdanov
Tran Nam Dung
Le Tuan Hoa
Géza Kós
Algebra
and let
d = max{di : 1 ≤ i ≤ n}.
(a) Prove that for arbitrary real numbers x1 ≤ x2 ≤ . . . ≤ xn ,
d
max |xi − ai | : 1 ≤ i ≤ n ≥ . (1)
2
(b) Show that there exists a sequence x1 ≤ x2 ≤ . . . ≤ xn of real numbers such that we have
equality in (1).
(New Zealand)
Solution 1. (a) Let 1 ≤ p ≤ q ≤ r ≤ n be indices for which
p q r
For arbitrary real numbers x1 ≤ x2 ≤ . . . ≤ xn , consider just the two quantities |xp − ap |
and |xr − ar |. Since
d d
we have either ap − xp ≥ or xr − ar ≥ . Hence,
2 2
d
max{|xi − ai | : 1 ≤ i ≤ n} ≥ max |xp − ap |, |xr − ar | ≥ max{ap − xp , xr − ar } ≥ .
2
8
d
xk = x` = a` − . (3)
2
Since
a` − ak ≤ max{aj : 1 ≤ j ≤ k} − min{aj : k ≤ j ≤ n} = dk ≤ d,
equality (3) implies
d d d
xk − ak = a` − ak − ≤ d− = .
2 2 2
d d
We obtained that − ≤ xk − ak ≤ for all 1 ≤ k ≤ n, so
2 2
d
max |xi − ai | : 1 ≤ i ≤ n ≤ .
2
d
We have equality because |x1 − a1 | = .
2
and
mi = min{ai , ai+1 , . . . , an } ≤ min{ai+1 , . . . , an } = mi+1 .
Therefore sequences (Mi ) and (mi ) are non-decreasing. Moreover, since ai is listed in both
definitions,
mi ≤ ai ≤ Mi .
To achieve equality in (1), set
Mi + mi
xi = .
2
Since sequences (Mi ) and (mi ) are non-decreasing, this sequence is non-decreasing as well.
9
We show that these functions satisfy the condition (1) and clearly fj (2007) = j.
To check the condition (1) for the function fj (j ≤ 2007), note first that fj is nondecreasing
and fj (n) ≤ n, hence fj fj (n) ≤ fj (n) ≤ n for all n ∈ N. Now, if fj (m) = 1, then the
inequality (1) is clear since fj (m + n) ≥ fj (n) ≥ fj fj (n) = fj (m) + fj fj (n) − 1. Otherwise,
fj (m) + fj fj (n) − 1 ≤ (m + j − 2007) + n = (m + n) + j − 2007 = fj (m + n).
In the case j = 2008, clearly n + 1 ≥ f2008 (n) ≥ n for all n ∈ N; moreover, n + 1 ≥
f2008 f2008 (n) as well. Actually, the latter is trivial if f2008 (n) = n; otherwise,
f2008 (n) = n + 1,
which implies 2007
6 n + 1 and hence n + 1 = f2008 (n + 1) = f2008 f2008 (n) .
So, if 2007 m + n, then
f2008 (m + n) = m + n + 1 = (m + 1) + (n + 1) − 1 ≥ f2008 (m) + f2008 f2008 (n) − 1.
Otherwise, 2007 6 m+n, hence 2007 6 m or 2007 6 n. In the former case we have f2008 (m) = m,
while in the latter one f2008 f2008 (n) = f2008 (n) = n, providing
f2008 (m) + f2008 f2008 (n) − 1 ≤ (m + n + 1) − 1 = f2008 (m + n).
11
Comment. The examples above are not unique. The values 1, 2, . . . , 2008 can be realized in several
ways. Here we present other two constructions for j ≤ 2007, without proof:
1, n < 2007,
jn
gj (n) = j, n = 2007, hj (n) = max 1, .
2007
n, n > 2007;
Also the example for j = 2008 can be generalized. In particular, choosing a divisor d > 1 of 2007,
one can set (
n, d 6 n,
f2008,d (n) =
n + 1, d n.
12
A3. Let n be a positive integer, and let x and y be positive real numbers such that xn +y n = 1.
Prove that ! !
n n
X 1 + x2k X 1 + y 2k 1
< .
k=1
1+ x4k k=1
1+ y 4k (1 − x)(1 − y)
(Estonia)
Solution 1. For each real t ∈ (0, 1),
1 + t2 1 (1 − t)(1 − t3 ) 1
4
= − 4
< .
1+t t t(1 + t ) t
Substituting t = xk and t = y k ,
n n n n
X 1 + x2k X 1 1 − xn X 1 + y 2k X 1 1 − yn
0< < = and 0 < < = .
k=1
1 + x4k k=1
xk xn (1 − x) k=1
1 + y 4k k=1
yk y n (1 − y)
Since 1 − y n = xn and 1 − xn = y n ,
1 − xn yn 1 − yn xn
= , =
xn (1 − x) xn (1 − x) y n (1 − y) y n (1 − y)
and therefore
n
! n
!
X 1 + x2k X 1 + y 2k yn xn 1
< · = .
1 + x4k 1 + y 4k n n
x (1 − x) y (1 − y) (1 − x)(1 − y)
k=1 k=1
Solution 2. We prove
! ! √ 2
1+ 2
n
X 1 + x2k
n
X 1 + y 2k 2
ln 2 0.7001
< < . (1)
1+ x4k 1+ y 4k (1 − x)(1 − y) (1 − x)(1 − y)
k=1 k=1
The idea is to estimate each term on the left-hand side with the same constant. To find the
1 + x2k 1+t
upper bound for the expression 4k
, consider the function f (t) = in interval (0, 1).
1+x 1 + t2
Since √ √
0 1 − 2t − t2 ( 2 + 1 + t)( 2 − 1 − t)
f (t) = = ,
(1 + t2 )2 (1 + t2 )2
√ √
the function increases
√ in interval (0, 2−1] and decreases in [ 2−1, 1). Therefore the maximum
is at point t0 = 2 − 1 and
√
1+t 1+ 2
f (t) = ≤ f (t0 ) = = α.
1 + t2 2
Applying this to each term on the left-hand side of (1), we obtain
n
! n !
X 1 + x2k X 1 + y 2k
4k 4k
≤ nα · nα = (nα)2 . (2)
k=1
1 + x k=1
1 + y
is obviously increasing for t ∈ (0, 1), hence for these values of t we have
1
g 0(t) > 0 ⇐⇒ h(t) < h(s) ⇐⇒ t < s = 1 − t ⇐⇒ t < .
2
Then, the maximum of g(t) in (0, 1) is attained at point t1 = 1/2 and therefore
1
g(t) ≤ g = 2 ln(1 − 2−1/n ), t ∈ (0, 1).
2
Substituting t = xn , we have 1 − t = y n , (1 − x)(1 − y) = exp g(t) and hence
(1 − x)(1 − y) = exp g(t) ≤ (1 − 2−1/n )2 . (3)
ln 2
Applying the inequality 1 − exp(−t) < t for t = , we obtain
n
√
ln 2 ln 2 1 + 2
αn(1 − 2−1/n ) = αn 1 − exp − < αn · = α ln 2 = ln 2.
n n 2
Hence,
! ! √ 2
1+ 2
n
X 1 + x2k
n
X 1 + y 2k 2
ln 2
< .
k=1
1 + x4k k=1
1 + y 4k (1 − x)(1 − y)
n
X 1 + x2k
Comment. It is a natural idea to compare the sum Sn (x) = with the integral In (x) =
1 + x4k
Z k=1
n
1 + x2t
4t
dt. Though computing the integral is quite standard, many difficulties arise. First, the
0 1+x
1 + x2k
integrand has an increasing segment and, depending on x, it can have a decreasing segment as
1 + x4k
well. So comparing Sn (x) and In (x) is not completely obvious. We can add a term to fix the estimate,
e.g. Sn ≤ In + (α − 1), but then the final result will be weak for the small values of n. Second, we
have to minimize (1 − x)(1 − y)In (x)In (y) which leads to very unpleasant computations.
However, by computer search we found that the maximum of In (x)In (y) is at x = y = 2−1/n , as
well as the maximum of Sn (x)Sn (y), and the latter is less. Hence, one can conjecture that the exact
constant which can be put into the numerator on the right-hand side of (1) is
2
π 2
Z 1
1 + 4−t 1 1 17
ln 2 · −t
dt = ln + arctan 4 − ≈ 0.6484.
0 1 + 16 4 2 2 4
14
for all x, y ∈ R+ . (Symbol R+ denotes the set of all positive real numbers.)
(Thaliand)
Answer. f (x) = 2x.
+
Solution 1. First we show that f (y) > y for all y ∈ R . Functional equation (1) yields
f x + f (y) > f (x + y) and hence f (y) 6= y immediately. If f (y) < y for some y, then setting
x = y − f (y) we get
f (y) = f y − f (y) + f (y) = f y − f (y) + y + f (y) > f (y),
By the injective property we conclude that t + g(u) + g(v) = t + g(u + v), hence
Since function g(v) is positive, equation (3) also shows that g is an increasing function.
Finally we prove that g(x) = x. Combining (2) and (3), we obtain
g(t) + y = g t + g(y) = g(t) + g g(y)
and hence
g g(y) = y.
Suppose that there exists an x ∈ R+ such that g(x) 6= x. By the monotonicity
of g, if
x > g(x) then g(x) > g g(x) = x. Similarly, if x < g(x) then g(x) < g g(x) = x. Both cases
lead to contradiction, so there exists no such x.
We have proved that g(x) = x and therefore f (x) = g(x) + x = 2x for all x ∈ R+ . This
function indeed satisfies the functional equation (1).
15
Comment. It is well-known that the additive property (3) together with g(x) ≥ 0 (for x > 0) imply
g(x) = cx. So, after proving (3), it is sufficient to test functions f (x) = (c + 1)x.
Solution 2. We prove that f (y) > y and introduce function g(x) = f (x) − x > 0 in the same
way as in Solution 1.
For arbitrary t > y > 0, substitute x = t − y into (1) to obtain
f t + g(y) = f (t) + f (y)
and therefore
f (y) f (z)
≤ .
y z
f (y) f (z)
Exchanging variables y and z, we obtain the reverse inequality. Hence, = for arbi-
y z
f (x)
trary y and z; so function is constant, f (x) = cx.
x
Substituting back into (1), we find that f (x) = cx is a solution if and only if c = 2. So the
only solution for the problem is f (x) = 2x.
16
A5. Let c > 2, and let a(1), a(2), . . . be a sequence of nonnegative real numbers such that
a(m + n) ≤ 2a(m) + 2a(n) for all m, n ≥ 1, (1)
and
1
a(2k ) ≤ for all k ≥ 0. (2)
(k + 1)c
Prove that the sequence a(n) is bounded.
(Croatia)
Solution 1. For convenience, define a(0) = 0; then condition (1) persists for all pairs of
nonnegative indices.
Lemma 1. For arbitrary nonnegative indices n1 , . . . , nk , we have
k
! k
X X
a ni ≤ 2i a(ni ) (3)
i=1 i=1
and !
k
X k
X
a ni ≤ 2k a(ni ). (4)
i=1 i=1
Proof. Inequality (3) is proved by induction on k. The base case k = 1 is trivial, while the
induction step is provided by
k+1
! k+1
! k
! k k+1
X X X X X
i
a ni = a n1 + ni ≤ 2a(n1 )+2a ni+1 ≤ 2a(n1 )+2 2 a(ni+1 ) = 2i a(ni ).
i=1 i=2 i=1 i=1 i=1
can be proved by an obvious induction on d. Then, turning to (4), we find an integer d such
that 2d−1 < k ≤ 2d to obtain
k
! k 2d
! k 2d
! k k
X X X X X X X
d d
a ni = a ni + 0 ≤2 a(ni ) + a(0) = 2 a(ni ) ≤ 2k a(ni ).
i=1 i=1 i=k+1 i=1 i=k+1 i=1 i=1
Fix an increasing unbounded sequence 0 = M0 < M1 < M2 < . . . of real numbers; the exact
values will be defined later. Let n be an arbitrary positive integer and write
d
X
n= ε i · 2i , where εi ∈ {0, 1}.
i=0
Set εi = 0 for i > d, and take some positive integer f such that Mf > d. Applying (3), we get
f
! f
!
X X X X
a(n) = a ε i · 2i ≤ 2k a ε i · 2i .
k=1 Mk−1 ≤i<Mk k=1 Mk−1 ≤i<Mk
17
Note that there are less than Mk − Mk−1 + 1 integers in interval [Mk−1 , Mk ); hence, using (4)
we have
f
X X
a(n) ≤ 2k · 2(Mk − Mk−1 + 1) εi · a(2i )
k=1 Mk−1 ≤i<Mk
f
X
≤ 2k · 2(Mk − Mk−1 + 1)2 max a(2i )
Mk−1 ≤i<Mk
k=1
f f 2
X
k+1 2
X 1 Mk + 1 2k+1
≤ 2 (Mk + 1) · = .
k=1
(Mk−1 + 1)c k=1
Mk−1 + 1 (Mk−1 + 1)
c−2
Proof. Apply an induction on k. The base cases are k = 1 (trivial) and k = 2 (follows from the
condition (1)). Suppose that k > 2. We can assume that s1 ≤ s2 ≤ · · · ≤ sk . Note that
k−1
X
2−si ≤ 1 − 2−sk−1 ,
i=1
since the left-hand side is a fraction with the denominator 2sk−1 , and this fraction is less than 1.
Define s0k−1 = sk−1 − 1 and n0k−1 = nk−1 + nk ; then we have
k−2
X 0
2−si + 2−sk−1 ≤ (1 − 2 · 2−sk−1 ) + 21−sk−1 = 1.
i=1
We show that this sequence satisfies the conditions of the problem. Take two arbitrary indices m
and n. Let
k
X k
X k
X
a(m) = 2si b(ui ), 2−si ≤ 1, 2ui = m;
i=1 i=1 i=1
Xl Xl Xl
a(n) = 2ri b(wi ), 2−ri ≤ 1, 2wi = n.
i=1 i=1 i=1
Then we have
k l k l
X X 1 1 X X
2−1−si + 2−1−ri ≤ + = 1, 2ui + 2wi = m + n,
2 2
i=1 i=1 i=1 i=1
so by (5) we obtain
k
X l
X
1+si
a(n + m) ≤ 2 b(ui ) + 21+ri b(wi ) = 2a(m) + 2a(n).
i=1 i=1
19
Comment 2. The condition c > 2 is sharp; we show that the sequence (5) is not bounded if c ≤ 2.
First, we prove that for an arbitrary n the minimum in (5) is attained with a sequence (ui )
consisting of distinct numbers. To the contrary, assume that uk−1 = uk . Replace uk−1 and uk by
a single number u0k−1 = uk + 1, and sk−1 and sk by s0k−1 = min{sk−1 , sk }. The modified sequences
provide a better bound since
0 0
2sk−1 b(u0k−1 ) = 2sk−1 b(uk + 1) < 2sk−1 b(uk−1 ) + 2sk b(uk )
k
! k k
k
X
i−1
X
si
X 2si
a(2 − 1) = a 2 = 2 b(i − 1) = .
ic
i=1 i=1 i=1
which is unbounded.
For c ≤ 2, it is also possible to show a concrete counterexample. Actually, one can prove that the
sequence !
k k
X
ui
X i
a 2 = (0 ≤ u1 < . . . < uk )
(ui + 1)2
i=1 i=1
A6. Let a1 , a2 , . . . , a100 be nonnegative real numbers such that a21 + a22 + . . . + a2100 = 1. Prove
that
12
a21 a2 + a22 a3 + . . . + a2100 a1 < .
25
(Poland)
100
X
Solution. Let S = a2k ak+1 . (As usual, we consider the indices modulo 100, e.g. we set
k=1
a101 = a1 and a102 = a2 .)
Applying the Cauchy-Schwarz inequality to sequences (ak+1 ) and (a2k + 2ak+1 ak+2 ), and then
the AM-GM inequality to numbers a2k+1 and a2k+2 ,
100
!2 100
! 100 !
X X X
(3S)2 = ak+1 (a2k + 2ak+1 ak+2 ) ≤ a2k+1 (a2k + 2ak+1 ak+2 )2 (1)
k=1 k=1 k=1
100
X 100
X
=1· (a2k + 2ak+1 ak+2 )2 = (a4k + 4a2k ak+1 ak+2 + 4a2k+1 a2k+2 )
k=1 k=1
100
X 100
X
≤ a4k + 2a2k (a2k+1 + a2k+2 ) + 4a2k+1 a2k+2 = a4k + 6a2k a2k+1 + 2a2k a2k+2 .
k=1 k=1
we obtain that
100
!2 50
! 50
! 50 50
!2
X X X X X
(3S)2 ≤ a2k +4 a22i−1 a22j ≤1+ a22i−1 + a22j = 2,
k=1 i=1 j=1 i=1 j=1
hence √
2 12
S≤ ≈ 0.4714 < = 0.48.
3 25
Comment 1. By applying the Lagrange multiplier method, one can see that the maximum is
attained at values of ai satisfying
a2k−1 + 2ak ak+1 = 2λak (2)
for all k = 1, 2, . . . , 100. Though this system of equations seems hard to solve, it can help to find the
estimate above; it may suggest to have a closer look at the expression a2k−1 ak + 2a2k ak+1 .
Moreover, if the numbers a1 , . . . , a100 satisfy (2), we have equality in (1). (See also Comment 3.)
Comment 2. It is natural to ask what is the best constant cn in the inequality
3/2
a21 a2 + a22 a3 + . . . + a2n a1 ≤ cn a21 + a22 + . . . + a2n . (3)
√
For 1 ≤ n ≤ 4 one may prove cn = 1/ n which is achieved when a1 = a2 = . . . = an . However, the
situation changes completely if n ≥ 5. In this case we do not know the exact value of cn . By computer
search it can be found that cn ≈ 0.4514 and it is realized for example if
Comment 3. The solution can be improved in several ways to give somewhat better bounds for cn .
Here we show a variant which proves cn < 0.4589 for n ≥ 5.
The value of cn does not change if negative values are also allowed in (3). So the problem is
equivalent to maximizing
on the unit sphere a21 + a22 + . . . + a2n = 1 in Rn . Since the unit sphere is compact, the function has a
maximum and we can apply the Lagrange multiplier method; for each maximum point there exists a
real number λ such that
Then
n
X n
X
3S = a2k−1 ak + 2a2k ak+1 = 2λa2k = 2λ
k=1 k=1
and therefore
a2k−1 + 2ak ak+1 = 3Sak for all k = 1, 2, . . . , n. (4)
and
n
X n
X n
X n
X
2
3S = 3Sa2k−1 ak = a2k−1 a2k−1 + 2ak ak+1 = a4k +2 a2k ak+1 ak+2 . (6)
k=1 k=1 k=1 k=1
Let p be a positive number. Combining (5) and (6) and applying the AM-GM inequality,
n
X n
X n
X
(9 + 3p)S 2 = (1 + p) a4k + 4 a2k a2k+1 + (4 + 2p) a2k ak+1 ak+2
k=1 k=1 k=1
n n n
X X X (2 + p)2 2 2
≤ (1 + p) a4k + 4 a2k a2k+1 + 2(1 + p)a2k a2k+2 +
ak ak+1
2(1 + p)
k=1 k=1 k=1
n X n
X
4 2 2 2 2 (2 + p)2
= (1 + p) (ak + 2ak ak+1 + 2ak ak+2 ) + 4 + − 2(1 + p) a2k a2k+1
2(1 + p)
k=1 k=1
n
!2 n
X 8 + 4p − 3p2 X 2 2
≤ (1 + p) a2k + ak ak+1
2(1 + p)
k=1 k=1
n
8 + 4p − 3p2 X
= (1 + p) + a2k a2k+1 .
2(1 + p)
k=1
√
2+2 7
Setting p = which is the positive root of 8 + 4p − 3p2 = 0, we obtain
3
r s √
1+p 5+2 7
S≤ = √ ≈ 0.458879.
9 + 3p 33 + 6 7
22
Find the smallest number of planes that jointly contain all (n + 1)3 − 1 points of S but none of
them passes through the origin.
(Netherlands)
Answer. 3n planes.
Solution. It is easy to find 3n such planes. For example, planes x = i, y = i or z = i
(i = 1, 2, . . . , n) cover the set S but none of them contains the origin. Another such collection
consists of all planes x + y + z = k for k = 1, 2, . . . , 3n.
We show that 3n is the smallest possible number.
Lemma 1. Consider a nonzero polynomial P (x1 , . . . , xk ) in k variables. Suppose that P
vanishes at all points (x1 , . . . , xk ) such that x1 , . . . , xk ∈ {0, 1, . . . , n} and x1 + · · · + xk > 0,
while P (0, 0, . . . , 0) 6= 0. Then deg P ≥ kn.
Proof. We use induction on k. The base case k = 0 is clear since P 6= 0. Denote for clarity
y = xk .
Let R(x1 , . . . , xk−1 , y) be the residue of P modulo Q(y) = y(y − 1) . . . (y − n). Polyno-
mial Q(y) vanishes at each y = 0, 1, . . . , n, hence P (x1 , . . . , xk−1 , y) = R(x1 , . . . , xk−1 , y) for
all x1 , . . . , xk−1 , y ∈ {0, 1, . . . , n}. Therefore, R also satisfies the condition of the Lemma;
moreover, degy R ≤ n. Clearly, deg R ≤ deg P , so it suffices to prove that deg R ≥ nk.
Now, expand polynomial R in the powers of y:
R(x1 , . . . , xk−1 , y) = Rn (x1 , . . . , xk−1 )y n + Rn−1 (x1 , . . . , xk−1 )y n−1 + · · · + R0 (x1 , . . . , xk−1 ).
We show that polynomial Rn (x1 , . . . , xk−1 ) satisfies the condition of the induction hypothesis.
Consider the polynomial T (y) = R(0, . . . , 0, y) of degree ≤ n. This polynomial has n roots
y = 1, . . . , n; on the other hand, T (y) 6≡ 0 since T (0) 6= 0. Hence deg T = n, and its leading
coefficient is Rn (0, 0, . . . , 0) 6= 0. In particular, in the case k = 1 we obtain that coefficient Rn
is nonzero.
Similarly, take any numbers a1 , . . . , ak−1 ∈ {0, 1, . . . , n} with a1 +· · ·+ak−1 > 0. Substituting
xi = ai into R(x1 , . . . , xk−1 , y), we get a polynomial in y which vanishes at all points y = 0, . . . , n
and has degree ≤ n. Therefore, this polynomial is null, hence Ri (a1 , . . . , ak−1) = 0 for all
i = 0, 1, . . . , n. In particular, Rn (a1 , . . . , ak−1 ) = 0.
Thus, the polynomial Rn (x1 , . . . , xk−1 ) satisfies the condition of the induction hypothesis.
So, we have deg Rn ≥ (k − 1)n and deg P ≥ deg R ≥ deg Rn + n ≥ kn.
Now we can finish the solution. Suppose that there are N planes covering all the points
of S but not containing the origin. Let their equations be ai x + bi y + ci z + di = 0. Consider
the polynomial
N
Y
P (x, y, z) = (ai x + bi y + ci z + di ).
i=1
It has total degree N. This polynomial has the property that P (x0 , y0, z0 ) = 0 for any
(x0 , y0, z0 ) ∈ S, while P (0, 0, 0) 6= 0. Hence by Lemma 1 we get N = deg P ≥ 3n, as de-
sired.
Comment 1. There are many other collections of 3n planes covering the set S but not covering the
origin.
23
Solution 2. We present a different proof of the main Lemma 1. Here we confine ourselves to
the case k = 3, which is applied in the solution, and denote the variables by x, y and z. (The
same proof works for the general statement as well.)
The following fact is known with various proofs; we provide one possible proof for the
completeness.
Lemma 2. For arbitrary integers 0 ≤ m < n and for an arbitrary polynomial P (x) of degree m,
n
k n
X
(−1) P (k) = 0. (1)
k=0
k
Now return to the proof of Lemma 1. Suppose, to the contrary, that deg P = N < 3n.
Consider the sum n X n X n
i+j+k n n n
X
Σ= (−1) P (i, j, k).
i=0 j=0 k=0
i j k
3
n
The only nonzero term in this sum is P (0, 0, 0) and its coefficient is = 1; therefore
0
Σ = P (0, 0, 0) 6= 0. X
On the other hand, if P (x, y, z) = pα,β,γ xα y β z γ , then
α+β+γ≤N
n X
n X
n X
X n n n
i+j+k
Σ= (−1) pα,β,γ iα j β k γ
i=0 j=0 k=0
i j k α+β+γ≤N
n ! n ! X n !
X X n X n n γ
= pα,β,γ (−1)i iα (−1)j jβ (−1)k k .
α+β+γ≤N i=0
i j=0
j k=0
k
Consider an arbitrary term in this sum. We claim that it is zero. Since N < 3n, one of three
inequalities α < n, β < n or γ < n is valid. For the convenience, suppose that α < n. Applying
n
i n
X
α
Lemma 2 to polynomial x , we get (−1) iα = 0, hence the term is zero as required.
i=0
i
This yields Σ = 0 which is a contradiction. Therefore, deg P ≥ 3n.
24
Comment 2. The proof does not depend on the concrete coefficients in Lemma 2. Instead of this
Lemma, one can simply use the fact that there exist numbers α0 , α1 , . . . , αn (α0 6= 0) such that
n
X
αk km = 0 for every 0 ≤ m < n.
k=0
This is a system of homogeneous linear equations in variables αi . Since the number of equations is
less than the number of variables, the only nontrivial thing is that there exists a solution with α0 6= 0.
It can be shown in various ways.
Combinatorics
C1. Let n > 1 be an integer. Find all sequences a1 , a2 , . . . , an2 +n satisfying the following
conditions:
(a) ai ∈ {0, 1} for all 1 ≤ i ≤ n2 + n;
(b) ai+1 + ai+2 + . . . + ai+n < ai+n+1 + ai+n+2 + . . . + ai+2n for all 0 ≤ i ≤ n2 − n.
(Serbia)
Answer. Such a sequence is unique. It can be defined as follows:
(
0, u + v ≤ n,
au+vn = for all 1 ≤ u ≤ n and 0 ≤ v ≤ n. (1)
1, u + v ≥ n + 1
(0| .{z
. . 0)
} (0| .{z
. . 0} 1) (0| .{z
. . 0} 1 1) . . . (|0 .{z
. . 0} |1 .{z
. . 1)
} . . . (0 1| .{z
. . 1)
} (|1 .{z
. . 1).
}
n n−1 n−2 n−v v n−1 n
Solution 1. Consider a sequence (ai ) satisfying the conditions. For arbitrary integers 0 ≤
k ≤ l ≤ n2 + n denote S(k, l] = ak+1 + · · · + al . (If k = l then S(k, l] = 0.) Then condition (b)
can be rewritten as S(i, i + n] < S(i + n, i + 2n] for all 0 ≤ i ≤ n2 − n. Notice that for
0 ≤ k ≤ l ≤ m ≤ n2 + n we have S(k, m] = S(k, l] + S(l, m].
By condition (b),
a1 = a2 = . . . = an = 0, (3)
an2 +1 = an2 +2 = . . . = an2 +n = 1. (4)
Subdivide sequence (ai ) into n+1 blocks, each consisting of n consecutive terms, and number
them from 0 to n. We show by induction on v that the vth blocks has the form
(0| .{z
. . 0} 1| .{z
. . 1).
}
n−v v
Consider the vth block for v > 0. By (2), it contains some “ones”. Let the first “one” in this
block be at the uth position (that is, au+vn = 1). By the induction hypothesis, the (v − 1)th
and vth blocks of (ai ) have the form
P
=v
(0| . . . {z
0 . . . 0} |1 .{z
. . 1)
} (0| .{z
. . 0} 1 ∗ . . . ∗),
n−v+1 v−1 u−1
where each star can appear to be any binary digit. Observe that u ≤ n − v + 1, since the sum
in this block is v. Then, the fragment of length n bracketed above has exactly (v − 1) + 1 ones,
i. e. S u + (v − 1)n, u + vn = v. Hence,
v = S u + (v − 1)n, u + vn < S u + vn, u + (v + 1)n < · · · < S u + (n − 1)n, u + n2 ≤ n;
we have n − v + 1 distinct integers in the interval [v, n], therefore S(u + (t − 1)n, u + tn] = t for
each t = v, . . . , n.
Thus, the end of sequence (ai ) looks as following:
u zeroes n − u ones
P P P
=v =v+1 ··· =n
z }| { z }| {
(|0 . . . 0 0 .{z
. . 0 1 . . . 1)
} |(0 . . . 0 1
{z ∗ . . . ∗) ( ∗ .
} | P {z . . ∗ ∗ . . . ∗)
} . . . ( 1
| . . . {z1 . . . 1)
1 }
P P P
=v−1 =v =v+1 =n
(each bracketed fragment contains n terms). Computing in two ways the sum of all digits
above, we obtain n − u = v − 1 and u = n − v + 1. Then, the first n − v terms in the vth
block are zeroes, and the next v terms are ones, due to the sum of all terms in this block. The
statement is proved.
We are left to check that the sequence obtained satisfies the condition. Notice that ai ≤ ai+n
for all 1 ≤ i ≤ n2 . Moreover, if 1 ≤ u ≤ n and 0 ≤ v ≤ n − 1, then au+vn < au+vn+n exactly
when u + v = n. In this case we have u + vn = n + v(n − 1).
Consider now an arbitrary index 0 ≤ i ≤ n2 − n. Clearly, there exists an integer v such that
n + v(n − 1) ∈ [i + 1, i + n]. Then, applying the above inequalities we obtain that condition (b)
is valid.
Solution 2. Similarly to Solution 1, we introduce the notation S(k, l] and obtain (2), (3),
and (4) in the same way. The sum of all elements of the sequence can be computed as
They are n distinct integers from the n + 1 possible values 0, 1, 2, . . . , n. Denote by m the
“missing” value which is not listed. We determine m from S(0, n2 + n]. Write this sum as
S(0, n2 +n] = S(0, u]+S(u, u+n]+S(u+n, u+2n]+. . .+S(u+(n−1)n, u+n2 ]+S(u+n2 , n2 +n].
so m = n − u.
Hence, the numbers listed in (5) are 0, 1, . . . , n − u − 1 and n − u + 1, . . . , n, respectively,
therefore
(
v, v ≤ n − u − 1,
S u + vn, u + (v + 1)n = for all 0 ≤ u ≤ n, 0 ≤ v ≤ n − 1. (6)
v + 1, v ≥ n − u
Conditions (6), together with (3), provide a system of linear equations in variables ai . Now
we solve this system and show that the solution is unique and satisfies conditions (a) and (b).
First, observe that any solution of the system (3), (6) satisfies the condition (b). By the con-
struction, equations (6) immediately imply (5). On the other hand, all inequalities mentioned
in condition (b) are included into the chain (5) for some value of u.
Next, note that the system (3), (6) is redundant. The numbers S kn, (k + 1)n , where
1 ≤ k ≤ n − 1,appear twice in (6). For u = 0 and v = k we have v ≤ n − u − 1, and (6) gives
S kn, (k + 1)n = v = k. For u = n and v = k − 1 we have v ≥ n − u and we obtain the same
value, S kn, (k + 1)n = v + 1 = k. Therefore, deleting one equation from each redundant pair,
we can make every sum S(k, k + n] appear exactly once on the left-hand side in (6).
Now, from (3), (6), the sequence (ai ) can be reconstructed inductively by
taking the values of S(k, k + n] from (6). This means first that there exists at most one solution
of our system. Conversely, the constructed sequence obviously satisfies all equations (3), (6)
(the only missing equation is an = 0, which follows from S(0, n] = 0). Hence it satisfies
condition (b), and we are left to check condition (a) only.
For arbitrary integers 1 ≤ u, t ≤ n we get
au+tn − au+(t−1)n = S u + (t − 1)n, u + tn − S (u − 1) + (t − 1)n, (u − 1) + tn
(t − 1) − (t − 1) = 0, t ≤ n − u,
= t − (t − 1) = 1, t = n − u + 1,
t − t = 0, t ≥ n − u + 2.
Since au = 0, we have
v
X
au+vn = au+vn − au = (au+tn − au+(t−1)n )
t=1
for all 1 ≤ u, v ≤ n. If v < n−u + 1 then all terms are 0 on the right-hand side. If v ≥ n−u + 1,
then variable t attains the value n − u + 1 once. Hence,
(
0, u + v ≤ n,
au+vn =
1, u + v ≥ n + 1,
according with (1). Note that the formula is valid for v = 0 as well.
Finally, we presented the direct formula for (ai ), and we have proved that it satisfies condi-
tion (a). So, the solution is complete.
28
C2. A unit square is dissected into n > 1 rectangles such that their sides are parallel to the
sides of the square. Any line, parallel to a side of the square and intersecting its interior, also
intersects the interior of some rectangle. Prove that in this dissection, there exists a rectangle
having no point on the boundary of the square.
(Japan)
Solution 1. Call the directions of the sides of the square horizontal and vertical. A horizontal
or vertical line, which intersects the interior of the square but does not intersect the interior of
any rectangle, will be called a splitting line. A rectangle having no point on the boundary of
the square will be called an interior rectangle.
Suppose, to the contrary, that there exists a dissection of the square into more than one
rectangle, such that no interior rectangle and no splitting line appear. Consider such a dissection
with the least possible number of rectangles. Notice that this number of rectangles is greater
than 2, otherwise their common side provides a splitting line.
If there exist two rectangles having a common side, then we can replace them by their union
(see Figure 1). The number of rectangles was greater than 2, so in a new dissection it is greater
than 1. Clearly, in the new dissection, there is also no splitting line as well as no interior
rectangle. This contradicts the choice of the original dissection.
Denote the initial square by ABCD, with A and B being respectively the lower left and lower
right vertices. Consider those two rectangles a and b containing vertices A and B, respectively.
(Note that a 6= b, otherwise its top side provides a splitting line.) We can assume that the
height of a is not greater than that of b. Then consider the rectangle c neighboring to the lower
right corner of a (it may happen that c = b). By aforementioned, the heights of a and c are
distinct. Then two cases are possible.
D C D C
d d
a b c
c a b
A B A B
c
b
X Y b0
L a0
a
a
X b
p
C3. Find all positive integers n, for which the numbers in the set S = {1, 2, . . . , n} can be
colored red and blue, with the following condition being satisfied: the set S × S × S contains
exactly 2007 ordered triples (x, y, z) such that (i) x, y, z are of the same color and (ii) x + y + z
is divisible by n.
(Netherlands)
Answer. n = 69 and n = 84.
Solution. Suppose that the numbers 1, 2, . . . , n are colored red and blue. Denote by R and B
the sets of red and blue numbers, respectively; let |R| = r and |B| = b = n − r. Call a
triple (x, y, z) ∈ S × S × S monochromatic if x, y, z have the same color, and bichromatic
otherwise. Call a triple (x, y, z) divisible if x + y + z is divisible by n. We claim that there are
exactly r 2 − rb + b2 divisible monochromatic triples.
For any pair (x, y) ∈ S × S there exists a unique zx,y ∈ S such that the triple (x, y, zx,y ) is
divisible; so there are exactly n2 divisible triples. Furthermore, if a divisible triple (x, y, z) is
bichromatic, then among x, y, z there are either one blue and two red numbers, or vice versa.
In both cases, exactly one of the pairs (x, y), (y, z) and (z, x) belongs to the set R × B. Assign
such pair to the triple (x, y, z).
Conversely, consider any pair (x, y) ∈ R × B, and denote z = zx,y . Since x 6= y, the
triples (x, y, z), (y, z, x) and (z, x, y) are distinct, and (x, y) is assigned to each of them. On the
other hand, if (x, y) is assigned to some triple, then this triple is clearly one of those mentioned
above. So each pair in R × B is assigned exactly three times.
Thus, the number of bichromatic divisible triples is three times the number of elements
in R × B, and the number of monochromatic ones is n2 − 3rb = (r + b)2 − 3rb = r 2 − rb + b2 ,
as claimed.
So, to find all values of n for which the desired coloring is possible, we have to find all
n, for which there exists a decomposition n = r + b with r 2 − rb + b2 = 2007. Therefore,
9 r 2 − rb +b2 = (r + b)2 − 3rb. From this it consequently follows that 3 r + b, 3 rb, and
then 3 r, 3 b. Set r = 3s, b = 3c. We can assume that s ≥ c. We have s2 − sc + c2 = 223.
Furthermore,
892 = 4(s2 − sc + c2 ) = (2c − s)2 + 3s2 ≥ 3s2 ≥ 3s2 − 3c(s − c) = 3(s2 − sc + c2 ) = 669,
which is impossible for an integer c. In a similar way, if s = 16 then c(16 − c) = 33, which is
also impossible. Finally, if s = 17 then c(17 − c) = 66, and the solutions are c = 6 and c = 11.
Hence, (r, b) = (51, 18) or (r, b) = (51, 33), and the possible values of n are n = 51 + 18 = 69
and n = 51 + 33 = 84.
Comment. After the formula for the number of monochromatic divisible triples is found, the solution
can be finished in various ways. The one presented is aimed to decrease the number of considered
cases.
31
C4. Let A0 = (a1 , . . . , an) be a finite sequence of real numbers. For each k ≥ 0, from the
sequence Ak = (x1 , . . . , xn ) we construct a new sequence Ak+1 in the following way.
1. We choose a partition {1, . . . , n} = I ∪ J, where I and J are two disjoint sets, such that
the expression
X X
xi − xj
i∈I j∈J
attains the smallest possible value. (We allow the sets I or J to be empty; in this case the
corresponding sum is 0.) If there are several such partitions, one is chosen arbitrarily.
2. We set Ak+1 = (y1 , . . . , yn ), where yi = xi + 1 if i ∈ I, and yi = xi − 1 if i ∈ J.
Prove that for some k, the sequence Ak contains an element x such that |x| ≥ n/2.
(Iran)
Solution.
Lemma. Suppose that all terms of the sequence (x1 , . . . , xn ) satisfy the inequality |xi | < a.
Then there exists a partition {1, 2, . . . , n} = I ∪ J into two disjoint sets such that
X X
xi − xj < a. (1)
i∈I j∈J
Proof. Apply an induction on n. The base case n = 1 is trivial. For the induction step,
consider a sequence (x1 , . . . , xn ) (n > 1). By the induction hypothesis there exists a splitting
{1, . . . , n − 1} = I 0 ∪ J 0 such that
X X
xi − xj < a.
i∈I 0 j∈J 0
P P
For convenience, suppose that xj . If xn ≥ 0 then choose I = I 0 , J = J ∪ {n}; other-
xi ≥
i∈I 0 j∈J 0 P P
wise choose I = I 0 ∪ {n}, J = J . In both cases, we have
0
xi − xj ∈ [0, a) and |xn | ∈ [0, a);
i∈I 0 j∈J 0
hence X X X X
xi − xj = xi − xj − |xn | ∈ (−a, a),
i∈I j∈J i∈I 0 j∈J 0
as desired.
Let us turn now to the problem. To the contrary, assume that for all k, all the numbers
in Ak lie in interval (−n/2, n/2). Consider an arbitrary sequence Ak = (b1 , . . . , bn ). To obtain
the term bi , we increased and decreased number ai by one several times. Therefore bi − ai is
always an integer, and there are not more than n possible values for bi . So, there are not more
than nn distinct possible sequences Ak , and hence two of the sequences A1 , A2 , . . . , Ann +1
should be identical, say Ap = Aq for some p < q.
For any positive integer k, let Sk be the sum of squares of elements in Ak . Consider two
consecutive sequences Ak = (x1 , . . . , xn ) and Ak+1 = (y1 , . . . , yn ). Let {1, 2, . . . , n} = I ∪ J be
the partition used inPthis step — that is, yi = xi + 1 for all i ∈ I and yj = xj − 1 for all j ∈ J.
P
Since the value of xi − xj is the smallest possible, the Lemma implies that it is less
i∈I j∈J
than n/2. Then we have
X X
X
2 2
X 2 2
n
Sk+1 − Sk = (xi + 1) − xi + (xj − 1) − xj = n + 2 xi − xj > n − 2 · = 0.
i∈I j∈J i∈I j∈J
2
Thus we obtain Sq > Sq−1 > · · · > Sp . This is impossible since Ap = Aq and hence Sp = Sq .
32
C5. In the Cartesian coordinate plane define the strip Sn = {(x, y) | n ≤ x < n + 1} for
every integer n. Assume that each strip Sn is colored either red or blue, and let a and b be two
distinct positive integers. Prove that there exists a rectangle with side lengths a and b such
that its vertices have the same color.
(Romania)
Solution. If Sn and Sn+a have the same color for some integer n, then we can choose the
rectangle with vertices (n, 0) ∈ Sn , (n, b) ∈ Sn , (n + a, 0) ∈ Sn+a , and (n + a, b) ∈ Sn+a , and we
are done. So it can be assumed that Sn and Sn+a have opposite colors for each n.
Similarly, it also can be assumed that Sn and Sn+b have opposite colors. Then, by induction
on |p| + |q|, we obtain that for arbitrary integers p and q, strips Sn and Sn+pa+qb have the same
color if p + q is even, and these two strips have opposite colors if p + q is odd.
Let d = gcd(a, b), a1 = a/d and b1 = b/d. Apply the result above for p = b1 and q = −a1 .
The strips S0 and S0+b1 a−a1 b are identical and therefore they have the same color. Hence, a1 +b1
is even. By the construction, a1 and b1 are coprime, so this is possible only if both are odd.
Without loss of generality, we can assume a > b. Then a1 > b1 ≥ 1, so a1 ≥ 3.
Choose integers k and ` such that ka1 − `b1 = 1 and therefore ka − `b = d. Since a1 and b1
are odd, k + ` is odd as well. Hence, for every integer n, strips Sn and Sn+ka−`b = Sn+d have
opposite colors. This also implies that the coloring is periodic with period 2d, i.e. strips Sn
and Sn+2d have the same color for every n.
b
C
a
a
A B0 D0
t t + 2d u u + 2d x
b
D
Figure 1
So, by the similar triangles ADD0 and BAB0 , we have the constraint
AD bd
u − t = AD0 = · BB0 = ϕ (1)
AB a
33
for numbers t and u. Computing the numbers y2 and y3 is not required since they have no
effect to the colors.
2
p Observe that the number ϕ is irrational, because ϕ is an integer, but ϕ is not: a1 > ϕ ≥
a21 − 2a1 + 2 > a1 − 1.
By the periodicity, points A and B have the same color; similarly, points C and D have the
same color. Furthermore, these colors depend only on the values of t and u. So it is sufficient
to choose numbers t and u such that vertices A and D have the same color.
Let w be the largest positive integer such that there exist w consecutive strips Sn0 , Sn0 +1 , . . . ,
Sn0 +w−1 with the same color, say red. (Since Sn0 +d must be blue, we have w ≤ d.) We will
choose t from the interval (n0 , n0 + w).
A B0 D0
( ) ( ) x
n0 t n0 + w t + 2d u
I
Figure 2
bd bd
Consider the interval I = n0 + ϕ, n0 + ϕ + w on the x-axis (see Figure 2). Its length
a a
is w, and the end-points are irrational. Therefore, this interval intersects w + 1 consecutive
strips. Since at most w consecutive strips may have the same color, interval I must contain both
bd
red and blue points. Choose u ∈ I such that the line x = u is red and set t = u− ϕ, according
a
to the constraint (1). Then t ∈ (n0 , n0 + w) and A = (t, 0) is red as well as D = (u, y3).
Hence, variables u and t can be set such that they provide a rectangle with four red vertices.
Comment. The statement is false for squares, i.e. in the case a = b. If strips S2ka , S2ka+1 , . . .,
S(2k+1)a−1 are red, and strips S(2k+1)a , S(2k+1)a+1 , . . ., S(2k+2)a−1 are blue for every integer k, then
each square of size a × a has at least one red and at least one blue vertex as well.
34
C6. In a mathematical competition some competitors are friends; friendship is always mutual.
Call a group of competitors a clique if each two of them are friends. The number of members
in a clique is called its size.
It is known that the largest size of cliques is even. Prove that the competitors can be
arranged in two rooms such that the largest size of cliques in one room is the same as the
largest size of cliques in the other room.
(Russia)
Solution. We present an algorithm to arrange the competitors. Let the two rooms be Room A
and Room B. We start with an initial arrangement, and then we modify it several times by
sending one person to the other room. At any state of the algorithm, A and B denote the sets
of the competitors in the rooms, and c(A) and c(B) denote the largest sizes of cliques in the
rooms, respectively.
Step 1. Let M be one of the cliques of largest size, |M| = 2m. Send all members of M to
Room A and all other competitors to Room B.
Since M is a clique of the largest size, we have c(A) = |M| ≥ c(B).
Step 2. While c(A) > c(B), send one person from Room A to Room B.
Room A Room B
A∩M B∩M
Note that c(A) > c(B) implies that Room A is not empty.
In each step, c(A) decreases by one and c(B) increases by at most one. So at the end we
have c(A) ≤ c(B) ≤ c(A) + 1.
We also have c(A) = |A| ≥ m at the end. Otherwise we would have at least m + 1 members
of M in Room B and at most m − 1 in Room A, implying c(B) − c(A) ≥ (m + 1) − (m − 1) = 2.
Step 3. Let k = c(A). If c(B) = k then STOP.
If we reached c(A) = c(B) = k then we have found the desired arrangement.
In all other cases we have c(B) = k + 1.
From the estimate above we also know that k = |A| = |A ∩ M| ≥ m and |B ∩ M| ≤ m.
Step 4. If there exists a competitor x ∈ B ∩ M and a clique C ⊂ B such that |C| = k + 1
and x ∈/ C, then move x to Room A and STOP.
Room A Room B
A∩M B∩M
x C
A∩M C B∩M
therefore
|Q| ≤ |A ∩ M| = k.
Finally, after Step 5 we have c(A) = c(B) = k.
Comment. Obviously, the statement is false without the assumption that the largest clique size is
even.
36
√
3− 5
C7. Let α < be a positive real number. Prove that there exist positive integers n
n
2
and p > α · 2 for which one can select 2p pairwise distinct subsets S1 , . . . , Sp , T1 , . . . , Tp of
the set {1, 2, . . . , n} such that Si ∩ Tj 6= ∅ for all 1 ≤ i, j ≤ p.
(Austria)
Solution. Let k and m be positive integers (to be determined later) and set n = km. De-
compose the set {1, 2, . . . , n} into k disjoint subsets, each of size m; denote these subsets
by A1 , . . . , Ak . Define the following families of sets:
S = S ⊂ {1, 2, . . . , n} : ∀i S ∩ Ai 6= ∅ ,
T1 = T ⊂ {1, 2, . . . , n} : ∃i Ai ⊂ T , T = T1 \ S.
For each set T ∈ T ⊂ T1 , there exists an index 1 ≤ i ≤ k such that Ai ⊂ T . Then for all S ∈ S,
S ∩ T ⊃ S ∩ Ai 6= ∅. Hence, each S ∈ S and each T ∈ T have at least one common element.
Below we showthat the numbers m and k can be chosen such that |S|, |T | > α · 2n . Then,
choosing p = min |S|, |T | , one can select the desired 2p sets S1 , . . . , Sp and T1 , . . . , Tp from
families S and T , respectively. Since families S and T are disjoint, sets Si and Tj will be
pairwise distinct.
To count the sets S ∈ S, observe that each Ai has 2m −1 nonempty subsets so we have 2m −1
choices for S ∩ Ai . These intersections uniquely determine set S, so
|S| = (2m − 1)k . (1)
Similarly, if a set H ⊂ {1, 2, . . . , n} does not contain a certain set Ai then we have 2m − 1
choices for H ∩ Ai : all subsets of Ai , except Ai itself. Therefore, the complement of T1 con-
tains (2m − 1)k sets and
|T1 | = 2km − (2m − 1)k . (2)
Next consider the family S \ T1 . If a set S intersects all Ai but does not contain any of them,
then there exists 2m − 2 possible values for each S ∩ Ai : all subsets of Ai except ∅ and Ai .
Therefore the number of such sets S is (2m − 2)k , so
|S \ T1 | = (2m − 2)k . (3)
From (1), (2), and (3) we obtain
|T | = |T1 | − |S ∩ T1 | = |T1 | − |S| − |S \ T1 | = 2km − 2(2m − 1)k + (2m − 2)k .
√
3− 5
Let δ = and k = k(m) = 2m log 1δ . Then
2
k
|S| 1 k
lim = lim 1 − m = exp − lim m = δ
m→∞ 2km m→∞ 2 m→∞ 2
and similarly
k k
|T | 1 2
lim = 1 − 2 lim 1 − m + lim 1 − m = 1 − 2δ + δ 2 = δ.
m→∞ 2km m→∞ 2 m→∞ 2
|S| |T |
Hence, if m is sufficiently large then and are greater than α (since α < δ). So
2mk 2mk
|S|, |T | > α · 2mk = α · 2n .
√
3− 5
Comment. It can be proved that the constant is sharp. Actually, if S1 , . . . , Sp , T1 , . . . , Tp
2 √
3− 5 n
are distinct subsets of {1, 2, . . . , n} such that each Si intersects each Tj , then p < ·2 .
2
37
C8. Given a convex n-gon P in the plane. For every three vertices of P , consider the triangle
determined by them. Call such a triangle good if all its sides are of unit length.
Prove that there are not more than 23 n good triangles.
(Ukraine)
Solution. Consider all good triangles containing a certain vertex A. The other two vertices
of any such triangle lie on the circle ωA with unit radius and center A. Since P is convex, all
these vertices lie on an arc of angle less than 180◦ . Let LA RA be the shortest such arc, oriented
clockwise (see Figure 1). Each of segments ALA and ARA belongs to a unique good triangle.
We say that the good triangle with side ALA is assigned counterclockwise to A, and the second
one, with side ARA , is assigned clockwise to A. In those cases when there is a single good
triangle containing vertex A, this triangle is assigned to A twice.
There are at most two assignments to each vertex of the polygon. (Vertices which do not
belong to any good triangle have no assignment.) So the number of assignments is at most 2n.
Consider an arbitrary good triangle ABC, with vertices arranged clockwise. We prove
that ABC is assigned to its vertices at least three times. Then, denoting the number of good
triangles by t, we obtain that the number K of all assignments is at most 2n, while it is not
less than 3t. Then 3t ≤ K ≤ 2n, as required.
Actually, we prove that triangle ABC is assigned either counterclockwise to C or clockwise
to B. Then, by the cyclic symmetry of the vertices, we obtain that triangle ABC is assigned
either counterclockwise to A or clockwise to C, and either counterclockwise to B or clockwise
to A, providing the claim.
LA
A0
ωA ωC ωB
RA
B C
A A
ωA
LA
ωA
C0 A B0
(0) (0)
RA Y =RB X=LC
Figure 1 Figure 2
Assume, to the contrary, that LC 6= A and RB 6= A. Denote by A0 , B 0 , C 0 the intersection
points of circles ωA , ωB and ωC , distinct from A, B, C (see Figure 2). Let CLC L0C be the good
triangle containing CLC . Observe that the angle of arc LC A is less than 120◦ . Then one of the
points LC and L0C belongs to arc B 0 A of ωC ; let this point be X. In the case when LC = B 0
and L0C = A, choose X = B 0 .
0
Analogously, considering the good triangle BRB RB which contains BRB as an edge, we see
0 0
that one of the points RB and RB lies on arc AC of ωB . Denote this point by Y , Y 6= A.
Then angles XAY , Y AB, BAC and CAX (oriented clockwise) are not greater than 180◦ .
Hence, point A lies in quadrilateral XY BC (either in its interior or on segment XY ). This is
impossible, since all these five points are vertices of P .
Hence, each good triangle has at least three assignments, and the statement is proved.
Comment 1. Considering a diameter AB of the polygon, one can prove that every good triangle
containing either A or B has at least four assignments. This observation leads to t ≤ 32 (n − 1) .
38
Comment 2. The result t ≤ 23 (n − 1) is sharp. To
C1 Cn
construct a polygon with n = 3k + 1 vertices and t = 2k tri-
D1
angles, take a rhombus AB1 C1 D1 with unit side length and Bn
∠B1 = 60◦ . Then rotate it around A by small angles ob-
taining rhombi AB2 C2 D2 , . . . , ABk Ck Dk (see Figure 3). The
polygon AB1 . . . Bk C1 . . . Ck D1 . . . Dk has 3k + 1 vertices and Dn
B1
contains 2k good triangles.
The construction for n = 3k and n = 3k − 1 can be A
obtained by deleting vertices Dn and Dn−1 . Figure 3
Geometry
G1. In triangle ABC, the angle bisector at vertex C intersects the circumcircle and the per-
pendicular bisectors of sides BC and CA at points R, P , and Q, respectively. The midpoints of
BC and CA are S and T , respectively. Prove that triangles RQT and RP S have the same area.
(Czech Republic)
Solution 1. If AC = BC then triangle ABC is isosceles, triangles RQT and RP S are
symmetric about the bisector CR and the statement is trivial. If AC 6= BC then it can be
assumed without loss of generality that AC < BC.
T Q S
O
`
P
A B
Denote the circumcenter by O. The right triangles CT Q and CSP have equal angles at
vertex C, so they are similar, ∠CP S = ∠CQT = ∠OQP and
QT CQ
= . (1)
PS CP
Let ` be the perpendicular bisector of chord CR; of course, ` passes through the circum-
center O. Due to the equal angles at P and Q, triangle OP Q is isosceles with OP = OQ.
Then line ` is the axis of symmetry in this triangle as well. Therefore, points P and Q lie
symmetrically on line segment CR,
RP = CQ and RQ = CP. (2)
Triangles RQT and RP S have equal angles at vertices Q and P , respectively. Then
1
area(RQT ) 2
· RQ · QT · sin ∠RQT RQ QT
= 1 = · .
area(RP S) 2
· RP · P S · sin ∠RP S RP P S
Substituting (1) and (2),
area(RQT ) RQ QT CP CQ
= · = · = 1.
area(RP S) RP P S CQ CP
Hence, area(RQT ) = area(RSP ).
40
Solution 2. Assume again AC < BC. Denote the circumcenter by O, and let γ be the
angle at C. Similarly to the first solution, from right triangles CT Q and CSP we obtain
that ∠OP Q = ∠OQP = 90◦ − γ2 . Then triangle OP Q is isosceles, OP = OQ and moreover
∠P OQ = γ.
As is well-known, point R is the midpoint of arc AB and ∠ROA = ∠BOR = γ.
C
γ
T Q S
O
γ
γ
P
A B
area(RP S) CS 1
= = .
area(BP R) CB 2
G2. Given an isosceles triangle ABC with AB = AC. The midpoint of side BC is denoted
by M. Let X be a variable point on the shorter arc MA of the circumcircle of triangle ABM.
Let T be the point in the angle domain BMA, for which ∠T MX = 90◦ and T X = BX. Prove
that ∠MT B − ∠CT M does not depend on X.
(Canada)
Solution 1. Let N be the midpoint of segment BT (see Figure 1). Line XN is the axis of
symmetry in the isosceles triangle BXT , thus ∠T NX = 90◦ and ∠BXN = ∠NXT . Moreover,
in triangle BCT , line MN is the midline parallel to CT ; hence ∠CT M = ∠NMT .
Due to the right angles at points M and N, these points lie on the circle with diameter XT .
Therefore,
Hence
∠MT B − ∠CT M = ∠MXN − ∠BXN = ∠MXB = ∠MAB
which does not depend on X.
A
T
X
B M C
N
B M C S
Figure 1 Figure 2
Solution 2. Let S be the reflection of point T over M (see Figure 2). Then XM is the per-
pendicular bisector of T S, hence XB = XT = XS, and X is the circumcenter of triangle BST .
Moreover, ∠BSM = ∠CT M since they are symmetrical about M. Then
∠SXB − ∠BXT
∠MT B − ∠CT M = ∠ST B − ∠BST = .
2
Observe that ∠SXB = ∠SXT − ∠BXT = 2∠MXT − ∠BXT , so
2∠MXT − 2∠BXT
∠MT B − ∠CT M = = ∠MXB = ∠MAB,
2
which is constant.
42
G3. The diagonals of a trapezoid ABCD intersect at point P . Point Q lies between the
parallel lines BC and AD such that ∠AQD = ∠CQB, and line CD separates points P and Q.
Prove that ∠BQP = ∠DAQ.
(Ukraine)
AD
Solution. Let t = . Consider the homothety h with center P and scale −t. Triangles P DA
BC
and P BC are similar with ratio t, hence h(B) = D and h(C) = A.
B C
0
Q
P
Q
A D
Let Q0 = h(Q) (see Figure 1). Then points Q, P and Q0 are obviously collinear. Points Q
and P lie on the same side of AD, as well as on the same side of BC; hence Q0 and P are
also on the same side of h(BC) = AD, and therefore Q and Q0 are on the same side of AD.
Moreover, points Q and C are on the same side of BD, while Q0 and A are on the opposite
side (see Figure above).
By the homothety, ∠AQ0 D = ∠CQB = ∠AQD, hence quadrilateral AQ0 QD is cyclic. Then
∠DAQ = ∠DQ0 Q = ∠DQ0 P = ∠BQP
(the latter equality is valid by the homothety again).
Comment. The statement of the problem is a limit case of the following result.
In an arbitrary quadrilateral ABCD, let P = AC ∩ BD, I = AD ∩ BC, and let Q be an arbitrary
point which is not collinear with any two of points A, B, C, D. Then ∠AQD = ∠CQB if and only if
∠BQP = ∠IQA (angles are oriented; see Figure below to the left).
In the special case of the trapezoid, I is an ideal point and ∠DAQ = ∠IQA = ∠BQP .
C
b B C
B
c I
p
I P P
i Q
Q I
U a A V
d
D I A D
Let a = QA, b = QB, c = QC, d = QD, i = QI and p = QP . Let line QA intersect lines BC
and BD at points U and V , respectively. On lines BC and BD we have
(abci) = (U BCI) and (badp) = (abpd) = (V BP D).
Projecting from A, we get
(abci) = (U BCI) = (V BP D) = (badp).
Suppose that ∠AQD = ∠CQB. Let line p0 be the reflection of line i about the bisector of
angle AQB. Then by symmetry we have (badp0 ) = (abci) = (badp). Hence p = p0 , as desired.
The converse statement can be proved analogously.
43
G4. Consider five points A, B, C, D, E such that ABCD is a parallelogram and BCED is
a cyclic quadrilateral. Let ` be a line passing through A, and let ` intersect segment DC and
line BC at points F and G, respectively. Suppose that EF = EG = EC. Prove that ` is the
bisector of angle DAB.
(Luxembourg)
Solution. If CF = CG, then ∠F GC = ∠GF C, hence ∠GAB = ∠GF C = ∠F GC = ∠F AD,
and ` is a bisector.
Assume that CF < GC. Let EK and EL be the altitudes in the isosceles triangles ECF
and EGC, respectively. Then in the right triangles EKF and ELC we have EF = EC and
CF GC
KF = < = LC,
2 2
so √ √
KE = EF 2 − KF 2 > EC 2 − LC 2 = LE.
Since quadrilateral BCED is cyclic, we have ∠EDC = ∠EBC, so the right triangles BEL
and DEK are similar. Then KE > LE implies DK > BL, and hence
DF = DK − KF > BL − LC = BC = AD.
AD GC
But triangles ADF and GCF are similar, so we have 1 > = ; this contradicts our
DF CF
assumption.
The case CF > GC is completely similar. We consequently obtain the converse inequalities
AD GC
KF > LC, KE < LE, DK < BL, DF < AD, hence 1 < = ; a contradiction.
DF CF
`
B C L G
K E
A D
44
G5. Let ABC be a fixed triangle, and let A1 , B1 , C1 be the midpoints of sides BC, CA, AB,
respectively. Let P be a variable point on the circumcircle. Let lines P A1 , P B1 , P C1 meet the
circumcircle again at A0 , B 0 , C 0 respectively. Assume that the points A, B, C, A0 , B 0 , C 0 are
distinct, and lines AA0 , BB 0 , CC 0 form a triangle. Prove that the area of this triangle does not
depend on P .
(United Kingdom)
Solution 1. Let A0 , B0 , C0 be the points of intersection of the lines AA0 , BB 0 and CC 0 (see
Figure). We claim that area(A0 B0 C0 ) = 21 area(ABC), hence it is constant.
Consider the inscribed hexagon ABCC 0 P A0 . By Pascal’s theorem, the points of intersection
of its opposite sides (or of their extensions) are collinear. These points are AB ∩ C 0 P = C1 ,
BC ∩ P A0 = A1 , CC 0 ∩ A0 A = B0 . So point B0 lies on the midline A1 C1 of triangle ABC.
Analogously, points A0 and C0 lie on lines B1 C1 and A1 B1 , respectively.
Lines AC and A1 C1 are parallel, so triangles B0 C0 A1 and AC0 B1 are similar; hence we have
B0 C0 A1 C0 C
= .
AC0 B1 C0 P
Analogously, from BC k B1 C1 we obtain
B1 C0
A1 C0 BC0 A1
= .
B1 C0 A0 C0 A0
B0
Combining these equalities, we get
B0 C0 BC0 A B
= , C1
AC0 A0 C0 B0
or
A0 C0 · B0 C0 = AC0 · BC0 . C0
Hence we have A0
1 1
area(A0 B0 C0 ) = A0 C0 · B0 C0 sin ∠A0 C0 B0 = AC0 · BC0 sin ∠AC0 B = area(ABC0 ).
2 2
Since C0 lies on the midline, we have d(C0 , AB) = 12 d(C, AB) (we denote by d(X, Y Z) the
distance between point X and line Y Z). Then we obtain
1 1 1
area(A0 B0 C0 ) = area(ABC0 ) = AB · d(C0 , AB) = AB · d(C, AB) = area(ABC).
2 4 2
P C
L A1 C0
0
A
X=B0
A
C1 B M
A0
B0 C0 K
Let line CC intersect BL at point X. Note that ∠LCX = ∠ACC 0 = ∠AP C 0 = ∠AP C1 ,
0
Finally,
1 1 1
area(A0 B0 C0 ) = area(ACB0 ) = B0 L · AC sin ALB0 = BL · AC sin ALB = area(ABC).
2 4 2
Comment 1. The equality area(A0 B0 C0 ) = area(ACB0 ) in Solution 2 does not need to be proved
since the following fact is frequently known.
Suppose that the lines KL and M N are parallel, while the lines KM and LN intersect in a point E.
Then area(KEN ) = area(M EL).
Comment 2. It follows immediately from both solutions that AA0 k BB0 k CC0 . These lines pass
through an ideal point which is isogonally conjugate to P . It is known that they are parallel to the
Simson line of point Q which is opposite to P on the circumcircle.
Comment 3. If A = A0 , then one can define the line AA0 to be the tangent to the circumcircle at
point A. Then the statement of the problem is also valid in this case.
46
G6. Determine the smallest positive real number k with the following property.
Let ABCD be a convex quadrilateral, and let points A1 , B1 , C1 and D1 lie on sides AB, BC,
CD and DA, respectively. Consider the areas of triangles AA1 D1 , BB1 A1 , CC1 B1 , and DD1 C1 ;
let S be the sum of the two smallest ones, and let S1 be the area of quadrilateral A1 B1 C1 D1 .
Then we always have kS1 ≥ S.
(U.S.A.)
Answer. k = 1.
Solution. Throughout the solution, triangles AA1 D1 , BB1 A1 , CC1 B1 , and DD1 C1 will be
referred to as border triangles. We will denote by [R] the area of a region R.
First, we show that k ≥ 1. Consider a triangle ABC with unit area; let A1 , B1 , K be
the midpoints of its sides AB, BC, AC, respectively. Choose a point D on the extension
of BK, close to K. Take points C1 and D1 on sides CD and DA close to D (see Figure 1).
We have [BB1 A1 ] = 14 . Moreover, as C1 , D1 , D → K, we get [A1 B1 C1 D1 ] → [A1 B1 K] = 14 ,
[AA1 D1 ] → [AA1 K] = 14 , [CC1 B1 ] → [CKB1 ] = 41 and [DD1 C1 ] → 0. Hence, the sum of the
two smallest areas of border triangles tends to 41 , as well as [A1 B1 C1 D1 ]; therefore, their ratio
tends to 1, and k ≥ 1.
We are left to prove that k = 1 satisfies the desired property.
B Y
B
B
A1
A1
A1 B1
C0 A0 C0 A0
C1
C1 X
K
A D1 C1 C
D A B1 B0 C Z A B 0 B1 C
Figure 1 Figure 2 Figure 3
Lemma. Let points A1 , B1 , C1 lie respectively on sides BC, CA, AB of a triangle ABC. Then
[A1 B1 C1 ] ≥ min [AC1 B1 ], [BA1 C1 ], [CB1 A1 ] .
Proof. Let A0 , B 0 , C 0 be the midpoints of sides BC, CA and AB, respectively.
Suppose that two of points A1 , B1 , C1 lie in one of triangles AC 0 B 0 , BA0 C 0 and CB 0 A0
(for convenience, let points B1 and C1 lie in triangle AC 0 B 0 ; see Figure 2). Let segments B1 C1
and AA1 intersect at point X. Then X also lies in triangle AC 0 B 0 . Hence A1 X ≥ AX, and we
have
1
[A1 B1 C1 ] A1 X · B1 C1 · sin ∠A1 XC1 A1 X
= 21 = ≥ 1,
[AC1 B1 ] 2
AX · B1 C 1 · sin ∠AXB1 AX
as required.
Otherwise, each one of triangles AC 0 B 0 , BA0 C 0 , CB 0 A0 contains exactly one of points A1 ,
B1 , C1 , and we can assume that BA1 < BA0 , CB1 < CB 0 , AC1 < AC 0 (see Figure 3). Then
lines B1 A1 and AB intersect at a point Y on the extension of AB beyond point B, hence
[A1 B1 C1 ] C1 Y
0
= 0 > 1; also, lines A1 C 0 and CA intersect at a point Z on the extension
[A1 B1 C ] CY
[A1 B1 C 0 ] B1 Z
of CA beyond point A, hence = > 1. Finally, since A1 A0 k B 0 C 0 , we have
[A1 B 0 C 0 ] B0Z
[A1 B1 C1 ] > [A1 B1 C 0 ] > [A1 B 0 C 0 ] = [A0 B 0 C 0 ] = 14 [ABC].
47
Now, from [A1 B1 C1 ] + [AC1 B1 ] + [BA1 C1 ] + [CB1 A1 ] = [ABC] we obtain that one of
the remaining triangles AC1 B1 , BA1 C1 , CB1 A1 has an area less than 14 [ABC], so it is less
than [A1 B1 C1 ].
Now we return to the problem. We say that triangle A1 B1 C1 is small if [A1 B1 C1 ] is less than
each of [BB1 A1 ] and [CC1 B1 ]; otherwise this triangle is big (the similar notion is introduced
for triangles B1 C1 D1 , C1 D1 A1 , D1 A1 B1 ). If both triangles A1 B1 C1 and C1 D1 A1 are big,
then [A1 B1 C1 ] is not less than the area of some border triangle, and [C1 D1 A1 ] is not less than
the area of another one; hence, S1 = [A1 B1 C1 ] + [C1 D1 A1 ] ≥ S. The same is valid for the pair
of B1 C1 D1 and D1 A1 B1 . So it is sufficient to prove that in one of these pairs both triangles
are big.
Suppose the contrary. Then there is a small triangle in each pair. Without loss of generality,
assume that triangles A1 B1 C1 and D1 A1 B1 are small. We can assume also that [A1 B1 C1 ] ≤
[D1 A1 B1 ]. Note that in this case ray D1 C1 intersects line BC.
Consider two cases.
B
K A A1
B
A1 B1
B1 C
A
C D1 C1
D
K L
D1 C1
D L
Figure 4 Figure 5
Case 1. Ray C1 D1 intersects line AB at some point K. Let ray D1 C1 intersect line BC at
point L (see Figure 4). Then we have [A1 B1 C1 ] < [CC1 B1 ] < [LC1 B1 ], [A1 B1 C1 ] < [BB1 A1 ]
(both — since [A1 B1 C1 ] is small), and [A1 B1 C1 ] ≤ [D1 A1 B1 ] < [AA1 D1 ] < [KA1 D1 ] < [KA1 C1 ]
(since triangle D1 A1 B1 is small). This contradicts the Lemma, applied for triangle A1 B1 C1
inside LKB.
Case 2. Ray C1 D1 does not intersect AB. Then choose a “sufficiently far” point K on
ray BA such that [KA1 C1 ] > [A1 B1 C1 ], and that ray KC1 intersects line BC at some point L
(see Figure 5). Since ray C1 D1 does not intersect line AB, the points A and D1 are on different
sides of KL; then A and D are also on different sides, and C is on the same side as A and B.
Then analogously we have [A1 B1 C1 ] < [CC1 B1 ] < [LC1 B1 ] and [A1 B1 C1 ] < [BB1 A1 ] since
triangle A1 B1 C1 is small. This (together with [A1 B1 C1 ] < [KA1 C1 ]) contradicts the Lemma
again.
48
49
G7. Given an acute triangle ABC with angles α, β and γ at vertices A, B and C, respectively,
such that β > γ. Point I is the incenter, and R is the circumradius. Point D is the foot of
the altitude from vertex A. Point K lies on line AD such that AK = 2R, and D separates A
and K. Finally, lines DI and KI meet sides AC and BC at E and F , respectively.
Prove that if IE = IF then β ≤ 3γ.
(Iran)
D0 B1
I O
T
D
B C
A1
Denote the perpendicular feet of incenter I on lines BC, AC, and AD by A1 , B1 , and T ,
respectively. Quadrilateral DA1 IT is a rectangle, hence T D = IA1 = IB1 .
Due to the right angles at T and B1 , quadrilateral AB1 IT is cyclic. Hence ∠B1 T I =
∠B1 AI = ∠CAM = ∠BAM = ∠BP M and ∠IB1 T = ∠IAT = ∠MAK = ∠MAP =
IT MP
∠MBP . Therefore, triangles B1 T I and BP M are similar and = .
IB1 MB
It is well-known that MB = MC = MI. Then right triangles IT D and KMI are also
50
IT IT MP KM
similar, because = = = . Hence, ∠KIM = ∠IDT = ∠IDA, and
TD IB1 MB MI
∠KID = ∠MID − ∠KIM = (∠IAD + ∠IDA) − ∠IDA = ∠IAD.
Now let us turn to the statement and suppose that IE = IF . Since IA1 = IB1 , the right
triangles IEB1 and IF A1 are congruent and ∠IEB1 = ∠IF A1 . Since β > γ, A1 lies in the
interior of segment CD and F lies in the interior of A1 D. Hence, ∠IF C is acute. Then two
cases are possible depending on the order of points A, C, B1 and E.
A A
B1
E
E
B1
I
D I
B C
F A1
D
B C
F A1
M
K K
If point E lies between C and B1 then ∠IF C = ∠IEA, hence quadrilateral CEIF is cyclic
and ∠F CE = 180◦ − ∠EIF = ∠KID. By (1), in this case we obtain ∠F CE = γ = ∠KID =
β−γ
and β = 3γ.
2
Otherwise, if point E lies between A and B1 , quadrilateral CEIF is a deltoid such that
∠IEC = ∠IF C < 90◦ . Then we have ∠F CE > 180◦ − ∠EIF = ∠KID. Therefore,
β−γ
∠F CE = γ > ∠KID = and β < 3γ.
2
Comment 1. In the case when quadrilateral CEIF is a deltoid, one can prove the desired inequality
without using (1). Actually, from ∠IEC = ∠IF C < 90◦ it follows that ∠ADI = 90◦ − ∠EDC <
∠AED − ∠EDC = γ. Since the incircle lies inside triangle ABC, we have AD > 2r (here r is the
β−γ
inradius), which implies DT < T A and DI < AI; hence = ∠IAD < ∠ADI < γ.
2
Solution 2. We give a different proof for (1). Then the solution can be finished in the same
way as above.
Define points M and P again; it can be proved in the same way that AM is the perpendicular
bisector of KP . Let J be the center of the excircle touching side BC. It is well-known that
points B, C, I, J lie on a circle with center M; denote this circle by ω1 .
Let B 0 be the reflection of point B about the angle bisector AM. By the symmetry, B 0 is the
second intersection point of circle ω1 and line AC. Triangles P BA and KB 0 A are symmetrical
51
with respect to line AM, therefore ∠KB 0 A = ∠P BA = 90◦. By the right angles at D and B 0 ,
points K, D, B 0 , C are concyclic and
AD · AK = AB 0 · AC.
AD · AK = AB 0 · AC = AI · AJ
I B0
C
B N
D
P
M
K
ω2
ω1
Let N be the point on circle ω2 which is opposite to K. Since ∠NDK = 90◦ = ∠CDK,
point N lies on line BC. Point M, being the center of circle ω1 , is the midpoint of segment IJ,
and KM is perpendicular to IJ. Therefore, line KM is the perpendicular bisector of IJ and
hence it passes through N.
From the cyclic quadrilateral IDKN we obtain
β−γ
∠KID = ∠KND = 90◦ − ∠DKN = 90◦ − ∠AKM = ∠MAK = .
2
Comment 2. The main difficulty in the solution is finding (1). If someone can guess this fact, he or
she can compute it in a relatively short way.
One possible way is finding and applying the relation AI 2 = 2R(ha − 2r), where ha = AD is the
length of the altitude. Using this fact, one can see that triangles AKI and AID0 are similar (here D 0
is the point symmetrical to D about T ). Hence, ∠M IK = ∠DD0 I = ∠IDD0 . The proof can be
finished as in Solution 1.
52
G8. Point P lies on side AB of a convex quadrilateral ABCD. Let ω be the incircle
of triangle CP D, and let I be its incenter. Suppose that ω is tangent to the incircles of
triangles AP D and BP C at points K and L, respectively. Let lines AC and BD meet at E,
and let lines AK and BL meet at F . Prove that points E, I, and F are collinear.
(Poland)
Solution. Let Ω be the circle tangent to segment AB and to rays AD and BC; let J be its
center. We prove that points E and F lie on line IJ.
Ω
D
ω
C
J
F
ωA I
K
IA L ωB
IB
A P B
Denote the incircles of triangles ADP and BCP by ωA and ωB . Let h1 be the homothety
with a negative scale taking ω to Ω. Consider this homothety as the composition of two
homotheties: one taking ω to ωA (with a negative scale and center K), and another one
taking ωA to Ω (with a positive scale and center A). It is known that in such a case the three
centers of homothety are collinear (this theorem is also referred to as the theorem on the three
similitude centers). Hence, the center of h1 lies on line AK. Analogously, it also lies on BL,
so this center is F . Hence, F lies on the line of centers of ω and Ω, i. e. on IJ (if I = J,
then F = I as well, and the claim is obvious).
Consider quadrilateral AP CD and mark the equal segments of tangents to ω and ωA (see the
figure below to the left). Since circles ω and ωA have a common point of tangency with P D,
one can easily see that AD + P C = AP + CD. So, quadrilateral AP CD is circumscribed;
analogously, circumscribed is also quadrilateral BCDP . Let ΩA and ΩB respectively be their
incircles.
Ω
D
D
ω
C
ω C J ΩB
ΩA
ΩA
ωA I JB
JA
E
A P
A P B
53
Consider the homothety h2 with a positive scale taking ω to Ω. Consider h2 as the compo-
sition of two homotheties: taking ω to ΩA (with a positive scale and center C), and taking ΩA
to Ω (with a positive scale and center A), respectively. So the center of h2 lies on line AC. By
analogous reasons, it lies also on BD, hence this center is E. Thus, E also lies on the line of
centers IJ, and the claim is proved.
Comment. In both main steps of the solution, there can be several different reasonings for the same
claims. For instance, one can mostly use Desargues’ theorem instead of the three homotheties theorem.
Namely, if IA and IB are the centers of ωA and ωB , then lines IA IB , KL and AB are concurrent (by
the theorem on three similitude centers applied to ω, ωA and ωB ). Then Desargues’ theorem, applied
to triangles AIA K and BIB L, yields that the points J = AIA ∩BIB , I = IA K ∩IB L and F = AK ∩BL
are collinear.
For the second step, let JA and JB be the centers of ΩA and ΩB . Then lines JA JB , AB and CD are
concurrent, since they appear to be the two common tangents and the line of centers of ΩA and ΩB .
Applying Desargues’ theorem to triangles AJA C and BJB D, we obtain that the points J = AJA ∩BJB ,
I = CJA ∩ DJB and E = AC ∩ BD are collinear.
54
Number Theory
N1. Find all pairs (k, n) of positive integers for which 7k − 3n divides k4 + n2 .
(Austria)
Answer. (2, 4).
Solution. Suppose that a pair (k, n) satisfies the condition of the problem. Since 7k − 3n is
even, k 4 + n2 is also even, hence k and n have the same parity. If k and n are odd, then
k 4 + n2 ≡ 1 + 1 = 2 (mod 4), while 7k − 3n ≡ 7 − 3 ≡ 0 (mod 4), so k 4 + n2 cannot be divisible
by 7k − 3n . Hence, both k and n must be even.
7a − 3b
Write k = 2a, n = 2b. Then 7k − 3n = 72a − 32b = · 2(7a + 3b ), and both factors are
2
integers. So 2(7a + 3b ) 7k − 3n and 7k − 3n k 4 + n2 = 2(8a4 + 2b2 ), hence
We prove by induction that 8a4 < 7a for a ≥ 4, 2b2 < 3b for b ≥ 1 and 2b2 + 9 ≤ 3b for b ≥ 3.
In the initial cases a = 4, b = 1, b = 2 and b = 3 we have 8 · 44 = 2048 < 74 = 2401, 2 < 3,
2 · 22 = 8 < 32 = 9 and 2 · 32 + 9 = 33 = 27, respectively.
If 8a4 < 7a (a ≥ 4) and 2b2 + 9 ≤ 3b (b ≥ 3), then
4 4
4 a+1
4 a 5 625
8(a + 1) = 8a <7 = 7a < 7a+1 and
a 4 256
2 2
2 2 b+1 b 4 16
2(b + 1) + 9 < (2b + 9) ≤3 = 3b < 3b+1 ,
b 3 9
as desired.
For a ≥ 4 we obtain 7a + 3b > 8a4 + 2b2 and inequality (1) cannot hold. Hence a ≤ 3, and
three cases are possible.
Case 1: a = 1. Then k = 2 and 8 + 2b2 ≥ 7 + 3b , thus 2b2 + 1 ≥ 3b . This is possible only
k 4 + n2 2 4 + 22 1
if b ≤ 2. If b = 1 then n = 2 and k n
= 2 2
= , which is not an integer. If b = 2
7 −3 7 −3 2
k 4 + n2 2 4 + 42
then n = 4 and k = 2 = −1, so (k, n) = (2, 4) is a solution.
7 − 3n 7 − 34
Case 2: a = 2. Then k = 4 and k 4 + n2 = 256 + 4b2 ≥ |74 − 3n | = |49 − 3b | · (49 + 3b ). The
smallest value of the first factor is 22, attained at b = 3, so 128 + 2b2 ≥ 11(49 + 3b ), which is
impossible since 3b > 2b2 .
Case 3: a = 3. Then k = 6 and k 4 + n2 = 1296 + 4b2 ≥ |76 − 3n | = |343 − 3b | · (343 + 3b ).
Analogously, |343 − 3b | ≥ 100 and we have 324 + b2 ≥ 25(343 + 3b ), which is impossible again.
We find that there exists a unique solution (k, n) = (2, 4).
56
N2. Let b, n > 1 be integers. Suppose that for each k > 1 there exists an integer ak such
that b − ank is divisible by k. Prove that b = An for some integer A.
(Canada)
Solution. Let the prime factorization of b be b = pα1 1 . . . pαs s , where p1 , . . . , ps are distinct primes.
α /n α /n
Our goal is to show that all exponents αi are divisible by n, then we can set A = p1 1 . . . ps s .
Apply the condition for k = b2 . The number b − ank is divisible by b2 and hence, for
each 1 ≤ i ≤ s, it is divisible by p2α
i
i
> pαi i as well. Therefore
and
ank ≡ b 6≡ 0 (mod piαi +1 ),
which implies that the largest power of pi dividing ank is pαi i . Since ank is a complete nth power,
this implies that αi is divisible by n.
Comment. If n = 8 and b = 16, then for each prime p there exists an integer ap such that b − anp is
divisible by p. Actually, the congruency x8 − 16 ≡ 0 (mod p) expands as
N3. Let X be a set of 10 000 integers, none of them is divisible by 47. Prove that there
exists a 2007-element subset Y of X such that a − b + c − d + e is not divisible by 47 for any
a, b, c, d, e ∈ Y .
(Netherlands)
Solution. Call a set M of integers good if 47 6 a − b + c − d + e for any a, b, c, d, e ∈ M .
Consider the set J = {−9, −7, −5, −3, −1, 1, 3, 5, 7, 9}. We claim that J is good. Actually,
for any a, b, c, d, e ∈ J the number a − b + c − d + e is odd and
Ak = {x ∈ X | ∃j ∈ J : kx ≡ j (mod 47)}.
If Ak is not good, then 47 a − b + c − d + e for some a, b, c, d, e ∈ Ak , hence 47 ka − kb +
kc − kd + ke. But set J contains numbers with the same residues modulo 47, so J also is not
good. This is a contradiction; therefore each Ak is a good subset of X.
Then it suffices to prove that there exists a number k such that |Ak | ≥ 2007. Note that
each x ∈ X is contained in exactly 10 sets Ak . Then
46
X
|Ak | = 10|X| = 100 000,
k=1
N4. For every integer k ≥ 2, prove that 23k divides the number
k
2k+1 2
k
− k−1 (1)
2 2
We compute the exponent of 2 in the prime decomposition of each factor (the first one is a
rational number but not necessarily an integer; it is not important).
First, we show by induction on n that the exponent of 2 in (2n )! is 2n − 1. The base
n
case n = 1 is trivial. Suppose that (2n )! = 22 −1 (2d + 1) for some integer d. Then we have
n n n −1 n+1 −1
(2n+1 )! = 22 (2n )! (2n+1 − 1)!! = 22 22 · (2d + 1)(2n+1 − 1)!! = 22 · (2q + 1)
For any integer i = 1, . . . , 2k−1, denote by a2i−1 the residue inverse to 2i − 1 modulo 2k . Clearly,
when 2i − 1 runs through all odd residues, so does a2i−1 , hence
k−1
2X 2k−1 2k−1
(2k − 1)!! X (2k − 1)!! X
S= ≡ − ≡ − (2k − 1)!! a22i−1
i=1
(2i − 1)(2k − 2i + 1) i=1
(2i − 1) 2
i=1
k−1
2X
k 2 k 2k−1(22k − 1)
= −(2 − 1)!! (2i − 1) = −(2 − 1)!! (mod 2k ).
i=1
3
which is divisible exactly by 23k−1 . Thus, the exponent of 2 in (2) is 1 + (3k − 1) = 3k.
Comment. The fact that (1) is divisible by 22k is known; but it does not help in solving this problem.
60
N5. Find all surjective functions f : N → N such that for every m, n ∈ N and every prime p,
the number f (m + n) is divisible by p if and only if f (m) + f (n) is divisible by p.
(N is the set of all positive integers.)
(Iran)
Answer. f (n) = n.
Solution. Suppose that function f : N → N satisfies the problem conditions.
Lemma. For any prime p and any x, y ∈ N, we have x ≡ y (mod p) if and only if f (x) ≡ f (y)
(mod p). Moreover, p f (x) if and only if p x.
Proof. Consider an arbitrary prime p. Since f is surjective, there exists some x ∈ N such
that p f (x). Let
d = min x ∈ N : p f (x) .
By induction on k, we obtain that p f (kd) for all k ∈ N. The base
is true since p f (d).
Moreover, if p f (kd) and p f (d) then, by the problem condition, p f (kd + d) = f (k + 1)d
as required.
Suppose that there exists an x ∈ N such that d 6 x but p f (x). Let
y = min x ∈ N : d 6 x, p f (x) .
We are left to show that p = d: in this case (1) and (2) provide the desired statements.
The numbers 1, 2, . . . , d have distinct residues modulo d. By (2), numbers f (1), f (2), . . . ,
f (d) have distinct residues modulo p; hence there are at least d distinct residues, and p ≥ d.
On the other hand, by the surjectivity of f , there exist x1 , . . . , xp ∈ N such that f (xi ) = i for
any i = 1, 2, . . . , p. By (2), all these xi ’s have distinct residues modulo d. For the same reasons,
d ≥ p. Hence, d = p.
Now we prove that f (n) = n by induction on n. If n = 1 then, by the Lemma, p 6 f (1) for
any prime p, so f (1) = 1, and thebase is established. Suppose that n > 1 and denote k = f (n).
Note that there exists a prime q n, so by the Lemma q k and k > 1.
If k > n then k − n + 1 > 1, and there exists a prime p k − n + 1; we have k ≡ n − 1
(mod p). By the induction hypothesis we have f (n − 1) = n − 1 ≡ k = f (n) (mod p). Now,
by the Lemma we obtain n − 1 ≡ n (mod p) which cannot be true.
61
Analogously, if k < n,then f (k−1) = k−1 by induction hypothesis. Moreover, n−k+1 > 1,
so there exists a prime p n − k + 1 and n ≡ k − 1 (mod p). By the Lemma again, k = f (n) ≡
f (k − 1) = k − 1 (mod p), which is also false. The only remaining case is k = n, so f (n) = n.
Finally, the function f (n) = n obviously satisfies the condition.
62
N6. Let k be a positive integer. Prove that the number (4k2 − 1)2 has a positive divisor of
the form 8kn − 1 if and only if k is even.
(United Kingdom)
Solution. The statement follows from the following fact.
Lemma. For arbitrary positive integers x and y, the number 4xy − 1 divides (4x2 − 1)2 if and
only if x = y.
Proof. If x = y then 4xy − 1 = 4x2 − 1 obviously divides (4x2 − 1)2 so it is sufficient to consider
the opposite direction.
Call a pair (x, y) of positive integers bad if 4xy − 1 divides (4x2 − 1)2 but x 6= y. In order to
prove that bad pairs do not exist, we present two properties of them which provide an infinite
descent.
Property (i). If (x, y) is a bad pair and x < y then there exists a positive integer z < x such
that (x, z) is also bad.
(4x2 − 1)2
Let r = . Then
4xy − 1
and r = 4xz − 1 with some positive integer z. From x < y we obtain that
(4x2 − 1)2
4xz − 1 = < 4x2 − 1
4xy − 1
and therefore z < x. By the construction, the number 4xz − 1 is a divisor of (4x2 − 1)2 so (x, z)
is a bad pair.
Property (ii). If (x, y) is a bad pair then (y, x) is also bad.
Since 1 = 12 ≡ (4xy)2 (mod 4xy − 1), we have
2
(4y 2 − 1)2 ≡ 4y 2 − (4xy)2 = 16y 4(4x2 − 1)2 ≡ 0 (mod 4xy − 1).
N7. For a prime p and a positive integer n, denote by νp (n) the exponent of p in the prime
factorization of n!. Given a positive integer d and a finite set {p1 , . . . , pk } of primes. Show that
there are infinitely many positive integers n such that d νpi (n) for all 1 ≤ i ≤ k.
(India)
Solution 1. For arbitrary prime p and positive integer n, denote by ordp (n) the exponent of p
in n. Thus,
n
X
νp (n) = ordp (n!) = ordp (i).
i=1
Lemma. Let p be a prime number, q be a positive integer, k and r be positive integers such
that pk > r. Then νp (qpk + r) = νp (qpk ) + νp (r).
Proof. We claim that ordp (qpk + i) = ordp (i) for all 0 < i < pk . Actually, if d = ordp (i)
then d < k, so qpk + i is divisible by pd , but only the first term is divisible by pd+1 ; hence the
sum is not.
Using this claim, we obtain
qpk qpk +r k
qp
X X X r
X
k
νp (qp + r) = ordp (i) + ordp (i) = ordp (i) + ordp (i) = νp (qpk ) + νp (r).
i=1 i=qpk +1 i=1 i=1
For any integer a, denote by a its residue modulo d. The addition of residues will also be
performed modulo d, i. e. a+b = a + b. For any positive integer n, let f (n) = f1 (n), . . . , fk (n) ,
where fi (n) = νpi (n).
Define the sequence n1 = 1, n`+1 = (p1 p2 . . . pk )n` . We claim that
for any `1 < `2 < . . . < `m . (The addition of k-tuples is componentwise.) The base case m = 1
is trivial.
n
Suppose that m > 1. By the construction of the sequence, pi `1 divides n`2 +. . .+n`m ; clearly,
n
pi `1 > n`1 for all 1 ≤ i ≤ k. Therefore the Lemma can be applied for p = pi , k = r = n`1
and qpk = n`2 + . . . + n`m to obtain
and hence
Algebra 7
Problem A1 . . . . . . . . . . . . . . . . . . . . . . . . . . . . . . . . . . . . . . 7
Problem A2 . . . . . . . . . . . . . . . . . . . . . . . . . . . . . . . . . . . . . . 9
Problem A3 . . . . . . . . . . . . . . . . . . . . . . . . . . . . . . . . . . . . . . 11
Problem A4 . . . . . . . . . . . . . . . . . . . . . . . . . . . . . . . . . . . . . . 12
Problem A5 . . . . . . . . . . . . . . . . . . . . . . . . . . . . . . . . . . . . . . 14
Problem A6 . . . . . . . . . . . . . . . . . . . . . . . . . . . . . . . . . . . . . . 15
Problem A7 . . . . . . . . . . . . . . . . . . . . . . . . . . . . . . . . . . . . . . 17
Combinatorics 21
Problem C1 . . . . . . . . . . . . . . . . . . . . . . . . . . . . . . . . . . . . . . 21
Problem C2 . . . . . . . . . . . . . . . . . . . . . . . . . . . . . . . . . . . . . . 23
Problem C3 . . . . . . . . . . . . . . . . . . . . . . . . . . . . . . . . . . . . . . 24
Problem C4 . . . . . . . . . . . . . . . . . . . . . . . . . . . . . . . . . . . . . . 25
Problem C5 . . . . . . . . . . . . . . . . . . . . . . . . . . . . . . . . . . . . . . 26
Problem C6 . . . . . . . . . . . . . . . . . . . . . . . . . . . . . . . . . . . . . . 27
Geometry 29
Problem G1 . . . . . . . . . . . . . . . . . . . . . . . . . . . . . . . . . . . . . . 29
Problem G2 . . . . . . . . . . . . . . . . . . . . . . . . . . . . . . . . . . . . . . 31
Problem G3 . . . . . . . . . . . . . . . . . . . . . . . . . . . . . . . . . . . . . . 32
Problem G4 . . . . . . . . . . . . . . . . . . . . . . . . . . . . . . . . . . . . . . 34
Problem G5 . . . . . . . . . . . . . . . . . . . . . . . . . . . . . . . . . . . . . . 36
Problem G6 . . . . . . . . . . . . . . . . . . . . . . . . . . . . . . . . . . . . . . 37
Problem G7 . . . . . . . . . . . . . . . . . . . . . . . . . . . . . . . . . . . . . . 40
Number Theory 43
Problem N1 . . . . . . . . . . . . . . . . . . . . . . . . . . . . . . . . . . . . . . 43
Problem N2 . . . . . . . . . . . . . . . . . . . . . . . . . . . . . . . . . . . . . . 45
Problem N3 . . . . . . . . . . . . . . . . . . . . . . . . . . . . . . . . . . . . . . 46
Problem N4 . . . . . . . . . . . . . . . . . . . . . . . . . . . . . . . . . . . . . . 47
Problem N5 . . . . . . . . . . . . . . . . . . . . . . . . . . . . . . . . . . . . . . 49
Problem N6 . . . . . . . . . . . . . . . . . . . . . . . . . . . . . . . . . . . . . . 50
Contributing Countries
Solution. Let f satisfy the given condition. Setting p = q = r = s =√1 yields f (1)2 = f (1) and
hence f (1) = 1. Now take any x > 0 and set p = x, q = 1, r = s = x to obtain
f (x)2 + 1 x2 + 1
= .
2f (x) 2x
And thus,
1
for every x > 0, either f (x) = x or f (x) = . (1)
x
Obviously, if
1
f (x) = x for all x > 0 or f (x) = for all x > 0 (2)
x
then the condition of the problem is satisfied. We show that actually these two functions are
the only solutions.
So let us assume that there exists a function f satisfying the requirement, other than
those in (2). Then f (a) 6= a and f (b) 6= 1/b for some a, b > 0. By (1), these √
values must be
f (a) = 1/a, f (b) = b. Applying now the equation with p = a, q = b, r = s = ab we obtain
(a−2 + b2 )/2f (ab) = (a2 + b2 )/2ab ; equivalently,
ab(a−2 + b2 )
f (ab) = . (3)
a2 + b2
We know however (see (1)) that f (ab) must be either ab or 1/ab . If f (ab) = ab then by (3)
a−2 + b2 = a2 + b2 , so that a = 1. But, as f (1) = 1, this contradicts the relation f (a) 6= a.
Likewise, if f (ab) = 1/ab then (3) gives a2 b2 (a−2 + b2 ) = a2 + b2 , whence b = 1, in contradiction
to f (b) 6= 1/b . Thus indeed the functions listed in (2) are the only two solutions.
8
Comment. The equation has as many as four variables with only one constraint pq = rs, leaving
three degrees of freedom and providing a lot of information. Various substitutions force various useful
properties of the function searched. We sketch one more method to reach conclusion (1); certainly
there are many others. √ √
Noticing that f (1) = 1 and setting, first, p = q = 1, r = x, s = 1/ x, and then p = x, q = 1/x,
r = s = 1, we obtain two relations, holding for every x > 0,
2
1 1 2 1 1
f (x) + f =x+ and f (x) + f = x2 + 2 . (4)
x x x x
Squaring the first and subtracting the second gives 2f (x)f (1/x) = 2. Subtracting this from the second
relation of (4) leads to
2
1 1 2 1 1
f (x) − f = x− or f (x) − f =± x− .
x x x x
The last two alternatives combined with the first equation of (4) imply the two alternatives of (1).
9
The inequality to be proved reads a2 + b2 + c2 ≥ 1. The new variables are subject to the
constraints a, b, c 6= 1 and the following one coming from the condition xyz = 1,
a+b+c−1 = ab + bc + ca,
2(a + b + c − 1) = (a + b + c)2 − (a2 + b2 + c2 ),
a2 + b2 + c2 − 2 = (a + b + c)2 − 2(a + b + c),
a2 + b2 + c2 − 1 = (a + b + c − 1)2 .
a + b + c = 1, ab + bc + ca = 0
b2 + (a − 1)b + a(a − 1) = 0,
with discriminant
∆ = (a − 1)2 − 4a(a − 1) = (1 − a)(1 + 3a).
We are looking for rational triples (a, b, c); it will suffice to have a rational such that 1 − a
and 1 + 3a are both squares of rational numbers (then ∆ will be so too). Set a = k/m. We
want m − k and m + 3k to be squares of integers. This is achieved for instance by taking
m = k 2 − k + 1 (clearly nonzero); then m − k = (k − 1)2 , m + 3k = (k + 1)2 . Note that dis-
tinct integers k yield distinct values of a = k/m.
And thus, if k is any integer and m = k 2 − k + 1, a = k/m then ∆ = (k 2 − 1)2 /m2 and the
quadratic equation has rational roots b = (m − k ± k 2 ∓ 1)/(2m). Choose e.g. the larger root,
m − k + k2 − 1 m + (m − 2) m−1
b= = = .
2m 2m m
10
Comment 1. There are many possible variations in handling the equation system a2 + b2 + c2 = 1,
a + b + c = 1 (a, b, c 6= 1) which of course describes a circle in the (a, b, c)-space (with three points
excluded), and finding infinitely many rational points on it.
Also the initial substitution x = a/(a − 1) (etc.) can be successfully replaced by other similar
substitutions, e.g. x = 1 − 1/α (etc.); or x = x0 − 1 (etc.); or 1 − yz = u (etc.)—eventually reducing
the inequality to (· · · )2 ≥ 0, the expression in the parentheses depending on the actual substitution.
Depending on the method chosen, one arrives at various sequences of rational triples (x, y, z)
as needed; let us produce just one more such example: x = (2r − 2)/(r + 1)2 , y = (2r + 2)/(r − 1)2 ,
z = (r 2 − 1)/4 where r can be any rational number different from 1 or −1.
Solution 2 (an outline). (a) Without changing variables, just setting z = 1/xy and clearing
fractions, the proposed inequality takes the form
(xy − 1)2 x2 (y − 1)2 + y 2 (x − 1)2 + (x − 1)2 (y − 1)2 ≥ (x − 1)2 (y − 1)2 (xy − 1)2 .
With the notation p = x + y, q = xy this becomes, after lengthy routine manipulation and a
lot of cancellation
q 4 − 6q 3 + 2pq 2 + 9q 2 − 6pq + p2 ≥ 0.
It is not hard to notice that the expression on the left is just (q 2 − 3q + p)2 , hence nonnegative.
(Without introducing p and q, one is of course led with some more work to the same
expression, just written in terms of x and y; but then it is not that easy to see that it is a
square.)
(b) To have equality, one needs q 2 − 3q + p = 0. Note that x and y are the roots of
the quadratic trinomial (in a formal variable t): t2 − pt + q. When q 2 − 3q + p = 0, the
discriminant equals
Comment 2. Part (a) alone might also be considered as a possible contest problem (in the category
of easy problems).
11
A3. Let S ⊆ R be a set of real numbers. We say that a pair (f, g) of functions from S into S
is a Spanish Couple on S, if they satisfy the following conditions:
(i) Both functions are strictly increasing, i.e. f (x) < f (y) and g(x) < g(y) for all x, y ∈ S
with x < y;
(ii) The inequality f (g(g(x))) < g(f (x)) holds for all x ∈ S.
Decide whether there exists a Spanish Couple
(a) on the set S = N of positive integers;
(b) on the set S = {a − 1/b : a, b ∈ N}.
Solution. We show that the answer is NO for part (a), and YES for part (b).
k
z }| {
(a) Throughout the solution, we will use the notation gk (x) = g(g(. . . g(x) . . .)), including
g0 (x) = x as well.
Suppose that there exists a Spanish Couple (f, g) on the set N. From property (i) we have
f (x) ≥ x and g(x) ≥ x for all x ∈ N.
We claim that gk (x) ≤ f (x) for all k ≥ 0 and all positive integers x. The proof is done by
induction on k. We already have the base case k = 0 since x ≤ f (x). For the induction step
from k to k + 1, apply the induction hypothesis on g2 (x) instead of x, then apply (ii):
g(gk+1(x)) = gk g2 (x) ≤ f g2 (x) < g(f (x)).
Since g is increasing, it follows that gk+1(x) < f (x). The claim is proven.
If g(x) = x for all x ∈ N then f (g(g(x))) = f (x) = g(f (x)), and we have a contradiction
with (ii). Therefore one can choose an x0 ∈ S for which x0 < g(x0 ). Now consider the sequence
x0 , x1 , . . . where xk = gk (x0 ). The sequence is increasing. Indeed, we have x0 < g(x0 ) = x1 ,
and xk < xk+1 implies xk+1 = g(xk ) < g(xk+1) = xk+2 .
Hence, we obtain a strictly increasing sequence x0 < x1 < . . . of positive integers which on
the other hand has an upper bound, namely f (x0 ). This cannot happen in the set N of positive
integers, thus no Spanish Couple exists on N.
f (a − 1/b) = a + 1 − 1/b,
g(a − 1/b) = a − 1/(b + 3a ).
Comment. Another example of a Spanish couple is f (a − 1/b) = 3a − 1/b, g(a − 1/b) = a − 1/(a+b).
More generally, postulating f (a − 1/b) = h(a) − 1/b, g(a − 1/b) = a − 1/G(a, b) with h increasing
and G increasing in both variables, we get that f ◦ g ◦ g < g ◦ f holds if G a, G(a, b) < G h(a), b .
A search just among linear functions h(a) = Ca, G(a, b) = Aa + Bb results in finding that any in-
tegers A > 0, C > 2 and B = 1 produce a Spanish couple (in the example above, A = 1, C = 3). The
proposer’s example results from taking h(a) = a + 1, G(a, b) = 3a + b.
12
A4. For an integer m, denote by t(m) the unique number in {1, 2, 3} such that m + t(m) is a
multiple of 3. A function f : Z → Z satisfies f (−1) = 0, f (0) = 1, f (1) = −1 and
Solution. The given conditions determine f uniquely on the positive integers. The signs of
f (1), f (2), . . . seem to change quite erratically. However values of the form f (2n − t(m)) are
sufficient to compute directly any functional value. Indeed, let n > 0 have base 2 representation
n = 2a0 +2a1 +· · ·+2ak , a0 > a1 > · · · > ak ≥ 0, and let nj = 2aj +2aj−1 +· · ·+2ak , j = 0, . . . , k.
Repeated applications of the recurrence show that f (n) is an alternating sum of the quantities
f (2aj − t(nj+1 )) plus (−1)k+1 . (The exact formula is not needed for our proof.)
So we focus attention on the values f (2n − 1), f (2n − 2) and f (2n − 3). Six cases arise; more
specifically,
t(22k −3) = 2, t(22k −2) = 1, t(22k −1) = 3, t(22k+1 −3) = 1, t(22k+1 −2) = 3, t(22k+1 −1) = 2.
Proof. By induction on k. The base k = 0 comes down to checking that f (2) = −1 and
f (3) = 2; the given values f (−1) = 0, f (0) = 1, f (1) = −1 are also needed. Suppose the claim
holds for k − 1. For f (22k+1 − t(m)), the recurrence formula and the induction hypothesis yield
One more (direct) consequence of the claim is that |f (2n − t(m))| ≤ 32 · 3n/2 for all m, n ≥ 0.
The last inequality enables us to find an upper bound for |f (m)| for m less than a given
power of 2. We prove by induction on n that |f (m)| ≤ 3n/2 holds true for all integers m, n ≥ 0
with 2n > m.
13
The base n = 0 is clear as f (0) = 1. For the inductive step from n to n + 1, let m and n
satisfy 2n+1 > m. If m < 2n , we are done by the inductive hypothesis. If m ≥ 2n then
m = 2n + k where 2n > k ≥ 0. Now, by |f (2n − t(k))| ≤ 32 · 3n/2 and the inductive assumption,
2 n/2
|f (m)| = |f (2n − t(k)) − f (k)| ≤ |f (2n − t(k))| + |f (k)| ≤ · 3 + 3n/2 < 3(n+1)/2 .
3
The induction is complete.
We proceed to prove that f (3p) ≥ 0 for all integers p ≥ 0. Since 3p is not a power of 2, its
binary expansion contains at least two summands. Hence one can write 3p = 2a + 2b + c where
a > b and 2b > c ≥ 0. Applying the recurrence formula twice yields
Solution. We show that if abcd = 1, the sum a + b + c + d cannot exceed a certain weighted
a b c d b c d a
mean of the expressions + + + and + + + .
b c d a a b c d
a a b a
By applying the AM-GM inequality to the numbers , , and , we obtain
b b c d
r r
4 a4 4 a a b a 1 a a b a
a= = · · · ≤ + + + .
abcd b b c d 4 b b c d
Analogously,
1 b b c b 1 c c d c 1 d d a d
b≤ + + + , c≤ + + + and d ≤ + + + .
4 c c d a 4 d d a b 4 a a b c
a b c d b c d a
In particular, if a + b + c + d > + + + then a + b + c + d < + + + .
b c d a a b c d
Comment. The estimate in the above solution was obtained by applying the AM-GM inequality to
each column of the 4 × 4 array
a/b b/c c/d d/a
a/b b/c c/d d/a
b/c c/d d/a a/b
a/d b/a c/b d/c
and adding up the resulting inequalities. The same table yields a stronger bound: If a, b, c, d > 0 and
abcd = 1 then
a b c d 3 b c d a
+ + + + + + ≥ (a + b + c + d)4 .
b c d a a b c d
It suffices to apply Hölder’s inequality to the sequences in the four rows, with weights 1/4:
a b c d 1/4 a b c d 1/4 b c d a 1/4 a b c d 1/4
+ + + + + + + + + + + +
b c d a b c d a c d a b d a b c
aaba 1/4 bbcb 1/4 ccdc 1/4 ddad 1/4
≥ + + + = a + b + c + d.
bbcd ccda ddab aabc
15
Solution. Suppose that the statement is false and f (R) = N. We prove several properties of
the function f in order to reach a contradiction.
To start with, observe that one can assume f (0) = 1. Indeed, let a ∈ R be such that
f (a) = 1, and consider the function g(x) = f (x + a). By substituting x + a and y + a for x
and y in (1), we have
1 1 1 1
g x+ =f x+a+ =f y+a+ =g y+ .
g(y) f (y + a) f (x + a) g(x)
So g satisfies the functional equation (1), with the additional property g(0) = 1. Also, g and f
have the same set of values: g(R) = f (R) = N. Henceforth we assume f (0) = 1.
1
Claim 1. For an arbitrary fixed c ∈ R we have f c + : n ∈ N = N.
n
Proof. Equation (1) and f (R) = N imply
1 1 1
f (R) = f x + :x∈R = f c+ :x∈R ⊂ f c+ : n ∈ N ⊂ f (R).
f (c) f (x) n
The claim follows.
We will use Claim 1 in the special cases c = 0 and c = 1/3:
1 1 1
f :n∈N = f + : n ∈ N = N. (2)
n 3 n
Claim 2. If f (u) = f (v) for some u, v ∈ R then f (u+q) = f (v+q) for all nonnegative rational q.
Furthermore, if f (q) = 1 for some nonnegative rational q then f (kq) = 1 for all k ∈ N.
Proof. For all x ∈ R we have by (1)
1 1 1 1
f u+ =f x+ =f x+ =f v+ .
f (x) f (u) f (v) f (x)
Since f (x) attains all positive integer values, this yields f (u + 1/n) = f (v + 1/n) for all n ∈ N.
Let q = k/n be a positive rational number. Then k repetitions of the last step yield
k k
f (u + q) = f u + =f v+ = f (v + q).
n n
Now let f (q) = 1 for some nonnegative rational q, and let k ∈ N. As f (0) = 1, the previous
conclusion yields successively f (q) = f (2q), f (2q) = f (3q), . . . , f ((k − 1)q) = f (kq), as needed.
Claim 3. The equality f (q) = f (q + 1) holds for all nonnegative rational q.
Proof. Let m be a positive integer such that f (1/m) = 1. Such an m exists by (2). Applying
the second statement of Claim 2 with q = 1/m and k = m yields f (1) = 1.
Given that f (0) = f (1) = 1, the first statement of Claim 2 implies f (q) = f (q + 1) for all
nonnegative rational q.
16
1
Claim 4. The equality f = n holds for every n ∈ N.
n
Proof. For a nonnegative rational q we set x = q, y = 0 in (1) and use Claim 3 to obtain
1 1
f =f q+ = f (q + 1) = f (q).
f (q) f (0)
By (2), for each n ∈ N there exists a k ∈ N such that f (1/k) = n. Applying the last equation
with q = 1/k, we have
1 1 1
n=f =f =f .
k f (1/k) n
Now we are ready to obtain a contradiction. Let n ∈ N be such that f (1/3 + 1/n) = 1.
Such an n exists by (2). Let 1/3 + 1/n = s/t, where s, t ∈ N are coprime. Observe that t > 1
as 1/3 + 1/n is not an integer. Choose k, l ∈ N so that that ks − lt = 1.
Because f (0) = f (s/t) = 1, Claim 2 implies f (ks/t) = 1. Now f (ks/t) = f (1/t + l); on the
other hand f (1/t + l) = f (1/t) by l successive applications of Claim 3. Finally, f (1/t) = t by
Claim 4, leading to the impossible t = 1. The solution is complete.
17
A7. Prove that for any four positive real numbers a, b, c, d the inequality
(a − b)(a − c) (b − c)(b − d) (c − d)(c − a) (d − a)(d − b)
+ + + ≥0
a+b+c b+c+d c+d+a d+a+b
holds. Determine all cases of equality.
Note that
MN > ac(a + c) + bd(b + d) s ≥ |W | · s. (4)
Now (2) and (4) yield
3 · |a − c| · |b − d|
|A00 + B 00 + C 00 + D 00 | ≤ . (5)
s
Combined with (1) this results in
2(A + B + C + D) = (A0 + B 0 + C 0 + D 0 ) + (A00 + B 00 + C 00 + D 00 )
16 · |a − c| · |b − d| 3 · |a − c| · |b − d| 7 · |a − c| · |b − d|
≥ − = ≥ 0.
3s s 3(a + b + c + d)
This is the required inequality. From the last line we see that equality can be achieved only if
either a = c or b = d. Since we also need equality in (1), this implies that actually a = c and
b = d must hold simultaneously, which is obviously also a sufficient condition.
Solution 2. We keep the notations A, B, C, D, s, and also M, N, W from the preceding
solution; the definitions of M, N, W and relations (3), (4) in that solution did not depend on
the foregoing considerations. Starting from
(a − c)2 + 3(a + c)(a − c)
2A = − 2a + 2c,
a+b+c
we get
2 1 1 1 1
2(A + C) = (a − c) + + 3(a + c)(a − c) −
s−d s−b s−d s−b
2s − b − d d−b p(a − c)2 − 3(a + c)(a − c)(b − d)
= (a − c)2 + 3(a + c)(a − c) · =
M M M
where p = 2s − b − d = s + a + c. Similarly, writing q = s + b + d we have
q(b − d)2 − 3(b + d)(b − d)(c − a)
2(B + D) = ;
N
specific grouping of terms in the numerators has its aim. Note that pq > 2s2 . By adding the
fractions expressing 2(A + C) and 2(B + D),
p(a − c)2 3(a − c)(b − d)W q(b − d)2
2(A + B + C + D) = + +
M MN N
with W defined by (3).
Substitution x = (a − c)/M, y = (b − d)/N brings the required inequality to the form
2(A + B + C + D) = Mpx2 + 3W xy + Nqy 2 ≥ 0. (6)
It will be enough to verify that the discriminant ∆ = 9W 2 − 4MNpq of the quadratic trinomial
Mpt2 + 3W t + Nq is negative; on setting t = x/y one then gets (6).
The first inequality in (4)
together with pq > 2s2 imply 4MNpq > 8s3 ac(a + c) + bd(b + d) . Since
(a + c)s3 > (a + c)4 ≥ 4ac(a + c)2 and likewise (b + d)s3 > 4bd(b + d)2 ,
the estimate continues as follows,
2
4MNpq > 8 4(ac)2 (a + c)2 + 4(bd)2 (b + d)2 > 32 bd(b + d) − ac(a + c) = 32W 2 ≥ 9W 2 .
Thus indeed ∆ < 0. The desired inequality (6) hence results. It becomes an equality if and
only if x = y = 0; equivalently, if and only if a = c and simultaneously b = d.
19
Comment. The two solutions presented above do not differ significantly; large portions overlap. The
properties of the number W turn out to be crucial in both approaches. The Cauchy-Schwarz inequality,
applied in the first solution, is avoided in the second, which requires no knowledge beyond quadratic
trinomials.
The estimates in the proof of ∆ < 0 in the second solution seem to be very wasteful. However,
they come close to sharp when the terms in one of the pairs (a, c), (b, d) are equal and much bigger
than those in the other pair.
In attempts to prove the inequality by just considering the six cases of arrangement of the numbers
a, b, c, d on the real line, one soon discovers that the cases which create real trouble are precisely
those in which a and c are both greater or both smaller than b and d.
Solution 3.
(a − b)(a − c)(a + b + d)(a + c + d)(b + c + d) =
= (a − b)(a + b + d) (a − c)(a + c + d) (b + c + d) =
= (a2 + ad − b2 − bd)(a2 + ad − c2 − cd)(b + c + d) =
= a4 +2a3 d−a2 b2 −a2 bd−a2 c2 −a2 cd+a2 d2 −ab2 d−abd2 −ac2 d−acd2 +b2 c2 +b2 cd+bc2 d+bcd2 (b+c+d) =
= a4 b + a4 c + a4 d + (b3 c2 + a2 d3 ) − a2 c3 + (2a3 d2 − b3 a2 + c3 b2 )+
+(b3 cd − c3 da − d3 ab) + (2a3 bd + c3 db − d3 ac) + (2a3 cd − b3 da + d3 bc)
+(−a2 b2 c + 3b2 c2 d − 2ac2 d2 ) + (−2a2 b2 d + 2bc2 d2 ) + (−a2 bc2 − 2a2 c2 d − 2ab2 d2 + 2b2 cd2 )+
+(−2a2 bcd − ab2 cd − abc2 d − 2abcd2 )
P
Introducing the notation Sxyzw = ax by cz dw , one can write
cyc
X
(a − b)(a − c)(a + b + d)(a + c + d)(b + c + d) =
cyc
= S4100 + S4010 + S4001 + 2S3200 − S3020 + 2S3002 − S3110 + 2S3101 + 2S3011 − 3S2120 − 6S2111 =
1 1
+ S4100 + S4001 + S3110 + S3011 − 3S2120 +
2 2
3 3 9 9 9
+ S4010 − S3020 − S3110 + S3011 + S2210 + S2201 − S2111 +
2 2 16 16 8
9 23 39
+ S3200 − S2210 − S2201 + S3002 + S3200 − 2S3101 + S3002 + S3101 − S2111 ,
16 16 8
where the expressions
1 1 X 1 3 1 3
4 4 2 2
S4100 + S4001 + S3110 + S3011 − 3S2120 = a b + bc + a bc + abc − 3a bc ,
2 2 cyc
2 2
2
3 3 9 9 9 X
2 3 3
S4010 − S3020 − S3110 + S3011 + S2210 + S2201 − S2111 = a c a−c− b+ d ,
2 2 16 16 8 cyc
4 4
X X
S3200 − S2210 − S2201 + S3002 = b2 (a3 − a2 c − ac2 + c3 ) = b2 (a + c)(a − c)2 ,
cyc cyc
X 1X
S3200 − 2S3101 + S3002 = a3 (b − d)2 and S3101 − S2111 = bd(2a3 + c3 − 3a2 c)
cyc
3 cyc
are all nonnegative.
20
Combinatorics
C1. In the plane we consider rectangles whose sides are parallel to the coordinate axes and
have positive length. Such a rectangle will be called a box . Two boxes intersect if they have a
common point in their interior or on their boundary.
Find the largest n for which there exist n boxes B1 , . . . , Bn such that Bi and Bj intersect if
and only if i 6≡ j ± 1 (mod n).
Solution. The maximum number of such boxes is 6. One example is shown in the figure.
B5
B1 B2
B3 B4
B6
Now we show that 6 is the maximum. Suppose that boxes B1 , . . . , Bn satisfy the condition.
Let the closed intervals Ik and Jk be the projections of Bk onto the x- and y-axis, for 1 ≤ k ≤ n.
If Bi and Bj intersect, with a common point (x, y), then x ∈ Ii ∩ Ij and y ∈ Ji ∩ Jj . So the
intersections Ii ∩ Ij and Ji ∩ Jj are nonempty. Conversely, if x ∈ Ii ∩ Ij and y ∈ Ji ∩ Jj for some
real numbers x, y, then (x, y) is a common point of Bi and Bj . Putting it around, Bi and Bj
are disjoint if and only if their projections on at least one coordinate axis are disjoint.
For brevity we call two boxes or intervals adjacent if their indices differ by 1 modulo n, and
nonadjacent otherwise.
The adjacent boxes Bk and Bk+1 do not intersect for each k = 1, . . . , n. Hence (Ik , Ik+1 )
or (Jk , Jk+1) is a pair of disjoint intervals, 1 ≤ k ≤ n. So there are at least n pairs of disjoint
intervals among (I1 , I2 ), . . . , (In−1 , In ), (In , I1 ); (J1 , J2 ), . . . , (Jn−1 , Jn ), (Jn , J1 ).
Next, every two nonadjacent boxes intersect, hence their projections on both axes intersect,
too. Then the claim below shows that at most 3 pairs among (I1 , I2 ), . . . , (In−1 , In ), (In , I1 ) are
disjoint, and the same holds for (J1 , J2 ), . . . , (Jn−1 , Jn ), (Jn , J1 ). Consequently n ≤ 3 + 3 = 6,
as stated. Thus we are left with the claim and its justification.
Claim. Let ∆1 , ∆2 , . . . , ∆n be intervals on a straight line such that every two nonadjacent
intervals intersect. Then ∆k and ∆k+1 are disjoint for at most three values of k = 1, . . . , n.
Proof. Denote ∆k = [ak , bk ], 1 ≤ k ≤ n. Let α = max(a1 , . . . , an ) be the rightmost among
the left endpoints of ∆1 , . . . , ∆n , and let β = min(b1 , . . . , bn ) be the leftmost among their right
endpoints. Assume that α = a2 without loss of generality.
If α ≤ β then ai ≤ α ≤ β ≤ bi for all i. Every ∆i contains α, and thus no disjoint pair
(∆i , ∆i+1 ) exists.
22
If β < α then β = bi for some i such that ai < bi = β < α = a2 < b2 , hence ∆2 and ∆i are
disjoint. Now ∆2 intersects all remaining intervals except possibly ∆1 and ∆3 , so ∆2 and ∆i
can be disjoint only if i = 1 or i = 3. Suppose by symmetry that i = 3; then β = b3 . Since
each of the intervals ∆4 , . . . , ∆n intersects ∆2 , we have ai ≤ α ≤ bi for i = 4, . . . , n. Therefore
α ∈ ∆4 ∩ . . . ∩ ∆n , in particular ∆4 ∩ . . . ∩ ∆n 6= ∅. Similarly, ∆5 , . . . , ∆n , ∆1 all intersect ∆3 ,
so that ∆5 ∩ . . . ∩ ∆n ∩ ∆1 6= ∅ as β ∈ ∆5 ∩ . . . ∩ ∆n ∩ ∆1 . This leaves (∆1 , ∆2 ), (∆2 , ∆3 ) and
(∆3 , ∆4 ) as the only candidates for disjoint interval pairs, as desired.
Comment. The problem is a two-dimensional version of the original proposal which is included below.
The extreme shortage of easy and appropriate submissions forced the Problem Selection Committee
to shortlist a simplified variant. The same one-dimensional Claim is used in both versions.
Original proposal. We consider parallelepipeds in three-dimensional space, with edges par-
allel to the coordinate axes and of positive length. Such a parallelepiped will be called a box .
Two boxes intersect if they have a common point in their interior or on their boundary.
Find the largest n for which there exist n boxes B1 , . . . , Bn such that Bi and Bj intersect if
and only if i 6≡ j ± 1 (mod n).
The maximum number of such boxes is 9. Suppose that boxes B1 , . . . , Bn satisfy the con-
dition. Let the closed intervals Ik , Jk and Kk be the projections of box Bk onto the x-, y-
and z-axis, respectively, for 1 ≤ k ≤ n. As before, Bi and Bj are disjoint if and only if their
projections on at least one coordinate axis are disjoint.
We call again two boxes or intervals adjacent if their indices differ by 1 modulo n, and
nonadjacent otherwise.
The adjacent boxes Bi and Bi+1 do not intersect for each i = 1, . . . , n. Hence at least one of
the pairs (Ii , Ii+1 ), (Ji , Ji+1 ) and (Ki , Ki+1 ) is a pair of disjoint intervals. So there are at least
n pairs of disjoint intervals among (Ii , Ii+1 ), (Ji , Ji+1 ), (Ki , Ki+1 ), 1 ≤ i ≤ n.
Next, every two nonadjacent boxes intersect, hence their projections on the three axes
intersect, too. Referring to the Claim in the solution of the two-dimensional version, we
cocnclude that at most 3 pairs among (I1 , I2 ), . . . , (In−1 , In ), (In , I1 ) are disjoint; the same
holds for (J1 , J2 ), . . . , (Jn−1 , Jn ), (Jn , J1 ) and (K1 , K2 ), . . . , (Kn−1 , Kn ), (Kn , K1 ). Consequently
n ≤ 3 + 3 + 3 = 9, as stated.
For n = 9, the desired system of boxes exists. Consider the intervals in the following table:
i Ii Ji Ki
1 [1, 4] [1, 6] [3, 6]
2 [5, 6] [1, 6] [1, 6]
3 [1, 2] [1, 6] [1, 6]
4 [3, 6] [1, 4] [1, 6]
5 [1, 6] [5, 6] [1, 6]
6 [1, 6] [1, 2] [1, 6]
7 [1, 6] [3, 6] [1, 4]
8 [1, 6] [1, 6] [5, 6]
9 [1, 6] [1, 6] [1, 2]
C2. For every positive integer n determine the number of permutations (a1 , a2 , . . . , an ) of the
set {1, 2, . . . , n} with the following property:
Solution. For each n let Fn be the number of permutations of {1, 2, . . . , n} with the required
property; call them nice. For n = 1, 2, 3 every permutation is nice, so F1 = 1, F2 = 2, F3 = 6.
Take an n > 3 and consider any nice permutation (a1 , a2 , . . . , an ) of {1, 2, . . . , n}. Then
n − 1 must be a divisor of the number
2(a1 + a2 + · · · + an−1 ) = 2 (1 + 2 + · · · + n) − an
= n(n + 1) − 2an = (n + 2)(n − 1) + (2 − 2an ).
is divisible by k, for any k ≤ n − 1. And again, any one of the Fn−1 nice permutations
(b1 , b2 , . . . , bn−1 ) of {1, 2, . . . , n−1} gives rise to a nice permutation of {1, 2, . . . , n} whose last
term is 1, namely (b1 +1, b2 +1, . . . , bn−1 +1, 1).
The bijective correspondences established in both cases show that there are Fn−1 nice per-
mutations of {1, 2, . . . , n} with the last term 1 and also Fn−1 nice permutations of {1, 2, . . . , n}
with the last term n. Hence follows the recurrence Fn = 2Fn−1 . With the base value F3 = 6
this gives the outcome formula Fn = 3 · 2n−2 for n ≥ 3.
24
C3. In the coordinate plane consider the set S of all points with integer coordinates. For a
positive integer k, two distinct points A, B ∈ S will be called k-friends if there is a point C ∈ S
such that the area of the triangle ABC is equal to k. A set T ⊂ S will be called a k-clique
if every two points in T are k-friends. Find the least positive integer k for which there exists
a k-clique with more than 200 elements.
Solution. To begin, let us describe those points B ∈ S which are k-friends of the point (0, 0).
By definition, B = (u, v) satisfies this condition if and only if there is a point C = (x, y) ∈ S
such that 21 |uy − vx| = k. (This is a well-known formula expressing the area of triangle ABC
when A is the origin.)
To say that there exist integers x, y for which |uy − vx| = 2k, is equivalent to saying that the
greatest common divisor of u and v is also a divisor of 2k. Summing up, a point B = (u, v) ∈ S
is a k-friend of (0, 0) if and only if gcd(u, v) divides 2k.
Translation by a vector with integer coordinates does not affect k-friendship; if two points are
k-friends, so are their translates. It follows that two points A, B ∈ S, A = (s, t), B = (u, v), are
k-friends if and only if the point (u − s, v − t) is a k-friend of (0, 0); i.e., if gcd(u − s, v − t)|2k.
Let n be a positive integer which does not divide 2k. We claim that a k-clique cannot have
more than n2 elements.
Indeed, all points (x, y) ∈ S can be divided into n2 classes determined by the remainders
that x and y leave in division by n. If a set T has more than n2 elements, some two points
A, B ∈ T , A = (s, t), B = (u, v), necessarily fall into the same class. This means that n|u − s
and n|v − t. Hence n|d where d = gcd(u − s, v − t). And since n does not divide 2k, also d
does not divide 2k. Thus A and B are not k-friends and the set T is not a k-clique.
Now let M(k) be the least positive integer which does not divide 2k. Write M(k) = m for
the moment and consider the set T of all points (x, y) with 0 ≤ x, y < m. There are m2 of
them. If A = (s, t), B = (u, v) are two distinct points in T then both differences |u − s|, |v − t|
are integers less than m and at least one of them is positive. By the definition of m, every
positive integer less than m divides 2k. Therefore u − s (if nonzero) divides 2k, and the same
is true of v − t. So 2k is divisible by gcd(u − s, v − t), meaning that A, B are k-friends. Thus
T is a k-clique.
It follows that the maximum size of a k-clique is M(k)2 , with M(k) defined as above. We
are looking for the minimum k such that M(k)2 > 200.
By the definition of M(k), 2k is divisible by the numbers 1, 2, . . . , M(k)−1, but not by
M(k) itself. If M(k)2 > 200 then M(k) ≥ 15. Trying to hit M(k) = 15 we get a contradiction
immediately (2k would have to be divisible by 3 and 5, but not by 15).
So let us try M(k) = 16. Then 2k is divisible by the numbers 1, 2, . . . , 15, hence also by
their least common multiple L, but not by 16. And since L is not a multiple of 16, we infer
that k = L/2 is the least k with M(k) = 16.
Finally, observe that if M(k) ≥ 17 then 2k must be divisible by the least common multiple
of 1, 2, . . . , 16, which is equal to 2L. Then 2k ≥ 2L, yielding k > L/2.
In conclusion, the least k with the required property is equal to L/2 = 180180.
25
C4. Let n and k be fixed positive integers of the same parity, k ≥ n. We are given 2n lamps
numbered 1 through 2n; each of them can be on or off. At the beginning all lamps are off. We
consider sequences of k steps. At each step one of the lamps is switched (from off to on or from
on to off).
Let N be the number of k-step sequences ending in the state: lamps 1, . . . , n on, lamps
n+1, . . . , 2n off.
Let M be the number of k-step sequences leading to the same state and not touching lamps
n+1, . . . , 2n at all.
Find the ratio N/M.
Solution. A sequence of k switches ending in the state as described in the problem statement
(lamps 1, . . . , n on, lamps n+1, . . . , 2n off ) will be called an admissible process. If, moreover,
the process does not touch the lamps n+1, . . . , 2n, it will be called restricted. So there are N
admissible processes, among which M are restricted.
In every admissible process, restricted or not, each one of the lamps 1, . . . , n goes from off
to on, so it is switched an odd number of times; and each one of the lamps n+1, . . . , 2n goes
from off to off, so it is switched an even number of times.
Notice that M > 0; i.e., restricted admissible processes do exist (it suffices to switch each
one of the lamps 1, . . . , n just once and then choose one of them and switch it k − n times,
which by hypothesis is an even number).
Consider any restricted admissible process p. Take any lamp `, 1 ≤ ` ≤ n, and suppose
that it was switched k` times. As noticed, k` must be odd. Select arbitrarily an even number
of these k` switches and replace each of them by the switch of lamp n+`. This can be done
in 2k` −1 ways (because a k` -element set has 2k` −1 subsets of even cardinality). Notice that
k1 + · · · + kn = k.
These actions are independent, in the sense that the action involving lamp ` does not
affect the action involving any other lamp. So there are 2k1 −1 · 2k2 −1 · · · 2kn −1 = 2k−n ways of
combining these actions. In any of these combinations, each one of the lamps n+1, . . . , 2n gets
switched an even number of times and each one of the lamps 1, . . . , n remains switched an odd
number of times, so the final state is the same as that resulting from the original process p.
This shows that every restricted admissible process p can be modified in 2k−n ways, giving
rise to 2k−n distinct admissible processes (with all lamps allowed).
Now we show that every admissible process q can be achieved in that way. Indeed, it is
enough to replace every switch of a lamp with a label ` > n that occurs in q by the switch of
the corresponding lamp `−n; in the resulting process p the lamps n+1, . . . , 2n are not involved.
Switches of each lamp with a label ` > n had occurred in q an even number of times. So
the performed replacements have affected each lamp with a label ` ≤ n also an even number of
times; hence in the overall effect the final state of each lamp has remained the same. This means
that the resulting process p is admissible—and clearly restricted, as the lamps n+1, . . . , 2n are
not involved in it any more.
If we now take process p and reverse all these replacements, then we obtain process q.
These reversed replacements are nothing else than the modifications described in the foregoing
paragraphs.
Thus there is a one–to–(2k−n ) correspondence between the M restricted admissible processes
and the total of N admissible processes. Therefore N/M = 2k−n .
26
C5. Let S = {x1 , x2 , . . . , xk+`} be a (k + `)-element set of real numbers contained in the
interval [0, 1]; k and ` are positive integers. A k-element subset A ⊂ S is called nice if
X
1 1 X k+`
k xi − xj
≤ .
x ∈A `
2k`
i xj ∈S\A
2 k+`
Prove that the number of nice subsets is at least .
k+` k
1 X 1 X k+`
Solution. For a k-element subset A ⊂ S, let f (A) = xi − xj . Denote = d.
k x ∈A ` 2k`
i xj ∈S\A
By definition a subset A is nice if |f (A)| ≤ d.
To each permutation (y1 , y2, . . . , yk+` ) of the set S = {x1 , x2 , . . . , xk+` } we assign k+` subsets
of S with k elements each, namely Ai = {yi , yi+1, . . . , yi+k−1}, i = 1, 2, . . . , k + `. Indices are
taken modulo k + ` here and henceforth. In other words, if y1 , y2, . . . , yk+` are arranged around
a circle in this order, the sets in question are all possible blocks of k consecutive elements.
Claim. At least two nice sets are assigned to every permutation of S.
Proof. Adjacent sets Ai and Ai+1 differ only by the elements yi and yi+k , i = 1, . . . , k + `. By
the definition of f , and because yi , yi+k ∈ [0, 1],
1 1 1 1
|f (Ai+1 ) − f (Ai )| = + (yi+k − yi) ≤ + = 2d.
k ` k `
Each element yi ∈ S belongs to exactly k of the sets A1 , . . . , Ak+` . Hence in k of the
expressions f (A1 ), . . . , f (Ak+` ) the coefficient of yi is 1/k; in the remaining ` expressions, its
coefficient is −1/`. So the contribution of yi to the sum of all f (Ai ) equals k · 1/k − ` · 1/` = 0.
Since this holds for all i, it follows that f (A1 ) + · · · + f (Ak+` ) = 0.
If f (Ap ) = min f (Ai ), f (Aq ) = max f (Ai ), we obtain in particular f (Ap ) ≤ 0, f (Aq ) ≥ 0.
Let p < q (the case p > q is analogous; and the claim is true for p = q as f (Ai ) = 0 for all i).
We are ready to prove that at least two of the sets A1 , . . . , Ak+` are nice. The interval [−d, d]
has length 2d, and we saw that adjacent numbers in the circular arrangement f (A1 ), . . . , f (Ak+` )
differ by at most 2d. Suppose that f (Ap ) < −d and f (Aq ) > d. Then one of the numbers
f (Ap+1), . . . , f (Aq−1 ) lies in [−d, d], and also one of the numbers f (Aq+1 ), . . . , f (Ap−1 ) lies there.
Consequently, one of the sets Ap+1 , . . . , Aq−1 is nice, as well as one of the sets Aq+1 , . . . , Ap−1.
If −d ≤ f (Ap ) and f (Aq ) ≤ d then Ap and Aq are nice. P
Let now f (Ap ) < −d and f (Aq ) ≤ d. Then f (Ap ) + f (Aq ) < 0, and since f (Ai) = 0,
there is an r 6= q such that f (Ar ) > 0. We have 0 < f (Ar ) ≤ f (Aq ) ≤ d, so the sets f (Ar )
and f (Aq ) are nice. The only case remaining, −d ≤ f (Ap ) and d < f (Aq ), is analogous.
Apply the claim to each of the (k + `)! permutations of S = {x1 , x2 , . . . , xk+` }. This gives
at least 2(k + `)! nice sets, counted with repetitions: each nice set is counted as many times as
there are permutations to which it is assigned.
On the other hand, each k-element set A ⊂ S is assigned to exactly (k+`) k! `! permutations.
Indeed, such a permutation (y1 , y2, . . . , yk+` ) is determined by three independent choices: an in-
dex i ∈ {1, 2, . . . , k + `} such that A = {yi , yi+1, . . . , yi+k−1}, a permutation (yi , yi+1 , . . . , yi+k−1)
of the set A, and a permutation (yi+k , yi+k+1, . . . , yi−1 ) of the set S \ A.
2(k + `)! 2 k+`
In summary, there are at least = nice sets.
(k + `) k! `! k+` k
27
C6. For n ≥ 2, let S1 , S2 , . . . , S2n be 2n subsets of A = {1, 2, 3, . . . , 2n+1 } that satisfy the
following property: There do not exist indices a and b with a < b and elements x, y, z ∈ A with
x < y < z such that y, z ∈ Sa and x, z ∈ Sb . Prove that at least one of the sets S1 , S2 , . . . , S2n
contains no more than 4n elements.
Solution 2. We show that one of the sets Sa has at most 2n + 1 elements. In the sequel | · |
denotes the cardinality of a (finite) set.
Claim. For n ≥ 2, suppose that k subsets S1 , . . . , Sk of {1, 2, . . . , 2n } (not necessarily different)
satisfy the condition of the problem. Then
k
X
(|Si| − n) ≤ (2n − 1)2n−2 .
i=1
Proof. Observe that if the sets Si (1 ≤ i ≤ k) satisfy the condition then so do their arbitrary
subsets Ti (1 ≤ i ≤ k). The condition also holds for the sets t + Si = {t + x | x ∈ Si } where t
is arbitrary.
Note also that a set may occur more than once among
P S1 , . . . , Sk only if its cardinality is
less than 3, in which case its contribution to the sum ki=1 (|Si | − n) is nonpositive (as n ≥ 2).
The proof is by induction on n. In the base case n = 2 we have subsets Si of {1, 2, 3, 4}.
Only the ones of cardinality 3 and 4 need to be considered by the remark above; each one of
28
I = {i | 1 ≤ i ≤ k, |Ui | =
6 0}, J = {1, . . . , k} \ I.
The sets Sj with j ∈ J are all contained in {2n + 1, . . . , 2n+1 }, so the induction hypothesis
applies to their translates −2n + Sj which have the same cardinalities. Consequently, this gives
P n−2
j∈J (|Sj | − n) ≤ (2n − 1)2 , so that
X X
(|Sj | − (n + 1)) ≤ (|Sj | − n) ≤ (2n − 1)2n−2 . (1)
j∈J j∈J
For i ∈ I, denote by vi the least element of Vi . Observe that if Va and Vb intersect, with a < b,
a, b ∈ I, then va is their unique common element. Indeed, let z ∈ Va ∩ Vb ⊆ Sa ∩ Sb and let m
be the least element of Sb . Since b ∈ I, we have m ≤ 2n . By the condition, there is no element
of Sa strictly between m ≤ 2n and z > 2n , which implies z = va .
It follows that if the element vi is removed from each Vi , a family of pairwise disjoint sets
Wi = Vi \ {vi } is obtained,
P i ∈ I (we assume WP As Wi ⊆ {2n + 1, . . . , 2n+1 } for
i = ∅ if Vi = ∅). P
all i, we infer that i∈I |Wi | ≤ 2n . Therefore i∈I (|Vi | − 1) ≤ i∈I |Wi | ≤ 2n .
P On the other hand, the n−2 induction hypothesis applies directly to the sets Ui , i ∈ I, so that
i∈I (|Ui | − n) ≤ (2n − 1)2 . In summary,
X X X
(|Si | − (n + 1)) = (|Ui | − n) + (|Vi | − 1) ≤ (2n − 1)2n−2 + 2n . (2)
i∈I i∈I i∈I
The estimates (1) and (2) are sufficient to complete the inductive step:
k
X X X
(|Si| − (n + 1)) = (|Si | − (n + 1)) + (|Sj | − (n + 1))
i=1 i∈I j∈J
n−2
≤ (2n − 1)2 + 2 + (2n − 1)2n−2 = (2n + 1)2n−1.
n
Comment. It can happen that each set Si has cardinality at least n + 1. Here is an example by the
proposer.
For i = 1, . . . , 2n , let Si = {i + 2k | 0 ≤ k ≤ n}. Then |Si | = n + 1 for all i. Suppose that there
exist a < b and x < y < z such that y, z ∈ Sa and x, z ∈ Sb . Hence z = a + 2k = b + 2l for some k > l.
Since y ∈ Sa and y < z, we have y ≤ a + 2k−1 . So the element x ∈ Sb satisfies
G1. In an acute-angled triangle ABC, point H is the orthocentre and A0 , B0 , C0 are the
midpoints of the sides BC, CA, AB, respectively. Consider three circles passing through
H: ωa around A0 , ωb around B0 and ωc around C0 . The circle ωa intersects the line BC at
A1 and A2 ; ωb intersects CA at B1 and B2 ; ωc intersects AB at C1 and C2 . Show that the
points A1 , A2 , B1 , B2 , C1 , C2 lie on a circle.
Let K be the midpoint of AH and let L be the midpoint of CH. Since A0 and B0 are the
midpoints of BC and CA, we see that A0 LkBH and B0 LkAH. Thus the segments A0 L and B0 L
are perpendicular to AC and BC, hence parallel to OB0 and OA0 , respectively. Consequently
OA0 LB0 is a parallelogram, so that OA0 and B0 L are equal and parallel. Also, the midline B0 L
of triangle AHC is equal and parallel to AK and KH.
It follows that AKA0 O and HA0 OK are parallelograms. The first one gives A0 K = OA = R,
where R is the circumradius of ABC. From the second one we obtain
2(OA20 + A0 H 2 ) = OH 2 + A0 K 2 = OH 2 + R2 . (2)
(In a parallelogram, the sum of squares of the diagonals equals the sum of squares of the sides).
From (1) and (2) we get OA21 = (OH 2 + R2 )/2. By symmetry, the same holds for the
distances OA2 , OB1 , OB2 , OC1 and OC2 . Thus A1 , A2 , B1 , B2 , C1 , C2 all lie on a circle with
centre at O and radius (OH 2 + R2 )/2.
B0
H O
B A1 A0 C
30
Solution 2. We are going to show again that the circumcentre O is equidistant from the six
points in question.
Let A0 be the second intersection point of ωb and ωc . The line B0 C0 , which is the line of
centers of circles ωb and ωc , is a midline in triangle ABC, parallel to BC and perpendicular
to the altitude AH. The points A0 and H are symmetric with respect to the line of centers.
Therefore A0 lies on the line AH.
From the two circles ωb and ωc we obtain AC1 · AC2 = AA0 · AH = AB1 · AB2 . So the
quadrilateral B1 B2 C1 C2 is cyclic. The perpendicular bisectors of the sides B1 B2 and C1 C2
meet at O. Hence O is the circumcentre of B1 B2 C1 C2 and so OB1 = OB2 = OC1 = OC2 .
Analogous arguments yield OA1 = OA2 = OB1 = OB2 and OA1 = OA2 = OC1 = OC2.
Thus A1 , A2 , B1 , B2 , C1 , C2 lie on a circle centred at O.
A
B2
C1
A0 ωb
ωc
C0 B0
H O
C2
B1
B C
A1 A0 A2
Comment. The problem can be solved without much difficulty in many ways by calculation, using
trigonometry, coordinate geometry or complex numbers. As an example we present a short proof using
vectors.
Solution 3. Let again O and R be the circumcentre and circumradius. Consider the vectors
−→ −−→ −→
OA = a, OB = b, OC = c, where a2 = b2 = c2 = R2 .
−−→
It is well known that OH = a + b + c. Accordingly,
−−→ −−→ −−→ b+c 2a + b + c
A0 H = OH − OA0 = (a + b + c) − = ,
2 2
and
2 2
b+c 2a + b + c
OA21 = OA20 + A0 A21 = OA20 + A0 H =2
+
2 2
1 1
= (b2 + 2bc + c2 ) + (4a2 + 4ab + 4ac + b2 + 2bc + c2 ) = 2R2 + (ab + ac + bc);
4 4
here ab, bc, etc. denote dot products of vectors. We get the same for the distances OA2 , OB1,
OB2 , OC1 and OC2.
31
G2. Given trapezoid ABCD with parallel sides AB and CD, assume that there exist points
E on line BC outside segment BC, and F inside segment AD, such that ∠DAE = ∠CBF .
Denote by I the point of intersection of CD and EF , and by J the point of intersection of AB
and EF . Let K be the midpoint of segment EF ; assume it does not lie on line AB.
Prove that I belongs to the circumcircle of ABK if and only if K belongs to the circumcircle
of CDJ.
IF · F E
FJ = .
2IF + F E
Since AEBF is cyclic and AB, CD are parallel, ∠F EC = ∠F AB = 180◦ − ∠CDF . Then
CDF E is also cyclic, yielding ID · IC = IF · IE. It follows that K belongs to the circumcircle
of CDJ if and only if IJ · IK = IF · IE. Expressing IJ = IF + F J, IK = IF + 12 F E, and
IE = IF + F E, we find that K is on the circumcircle of CDJ if and only if
IF · F E
FJ = .
2IF + F E
The conclusion follows.
E
A J
B
I D C
Comment. While the figure shows B inside segment CE, it is possible that C is inside segment BE.
Consequently, I would be inside segment EF and J outside segment EF . The position of point K on
line EF with respect to points I, J may also vary.
Some case may require that an angle ϕ be replaced by 180◦ − ϕ, and in computing distances, a
sum may need to become a difference. All these cases can be covered by the proposed solution if it is
clearly stated that signed distances and angles are used.
32
G3. Let ABCD be a convex quadrilateral and let P and Q be points in ABCD such that
P QDA and QP BC are cyclic quadrilaterals. Suppose that there exists a point E on the line
segment P Q such that ∠P AE = ∠QDE and ∠P BE = ∠QCE. Show that the quadrilateral
ABCD is cyclic.
Solution 1. Let F be the point on the line AD such that EF kP A. By hypothesis, the quadri-
lateral P QDA is cyclic. So if F lies between A and D then ∠EF D = ∠P AD = 180◦ − ∠EQD;
the points F and Q are on distinct sides of the line DE and we infer that EF DQ is a
cyclic quadrilateral. And if D lies between A and F then a similar argument shows that
∠EF D = ∠EQD; but now the points F and Q lie on the same side of DE, so that EDF Q is
a cyclic quadrilateral.
In either case we obtain the equality ∠EF Q = ∠EDQ = ∠P AE which implies that F QkAE.
So the triangles EF Q and P AE are either homothetic or parallel-congruent. More specifically,
triangle EF Q is the image of P AE under the mapping f which carries the points P , E respec-
tively to E, Q and is either a homothety or translation by a vector. Note that f is uniquely
determined by these conditions and the position of the points P , E, Q alone.
Let now G be the point on the line BC such that EGkP B. The same reasoning as above
applies to points B, C in place of A, D, implying that the triangle EGQ is the image of P BE
under the same mapping f . So f sends the four points A, P, B, E respectively to F, E, G, Q.
If P E 6= QE, so that f is a homothety with a centre X, then the lines AF , P E, BG—i.e. the
lines AD, P Q, BC—are concurrent at X. And since P QDA and QP BC are cyclic quadri-
laterals, the equalities XA · XD = XP · XQ = XB · XC hold, showing that the quadrilateral
ABCD is cyclic.
Finally, if P E = QE, so that f is a translation, then ADkP QkBC. Thus P QDA and
QP BC are isosceles trapezoids. Then also ABCD is an isosceles trapezoid, hence a cyclic
quadrilateral.
A
P E Q X
Y
G
B
Solution 2. Here is another way to reach the conclusion that the lines AD, BC and P Q are
either concurrent or parallel. From the cyclic quadrilateral P QDA we get
Comment. After reaching the conclusion that the circles (EDA) and (EBC) are tangent to P Q one
may continue as follows. Denote the circles (P QDA), (EDA), (EBC), (QP BC) by ω1 , ω2 , ω3 , ω4
respectively. Let `ij be the radical axis of the pair (ωi , ωj ) for i < j. As is well-known, the lines
`12 , `13 , `23 concur, possibly at infinity (let this be the meaning of the word concur in this comment).
So do the lines `12 , `14 , `24 . Note however that `23 and `14 both coincide with the line P Q. Hence the
pair `12 , P Q is in both triples; thus the four lines `12 , `13 , `24 and P Q are concurrent.
Similarly, `13 , `14 , `34 concur, `23 , `24 , `34 concur, and since `14 = `23 = P Q, the four lines
`13 , `24 , `34 and P Q are concurrent. The lines `13 and `24 are present in both quadruples, there-
fore all the lines `ij are concurrent. Hence the result.
34
G4. In an acute triangle ABC segments BE and CF are altitudes. Two circles passing
through the points A and F are tangent to the line BC at the points P and Q so that B lies
between C and Q. Prove that the lines P E and QF intersect on the circumcircle of triangle
AEF .
Solution 1. To approach the desired result we need some information about the slopes of the
lines P E and QF ; this information is provided by formulas (1) and (2) which we derive below.
The tangents BP and BQ to the two circles passing through A and F are equal, as
BP 2 = BA · BF = BQ2 . Consider the altitude AD of triangle ABC and its orthocentre H.
From the cyclic quadrilaterals CDF A and CDHE we get BA · BF = BC · BD = BE · BH.
Thus BP 2 = BE · BH, or BP/BH = BE/BP , implying that the triangles BP H and BEP
are similar. Hence
∠BP E = ∠BHP . (1)
The point P lies between D and C; this follows from the equality BP 2 = BC · BD. In view
of this equality, and because BP = BQ,
Also AD · DH = BD · DC, as is seen from the similar triangles BDH and ADC. Combining
these equalities we obtain AD · DH = DP · DQ. Therefore DH/DP = DQ/DA, showing that
the triangles HDP and QDA are similar. Hence ∠HP D = ∠QAD, which can be rewritten as
∠BP H = ∠BAD + ∠BAQ. And since BQ is tangent to the circumcircle of triangle F AQ,
Thus ∠BP E + ∠BQF < 180◦ , which means that the rays P E and QF meet. Let S be the
point of intersection. Then ∠P SQ = 180◦ − (∠BP E + ∠BQF ) = ∠CAB = ∠EAF .
If S lies between P and E then ∠P SQ = 180◦ − ∠ESF ; and if E lies between P and S
then ∠P SQ = ∠ESF . In either case the equality ∠P SQ = ∠EAF which we have obtained
means that S lies on the circumcircle of triangle AEF .
A
E
F
H
Q B D P C
35
Solution 2. Let H be the orthocentre of triangle ABC and let ω be the circle with diameter
AH, passing through E and F . Introduce the points of intersection of ω with the following lines
emanating from P : P A ∩ ω = {A, U}, P H ∩ ω = {H, V }, P E ∩ ω = {E, S}. The altitudes of
triangle AHP are contained in the lines AV , HU, BC, meeting at its orthocentre Q0 .
By Pascal’s theorem applied to the (tied) hexagon AESF HV , the points AE ∩ F H = C,
ES ∩ HV = P and SF ∩ V A are collinear, so F S passes through Q0 .
Denote by ω1 and ω2 the circles with diameters BC and P Q0 , respectively. Let D be the
foot of the altitude from A in triangle ABC. Suppose that AD meets the circles ω1 and ω2 at
the respective points K and L.
Since H is the orthocentre of ABC, the triangles BDH and ADC are similar, and so
DA · DH = DB · DC = DK 2 ; the last equality holds because BKC is a right triangle. Since
H is the orthocentre also in triangle AQ0 P , we analogously have DL2 = DA · DH. Therefore
DK = DL and K = L.
Also, BD · BC = BA · BF , from the similar triangles ABD, CBF . In the right triangle
BKC we have BK 2 = BD · BC. Hence, and because BA · BF = BP 2 = BQ2 (by the defini-
tion of P and Q in the problem statement), we obtain BK = BP = BQ. It follows that B is
the centre of ω2 and hence Q0 = Q. So the lines P E and QF meet at the point S lying on the
circumcircle of triangle AEF .
A
ω
V S
ω2
K E
F
ω1
H U
T
Q0 B D P C
Comment 1. If T is the point defined by P F ∩ ω = {F, T }, Pascal’s theorem for the hexagon
AF T EHV will analogously lead to the conclusion that the line ET goes through Q0 . In other words,
the lines P F and QE also concur on ω.
Comment 2. As is known from algebraic geometry, the points of the circle ω form a commutative
groups with the operation defined as follows. Choose any point 0 ∈ ω (to be the neutral element of
the group) and a line ` exterior to the circle. For X, Y ∈ ω, draw the line from the point XY ∩ `
through 0 to its second intersection with ω and define this point to be X + Y .
In our solution we have chosen H to be the neutral element in this group and line BC to be `. The
fact that the lines AV , HU , ET , F S are concurrent can be deduced from the identities A + A = 0,
F = E + A, V = U + A = S + E = T + F .
G5. Let k and n be integers with 0 ≤ k ≤ n − 2. Consider a set L of n lines in the plane such
that no two of them are parallel and no three have a common point. Denote by I the set of
intersection points of lines in L. Let O be a point in the plane not lying on any line of L.
A point X ∈ I is colored red if the open line segment OX intersects at most k lines in L.
Prove that I contains at least 21 (k + 1)(k + 2) red points.
Solution. There are at least 21 (k + 1)(k + 2) points in the intersection set I in view of the
condition n ≥ k + 2.
For each point P ∈ I, define its order as the number of lines that intersect the open line
segment OP . By definition, P is red if its order is at most k. Note that there is always at
least one point X ∈ I of order 0. Indeed, the lines in L divide the plane into regions, bounded
or not, and O belongs to one of them. Clearly any corner of this region is a point of I with
order 0.
Claim. Suppose that two points P, Q ∈ I lie on the same line of L, and no other line of L
intersects the open line segment P Q. Then the orders of P and Q differ by at most 1.
Proof. Let P and Q have orders p and q, respectively, with p ≥ q. Consider triangle OP Q.
Now p equals the number of lines in L that intersect the interior of side OP . None of these
lines intersects the interior of side P Q, and at most one can pass through Q. All remaining
lines must intersect the interior of side OQ, implying that q ≥ p − 1. The conclusion follows.
We prove the main result by induction on k. The base k = 0 is clear since there is a point
of order 0 which is red. Assuming the statement true for k − 1, we pass on to the inductive
step. Select a point P ∈ I of order 0, and consider one of the lines ` ∈ L that pass through P .
There are n − 1 intersection points on `, one of which is P . Out of the remaining n − 2 points,
the k closest to P have orders not exceeding k by the Claim. It follows that there are at least
k + 1 red points on `.
Let us now consider the situation with ` removed (together with all intersection points
it contains). By hypothesis of induction, there are at least 12 k(k + 1) points of order not
exceeding k − 1 in the resulting configuration. Restoring ` back produces at most one new
intersection point on each line segment joining any of these points to O, so their order is at
most k in the original configuration. The total number of points with order not exceeding k is
therefore at least (k + 1) + 21 k(k + 1) = 12 (k + 1)(k + 2). This completes the proof.
Comment. The steps of the proof can be performed in reverse order to obtain a configuration of n
lines such that equality holds simultaneously for all 0 ≤ k ≤ n − 2. Such a set of lines is illustrated in
the Figure.
O
0
1 1
2
2 3 3 2
3 4 3
4 4
4 4
37
G6. There is given a convex quadrilateral ABCD. Prove that there exists a point P inside
the quadrilateral such that
∠P AB + ∠P DC = ∠P BC + ∠P AD = ∠P CD + ∠P BA = ∠P DA + ∠P CB = 90◦ (1)
Solution 1. For a point P in ABCD which satisfies (1), let K, L, M, N be the feet of per-
pendiculars from P to lines AB, BC, CD, DA, respectively. Note that K, L, M, N are interior
to the sides as all angles in (1) are acute. The cyclic quadrilaterals AKP N and DNP M give
∠P AB + ∠P DC = ∠P NK + ∠P NM = ∠KNM.
N M
VC VA
P
UA UC
OA OC
A
R C
K L
B
(ii) Suppose that AC and BD are perpendicular and meet at R. If ABCD is a rhombus, P
can be chosen to be its centre. So assume that ABCD is not a rhombus, and let BR < DR
without loss of generality.
Denote by UA and UC the circumcentres of the triangles ABD and CDB, respectively. Let
AVA and CVC be the diameters through A and C of the two circumcircles. Since AR is an
altitude in triangle ADB, lines AC and AVA are isogonal conjugates, i. e. ∠DAVA = ∠BAC.
Now BR < DR implies that ray AUA lies in ∠DAC. Similarly, ray CUC lies in ∠DCA. Both
diameters AVA and CVC intersect BD as the angles at B and D of both triangles are acute.
Also UA UC is parallel to AC as it is the perpendicular bisector of BD. Hence VA VC is parallel
to AC, too. We infer that AVA and CVC intersect at a point P inside triangle ACD, hence
inside ABCD.
38
N M
VC VA
P
UA UC
OA OC
A
R C
K L
B
Solution 2. For a point P distinct from A, B, C, D, let circles (AP D) and (BP C) inter-
sect again at Q (Q = P if the circles are tangent). Next, let circles (AQB) and (CQD)
intersect again at R. We show that if P lies in ABCD and satisfies (1) then AC and BD
intersect at R and are perpendicular; the converse is also true. It is convenient to use directed
angles. Let ](UV, XY ) denote the angle of counterclockwise rotation that makes line UV
parallel to line XY . Recall that four noncollinear points U, V, X, Y are concyclic if and only if
](UX, V X) = ](UY, V Y ).
The definitions of points P , Q and R imply
](AR, BR) = ](AQ, BQ) = ](AQ, P Q) + ](P Q, BQ) = ](AD, P D) + ](P C, BC),
](CR, DR) = ](CQ, DQ) = ](CQ, P Q) + ](P Q, DQ) = ](CB, P B) + ](P A, DA),
](BR, CR) = ](BR, RQ) + ](RQ, CR) = ](BA, AQ) + ](DQ, CD)
= ](BA, AP ) + ](AP, AQ) + ](DQ, DP ) + ](DP, CD)
= ](BA, AP ) + ](DP, CD).
Observe that the whole construction is reversible. One may start with point R, define Q as the
second intersection of circles (ARB) and (CRD), and then define P as the second intersection
of circles (AQD) and (BQC). The equalities above will still hold true.
39
](AD, P D) = ∠P DA, ](P C, BC) = ∠P CB, ](CB, P B) = ∠P BC, ](P A, DA) = ∠P AD,
](BA, AP ) = ∠P AB, ](DP, CD) = ∠P DC.
(i) Suppose that P lies in ABCD and satisfies (1). Then ](AR, BR) = ∠P DA + ∠P CB = 90◦
and similarly ](BR, CR) = ](CR, DR) = 90◦ . It follows that R is the common point of
lines AC and BD, and that these lines are perpendicular.
(ii) Suppose that AC and BD are perpendicular and intersect at R. We show that the point P
defined by the reverse construction (starting with R and ending with P ) lies in ABCD. This
is enough to finish the solution, because then the angle equalities above will imply (1).
One can assume that Q, the second common point of circles (ABR) and (CDR), lies
in ∠ARD. Then in fact Q lies in triangle ADR as angles AQR and DQR are obtuse. Hence
∠AQD is obtuse, too, so that B and C are outside circle (ADQ) (∠ABD and ∠ACD are
acute).
Now ∠CAB+∠CDB = ∠BQR+∠CQR = ∠CQB implies ∠CAB < ∠CQB and ∠CDB <
∠CQB. Hence A and D are outside circle (BCQ). In conclusion, the second common point P
of circles (ADQ) and (BCQ) lies on their arcs ADQ and BCQ.
We can assume that P lies in ∠CQD. Since
point P lies in triangle CDQ, and hence in ABCD. The proof is complete.
D
Q
A R
C
B
40
G7. Let ABCD be a convex quadrilateral with AB 6= BC. Denote by ω1 and ω2 the incircles
of triangles ABC and ADC. Suppose that there exists a circle ω inscribed in angle ABC,
tangent to the extensions of line segments AD and CD. Prove that the common external
tangents of ω1 and ω2 intersect on ω.
A0
C0
P0
ω1
A
P
Q
ω2 C
Q0
ω
T
M
K L
For brevity, in the sequel we write “excircle AC” for the excircle of a triangle with side AC
which is tangent to line segment AC and the extensions of the other two sides.
Lemma 2. The incircle of triangle ABC is tangent to its side AC at P . Let P P 0 be the diameter
of the incircle through P , and let line BP 0 intersect AC at Q. Then Q is the point of tangency
of side AC and excircle AC.
Proof. Let the tangent at P 0 to the incircle ω1 meet BA and BC at A0 and C 0 . Now ω1 is the
excircle A0 C 0 of triangle A0 BC 0 , and it touches side A0 C 0 at P 0 . Since A0 C 0 k AC, the homothety
with centre B and ratio BQ/BP 0 takes ω1 to the excircle AC of triangle ABC. Because this
homothety takes P 0 to Q, the lemma follows.
41
Recall also that if the incircle of a triangle touches its side AC at P , then the tangency
point Q of the same side and excircle AC is the unique point on line segment AC such that
AP = CQ.
We pass on to the main proof. Let ω1 and ω2 touch AC at P and Q, respectively; then
AP = (AC + AB − BC)/2, CQ = (CA + CD − AD)/2. Since AB − BC = CD − AD
by Lemma 1, we obtain AP = CQ. It follows that in triangle ABC side AC and excircle AC
are tangent at Q. Likewise, in triangle ADC side AC and excircle AC are tangent at P . Note
that P 6= Q as AB 6= BC.
Let P P 0 and QQ0 be the diameters perpendicular to AC of ω1 and ω2 , respectively. Then
Lemma 2 shows that points B, P 0 and Q are collinear, and so are points D, Q0 and P .
Consider the diameter of ω perpendicular to AC and denote by T its endpoint that is closer
to AC. The homothety with centre B and ratio BT /BP 0 takes ω1 to ω. Hence B, P 0 and T
are collinear. Similarly, D, Q0 and T are collinear since the homothety with centre D and
ratio −DT /DQ0 takes ω2 to ω.
We infer that points T, P 0 and Q are collinear, as well as T, Q0 and P . Since P P 0 k QQ0 , line
segments P P 0 and QQ0 are then homothetic with centre T . The same holds true for circles ω1
and ω2 because they have P P 0 and QQ0 as diameters. Moreover, it is immediate that T lies on
the same side of line P P 0 as Q and Q0 , hence the ratio of homothety is positive. In particular
ω1 and ω2 are not congruent.
In summary, T is the centre of a homothety with positive ratio that takes circle ω1 to
circle ω2 . This completes the solution, since the only point with the mentioned property is the
intersection of the the common external tangents of ω1 and ω2 .
42
Number Theory
N1. Let n be a positive integer and let p be a prime number. Prove that if a, b, c are integers
(not necessarily positive) satisfying the equations
an + pb = bn + pc = cn + pa,
then a = b = c.
Solution 1. If two of a, b, c are equal, it is immediate that all the three are equal. So we
may assume that a 6= b 6= c 6= a. Subtracting the equations we get an − bn = −p(b − c) and two
cyclic copies of this equation, which upon multiplication yield
an − bn bn − cn cn − an
· · = −p3 . (1)
a−b b−c c−a
If n is odd then the differences an − bn and a − b have the same sign and the product on the
left is positive, while −p3 is negative. So n must be even.
Let d be the greatest common divisor of the three differences a − b, b − c, c − a, so that
a − b = du, b − c = dv, c − a = dw; gcd(u, v, w) = 1, u + v + w = 0.
From an − bn = −p(b − c) we see that (a − b)|p(b − c), i.e., u|pv; and cyclically v|pw, w|pu.
As gcd(u, v, w) = 1 and u + v + w = 0, at most one of u, v, w can be divisible by p. Sup-
posing that the prime p does not divide any one of them, we get u|v, v|w, w|u, whence
|u| = |v| = |w| = 1; but this quarrels with u + v + w = 0.
Thus p must divide exactly one of these numbers. Let e.g. p|u and write u = pu1 . Now
we obtain, similarly as before, u1 |v, v|w, w|u1 so that |u1 | = |v| = |w| = 1. The equation
pu1 + v + w = 0 forces that the prime p must be even; i.e. p = 2. Hence v + w = −2u1 = ±2,
implying v = w (= ±1) and u = −2v. Consequently a − b = −2(b − c).
Knowing that n is even, say n = 2k, we rewrite the equation an − bn = −p(b − c) with p = 2
in the form
(ak + bk )(ak − bk ) = −2(b − c) = a − b.
The second factor on the left is divisible by a − b, so the first factor (ak + bk ) must be ±1.
Then exactly one of a and b must be odd; yet a − b = −2(b − c) is even. Contradiction ends
the proof.
Solution 2. The beginning is as in the first solution. Assuming that a, b, c are not all equal,
hence are all distinct, we derive equation (1) with the conclusion that n is even. Write n = 2k.
Suppose that p is odd. Then the integer
an − bn
= an−1 + an−2 b + · · · + bn−1 ,
a−b
44
which is a factor in (1), must be odd as well. This sum of n = 2k summands is odd only if
a and b have different parities. The same conclusion holding for b, c and for c, a, we get that
a, b, c, a alternate in their parities, which is clearly impossible.
Thus p = 2. The original system shows that a, b, c must be of the same parity. So we may
divide (1) by p3 , i.e. 23 , to obtain the following product of six integer factors:
ak + bk ak − bk bk + ck bk − ck ck + ak ck − ak
· · · · · = −1. (2)
2 a−b 2 b−c 2 c−a
Each one of the factors must be equal to ±1. In particular, ak + bk = ±2. If k is even, this
becomes ak + bk = 2 and yields |a| = |b| = 1, whence ak − bk = 0, contradicting (2).
Let now k be odd. Then the sum ak + bk , with value ±2, has a + b as a factor. Since a and b
are of the same parity, this means that a + b = ±2; and cyclically, b + c = ±2, c + a = ±2. In
some two of these equations the signs must coincide, hence some two of a, b, c are equal. This
is the desired contradiction.
Comment. Having arrived at the equation (1) one is tempted to write down all possible decomposi-
tions of −p3 (cube of a prime) into a product of three integers. This leads to cumbersome examination
of many cases, some of which are unpleasant to handle. One may do that just for p = 2, having earlier
in some way eliminated odd primes from consideration.
However, the second solution shows that the condition of p being a prime is far too strong. What
is actually being used in that solution, is that p is either a positive odd integer or p = 2.
45
N2. Let a1 , a2 , . . . , an be distinct positive integers, n ≥ 3. Prove that there exist distinct
indices i and j such that ai + aj does not divide any of the numbers 3a1 , 3a2 , . . . , 3an .
Solution. Without loss of generality, let 0 < a1 < a2 < · · · < an . One can also assume that
a1 , a2 , . . . , an are coprime. Otherwise division by their greatest common divisor reduces the
question to the new sequence whose terms are coprime integers.
Suppose that the claim is false. Then for each i < n there exists a j such that an + ai
divides 3aj . If an + ai is not divisible by 3 then an + ai divides aj which is impossible as
0 < aj ≤ an < an +ai . Thus an +ai is a multiple of 3 for i = 1, . . . , n−1, so that a1 , a2 , . . . , an−1
are all congruent (to −an ) modulo 3.
Now an is not divisible by 3 or else so would be all remaining ai ’s, meaning that a1 , a2 , . . . , an
are not coprime. Hence an ≡ r (mod 3) where r ∈ {1, 2}, and ai ≡ 3 − r (mod 3) for all
i = 1, . . . , n − 1.
Consider a sum an−1 + ai where 1 ≤ i ≤ n − 2. There is at least one such sum as n ≥ 3. Let
j be an index such that an−1 + ai divides 3aj . Observe that an−1 + ai is not divisible by 3 since
an−1 + ai ≡ 2ai 6≡ 0 (mod 3). It follows that an−1 + ai divides aj , in particular an−1 + ai ≤ aj .
Hence an−1 < aj ≤ an , implying j = n. So an is divisible by all sums an−1 + ai , 1 ≤ i ≤ n − 2.
In particular an−1 + ai ≤ an for i = 1, . . . , n − 2.
Let j be such that an + an−1 divides 3aj . If j ≤ n − 2 then an + an−1 ≤ 3aj < aj + 2an−1 .
This yields an < an−1 + aj ; however an−1 + aj ≤ an for j ≤ n − 2. Therefore j = n − 1 or j = n.
For j = n − 1 we obtain 3an−1 = k(an + an−1 ) with k an integer, and it is straightforward
that k = 1 (k ≤ 0 and k ≥ 3 contradict 0 < an−1 < an ; k = 2 leads to an−1 = 2an > an−1 ).
Thus 3an−1 = an + an−1 , i. e. an = 2an−1 .
Similarly, if j = n then 3an = k(an + an−1 ) for some integer k, and only k = 2 is possible.
Hence an = 2an−1 holds true in both cases remaining, j = n − 1 and j = n.
Now an = 2an−1 implies that the sum an−1 + a1 is strictly between an /2 and an . But an−1
and a1 are distinct as n ≥ 3, so it follows from the above that an−1 + a1 divides an . This
provides the desired contradiction.
46
N3. Let a0 , a1 , a2 , . . . be a sequence of positive integers such that the greatest common divisor
of any two consecutive terms is greater than the preceding term; in symbols, gcd(ai , ai+1 ) > ai−1 .
Prove that an ≥ 2n for all n ≥ 0.
Solution. Since ai ≥ gcd(ai , ai+1 ) > ai−1 , the sequence is strictly increasing. In particular
a0 ≥ 1, a1 ≥ 2. For each i ≥ 1 we also have ai+1 − ai ≥ gcd(ai , ai+1 ) > ai−1 , and consequently
ai+1 ≥ ai + ai−1 + 1. Hence a2 ≥ 4 and a3 ≥ 7. The equality a3 = 7 would force equalities
in the previous estimates, leading to gcd(a2 , a3 ) = gcd(4, 7) > a1 = 2, which is false. Thus
a3 ≥ 8; the result is valid for n = 0, 1, 2, 3. These are the base cases for a proof by induction.
Take an n ≥ 3 and assume that ai ≥ 2i for i = 0, 1, . . . , n. We must show that an+1 ≥ 2n+1 .
Let gcd(an , an+1 ) = d. We know that d > an−1 . The induction claim is reached immediately
in the following cases:
The only remaining possibility is that an = 2d and an+1 = 3d, which we assume for the
sequel. So an+1 = 32 an .
Let now gcd(an−1 , an ) = d0 ; then d0 > an−2 . Write an = md0 (m an integer). Keeping
in mind that d0 ≤ an−1 < d and an = 2d, we get that m ≥ 3. Also an−1 < d = 12 md0 ,
an+1 = 32 md0 . Again we single out the cases which imply the induction claim immediately:
So we are left with the case m = 5, which means that an = 5d0, an+1 = 15 2
d0 , an−1 < d = 52 d0 .
The last relation implies that an−1 is either d0 or 2d0 . Anyway, an−1 |2d0 .
The same pattern repeats once more. We denote gcd(an−2 , an−1 ) = d00 ; then d00 > an−3 .
Because d00 is a divisor of an−1 , hence also of 2d0 , we may write 2d0 = m0 d00 (m0 an integer).
Since d00 ≤ an−2 < d0 , we get m0 ≥ 3. Also, an−2 < d0 = 12 m0 d00 , an+1 = 15 2
d0 = 15
4
m0 d00 . As
before, we consider the cases:
15 0 00 75 00 75 75
if m0 ≥ 5 then an+1 = 4
md ≥ 4
d > a
4 n−3
≥ 4
·2n−3 > 2n+1 ;
1
if 3 ≤ m0 ≤ 4 then an−2 < 2
· 4d00 , and hence an−2 = d00 ,
15 0 15 45 n−2
an+1 = 4
m an−2 ≥ 4
·3an−2 ≥ 4
·2 > 2n+1 .
Both of them have produced the induction claim. But now there are no cases left. Induction
is complete; the inequality an ≥ 2n holds for all n.
47
This prepares ground for a proof of the required result by induction on n. The base case
n−1
n = 1 is obvious. Assume the assertion is true for n − 1 and pass to n, denoting ak = 2 k −1 ,
n
bm = 2 m−1 . The induction hypothesis is that all the numbers ak (0 ≤ k < 2n−2 ) are distinct
(mod 2n−1 ); the claim is that all the numbers bm (0 ≤ m < 2n−1 ) are distinct (mod 2n ).
The congruence relations (1) are restated as
b2k ≡ (−1)k ak ≡ −b2k+1 (mod 2n ). (2)
Shifting the exponent in the first relation of (1) from n to n − 1 we also have the congruence
a2i+1 ≡ −a2i (mod 2n−1 ). We hence conclude:
If, for some j, k < 2n−2 , ak ≡ −aj (mod 2n−1 ), then {j, k} = {2i, 2i+1} for some i. (3)
This is so because in the sequence (ak : k < 2n−2 ) each term aj is complemented to 0 (mod 2n−1 )
by only one other term ak , according to the induction hypothesis.
From (2) we see that b4i ≡ a2i and b4i+3 ≡ a2i+1 (mod 2n ). Let
M = {m : 0 ≤ m < 2n−1 , m ≡ 0 or 3 (mod 4)}, L = {l : 0 ≤ l < 2n−1 , l ≡ 1 or 2 (mod 4)}.
The last two congruences take on the unified form
bm ≡ abm/2c (mod 2n ) for all m ∈ M. (4)
Thus all the numbers bm for m ∈ M are distinct (mod 2n ) because so are the numbers ak (they
are distinct (mod 2n−1 ), hence also (mod 2n )).
Every l ∈ L is paired with a unique m ∈ M into a pair of the form {2k, 2k+1}. So (2) implies
that also all the bl for l ∈ L are distinct (mod 2n ). It remains to eliminate the possibility that
bm ≡ bl (mod 2n ) for some m ∈ M , l ∈ L.
Suppose that such a situation occurs. Let m0 ∈ M be such that {m0 , l} is a pair of the form
{2k, 2k+1}, so that (see (2)) bm0 ≡ −bl (mod 2n ). Hence bm0 ≡ −bm (mod 2n ). Since both
m0 and m are in M, we have by (4) bm0 ≡ aj , bm ≡ ak (mod 2n ) for j = bm0 /2c, k = bm/2c.
Then aj ≡ −ak (mod 2n ). Thus, according to (3), j = 2i, k = 2i + 1 for some i (or vice
n−1 n−1
versa). The equality a2i+1 ≡ −a2i (mod 2n ) now means that 2 2i −1 + 2 2i+1−1 ≡ 0 (mod 2n ).
n−1
However, the sum on the left is equal to 22i+1 . A number of this form cannot be divisible
by 2n . This is a contradiction which concludes the induction step and proves the result.
48
Given this, the conclusion is immediate: the first formula of (5) together with the induction
hypothesis tells us that (b0 , b4 , b8 , . . . , b2N −4 ) (mod N) is a permutation of (1, 3, 5, . . . , N−1).
Then the second formula of (5) shows that (b2 , b6 , b10 , . . . , b2N −2 ) (mod N) is exactly the same
permutation; moreover, this formula distinguishes (mod 2N) each b4i from b4i+2 .
Consequently, these two sequences combined represent (mod 2N) a permutation of the
sequence (1, 3, 5, . . . , N−1, N+1, N+3, N+5, . . . , N+N−1), and this is precisely the induction
claim.
Now we prove formulas (5); we begin with the second one. Since bm+1 = bm · 2Nm+1 −m−1
,
2N − 4i − 1 2N − 4i − 2 2N − 4i − 1 N − 2i − 1
b4i+2 = b4i · · = b4i · · .
4i + 1 4i + 2 4i + 1 2i + 1
The desired congruence b4i+2 ≡ b4i + N may be multiplied by the odd number (4i + 1)(2i + 1),
giving rise to a chain of successively equivalent congruences:
and the last one is satisfied, as b4i is odd. This settles the second relation in (5).
The first one is proved by induction on i. It holds for i = 0. Assume b4i ≡ a2i (mod 2N)
and consider i + 1:
2N − 4i − 3 2N − 4i − 4 N − 2i − 1 N − 2i − 2
b4i+4 = b4i+2 · · ; a2i+2 = a2i · · .
4i + 3 4i + 4 2i + 1 2i + 2
Both expressions have the fraction N 2i+2
−2i−2
as the last factor. Since 2i + 2 < N = 2n−1 , this
fraction reduces to `/m with ` and m odd. In showing that b4i+4 ≡ a2i+2 (mod 2N), we may
ignore this common factor `/m. Clearing other odd denominators reduces the claim to
By the inductive assumption (saying that b4i ≡ a2i (mod 2N)) and by the second relation of (5),
this is equivalent to
a congruence which we have already met in the preceding proof a few lines above. This com-
pletes induction (on i) and the proof of (5), hence also the whole solution.
Comment. One can avoid the words congruent modulo in the problem statement by rephrasing the
assertion into: Show that these numbers leave distinct remainders in division by 2n .
49
N5. For every n ∈ N let d(n) denote the number of (positive) divisors of n. Find all func-
tions f : N → N with the following properties:
Hence all inequalities bi ≤ pai i − 1 must be equalities, i = 1, . . . , k, implying that (1) holds true.
The proof is complete.
50
2
N6. Prove that there exist
√ infinitely many positive integers n such that n + 1 has a prime
divisor greater than 2n + 2n.
Solution. Let p ≡ 1 (mod 8) be a prime. The congruence x2 ≡ −1 (mod p) has two solutions
2
√ If n is the smaller one of them then p divides n +1 and n ≤ (p−1)/2.
in [1, p−1] whose sum is p.
We show that p > 2n + 10n.
Let n = (p − 1)/2 − ` where ` ≥ 0. Then n2 ≡ −1 (mod p) gives
2
p−1
−` ≡ −1 (mod p) or (2` + 1)2 + 4 ≡ 0 (mod p).
2
Problem Shortlist
with Solutions
gratefully received
Contents
Problem Shortlist 4
Algebra 12
Combinatorics 26
Geometry 47
Number Theory 69
Algebra Problem Shortlist 50th IMO 2009
Algebra
A1 CZE (Czech Republic)
Find the largest possible integer k, such that the following statement is true:
Let 2009 arbitrary non-degenerated triangles be given. In every triangle the three sides are
colored, such that one is blue, one is red and one is white. Now, for every color separately, let
us sort the lengths of the sides. We obtain
Then there exist k indices j such that we can form a non-degenerated triangle with side lengths
bj , rj , wj .
A2 EST (Estonia)
1 1 1
Let a, b, c be positive real numbers such that + + = a + b + c. Prove that
a b c
1 1 1 3
2
+ 2
+ 2
≤ .
(2a + b + c) (2b + c + a) (2c + a + b) 16
A3 FRA (France)
Determine all functions f from the set of positive integers into the set of positive integers such
that for all x and y there exists a non degenerated triangle with sides of lengths
A4 BLR (Belarus)
Let a, b, c be positive real numbers such that ab + bc + ca ≤ 3abc. Prove that
√ √ √
r r r
a2 + b 2 b2 + c 2 c 2 + a2 √
+ + +3≤ 2 a+b+ b+c+ c+a .
a+b b+c c+a
A5 BLR (Belarus)
Let f be any function that maps the set of real numbers into the set of real numbers. Prove
that there exist real numbers x and y such that
4
50th IMO 2009 Problem Shortlist Algebra
A7 JPN (Japan)
Find all functions f from the set of real numbers into the set of real numbers which satisfy for
all real x, y the identity
5
Combinatorics Problem Shortlist 50th IMO 2009
Combinatorics
C1 NZL (New Zealand)
Consider 2009 cards, each having one gold side and one black side, lying in parallel on a long
table. Initially all cards show their gold sides. Two players, standing by the same long side of
the table, play a game with alternating moves. Each move consists of choosing a block of 50
consecutive cards, the leftmost of which is showing gold, and turning them all over, so those
which showed gold now show black and vice versa. The last player who can make a legal move
wins.
(a) Does the game necessarily end?
(b) Does there exist a winning strategy for the starting player?
C2 ROU (Romania)
For any integer n ≥ 2, let N (n) be the maximal number of triples (ai , bi , ci ), i = 1, . . . , N (n),
consisting of nonnegative integers ai , bi and ci such that the following two conditions are satis-
fied:
(1) ai + bi + ci = n for all i = 1, . . . , N (n),
(2) If i 6= j, then ai 6= aj , bi 6= bj and ci 6= cj .
Determine N (n) for all n ≥ 2.
Comment. The original problem was formulated for m-tuples instead for triples. The numbers
N (m, n) are then defined similarly to N (n) in the case m = 3. The numbers N (3, n) and
N (n, n) should be determined. The case m = 3 is the same as in the present problem. The
upper bound for N (n, n) can be proved by a simple generalization. The construction of a set
of triples attaining the bound can be easily done by induction from n to n + 2.
a0 = b0 = 1, a1 = b1 = 7,
(
2ai−1 + 3ai , if εi = 0,
ai+1 = for each i = 1, . . . , n − 1,
3ai−1 + ai , if εi = 1,
(
2bi−1 + 3bi , if εn−i = 0,
bi+1 = for each i = 1, . . . , n − 1.
3bi−1 + bi , if εn−i = 1,
Prove that an = bn .
C4 NLD (Netherlands)
For an integer m ≥ 1, we consider partitions of a 2m × 2m chessboard into rectangles consisting
of cells of the chessboard, in which each of the 2m cells along one diagonal forms a separate
rectangle of side length 1. Determine the smallest possible sum of rectangle perimeters in such
a partition.
6
50th IMO 2009 Problem Shortlist Combinatorics
C5 NLD (Netherlands)
Five identical empty buckets of 2-liter capacity stand at the vertices of a regular pentagon.
Cinderella and her wicked Stepmother go through a sequence of rounds: At the beginning of
every round, the Stepmother takes one liter of water from the nearby river and distributes it
arbitrarily over the five buckets. Then Cinderella chooses a pair of neighboring buckets, empties
them into the river, and puts them back. Then the next round begins. The Stepmother’s goal
is to make one of these buckets overflow. Cinderella’s goal is to prevent this. Can the wicked
Stepmother enforce a bucket overflow?
C6 BGR (Bulgaria)
On a 999 × 999 board a limp rook can move in the following way: From any square it can move
to any of its adjacent squares, i.e. a square having a common side with it, and every move
must be a turn, i.e. the directions of any two consecutive moves must be perpendicular. A non-
intersecting route of the limp rook consists of a sequence of pairwise different squares that the
limp rook can visit in that order by an admissible sequence of moves. Such a non-intersecting
route is called cyclic, if the limp rook can, after reaching the last square of the route, move
directly to the first square of the route and start over.
How many squares does the longest possible cyclic, non-intersecting route of a limp rook
visit?
Variant 1. A grasshopper jumps along the real axis. He starts at point 0 and makes 2009
jumps to the right with lengths 1, 2, . . . , 2009 in an arbitrary order. Let M be a set of 2008
positive integers less than 1005 · 2009. Prove that the grasshopper can arrange his jumps in
such a way that he never lands on a point from M .
Variant 2. Let n be a nonnegative integer. A grasshopper jumps along the real axis. He starts
at point 0 and makes n + 1 jumps to the right with pairwise different positive integral lengths
a1 , a2 , . . . , an+1 in an arbitrary order. Let M be a set of n positive integers in the interval (0, s),
where s = a1 + a2 + · · · + an+1 . Prove that the grasshopper can arrange his jumps in such a
way that he never lands on a point from M .
C8 AUT (Austria)
For any integer n ≥ 2, we compute the integer h(n) by applying the following procedure to its
decimal representation. Let r be the rightmost digit of n.
(1) If r = 0, then the decimal representation of h(n) results from the decimal representation
of n by removing this rightmost digit 0.
(2) If 1 ≤ r ≤ 9 we split the decimal representation of n into a maximal right part R that
solely consists of digits not less than r and into a left part L that either is empty or ends
with a digit strictly smaller than r. Then the decimal representation of h(n) consists of the
decimal representation of L, followed by two copies of the decimal representation of R − 1.
For instance, for the number n = 17,151,345,543, we will have L = 17,151, R = 345,543
and h(n) = 17,151,345,542,345,542.
Prove that, starting with an arbitrary integer n ≥ 2, iterated application of h produces the
integer 1 after finitely many steps.
7
Geometry Problem Shortlist 50th IMO 2009
Geometry
G1 BEL (Belgium)
Let ABC be a triangle with AB = AC. The angle bisectors of A and B meet the sides BC
and AC in D and E, respectively. Let K be the incenter of triangle ADC. Suppose that
∠BEK = 45 ◦. Find all possible values of ∠BAC.
G5 POL (Poland)
Let P be a polygon that is convex and symmetric to some point O. Prove that for some
parallelogram R satisfying P ⊂ R we have
|R| √
≤ 2
|P |
where |R| and |P | denote the area of the sets R and P , respectively.
G6 UKR (Ukraine)
Let the sides AD and BC of the quadrilateral ABCD (such that AB is not parallel to CD)
intersect at point P . Points O1 and O2 are the circumcenters and points H1 and H2 are the
orthocenters of triangles ABP and DCP , respectively. Denote the midpoints of segments
O1 H1 and O2 H2 by E1 and E2 , respectively. Prove that the perpendicular from E1 on CD, the
perpendicular from E2 on AB and the line H1 H2 are concurrent.
8
50th IMO 2009 Problem Shortlist Geometry
G8 BGR (Bulgaria)
Let ABCD be a circumscribed quadrilateral. Let g be a line through A which meets the
segment BC in M and the line CD in N . Denote by I1 , I2 , and I3 the incenters of 4ABM ,
4M N C, and 4N DA, respectively. Show that the orthocenter of 4I1 I2 I3 lies on g.
9
Number Theory Problem Shortlist 50th IMO 2009
Number Theory
N1 AUS (Australia)
A social club has n members. They have the membership numbers 1, 2, . . . , n, respectively.
From time to time members send presents to other members, including items they have already
received as presents from other members. In order to avoid the embarrassing situation that a
member might receive a present that he or she has sent to other members, the club adds the
following rule to its statutes at one of its annual general meetings:
“A member with membership number a is permitted to send a present to a member with
membership number b if and only if a(b − 1) is a multiple of n.”
Prove that, if each member follows this rule, none will receive a present from another member
that he or she has already sent to other members.
N2 PER (Peru)
A positive integer N is called balanced, if N = 1 or if N can be written as a product of an
even number of not necessarily distinct primes. Given positive integers a and b, consider the
polynomial P defined by P (x) = (x + a)(x + b).
(a) Prove that there exist distinct positive integers a and b such that all the numbers P (1), P (2),
. . . , P (50) are balanced.
(b) Prove that if P (n) is balanced for all positive integers n, then a = b.
N3 EST (Estonia)
Let f be a non-constant function from the set of positive integers into the set of positive integers,
such that a − b divides f (a) − f (b) for all distinct positive integers a, b. Prove that there exist
infinitely many primes p such that p divides f (c) for some positive integer c.
N5 HUN (Hungary)
Let P (x) be a non-constant polynomial with integer coefficients. Prove that there is no function
T from the set of integers into the set of integers such that the number of integers x with
T n (x) = x is equal to P (n) for every n ≥ 1, where T n denotes the n-fold application of T .
10
50th IMO 2009 Problem Shortlist Number Theory
N6 TUR (Turkey)
Let k be a positive integer. Show that if there exists a sequence a0 , a1 , . . . of integers satisfying
the condition
an−1 + nk
an = for all n ≥ 1,
n
then k − 2 is divisible by 3.
N7 MNG (Mongolia)
Let a and b be distinct integers greater than 1. Prove that there exists a positive integer n such
that (an − 1)(bn − 1) is not a perfect square.
11
A1 Algebra 50th IMO 2009
Algebra
A1 CZE (Czech Republic)
Find the largest possible integer k, such that the following statement is true:
Let 2009 arbitrary non-degenerated triangles be given. In every triangle the three sides are
colored, such that one is blue, one is red and one is white. Now, for every color separately, let
us sort the lengths of the sides. We obtain
Then there exist k indices j such that we can form a non-degenerated triangle with side lengths
bj , rj , wj .
Solution. We will prove that the largest possible number k of indices satisfying the given
condition is one.
Firstly we prove that b2009 , r2009 , w2009 are always lengths of the sides of a triangle. Without
loss of generality we may assume that w2009 ≥ r2009 ≥ b2009 . We show that the inequality
b2009 + r2009 > w2009 holds. Evidently, there exists a triangle with side lengths w, b, r for the
white, blue and red side, respectively, such that w2009 = w. By the conditions of the problem
we have b + r > w, b2009 ≥ b and r2009 ≥ r. From these inequalities it follows
Secondly we will describe a sequence of triangles for which wj , bj , rj with j < 2009 are not the
lengths of the sides of a triangle. Let us define the sequence ∆j , j = 1, 2, . . . , 2009, of triangles,
where ∆j has
a blue side of length 2j,
a red side of length j for all j ≤ 2008 and 4018 for j = 2009,
and a white side of length j + 1 for all j ≤ 2007, 4018 for j = 2008 and 1 for j = 2009.
Since
12
50th IMO 2009 Algebra A2
A2 EST (Estonia)
1 1 1
Let a, b, c be positive real numbers such that + + = a + b + c. Prove that
a b c
1 1 1 3
2
+ 2
+ 2
≤ .
(2a + b + c) (2b + c + a) (2c + a + b) 16
we obtain
1 1
2
≤ .
(2x + y + z) 4(x + y)(x + z)
Applying this to the left-hand side terms of the inequality to prove, we get
1 1 1
2
+ 2
+
(2a + b + c) (2b + c + a) (2c + a + b)2
1 1 1
≤ + +
4(a + b)(a + c) 4(b + c)(b + a) 4(c + a)(c + b)
(b + c) + (c + a) + (a + b) a+b+c
= = . (1)
4(a + b)(b + c)(c + a) 2(a + b)(b + c)(c + a)
a2 b + a2 c + b2 a + b2 c + c2 a + c2 b ≥ 6abc,
or, equivalently,
a2 b2 + b2 c2 + c2 a2 ≥ a2 bc + ab2 c + abc2 ,
which is equivalent to
13
A2 Algebra 50th IMO 2009
Combining (1), (2), (3), and (4), we will finish the proof:
for all positive real numbers a, b, c. Without loss of generality we choose a + b + c = 1. Thus,
the problem is equivalent to prove for all a, b, c > 0, fulfilling this condition, the inequality
1 1 1 3 1 1 1
+ + ≤ + + . (5)
(1 + a)2 (1 + b)2 (1 + c)2 16 a b c
x
Applying Jensen’s inequality to the function f (x) = , which is concave for 0 ≤ x ≤ 2
(1 + x)2
and increasing for 0 ≤ x ≤ 1, we obtain
a b c A αa + βb + γc
α 2
+β 2
+γ 2
≤ (α + β + γ) , where A = .
(1 + a) (1 + b) (1 + c) (1 + A)2 α+β+γ
1 1 1
Choosing α = , β = , and γ = , we can apply the harmonic-arithmetic-mean inequality
a b c
3 a+b+c 1
A= 1 1 1 ≤ = < 1.
a
+ +
b c
3 3
14
50th IMO 2009 Algebra A3
A3 FRA (France)
Determine all functions f from the set of positive integers into the set of positive integers such
that for all x and y there exists a non degenerated triangle with sides of lengths
Solution. The identity function f (x) = x is the only solution of the problem.
If f (x) = x for all positive integers x, the given three lengths are x, y = f (y) and z =
f (y + f (x) − 1) = x + y − 1. Because of x ≥ 1, y ≥ 1 we have z ≥ max{x, y} > |x − y| and
z < x + y. From this it follows that a triangle with these side lengths exists and does not
degenerate. We prove in several steps that there is no other solution.
Hence,
z−1
f (t − w) ≥ f (t) − (z − 1) > (t − w) + M,
w
a contradiction to the minimality of t.
Therefore the inequality (1) fails for all t ≥ 1, we have proven
z−1
f (t) ≤ · t + M, (2)
w
instead.
15
A3 Algebra 50th IMO 2009
z−1
Now, using (2), we finish the proof of Step 3. Because of z ≤ w we have < 1 and we can
w
choose an integer t sufficiently large to fulfill the condition
2
z−1 z−1
t+ + 1 M < t.
w w
z = f (f (z)) ≤ f (z) ≤ z
16
50th IMO 2009 Algebra A4
A4 BLR (Belarus)
Let a, b, c be positive real numbers such that ab + bc + ca ≤ 3abc. Prove that
√ √ √
r r r
a2 + b 2 b2 + c 2 c 2 + a2 √
+ + +3≤ 2 a+b+ b+c+ c+a .
a+b b+c c+a
Solution. Starting with the terms of the right-hand side, the quadratic-arithmetic-mean in-
equality yields
s
√ √
r
ab 1 a2 + b 2
2 a+b=2 2+
a+b 2 ab
r ! r
1 √
r r
ab a2 + b 2 2ab a2 + b 2
≥2 · 2+ = +
a+b 2 ab a+b a+b
and, analogously,
√ √ √ √
r r r r
2bc b 2 + c2 2ca c 2 + a2
2 b+c≥ + , 2 c+a≥ + .
b+c b+c c+a c+a
Applying the inequality between the arithmetic mean and the squared harmonic mean will
finish the proof:
r r r v r
2ab 2bc 2ca u 3 3abc
+ + ≥3·u q 2 q 2 q 2 =3· ≥ 3.
a+b b+c c+a t
a+b b+c c+a ab + bc + ca
2ab
+ 2bc + 2ca
17
A5 Algebra 50th IMO 2009
A5 BLR (Belarus)
Let f be any function that maps the set of real numbers into the set of real numbers. Prove
that there exist real numbers x and y such that
Let a = f (0). Setting y = 0 in (1) gives f (x − a) ≤ x for all real x and, equivalently,
From (2) and (3) we obtain −1 ≤ f (f (y)) ≤ f (y) + a for all y > 0, so
yf (x) + x ≥ f (x − f (y)) ≥ −a − 1,
whence
−a − x − 1
y≥
f (x)
contrary to our choice of y. Thereby, we have established (5).
Setting x = 0 in (5) leads to a = f (0) ≤ 0 and (2) then yields
Now choose y such that y > 0 and y > −f (−1) − 1 and set x = f (y) − 1. From (1), (5) and
18
50th IMO 2009 Algebra A5
(6) we obtain
Let a = f (0). Setting y = 0 in (7) gives f (x − a) ≤ x for all real x and, equivalently,
With w = 0 we get
a ≤ (1 + 2z + · · · + nz n−1 + nz n )b + az n . (12)
In view of the assumption b < 0 we find some n such that
because the right hand side tends to −∞ as n → ∞. Now (12) and (13) give the desired
contradiction and (9) is established. In addition, we have for z = 1 the strict inequality
f (−1) ≤ −(n + 1) + a
19
A5 Algebra 50th IMO 2009
On the other hand, by (8) and the choice of t we have f (t) ≤ t + a ≤ 0 and hence 1 − f (t) ≥ 1.
The inequality (9) yields
f (1 − f (t)) ≥ 0,
which contradicts (15).
20
50th IMO 2009 Algebra A6
A6 USA (United States of America)
Suppose that s1 , s2 , s3 , . . . is a strictly increasing sequence of positive integers such that the
subsequences
Solution 1. Let D be the common difference of the progression ss1 , ss2 , . . . . Let for n =
1, 2, . . .
dn = sn+1 − sn .
We have to prove that dn is constant. First we show that the numbers dn are bounded. Indeed,
by supposition dn ≥ 1 for all n. Thus, we have for all n
It suffices to show that m = M . Assume that m < M . Choose n such that dn = m. Considering
a telescoping sum of m = dn = sn+1 − sn items not greater than M leads to
and equality holds if and only if all items of the sum are equal to M . Now choose n such that
dn = M . In the same way, considering a telescoping sum of M items not less than m we obtain
and equality holds if and only if all items of the sum are equal to m. The inequalities (1) and
(2) imply that D = M m and that
Hence, dn = m implies dsn = M . Note that sn ≥ s1 + (n − 1) ≥ n for all n and moreover sn > n
if dn = n, because in the case sn = n we would have m = dn = dsn = M in contradiction to
the assumption m < M . In the same way dn = M implies dsn = m and sn > n. Consequently,
there is a strictly increasing sequence n1 , n2 , . . . such that
The sequence ds1 , ds2 , . . . is the sequence of pairwise differences of ss1 +1 , ss2 +1 , . . . and ss1 , ss2 , . . . ,
hence also an arithmetic progression. Thus m = M .
21
A6 Algebra 50th IMO 2009
Solution 2. Let the integers D and E be the common differences of the progressions ss1 , ss2 , . . .
and ss1 +1 , ss2 +1 , . . . , respectively. Let briefly A = ss1 − D and B = ss1 +1 − E. Then, for all
positive integers n,
Since the sequence s1 , s2 , . . . is strictly increasing, we have for all positive integers n
which implies
A + nD < B + nE ≤ A + (n + 1)D,
and thereby
0 < B − A + n(E − D) ≤ D,
which implies D − E = 0 and thus
0 ≤ B − A ≤ D. (3)
and
D = A + (s1 + 1)D − (A + s1 D) = sss1 +1 − sss1 = sB+D − sA+D ≥ m(B − A). (5)
From (3) we consider two cases.
Case 1. B − A = D.
Then, for each positive integer n, ssn +1 = B + nD = A + (n + 1)D = ssn+1 , hence sn+1 = sn + 1
and s1 , s2 , . . . is an arithmetic progression with common difference 1.
i.e.,
(B − A − m) + (D − m(B − A)) ≤ 0. (6)
The inequalities (4)-(6) imply that
Assume that there is some positive integer n such that sn+1 > sn + m. Then
22
50th IMO 2009 Algebra A7
A7 JPN (Japan)
Find all functions f from the set of real numbers into the set of real numbers which satisfy for
all real x, y the identity
Solution 1. It is no hard to see that the two functions given by f (x) = x and f (x) = −x for
all real x respectively solve the functional equation. In the sequel, we prove that there are no
further solutions.
Let f be a function satisfying the given equation. It is clear that f cannot be a constant. Let us
t
first show that f (0) = 0. Suppose that f (0) 6= 0. For any real t, substituting (x, y) = (0, f (0) )
into the given functional equation, we obtain
contradicting the fact that f is not a constant function. Therefore, f (0) = 0. Next for any t,
substituting (x, y) = (t, 0) and (x, y) = (t, −t) into the given equation, we get
and
f (tf (0)) = f (−tf (t)) + t2 ,
respectively. Therefore, we conclude that
Consequently, for every real v, there exists a real u, such that f (u) = v. We also see that if
f (t) = 0, then 0 = f (tf (t)) = t2 so that t = 0, and thus 0 is the only real number satisfying
f (t) = 0.
We next show that for any real number s,
This is clear if f (s) = 0. Suppose now f (s) < 0, then we can find a number t for which
f (s) = −t2 . As t 6= 0 implies f (t) 6= 0, we can also find number a such that af (t) = s.
Substituting (x, y) = (t, a) into the given equation, we get
23
A7 Algebra 50th IMO 2009
we obtain
and
f ((−s − t)f (−t)) = f (sf (−s − t)) + (s + t)2 ,
respectively. Using the fact that f (−x) = −f (x) holds for all x to rewrite the second and the
third equation, and rearranging the terms, we obtain
Adding up these three equations now yields 2f (tf (s)) = 2ts, and therefore, we conclude that
f (tf (s)) = ts holds for every pair of real numbers s, t. By fixing s so that f (s) = 1, we obtain
f (x) = sx. In view of the given equation, we see that s = ±1. It is easy to check that both
functions f (x) = x and f (x) = −x satisfy the given functional equation, so these are the desired
solutions.
where the last statement follows from the given functional equation with x = r and y = s − r.
Hence, h = (s − r)f (r) satisfies f (h) = 0 which implies h2 = f (hf (h)) = f (0) = 0, i.e., h = 0.
Then, by s 6= r we have f (r) = 0 which implies r = 0, and finally f (s) = f (r) = f (0) = 0.
Analogously, it follows that s = 0 which gives the contradiction r = s.
To prove |f (1)| = 1 we apply (2) with t = 1 and also with t = f (1) and obtain f (f (1)) = 1 and
(f (1))2 = f (f (1) · f (f (1))) = f (f (1)) = 1.
Now we choose η ∈ {−1, 1} with f (1) = η. Using that f is odd and the given equation with
x = 1, y = z (second equality) and with x = −1, y = z + 2 (fourth equality) we obtain
= −ηf ((z + 2)f (−1)) = −ηf ((z + 2)(−η)) = ηf ((z + 2)η) = f (z + 2). (4)
Hence,
f (z + 2η) = ηf (ηz + 2) = η(f (ηz) + 2η) = f (z) + 2.
Using this argument twice we obtain
24
50th IMO 2009 Algebra A7
where the last equality follows from (4). Applying the given functional equation we proceed to
where the last equality follows again from (4) with z = 0, i.e., f (2) = 2η. Finally, f (2f (x)) =
f (2ηx) and by injectivity of f we get 2f (x) = 2ηx and hence the two solutions.
25
C1 Combinatorics 50th IMO 2009
Combinatorics
C1 NZL (New Zealand)
Consider 2009 cards, each having one gold side and one black side, lying in parallel on a long
table. Initially all cards show their gold sides. Two players, standing by the same long side of
the table, play a game with alternating moves. Each move consists of choosing a block of 50
consecutive cards, the leftmost of which is showing gold, and turning them all over, so those
which showed gold now show black and vice versa. The last player who can make a legal move
wins.
(a) Does the game necessarily end?
(b) Does there exist a winning strategy for the starting player?
Solution. (a) We interpret a card showing black as the digit 0 and a card showing gold as the
digit 1. Thus each position of the 2009 cards, read from left to right, corresponds bijectively to
a nonnegative integer written in binary notation of 2009 digits, where leading zeros are allowed.
Each move decreases this integer, so the game must end.
(b) We show that there is no winning strategy for the starting player. We label the cards from
right to left by 1, . . . , 2009 and consider the set S of cards with labels 50i, i = 1, 2, . . . , 40. Let
gn be the number of cards from S showing gold after n moves. Obviously, g0 = 40. Moreover,
|gn − gn+1 | = 1 as long as the play goes on. Thus, after an odd number of moves, the non-
starting player finds a card from S showing gold and hence can make a move. Consequently,
this player always wins.
26
50th IMO 2009 Combinatorics C2
C2 ROU (Romania)
For any integer n ≥ 2, let N (n) be the maximal number of triples (ai , bi , ci ), i = 1, . . . , N (n),
consisting of nonnegative integers ai , bi and ci such that the following two conditions are satis-
fied:
(1) ai + bi + ci = n for all i = 1, . . . , N (n),
(2) If i 6= j, then ai 6= aj , bi 6= bj and ci 6= cj .
Determine N (n) for all n ≥ 2.
Comment. The original problem was formulated for m-tuples instead for triples. The numbers
N (m, n) are then defined similarly to N (n) in the case m = 3. The numbers N (3, n) and
N (n, n) should be determined. The case m = 3 is the same as in the present problem. The
upper bound for N (n, n) can be proved by a simple generalization. The construction of a set
of triples attaining the bound can be easily done by induction from n to n + 2.
Solution. Let n ≥ 2 be an integer and let {T1 , . . . , TN } be any set of triples of nonnegative
integers satisfying the conditions (1) and (2). Since the a-coordinates are pairwise distinct we
have
N N
X X N (N − 1)
ai ≥ (i − 1) = .
i=1 i=1
2
Analogously,
N N
X N (N − 1) X N (N − 1)
bi ≥ and ci ≥ .
i=1
2 i=1
2
By constructing examples, we show that this upper bound can be attained, so N (n) = b 2n
3
c + 1.
27
C2 Combinatorics 50th IMO 2009
n = 3k − 1
2n 2n n = 3k 2n n = 3k + 1
3
+ 1 = 2k 3
+ 1 = 2k + 1 3
+ 1 = 2k + 1
ai bi ci ai bi ci ai bi ci
0 k + 1 2k − 2 0 k 2k 0 k 2k + 1
1 k + 2 2k − 4 1 k + 1 2k − 2 1 k + 1 2k − 1
.. .. .. .. .. .. .. .. ..
. . . . . . . . .
k−1 2k 0 k 2k 0 k 2k 1
k 0 2k − 1 k+1 0 2k − 1 k+1 0 2k
k+1 1 2k − 3 k+2 1 2k − 3 k+2 1 2k − 2
.. .. .. .. .. .. .. .. ..
. . . . . . . . .
2k − 1 k − 1 1 2k k−1 1 2k k−1 2
It can be easily seen that the conditions (1) and (2) are satisfied and that we indeed have
b 2n
3
c + 1 triples in each case.
28
50th IMO 2009 Combinatorics C3
C3 RUS (Russian Federation)
Let n be a positive integer. Given a sequence ε1 , . . . , εn−1 with εi = 0 or εi = 1 for each
i = 1, . . . , n − 1, the sequences a0 , . . . , an and b0 , . . . , bn are constructed by the following rules:
a0 = b0 = 1, a1 = b1 = 7,
(
2ai−1 + 3ai , if εi = 0,
ai+1 = for each i = 1, . . . , n − 1,
3ai−1 + ai , if εi = 1,
(
2bi−1 + 3bi , if εn−i = 0,
bi+1 = for each i = 1, . . . , n − 1.
3bi−1 + bi , if εn−i = 1,
Prove that an = bn .
(1, 7)wσ0 = 2(1, 7)w + 3(1, 7)wσ = 2(1, 7)w + 3(1, 7)σw = 2(2, 1)σw + 3(1, 7)σw
= (7, 23)σw = (1, 7)0σw .
29
C3 Combinatorics 50th IMO 2009
(1, 7)wσ1 = 3(1, 7)w + (1, 7)wσ = 3(1, 7)w + (1, 7)σw = 3(2, 1)σw + (1, 7)σw
= (7, 10)σw = (1, 7)1σw .
Thus the induction step is complete, (3) and hence also an = bn are proved.
which can be proved by induction on the length of w. Then (3) also follows by induction on
the length of w:
(1, 7)αβw = ((1, 7)w , (1, 7)βw )α = ((1, 7)w , (1, 7)wβ )α = (1, 7)wβα .
30
50th IMO 2009 Combinatorics C4
C4 NLD (Netherlands)
For an integer m ≥ 1, we consider partitions of a 2m × 2m chessboard into rectangles consisting
of cells of the chessboard, in which each of the 2m cells along one diagonal forms a separate
rectangle of side length 1. Determine the smallest possible sum of rectangle perimeters in such
a partition.
31
C4 Combinatorics 50th IMO 2009
perimeter is at least
Since the function f (x) = 2x log2 x is convex for x > 0, Jensen’s inequality immediately shows
that the minimum of the right hand sight of (1) is attained for i = k/2. Hence the total
perimeter of the optimal partition of Bk is at least 2k + 2k/2 log2 k/2 + 2(k/2) log2 (k/2) = Dk .
Solution 2. We start as in Solution 1 and present another proof that m2m+1 is a lower bound
for the total perimeter of a partition of B2m into n rectangles. Let briefly M = 2m . For
1 ≤ i ≤ M , let ri denote the number of rectangles in the partition that cover some cell from
row i and let cj be the number of rectangles that cover some cell from column j. Note that the
total perimeter p of all rectangles in the partition is
M M
!
X X
p=2 ri + ci .
i=1 i=1
No rectangle can simultaneously cover cells from row i and from column i since otherwise it
would also cover the cell Cii . We classify subsets S of rectangles of the partition as follows.
We say that S is of type i, 1 ≤ i ≤ M , if S contains all ri rectangles that cover some cell from
row i, but none of the ci rectangles that cover some cell from column i. Altogether there are
2n−ri −ci subsets of type i. Now we show that no subset S can be simultaneously of type i and of
type j if i 6= j. Assume the contrary and let without loss of generality i < j. The cell Cij must
be covered by some rectangle R. The subset S is of type i, hence R is contained in S. S is of
type j, thus R does not belong to S, a contradiction. Since there are 2n subsets of rectangles
of the partition, we infer
XM M
X
n−ri −ci
n
2 ≥ 2 =2n
2−(ri +ci ) . (2)
i=1 i=1
32
50th IMO 2009 Combinatorics C5
C5 NLD (Netherlands)
Five identical empty buckets of 2-liter capacity stand at the vertices of a regular pentagon.
Cinderella and her wicked Stepmother go through a sequence of rounds: At the beginning of
every round, the Stepmother takes one liter of water from the nearby river and distributes it
arbitrarily over the five buckets. Then Cinderella chooses a pair of neighboring buckets, empties
them into the river, and puts them back. Then the next round begins. The Stepmother’s goal
is to make one of these buckets overflow. Cinderella’s goal is to prevent this. Can the wicked
Stepmother enforce a bucket overflow?
Solution 1. No, the Stepmother cannot enforce a bucket overflow and Cinderella can keep
playing forever. Throughout we denote the five buckets by B0 , B1 , B2 , B3 , and B4 , where Bk
is adjacent to bucket Bk−1 and Bk+1 (k = 0, 1, 2, 3, 4) and all indices are taken modulo 5.
Cinderella enforces that the following three conditions are satisfied at the beginning of every
round:
(1) Two adjacent buckets (say B1 and B2 ) are empty.
(2) The two buckets standing next to these adjacent buckets (here B0 and B3 ) have total
contents at most 1.
(3) The remaining bucket (here B4 ) has contents at most 1.
These conditions clearly hold at the beginning of the first round, when all buckets are empty.
Assume that Cinderella manages to maintain them until the beginning of the r-th round (r ≥ 1).
Denote by xk (k = 0, 1, 2, 3, 4) the contents of bucket Bk at the beginning of this round and
by yk the corresponding contents after the Stepmother has distributed her liter of water in this
round.
By the conditions, we can assume x1 = x2 = 0, x0 + x3 ≤ 1 and x4 ≤ 1. Then, since the
Stepmother adds one liter, we conclude y0 + y1 + y2 + y3 ≤ 2. This inequality implies y0 + y2 ≤ 1
or y1 + y3 ≤ 1. For reasons of symmetry, we only consider the second case.
Then Cinderella empties buckets B0 and B4 .
At the beginning of the next round B0 and B4 are empty (condition (1) is fulfilled), due to
y1 + y3 ≤ 1 condition (2) is fulfilled and finally since x2 = 0 we also must have y2 ≤ 1 (condition
(3) is fulfilled).
Therefore, Cinderella can indeed manage to maintain the three conditions (1)–(3) also at the
beginning of the (r + 1)-th round. By induction, she thus manages to maintain them at the
beginning of every round. In particular she manages to keep the contents of every single bucket
at most 1 liter. Therefore, the buckets of 2-liter capacity will never overflow.
Solution 2. We prove that Cinderella can maintain the following two conditions and hence
she can prevent the buckets from overflow:
(10 ) Every two non-adjacent buckets contain a total of at most 1.
(20 ) The total contents of all five buckets is at most 23 .
We use the same notations as in the first solution. The two conditions again clearly hold at
the beginning. Assume that Cinderella maintained these two conditions until the beginning of
the r-th round. A pair of non-neighboring buckets (Bi , Bi+2 ), i = 0, 1, 2, 3, 4 is called critical
33
C5 Combinatorics 50th IMO 2009
if yi + yi+2 > 1. By condition (20 ), after the Stepmother has distributed her water we have
y0 + y1 + y2 + y3 + y4 ≤ 52 . Therefore,
and hence there is a pair of non-neighboring buckets which is not critical, say (B0 , B2 ). Now,
if both of the pairs (B3 , B0 ) and (B2 , B4 ) are critical, we must have y1 < 21 and Cinderella
can empty the buckets B3 and B4 . This clearly leaves no critical pair of buckets and the total
contents of all the buckets is then y1 + (y0 + y2 ) ≤ 32 . Therefore, conditions (10 ) and (20 ) are
fulfilled.
Now suppose that without loss of generality the pair (B3 , B0 ) is not critical. If in this case
y0 ≤ 12 , then one of the inequalities y0 + y1 + y2 ≤ 32 and y0 + y3 + y4 ≤ 23 must hold. But then
Cinderella can empty B3 and B4 or B1 and B2 , respectively and clearly fulfill the conditions.
Finally consider the case y0 > 12 . By y0 + y1 + y2 + y3 + y4 ≤ 25 , at least one of the pairs (B1 , B3 )
and (B2 , B4 ) is not critical. Without loss of generality let this be the pair (B1 , B3 ). Since the
pair (B3 , B0 ) is not critical and y0 > 12 , we must have y3 ≤ 12 . But then, as before, Cinderella
can maintain the two conditions at the beginning of the next round by either emptying B1 and
B2 or B4 and B0 .
34
50th IMO 2009 Combinatorics C5
A harder variant. Five identical empty buckets of capacity b stand at the vertices of a regular
pentagon. Cinderella and her wicked Stepmother go through a sequence of rounds: At the
beginning of every round, the Stepmother takes one liter of water from the nearby river and
distributes it arbitrarily over the five buckets. Then Cinderella chooses a pair of neighboring
buckets, empties them into the river, and puts them back. Then the next round begins. The
Stepmother’s goal is to make one of these buckets overflow. Cinderella’s goal is to prevent this.
Determine all bucket capacities b for which the Stepmother can enforce a bucket to overflow.
35
C6 Combinatorics 50th IMO 2009
C6 BGR (Bulgaria)
On a 999 × 999 board a limp rook can move in the following way: From any square it can move
to any of its adjacent squares, i.e. a square having a common side with it, and every move
must be a turn, i.e. the directions of any two consecutive moves must be perpendicular. A non-
intersecting route of the limp rook consists of a sequence of pairwise different squares that the
limp rook can visit in that order by an admissible sequence of moves. Such a non-intersecting
route is called cyclic, if the limp rook can, after reaching the last square of the route, move
directly to the first square of the route and start over.
How many squares does the longest possible cyclic, non-intersecting route of a limp rook
visit?
There is up to reflection only one way the rook can take from (a, b) to (a + 2, b + 2). Let this
way be (a, b) → (a, b + 1) → (a + 1, b + 1) → (a + 1, b + 2) → (a + 2, b + 2). Also let without
loss of generality the color of the cell (a, b + 1) be B (otherwise change the roles of columns and
rows).
Now consider the A-cell (a, b+2). The only way the rook can pass through it is via (a−1, b+2) →
(a, b + 2) → (a, b + 3) in this order, since according to our assumption after every A-cell the
rook passes through a B-cell. Hence, to connect these two parts of the path, there must be
36
50th IMO 2009 Combinatorics C6
a path connecting the cell (a, b + 3) and (a, b) and also a path connecting (a + 2, b + 2) and
(a − 1, b + 2).
But these four cells are opposite vertices of a convex quadrilateral and the paths are outside of
that quadrilateral and hence they must intersect. This is due to the following fact:
The path from (a, b) to (a, b + 3) together with the line segment joining these two cells form a
closed loop that has one of the cells (a − 1, b + 2) and (a + 2, b + 2) in its inside and the other
one on the outside. Thus the path between these two points must cross the previous path.
But an intersection is only possible if a cell is visited twice. This is a contradiction.
Hence the number of cells visited is at most 4 · (4992 − 1).
The following picture indicates a recursive construction for all n × n-chessboards with n ≡ 3
mod 4 which clearly yields a path that misses exactly one A-cell (marked with a dot, the center
cell of the 15 × 15-chessboard) and hence, in the case of n = 999 crosses exactly 4 · (4992 − 1)
cells.
37
C7 Combinatorics 50th IMO 2009
Variant 1. A grasshopper jumps along the real axis. He starts at point 0 and makes 2009
jumps to the right with lengths 1, 2, . . . , 2009 in an arbitrary order. Let M be a set of 2008
positive integers less than 1005 · 2009. Prove that the grasshopper can arrange his jumps in
such a way that he never lands on a point from M .
Variant 2. Let n be a nonnegative integer. A grasshopper jumps along the real axis. He starts
at point 0 and makes n + 1 jumps to the right with pairwise different positive integral lengths
a1 , a2 , . . . , an+1 in an arbitrary order. Let M be a set of n positive integers in the interval (0, s),
where s = a1 + a2 + · · · + an+1 . Prove that the grasshopper can arrange his jumps in such a
way that he never lands on a point from M .
The right hand side of the last inequality obviously attains its minimum for m = 1004 and this
minimum value is greater than 2008, a contradiction.
Case 2. M does contain a number µ divisible by 2009.
By the pigeonhole principle there exists some r ∈ {1, . . . , 2008} such that M does not contain
numbers with remainder r modulo 2009. We fix the numbers 2009(k − 1) + r as landing points,
k = 1, 2, . . . , 1005. Moreover, 1005 · 2009 is a landing point. Consider the open intervals
38
50th IMO 2009 Combinatorics C7
Ik = (2009(k − 1) + r, 2009k + r), k = 1, 2, . . . , 1004. Analogously to Case 1, it is enough to
show that we can choose in 1003 of these intervals exactly one landing point outside of M \ {µ}
such that each of the lengths of D = {1, 2, . . . , 1004} \ {r} are implemented. Note that r
and 2009 − r are realized by the first and last jump and that choosing µ would realize these
two differences again. Let nk , k = 1, 2, . . . , 1004, be the number of elements of M \ {µ} that
belong to the interval Ik and p1 , p2 , . . . , p1004 be a permutation of {1, 2, . . . , 1004} such that
np1 ≥ np2 ≥ · · · ≥ np1004 . With the same reasoning as in Case 1 we can verify that a greedy
choice of the landing points in Ip2 , Ip3 , . . . , Ip1004 is possible. We only have to replace (1) by
npm+1 ≥ 2(1004 − m)
Comment. The cardinality 2008 of M in the problem is the maximum possible value. For
M = {1, 2, . . . , 2009}, the grasshopper necessarily lands on a point from M .
Solution of Variant 2. First of all we remark that the statement in the problem implies a
strengthening of itself: Instead of |M | = n it is sufficient to suppose that |M ∩ (0, s − a]| ≤ n,
where a = min{a1 , a2 , . . . , an+1 }. This fact will be used in the proof.
We prove the statement by induction on n. The case n = 0 is obvious. Let n > 0 and let the
assertion be true for all nonnegative integers less than n. Moreover let a1 , a2 , . . . , an+1 , s and
M be given as in the problem. Without loss of generality we may assume that an+1 < an <
· · · < a2 < a1 . Set
Xk
Tk = ai for k = 0, 1, . . . , n + 1.
i=1
Note that 0 = T0 < T1 < · · · < Tn+1 = s. We will make use of the induction hypothesis as
follows:
Claim 1. It suffices to show that for some m ∈ {1, 2, . . . , n + 1} the grasshopper is able to do
at least m jumps without landing on a point of M and, in addition, after these m jumps he
has jumped over at least m points of M .
Proof. Note that m = n + 1 is impossible by |M | = n. Now set n0 = n − m. Then 0 ≤ n0 < n.
The remaining n0 + 1 jumps can be carried out without landing on one of the remaining at
most n0 forbidden points by the induction hypothesis together with a shift of the origin. This
proves the claim.
An integer k ∈ {1, 2, . . . , n + 1} is called smooth, if the grasshopper is able to do k jumps with
the lengths a1 , a2 , . . . , ak in such a way that he never lands on a point of M except for the very
last jump, when he may land on a point of M .
Obviously, 1 is smooth. Thus there is a largest number k ∗ , such that all the numbers 1, 2, . . . , k ∗
are smooth. If k ∗ = n + 1, the proof is complete. In the following let k ∗ ≤ n.
Claim 2. We have
Tk∗ ∈ M and |M ∩ (0, Tk∗ )| ≥ k ∗ . (3)
Proof. In the case Tk∗ 6∈ M any sequence of jumps that verifies the smoothness of k ∗ can be
extended by appending ak∗ +1 , which is a contradiction to the maximality of k ∗ . Therefore we
have Tk∗ ∈ M . If |M ∩ (0, Tk∗ )| < k ∗ , there exists an l ∈ {1, 2, . . . , k ∗ } with Tk∗ +1 − al 6∈ M . By
the induction hypothesis with k ∗ − 1 instead of n, the grasshopper is able to reach Tk∗ +1 − al
39
C7 Combinatorics 50th IMO 2009
with k ∗ jumps of lengths from {a1 , a2 , . . . , ak∗ +1 } \ {al } without landing on any point of M .
Therefore k ∗ + 1 is also smooth, which is a contradiction to the maximality of k ∗ . Thus Claim 2
is proved.
Now, by Claim 2, there exists a smallest integer k ∈ {1, 2, . . . , k ∗ } with
Tk ∈ M and |M ∩ (0, Tk )| ≥ k.
Proof. If k = 1, then (4) is clearly satisfied. In the following let k > 1. If Tk−1 ∈ M , then
(4) follows immediately by the minimality of k. If Tk−1 6∈ M , by the smoothness of k − 1, we
obtain a situation as in Claim 1 with m = k − 1 provided that |M ∩ (0, Tk−1 ]| ≥ k − 1. Hence,
we may even restrict ourselves to |M ∩ (0, Tk−1 ]| ≤ k − 2 in this case and Claim 3 is proved.
Choose an integer v ≥ 0 with |M ∩ (0, Tk )| = k + v. Let r1 > r2 > · · · > rl be exactly those
indices r from {k + 1, k + 2, . . . , n + 1} for which Tk + ar 6∈ M . Then
Tk + ar1 − a1 < Tk + ar1 − a2 < · · · < Tk + ar1 − ak < Tk + ar2 − ak < · · · < Tk + arv+2 − ak < Tk
and that this are k + v + 1 numbers from (0, Tk ). Therefore we find some r ∈ {k + 1, k +
2, . . . , n + 1} and some s ∈ {1, 2, . . . , k} with Tk + ar 6∈ M and Tk + ar − as 6∈ M . Consider the
set of jump lengths B = {a1 , a2 , . . . , ak , ar } \ {as }. We have
X
x = Tk + ar − as
x∈B
and
Tk + ar − as − min(B) = Tk − as ≤ Tk−1 .
By (4) and the strengthening, mentioned at the very beginning with k − 1 instead of n, the
grasshopper is able to reach Tk + ar − as by k jumps with lengths from B without landing on
any point of M . From there he is able to jump to Tk + ar and therefore we reach a situation as
in Claim 1 with m = k + 1, which completes the proof.
40
50th IMO 2009 Combinatorics C8
C8 AUT (Austria)
For any integer n ≥ 2, we compute the integer h(n) by applying the following procedure to its
decimal representation. Let r be the rightmost digit of n.
(1) If r = 0, then the decimal representation of h(n) results from the decimal representation
of n by removing this rightmost digit 0.
(2) If 1 ≤ r ≤ 9 we split the decimal representation of n into a maximal right part R that
solely consists of digits not less than r and into a left part L that either is empty or ends
with a digit strictly smaller than r. Then the decimal representation of h(n) consists of the
decimal representation of L, followed by two copies of the decimal representation of R − 1.
For instance, for the number n = 17,151,345,543, we will have L = 17,151, R = 345,543
and h(n) = 17,151,345,542,345,542.
Prove that, starting with an arbitrary integer n ≥ 2, iterated application of h produces the
integer 1 after finitely many steps.
Solution 1. We identify integers n ≥ 2 with the digit-strings, briefly strings, of their decimal
representation and extend the definition of h to all non-empty strings with digits from 0 to
9. We recursively define ten functions f0 , . . . , f9 that map some strings into integers for k =
9, 8, . . . , 1, 0. The function f9 is only defined on strings x (including the empty string ε) that
entirely consist of nines. If x consists of m nines, then f9 (x) = m + 1, m = 0, 1, . . . . For k ≤ 8,
the domain of fk (x) is the set of all strings consisting only of digits that are ≥ k. We write x
in the form x0 kx1 kx2 k . . . xm−1 kxm where the strings xs only consist of digits ≥ k + 1. Note
that some of these strings might equal the empty string ε and that m = 0 is possible, i.e. the
digit k does not appear in x. Then we define
m
X
fk (x) = 4fk+1 (xs ) .
s=0
41
C8 Combinatorics 50th IMO 2009
than r. Then n = ykzr and h(n) = ykz(r − 1)z(r − 1). Let d(y) be the smallest digit of y. We
consider two cases which do not exclude each other.
Case 1. d(y) ≥ k.
Then
fk (n) − fk (h(n)) = fk (zr) − fk (z(r − 1)z(r − 1)).
In view of Fact 1 this difference is positive if and only if
Here we use the additional definition f10 (ε) = 0 if r = 9. Consequently, fk (n) − fk (h(n)) > 0
and according to Fact 1, f0 (n) − f0 (h(n)) > 0.
Case 2. d(y) ≤ k.
We prove by induction on d(y) = k, k − 1, . . . , 0 that fi (n) − fi (h(n)) > 0 for all i = 0, . . . , d(y).
By Fact 1, it suffices to do so for i = d(y). The initialization d(y) = k was already treated in
Case 1. Let t = d(y) < k. Write y in the form utv where v does not contain digits ≤ t. Then,
in view of the induction hypothesis,
ft (n) − ft (h(n)) = ft (vkzr) − ft (vkz(r − 1)z(r − 1)) = 4ft+1 (vkzr) − 4ft+1 (vkz(r−1)z(r−1)) > 0.
Thus the inequality fd(y) (n) − fd(y) (h(n)) > 0 is established and from Fact 1 it follows that
f0 (n) − f0 (h(n)) > 0.
Solution 2. We identify integers n ≥ 2 with the digit-strings, briefly strings, of their decimal
representation and extend the definition of h to all non-empty strings with digits from 0 to
9. Moreover, let us define that the empty string, ε, is being mapped to the empty string. In
the following all functions map the set of strings into the set of strings. For two functions f
and g let g ◦ f be defined by (g ◦ f )(x) = g(f (x)) for all strings x and let, for non-negative
integers n, f n denote the n-fold application of f . For any string x let s(x) be the smallest digit
of x, and for the empty string let s(ε) = ∞. We define nine functions g1 , . . . , g9 as follows: Let
k ∈ {1, . . . , 9} and let x be a string. If x = ε then gk (x) = ε. Otherwise, write x in the form
x = yzr where y is either the empty string or ends with a digit smaller than k, s(z) ≥ k and r
is the rightmost digit of x. Then gk (x) = zr.
Lemma 1. We have gk ◦ h = gk ◦ h ◦ gk for all k = 1, . . . , 9.
Proof of Lemma 1. Let x = yzr be as in the definition of gk . If y = ε, then gk (x) = x, whence
So let y 6= ε.
Case 1. z contains a digit smaller than r.
Let z = uav where a < r and s(v) ≥ r. Then
(
yuav if r = 0,
h(x) =
yuav(r − 1)v(r − 1) if r > 0
42
50th IMO 2009 Combinatorics C8
and (
uav if r = 0,
h(gk (x)) = h(zr) = h(uavr) =
uav(r − 1)v(r − 1) if r > 0.
Since y ends with a digit smaller than k, (1) is obviously true.
Case 2. z does not contain a digit smaller than r.
Let y = uv where u is either the empty string or ends with a digit smaller than r and s(v) ≥ r.
We have (
uvz if r = 0,
h(x) =
uvz(r − 1)vz(r − 1) if r > 0
and (
z if r = 0,
h(gk (x)) = h(zr) =
z(r − 1)z(r − 1) if r > 0.
Recall that y and hence v ends with a digit smaller than k, but all digits of v are at least r.
Now if r > k, then v = ε, whence the terminal digit of u is smaller than k, which entails
If r ≤ k, then
gk (h(x)) = z(r − 1) = gk (h(gk (x))) ,
so that in both cases (1) is true. Thus Lemma 1 is proved.
Lemma 2. Let k ∈ {1, . . . , 9}, let x be a non-empty string and let n be a positive integer. If
hn (x) = ε then (gk ◦ h)n (x) = ε.
Proof of Lemma 2. We proceed by induction on n. If n = 1 we have
Now consider the step from n − 1 to n where n ≥ 2. Let hn (x) = ε and let y = h(x). Then
hn−1 (y) = ε and by the induction hypothesis (gk ◦ h)n−1 (y) = ε. In view of Lemma 1,
43
C8 Combinatorics 50th IMO 2009
1) = ygk (h(zr))ygk (h(zr)) and we may apply the induction hypothesis to see that if ygk h(zr))
terminates, then h(yzr) terminates.
Case 3. r > k.
Then h(yzr) = yh(zr) = ygk (h(zr)).
Note that ygk (h(zr)) has the form yz 0 r0 where s(z 0 ) ≥ k. By the same arguments it is sufficient
to prove that ygk (h(z 0 r0 )) = y(gk ◦ h)2 (zr) terminates and, by induction, that y(gk ◦ h)m (zr)
terminates for some positive integer m. In view of Lemma 2 there is some m such that (gk ◦
h)m (zr) = , so x = yzr terminates if y terminates. Thus Lemma 3 is proved.
Now assume that there is some string x that does not terminate. We choose x minimal. If
x ≥ 10, we can write x in the form x = yzr of Lemma 3 and by this lemma x terminates since
y and zr are smaller than x. If x ≤ 9, then h(x) = (x − 1)(x − 1) and h(x) terminates again
by Lemma 3 and the minimal choice of x.
Step 1. fk is separating.
Before embarking on the proof of this, we record a useful observation which is easily proved by
induction on M .
44
50th IMO 2009 Combinatorics C8
Claim 1. For any strings A, B and any positive integer M such that fkM −1 (B) 6= , we have
Now we call a pair (a, b) of strings wicked provided that a is an initial segment of b, but there
is no N ∈ N0 such that fkN (b) = a. We need to show that there are none, so assume that
there were such pairs. Choose a wicked pair (a, b) for which gk+1 (b) attains its minimal possible
value. Obviously b 6= for any wicked pair (a, b). Let z denote the terminal digit of b. Observe
that a 6= b, which means that a is also an initial segment of λ(b). To facilitate the construction
of the eventual contradiction, we prove
Claim 2. There cannot be an N ∈ N0 such that
Proof of Claim 2. For suppose that such an N existed. Because gk+1 (λ(b)) < gk+1 (b) in view
of the coherency of fk+1 , the pair (a, λ(b)) is not wicked. But then there is some N 0 for which
0 0
fkN (λ(b)) = a which entails fkN +N (b) = a, contradiction. Hence Claim 2 is proved.
fk (b) = LR∗ kR∗ k , fk2 (b) = LR∗ kR∗ and fk+1 (b) = LR∗ .
Using that R∗ is a terminal segment of LR∗ and the coherency of fk+1 , we infer
Hence the pair (, R∗ ) is not wicked, so there is some minimal M ∈ N0 with fkM (R∗ ) = and
by Claim 1 it follows that fk2+M (b) = LR∗ k. Finally, we infer that λ(b) = LR∗ = fk (LR∗ k) =
fk3+M (b), which yields a contradiction to Claim 2.
This final contradiction establishes that fk is indeed separating.
Step 2. fk is coherent.
To prepare the proof of this, we introduce some further pieces of terminology. A nonempty
string (a1 , a2 , . . . , an ) is called a hypostasis, if an < ai for all i = 1, . . . , n − 1. Reading an
arbitrary string a backwards, we easily find a, possibly empty, sequence (A1 , A2 , . . . , Am ) of
hypostases such that ρ(A1 ) ≤ ρ(A2 ) ≤ · · · ≤ ρ(Am ) and, symbolically, a = A1 A2 . . . Am .
The latter sequence is referred to as the decomposition of a. So, for instance, (20, 0, 9) is the
decomposition of 2009 and the string 50 is a hypostasis. Next we explain when we say about
two strings a and b that a is injectible into b. The definition is by induction on the length
of b. Let (B1 , B2 , . . . , Bn ) be the decomposition of b into hypostases. Then a is injectible
into b if for the decomposition (A1 , A2 , . . . , Am ) of a there is a strictly increasing function
H : {1, 2, . . . , m} −→ {1, 2, . . . , n} satisfying
ρ(Ai ) = ρ(BH(i) ) for all i = 1, . . . , m;
45
C8 Combinatorics 50th IMO 2009
A pair (a, b) of strings is called aggressive if a is injectible into b and nevertheless gk (a) > gk (b).
Observe that if fk was incoherent, which we shall assume from now on, then such pairs existed.
Now among all aggressive pairs we choose one, say (a, b), for which gk (b) attains its least possible
value. Obviously fk (a) cannot be injectible into fk (b), for otherwise the pair (fk (a), fk (b)) was
aggressive and contradicted our choice of (a, b). Let (A1 , A2 , . . . , Am ) and (B1 , B2 , . . . , Bn )
be the decompositions of a and b and take a function H : {1, 2, . . . , m} −→ {1, 2, . . . , n}
exemplifying that a is indeed injectible into b. If we had H(m) < n, then a was also injectible
into the number b0 whose decomposition is (B1 , B2 , . . . , Bn−1 ) and by separativity of fk we
obtained gk (b0 ) < gk (b), whence the pair (a, b0 ) was also aggressive, contrary to the minimality
condition imposed on b. Therefore a is strongly injectible into b. In particular, a and b have a
common terminal digit, say z. If we had z ≤ k, then fk (a) = λ(a) and fk (b) = λ(b), so that by
Claim 3, fk (a) was injectible into fk (b), which is a contradiction. Hence, z ≥ k + 1.
Now let r be the minimal element of {1, 2, . . . , m} for which ρ(Ar ) = z. Then the maximal
right part of a consisting of digits ≥ z is equal to Ra , the string whose decomposition is
(Ar , Ar+1 , . . . , Am ). Then Ra − 1 is a hypostasis and (A1 , . . . , Ar−1 , Ra − 1, Ra − 1) is the
decomposition of fk (a). Defining s and Rb in a similar fashion with respect to b, we see that
(B1 , . . . , Bs−1 , Rb − 1, Rb − 1) is the decomposition of fk (b). The definition of injectibility then
easily entails that Ra is strongly injectible into Rb . It follows from Claim 3 that λ(Ra ) =
λ(Ra − 1) is injectible into λ(Rb ) = λ(Rb − 1), whence the function H 0 : {1, 2, . . . , r + 1} −→
{1, 2, . . . , s + 1}, given by H 0 (i) = H(i) for i = 1, 2, . . . , r − 1, H 0 (r) = s and H 0 (r + 1) = s + 1
exemplifies that fk (a) is injectible into fk (b), which yields a contradiction as before.
This shows that aggressive pairs cannot exist, whence fk is indeed coherent, which finishes the
proof of the seductivity of k, whereby the problem is finally solved.
46
50th IMO 2009 Geometry G1
Geometry
G1 BEL (Belgium)
Let ABC be a triangle with AB = AC. The angle bisectors of A and B meet the sides BC
and AC in D and E, respectively. Let K be the incenter of triangle ADC. Suppose that
∠BEK = 45 ◦. Find all possible values of ∠BAC.
Solution 1. Answer: ∠BAC = 60 ◦ or ∠BAC = 90 ◦ are possible values and the only possible
values.
Let I be the incenter of triangle ABC, then K lies on the line CI. Let F be the point, where
the incircle of triangle ABC touches the side AC; then the segments IF and ID have the same
length and are perpendicular to AC and BC, respectively.
A
A
F
E=F E
S R
S I
I R
P K
P K
B D Q C B D Q C
Figure 1 Figure 2
Let P , Q and R be the points where the incircle of triangle ADC touches the sides AD, DC
and CA, respectively. Since K and I lie on the angle bisector of ∠ACD, the segments ID and
IF are symmetric with respect to the line IC. Hence there is a point S on IF where the incircle
of triangle ADC touches the segment IF . Then segments KP , KQ, KR and KS all have the
same length and are perpendicular to AD, DC, CA and IF , respectively. So – regardless of
the value of ∠BEK – the quadrilateral KRF S is a square and ∠SF K = ∠KF C = 45 ◦.
Consider the case ∠BAC = 60 ◦ (see Figure 1). Then triangle ABC is equilateral. Furthermore
we have F = E, hence ∠BEK = ∠IF K = ∠SEK = 45 ◦. So 60 ◦ is a possible value for ∠BAC.
Now consider the case ∠BAC = 90 ◦ (see Figure 2). Then ∠CBA = ∠ACB = 45 ◦. Fur-
thermore, ∠KIE = 12 ∠CBA + 12 ∠ACB = 45 ◦, ∠AEB = 180 ◦ − 90 ◦ − 22.5 ◦ = 67.5 ◦ and
∠EIA = ∠BID = 180 ◦ − 90 ◦ − 22.5 ◦ = 67.5 ◦. Hence triangle IEA is isosceles and a reflection
of the bisector of ∠IAE takes I to E and K to itself. So triangle IKE is symmetric with
respect to this axis, i.e. ∠KIE = ∠IEK = ∠BEK = 45 ◦. So 90 ◦ is a possible value for
∠BAC, too.
If, on the other hand, ∠BEK = 45 ◦ then ∠BEK = ∠IEK = ∠IF K = 45 ◦. Then
• either F = E, which makes the angle bisector BI be an altitude, i.e., which makes triangle
ABC isosceles with base AC and hence equilateral and so ∠BAC = 60 ◦,
• or E lies between F and C, which makes the points K, E, F and I concyclic, so 45 ◦ =
∠KF C = ∠KF E = ∠KIE = ∠CBI + ∠ICB = 2 · ∠ICB = 90 ◦ − 21 ∠BAC, and so
∠BAC = 90 ◦,
47
G1 Geometry 50th IMO 2009
A α
2
E 3α
4
45˚ α
α I 45˚
2
45˚
4
K α
45˚
45˚ 4
β 45˚
B D C
Figure 3
Multiplication of these four equations yields
sin(45 ◦ + 43 α) sin(45 ◦ + α2 )
1= .
sin(90 ◦ − α4 )
But, since
this is equivalent to
sin(45 ◦ + 34 α) sin(45 ◦ + α2 ) = cos (45 ◦ + 43 α) cos (45 ◦ + α2 ) + sin (45 ◦ + 34 α) sin (45 ◦ + α2 )
and finally
cos (45 ◦ + 43 α) cos (45 ◦ + α2 ) = 0.
48
50th IMO 2009 Geometry G1
But this means cos (45 ◦ + 34 α) = 0, hence 45 ◦ + 34 α = 90 ◦, i.e. α = 60 ◦ or cos (45 ◦ + α2 ) = 0,
hence 45 ◦ + α2 = 90 ◦, i.e. α = 90 ◦. So these values are the only two possible values for α.
On the other hand, both α = 90 ◦ and α = 60 ◦ yield ∠BEK = 45 ◦, this was shown in
Solution 1.
49
G2 Geometry 50th IMO 2009
Now (1) is equivalent to AP · P C = AQ · QB which means that the power of points P and Q
with respect to the circumcircle of 4ABC are equal, hence OP = OQ.
A
Q
M
k
P
C′ B′
L
O
K
B C
Figure 1
Comment. The last argument can also be established by the following calculation:
OP 2 − OQ2 = OB 02 + B 0 P 2 − OC 02 − C 0 Q2
= (OA2 − AB 02 ) + B 0 P 2 − (OA2 − AC 02 ) − C 0 Q2
= (AC 02 − C 0 Q2 ) − (AB 02 − B 0 P 2 )
= (AC 0 − C 0 Q)(AC 0 + C 0 Q) − (AB 0 − B 0 P )(AB 0 + B 0 P )
= AQ · QB − AP · P C.
50
50th IMO 2009 Geometry G2
Solution 2. Again, denote by K, L, M the midpoints of segments BP , CQ, and P Q, respec-
tively. Let O, S, T be the circumcenters of triangles ABC, KLM , and AP Q, respectively (see
Figure 2). Note that M K and LM are the midlines in triangles BP Q and CP Q, respectively, so
−−→ 1 −−→ −−→ −→
M K = 2 QB and M L = 21 P C. Denote by prl (→ −v ) the projection of vector →
−
v onto line l. Then
−→ −→ −→ 1 − → 1 −→ 1 −
−→ −−→ −−→ −−→
prAB (OT ) = prAB (OA − T A) = 2 BA − 2 QA = 2 BQ = KM and prAB (SM ) = prM K (SM ) =
1 −−→ −→ −−→ −→
2
KM = 12 prAB (OT ). Analogously we get prCA (SM ) = 21 prCA (OT ). Since AB and CA are not
−−→ −→
parallel, this implies that SM = 12 OT .
A
Q T
k M
P
O L
B C
Figure 2
Now, since the circle k touches P Q at M , we get SM ⊥ P Q, hence OT ⊥ P Q. Since T is
equidistant from P and Q, the line OT is a perpendicular bisector of segment P Q, and hence
O is equidistant from P and Q which finishes the proof.
51
G3 Geometry 50th IMO 2009
Solution 1. Denote by k the incircle and by ka the excircle opposite to A of triangle ABC.
Let k and ka touch the side BC at the points X and T , respectively, let ka touch the lines AB
and AC at the points P and Q, respectively. We use several times the fact that opposing sides
of a parallelogram are of equal length, that points of contact of the excircle and incircle to a
side of a triangle lie symmetric with respect to the midpoint of this side and that segments on
two tangents to a circle defined by the points of contact and their point of intersection have
the same length. So we conclude
ZP = ZB + BP = XB + BT = BX + CX = ZS and
CQ = CT = BX = BZ = CS.
A
x x
k
R Y
Z y+z S
p
z
y G y
q
B C
y X T z
ka
Q
Ia
So for each of the points Z, C, their distances to S equal the length of a tangent segment from
this point to ka . It is well-known, that all points with this property lie on the line ZC, which
is the radical axis of S and ka . Similar arguments yield that BY is the radical axis of R and
ka . So the point of intersection of ZC and BY , which is G by definition, is the radical center
of R, S and ka , from which the claim GR = GS follows immediately.
52
50th IMO 2009 Geometry G3
and GS in terms of x, y, and z that can be resolved for GS. The result is symmetric in y and
z, so GR = GS. More in detail this means:
The line BY intersects the sides of triangle AZC, so Menelaos’ theorem yields pq · xz · x+y
y
= 1,
hence
p xy
= . (1)
q yz + zx
Since we only want to show that the term for GS is symmetric in y and z, we abbreviate terms
that are symmetric in y and z by capital letters, starting with N = xy + yz + zx. So (1) implies
p xy xy q yz + zx yz + zx
= = and = = . (2)
p+q xy + yz + zx N p+q xy + yz + zx N
Now the law of Cosines in triangle ABC yields
Now in triangle ZCS the segment GS is a cevian, so with Stewart’s theorem we have
py 2 + q(y + z)2 = (p + q)(GS 2 + pq), hence
p q p q
GS 2 = · y2 + · (y + z)2 − · · (p + q)2 .
p+q p+q p+q p+q
Replacing the p’s and q’s herein by (2) and (3) yields
2 xy 2 yz + zx 2 xy yz + zx 2 4xyz
GS = y + (y + z) − · · z +
N N N N x+y
3 2 2
xy yz(y + z) zx(y + z) xyz (x + y) 4x2 y 2 z 2
3
= + + − −
N | {zN } N N2 | N
2
{z }
M1 M2
3 2 3
xy + zx(y + z) xyz (x + y)
= − + M1 − M2
N N2
x(y 3 + y 2 z + yz 2 + z 3 ) xyz 2 N xyz 3 (x + y)
= + − + M1 − M2
| N
{z } N2 N2
M3
x y z + xy z + x2 yz 3 − x2 yz 3 − xy 2 z 3
2 2 2 2 3
= + M1 − M2 + M3
N2
x2 y 2 z 2
= + M1 − M2 + M3 ,
N2
a term that is symmetric in y and z, indeed.
53
G4 Geometry 50th IMO 2009
Solution 1. It suffices to show that ∠HEF = ∠HGE (see Figure 1), since in circle EGH the
angle over the chord EH at G equals the angle between the tangent at E and EH.
First, ∠BAD = 180 ◦ − ∠DCB = ∠F CD. Since triangles F AB and F CD have also a common
interior angle at F , they are similar.
A
D
E H M F
G
Y
X
C
Figure 1
Denote by T the transformation consisting of a reflection at the bisector of ∠DF C followed
by a dilation with center F and factor of FF CA . Then T maps F to F , C to A, D to B, and H
to G. To see this, note that 4F CA ∼ 4F DB, so FF CA = FF D B
. Moreover, as ∠ADB = ∠ACB,
the image of the line DE under T is parallel to AC (and passes through B) and similarly the
image of CE is parallel to DB and passes through A. Hence E is mapped to the point X which
is the fourth vertex of the parallelogram BEAX. Thus, in particular ∠HEF = ∠F XG.
As G is the midpoint of the diagonal AB of the parallelogram BEAX, it is also the midpoint
of EX. In particular, E, G, X are collinear, and EX = 2 · EG.
Denote by Y the fourth vertex of the parallelogram DECY . By an analogous reasoning as
before, it follows that T maps Y to E, thus E, H, Y are collinear with EY = 2 · EH.
Therefore, by the intercept theorem, HG k XY .
From the construction of T it is clear that the lines F X and F E are symmetric with respect
to the bisector of ∠DF C, as are F Y and F E. Thus, F , X, Y are collinear, which together
with HG k XY implies ∠F XE = ∠HGE. This completes the proof.
54
50th IMO 2009 Geometry G4
Solution 2. We use the following
Lemma (Gauß). Let ABCD be a quadrilateral. Let AB and CD intersect at P , and BC
and DA intersect at Q. Then the midpoints K, L, M of AC, BD, and P Q, respectively, are
collinear.
Proof: Let us consider the points Z that fulfill the equation
where (RST ) denotes the oriented area of the triangle RST (see Figure 2).
P
A
D
K
L Q
C
B
Figure 2
As (1) is linear in Z, it can either characterize a line, or be contradictory, or be trivially fulfilled
for all Z in the plane. If (1) was fulfilled for all Z, then it would hold for Z = A, Z = B, which
gives (CDA) = (BCA), (CDB) = (DAB), respectively, i.e. the diagonals of ABCD would
bisect each other, thus ABCD would be a parallelogram. This contradicts the hypothesis that
AD and BC intersect. Since E, F, G fulfill (1), it is the equation of a line which completes the
proof of the lemma.
Now consider the parallelograms EAXB and ECY D (see Figure 1). Then G, H are the
midpoints of EX, EY , respectively. Let M be the midpoint of EF . By applying the Lemma to
the (re-entrant) quadrilateral ADBC, it is evident that G, H, and M are collinear. A dilation
by a factor of 2 with center E shows that X, Y , F are collinear. Since AX k DE and BX k CE,
we have pairwise equal interior angles in the quadrilaterals F DEC and F BXA. Since we have
also ∠EBA = ∠DCA = ∠CDY , the quadrilaterals are similar. Thus, ∠F XA = ∠CEF .
Clearly the parallelograms ECY D and EBXA are similar, too, thus ∠EXA = ∠CEY . Con-
sequently, ∠F XE = ∠F XA − ∠EXA = ∠CEF − ∠CEY = ∠Y EF . By the converse of the
tangent-chord angle theorem EF is tangent to the circle XEY . A dilation by a factor of 21
completes the proof.
55
G4 Geometry 50th IMO 2009
hence
(e − a)(e − c) = e2 − R2 , (3)
(e − b)(e − d) = e2 − R2 , (4)
(f − a)(f − d) = f 2 − R2 , (5)
2 2
(f − b)(f − c) = f − R . (6)
4 ef = 4R2 . (7)
56
50th IMO 2009 Geometry G5
G5 POL (Poland)
Let P be a polygon that is convex and symmetric to some point O. Prove that for some
parallelogram R satisfying P ⊂ R we have
|R| √
≤ 2
|P |
where |R| and |P | denote the area of the sets R and P , respectively.
Solution 1. We will construct two parallelograms √ R1 and R3 , each√of them containing P , and
prove that at least one of the inequalities |R1 | ≤ 2 |P | and |R3 | ≤ 2 |P | holds (see Figure 1).
First we will construct a parallelogram R1 ⊇ P with the property that the midpoints of the
sides of R1 are points of the boundary of P .
Choose two points A and B of P such that the triangle OAB has maximal area. Let a be the
line through A parallel to OB and b the line through B parallel to OA. Let A0 , B 0 , a0 and b0 be
the points or lines, that are symmetric to A, B, a and b, respectively, with respect to O. Now
let R1 be the parallelogram defined by a, b, a0 and b0 .
R3
a A C
R1 X′ *
X
R2
X
B′ B
a* O
Y
b′ Y′
b
a′ A′ D
Figure 1
Obviously, A and B are located on the boundary of the polygon P , and A, B, A0 and B 0 are
midpoints of the sides of R1 . We note that P ⊆ R1 . Otherwise, there would be a point Z ∈ P
but Z ∈ / R1 , i.e., one of the lines a, b, a0 or b0 were between O and Z. If it is a, we have
|OZB| > |OAB|, which is contradictory to the choice of A and B. If it is one of the lines b, a0
or b0 almost identical arguments lead to a similar contradiction.
Let R2 be the parallelogram ABA0 B 0 . Since A and B are points of P , segment AB ⊂ P and
so R2 ⊂ R1 . Since A, B, A0 and B 0 are midpoints of the sides of R1 , an easy argument yields
Let R3 be the smallest parallelogram enclosing P defined by lines parallel to AB and BA0 .
Obviously R2 ⊂ R3 and every side of R3 contains at least one point of the boundary of P .
Denote by C the intersection point of a and b, by X the intersection point of AB and OC, and
by X 0 the intersection point of XC and the boundary of R3 . In a similar way denote by D
57
G5 Geometry 50th IMO 2009
the intersection point of b and a0 , by Y the intersection point of A0 B and OD, and by Y 0 the
intersection point of Y D and the boundary of R3 .
Note that OC = 2·OX and OD = 2·OY , so there exist real numbers x and y with 1 ≤ x, y ≤ 2
and OX 0 = x · OX and OY 0 = y · OY . Corresponding sides of R3 and R2 are parallel which
yields
The side of R3 containing X 0 contains at least one point X ∗ of P ; due to the convexity of
P we have AX ∗ B ⊂ P . Since this side of the parallelogram R3 is parallel to AB we have
|AX ∗ B| = |AX 0 B|, so |OAX 0 B| does not exceed the area of P confined to the sector defined
by the rays OB and OA. In a similar way we conclude that |OB 0 Y 0 A0 | does not exceed the
area of P confined to the sector defined by the rays OB and OA0 . Putting things together we
have |OAX 0 B| = x · |OAB|, |OBDA0 | = y · |OBA0 |. Since |OAB| = |OBA0 |, we conclude that
|P | ≥ 2 · |AX 0 BY 0 A0 | = 2 · (x · |OAB| + y · |OBA0 |) = 4 · x+y
2
· |OAB| = x+y
2
· R2 ; this is in short
x+y
· |R2 | ≤ |P |. (3)
2
Since all numbers concerned are positive, we can combine (1)–(3). Using the arithmetic-
geometric-mean inequality we obtain
2
2 x+y
|R1 | · |R3 | = 2 · |R2 | · xy · |R2 | ≤ 2 · |R2 | ≤ 2 · |P |2 .
2
√ √
This implies immediately the desired result |R1 | ≤ 2 · |P | or |R3 | ≤ 2 · |P |.
B′ B
R2
a
A′
Figure 2
Recall that affine one-to-one maps of the plane preserve the ratio of areas of subsets of the
plane. On the other hand, every parallelogram can be transformed with an affine map onto
a square. It follows that without loss of generality we may assume that R1 is a square (see
Figure 2).
Then R2 , whose vertices are the midpoints of the sides of R1 , is a square too, and R3 , whose
sides are parallel to the diagonals of R1 , is a rectangle.
Let a > 0, b ≥ 0 and c ≥ 0 be the distances introduced in Figure 2. Then |R1 | = 2a2 and
58
50th IMO 2009 Geometry G5
|R3 | = (a + 2b)(a + 2c).
Points A, A0 , B and B 0 are in the convex polygon P . Hence the square ABA0 B 0 is a subset of
P . Moreover, each of the sides of the rectangle R3 contains a point of P , otherwise R3 would
not be minimal. It follows that
ab ac
|P | ≥ a2 + 2 · +2· = a(a + b + c).
2 2
|R1 |
√ √
Now assume that both |P |
> 2 and |R 3|
|P |
> 2, then
√ √
2a2 = |R1 | > 2 · |P | ≥ 2 · a(a + b + c)
and √ √
(a + 2b)(a + 2c) = |R3 | > 2 · |P | ≥ 2 · a(a + b + c).
All numbers concerned are positive, so after multiplying these inequalities we get
59
G6 Geometry 50th IMO 2009
G6 UKR (Ukraine)
Let the sides AD and BC of the quadrilateral ABCD (such that AB is not parallel to CD)
intersect at point P . Points O1 and O2 are the circumcenters and points H1 and H2 are the
orthocenters of triangles ABP and DCP , respectively. Denote the midpoints of segments
O1 H1 and O2 H2 by E1 and E2 , respectively. Prove that the perpendicular from E1 on CD, the
perpendicular from E2 on AB and the line H1 H2 are concurrent.
Solution 1. We keep triangle ABP fixed and move the line CD parallel to itself uniformly,
i.e. linearly dependent on a single parameter λ (see Figure 1). Then the points C and D also
move uniformly. Hence, the points O2 , H2 and E2 move uniformly, too. Therefore also the
perpendicular from E2 on AB moves uniformly. Obviously, the points O1 , H1 , E1 and the
perpendicular from E1 on CD do not move at all. Hence, the intersection point S of these
two perpendiculars moves uniformly. Since H1 does not move, while H2 and S move uniformly
along parallel lines (both are perpendicular to CD), it is sufficient to prove their collinearity
for two different positions of CD.
P
O2
D H2 E2
O1
S C
E1
H1
A B
Figure 1
Let CD pass through either point A or point B. Note that by hypothesis these two cases
are different. We will consider the case A ∈ CD, i.e. A = D. So we have to show that the
perpendiculars from E1 on AC and from E2 on AB intersect on the altitude AH of triangle
ABC (see Figure 2).
60
50th IMO 2009 Geometry G6
P
O1
E1
O2
E2
S H
H2 H1
B
A=D C1
A1
B1
C
Figure 2
To this end, we consider the midpoints A1 , B1 , C1 of BC, CA, AB, respectively. As E1 is the
center of Feuerbach’s circle (nine-point circle) of 4ABP , we have E1 C1 = E1 H. Similarly,
E2 B1 = E2 H. Note further that a point X lies on the perpendicular from E1 on A1 C1 if and
only if
XC12 − XA21 = E1 C12 − E1 A21 .
Similarly, the perpendicular from E2 on A1 B1 is characterized by
61
G6 Geometry 50th IMO 2009
E1 A21 − E2 A21 − E1 H 2 + E2 H 2
= F1 A21 − F1 H 2 − F2 A21 + F2 H 2
= (F1 A1 − F1 H)(F1 A1 + F1 H) − (F2 A1 − F2 H)(F2 A1 + F2 H)
= A1 H · (A1 P1 − A1 P2 )
A1 H · BC
=
2
AC 2 − AB 2
= ,
4
which proves the claim.
Q
H1
ϕ
B
X C
M
E1
β
E2
ψ
P α D A
Y N
H2
Figure 3
We will prove now that triangles E1 XM and E2 Y N have equal angles at E1 , E2 , and supple-
mentary angles at X, Y .
In the following, angles are understood as oriented, and equalities of angles modulo 180◦ .
Let α = ∠H2 P D, ψ = ∠DP C, β = ∠CP H1 . Then α + ψ + β = ϕ, ∠E1 XH1 = ∠H2 Y E2 = ϕ,
thus ∠M XE1 + ∠N Y E2 = 180◦ .
By considering the Feuerbach circle of 4ABP whose center is E1 and which goes through M ,
we have ∠E1 M H1 = ψ + 2β. Analogous considerations with the Feuerbach circle of 4DCP
yield ∠H2 N E2 = ψ + 2α. Hence indeed ∠XE1 M = ϕ − (ψ + 2β) = (ψ + 2α) − ϕ = ∠Y E2 N .
It follows now that
XM YN
= .
M E1 N E2
Furthermore, M E1 is half the circumradius of 4ABP , while P H1 is the distance of P to the
orthocenter of that triangle, which is twice the circumradius times the cosine of ψ. Together
62
50th IMO 2009 Geometry G6
with analogous reasoning for 4DCP we have
M E1 1 N E2
= = .
P H1 4 cos ψ P H2
By multiplication,
XM YN
= ,
P H1 P H2
and therefore
PX H2 Y
= .
XH1 YP
Let E1 X, E2 Y meet H1 H2 in R, S respectively.
Applying the intercept theorem to the parallels E1 X, P H2 and center H1 gives
H2 R PX
= ,
RH1 XH1
while with parallels E2 Y , P H1 and center H2 we obtain
H2 S H2 Y
= .
SH1 YP
Combination of the last three equalities yields that R and S coincide.
63
G7 Geometry 50th IMO 2009
Solution. AZ, AI and AY divide ∠BAC into four equal angles; denote them by α. In
the same way we have four equal angles β at B and four equal angles γ at C. Obviously
◦
α + β + γ = 180
4
= 45 ◦; and 0 ◦ < α, β, γ < 45 ◦.
A
Z Y
I
γ
X
β
B C
Easy calculations in various triangles yield ∠BIC = 180 ◦ − 2β − 2γ = 180 ◦ − (90 ◦ − 2α) =
90 ◦ + 2α, hence (for X is the incenter of triangle BCI, so IX bisects ∠BIC) we have ∠XIC =
∠BIX = 12 ∠BIC = 45 ◦ + α and with similar aguments ∠CIY = ∠Y IA = 45 ◦ + β and
∠AIZ = ∠ZIB = 45 ◦ + γ. Furthermore, we have ∠XIY = ∠XIC + ∠CIY = (45 ◦ + α) +
(45 ◦ + β) = 135 ◦ − γ, ∠Y IZ = 135 ◦ − α, and ∠ZIX = 135 ◦ − β.
Now we calculate the lengths of IX, IY and IZ in terms of α, β and γ. The perpendicular
from I on CX has length IX · sin ∠CXI = IX · sin (90 ◦ + β) = IX · cos β. But CI bisects
∠Y CX, so the perpendicular from I on CY has the same length, and we conclude
IX · cos β = IY · cos α.
To make calculations easier we choose a length unit that makes IX = cos α. Then IY = cos β
and with similar arguments IZ = cos γ.
Since XY Z is equilateral we have ZX = ZY . The law of Cosines in triangles XY I, Y ZI yields
ZX 2 = ZY 2
=⇒ IZ 2 + IX 2 − 2 · IZ · IX · cos ∠ZIX = IZ 2 + IY 2 − 2 · IZ · IY · cos ∠Y IZ
=⇒ IX 2 − IY 2 = 2 · IZ · (IX · cos ∠ZIX − IY · cos ∠Y IZ)
=⇒ cos 2 α − cos 2 β = 2 · cos γ · (cos α · cos (135◦ − β) − cos β · cos (135◦ − α)) .
| {z } | {z }
L.H.S. R.H.S.
64
50th IMO 2009 Geometry G7
= (cos α · sin β + cos β · sin α) · (cos α · sin β − cos β · sin α)
= sin (β + α) · sin (β − α) = sin (45◦ − γ) · sin (β − α)
R.H.S. = 2 · cos γ · (cos α · (− cos (45◦ + β)) − cos β · (− cos (45◦ + α)))
√
2
=2· · cos γ · (cos α · (sin β − cos β) + cos β · (cos α − sin α))
√ 2
= 2 · cos γ · (cos α · sin β − cos β · sin α)
√
= 2 · cos γ · sin (β − α).
65
G8 Geometry 50th IMO 2009
G8 BGR (Bulgaria)
Let ABCD be a circumscribed quadrilateral. Let g be a line through A which meets the
segment BC in M and the line CD in N . Denote by I1 , I2 , and I3 the incenters of 4ABM ,
4M N C, and 4N DA, respectively. Show that the orthocenter of 4I1 I2 I3 lies on g.
Solution 1. Let k1 , k2 and k3 be the incircles of triangles ABM , M N C, and N DA, respec-
tively (see Figure 1). We shall show that the tangent h from C to k1 which is different from
CB is also tangent to k3 .
B
k1
A
h
g
I1
k3
X
H I3
M L1
L3 D
k2 I2
C
N
Figure 1
To this end, let X denote the point of intersection of g and h. Then ABCX and ABCD are
circumscribed quadrilaterals, whence
i.e.
AX + CD = CX + AD
which in turn reveals that the quadrilateral AXCD is also circumscribed. Thus h touches
indeed the circle k3 .
Moreover, we find that ∠I3 CI1 = ∠I3 CX + ∠XCI1 = 21 (∠DCX + ∠XCB) = 12 ∠DCB =
1
2
(180◦ − ∠M CN ) = 180◦ − ∠M I2 N = ∠I3 I2 I1 , from which we conclude that C, I1 , I2 , I3 are
concyclic.
Let now L1 and L3 be the reflection points of C with respect to the lines I2 I3 and I1 I2 respec-
tively. Since I1 I2 is the angle bisector of ∠N M C, it follows that L3 lies on g. By analogous
reasoning, L1 lies on g.
Let H be the orthocenter of 4I1 I2 I3 . We have ∠I2 L3 I1 = ∠I1 CI2 = ∠I1 I3 I2 = 180◦ − ∠I1 HI2 ,
which entails that the quadrilateral I2 HI1 L3 is cyclic. Analogously, I3 HL1 I2 is cyclic.
66
50th IMO 2009 Geometry G8
Then, working with oriented angles modulo 180 ◦, we have
Comment. The last part of the argument essentially reproves the following fact: The Simson
line of a point P lying on the circumcircle of a triangle ABC with respect to that triangle bisects
the line segment connecting P with the orthocenter of ABC.
I1
Z I
I3
M
δ
D
g I2 γ
C
N
Figure 2
To this end, notice first that I2 , M , I1 are collinear, as are N , I2 , I3 (see Figure 2). Denote by
α, β, γ, δ the internal angles of ABCD. By considerations in triangle CM N , it follows that
∠I3 I2 I1 = γ2 . We will show that ∠I3 CI1 = γ2 , too. Denote by I the incenter of ABCD. Clearly,
I1 ∈ BI, I3 ∈ DI, ∠I1 AI3 = α2 .
Using the abbreviation [X, Y Z] for the distance from point X to the line Y Z, we have because
of ∠BAI1 = ∠IAI3 and ∠I1 AI = ∠I3 AD that
[I1 , AB] [I3 , AI]
= .
[I1 , AI] [I3 , AD]
Furthermore, consideration of the angle sums in AIB, BIC, CID and DIA implies ∠AIB +
∠CID = ∠BIC + ∠DIA = 180◦ , from which we see
Because of [I1 , AB] = [I1 , BC], [I3 , AD] = [I3 , CD], multiplication yields
By ∠DCI = ∠ICB = γ/2 it follows that ∠I1 CB = ∠I3 CI which concludes the proof of the
67
G8 Geometry 50th IMO 2009
above statement.
68
50th IMO 2009 Number Theory N1
Number Theory
N1 AUS (Australia)
A social club has n members. They have the membership numbers 1, 2, . . . , n, respectively.
From time to time members send presents to other members, including items they have already
received as presents from other members. In order to avoid the embarrassing situation that a
member might receive a present that he or she has sent to other members, the club adds the
following rule to its statutes at one of its annual general meetings:
“A member with membership number a is permitted to send a present to a member with
membership number b if and only if a(b − 1) is a multiple of n.”
Prove that, if each member follows this rule, none will receive a present from another member
that he or she has already sent to other members.
gcd(i1 , n)| gcd(ir , n)| gcd(ir−1 , n)| . . . | gcd(i2 , n)| gcd(i1 , n),
which implies that all these greatest common divisors must be equal, say be equal to t.
Now we pick any of the ik , without loss of generality let it be i1 . Then ir (i1 − 1) is a multiple of
n and hence also (by dividing by t), i1 − 1 is a multiple of nt . Since i1 and i1 − 1 are relatively
prime, also t and nt are relatively prime. So, by the Chinese remainder theorem, the value of
i1 is uniquely determined modulo n = t · nt by the value of t. But, as i1 was chosen arbitrarily
among the ik , this implies that all the ik have to be equal, a contradiction.
Solution 2. If a, b, c are integers such that ab − a and bc − b are multiples of n, then also
ac − a = a(bc − b) + (ab − a) − (ab − a)c is a multiple of n. This implies that if there is an
edge from va to vb and an edge from vb to vc , then there also must be an edge from va to vc .
Therefore, if there are any cycles at all, the smallest cycle must have length 2. But suppose
the vertices va and vb form such a cycle, i. e., ab − a and ab − b are both multiples of n. Then
a − b is also a multiple of n, which can only happen if a = b, which is impossible.
Solution 3. Suppose there was a cycle vi1 → vi2 → · · · → vir → vi1 . Then i1 (i2 − 1)
is a multiple of n, i. e., i1 ≡ i1 i2 mod n. Continuing in this manner, we get i1 ≡ i1 i2 ≡
i1 i2 i3 ≡ i1 i2 i3 . . . ir mod n. But the same holds for all ik , i. e., ik ≡ i1 i2 i3 . . . ir mod n. Hence
i1 ≡ i2 ≡ · · · ≡ ir mod n, which means i1 = i2 = · · · = ir , a contradiction.
69
N1 Number Theory 50th IMO 2009
Solution 4. Let n = k be the smallest value of n for which the corresponding graph has a
cycle. We show that k is a prime power.
If k is not a prime power, it can be written as a product k = de of relatively prime integers
greater than 1. Reducing all the numbers modulo d yields a single vertex or a cycle in the
corresponding graph on d vertices, because if a(b − 1) ≡ 0 mod k then this equation also holds
modulo d. But since the graph on d vertices has no cycles, by the minimality of k, we must
have that all the indices of the cycle are congruent modulo d. The same holds modulo e and
hence also modulo k = de. But then all the indices are equal, which is a contradiction.
Thus k must be a prime power k = pm . There are no edges ending at vk , so vk is not contained
in any cycle. All edges not starting at vk end at a vertex belonging to a non-multiple of p, and
all edges starting at a non-multiple of p must end at v1 . But there is no edge starting at v1 .
Hence there is no cycle.
Solution 5. Suppose there was a cycle vi1 → vi2 → · · · → vir → vi1 . Let q = pm be a prime
power dividing n. We claim that either i1 ≡ i2 ≡ · · · ≡ ir ≡ 0 mod q or i1 ≡ i2 ≡ · · · ≡ ir ≡
1 mod q.
Suppose that there is an is not divisible by q. Then, as is (is+1 − 1) is a multiple of q, is+1 ≡
1 mod p. Similarly, we conclude is+2 ≡ 1 mod p and so on. So none of the labels is divisible by
p, but since is (is+1 − 1) is a multiple of q = pm for all s, all is+1 are congruent to 1 modulo q.
This proves the claim.
Now, as all the labels are congruent modulo all the prime powers dividing n, they must all be
equal by the Chinese remainder theorem. This is a contradiction.
70
50th IMO 2009 Number Theory N2
N2 PER (Peru)
A positive integer N is called balanced, if N = 1 or if N can be written as a product of an
even number of not necessarily distinct primes. Given positive integers a and b, consider the
polynomial P defined by P (x) = (x + a)(x + b).
(a) Prove that there exist distinct positive integers a and b such that all the numbers P (1), P (2),
. . . , P (50) are balanced.
(b) Prove that if P (n) is balanced for all positive integers n, then a = b.
Solution. Define a function f on the set of positive integers by f (n) = 0 if n is balanced and
f (n) = 1 otherwise. Clearly, f (nm) ≡ f (n) + f (m) mod 2 for all positive integers n, m.
(a) Now for each positive integer n consider the binary sequence (f (n+1), f (n+2), . . . , f (n+
50)). As there are only 250 different such sequences there are two different positive integers
a and b such that
But this implies that for the polynomial P (x) = (x+a)(x+b) all the numbers P (1), P (2),
. . . , P (50) are balanced, since for all 1 ≤ k ≤ 50 we have f (P (k)) ≡ f (a + k) + f (b + k) ≡
2f (a + k) ≡ 0 mod 2.
(b) Now suppose P (n) is balanced for all positive integers n and a < b. Set n = k(b − a) − a
for sufficiently large k, such that n is positive. Then P (n) = k(k + 1)(b − a)2 , and this
number can only be balanced, if f (k) = f (k + 1) holds. Thus, the sequence f (k) must
become constant for sufficiently large k. But this is not possible, as for every prime p we
have f (p) = 1 and for every square t2 we have f (t2 ) = 0.
Hence a = b.
Comment. Given a positive integer k, a computer search for the pairs of positive integers
(a, b), for which P (1), P (2), . . . , P (k) are all balanced yields the following results with
minimal sum a + b and a < b:
k 3 4 5 10 20
(a, b) (2, 4) (6, 11) (8, 14) (20, 34) (1751, 3121)
Therefore, trying to find a and b in part (a) of the problem cannot be done by elementary
calculations.
71
N3 Number Theory 50th IMO 2009
N3 EST (Estonia)
Let f be a non-constant function from the set of positive integers into the set of positive integers,
such that a − b divides f (a) − f (b) for all distinct positive integers a, b. Prove that there exist
infinitely many primes p such that p divides f (c) for some positive integer c.
Solution 1. Denote by vp (a) the exponent of the prime p in the prime decomposition of a.
Assume that there are only finitely many primes p1 , p2 , . . . , pm that divide some function value
produced of f .
There are infinitely many positive integers a such that vpi (a) > vpi (f (1)) for all i = 1, 2, . . . , m,
e.g. a = (p1 p2 . . . pm )α with α sufficiently large. Pick any such a. The condition of the problem
then yields a| (f (a + 1) − f (1)). Assume f (a + 1) 6= f (1). Then we must have vpi (f (a + 1)) 6=
vpi (f (1)) for at least one i. This yields vpi (f (a + 1) − f (1)) = min {vpi (f (a + 1)), vpi (f (1))} ≤
vp1 (f (1)) < vpi (a). But this contradicts the fact that a| (f (a + 1) − f (1)).
Hence we must have f (a + 1) = f (1) for all such a.
Now, for any positive integer b and all such a, we have (a + 1 − b)|(f (a + 1) − f (b)), i.e.,
(a + 1 − b)|(f (1) − f (b)). Since this is true for infinitely many positive integers a we must have
f (b) = f (1). Hence f is a constant function, a contradiction. Therefore, our initial assumption
was false and there are indeed infinitely many primes p dividing f (c) for some positive integer
c.
Solution 2. Assume that there are only finitely many primes p1 , p2 , . . . , pm that divide some
function value of f . Since f is not identically 1, we must have m ≥ 1.
Then there exist non-negative integers α1 , . . . , αm such that
Then for all i ∈ {1, . . . , m} we have that pαi i +1 divides M − 1 and hence by the condition of the
problem also f (M ) − f (1). This implies that f (M ) is divisible by pαi i but not by pαi i +1 for all i
and therefore f (M ) = f (1).
Hence
But since M − r divides f (M ) − f (r) this can only be true if f (r) = f (M ) = f (1), which
contradicts the choice of r.
Comment. In the case that f is a polynomial with integer coefficients the result is well-known,
see e.g. W. Schwarz, Einführung in die Methoden der Primzahltheorie, 1969.
72
50th IMO 2009 Number Theory N4
N4 PRK (Democratic People’s Republic of Korea)
Find all positive integers n such that there exists a sequence of positive integers a1 , a2 , . . . , an
satisfying
a2 + 1
ak+1 = k −1
ak−1 + 1
for every k with 2 ≤ k ≤ n − 1.
Solution 1. Such a sequence exists for n = 1, 2, 3, 4 and no other n. Since the existence of
such a sequence for some n implies the existence of such a sequence for all smaller n, it suffices
to prove that n = 5 is not possible and n = 4 is possible.
Assume first that for n = 5 there exists a sequence of positive integers a1 , a2 , . . . , a5 satisfying
the conditions
Assume a1 is odd, then a2 has to be odd as well and as then a22 + 1 ≡ 2 mod 4, a3 has to be
even. But this is a contradiction, since then the even number a2 + 1 cannot divide the odd
number a23 + 1.
Hence a1 is even.
If a2 is odd, a23 + 1 is even (as a multiple of a2 + 1) and hence a3 is odd, too. Similarly we must
have a4 odd as well. But then a23 + 1 is a product of two even numbers (a2 + 1)(a4 + 1) and
thus is divisible by 4, which is a contradiction as for odd a3 we have a23 + 1 ≡ 2 mod 4.
Hence a2 is even. Furthermore a3 +1 divides the odd number a22 +1 and so a3 is even. Similarly,
a4 and a5 are even as well.
Now set x = a2 and y = a3 . From the given condition we get (x+1)|(y 2 +1) and (y +1)|(x2 +1).
We will prove that there is no pair of positive even numbers (x, y) satisfying these two conditions,
thus yielding a contradiction to the assumption.
Assume there exists a pair (x0 , y0 ) of positive even numbers satisfying the two conditions
(x0 + 1)|(y02 + 1) and (y0 + 1)|(x20 + 1).
Then one has (x0 + 1)|(y02 + 1 + x20 − 1), i.e., (x0 + 1)|(x20 + y02 ), and similarly (y0 + 1)|(x20 + y02 ).
Any common divisor d of x0 + 1 and y0 + 1 must hence also divide the number
(x20 + 1) + (y02 + 1) − (x20 + y02 ) = 2. But as x0 + 1 and y0 + 1 are both odd, we must have d = 1.
Thus x0 + 1 and y0 + 1 are relatively prime and therefore there exists a positive integer k such
that
k(x + 1)(y + 1) = x2 + y 2
has the solution (x0 , y0 ). We will show that the latter equation has no solution (x, y) in positive
even numbers.
Assume there is a solution. Pick the solution (x1 , y1 ) with the smallest sum x1 + y1 and assume
x1 ≥ y1 . Then x1 is a solution to the quadratic equation
73
N4 Number Theory 50th IMO 2009
Let x2 be the second solution, which by Vieta’s theorem fulfills x1 + x2 = k(y1 + 1) and
x1 x2 = y12 − k(y1 + 1). If x2 = 0, the second equation implies y12 = k(y1 + 1), which is
impossible, as y1 + 1 > 1 cannot divide the relatively prime number y12 . Therefore x2 6= 0.
Also we get (x1 + 1)(x2 + 1) = x1 x2 + x1 + x2 + 1 = y12 + 1 which is odd, and hence x2 must
y 2 +1 y 2 +1
be even and positive. Also we have x2 + 1 = x11 +1 ≤ y11 +1 ≤ y1 ≤ x1 . But this means that the
pair (x0 , y 0 ) with x0 = y1 and y 0 = x2 is another solution of k(x + 1)(y + 1) = x2 + y 2 in even
positive numbers with x0 + y 0 < x1 + y1 , a contradiction.
Therefore we must have n ≤ 4.
When n = 4, a possible example of a sequence is a1 = 4, a2 = 33, a3 = 217 and a4 = 1384.
Solution 2. It is easy to check that for n = 4 the sequence a1 = 4, a2 = 33, a3 = 217 and
a4 = 1384 is possible.
Now assume there is a sequence with n ≥ 5. Then we have in particular
Also assume without loss of generality that among all such quintuples (a1 , a2 , a3 , a4 , a5 ) we have
chosen one with minimal a1 .
One shows quickly the following fact:
If three positive integers x, y, z fulfill y 2 + 1 = (x + 1)(z + 1) and if y is even, then
x and z are even as well and either x < y < z or z < y < x holds. (1)
Indeed, the first part is obvious and from x < y we conclude
y2 + 1 y2 + 1
z+1= ≥ > y,
x+1 y
and similarly in the other case.
Now, if a3 was odd, then (a2 + 1)(a4 + 1) = a23 + 1 ≡ 2 mod 4 would imply that one of a2 or
a4 is even, this contradicts (1). Thus a3 and hence also a1 , a2 , a4 and a5 are even. According
to (1), one has a1 < a2 < a3 < a4 < a5 or a1 > a2 > a3 > a4 > a5 but due to the minimality of
a1 the first series of inequalities must hold.
Consider the identity
(a3 +1)(a1 +a3 ) = a23 −1+(a1 +1)(a3 +1) = a22 +a23 = a22 −1+(a2 +1)(a4 +1) = (a2 +1)(a2 +a4 ).
Any common divisor of the two odd numbers a2 + 1 and a3 + 1 must also divide (a2 + 1)(a4 +
1) − (a3 + 1)(a3 − 1) = 2, so these numbers are relatively prime. Hence the last identity shows
that a1 + a3 must be a multiple of a2 + 1, i.e. there is an integer k such that
74
50th IMO 2009 Number Theory N4
Thus a0 ≥ 0. If a0 > 0, then by (1) we would have a0 < a1 < a2 and then the quintuple
(a0 , a1 , a2 , a3 , a4 ) would contradict the minimality of a1 .
Hence a0 = 0, implying a2 = a21 . But also a2 = k(a1 + 1), which finally contradicts the fact
that a1 + 1 > 1 is relatively prime to a21 and thus cannot be a divisior of this number.
Hence n ≥ 5 is not possible.
Comment 1. Finding the example for n = 4 is not trivial and requires a tedious calculation,
but it can be reduced to checking a few cases. The equations (a1 + 1)(a3 + 1) = a22 + 1 and
(a2 + 1)(a4 + 1) = a23 + 1 imply, as seen in the proof, that a1 is even and a2 , a3 , a4 are odd. The
case a1 = 2 yields a22 ≡ −1 mod 3 which is impossible. Hence a1 = 4 is the smallest possibility.
In this case a22 ≡ −1 mod 5 and a2 is odd, which implies a2 ≡ 3 or a2 ≡ 7 mod 10. Hence we
have to start checking a2 = 7, 13, 17, 23, 27, 33 and in the last case we succeed.
Comment 2. The choice of a0 = k(a1 + 1) − a2 in the second solution appears more natural if
one considers that by the previous calculations one has a1 = k(a2 +1)−a3 and a2 = k(a3 +1)−a4 .
Alternatively, one can solve the equation (2) for a3 and use a22 + 1 = (a1 + 1)(a3 + 1) to get
a22 − k(a1 + 1)a2 + a21 − k(a1 + 1) = 0. Now a0 is the second solution to this quadratic equation
in a2 (Vieta jumping).
75
N5 Number Theory 50th IMO 2009
N5 HUN (Hungary)
Let P (x) be a non-constant polynomial with integer coefficients. Prove that there is no function
T from the set of integers into the set of integers such that the number of integers x with
T n (x) = x is equal to P (n) for every n ≥ 1, where T n denotes the n-fold application of T .
Solution 1. Assume there is a polynomial P of degree at least 1 with the desired property
for a given function T . Let A(n) denote the set of all x ∈ Z such that T n (x) = x and let
B(n) denote the set of all x ∈ Z for which T n (x) = x and T k (x) 6= x for all 1 ≤ k < n. Both
sets are finite under the assumption made. For each x ∈ A(n) there is a smallest k ≥ 1 such
that T k (x) = x, i.e., x ∈ B(k). Let d = gcd(k, n). There are positive integers r, s such that
rk − sn = d and hence x = T rk (x) = T sn+d (x) = T d (T sn (x)) = T d (x). The minimality of k
implies d = k, S i.e., k|n. On the other hand one clearly has B(k) ⊂ A(n) if k|n and thus we
have A(n) = d|n B(d) as a disjoint union and hence
X
|A(n)| = |B(d)|.
d|n
Furthermore, for every x ∈ B(n) the elements x, T 1 (x), T 2 (x), . . . , T n−1 (x) are n distinct
elements of B(n). The fact that they are in A(n) is obvious. If for some k < n and
some 0 ≤ i < n we had T k (T i (x)) = T i (x), i.e. T k+i (x) = T i (x), that would imply
x = T n (x) = T n−i (T i (x)) = T n−i (T k+i (x)) = T k (T n (x)) = T k (x) contradicting the minimality
of n. Thus T i (x) ∈ B(n) and T i (x) 6= T j (x) for 0 ≤ i < j ≤ n − 1.
So indeed, T permutes the elements of B(n) in (disjoint) cycles of length n and in particular
one has n|B(n)|.
Now let P (x) = ki=0 ai xi , ai ∈ Z, k ≥ 1, ak 6= 0 and suppose that |A(n)| = P (n) for all n ≥ 1.
P
Let p be any prime. Then
p2 |B(p2 )| = |A(p2 )| − |A(p)| = a1 (p2 − p) + a2 (p4 − p2 ) + . . .
Hence p|a1 and since this is true for all primes we must have a1 = 0.
Now consider any two different primes p and q. Since a1 = 0 we have that
This implies
p2 q |B(p2 )| = |A(p2 )| − |A(p)| = a2 (p4 − p2 ) + a3 (p6 − p3 ) + · · · + ak (p2k − pk ).
Since this is true for every prime q we must have a2 (p4 − p2 ) + a3 (p6 − p3 ) + · · · + ak (p2k − pk ) = 0
for every prime p. Since this expression is a polynomial in p of degree 2k (because ak 6= 0) this
is a contradiction, as such a polynomial can have at most 2k zeros.
76
50th IMO 2009 Number Theory N5
Comment. The last contradiction can also be reached via
1 4 2 6 3 2k k
ak = lim a 2 (p − p ) + a 3 (p − p ) + · · · + a k (p − p ) = 0.
p→∞ p2k
Solution 2. As in the first solution define A(n) and B(n) and assume that a polynomial P
with the required property exists. This again implies that |A(n)| and |B(n)| is finite for all
positive integers n and that
X
P (n) = |A(n)| = |B(d)| and n|B(n)|.
d|n
Thus, for any fixed p, the expression P (0) − |B(1)| − |B(p)| is divisible by arbitrarily large
primes q which means that P (0) = |B(1)| + |B(p)| = P (p) for any prime p. This implies that
the polynomial P is constant, a contradiction.
77
N6 Number Theory 50th IMO 2009
N6 TUR (Turkey)
Let k be a positive integer. Show that if there exists a sequence a0 , a1 , . . . of integers satisfying
the condition
an−1 + nk
an = for all n ≥ 1,
n
then k − 2 is divisible by 3.
Solution 1. Part A. For each positive integer k, there exists a polynomial Pk of degree k − 1
with integer coefficients, i. e., Pk ∈ Z[x], and an integer qk such that the polynomial identity
and determine the coefficients bk−1 , bk−2 , . . . , b0 and the number qk successively. Obviously, we
have bk−1 = 1. For m = k − 1, k − 2, . . . , 1, comparing the coefficients of xm in the identity (Ik )
results in an expression of bm−1 as an integer linear combination of bk−1 , . . . , bm , and finally
qk = −Pk (−1).
Part B. Let k be a positive integer, and let a0 , a1 , . . . be a sequence of real numbers satisfying
the recursion given in the problem. This recursion can be written as
an−1 − Pk (n − 1) qk
an − Pk (n) = − for all n ≥ 1,
n n
which by induction gives
n−1
a0 − Pk (0) X i!
an − Pk (n) = − qk for all n ≥ 1.
n! i=0
n!
a0 = Pk (0) and qk = 0.
Part C. Multiplying the identity (Ik ) by x2 + x and subtracting the identities (Ik+1 ), (Ik+2 ) and
qk x2 = qk x2 therefrom, we obtain
xTk (x) = Tk (x − 1) + 2x Pk (x − 1) + qk − (qk+2 + qk+1 + qk ),
where the polynomials Tk ∈ Z[x] are defined by Tk (x) = (x2 +x)Pk (x)−Pk+1 (x)−Pk+2 (x)−qk x.
Thus
78
50th IMO 2009 Number Theory N6
Comparing the degrees, we easily see that this is only possible if Tk is the zero polynomial
modulo 2, and
Solution 2. Part A and B. Let k be a positive integer, and suppose there is a sequence
a0 , a1 , . . . as required. We prove: There exists a polynomial P ∈ Z[x], i. e., with integer
coefficients, such that an = P (n), n = 0, 1, . . . , and xP (x) = xk + P (x − 1).
To prove this, we write P (x) = bk−1 xk−1 + · · · + b1 x + b0 and determine the coefficients
bk−1 , bk−2 , . . . , b0 successively such that
xP (x) − xk − P (x − 1) = q,
P (n − 1) + cn−1 + nk
P (n) + cn = , i. e.,
n
q + ncn = cn−1 ,
hence
c0 0! + 1! + · · · + (n − 1)!
cn = −q· .
n! n!
We conclude limn→∞ cn = 0, which, using cn ∈ Z, implies cn = 0 for sufficiently large n.
Therefore, we get q = 0 and cn = 0, n = 0, 1, . . . .
Part C. Suppose that q = qk = 0, i. e. xP (x) = xk + P (x − 1). To consider this identity for
arguments x ∈ F4 , we write F4 = {0, 1, α, α + 1}. Then we get
hence
79
N6 Number Theory 50th IMO 2009
P1 (x) = 1, q1 = −1,
P2 (x) = x + 1, q2 = 0,
P3 (x) = x2 + x − 1, q3 = 1,
P4 (x) = x3 + x2 − 2x − 1, q4 = −1,
P5 (x) = x4 + x3 − 3x2 + 5, q5 = −2,
P6 (x) = x5 + x4 − 4x3 + 2x2 + 10x − 5, q6 = 9,
q7 = −9, q8 = −50, q9 = 267, q10 = −413, q11 = −2180.
A lookup in the On-Line Encyclopedia of Integer Sequences (A000587) reveals that the sequence
q1 , −q2 , q3 , −q4 , q5 , . . . is known as Uppuluri-Carpenter numbers. The result that qk = 0
implies k ≡ 2 mod 3 is contained in
Murty, Summer: On the p-adic series ∞ k
P
n=0 n · n!. CRM Proc. and Lecture Notes 36, 2004.
As shown by Alexander (Non-Vanishing of Uppuluri-Carpenter Numbers, Preprint 2006),
Uppuluri-Carpenter numbers are zero at most twice.
Comment 3. The numbers qk can be written in terms of the Stirling numbers of the second
kind. To show this, we fix the notation such that
xk =Sk−1,k−1 x(x − 1) · · · (x − k + 1)
+ Sk−1,k−2 x(x − 1) · · · (x − k + 2) (∗)
+ · · · + Sk−1,0 x,
Ωk = Sk−1,k−1 − Sk−1,k−2 + − · · · .
xk =Sk−1,k−1 x(x + 1) · · · (x + k − 1)
− Sk−1,k−2 x(x + 1) · · · (x + k − 2)
+ − · · · ± Sk−1,0 x.
Defining
P (x) =Sk−1,k−1 (x + 1) · · · (x + k − 1)
+ (Sk−1,k−1 − Sk−1,k−2 )(x + 1) · · · (x + k − 2)
+ (Sk−1,k−1 − Sk−1,k−2 + Sk−1,k−3 )(x + 1) · · · (x + k − 3)
+ · · · + Ωk ,
we obtain
hence qk = −Ωk .
80
50th IMO 2009 Number Theory N7
N7 MNG (Mongolia)
Let a and b be distinct integers greater than 1. Prove that there exists a positive integer n such
that (an − 1)(bn − 1) is not a perfect square.
81
N7 Number Theory 50th IMO 2009
Because the series in (4) is obtained by a finite linear combination of the absolutely convergent
series (1), we conclude that in particular M1 < ∞. Since
√ √ √
ab ab ab
≤ λ := max , for all k, ` ≥ 0 such that k ≥ k0 or ` ≥ `0 ,
ak b ` ak0 b`0
we get the estimates Mn+1 ≤ λMn , n = 1, 2, . . . Our choice of k0 and `0 ensures λ < 1, which
implies Mn → 0 and consequently yn → 0 as n → ∞. It follows that yn = 0 for all sufficiently
large n.
P 0 ·`0
So, equation (3) reduces to ki=0 di xn+i = 0.
Using the theory of linear recursions again, for sufficiently large n we have
k0 −1,`0 −1 √ n
X ab
xn = fk,` k `
k=0,`=0
a b
for all real numbers α, β ∈ (0, 1). We choose k ∗ < k0 maximal such that there is some i
∗ ∗
with ck∗ ,i 6= 0. Squaring (5) and comparing coefficients of α2k β 2i , where i∗ is maximal with
ck∗ ,i∗ 6= 0, we see that k ∗ = 0. This means that the right hand side of (5) is independent of α,
which is clearly impossible.
We are left with the case that aµ = bν for some positive integers µ and ν. We may assume
that µ and ν are relatively prime. Then there is some positive integer c such that a = cν and
b = cµ . Now starting with the expansion (2), i. e.,
√
X cµ+ν n
xn = gj
j≥0
cj
for certain coefficients gj , and repeating the arguments above, we see that gj = 0 for sufficiently
large j, say j > j0 . But this means that
j0
1 1
X
µ ν
(1 − x ) (1 − x ) =
2 2 gj xj
j=0
(1 − xµ )(1 − xν )
is the square of a polynomial in x. In particular, all its zeros are of order at least 2, which
implies µ = ν by looking at roots of unity. So we obtain µ = ν = 1, i. e., a = b, a contradiction.
82
50th IMO 2009 Number Theory N7
p
Solution 2. We set a2 = A, b2 = B, and zn = (An − 1)(B n − 1). Let us assume that zn
is an integer for n = 1, 2, . . . Without loss of generality, we may suppose that b < a. We
determine an integer k ≥ 2 such that bk−1 ≤ a < bk , and define a sequence γ1 , γ2 , . . . of rational
numbers such that
n
X
2γ1 = 1 and 2γn+1 = γi γn−i for n = 1, 2, . . .
i=1
whence a n a n a n n
n b
zn = (ab) − γ1 − γ2 − · · · − γk 2k−1 + O .
b b3 b a
Now choose rational numbers r1 , r2 , . . . , rk+1 such that
(x − ab) · (x − ab ) . . . (x − a
b2k−1
) = xk+1 − r1 xk + − · · · ± rk+1 ,
and then a natural number M for which M r1 , M r2 , . . . M rk+1 are integers. For known reasons,
n
b
M (zn+k+1 − r1 zn+k + − · · · ± rk+1 zn ) = O
a
for all n ∈ N and thus there is a natural number N which is so large, that
holds for all n > N . Now the theory of linear recursions reveals that there are some rational
numbers δ0 , δ1 , δ2 , . . . , δk such that
a n a n a n
n
zn = δ0 (ab) − δ1 − δ2 3 − · · · − δk 2k−1
b b b
for sufficiently large n, where δ0 > 0 as zn > 0. As before, one obtains
An B n − An − B n + 1 = zn2
n a n a n a n o2
n
= δ0 (ab) − δ1 − δ2 3 − · · · − δk 2k−1
b b b !
i=k j=i−1 n n
2 n n n
X X A A
= δ0 A B − 2δ0 δ1 A − 2δ0 δi − δj δi−j i−1
+O k
.
i=2 j=1
B B
P (X)2 . But this cannot occur, for instance as X k−1 − 1 has no double zeros. Thus our
83
N7 Number Theory 50th IMO 2009
assumption that zn was an integer for n = 1, 2, . . . turned out to be wrong, which solves the
problem.
Original formulation of the problem. a, b are positive integers such that a·b is not a square of
an integer. Prove that there exists a (infinitely many) positive integer n such that (an −1)(bn −1)
is not a square of an integer.
Solution. Lemma. Let c be a positive integer, which is not a perfect square. Then there exists
an odd prime p such that c is not a quadratic residue modulo p.
0
Proof. Denoting the square-free part of c by c0 , we have the equality cp = pc of the corre-
sponding Legendre symbols. Suppose that c0 = q1 · · · qm , where q1 < · · · < qm are primes.
Then we have
c0 q q
1 m
= ··· .
p p p
Case 1. Let q1 be odd. We choose a quadratic nonresidue r1 modulo q1 and quadratic residues
ri modulo qi for i = 2, . . . , m. By the Chinese remainder theorem and the Dirichlet theorem,
there exists a (infinitely many) prime p such that
p ≡ r1 mod q1 ,
p ≡ r2 mod q2 ,
.. ..
. .
p ≡ rm mod qm ,
p ≡ 1 mod 4.
p ≡ r2 mod q2 ,
.. ..
. .
p ≡ rm mod qm ,
p ≡ 5 mod 8.
By the choice of the residues, we obtain qpi = rqii = 1 for i = 2, . . . , m. Since p ≡ 1 mod 4 we
have qpi = qpi , i = 2, . . . , m, by the law of quadratic reciprocity. The congruence p ≡ 5 mod 8
84
50th IMO 2009 Number Theory N7
2
implies that p
= −1. Thus
c0 2 q q
2 m
= ··· = −1,
p p p p
and the lemma is proved.
Applying the lemma for c = a · b, we find an odd prime p such that
ab a b
= · = −1.
p p p
This implies either
p−1 p−1 p−1 p−1
a 2 ≡ 1 mod p, b 2 ≡ −1 mod p, or a 2 ≡ −1 mod p, b 2 ≡ 1 mod p.
p−1 p−1
Without loss of generality, suppose that a 2 ≡ 1 mod p and b 2 ≡ −1 mod p. The second
p−1 p−1
congruence implies that b 2 −1 is not divisible by p. Hence, if the exponent νp (a 2 −1) of p in
p−1 p−1 p−1
the prime decomposition of (a 2 − 1) is odd, then (a 2 − 1)(b 2 − 1) is not a perfect square.
p−1 p−1
If νp (a 2 − 1) is even, then νp (a 2 p − 1) is odd by the well-known power lifting property
p−1 p−1
νp a 2 p − 1 = νp a 2 − 1 + 1.
p−1 p−1
p p
In this case, (a 2 − 1)(b 2 − 1) is not a perfect square.
Comment 2. There is also an elementary proof of the lemma. We cite Theorem 3 of Chapter 5
and its proof from the book
Ireland, Rosen: A Classical Introduction to Modern Number Theory, Springer 1982.
Theorem. Let a be a nonsquare integer. Then there are infinitely many primes p for which a is
a quadratic nonresidue.
Proof. It is easily seen that we may assume that a is square-free. Let a = 2e q1 q2 · · · qn , where
qi are distinct odd primes and e = 0 or 1. The case a = 2 has to be dealt with separately. We
shall assume to begin with that n ≥ 1, i. e., that a is divisible by an odd prime.
Let `1 , `2 , . . . , `k be a finite set of odd primes not including any qi . Let s be any quadratic
nonresidue modqn , and find a simultaneous solution to the congruences
x ≡ 1 mod `i , i = 1, . . . , k,
x ≡ 1 mod 8,
x ≡ 1 mod qi , i = 1, . . . , n − 1,
x ≡ s mod qn .
Call the solution b. b is odd. Suppose that b = p1 p2 · · · pm is its prime decomposition. Since
85
N7 Number Theory 50th IMO 2009
2 qi b
b ≡ 1 mod 8 we have b
= 1 and b
= qi
by a result on Jacobi symbols. Thus
a 2 e q q q b b b 1 1 s
1 n−1 n
= ··· = ··· = ··· = −1.
b b b b b q1 qn−1 qn q1 qn−1 qn
a a a a a
On the other hand, by the definition of b
, we have b
= p1 p2
··· pm
. It follows that
a
pi
= −1 for some i.
Notice that `j does not divide b. Thus pi ∈
/ {`1 , `2 , . . . , `k }.
To summarize, if a is a nonsquare, divisible by an odd prime,we have found a prime p, outside
of a given finite set of primes {2, `1 , `2 , . . . , `k }, such that ap = −1. This proves the theorem
in this case.
It remains to consider the case a = 2. Let `1 , `2 , . . . , `k be a finite set of primes, excluding 3, for
which `2i = −1. Let b = 8`1 `2 · · · `k + 3. b is not divisible by 3 or any `i . Since b ≡ 3 mod 8
b2 −1
we have 2b = (−1) 8 = −1. Suppose
that b = p1 p2 · · · pm is the prime decomposition of
2
b. Then, as before, we see that pi = −1 for some i. pi ∈ / {3, `1 , `2 , . . . , `k }. This proves the
theorem for a = 2.
This proof has also been posted to mathlinks, see http://www.mathlinks.ro/viewtopic.
php?t=150495 again.
86
51st International Mathematical Olympiad
Astana, Kazakhstan 2010
Note of Confidentiality 5
Algebra 7
Problem A1 . . . . . . . . . . . . . . . . . . . . . . . . . . . . . . . . . . . . . . 7
Problem A2 . . . . . . . . . . . . . . . . . . . . . . . . . . . . . . . . . . . . . . 8
Problem A3 . . . . . . . . . . . . . . . . . . . . . . . . . . . . . . . . . . . . . . 10
Problem A4 . . . . . . . . . . . . . . . . . . . . . . . . . . . . . . . . . . . . . . 12
Problem A5 . . . . . . . . . . . . . . . . . . . . . . . . . . . . . . . . . . . . . . 13
Problem A6 . . . . . . . . . . . . . . . . . . . . . . . . . . . . . . . . . . . . . . 15
Problem A7 . . . . . . . . . . . . . . . . . . . . . . . . . . . . . . . . . . . . . . 17
Problem A8 . . . . . . . . . . . . . . . . . . . . . . . . . . . . . . . . . . . . . . 19
Combinatorics 23
Problem C1 . . . . . . . . . . . . . . . . . . . . . . . . . . . . . . . . . . . . . . 23
Problem C2 . . . . . . . . . . . . . . . . . . . . . . . . . . . . . . . . . . . . . . 26
Problem C3 . . . . . . . . . . . . . . . . . . . . . . . . . . . . . . . . . . . . . . 28
Problem C4 . . . . . . . . . . . . . . . . . . . . . . . . . . . . . . . . . . . . . . 30
Problem C41 . . . . . . . . . . . . . . . . . . . . . . . . . . . . . . . . . . . . . 30
Problem C5 . . . . . . . . . . . . . . . . . . . . . . . . . . . . . . . . . . . . . . 32
Problem C6 . . . . . . . . . . . . . . . . . . . . . . . . . . . . . . . . . . . . . . 35
Problem C7 . . . . . . . . . . . . . . . . . . . . . . . . . . . . . . . . . . . . . . 38
Geometry 44
Problem G1 . . . . . . . . . . . . . . . . . . . . . . . . . . . . . . . . . . . . . . 44
Problem G2 . . . . . . . . . . . . . . . . . . . . . . . . . . . . . . . . . . . . . . 46
Problem G3 . . . . . . . . . . . . . . . . . . . . . . . . . . . . . . . . . . . . . . 50
Problem G4 . . . . . . . . . . . . . . . . . . . . . . . . . . . . . . . . . . . . . . 52
Problem G5 . . . . . . . . . . . . . . . . . . . . . . . . . . . . . . . . . . . . . . 54
Problem G6 . . . . . . . . . . . . . . . . . . . . . . . . . . . . . . . . . . . . . . 56
Problem G61 . . . . . . . . . . . . . . . . . . . . . . . . . . . . . . . . . . . . . 56
Problem G7 . . . . . . . . . . . . . . . . . . . . . . . . . . . . . . . . . . . . . . 60
Number Theory 64
Problem N1 . . . . . . . . . . . . . . . . . . . . . . . . . . . . . . . . . . . . . . 64
Problem N11 . . . . . . . . . . . . . . . . . . . . . . . . . . . . . . . . . . . . . 64
Problem N2 . . . . . . . . . . . . . . . . . . . . . . . . . . . . . . . . . . . . . . 66
Problem N3 . . . . . . . . . . . . . . . . . . . . . . . . . . . . . . . . . . . . . . 68
Problem N4 . . . . . . . . . . . . . . . . . . . . . . . . . . . . . . . . . . . . . . 70
Problem N5 . . . . . . . . . . . . . . . . . . . . . . . . . . . . . . . . . . . . . . 71
Problem N6 . . . . . . . . . . . . . . . . . . . . . . . . . . . . . . . . . . . . . . 72
Note of Confidentiality
Contributing Countries
The Organizing Committee and the Problem Selection Committee of IMO 2010 thank the
following 42 countries for contributing 158 problem proposals.
Yerzhan Baissalov
Ilya Bogdanov
Géza Kós
Nairi Sedrakyan
Damir Yeliussizov
Kuat Yessenov
Algebra
A1. Determine all functions f : R Ñ R such that the equality
f prxsy q f pxqrf py qs. (1)
holds for all x, y P R. Here, by rxs we denote the greatest integer not exceeding x.
(France)
Answer. f pxq const C, where C 0 or 1 ¨ C 2.
Solution 1. First, setting x 0 in (1) we get
f p0q f p0qrf py qs (2)
for all y P R. Now, two cases are possible.
Case 1. Assume that f p0q 0. Then from (2) we conclude that rf py qs 1 for all
y P R. Therefore, equation (1) becomes f prxsy q f pxq, and substituting y 0 we have
f pxq f p0q C 0. Finally, from rf py qs 1 rC s we obtain that 1 ¨ C 2.
Case 2. Now we have f p0q 0. Here we consider two subcases.
Subcase 2a. Suppose that there exists 0 α 1 such that f pαq 0. Then setting x α
in (1) we obtain 0 f p0q f pαqrf py qs for all y P R. Hence, rf py qs 0 for all y P R. Finally,
substituting x 1 in (1) provides f py q 0 for all y P R, thus contradicting the condition
f pαq 0.
Subcase 2b. Conversely, we have f pαq 0 for all 0 ¨ α 1. Consider any real z; there
exists an integer N such that α P r0, 1q (one may set N rzs 1 if z © 0 and N rzs 1
z
N
otherwise). Now, from (1) we get f pz q f prN sαq f pN qrf pαqs 0 for all z P R.
Finally, a straightforward check shows that all the obtained functions satisfy (1).
Solution 2. Assume that rf py qs 0 for some y; then the substitution x 1 provides
f py q f p1qrf py qs 0. Hence, if rf py qs 0 for all y, then f py q 0 for all y. This function
obviously satisfies the problem conditions.
So we are left to consider the case when rf paqs 0 for some a. Then we have
f prxsaq
f prxsaq f pxqrf paqs, or f pxq
rf paqs . (3)
This means that f px1 q f px2 q whenever rx1 s rx2 s, hence f pxq f prxsq, and we may assume
that a is an integer.
Now we have
f paq f 2a 12 f p2aq f 12 f p2aqrf p0qs;
this implies rf p0qs 0, so we may even assume that a 0. Therefore equation (3) provides
f p0q
f pxq C0
rf p0qs
for each x. Now, condition (1) becomes equivalent to the equation C C rC s which holds
exactly when rC s 1.
8
A2. Let the real numbers a, b, c, d satisfy the relations a b c d 6 and a2 b2 c2 d2 12.
Prove that
36 ¨ 4pa3 b3 c3 d3 q pa4 b4 c4 d4 q ¨ 48.
(Ukraine)
16 © x4 y 4 z 4 t4 © 4,
x 4
y 4
z 4
t 4
© px2 y2 z2 t2 q2
4.
4
For the other one, expanding the brackets we note that
px2 y2 z2 t2 q2 px4 y4 z4 t4 q q,
x4 y4 z4 t4 ¨ px2 y2 z2 t2 q2 16,
and we are done.
Comment 1. The estimates are sharp; the lower and upper bounds are attained at p3, 1, 1, 1q and
p0, 2, 2, 2q, respectively.
Comment 2. After the change of variables, one can finish the solution in several different ways.
The latter estimate, for instance, it can be performed by moving the variables – since we need only
the second of the two shifted conditions.
a b c 6 d, a2 b2 c2 12 d2,
hence the power mean inequality
a2 b2 c2 © pa b
3
cq2
rewrites as
© p6 3 dq
2
12 d2 ðñ 2dpd 3q ¨ 0,
9
which implies the desired inequalities for d; since the conditions are symmetric, we also have
the same estimate for the other variables.
Now, to prove the rightmost inequality, we use the obvious inequality x2 px 2q2 © 0 for
each real x; this inequality rewrites as 4x3 x4 ¨ 4x2 . It follows that
p4a3 a4 q p4b3 b4 q p4c3 c4q p4d3 d4q ¨ 4pa2 b2 c2 d2 q 48,
as desired.
Now we prove the leftmost inequality in an analogous way. For each x P r0, 3s, we have
px 1qpx 1q2px 3q ¨ 0 which is equivalent to 4x3 x4 © 2x2 4x 3. This implies that
p4a3 a4 q p4b3 b4 q p4c3 c4 q p4d3 d4q © 2pa2 b2 c2 d2 q 4 pa b c dq 12 36,
as desired.
Comment. It is easy to guess the extremal points p0, 2, 2, 2q and p3, 1, 1, 1q for this inequality. This
provides a method of finding the polynomials used in Solution 2. Namely, these polynomials should
have the form x4 4x3 ax2 bx c; moreover, the former polynomial should have roots at 2 (with
an even multiplicity) and 0, while the latter should have roots at 1 (with an even multiplicity) and 3.
These conditions determine the polynomials uniquely.
which follows from the previous estimate. The inequality 4a a2 ¨ 4b b2 can be proved
analogously.
Further, the inequalities a ¨ b ¨ c together with 4a a2 ¨ 4b b2 ¨ 4c c2 allow us to
apply the Chebyshev inequality obtaining
a2 p4a a2 q b2 p4b b2 q c2 p4c c2 q © pa2 b2 c2 q 4pa b cq pa2 b2 c2 q
1
3
p12 d qp4p6 3dq p12 d qq .
2 2
A3. Let x1 , . . . , x100 be nonnegative real numbers such that xi xi 1 xi 2 ¨ 1 for all
i 1, . . . , 100 (we put x101 x1 , x102 x2 ). Find the maximal possible value of the sum
¸
100
S xi xi 2 .
i 1
(Russia)
25
Answer. .
2
2
Solution 1. Let x2i 0, x2i1 12 for all i 1, . . . , 50. Then we have S 50 12 25
. So,
we are left to show that S ¨ 25
2
2
for all values of xi ’s satisfying the problem conditions.
Consider any 1 ¨ i ¨ 50. By the problem condition, we get x2i1 ¨ 1 x2i x2i 1 and
x2i 2 ¨ 1 x2i x2i 1 . Hence by the AM–GM inequality we get
x2i1 x2i 1 x2i x2i 2 ¨ p1 x2i x2i 1 qx2i 1 x2i p1 x2i x2i q 1
px2iqp1 x2i x2i 1 q ¨ px2i x2i
x2i
1q p1 x2i x2i 1 q
2 1 .
1
2 4
Summing up these inequalities for i 1, 2, . . . , 50, we get the desired inequality
¸
50
px2i1 x2i 1 x2i x2i 2 q ¨ 50 14 252 .
i 1
Comment. This solution shows that a bit more general fact holds. Namely, consider 2n nonnegative
numbers x1 , . . . , x2n in a row (with no cyclic notation) and suppose that xi xi 1 xi 2 ¨ 1 for all
¸
2n 2 n1
i 1, 2, . . . , 2n 2. Then xi xi 2 ¨ .
i1
4
The proof is the same as above, though if might be easier to find it (for instance, applying
induction). The original estimate can be obtained from this version by considering the sequence
x1 , x2 , . . . , x100 , x1 , x2 .
Solution 2. We present another proof of the estimate. From the problem condition, we get
¸
100 ¸
100 ¸
100 ¸
100 ¸
100
S xi xi 2 ¨ xi p1 xi xi 1q xi x2i xi xi 1
i 1 i 1
i 1
i 1 i 1
¸
100 ¸
100
xi
1
2 i1
pxi xi 1 q2.
i 1
° ° 2
By the AM–QM inequality, we have pxi xi 1 q2 © 1001 pxi xi 1 q , so
2 2
¸
100 ¸
100 ¸
100 ¸
100
S ¨ xi
1
200
pxi xi 1 q xi
2
100
xi
i 1 i 1
i 1 i1
100 ¸ ¸
100
2
100
xi
100
2
xi .
i 1
i 1
Comment. These solutions are not as easy as they may seem at the first sight. There are two
different optimal configurations in which the variables have different values, and not all of sums of
three consecutive numbers equal 1. Although it is easy to find the value 25 2 , the estimates must be
done with care to preserve equality in the optimal configurations.
12
Solution 1. We start with some observations. First, from the definition of xi it follows that
for each positive integer k we have
Solution 2. We will use the notation of Sn and the relations (1)–(3) from the previous
solution.
Assume the contrary and consider the minimal n such that Sn 1 0; surely n © 1, and
from Sn © 0 we get Sn 0, xn 1 1. Hence, we are especially interested in the set
M tn : Sn 0u; our aim is to prove that xn 1 1 whenever n P M thus coming to a
contradiction.
For this purpose, we first describe the set M inductively. We claim that (i) M consists only
of even numbers, (ii) 2 P M, and (iii) for every even n © 4 we have n P M ðñ rn{4s P M.
Actually, (i) holds since Sn n pmod 2q, (ii) is straightforward, while (iii) follows from the
relations S4k 2 S4k 2Sk .
Now, we are left to prove that xn 1 1 if n P M. We use the induction on n. The base
case is n 2, that is, the minimal element of M; here we have x3 1, as desired.
For the induction step, consider some 4 ¨ n P M and let m rn{4s P M; then m is even,
and xm 1 1 by the induction hypothesis. We prove that xn 1 xm 1 1. If n 4m then we
have xn 1 x2m 1 xm 1 since m is even; otherwise, n 4m 2, and xn 1 x2m 2 xm 1 ,
as desired. The proof is complete.
Comment. Using the inductive definition of set M , one can describe it explicitly. Namely, M consists
exactly of all positive integers not containing digits 1 and 3 in their 4-base representation.
13
A5. Denote by Q the set of all positive rational numbers. Determine all functions f : Q ÑQ
which satisfy the following equation for all x, y PQ :
f f pxq2 y x3 f pxy q. (1)
(Switzerland)
f pxy q2 f pxq2 f py q2 ,
f pxy q f pxqf py q.
Therefore, f is multiplicative. This also implies f p1q 1 and f pxn q f pxqn for all integers n.
Then the function equation (1) can be re-written as
2
f f pxq f py q x3 f pxqf py q,
a
f f pxq x3 f pxq. (3)
n 1
14
where the exponents should be integers. But this is not true for large values of n, for example
p 52 qnα1 cannot be a integer number when 2n α1. Therefore, gpxq 1 is impossible.
Hence, g pxq 1 and thus f pxq for all x.
1
x
The function f pxq satisfies the equation (1):
1
x
3
f pf pxq2 y q xy x3 f pxyq.
1 1 x
f pxq2 y 1 2
y
x
A6. Suppose that f and g are two functions defined on the set of positive integers and taking
positive integer values. Suppose also that the equations f pg pnqq f pnq 1 and g pf pnqq
g pnq 1 hold for all positive integers. Prove that f pnq g pnq for all positive integer n.
(Germany)
Solution 1. Throughout the solution, by N we denote the set of all positive integers. For
any function h : N Ñ N and for any positive integer k, define hk pxq h h . . . hpxq . . . (in
loooomoooon
Observe that f g k pxq f g k1pxq 1 f pxq k for any positive integer k, and
similarly g f k pxq g pxq k. Now let a and b are the minimal values attained by f and g,
respectively; say f pnf q a, g png q b. Then we have f g pnf q a k, g f png q b k, so
k k
the function f attains all values from the set Nf ta, a 1, . . . u, while g attains all the values
from the set Ng tb, b 1, . . . u.
Next, note that f pxq f py q implies g pxq g f pxq 1 g f py q 1 g py q; surely, the
converse implication also holds. Now, we say that x and y are similar (and write x y) if
f pxq f py q (equivalently, g pxq g py q). For every x P N, we define rxs ty P N : x y u;
surely, y1 y2 for all y1 , y2 P rxs, so rxs ry s whenever y P rxs.
Now we investigate the structure of the sets rxs.
Claim 1. Suppose that f pxq f py q; then x y, that is, f pxq f py q. Consequently, each
class rxs contains at most one element from Nf , as well as at most one element from Ng .
Proof. If f pxq f py q, then we have g pxq g f pxq 1 g f py q 1 g py q, so x y. The
second statement follows now from the sets of values of f and g. l
Next, we clarify which classes do not contain large elements.
Claim 2. For any x P N, we have rxs t1, 2, . . . , b 1u if and only if f pxq a. Analogously,
rxs t1, 2, . . . , a 1u if and only if gpxq b.
Proof. We will prove that rxs t1, 2, . . . , b 1u ðñ f pxq ¡ a; the proof of the second
statement is similar.
Note that f pxq ¡ a implies that there exists some y satisfying f py q f pxq 1, so f g py q
f py q 1 f pxq, and hence x g py q © b. Conversely, if b ¨ c x then c g py q for some y P N,
which in turn follows f pxq f g py q f py q 1 © a 1, and hence f pxq ¡ a. l
Claim 2 implies that there exists exactly one class contained in t1, . . . , a 1u (that is, the
class rng s), as well as exactly one class contained in t1, . . . , b 1u (the class rnf s). Assume for a
moment that a ¨ b; then rng s is contained in t1, . . . , b 1u as well, hence it coincides with rng s.
So, we get that
f pxq a ðñ g pxq b ðñ x nf ng . (1)
Claim 3. a b.
Proof. By Claim 2, we have ras rnf s, so ras should contain some element a1 © b by Claim 2
again. If a a1 , then ras contains two elements © a which is impossible by Claim 1. Therefore,
a a1 © b. Similarly, b © a. l
Now we are ready to prove the problem statement. First, we establish the following
Claim 4. For every integer d © 0, f d 1 pnf q g d 1 pnf q a d.
Proof. Induction on d. For d 0, the statement follows from 3. Next, for d ¡ 1
(1) and Claim
from the induction hypothesis we have f pnf q f f pnf q f g pnf q f pnf q d a d.
d 1 d d
Finally, for each x P N, we have f pxq a d for some d © 0, so f pxq f g dpnf q and
hence x g d pnf q. It follows that g pxq g g d pnf q g d 1 pnf q a d f pxq by Claim 4.
Solution 2. We start with the same observations, introducing the relation and proving
Claim 1 from the previous solution.
Note that f paq ¡ a since otherwise we have f paq a and hence g paq g f paq g paq 1,
which is false.
Claim 21 . a b.
Proof. We can assume that a ¨ b. Since f paq © a 1, there exists some x P N such that
f paq f pxq 1, which is equivalent to f paq f g pxq and a g pxq. Since g pxq © b © a, by
Claim 1 we have a g pxq © b, which together with a ¨ b proves the Claim. l
Now, almost the same method allows to find the values f paq and g paq.
Claim 31 . f paq g paq a 1.
Proof. Assume the contrary; then f paq © a 2, hence there exist some x, y P N such that
f pxq f paq 2 and f py q g pxq (as g pxq © a b). Now we get f paq f pxq 2 f g 2pxq ,
so a g 2pxq © a, and by Claim 1 we get a g 2 pxq g f py q 1 g py q © 1 a; this is
impossible. The equality g paq a 1 is similar.
Now, we are prepared for the proof of the problem statement. First, we prove it for n © a.
Claim 41 . For each integer x © a, we have f pxq g pxq x 1.
x a is provided by Claim 3 , while the induction
Proof. Induction on x. The base case 1
step follows from f px 1q f g pxq f pxq 1 px 1q 1 and the similar computation
for g px 1q.
Finally,
for an arbitrary n P N we have g pnq © a, so by Claim 41 we have f pnq 1
f g pnq g pnq 1, hence f pnq g pnq.
Comment. It is not hard now to describe all the functions f : N Ñ N satisfying the property f pf pnqq
f pnq 1. For each such function, there exists n0 P N such that f pnq n 1 for all n © n0 , while for
each n n0 , f pnq is an arbitrary number greater than of equal to n0 (these numbers may be different
for different n n0 ).
17
Prove that there exist positive integers ℓ ¨ r and N such that an anℓ aℓ for all n © N.
(Iran)
Solution 1. First, from the problem conditions we have that each an (n ¡ r) can be expressed
as an aj1 aj2 with j1 , j2 n, j1 j2 n. If, say, j1 ¡ r then we can proceed in the same
way with aj1 , and so on. Finally, we represent an in a form
ai
an 1 aik , (2)
1 ¨ ij ¨ r, i1 ik n. (3)
Moreover, if ai1 and ai2 are the numbers in (2) obtained on the last step, then i1 i2 ¡ r.
Hence we can adjust (3) as
1 ¨ ij ¨ r, i1 ik n, i1 i2 ¡ r. (4)
On the other hand, suppose that the indices i1 , . . . , ik satisfy the conditions (4). Then,
denoting sj i1 ij , from (1) we have
an as © as
k k 1
aik © as k 2
aik1 aik © © ai 1 aik .
Summarizing these observations we get the following
Claim. For every n ¡ r, we have
an maxtai 1 aik : the collection pi1 , . . . , ik q satisfies (4)u. l
Now we denote
s max
ai
1¨i¨r i
Solution 2. As in the previous solution, we involve the expansion (2), (3), and we fix some
index 1 ¨ ℓ ¨ r such that
aℓ
ℓ
s max .
ai
1¨i¨r i
bn 1¨max
k ¨n1
pak ank q ns max
¨¨
1 k n 1
pbk bnk nsq ns max
¨¨
1 k n 1
pbk bnk q ¨ 0,
as required.
Now, if bk 0 for all 1 ¨ k ¨ r, then bn 0 for all n, hence an sn, and the statement is
trivial. Otherwise, define
M 1max |b |,
¨i¨r i
ε mint|bi | : 1 ¨ i ¨ r, bi 0u.
Then for n ¡ r we obtain
bn 1¨max
k ¨n1
pbk bnk q © bℓ bnℓ bnℓ,
so
0 © bn © bnℓ © bn2ℓ © © M.
Thus, in view of the expansion (2), (3) applied to the sequence pbn q, we get that each bn is
contained in a set
We claim that this set is finite. Actually, for any x P T , let x bi bi (i1, . . . , ik ¨ r).
1 k
br t , br t ℓ , br t 2ℓ , . . .
is non-decreasing and attains the finite number of values; therefore it is constant from some
index. Thus, the sequence pbn q is periodic with period ℓ from some index N, which means that
and hence
as desired.
19
(South Korea)
α β γ T2T SS , αβ αγ βγ SτT Tσ
S
.
or
4S 2 T 2 ¡ 3pT S qpT σ Sτ q,
which is exactly what we need.
Comment 1. In fact, one can locate the roots of P pxq more narrowly: they should lie in the intervals
pa, bq, pc, dq, pe, f q.
Surely, if we change all inequality signs in the problem statement to non-strict ones, the (non-strict)
inequality will also hold by continuity. One can also find when the equality is achieved. This happens
in that case when P pxq is a perfect cube, which immediately implies that b c d ep α β γ q,
together with the additional condition that P 2 pbq 0. Algebraically,
6pT S qb 4T S 0 ðñ 3bpa 4b f q 2pa 2bqp2b fq
ðñ f bp2a
4b aq
b
b 1
3pb aq
2a b
¡ b.
This means that for every pair of numbers a, b such that 0 a b, there exists f ¡ b such that the
point pa, b, b, b, b, f q is a point of equality.
20
Solution 2. Let
U 12 pe aq2 pc aq2 pe cq2 S 2 3pac ae ceq
and
V 12 pf bq2 pf dq2 pd bq2 T 2 3pbd bf df q.
Then
pL.H.S.q2 pR.H.S.q2 p2ST q2 pS T q S 3pbd bf df q T 3pac ae ceq
4S 2T 2 pS T q S pT 2 V q T pS 2 U q pS T qpSV T U q ST pT S q2 ,
and the statement is equivalent with
pS T qpSV T U q ¡ ST pT S q2. (4)
By the Cauchy-Schwarz inequality,
? ? 2 ? ? 2
pS T qpT U SV q © S TU T SV ST U V . (5)
? ?
Estimate the quantities U and V by the QM–AM inequality with the positive terms pe cq2
and pd bq2 being omitted:
? ?
¡ pe aq 2 pc aq pf bq2 pf dq2
2 2
U V
2
¡ p e aq pc aq pf bq pf dq
f d b e c
a
2 2 2 2 2 2
pT S q
3
2
p e dq
3
2
pc bq ¡ T S. (6)
The estimates (5) and (6) prove (4) and hence the statement.
Solution 3. We keep using the notations σ and τ from Solution 1. Moreover, let s c e.
Note that
pc bqpc dq pe f qpe dq pe f qpc bq 0,
since each summand is negative. This rewrites as
pbd bf df q pac ce aeq pc eqpb d f a c eq, or
τ σ spT S q. (7)
Then we have
Sτ Tσ S pτ σq pS T qσ SspT S q
pS Tqpce asq
2
¨ SspT S q pS T q s4 pS sqs s 2ST 34 pS T qs .
3
4
pS T qpSτ T σq
3
4
pS T qs 2ST 3
4
pS T qs
¨ 4 pS T qs 34 pS T qs
3
2ST
2
ST.
Hence, b
2ST ¡ 3pS T q S pbd bf df q T pac ae ceq .
21
Comment 2. The expression (7) can be found by considering the sum of the roots of the quadratic
polynomial q pxq px bqpx dqpx f q px aqpx cqpx eq.
Solution 4. We introduce the expressions σ and τ as in the previous solutions. The idea of
the solution is to change the values of variables a, . . . , f keeping the left-hand side unchanged
and increasing the right-hand side; it will lead to a simpler inequality which can be proved in
a direct way.
Namely, we change the variables (i) keeping the (non-strict) inequalities a ¨ b ¨ c ¨ d ¨
e ¨ f ; (ii) keeping the values of sums S and T unchanged; and finally (iii) increasing the
values of σ and τ . Then the left-hand side of (1) remains unchanged, while the right-hand
side increases. Hence, the inequality (1) (and even a non-strict version of (1)) for the changed
values would imply the same (strict) inequality for the original values.
First, we find the sufficient conditions for (ii) and (iii) to be satisfied.
Lemma. Let x, y, z ¡ 0; denote U px, y, z q x y z, υ px, y, z q xy xz yz. Suppose that
x1 y 1 x y but |x y | © |x1 y 1 |; then we have U px1 , y 1 , z q U px, y, z q and υ px1 , y 1 , z q ©
υ px, y, z q with equality achieved only when |x y | |x1 y 1 |.
Proof. The first equality is obvious. For the second, we have
Solution. We say that an order of singers is good if it satisfied all their wishes. Next, we
say that a number N is realizable by k singers (or k-realizable) if for some set of wishes of
these singers there are exactly N good orders. Thus, we have to prove that a number 2010 is
20-realizable.
We start with the following simple
Lemma. Suppose that numbers n1 , n2 are realizable by respectively k1 and k2 singers. Then
the number n1 n2 is pk1 k2 q-realizable.
Proof. Let the singers A1 , . . . , Ak1 (with some wishes among them) realize n1 , and the singers B1 ,
. . . , Bk2 (with some wishes among them) realize n2 . Add to each singer Bi the wish to perform
later than all the singers Aj . Then, each good order of the obtained set of singers has the form
pAi1 , . . . , Aik1 , Bj1 , . . . , Bjk2 q, where pAi1 , . . . , Aik1 q is a good order for Ai’s and pBj1 , . . . , Bjk2 q
is a good order for Bj ’s. Conversely, each order of this form is obviously good. Hence, the
number of good orders is n1 n2 . l
In view of Lemma, we show how to construct sets of singers containing 4, 3 and 13 singers
and realizing the numbers 5, 6 and 67, respectively. Thus the number 2010 6 5 67 will be
realizable by 4 3 13 20 singers. These companies of singers are shown in Figs. 1–3; the
wishes are denoted by arrows, and the number of good orders for each Figure stands below in
the brackets.
a1
a2
a3
a4
a b a5
a6
y a7
c d a8 x
(5) (3)
Fig. 1 Fig. 2 a9
a10
a11
(67)
Fig. 3
For Fig. 1, there are exactly 5 good orders pa, b, c, dq, pa, b, d, cq, pb, a, c, dq, pb, a, d, cq,
pb, d, a, cq. For Fig. 2, each of 6 orders is good since there are no wishes.
24
Finally, for Fig. 3, the order of a1 , . . . , a11 is fixed; in this line, singer x can stand before
each of ai (i ¨ 9), and singer y can stand after each of aj (j © 5), thus resulting in 9 7 63
cases. Further, the positions of x and y in this line determine the whole order uniquely unless
both of them come between the same pair pai , ai 1 q (thus 5 ¨ i ¨ 8); in the latter cases, there
are two orders instead of 1 due to the order of x and y. Hence, the total number of good orders
is 63 4 67, as desired.
Comment. The number 20 in the problem statement is not sharp and is put there to respect the
original formulation. So, if necessary, the difficulty level of this problem may be adjusted by replac-
ing 20 by a smaller number. Here we present some improvements of the example leading to a smaller
number of singers.
Surely, each example with 20 singers can be filled with some “super-stars” who should perform
at the very end in a fixed order. Hence each of these improvements provides a different solution for
the problem. Moreover, the large variety of ideas standing behind these examples allows to suggest
that there are many other examples.
1. Instead of building the examples realizing 5 and 6, it is more economic to make an example
realizing 30; it may seem even simpler. Two possible examples consisting of 5 and 6 singers are shown
in Fig. 4; hence the number 20 can be decreased to 19 or 18.
For Fig. 4a, the order of a1 , . . . , a4 is fixed, there are 5 ways to add x into this order, and there
are 6 ways to add y into the resulting order of a1 , . . . , a4 , x. Hence there are 5 6 30 good orders.
On Fig. 4b, for 5 singers a, b1 , b2 , c1 , c2 there are 5! 120 orders at all. Obviously, exactly one half
of them satisfies the wish b1 b2 , and exactly one half of these orders satisfies another wish c1 c2 ;
hence, there are exactly 5!{4 30 good orders.
a1 b1 b2
b2 b1 b3
x b1 c1
b3 b4
a2 b4 b5 a5
a
y a6 y a6
a3 a7 x a7 x
y b2 c2 a8 a8
a4 a9 c9 c10
(30) a10
(30) b) a11 c11
a) (2010) (2010)
Fig. 4 Fig. 5 Fig. 6
2. One can merge the examples for 30 and 67 shown in Figs. 4b and 3 in a smarter way, obtaining
a set of 13 singers representing 2010. This example is shown in Fig. 5; an arrow from/to group
tb1 , . . . , b5 u means that there exists such arrow from each member of this group.
Here, as in Fig. 4b, one can see that there are exactly 30 orders of b1 , . . . , b5 , a6 , . . . , a11 satisfying
all their wishes among themselves. Moreover, one can prove in the same way as for Fig. 3 that each
of these orders can be complemented by x and y in exactly 67 ways, hence obtaining 30 67 2010
good orders at all.
Analogously, one can merge the examples in Figs. 1–3 to represent 2010 by 13 singers as is shown
in Fig. 6.
25
a1
a2 a5
a3 a4
b1 b3 b4
a4 a3
b4
a6 b2
b5 b2 a2
b6 b3 a1 b1
(67) (2010)
Fig. 7 Fig. 8
3. Finally, we will present two other improvements; the proofs are left to the reader. The graph in
Fig. 7 shows how 10 singers can represent 67. Moreover, even a company of 10 singers representing 2010
can be found; this company is shown in Fig. 8.
26
C2. On some planet, there are 2N countries (N © 4). Each country has a flag N units wide
and one unit high composed of N fields of size 1 1, each field being either yellow or blue. No
two countries have the same flag.
We say that a set of N flags is diverse if these flags can be arranged into an N N square so
that all N fields on its main diagonal will have the same color. Determine the smallest positive
integer M such that among any M distinct flags, there exist N flags forming a diverse set.
(Croatia)
Answer. M 2N 2 1.
Solution. When speaking about the diagonal of a square, we will always mean the main
diagonal.
Let MN be the smallest positive integer satisfying the problem condition. First, we show
that MN ¡ 2N 2 . Consider the collection of all 2N 2 flags having yellow first squares and blue
second ones. Obviously, both colors appear on the diagonal of each N N square formed by
these flags.
We are left to show that MN ¨ 2N 2 1, thus obtaining the desired answer. We start with
establishing this statement for N 4.
Suppose that we have 5 flags of length 4. We decompose each flag into two parts of 2 squares
each; thereby, we denote each flag as LR, where the 2 1 flags L, R P S tBB, BY, YB, YYu
are its left and right parts, respectively. First, we make two easy observations on the flags 2 1
which can be checked manually.
(i) For each A P S, there exists only one 2 1 flag C P S (possibly C A) such that A
and C cannot form a 2 2 square with monochrome diagonal (for part BB, that is YY, and
for BY that is YB).
(ii) Let A1 , A2 , A3 P S be three distinct elements; then two of them can form a 2 2 square
with yellow diagonal, and two of them can form a 2 2 square with blue diagonal (for all parts
but BB, a pair (BY, YB) fits for both statements, while for all parts but BY, these pairs are
(YB, YY) and (BB, YB)).
Now, let ℓ and r be the numbers of distinct left and right parts of our 5 flags, respectively.
The total number of flags is 5 ¨ rℓ, hence one of the factors (say, r) should be at least 3. On
the other hand, ℓ, r ¨ 4, so there are two flags with coinciding right part; let them be L1 R1
and L2 R1 (L1 L2 ).
Next, since r © 3, there exist some flags L3 R3 and L4 R4 such that R1 , R3 , R4 are distinct.
Let L1 R1 be the remaining flag. By (i), one of the pairs pL1 , L1 q and pL1 , L2 q can form a
2 2 square with monochrome diagonal; we can assume that L1 , L2 form a square with a blue
diagonal. Finally, the right parts of two of the flags L1 R1 , L3 R3 , L4 R4 can also form a 2 2
square with a blue diagonal by (ii). Putting these 2 2 squares on the diagonal of a 4 4
square, we find a desired arrangement of four flags.
We are ready to prove the problem statement by induction on N; actually, above we have
proved the base case N 4. For the induction step, assume that N ¡ 4, consider any 2N 2 1
flags of length N, and arrange them into a large flag of size p2N 2 1q N. This flag contains
a non-monochrome column since the flags are distinct; we may assumeR that this Vcolumn is the
2N 2 1
first one. By the pigeonhole principle, this column contains at least
2
2N 3 1
squares of one color (say, blue). We call the flags with a blue first square good.
Consider all the good flags and remove the first square from each of them. We obtain at
least 2N 3 1 © MN 1 flags of length N 1; by the induction hypothesis, N 1 of them
27
can form a square Q with the monochrome diagonal. Now, returning the removed squares, we
obtain a rectangle pN 1q N, and our aim is to supplement it on the top by one more flag.
If Q has a yellow diagonal, then we can take each flag with a yellow first square (it exists
by a choice of the first column; moreover, it is not used in Q). Conversely, if the diagonal of Q
is blue then we can take any of the © 2N 3 1 pN 1q ¡ 0 remaining good flags. So, in both
cases we get a desired N N square.
C3. 2500 chess kings have to be placed on a 100 100 chessboard so that
(i) no king can capture any other one (i.e. no two kings are placed in two squares sharing
a common vertex);
(ii) each row and each column contains exactly 25 kings.
Find the number of such arrangements. (Two arrangements differing by rotation or sym-
metry are supposed to be different.)
(Russia)
Answer. There are two such arrangements.
Solution. Suppose that we have an arrangement satisfying the problem conditions. Divide the
board into 2 2 pieces; we call these pieces blocks. Each block can contain not more than one
king (otherwise these two kings would attack each other); hence, by the pigeonhole principle
each block must contain exactly one king.
Now assign to each block a letter T or B if a king is placed in its top or bottom half,
respectively. Similarly, assign to each block a letter L or R if a king stands in its left or right
half. So we define T-blocks, B-blocks, L-blocks, and R-blocks. We also combine the letters; we call
a block a TL-block if it is simultaneously T-block and L-block. Similarly we define TR-blocks,
BL-blocks, and BR-blocks. The arrangement of blocks determines uniquely the arrangement of
kings; so in the rest of the solution we consider the 50 50 system of blocks (see Fig. 1). We
identify the blocks by their coordinate pairs; the pair pi, j q, where 1 ¨ i, j ¨ 50, refers to the
jth block in the ith row (or the ith block in the jth column). The upper-left block is p1, 1q.
The system of blocks has the following properties..
(i1 ) If pi, j q is a B-block then pi 1, j q is a B-block: otherwise the kings in these two blocks
can take each other. Similarly: if pi, j q is a T-block then pi 1, j q is a T-block; if pi, j q is an
L-block then pi, j 1q is an L-block; if pi, j q is an R-block then pi, j 1q is an R-block.
(ii1 ) Each column contains exactly 25 L-blocks and 25 R-blocks, and each row contains
exactly 25 T-blocks and 25 B-blocks. In particular, the total number of L-blocks (or R-blocks,
or T-blocks, or B-blocks) is equal to 25 50 1250.
Consider any B-block of the form p1, j q. By (i1 ), all blocks in the jth column are B-blocks;
so we call such a column B-column. By (ii1 ), we have 25 B-blocks in the first row, so we obtain
25 B-columns. These 25 B-columns contain 1250 B-blocks, hence all blocks in the remaining
columns are T-blocks, and we obtain 25 T-columns. Similarly, there are exactly 25 L-rows and
exactly 25 R-rows.
Now consider an arbitrary pair of a T-column and a neighboring B-column (columns with
numbers j and j 1).
1 2 3
k k k 1 TL BL TL
j j+1
k k k 2 TL BR TR
i
k L
k k k
i+1
3 BL BL TR
T B
Fig. 1 Fig. 2
Case 1. Suppose that the jth column is a T-column, and the pj 1qth column is a B-
column. Consider some index i such that the ith row is an L-row; then pi, j 1q is a BL-block.
Therefore, pi 1, j q cannot be a TR-block (see Fig. 2), hence pi 1, j q is a TL-block, thus the
29
pi 1qth row is an L-row. Now, choosing the ith row to be the topmost L-row, we successively
obtain that all rows from the ith to the 50th are L-rows. Since we have exactly 25 L-rows, it
follows that the rows from the 1st to the 25th are R-rows, and the rows from the 26th to the
50th are L-rows.
Now consider the neighboring R-row and L-row (that are the rows with numbers 25 and
26). Replacing in the previous reasoning rows by columns and vice versa, the columns from the
1st to the 25th are T-columns, and the columns from the 26th to the 50th are B-columns. So
we have a unique arrangement of blocks that leads to the arrangement of kings satisfying the
condition of the problem (see Fig. 3).
TR TR BR BR k k k k k k kk kk
TR TR BR BR k k k k k k k k
TL TL BL BL k k k k k k k k
TL TL BL BL k k k k k k
Fig. 3 Fig. 4
Case 2. Suppose that the jth column is a B-column, and the pj 1qth column is a T-column.
Repeating the arguments from Case 1, we obtain that the rows from the 1st to the 25th are
L-rows (and all other rows are R-rows), the columns from the 1st to the 25th are B-columns
(and all other columns are T-columns), so we find exactly one more arrangement of kings (see
Fig. 4).
30
C4. Six stacks S1 , . . . , S6 of coins are standing in a row. In the beginning every stack contains
a single coin. There are two types of allowed moves:
Move 1 : If stack Sk with 1 ¨ k ¨ 5 contains at least one coin, you may remove one coin
from Sk and add two coins to Sk 1 .
Move 2 : If stack Sk with 1 ¨ k ¨ 4 contains at least one coin, then you may remove
one coin from Sk and exchange stacks Sk 1 and Sk 2 .
Decide whether it is possible to achieve by a sequence of such moves that the first five stacks
2010
are empty, whereas the sixth stack S6 contains exactly 20102010 coins.
1 2010
C4 . Same as Problem C4, but the constant 20102010 is replaced by 20102010 .
(Netherlands)
Answer. Yes (in both variants of the problem). There exists such a sequence of moves.
Solution. Denote by pa1 , a2 , . . . , an q Ñ pa11 , a12 , . . . , a1n q the following: if some consecutive stacks
contain a1 , . . . , an coins, then it is possible to perform several allowed moves such that the stacks
contain a11 , . . . , a1n coins respectively, whereas the contents of the other stacks remain unchanged.
Let A 20102010 or A 20102010 , respectively. Our goal is to show that
2010
..
2. 2
Lemma 2. For every positive integer n, let Pn lo2omoon (e.g. P3 22 16). Then
To decrease the number of coins in stack S4 , apply Move 2 to this stack repeatedly until its
size decreases to A{4. (In every step, we remove a coin from S4 and exchange the empty stacks
S5 and S6 .)
For this reason, the PSC suggests to consider the problem C4 as well. Problem C4 requires more
invention and technical care. On the other hand, the problem statement in C41 hides the fact that the
resulting amount of coins can be such incredibly huge and leaves this discovery to the students.
32
C5. n © 4 players participated in a tennis tournament. Any two players have played exactly
one game, and there was no tie game. We call a company of four players bad if one player
was defeated by the other three players, and each of these three players won a game and lost
another game among themselves. Suppose that there is no bad company in this tournament.
Let wi and ℓi be respectively the number of wins and losses of the ith player. Prove that
¸
n
pwi ℓiq3 © 0. (1)
i 1
(South Korea)
Solution. For any tournament T satisfying the problem condition, denote by S pT q sum under
consideration, namely
¸
n
S pT q pwi ℓiq3.
i 1
First, we show that the statement holds if a tournament T has only 4 players. Actually, let
A pa1 , a2 , a3 , a4 q be the number of wins of the players; we may assume that a1 © a2 © a3 © a4 .
We have a1 a2 a3 a4 42 6, hence a4 ¨ 1. If a4 0, then we cannot have
a1 a2 a3 2, otherwise the company of all players is bad. Hence we should have
A p3, 2, 1, 0q, and S pT q 33 13 p1q3 p3q3 0. On the other hand, if a4 1, then
only two possibilities, A p3, 1, 1, 1q and A p2, 2, 1, 1q can take place. In the former case we
have S pT q 33 3 p2q3 ¡ 0, while in the latter one S pT q 13 13 p1q3 p1q3 0, as
desired.
Now we turn to the general problem. Consider a tournament T with no bad companies and
enumerate the players by the numbers from 1 to n. For every 4 players i1 , i2 , i3 , i4 consider a
“sub-tournament” Ti1 i2 i3 i4 consisting of only these players and the games which they performed
with each other. By the abovementioned, we have S pTi1 i2 i3 i4 q © 0. Our aim is to prove that
¸
S pT q S pTi1 i2 i3 i4 q, (2)
i1 ,i2 ,i3 ,i4
where the sum is taken over all 4-tuples of distinct numbers from the set t1, . . . , nu. This way
the problem statement will be established.
We interpret the number pwi ℓi q3 as following. For i j, let εij 1 if the ith player wins
against the jth one, and εij 1 otherwise. Then
3
¸ ¸
pwi ℓiq3 εij εij1 εij2 εij3 .
j i
j1 ,j2 ,j3 i
Hence, ¸
S pT q εij1 εij2 εij3 .
i Rtj1 ,j2,j3 u
To simplify this expression, consider all the terms in this sum where two indices are equal.
If, for instance, j1 j2 , then the term contains ε2ij1 1, so we can replace this term by εij3 .
Make such replacements for each such term; obviously, after this change each term of the form
εij3 will appear P pT q times, hence
¸ ¸
S pT q εij1 εij2 εij3 P pT q εij S 1 pT q P pT qS2pT q.
|ti,j1 ,j2,j3 u|4
i j
33
We show that S2 pT q 0 and hence S pT q S1 pT q for each tournament. Actually, note that
εij εji, and the whole sum can be split into such pairs. Since the sum in each pair is 0, so
is S2 pT q.
Thus the desired equality (2) rewrites as
¸
S1 pT q S1 pTi1 i2 i3 i4 q. (3)
i1 ,i2 ,i3 ,i4
Now, if all the numbers j1 , j2 , j3 are distinct, then the set ti, j1 , j2 , j3 u is contained in exactly
one 4-tuple, hence the term εij1 εij2 εij3 appears in the right-hand part of (3) exactly once, as
well as in the left-hand part. Clearly, there are no other terms in both parts, so the equality is
established.
Solution 2. Similarly to the first solution, we call the subsets of players as companies, and
the k-element subsets will be called as k-companies.
In any company of the players, call a player the local champion of the company if he defeated
all other members of the company. Similarly, if a player lost all his games against the others
in the company then call him the local loser of the company. Obviously every company has
at most one local champion and at most one local loser. By the condition of the problem,
whenever a 4-company has a local loser, then this company has a local champion as well.
Suppose that k is some positive integer, and let us count all cases when a player is the local
champion of some k-company. The ith player won against wi other player. To be the local
champion of a k-company, he must be a member of the company, and the other k 1 members
must be chosen
from those whom he defeated. Therefore, the ith player is the local champion
wi
k1
of k-companies. Hence, the total number of local champions of all k-companies is
ņ
wi
k1
.
i1
¸
n
ℓi
k1
Similarly, the total number of local losers of the k-companies is .
i 1
¸
n ¸
n
Since every game has a winner and a loser, we have wi ℓi n
2
, and hence
i 1
i 1
¸
n
w i ℓi 0. (4)
i 1
In every 3-company, either the players defeated one another in a cycle or the company has
both a local champion and a local loser. Therefore, the total number of local champions and
¸n
¸n
In every 4-company, by the problem’s condition, the number of local losers is less than or
equal to the number of local champions. Then the same holds for the total numbers of local
34
n
¸ ¸
n
Now we establish the problem statement (1) as a linear combination of (4), (5) and (6). It
is easy check that
px yq 24
3 x
3
y
3
24
x
2
y
2
3px y q2 4 px y q.
Apply this identity to x w1 and y ℓi . Since every player played n 1 games, we have
wi ℓi n 1, and thus
pwi ℓiq3 24 wi
3
ℓi
3
24
wi
2
ℓi
2
3pn 1q2 4 w i ℓi .
Then
¸ n
¸ n
¸ n
n
¸
pwi ℓiq 24
3 wi
3
ℓi
3
24
wi ℓi
2
2
3pn 1q 4 2
w i ℓi © 0.
i 1
looooooooomooooooooon
i 1
ilooooooooomooooooooon
1
looooomooooon
i 1
©0 0 0
35
C6. Given a positive integer k and other two integers b ¡ w ¡ 1. There are two strings of
pearls, a string of b black pearls and a string of w white pearls. The length of a string is the
number of pearls on it.
One cuts these strings in some steps by the following rules. In each step:
(i) The strings are ordered by their lengths in a non-increasing order. If there are some
strings of equal lengths, then the white ones precede the black ones. Then k first ones (if they
consist of more than one pearl) are chosen; if there are less than k strings longer than 1, then
one chooses all of them.
(ii) Next, one cuts each chosen string into two parts differing in length by at most one.
(For instance, if there are strings of 5, 4, 4, 2 black pearls, strings of 8, 4, 3 white pearls and
k 4, then the strings of 8 white, 5 black, 4 white and 4 black pearls are cut into the parts
p4, 4q, p3, 2q, p2, 2q and p2, 2q, respectively.)
The process stops immediately after the step when a first isolated white pearl appears.
Prove that at this stage, there will still exist a string of at least two black pearls.
(Canada)
Solution 1. Denote the situation after the ith step by Ai ; hence A0 is the initial situation, and
Ai1 Ñ Ai is the ith step. We call a string containing m pearls an m-string; it is an m-w-string
or a m-b-string if it is white or black, respectively.
We continue the process until every string consists of a single pearl. We will focus on three
moments of the process: (a) the first stage As when the first 1-string (no matter black or
white) appears; (b) the first stage At where the total number of strings is greater than k (if
such moment does not appear then we put t 8); and (c) the first stage Af when all black
pearls are isolated. It is sufficient to prove that in Af 1 (or earlier), a 1-w-string appears.
We start with some easy properties of the situations under consideration. Obviously, we
have s ¨ f . Moreover, all b-strings from Af 1 become single pearls in the f th step, hence all
of them are 1- or 2-b-strings.
Next, observe that in each step Ai Ñ Ai 1 with i ¨ t 1, all p¡1q-strings were cut since
there are not more than k strings at all; if, in addition, i s, then there were no 1-string, so
all the strings were cut in this step.
Now, let Bi and bi be the lengths of the longest and the shortest b-strings in Ai , and
let Wi and wi be the same for w-strings. We show by induction on i ¨ mints, tu that (i) the
situation Ai contains exactly 2i black and 2i white strings, (ii) Bi © Wi , and (iii) bi © wi .
The base case i 0 is obvious. For the induction step, if i ¨ mints, tu then in the ith step,
each string is cut, thus the claim (i) follows from the induction hypothesis; next, we have
Bi rBi1 {2s © rWi1 {2s Wi and bi tbi1 {2u © twi1 {2u wi , thus establishing (ii)
and (iii).
For the numbers s, t, f , two cases are possible.
Case 1. Suppose that s ¨ t or f ¨ t 1 (and hence s ¨ t 1); in particular, this is true
when t 8. Then in As1 we have Bs1 © Ws1 , bs1 © ws1 ¡ 1 as s 1 ¨ mints, tu.
Now, if s f , then in As1 , there is no 1-w-string as well as no p¡2q-b-string. That is,
2 Bs1 © Ws1 © bs1 © ws1 ¡ 1, hence all these numbers equal 2. This means that
in As1 , all strings contain 2 pearls, and there are 2s1 black and 2s1 white strings, which
means b 2 2s1 w. This contradicts the problem conditions.
Hence we have s ¨ f 1 and thus s ¨ t. Therefore, in the sth step each string is cut
into two parts. Now, if a 1-b-string appears in this step, then from ws1 ¨ bs1 we see that a
36
1-w-string appears as well; so, in each case in the sth step a 1-w-string appears, while not all
black pearls become single, as desired.
Case 2. Now assume that t 1 ¨ s and t 2 ¨ f . Then in At we have exactly 2t white
and 2t black strings, all being larger than 1, and 2t 1 ¡ k © 2t (the latter holds since 2t is the
total number of strings in At1 ). Now, in the pt 1qst step, exactly k strings are cut, not more
than 2t of them being black; so the number of w-strings in At 1 is at least 2t pk 2t q k.
Since the number of w-strings does not decrease in our process, in Af 1 we have at least k
white strings as well.
Finally, in Af 1 , all b-strings are not larger than 2, and at least one 2-b-string is cut in
the f th step. Therefore, at most k 1 white strings are cut in this step, hence there exists a
w-string W which is not cut in the f th step. On the other hand, since a 2-b-string is cut, all
p©2q-w-strings should also be cut in the f th step; hence W should be a single pearl. This is
exactly what we needed.
Comment. In this solution, we used the condition b w only to avoid the case b w 2t . Hence,
if a number b w is not a power of 2, then the problem statement is also valid.
Solution 2. We use the same notations as introduced in the first paragraph of the previous
solution. We claim that at every stage, there exist a u-b-string and a v-w-string such that
either
(i) u ¡ v © 1, or
(ii) 2 ¨ u ¨ v 2u, and there also exist k 1 of p¡v {2q-strings other than considered
above.
First, we notice that this statement implies the problem statement. Actually, in both
cases (i) and (ii) we have u ¡ 1, so at each stage there exists a p©2q-b-string, and for the last
stage it is exactly what we need.
Now, we prove the claim by induction on the number of the stage. Obviously, for A0 the
condition (i) holds since b ¡ w. Further, we suppose that the statement holds for Ai , and prove
it for Ai 1 . Two cases are possible.
Case 1. Assume that in Ai , there are a u-b-string and a v-w-string with u ¡ v. We can
assume that v is the length of the shortest w-string in Ai ; since we are not at the final stage,
we have v © 2. Now, in the pi 1qst step, two subcases may occur.
Subcase 1a. Suppose that either no u-b-string is cut, or both some u-b-string and some
v-w-string are cut. Then in Ai 1 , we have either a u-b-string and a p¨v q-w-string (and (i) is
valid), or we have a ru{2s-b-string and a tv {2u-w-string. In the latter case, from u ¡ v we get
ru{2s ¡ tv{2u, and (i) is valid again.
Subcase 1b. Now, some u-b-string is cut, and no v-w-string is cut (and hence all the strings
which are cut are longer than v). If u1 ru{2s ¡ v, then the condition (i) is satisfied since we
have a u1 -b-string and a v-w-string in Ai 1 . Otherwise, notice that the inequality u ¡ v © 2
implies u1 © 2. Furthermore, besides a fixed u-b-string, other k 1 of p©v 1q-strings should
be cut in the pi 1qst step, hence providing at least k 1 of p©rpv 1q{2sq-strings, and
rpv 1q{2s ¡ v{2. So, we can put v1 v, and we have u1 ¨ v u ¨ 2u1, so the condition (ii)
holds for Ai 1 .
Case 2. Conversely, assume that in Ai there exist a u-b-string, a v-w-string (2 ¨ u ¨ v 2u)
and a set S of k 1 other strings larger than v {2 (and hence larger than 1). In the pi 1qst
step, three subcases may occur.
Subcase 2a. Suppose that some u-b-string is not cut, and some v-w-string is cut. The latter
one results in a tv {2u-w-string, we have v 1 tv {2u u, and the condition (i) is valid.
37
Subcase 2b. Next, suppose that no v-w-string is cut (and therefore no u-b-string is cut as
u ¨ v). Then all k strings which are cut have the length ¡ v, so each one results in a p¡v {2q-
string. Hence in Ai 1 , there exist k © k 1 of p¡v {2q-strings other than the considered u- and
v-strings, and the condition (ii) is satisfied.
Subcase 2c. In the remaining case, all u-b-strings are cut. This means that all p©uq-strings
are cut as well, hence our v-w-string is cut. Therefore in Ai 1 there exists a ru{2s-b-string
together with a tv {2u-w-string. Now, if u1 ru{2s ¡ tv {2u v 1 then the condition (i) is
fulfilled. Otherwise, we have u1 ¨ v 1 u ¨ 2u1 . In this case, we show that u1 © 2. If, to the
contrary, u1 1 (and hence u 2), then all black and white p©2q-strings should be cut in the
pi 1qst step, and among these strings there are at least a u-b-string, a v-w-string, and k 1
strings in S (k 1 strings altogether). This is impossible.
Hence, we get 2 ¨ u1 ¨ v 1 2u1 . To reach (ii), it remains to check that in Ai 1 , there exists
a set S 1 of k 1 other strings larger than v 1 {2. These will be exactly the strings obtained from
the elements of S. Namely, each s P S was either cut in the pi 1qst step, or not. In the former
case, let us include into S 1 the largest of the strings obtained from s; otherwise we include s
itself into S 1 . All k 1 strings in S 1 are greater than v {2 © v 1 , as desired.
38
¸
k
s©1 αi ppi 1q.
i 1
(Germany)
Solution 1. First, we prove the key lemma, and then we show how to apply it to finish the
solution.
Let n1 , . . . , nk be positive integers. By an n1 n2 nk grid we mean the set N
tpa1 , . . . , ak q : ai P Z, 0 ¨ ai ¨ ni 1u; the elements of N will be referred to as points. In this
grid, we define a subgrid as a subset of the form
L tpb1 , . . . , bk q P N : bi1
xi , . . . , bi xi u,
1 t t (1)
|W Y X | |W | |X | ¡ |f pW q| |f pX q X U 1 | |f pW q Y pf pX q X U 1 q| |f pW Y X q|.
This contradicts the maximality of |W |.
Thus, applying Hall’s lemma, we can assign to each L P U some vertex vij P U 1 so that to
distinct elements of U, distinct vertices of U 1 are assigned. In this situation, we say that L P U
corresponds to the ith axis, and write g pLq i. Since there are ni 1 vertices of the form vij ,
we get that for each 1 ¨ i ¨ ℓ, not more than ni 1 subgrids correspond to the ith axis.
Finally, we are ready to present the desired point. Since W V , there exists a point
b pb1 , b2 , . . . , bk q P N zpYLPW Lq. On the other hand, for every 1 ¨ i ¨ ℓ, consider any subgrid
L P U with g pLq i. This means exactly that L is orthogonal to the ith axis, and hence all
its elements have the same ith coordinate cL . Since there are at most ni 1 such subgrids,
there exists a number 0 ¨ ai ¨ ni 1 which is not contained in a set tcL : g pLq iu. Choose
such number for every 1 ¨ i ¨ ℓ. Now we claim that point a pa1 , . . . , aℓ , bℓ 1 , . . . , bk q is not
covered, hence contradicting the Lemma condition.
Surely, point a cannot lie in some L P U, since all the points in L have g pLqth coordinate
cL agpLq . On the other hand, suppose that a P L for some L P W ; recall that b R L. But the
points a and b differ only at first ℓ coordinates, so L should be orthogonal to at least one of
the first ℓ axes, and hence our graph contains some edge pL, vij q for i ¨ ℓ. It contradicts the
definition of W 1 . The Lemma is proved. l
Now we turn to the problem. Let dj be the step of the progression Pj . Note that since
n l.c.m.pd1 , . . . , ds q, for each 1 ¨ i ¨ k there exists an index j piq such that pαi i dj piq . We
assume that n ¡ 1; otherwise the problem statement is trivial.
For each 0 ¨ m ¨ n 1 and 1 ¨ i ¨ k, let mi be the residue of m modulo pαi i , and let
mi riαi . . . ri1 be the base pi representation of mi (possibly, with some leading zeroes). Now,
we put into correspondence to m the sequence r pmq pr11 , . . . , r1α1 , r21 , . . . , rkαk q. Hence r pmq
1 p1 loooooomoooooon
lies in a ploooooomoooooon pk pk grid N.
α1 times
αk times
Surely, if r pm q r pm q then pi
1 αi
mi m1i , which follows pαi i m m1 for all 1 ¨ i ¨ k;
consequently, n m m1 . So, when m runs over the set t0, . . . , n 1u, the sequences r pmq do
not repeat; since |N | n, this means that r is a bijection between t0, . . . , n 1u and N. Now
we will show that for each 1 ¨ i ¨ s, the set Li tr pmq : m P Pi u is a subgrid, and that for
each axis there exists a subgrid orthogonal to this axis. Obviously, these subgrids cover N, and
the condition (ii1 ) follows directly from (ii). Hence the Lemma provides exactly the estimate
we need.
Consider some 1 ¨ j ¨ s and let dj pγ11 . . . pγkk . Consider some q P Pj and let r pq q
pr11 , . . . , rkαk q. Then for an arbitrary q1 , setting rpq1q pr111 , . . . , rkα
1 q we have
k
q1P Pj ðñ pγi q q1 for each 1 ¨ i ¨ k ðñ ri,t ri,t1 for all t ¨ γi.
i
Comment 1. The estimate in the problem is sharp for every n. One of the possible examples is the
following one. For each 1 ¨ i ¨ k, 0 ¨ j ¨ αi 1, 1 ¨ k ¨ p 1, let
and add the progression P0 nZ. One can easily check that this set satisfies all the problem conditions.
There also exist other examples.
On the other hand, the estimate can be adjusted in the following sense. For every 1 ¨ i ¨ k, let
0 αi0 , αi1 , . . . , αihi be all the numbers of the form ordpi pdj q in an increasing order (we delete the
repeating occurences of a number, and add a number 0 αi0 if it does not occur). Then, repeating
the arguments from the solution one can obtain that
ķ ¸
hi
s©1 ppα α 1q.
j j 1
i 1j 1
Note that pα 1 © αpp 1q, and the equality is achieved only for α 1. Hence, for reaching the
minimal number of the progressions, one should have αi,j j for all i, j. In other words, for each
1 ¨ j ¨ αi , there should be an index t such that ordpi pdt q j.
Solution 2. We start with introducing some notation. For positive integer r, we denote
rrs t1, 2, . . . , ru. Next, we say that a set of progressions P tP1 , . . . , Psu cover Z if each
integer belongs to some of them; we say that this covering is minimal if no proper subset of P
covers Z. Obviously, each covering contains a minimal subcovering.
Next, for a minimal covering tP1 , . . . , Ps u and for every 1 ¨ i ¨ s, let di be the step of
progression Pi , and hi be some number which is contained in Pi but in none of the other
progressions. We assume that n ¡ 1, otherwise the problem is trivial. This implies di ¡ 1,
otherwise the progression Pi covers all the numbers, and n 1.
We will prove a more general statement, namely the following
Claim. Assume that the progressions P1 , . . . , Ps and number n pα1 1 . . . pαk k ¡ 1 are chosen as
in the problem statement. Moreover, choose some nonempty set of indices I ti1 , . . . , it u rk s
and some positive integer βi ¨ αi for every i P I. Consider the set of indices
! )
T j:1¨j ¨ s, and pαi β i i 1 dj for some i PI .
Then ¸
|T | © 1 βi ppi 1q. (2)
P
i I
Observe that the Claim for I rk s and βi αi implies the problem statement, since the
left-hand side in (2) is not greater than s. Hence, it suffices to prove the Claim.
1. First, we prove the Claim assuming that all dj ’s are prime numbers. If for some 1 ¨ i ¨ k
we have at least pi progressions with the step pi , then they do not intersect and hence cover all
the integers; it means that there are no other progressions, and n pi ; the Claim is trivial in
this case.
Now assume that for every 1 ¨ i ¨ k, there are not more than pi 1 progressions with
step pi ; each such progression covers the numbers with a fixed residue modulo pi , therefore
there exists a residue qi mod pi which is not touched by these progressions. By the Chinese
Remainder Theorem, there exists a number q such that q qi pmod pi q for all 1 ¨ i ¨ k; this
number cannot be covered by any progression with step pi , hence it is not covered at all. A
contradiction.
41
2. Now, we assume that the general Claim is not valid, and hence we consider a counterex-
ample tP1 , . . . , Ps u for the Claim; we can choose it to be minimal in the following sense:
the number ° n is minimal possible among all the counterexamples;
the sum i di is minimal possible among all the counterexamples having the chosen value
of n.
As was mentioned above, not all numbers di are primes; hence we can assume that d1 is
composite, say p1 d1 and d11 dp11 ¡ 1. Consider a progression P11 having the step d11 , and
containing P1 . We will focus on two coverings constructed as follows.
(i) Surely, the progressions P11 , P2 , . . . , Ps cover Z, though this covering in not necessarily
minimal. So, choose some minimal subcovering P 1 in it; surely P11 P P 1 since h1 is not covered
by P2 , . . . , Ps , so we may assume that P 1 tP11 , P2 , . . . , Ps1 u for some s1 ¨ s. Furthermore, the
period of the covering P 1 can appear to be less than n; so we denote this period by
n1 pα1 σ
1 1
. . . pkαk σk l.c.m. d11 , d2 , . . . , ds1 .
Observe that for each Pj R P 1 , we have hj P P11 , otherwise hj would not be covered by P.
(ii) On the other hand, each nonempty set of the form Ri Pi X P11 (1 ¨ i ¨ s) is also a
progression with a step ri l.c.m.pdi , d11q, and such sets cover P11 . Scaling these progressions
with the ratio 1{d11 , we obtain the progressions Qi with steps qi ri {d11 which cover Z. Now we
choose a minimal subcovering Q of this covering; again we should have Q1 P Q by the reasons
of h1 . Now, denote the period of Q by
l.c.m.tqi : Qi P Qu l.c.m.trid:1 Qi P Qu p1
γ1
. . . pγkk
n2 .
1 d11
Note that if hj P P11 , then the image of hj under the scaling can be covered by Qj only; so, in
this case we have Qj P Q.
Our aim is to find the desired number of progressions in coverings P and Q. First, we have
n © n1 , and the sum of the steps in P 1 is less than that in P; hence the Claim is valid for P 1 .
We apply it to the set of indices I 1 ti P I : βi ¡ σi u and the exponents βi1 βi σi ; hence
the set under consideration is
! 1 )
T1 j : 1 ¨ j ¨ s1 , and ppαi σi qβi
i
1
pαi i βi 1 dj for some i P I1 T X rs1s,
and we obtain that
¸ ¸
|T X rs1s| © |T 1| © 1 pβi σiqppi 1q 1 pβi σi q ppi 1q,
i I1P P
i I
where pxq maxtx, 0u; the latter equality holds as for i R I 1 we have βi ¨ σi .
Observe that x px y q mintx, y u for all x, y. So, if we find at least
¸
G mintβi , σi uppi 1q
P
i I
thus leading to a contradiction with the choice of P. We will find those indices among the
indices of progressions in Q.
42
3. Now denote I 2 ti P I : σi ¡ 0u and consider some i P I 2 ; then pαi i n1 . On the
other hand, there exists an index j piq such that pαi i dj piq ; this means that dj piq n1 and hence
Pj piq cannot appear in P 1 , so j piq ¡ s1 . Moreover, we have observed before that in this case
hj piq P P11 , hence Qj piq P Q. This means that
q j piq n , therefore γi αi for each i P I (recall
2 2
here that qi ri {d11 and hence dj piq rj piq d11 n2 ).
Let d11 pτ11 . . . pτkk . Then n2 pγ11 τ1 . . . pkγi τi . Now, if i P I 2 , then for every β the condition
pγ τ qβ 1 q is equivalent to pαi β 1 r .
pi i i j j
Note that n ¨ n{d1 n, hence we can apply the Claim to the covering Q. We perform
i
2 1
this with the set of indices I 2 and the exponents βi2 mintβi , σi u ¡ 0. So, the set under
consideration is
! )
T2 j : Qj P Q, and pipγ τ qmintβ ,σ u 1 qj for some i P I 2
i i i i
! )
α mintβ ,σ u 1 2
j : Qj P Q, and pi i i
rj for some i P I ,
i
and we obtain |T 2 | © 1 G. Finally, we claim that T 2 T X t1u Y ts1 1, . . . , su ; then we
will obtain |T X ts1 1, . . . , su| © G, which is exactly what we need.
To prove this, consider any j P T 2 . Observe first that αi mintβi , σi u 1 ¡ αi σi © τi ,
α mintβi ,σi u 1 α mintβi ,σi u 1
hence from pi i rj l.c.m.pd11 , dj q we have pi i dj , which means that
j P T . Next, the exponent of pi in dj is greater than that in n1 , which means that Pj R P 1 . This
may appear only if j 1 or j ¡ s1 , as desired. This completes the proof.
Comment 2. A grid analogue of the Claim is also valid. It reads as following.
Claim. Assume that the grid N is covered by subgrids L1 , L2 , . . . , Ls so that
(ii1 ) each subgrid contains a point which is not covered by other subgrids;
(iii) for each coordinate axis, there exists a subgrid Li orthogonal to this axis.
Choose some set of indices I ti1 , . . . , it u rks, and consider the set of indices
Solution 1. The line EF intersects the circumcircle at two points. Depending on the choice
of P , there are two different cases to consider.
Case 1 : The point P lies on the ray EF (see Fig. 1).
Let =CAB α, =ABC β and =BCA γ. The quadrilaterals BCEF and CAF D are
cyclic due to the right angles at D, E and F . So,
Q
γ
A A
α
E E γ P
γ γ γ γ
F F
P γ γ
γ
γ Q
β α γ γ
B D C B D C
Fig. 1 Fig. 2
45
Case 2 : The point P lies on the ray F E (see Fig. 2). In this case the point Q lies inside
the segment F D.
Similarly to the first case, we have
E P′
F γ
P γ
γ
Q′
γ
B D C
Fig. 3
Like in the first solution, we have =AF E =BF P =DF B =BCA γ from the
cyclic quadrilaterals BCEF and CAF D.
By P B P 1 A 2=AF P 1 2γ 2=BCA AP
P B, we have
AP 1 A, =P BA =ABP 1
P and AP AP 1. p1q
P
Due to AP 1 A, the lines BP and BQ1 are symmetrical about line AB.
Similarly, by =BF P =Q1 F B, the lines F P and F Q1 are symmetrical about AB. It
follows that also the points P and P 1 are symmetrical to Q1 and Q, respectively. Therefore,
AP AQ1 and AP 1
AQ. p2q
The relations (1) and (2) together prove AP AP 1 AQ AQ1 .
46
G2. Point P lies inside triangle ABC. Lines AP , BP , CP meet the circumcircle of ABC
again at points K, L, M, respectively. The tangent to the circumcircle at C meets line AB
at S. Prove that SC SP if and only if MK ML.
(Poland)
and
LM
PM
CB
PB
. Multiplying these two equalities, we get
LM
KM
CB PA
CA P B
.
Ω
L C
C
C
C
P
P
P
S
A E B
M
Fig. 1
Now we prove that S Q, thus establishing the problem statement. We have =CES
=CAE =ACE =BCS =ECB =ECS, so SC SE. Hence, the point S lies on AB
as well as on the perpendicular bisector of CE and therefore coincides with Q.
Solution 2. As in the previous solution, we assume that S lies on the ray AB.
1. Let P be an arbitrary point inside both the circumcircle ω of the triangle ABC and the
angle ASC, the points K, L, M defined as in the problem. We claim that SP SC implies
MK ML.
Let E and F be the points of intersection of the line SP with ω, point E lying on the
segment SP (see Fig. 2).
47
F L
K
ω
C
P
P
P
M
E
A B S
Fig. 2
We have SP 2 SA SB, so SB
SC 2
SP
SP
SA
, and hence △P SA △BSP . Then
=BP S =SAP . Since 2=BP S BE
LF and 2=SAP BE we have
EK
LF EK.
(1)
MF EM
. (2)
Ǒ
From (1) and (2) we get MF L MF
FL ME
EK MEK
Ǒ and hence MK ML.
The claim is proved.
2. We are left to prove the converse. So, assume that MK ML, and introduce the
points E and F as above. We have SC 2 SE SF ; hence, there exists a point P 1 lying on the
segment EF such that SP 1 SC (see Fig. 3).
L
L′ C
F
K′′′′′
K
K
P′′′′′
P
P K
P
ω
E
A B S
M
M′
Fig. 3
48
L C
F K
K
K
A A′ B S
ω
M
Fig. 4
Consider the point A1 on the ray SA for which =SP A1 =SBP ; in our case, this point lies
on the segment SA (see Fig. 4). Then △SBP △SP A1 and SP 2 SB SA1 SB SA SC 2 .
Therefore, SP SC which contradicts SP ¡ SC.
MF
Similarly, one can prove that the inequality ME is also impossible. So, we get
MF
ME and therefore 2=SCM EC EC
ME 2=SP C, which implies
MF
SC SP .
49
50
G3. Let A1 A2 . . . An be a convex polygon. Point P inside this polygon is chosen so that its
projections P1 , . . . , Pn onto lines A1 A2 , . . . , An A1 respectively lie on the sides of the polygon.
Prove that for arbitrary points X1 , . . . , Xn on sides A1 A2 , . . . , An A1 respectively,
" *
max
X 1 X2
P1 P2
,...,
X n X1
Pn P1
© 1.
(Armenia)
QED.
A4
X4 X3
Ω
A3
P
A5 Xi+1
Q
X2 ω Pi+1
X5
A1 X1 A2 Pi Xi Ai+1
Fig. 1 Fig. 2
51
Solution 2. As in Solution 1, we assume that all indices of points are considered modulo n.
We will prove a bit stronger inequality, namely
" *
cos αn © 1,
X1 X2 Xn X1
max cos α1 , . . . ,
P1 P2 Pn P1
where αi (1 ¨ i ¨ n) is the angle between lines Xi Xi 1 and Pi Pi 1 . We denote βi =Ai Pi Pi1
and γi =Ai 1 Pi Pi 1 for all 1 ¨ i ¨ n.
Suppose that for some 1 ¨ i ¨ n, point Xi lies on the segment Ai Pi , while point Xi 1 lies on
the segment Pi 1 Ai 2 . Then the projection of the segment Xi Xi 1 onto the line Pi Pi 1 contains
segment Pi Pi 1 , since γi and βi 1 are acute angles (see Fig. 3). Therefore, Xi Xi 1 cos αi ©
Pi Pi 1 , and in this case the statement is proved.
So, the only case left is when point Xi lies on segment Pi Ai 1 for all 1 ¨ i ¨ n (the case
when each Xi lies on segment Ai Pi is completely analogous).
Now, assume to the contrary that the inequality
Xi Xi 1 cos αi Pi Pi 1 (1)
holds for every 1 ¨ i ¨ n. Let Yi and Yi1 1 be the projections of Xi and Xi 1 onto Pi Pi 1 . Then
inequality (1) means exactly that YiYi1 1 Pi Pi 1 , or Pi Yi ¡ Pi 1 Yi1 1 (again since γi and βi 1
are acute; see Fig. 4). Hence, we have
Xi Pi cos γi ¡ Xi 1 Pi 1 cos βi 1 , 1 ¨ i ¨ n.
Multiplying these inequalities, we get
cos γ1 cos γ2 cos γn ¡ cos β1 cos β2 cos βn. (2)
On the other hand, the sines theorem applied to triangle P Pi Pi 1 provides
sinsin πβγi 1 cos
π
P Pi 2 βi 1
.
P Pi 1 2 i cos γi
Multiplying these equalities we get
1 cos
cos β2 cos β3 cos β1
cos γ1 cos γ2 γn
which contradicts (2).
′
Xi+1 Yi+1
Pi+1
βi+1
αi Ai+1
αi
Yi
Xi+1
Xi
P
Pi+1 P
βi+1 γi
γi Pi
βi
Xi Pi Ai+1
Pi−1 Xi−1 Ai
Fig. 3 Fig. 4
52
G4. Let I be the incenter of a triangle ABC and Γ be its circumcircle. Let the line AI
intersect Γ at a point D A. Let F and E be points on side BC and arc BDC respectively
such that =BAF =CAE 12 =BAC. Finally, let G be the midpoint of the segment IF .
Prove that the lines DG and EI intersect on Γ.
(Hong Kong)
Solution 1. Let X be the second point of intersection of line EI with Γ, and L be the foot
of the bisector of angle BAC. Let G1 and T be the points of intersection of segment DX with
lines IF and AF , respectively. We are to prove that G G1 , or IG1 G1 F . By the Menelaus
theorem applied to triangle AIF and line DX, it means that we need the relation
G1 F
1 TATF AD AD
TF ID
, or .
IG1 ID AT
Let the line AF intersect Γ at point K A (see Fig. 1); since =BAK =CAE we have
BK hence KE k BC. Notice that =IAT =DAK =EAD =EXD =IXT , so
CE,
the points I, A, X, T are concyclic. Hence we have =IT A =IXA =EXA =EKA, so
IT k KE k BC. Therefore we obtain
TF
AT
IL
AI
.
CL
AC
DC
AD
. Finally, it is known that the midpoint D of arc BC is equidistant from points I,
B, C, hence
DC
AD
ID
AD
.
Summarizing all these equalities, we get
TF
AT
AI
IL
AC
CL
DC
AD
ID
AD
,
as desired.
X A
A
B C
IIII
TT
G′
D
F
B L C
K E
D L
Fig. 1 Fig. 2
53
pDI AD q AI AB AC AF AE ñ AI
AE
ADAF DI ñ AT
AD
ADAF DI ,
as desired.
Comment. In fact, point J is an excenter of triangle ABC.
54
G5. Let ABCDE be a convex pentagon such that BC k AE, AB BC AE, and =ABC
=CDE. Let M be the midpoint of CE, and let O be the circumcenter of triangle BCD. Given
that =DMO 90 , prove that 2=BDA =CDE.
(Ukraine)
Solution 1. Choose point T on ray AE such that AT AB; then from AE k BC we have
=CBT =AT B =ABT , so BT is the bisector of =ABC. On the other hand, we have
ET AT AE AB AE BC, hence quadrilateral BCT E is a parallelogram, and the
midpoint M of its diagonal CE is also the midpoint of the other diagonal BT .
Next, let point K be symmetrical to D with respect to M. Then OM is the perpendicular
bisector of segment DK, and hence OD OK, which means that point K lies on the cir-
cumcircle of triangle BCD. Hence we have =BDC =BKC. On the other hand, the angles
BKC and T DE are symmetrical with respect to M, so =T DE =BKC =BDC.
Therefore, =BDT =BDE =EDT =BDE =BDC =CDE =ABC 180
=BAT . This means that the points A, B, D, T are concyclic, and hence =ADB =AT B
1
2
=ABC 12 =CDE, as desired.
B C
B C
α α
+
O 2ϕ β
D
M
M
M β
K 2ϕ − β − γ D
γγ
A E T −γ −β
α−β −α
2ϕ − 2ϕ
A E
N
N
N
N
N
N
O O
O
O
D M
M
M
M
M
D
D
D
E E
Let N be the midpoint of CD; then =DNO 90 =DMO, hence points M, N lie on
the circle with diameter OD. Now, if points O and M lie on the same side of CD, we have
=DMN =DON 12 =DOC α; in the other case, we have =DMN 180 =DON α;
55
so, in both cases =DMN α (see Figures). Next, since MN is a midline in triangle CDE,
we have =MDE =DMN α and =NDM 2ϕ α.
Now we apply the sine rule to the triangles ABD, ADE (twice), BCD and MND obtaining
AB p2ϕ β γ q ,
sinsin AE
sin γ DE
sinp2ϕ α β γ q
AD p2ϕ αq AD sinp2ϕ α β q
,
AD sinp2ϕ α β q
,
CD{2
BC
CD
sin β
sin α
,
CD
DE
DE {2
ND
NM
sin α
sinp2ϕ αq
,
which implies
BC
AD
CD DE AD sinsinp2ϕβ sinαpq2ϕ
BC CD DE α β γq .
sinp2ϕ α β q
Hence, the condition AB AE BC, or equivalently AEADBC , after multiplying
AB
AD
by the common denominator rewrites as
G6. The vertices X, Y , Z of an equilateral triangle XY Z lie respectively on the sides BC,
CA, AB of an acute-angled triangle ABC. Prove that the incenter of triangle ABC lies inside
triangle XY Z.
1
G6 . The vertices X, Y , Z of an equilateral triangle XY Z lie respectively on the sides
BC, CA, AB of a triangle ABC. Prove that if the incenter of triangle ABC lies outside
triangle XY Z, then one of the angles of triangle ABC is greater than 120 .
(Bulgaria)
Solution 1 for G6. We will prove a stronger fact; namely, we will show that the incenter I of
triangle ABC lies inside the incircle of triangle XY Z (and hence surely inside triangle XY Z
itself). We denote by dpU, V W q the distance between point U and line V W .
Denote by O the incenter of △XY Z and by r, r 1 and R1 the inradii of triangles ABC, XY Z
and the circumradius of XY Z, respectively. Then we have R1 2r 1 , and the desired inequality
is OI ¨ r 1 . We assume that O I; otherwise the claim is trivial.
Let the incircle of △ABC touch its sides BC, AC, AB at points A1 , B1 , C1 respectively.
The lines IA1 , IB1 , IC1 cut the plane into 6 acute angles, each one containing one of the
points A1 , B1 , C1 on its border. We may assume that O lies in an angle defined by lines IA1 ,
IC1 and containing point C1 (see Fig. 1). Let A1 and C 1 be the projections of O onto lines IA1
and IC1 , respectively.
Since OX R1 , we have dpO, BC q ¨ R1 . Since OA1 k BC, it follows that dpA1 , BC q
A1 I r ¨ R1 , or A1 I ¨ R1 r. On the other hand, the incircle of △XY Z lies inside △ABC,
hence dpO, AB q © r 1 , and analogously we get dpO, AB q C 1 C1 r IC 1 © r 1 , or IC 1 ¨ r r 1 .
B
X
C1
Z C′′′′′
C
C
C A1 C′
O
II
A′ O I
A B1 Y C A′
Fig. 1 Fig. 2
Finally, the quadrilateral IA1 OC 1 is circumscribed due to the right angles at A1 and C 1
(see Fig. 2). On its circumcircle, we have A Ǒ1 OC 1 2=A1 IC 1 180 OC
1I, hence 180 ©
1 ¡ A
IC 1 O. This means that IC 1 ¡ A1 O. Finally, we have OI ¨ IA1 A1 O IA1 IC 1 ¨
pR1 rq pr r1q R1 r1 r1, as desired.
Solution 2 for G6. Assume the contrary. Then the incenter I should lie in one of trian-
gles AY Z, BXZ, CXY — assume that it lies in △AY Z. Let the incircle ω of △ABC touch
sides BC, AC at point A1 , B1 respectively. Without loss of generality, assume that point A1
lies on segment CX. In this case we will show that =C ¡ 90 thus leading to a contradiction.
Note that ω intersects each of the segments XY and Y Z at two points; let U, U 1 and V ,
V 1 be the points of intersection of ω with XY and Y Z, respectively (UY ¡ U 1 Y , V Y ¡ V 1 Y ;
see Figs. 3 and 4). Note that 60 =XY Z 12 pUV U 1 V 1 q ¨ 1 UV
2
© 120 .
, hence UV
57
A
ω C1 B1
C1
VVV′′′′′ VV′′′′′
Y VV
Y V ′′ I I
U
U′′′
U
U V U′
B1 Z V
Z
U
U
U ω
U
C A1 X B C A1 X B
Fig. 3 Fig. 4
Case 1. Let point Y lie on the segment AB1 (see Fig. 3). Then we have =Y XC
1
A1U A1 U ¨ 2 UA1 U 30; analogously, we get =XY C ¨ 12 UA
1 1 Ǒ1 Ǒ 1 U 30 . Therefore,
1
=Y CX 180 =Y XC =XY C ¡ 120 , as desired.
2
Case 2. Now let point Y lie on the segment CB1 (see Fig. 4). Analogously, we obtain
=Y XC 30. Next, =IY X ¡ =ZY X 60, but =IY X =IY B1, since Y B1 is a tangent
and Y X is a secant line to circle ω from point Y . Hence, we get 120 =IY B1 =IY X
=B1 Y X =Y XC =Y CX 30 =Y CX, hence =Y CX ¡ 120 30 90, as desired.
Comment. In the same way, one can prove a more general
Claim. Let the vertices X, Y , Z of a triangle XY Z lie respectively on the sides BC, CA, AB of a
triangle ABC. Suppose that the incenter of triangle ABC lies outside triangle XY Z, and α is the
least angle of △XY Z. Then one of the angles of triangle ABC is greater than 3α 90 .
Solution for G61 . Assume the contrary. As in Solution 2, we assume that the incenter I of
△ABC lies in △AY Z, and the tangency point A1 of ω and BC lies on segment CX. Surely,
=Y ZA ¨ 180 =Y ZX 120, hence points I and Y lie on one side of the perpendicular
bisector to XY ; therefore IX ¡ IY . Moreover, ω intersects segment XY at two points, and
therefore the projection M of I onto XY lies on the segment XY . In this case, we will prove
that =C ¡ 120 .
Let Y K, Y L be two tangents from point Y to ω (points K and A1 lie on one side of XY ;
if Y lies on ω, we say K L Y ); one of the points K and L is in fact a tangency point B1
of ω and AC. From symmetry, we have =Y IK =Y IL. On the other hand, since IX ¡ IY ,
we get XM XY which implies =A1 XY =KY X.
Next, we have =MIY 90 =IY X 90 =ZY X 30 . Since IA1 K A1 X, IM K XY ,
IK K Y K we get =MIA1 =A1 XY =KY X =MIK. Finally, we get
A1 K(= B1 )
X Y
M
C L(= B1 )
A1 M
M
MM Y
B Z A
I
Fig. 5 Fig. 6
Comment 1. The estimate claimed in G61 is sharp. Actually, if =BAC ¡ 120 , one can consider an
equilateral triangle XY Z with Z A, Y P AC, X P BC (such triangle exists since =ACB 60 ). It
intersects with the angle bisector of =BAC only at point A, hence it does not contain I.
Comment 2. As in the previous solution, there is a generalization for an arbitrary triangle XY Z,
but here we need some additional condition. The statement reads as follows.
Claim. Let the vertices X, Y , Z of a triangle XY Z lie respectively on the sides BC, CA, AB of a
triangle ABC. Suppose that the incenter of triangle ABC lies outside triangle XY Z, α is the least
angle of △XY Z, and all sides of triangle XY Z are greater than 2r cot α, where r is the inradius
of △ABC. Then one of the angles of triangle ABC is greater than 2α.
The additional condition is needed to verify that XM ¡ Y M since it cannot be shown in the
original way. Actually, we have =M Y I ¡ α, IM r, hence Y M r cot α. Now, if we have
XY XM Y M ¡ 2r cot α, then surely XM ¡ Y M .
On the other hand, this additional condition follows easily from the conditions of the original
problem. Actually, if I P △AY Z, then the diameter of ω parallel to Y Z is contained in △AY Z and
is thus shorter than Y Z. Hence Y Z ¡ 2r ¡ 2r cot 60 .
59
60
G7. Three circular arcs γ1 , γ2 , and γ3 connect the points A and C. These arcs lie in the same
half-plane defined by line AC in such a way that arc γ2 lies between the arcs γ1 and γ3 . Point
B lies on the segment AC. Let h1 , h2 , and h3 be three rays starting at B, lying in the same
half-plane, h2 being between h1 and h3 . For i, j 1, 2, 3, denote by Vij the point of intersection
of hi and γj (see the Figure below).
Denote by VǑ Ǒ
ij Vkj Vkℓ Viℓ the curved quadrilateral, whose sides are the segments Vij Viℓ , Vkj Vkℓ
and arcs Vij Vkj and Viℓ Vkℓ . We say that this quadrilateral is circumscribed if there exists a circle
touching these two segments and two arcs.
Prove that if the curved quadrilaterals VǑ Ǒ ǑǑ ǑǑ
11 V21 V22 V12 , V12 V22 V23 V13 , V21 V31 V32 V22 are circum-
Ǒ Ǒ
scribed, then the curved quadrilateral V22 V32 V33 V23 is circumscribed, too.
h2
V23 h3
h1
V33
V13
V22
V12 V32
γ3
γ2 V21 V31
V11
γ1
A B C
Fig. 1
(Hungary)
Solution. Denote by Oi and Ri the center and the radius of γi , respectively. Denote also by H
the half-plane defined by AC which contains the whole configuration. For every point P in
the half-plane H, denote by dpP q the distance between P and line AC. Furthermore, for any
r ¡ 0, denote by ΩpP, r q the circle with center P and radius r.
Lemma 1. For every 1 ¨ i j ¨ 3, consider those circles ΩpP, r q in the half-plane H which
are tangent to hi and hj .
(a) The locus of the centers of these circles is the angle bisector βij between hi and hj .
(b) There is a constant uij such that r uij dpP q for all such circles.
Proof. Part (a) is obvious. To prove part (b), notice that the circles which are tangent to hi
and hj are homothetic with the common homothety center B (see Fig. 2). Then part (b) also
becomes trivial. l
Lemma 2. For every 1 ¨ i j ¨ 3, consider those circles ΩpP, r q in the half-plane H which
are externally tangent to γi and internally tangent to γj .
(a) The locus of the centers of these circles is an ellipse arc εij with end-points A and C.
(b) There is a constant vij such that r vij dpP q for all such circles.
Proof. (a) Notice that the circle ΩpP, r q is externally tangent to γi and internally tangent to γj
if and only if Oi P Ri r and Oj Rj r. Therefore, for each such circle we have
Oi P Oj P Oi A Oj A Oi C Oj C Ri Rj .
Such points lie on an ellipse with foci Oi and Oj ; the diameter of this ellipse is Ri Rj , and it
passes through the points A and C. Let εij be that arc AC of the ellipse which runs inside the
half plane H (see Fig. 3.)
This ellipse arc lies between the arcs γi and γj . Therefore, if some point P lies on εij ,
then Oi P ¡ Ri and Oj P Rj . Now, we choose r Oi P Ri Rj Oj P ¡ 0; then the
61
γj
Ω(P, r)
hi βij r P Rj εij
P′ r γi
Oj
r′ ~ρ
ρ~ρ
~
ρ
~j Ri
hj ~v
P A C
r
d(P ′ ) ρ
~i
Oi
d(P )
B
Fig. 2 Fig. 3
circle ΩpP, r q touches γi externally and touches γj internally, so P belongs to the locus under
investigation.
ÝÑ ÝÝÑ ÝÝÑ
(b) Let ρ~ AP , ρ~i AOi, and ρ~j AOj ; let dij OiOj , and let ~v be a unit vector
ÝÝÑ
orthogonal to AC and directed toward H. Then we have |ρ~i | Ri , |ρ~j | Rj , |OiP |
|ρ~ ρ~i| Ri r, |ÝOÝjÑ
P | |ρ~ ρ~j | Rj r, hence
Therefore,
r R dij R dpP q,
i j
Comment 1. Lemma 2(b) (together with the easy Lemma 1(b)) is the key tool in this solution.
If one finds this fact, then the solution can be finished in many ways. That is, one can find a circle
touching three of h2 , h3 , γ2 , and γ3 , and then prove that it is tangent to the fourth one in either
synthetic or analytical way. Both approaches can be successful.
Here we present some discussion about this key Lemma.
1. In the solution above we chose an analytic proof for Lemma 2(b) because we expect that most
students will use coordinates or vectors to examine the locus of the centers, and these approaches are
less case-sensitive.
Here we outline a synthetic proof. We consider only the case when P does not lie in the line Oi Oj .
The other case can be obtained as a limit case, or computed in a direct way.
Let S be the internal homothety center between the circles of γi and γj , lying on Oi Oj ; this point
does not depend on P . Let U and V be the points of tangency of circle σ ΩpP, r q with γi and γj ,
respectively (then r P U P V ); in other words, points U and V are the intersection points of
rays Oi P , Oj P with arcs γi , γj respectively (see Fig. 4).
Due to the theorem on three homothety centers (or just to the Menelaus theorem applied to
triangle Oi Oj P ), the points U , V and S are collinear. Let T be the intersection point of line AC and
the common tangent to σ and γi at U ; then T is the radical center of σ, γi and γj , hence T V is the
common tangent to σ and γj .
Let Q be the projection of P onto the line AC. By the right angles, the points U , V and Q lie on
the circle with diameter P T . From this fact and the equality P U P V we get =U QP =U V P
=V U P =SU Oi. Since Oi S k P Q, we have =SOi U =QP U . Hence, the triangles SOiU and U P Q
are similar and thus
r
dpP q
PU
PQ
Oi S
Oi U
Oi S
Ri
; the last expression is constant since S is a constant
point. l
ℓ
γj dℓ (P )
σ εij
V P
dℓ (A) Oj
P
d(P ))
d(P
U
U
U γi
Oj
A C
S
T A Q C Oi
Oi
Fig. 4 Fig. 5
2. Using some known facts about conics, the same statement can be proved in a very short way.
Denote by ℓ the directrix of ellipse of εij related to the focus Oj ; since εij is symmetrical about Oi Oj ,
we have ℓ k AC. Recall that for each point P P εij , we have P Oj ǫ dℓ pP q, where dℓ pP q is the
distance from P to ℓ, and ǫ is the eccentricity of εij (see Fig. 5).
Now we have
r Rj pRj rq AOj P Oj ǫ dℓ pAq dℓpP q ǫ dpP q dpAq ǫ dpP q,
and ǫ does not depend on P . l
63
Comment 2. One can find a spatial interpretations of the problem and the solution.
For every point px, y q and radius r ¡ 0, represent the circle Ω px, y q, r by the point px, y, r q
in space. This point is the apex of the cone with base circle Ω px, y q, r and height r. According to
Lemma 1, the circles which are tangent to hi and hj correspond to the points of a half line βij1 , starting
at B.
Now we translate Lemma 2. Take some 1 ¨ i j ¨ 3, and consider those circles which are
internally tangent to γj . It is easy to see that the locus of the points which represent these circles is
a subset of a cone, containing γj . Similarly, the circles which are externally tangent to γi correspond
to the points on the extension of another cone, which has its apex on the opposite side of the base
plane Π. (See Fig. 6; for this illustration, the z-coordinates were multiplied by 2.)
The two cones are symmetric to each other (they have the same aperture, and their axes are
parallel). As is well-known, it follows that the common points of the two cones are co-planar. So the
intersection of the two cones is a a conic section — which is an ellipse, according to Lemma 2(a). The
points which represent the circles touching γi and γj is an ellipse arc ε1ij with end-points A and C.
′
Σ β12 ′
β23
ε′23
ε′ij
ε′12
γi
γj
Π
Fig. 6 Fig. 7
1
Thus, the curved quadrilateral Qij is circumscribed if and only if βi,i and ε1j,j 1 intersect, i.e. if
they are coplanar. If three of the four curved quadrilaterals are circumscribed, it means that ε112 , ε123 ,
1
1 and β 1 lie in the same plane Σ, and the fourth intersection comes to existence, too (see Fig. 7).
β12 23
1 1 ... 1 2010
1 1 1 51
.
s1 s2 sn
1
51 42
N1 . Same as Problem N1, but the constant is replaced by .
2010 2010
(Canada)
Answer for Problem N1. n 39.
Solution for Problem N1. Suppose that for some n there exist the desired numbers; we
may assume that s1 s2 sn . Surely s1 ¡ 1 since otherwise 1 0. So we have
1
s1
2 ¨ s1 ¨ s2 1 ¨ ¨ sn pn 1q, hence si © i 1 for each i 1, . . . , n. Therefore
51
2010
1
1
s1
1
1
s2
... 1
1
sn
© 1
1
2
1
1
3
... 1
1
n 1
1 2
2 3
n
n 1
n 1 1,
which implies
1© 670 ¡ 39,
2010
n
51 17
so n © 39.
Now we are left to show that n 39 fits. Consider the set t2, 3, . . . , 33, 35, 36, . . . , 40, 67u
which contains exactly 39 numbers. We have
1 2
2 3
32 34
33 35
39 66
40 67
331 34
40 67
66
17
670
51
2010
, p1q
hence for n 39 there exists a desired example.
Comment. One can show that the example p1q is unique.
Answer for Problem N11 . n 48.
Solution for Problem N11 . Suppose that for some n there exist the desired numbers. In
the same way we obtain that si © i 1. Moreover, since the denominator of the fraction
42
2010
7
335
is divisible by 67, some of si ’s should be divisible by 67, so sn © si © 67. This
means that
n1
42
2010
©
1 2
2 3
n
1
1
67
66
67n
,
65
which implies
2010 66
n© 330 ¡ 47,
42 67 7
so n © 48.
Now we are left to show that n 48 fits. Consider the set t2, 3, . . . , 33, 36, 37, . . . , 50, 67u
which contains exactly 48 numbers. We have
1 2
2 3
32 35
33 36
49 66
50 67
331 35
50 67
66
7
335
42
2010
,
1 2
2 3
46 66 329
47 67 330
671 330 47 677 5 2010
66 329 42
.
Comment 2. N11 was the Proposer’s formulation of the problem. We propose N1 according to the
number of current IMO.
66
(Australia)
Answer. p6, 3q, p9, 3q, p9, 5q, p54, 5q.
Solution. For fixed values of n, the equation (1) is a simple quadratic equation in m. For
n ¨ 5 the solutions are listed in the following table.
case equation discriminant integer roots
n0 m2 m 2 0 7 none
n1 m2 3m 6 0 15 none
n2 m2 7m 18 0 23 none
n3 m2 15m 54 0 9 m 6 and m 9
n4 m2 31m 162 0 313 none
n5 m2 63m 486 0 2025 452 m 9 and m 54
We prove that there is no solution for n © 6.
Suppose that pm, nq satisfies (1) and n © 6. Since m 2 3n m 2n 1
1 m2 , we have
m 3p with some 0 ¨ p ¨ n or m 2 3q with some 0 ¨ q ¨ n.
In the first case, let q n p; then
2 3n
2n 1
1m m
3p 2 3q .
3p 2 3q 2n 1 1, p q n. (2)
3p 2n 1 8 n 1
3 9 n 1
3 3 p 2 n 1
3
q
and
2 3q 2n 1 2 8 2 9 2 3 2 3 p q ,
n
3
n
3
2n
3
2 n 1
3
n2
p, q 2pn 1q .
3 3
(3)
2n 1
1 43r 1 p42r 4r 1qp2r 1qp2r 1q. (4)
67
Notice that the factor 42r 4r 1 p4r 1q2 3 4r is divisible by 3, but it is never
divisible by 9. The other two factors in (4), 2r 1 and 2r 1 are coprime: both are odd and
their difference is 2. Since the whole product is divisible by 3h , we have either 3h1 2r 1 or
3h1 2r 1. In any case, we have 3h1 ¨ 2r 1. Then
3h1 ¨ 2r 1 ¨ 3r 3 6 ,
n 1
n2
3
1 h1¨
n 1
6
,
n 11.
But this is impossible since we assumed n © 6, and we proved 6 n 1.
68
N3. Find the smallest number n such that there exist polynomials f1 , f2 , . . . , fn with rational
coefficients satisfying
x2 7 f1 pxq2 f2 pxq2 fn pxq2 .
(Poland)
Answer. The smallest n is 5.
¸
4 ¸
4 ¸
4
(i) x2i 8m2, (ii) yi2 8m2, (iii) xi yi 6m2.
i 1
i 1
i 1
pa21 a22 a23 a24 qpb21 b22 b23 b24 q pa1 b1 a2 b2 a3 b3 a4 b4 q2 pa1 b2 a2b1 a3 b4 a4 b3 q2
pa1 b3 a3b1 a4 b2 a2 b4 q2 pa1 b4 a4b1 a2 b3 a3 b2 q2 .
69
So, using the relations (1) from the Solution 1 we get that
m 2 m 2 m 2
7
1 2 3
, (2)
m m m
where
m1
m
a1 b2 a2b1 a3 b4 a4 b3 ,
m2
m
a1 b3 a3b1 a4 b2 a2 b4 ,
m3
m
a1 b4 a4b1 a2 b3 a3 b2
and m1 , m2 , m3 P Z, m P N.
Let m be a minimum positive integer number for which (2) holds. Then
N4. Let a, b be integers, and let P pxq ax3 bx. For any positive integer n we say that the
pair pa, bq is n-good if n P pmq P pk q implies n m k for all integers m, k. We say that
pa, bq is very good if pa, bq is n-good for infinitely many positive integers n.
(a) Find a pair pa, bq which is 51-good, but not very good.
(b) Show that all 2010-good pairs are very good.
(Turkey)
Solution. (a) We show that the pair p1, 512 q is good but not very good. Let P pxq x3 512x.
Since P p51q P p0q, the pair p1, 512q is not n-good for any positive integer that does not
divide 51. Therefore, p1, 512 q is not very good.
On the other hand, if P pmq P pk q pmod 51q, then m3 k 3 pmod 51q. By Fermat’s
theorem, from this we obtain
m m3 k3 k pmod 3q and m m33 k33 k pmod 17q.
Hence we have m k pmod 51q. Therefore p1, 512q is 51-good.
(b) We will show that if a pair pa, bq is 2010-good then pa, bq is 67i -good for all positive
integer i.
Claim 1. If pa, bq is 2010-good then pa, bq is 67-good.
Proof. Assume that P pmq P pk q pmod 67q. Since 67 and 30 are coprime, there exist integers
m1 and k 1 such that k 1 k pmod 67q, k 1 0 pmod 30q, and m1 m pmod 67q, m1 0
pmod 30q. Then we have P pm1q P p0q P pk1q pmod 30q and P pm1q P pmq P pkq P pk1q
pmod 67q, hence P pm1q P pk1q pmod 2010q. This implies m1 k1 pmod 2010q as pa, bq is
2010-good. It follows that m m1 k 1 k pmod 67q. Therefore, pa, bq is 67-good.
l
Claim 2. If pa, bq is 67-good then 67 a.
Proof. Suppose that 67 a. Consider the sets tat2 pmod 67q : 0 ¨ t ¨ 33u and t3as2 b
mod 67 : 0 ¨ s ¨ 33u. Since a 0 pmod 67q, each of these sets has 34 elements. Hence they
have at least one element in common. If at2 3as2 b pmod 67q then for m t s, k 2s
we have
P pmq P pk q apm3 k 3 q bpm k q pm k q apm2 mk k 2 q b
pt 3sqpat2 3as2 bq 0 pmod 67q.
Since pa, bq is 67-good, we must have m k pmod 67q in both cases, that is, t 3s pmod 67q
and t 3s pmod 67q. This means t s 0 pmod 67q and b 3as2 at2 0 pmod 67q.
But then 67 P p7q P p2q 67 5a 5b and 67 7 2, contradicting that pa, bq is 67-good. l
Claim 3. If pa, bq is 2010-good then pa, bq is 67i-good all i © 1.
Proof. By Claim 2, we have 67 a. If 67 b, then P pxq P p0q pmod 67q for all x, contradicting
that pa, bq is 67-good. Hence, 67 b.
Suppose that 67i P pmq P pk q pm k q apm2 mk k 2 q b . Since 67 a and 67 b,
the second factor apm2 mk k 2 q b is coprime to 67 and hence 67i m k. Therefore, pa, bq
is 67i -good. l
Comment 1. In the proof of Claim 2, the following reasoning can also be used. Since 3 is not
a quadratic residue modulo 67, either au2 b pmod 67q or 3av 2 b pmod 67q has a solution.
The settings pm, kq pu, 0q in the first case and pm, kq pv, 2v q in the second case lead to b 0
pmod 67q.
Comment 2. The pair p67, 30q is n-good if and only if n d 67i , where d 30 and i © 0. It shows
that in part (b), one should deal with the large powers of 67 to reach the solution. The key property
of number 67 is that it has the form 3k 1, so there exists a nontrivial cubic root of unity modulo 67.
71
N5. Let N be the set of all positive integers. Find all functions f : N Ñ N such that the
number f pmq n m f pnq is a square for all m, n P N.
(U.S.A.)
Answer. All functions of the form f pnq n c, where c P N Y t0u.
Solution. First, it is clear that all functions of the form f pnq n c with a constant nonneg-
ative integer c satisfy the problem conditions since f pmq n f pnq m pn m cq2 is a
square.
We are left to prove that there are no other functions. We start with the following
Lemma. Suppose that p f pk q f pℓq for some prime p and positive integers k, ℓ. Then p k ℓ.
Proof. Suppose first that p2 f pk q f pℓq, so f pℓq f pk q p2 a for some integer a. Take some
positive integer D ¡ maxtf pk q, f pℓqu which is not divisible by p and set n pD f pk q. Then
the positive numbers n f pk q pD and n f pℓq pD f pℓq f pk q ppD paq are
both divisible by p but not byp2 . Now, applying the problem
conditions, we get that both the
numbers f pk q n f pnq k and f pℓq n f pnq ℓ are squares divisible by p (and thus
by p2 ); this means
that the multipliers f pnq k and f pnq ℓ are also divisible by p, therefore
p f pnq k f pnq ℓ k ℓ as well.
On the other hand, if f pk q f pℓq is divisible by p but not by p2 , then choose the same
number D and set n p3 D f pk q. Then the positive numbers f pk q n p3 D and f pℓq n
p3 D f pℓq f pk q are respectively divisible by p3 (but not by p4 ) and by p (but not by p2 ).
Hence in analogous
way
we obtain that
the numbers f pnq k and f pnq ℓ are divisible by p,
therefore p f pnq k f pnq ℓ k ℓ.
l
We turn to the problem. First, suppose that f pk q f pℓq for some k, ℓ P N. Then by Lemma
we have that k ℓ is divisible by every prime number, so k ℓ 0, or k ℓ. Therefore, the
function f is injective.
Next, consider the numbers f pk q and f pk 1q. Since the number pk 1q k 1 has no
prime divisors, by Lemma the same holds for f pk 1q f pk q; thus |f pk 1q f pk q| 1.
Now, let f p2q f p1q q, |q | 1. Then we prove by induction that f pnq f p1q q pn 1q.
The base for n 1, 2 holds by the definition of q. For the step, if n ¡ 1 we have f pn 1q
f pnq q f p1q q pn 1q q. Since f pnq f pn 2q f p1q q pn 2q, we get f pnq f p1q qn,
as desired.
Finally, we have f pnq f p1q q pn 1q. Then q cannot be 1 since otherwise for n © f p1q 1
we have f pnq ¨ 0 which is impossible. Hence q 1 and f pnq pf p1q 1q n for each n P N,
and f p1q 1 © 0, as desired.
72
N6. The rows and columns of a 2n 2n table are numbered from 0 to 2n 1. The cells of the
table have been colored with the following property being satisfied: for each 0 ¨ i, j ¨ 2n 1,
the jth cell in the ith row and the pi j qth cell in the jth row have the same color. (The
indices of the cells in a row are considered modulo 2n .)
Prove that the maximal possible number of colors is 2n .
(Iran)
Solution. Throughout the solution we denote the cells of the table by coordinate pairs; pi, j q
refers to the jth cell in the ith row.
Consider the directed graph, whose vertices are the cells of the board, and the edges are
the arrows pi, j q Ñ pj, i j q for all 0 ¨ i, j ¨ 2n 1. From each vertex pi, j q, exactly one edge
passes (to pj, i j mod 2n q); conversely, to each cell pj, k q exactly one edge is directed (from
the cell pk j mod 2n , j qq. Hence, the graph splits into cycles.
Now, in any coloring considered, the vertices of each cycle should have the same color by
the problem condition. On the other hand, if each cycle has its own color, the obtained coloring
obviously satisfies the problem conditions. Thus, the maximal possible number of colors is the
same as the number of cycles, and we have to prove that this number is 2n .
Next, consider any cycle pi1 , j1 q, pi2 , j2 q, . . . ; we will describe it in other terms. Define a
sequence pa0 , a1 , . . . q by the relations a0 i1 , a1 j1 , an 1 an an1 for all n © 1 (we
say that such a sequence is a Fibonacci-type sequence). Then an obvious induction shows
that ik ak1 pmod 2n q, jk ak pmod 2n q. Hence we need to investigate the behavior of
Fibonacci-type sequences modulo 2n .
Denote by F0 , F1 , . . . the Fibonacci numbers defined by F0 0, F1 1, and Fn 2
Fn 1 Fn for n © 0. We also set F1 1 according to the recurrence relation.
For every positive integer m, denote by ν pmq the exponent of 2 in the prime factorization
of m, i.e. for which 2 ν pmq
m but 2 m.
ν pmq 1
Comment. We outline a different proof for the essential part of Lemma 3. That is, we assume that
k 0 and show that in this case the period of pai q modulo 2n coincides with the period of the Fibonacci
numbers modulo 2n ; then the proof can be finished by the arguments from Lemma 2..
Note that p is a (not necessarily minimal) period of the sequence pai q modulo 2n if and only if we
have a0 ap pmod 2n q, a1 ap 1 pmod 2n q, that is,
Problem Shortlist
with Solutions
International
IMO2011 Mathematical
Amsterdam Olympiad Am
sterdam 2011
52nd International
Mathematical Olympiad
12-24 July 2011
Amsterdam
The Netherlands
Problem shortlist
with solutions
IMPORTANT
IMO regulation:
these shortlist problems have to
be kept strictly confidential
until IMO 2012.
Algebra
A1
A1
For any set A = {a1 , a2 , a3 , a4 } of four distinct positive integers with sum sA = a1 + a2 + a3 + a4 ,
let pA denote the number of pairs (i, j) with 1 ≤ i < j ≤ 4 for which ai + aj divides sA . Among
all sets of four distinct positive integers, determine those sets A for which pA is maximal.
A2
A2
Determine all sequences (x1 , x2 , . . . , x2011 ) of positive integers such that for every positive inte-
ger n there is an integer a with
A3
A3
Determine all pairs (f, g) of functions from the set of real numbers to itself that satisfy
A4
A4
Determine all pairs (f, g) of functions from the set of positive integers to itself that satisfy
A5
A5
Prove that for every positive integer n, the set {2, 3, 4, . . . , 3n + 1} can be partitioned into
n triples in such a way that the numbers from each triple are the lengths of the sides of some
obtuse triangle.
4
52nd IMO 2011 Problem shortlist Algebra
A6
A6
Let f be a function from the set of real numbers to itself that satisfies
f (x + y) ≤ yf (x) + f (f (x))
for all real numbers x and y. Prove that f (x) = 0 for all x ≤ 0.
A7
A7
√
Let a, b, and c be positive real numbers satisfying min(a+b, b+c, c+a) > 2 and a2 +b2 +c2 = 3.
Prove that
a b c 3
2
+ 2
+ 2
≥ .
(b + c − a) (c + a − b) (a + b − c) (abc)2
5
Combinatorics Problem shortlist 52nd IMO 2011
Combinatorics
C1
C1
Let n > 0 be an integer. We are given a balance and n weights of weight 20 , 21 , . . . , 2n−1 . In a
sequence of n moves we place all weights on the balance. In the first move we choose a weight
and put it on the left pan. In each of the following moves we choose one of the remaining
weights and we add it either to the left or to the right pan. Compute the number of ways in
which we can perform these n moves in such a way that the right pan is never heavier than the
left pan.
C2
C2
Suppose that 1000 students are standing in a circle. Prove that there exists an integer k with
100 ≤ k ≤ 300 such that in this circle there exists a contiguous group of 2k students, for which
the first half contains the same number of girls as the second half.
C3
C3
Let S be a finite set of at least two points in the plane. Assume that no three points of S are
collinear. By a windmill we mean a process as follows. Start with a line ℓ going through a
point P ∈ S. Rotate ℓ clockwise around the pivot P until the line contains another point Q
of S. The point Q now takes over as the new pivot. This process continues indefinitely, with
the pivot always being a point from S.
Show that for a suitable P ∈ S and a suitable starting line ℓ containing P , the resulting
windmill will visit each point of S as a pivot infinitely often.
C4
C4
Determine the greatest positive integer k that satisfies the following property: The set of positive
integers can be partitioned into k subsets A1 , A2 , . . . , Ak such that for all integers n ≥ 15 and
all i ∈ {1, 2, . . . , k} there exist two distinct elements of Ai whose sum is n.
6
52nd IMO 2011 Problem shortlist Combinatorics
C5
C5
Let m be a positive integer and consider a checkerboard consisting of m by m unit squares.
At the midpoints of some of these unit squares there is an ant. At time 0, each ant starts
moving with speed 1 parallel to some edge of the checkerboard. When two ants moving in
opposite directions meet, they both turn 90◦ clockwise and continue moving with speed 1.
When more than two ants meet, or when two ants moving in perpendicular directions meet,
the ants continue moving in the same direction as before they met. When an ant reaches one
of the edges of the checkerboard, it falls off and will not re-appear.
Considering all possible starting positions, determine the latest possible moment at which the
last ant falls off the checkerboard or prove that such a moment does not necessarily exist.
C6
C6
Let n be a positive integer and let W = . . . x−1 x0 x1 x2 . . . be an infinite periodic word consisting
of the letters a and b. Suppose that the minimal period N of W is greater than 2n .
A finite nonempty word U is said to appear in W if there exist indices k ≤ ℓ such that
U = xk xk+1 . . . xℓ . A finite word U is called ubiquitous if the four words Ua, Ub, aU, and bU
all appear in W . Prove that there are at least n ubiquitous finite nonempty words.
C7
C7
On a square table of 2011 by 2011 cells we place a finite number of napkins that each cover
a square of 52 by 52 cells. In each cell we write the number of napkins covering it, and we
record the maximal number k of cells that all contain the same nonzero number. Considering
all possible napkin configurations, what is the largest value of k?
7
Geometry Problem shortlist 52nd IMO 2011
Geometry
G1
G1
Let ABC be an acute triangle. Let ω be a circle whose center L lies on the side BC. Suppose
that ω is tangent to AB at B ′ and to AC at C ′ . Suppose also that the circumcenter O of the
triangle ABC lies on the shorter arc B ′ C ′ of ω. Prove that the circumcircle of ABC and ω
meet at two points.
G2
G2
Let A1 A2 A3 A4 be a non-cyclic quadrilateral. Let O1 and r1 be the circumcenter and the
circumradius of the triangle A2 A3 A4 . Define O2 , O3 , O4 and r2 , r3 , r4 in a similar way. Prove
that
1 1 1 1
+ + + = 0.
O1 A1 − r1 O2 A2 − r2 O3 A3 − r3 O4 A4 − r42
2 2 2 2 2 2 2
G3
G3
Let ABCD be a convex quadrilateral whose sides AD and BC are not parallel. Suppose that the
circles with diameters AB and CD meet at points E and F inside the quadrilateral. Let ωE be
the circle through the feet of the perpendiculars from E to the lines AB, BC, and CD. Let ωF
be the circle through the feet of the perpendiculars from F to the lines CD, DA, and AB.
Prove that the midpoint of the segment EF lies on the line through the two intersection points
of ωE and ωF .
G4
G4
Let ABC be an acute triangle with circumcircle Ω. Let B0 be the midpoint of AC and let C0
be the midpoint of AB. Let D be the foot of the altitude from A, and let G be the centroid
of the triangle ABC. Let ω be a circle through B0 and C0 that is tangent to the circle Ω at a
point X 6= A. Prove that the points D, G, and X are collinear.
G5
G5
Let ABC be a triangle with incenter I and circumcircle ω. Let D and E be the second
intersection points of ω with the lines AI and BI, respectively. The chord DE meets AC at a
point F , and BC at a point G. Let P be the intersection point of the line through F parallel to
AD and the line through G parallel to BE. Suppose that the tangents to ω at A and at B meet
at a point K. Prove that the three lines AE, BD, and KP are either parallel or concurrent.
8
52nd IMO 2011 Problem shortlist Geometry
G6
G6
Let ABC be a triangle with AB = AC, and let D be the midpoint of AC. The angle bisector
of ∠BAC intersects the circle through D, B, and C in a point E inside the triangle ABC.
The line BD intersects the circle through A, E, and B in two points B and F . The lines AF
and BE meet at a point I, and the lines CI and BD meet at a point K. Show that I is the
incenter of triangle KAB.
G7
G7
Let ABCDEF be a convex hexagon all of whose sides are tangent to a circle ω with center O.
Suppose that the circumcircle of triangle ACE is concentric with ω. Let J be the foot of the
perpendicular from B to CD. Suppose that the perpendicular from B to DF intersects the
line EO at a point K. Let L be the foot of the perpendicular from K to DE. Prove that
DJ = DL.
G8
G8
Let ABC be an acute triangle with circumcircle ω. Let t be a tangent line to ω. Let ta , tb ,
and tc be the lines obtained by reflecting t in the lines BC, CA, and AB, respectively. Show
that the circumcircle of the triangle determined by the lines ta , tb , and tc is tangent to the
circle ω.
9
Number Theory Problem shortlist 52nd IMO 2011
Number Theory
N1
N1
For any integer d > 0, let f (d) be the smallest positive integer that has exactly d positive
divisors (so for example we have f (1) = 1, f (5) = 16, and f (6) = 12). Prove that for every
integer k ≥ 0 the number f (2k ) divides f (2k+1).
N2
N2
Consider a polynomial P (x) = (x + d1 )(x + d2 ) · . . . · (x + d9 ), where d1 , d2 , . . . , d9 are nine
distinct integers. Prove that there exists an integer N such that for all integers x ≥ N the
number P (x) is divisible by a prime number greater than 20.
N3
N3
Let n ≥ 1 be an odd integer. Determine all functions f from the set of integers to itself such
that for all integers x and y the difference f (x) − f (y) divides xn − y n .
N4
N4
For each positive integer k, let t(k) be the largest odd divisor of k. Determine all positive
integers a for which there exists a positive integer n such that all the differences
are divisible by 4.
N5
N5
Let f be a function from the set of integers to the set of positive integers. Suppose that for
any two integers m and n, the difference f (m) − f (n) is divisible by f (m − n). Prove that for
all integers m, n with f (m) ≤ f (n) the number f (n) is divisible by f (m).
N6
N6
Let P (x) and Q(x) be two polynomials with integer coefficients such that no nonconstant
polynomial with rational coefficients divides both P (x) and Q(x). Suppose that for every
positive integer n the integers P (n) and Q(n) are positive, and 2Q(n) − 1 divides 3P (n) − 1.
Prove that Q(x) is a constant polynomial.
10
52nd IMO 2011 Problem shortlist Number Theory
N7
N7
Let p be an odd prime number. For every integer a, define the number
a a2 ap−1
Sa = + +···+ .
1 2 p−1
N8
N8
Let k be a positive integer and set n = 2k + 1. Prove that n is a prime number if and only if
the following holds: there is a permutation a1 , . . . , an−1 of the numbers 1, 2, . . . , n − 1 and a
sequence of integers g1 , g2 , . . . , gn−1 such that n divides giai − ai+1 for every i ∈ {1, 2, . . . , n − 1},
where we set an = a1 .
11
A1 Algebra – solutions 52nd IMO 2011
A1
For any set A = {a1 , a2 , a3 , a4 } of four distinct positive integers with sum sA = a1 + a2 + a3 + a4 ,
let pA denote the number of pairs (i, j) with 1 ≤ i < j ≤ 4 for which ai + aj divides sA . Among
all sets of four distinct positive integers, determine those sets A for which pA is maximal.
Answer. The sets A for which pA is maximal are the sets the form {d, 5d, 7d, 11d} and
{d, 11d, 19d, 29d}, where d is any positive integer. For all these sets pA is 4.
Solution. Firstly, we will prove that the maximum value of pA is at most 4. Without loss
of generality, we may assume that a1 < a2 < a3 < a4 . We observe that for each pair of
indices (i, j) with 1 ≤ i < j ≤ 4, the sum ai + aj divides sA if and only if ai + aj divides
sA − (ai + aj ) = ak + al , where k and l are the other two indices. Since there are 6 distinct
pairs, we have to prove that at least two of them do not satisfy the previous condition. We
claim that two such pairs are (a2 , a4 ) and (a3 , a4 ). Indeed, note that a2 + a4 > a1 + a3 and
a3 + a4 > a1 + a2 . Hence a2 + a4 and a3 + a4 do not divide sA . This proves pA ≤ 4.
Hence, there exist positive integers m and n with m > n ≥ 2 such that
a1 + a4 = a2 + a3
m(a1 + a2 ) = a3 + a4
n(a1 + a3 ) = a2 + a4 .
Adding up the first equation and the third one, we get n(a1 + a3 ) = 2a2 + a3 − a1 . If n ≥ 3,
then n(a1 + a3 ) > 3a3 > 2a2 + a3 > 2a2 + a3 − a1 . This is a contradiction. Therefore n = 2. If
we multiply by 2 the sum of the first equation and the third one, we obtain
while the sum of the first one and the second one is
(m + 7)a1 = (5 − m)a2 .
12
52nd IMO 2011 Algebra – solutions A1
It follows that 5 − m ≥ 1, because the left-hand side of the last equation and a2 are positive.
Since we have m > n = 2, the integer m can be equal only to either 3 or 4. Substituting
(3, 2) and (4, 2) for (m, n) and solving the previous system of equations, we find the families of
solutions {d, 5d, 7d, 11d} and {d, 11d, 19d, 29d}, where d is any positive integer.
13
A2 Algebra – solutions 52nd IMO 2011
A2
Determine all sequences (x1 , x2 , . . . , x2011 ) of positive integers such that for every positive inte-
ger n there is an integer a with
Solution. Throughout this solution, the set of positive integers will be denoted by Z+ .
1n + 2k n + · · · 2011k n = 1 + k · k n = k n+1 + 1
for all n, so (1, k, . . . , k) is a valid sequence. We shall prove that it is the only one.
Let a valid sequence (x1 , . . . , x2011 ) be given. For each n ∈ Z+ we have some yn ∈ Z+ with
Note that xn1 + 2xn2 + · · · + 2011xn2011 < (x1 + 2x2 + · · · + 2011x2011 )n+1 , which implies that
the sequence (yn ) is bounded. In particular, there is some y ∈ Z+ with yn = y for infinitely
many n.
Let m be the maximum of all the xi . Grouping terms with equal xi together, the sum xn1 +
2xn2 + · · · + 2011xn2011 can be written as
Lemma. Let integers b1 , . . . , bN be given and assume that there are arbitrarily large positive
integers n with b1 + b2 2n + · · · + bN N n = 0. Then bi = 0 for all i.
Proof. Suppose that not all bi are zero. We may assume without loss of generality that bN 6= 0.
14
52nd IMO 2011 Algebra – solutions A2
Dividing through by N n gives
n n n
N − 1 1 N −1
|bN | = bN −1
+ · · · + b1 ≤ (|bN −1 | + · · · + |b1 |) .
N N N
n
The expression NN−1 can be made arbitrarily small for n large enough, contradicting the
assumption that bN be non-zero.
15
A3 Algebra – solutions 52nd IMO 2011
A3
Determine all pairs (f, g) of functions from the set of real numbers to itself that satisfy
Answer. Either both f and g vanish identically, or there exists a real number C such that
f (x) = x2 + C and g(x) = x for all real numbers x.
Solution. Clearly all these pairs of functions satisfy the functional equation in question, so it
suffices to verify that there cannot be any further ones. Substituting −2x for y in the given
functional equation we obtain
g(f (−x)) = f (x). (1)
Now for any two real numbers a and b, setting x = −b and y = a + b we get
as well as
f (−c) = f (−a) + (c − a)g(c + a).
Now given any three real numbers x, y, and z one may determine three reals a, b, and c such
that x = b + c, y = c + a, and z = a + b, so that we get
This implies that the three points (x, g(x)), (y, g(y)), and (z, g(z)) from the graph of g are
collinear. Hence that graph is a line, i.e., g is either a constant or a linear function.
16
52nd IMO 2011 Algebra – solutions A3
Let us write g(x) = Ax + B, where A and B are two real numbers. Substituting (0, −y) for
(x, y) in (2) and denoting C = f (0), we have f (y) = Ay 2 − By + C. Now, comparing the
coefficients of x2 in (1) we see that A2 = A, so A = 0 or A = 1.
If A = 0, then (1) becomes B = −Bx + C and thus B = C = 0, which provides the first of the
two solutions mentioned above.
Now suppose A = 1. Then (1) becomes x2 − Bx + C + B = x2 − Bx + C, so B = 0. Thus,
g(x) = x and f (x) = x2 + C, which is the second solution from above.
Comment. Another way to show that g(x) is either a constant or a linear function is the following.
If we interchange x and y in the given functional equation and subtract this new equation from the
given one, we obtain
f (x) − f (y) = (2y + x)g(x) − (2x + y)g(y).
Substituting (x, 0), (1, x), and (0, 1) for (x, y), we get
17
A4 Algebra – solutions 52nd IMO 2011
A4
Determine all pairs (f, g) of functions from the set of positive integers to itself that satisfy
Answer. The only pair (f, g) of functions that satisfies the equation is given by f (n) = n and
g(n) = 1 for all n.
Let y1 < y2 < . . . be all the values attained by f (this sequence might be either finite or
infinite). We will prove that for every positive n the function f attains at least n values, and
we have (i)n : f (x) = yn if and only if x = n, and (ii)n : yn = n. The proof will follow the
scheme
(i)1 , (ii)1 , (i)2 , (ii)2 , . . . , (i)n , (ii)n , . . . (2)
To start, consider any x such that f (x) = y1 . If x > 1, then (1) reads f f g(x−1) (x − 1) < y1 ,
contradicting the minimality of y1 . So we have that f (x) = y1 is equivalent to x = 1, establish-
ing (i)1 .
Next, assume that for some n statement (i)n is established, as well as all the previous statements
in (2). Note that these statements imply that for all k ≥ 1 and a < n we have f k (x) = a if
and only if x = a.
Now, each value yi with 1 ≤ i ≤ n is attained at the unique integer i, so yn+1 exists. Choose
an arbitrary x such that f (x) = yn+1 ; we necessarily have x > n. Substituting x − 1 into (1)
we have f f g(x−1) (x − 1) < yn+1, which implies
Set b = f g(x−1) (x − 1). If b < n then we would have x − 1 = b which contradicts x > n. So
b = n, and hence yn = n, which proves (ii)n . Next, from (i)n we now get f (k) = n ⇐⇒ k = n,
so removing all the iterations of f in (3) we obtain x − 1 = b = n, which proves (i)n+1 .
So, all the statements in (2) are valid and hence f (n) = n for all n. The given relation between
f and g now reads n + g n (n) = n + 1 − g(n + 1) + 1 or g n (n) + g(n + 1) = 2, from which it
18
52nd IMO 2011 Algebra – solutions A4
immediately follows that we have g(n) = 1 for all n.
Comment. Several variations of the above solution are possible. For instance, one may first prove by
induction that the smallest n values of f are exactly f (1) < · · · < f (n) and proceed as follows. We
certainly have f (n) ≥ n for all n. If there is an n with f (n) > n, then f (x) > x for all x ≥ n. From
this we conclude f g(n)+1 (n) > f g(n) (n) > · · · > f (n). But we also have f g(n)+1 < f (n + 1). Having
squeezed in a function value between f (n) and f (n + 1), we arrive at a contradiction.
In any case, the inequality (1) plays an essential rôle.
19
A5 Algebra – solutions 52nd IMO 2011
A5
Prove that for every positive integer n, the set {2, 3, 4, . . . , 3n + 1} can be partitioned into
n triples in such a way that the numbers from each triple are the lengths of the sides of some
obtuse triangle.
Solution. Throughout the solution, we denote by [a, b] the set {a, a + 1, . . . , b}. We say that
{a, b, c} is an obtuse triple if a, b, c are the sides of some obtuse triangle.
We prove by induction on n that there exists a partition of [2, 3n + 1] into n obtuse triples Ai
(2 ≤ i ≤ n + 1) having the form Ai = {i, ai , bi }. For the base case n = 1, one can simply set
A2 = {2, 3, 4}. For the induction step, we need the following simple lemma.
Lemma. Suppose that the numbers a < b < c form an obtuse triple, and let x be any positive
number. Then the triple {a, b + x, c + x} is also obtuse.
Proof. The numbers a < b + x < c + x are the sides of a triangle because (c + x) − (b + x) =
c−b < a. This triangle is obtuse since (c+x)2 −(b+x)2 = (c−b)(c+b+2x) > (c−b)(c+b) > a2 .
Now we turn to the induction step. Let n > 1 and put t = ⌊n/2⌋ < n. By the induction
hypothesis, there exists a partition of the set [2, 3t + 1] into t obtuse triples A′i = {i, a′i , b′i }
(i ∈ [2, t + 1]). For the same values of i, define Ai = {i, a′i + (n − t), b′i + (n − t)}. The
constructed triples are obviously disjoint, and they are obtuse by the lemma. Moreover, we
have
t+1
[
Ai = [2, t + 1] ∪ [n + 2, n + 2t + 1].
i=2
Next, for each i ∈ [t + 2, n + 1], define Ai = {i, n + t + i, 2n + i}. All these sets are disjoint, and
n+1
[
Ai = [t + 2, n + 1] ∪ [n + 2t + 2, 2n + t + 1] ∪ [2n + t + 2, 3n + 1],
i=t+2
so
n+1
[
Ai = [2, 3n + 1].
i=2
Thus, we are left to prove that the triple Ai is obtuse for each i ∈ [t + 2, n + 1].
Since (2n + i) − (n + t + i) = n − t < t + 2 ≤ i, the elements of Ai are the sides of a triangle.
Next, we have
n n 9n
(2n + i)2 − (n + t + i)2 = (n − t)(3n + t + 2i) ≥ · (3n + 3(t + 1) + 1) > · ≥ (n + 1)2 ≥ i2 ,
2 2 2
so this triangle is obtuse. The proof is completed.
20
52nd IMO 2011 Algebra – solutions A6
A6
Let f be a function from the set of real numbers to itself that satisfies
for all real numbers x and y. Prove that f (x) = 0 for all x ≤ 0.
Consider now some real numbers a, b and use (2) with t = f (a), x = b as well as with t = f (b),
x = a. We get
Now, substitute b = 2f (a) to obtain 2f (a)f (b) ≥ af (a) + 2f (a)f (b), or af (a) ≤ 0. So, we get
Now suppose f (x) > 0 for some real number x. From (2) we immediately get that for every
xf (x) − f (f (x))
t< we have f (t) < 0. This contradicts (3); therefore
f (x)
0 ≤ 0 − 0 + f (0),
Solution 2. We will also use the condition of the problem in form (2). For clarity we divide
the argument into four steps.
21
A6 Algebra – solutions 52nd IMO 2011
Step 1. We begin by proving that f attains nonpositive values only. Assume that there
exist some real number z with f (z) > 0. Substituting x = z into (2) and setting A = f (z),
B = −zf (z) − f (f (z)) we get f (t) ≤ At + B for all real t. Hence, if for any positive real
number t we substitute x = −t, y = t into (1), we get
But surely this cannot be true if we take t to be large enough. This contradiction proves that
we have indeed f (x) ≤ 0 for all real numbers x. Note that for this reason (1) entails
f (x + y) ≤ yf (x) (5)
Step 2. We proceed by proving that f has at least one zero. If f (0) = 0, we are done.
Otherwise, in view of Step 1 we get f (0) < 0. Observe that (5) tells us now f (y) ≤ yf (0) for all
real numbers y. Thus we can specify a positive real number a that is so large that f (a)2 > −f (0).
Put b = f (a) and substitute x = b and y = −b into (5); we learn −b2 < f (0) ≤ −bf (b), i.e.
b < f (b). Now we apply (2) to x = b and t = f (b), which yields
f (f (b)) ≤ f (b) − b f (b) + f (f (b)),
Step 3. Next we show that if f (a) = 0 and b < a, then f (b) = 0 as well. To see this, we just
substitute x = b and y = a − b into (5), thus getting f (b) ≥ 0, which suffices by Step 1.
Step 4. By Step 3, the solution of the problem is reduced to showing f (0) = 0. Pick any
zero r of f and substitute x = r and y = −1 into (1). Because of f (r) = f (r − 1) = 0 this gives
f (0) ≥ 0 and hence f (0) = 0 by Step 1 again.
Comment 1. Both of these solutions also show f (x) ≤ 0 for all real numbers x. As one can see
from Solution 1, this task gets much easier if one already knows that f takes nonnegative values for
sufficiently small arguments. Another way of arriving at this statement, suggested by the proposer, is
as follows:
Put a = f (0) and substitute x = 0 into (1). This gives f (y) ≤ ay + f (a) for all real numbers y. Thus
if for any real number x we plug y = a − x into (1), we obtain
and hence 0 ≤ (2a − x)f (x). In particular, if x < 2a, then f (x) ≥ 0.
Having reached this point, one may proceed almost exactly as in the first solution to deduce f (x) ≤ 0
for all x. Afterwards the problem can be solved in a few lines as shown in steps 3 and 4 of the second
22
52nd IMO 2011 Algebra – solutions A6
solution.
Comment 2. The original problem also contained the question whether a nonzero function satisfying
the problem condition exists. Here we present a family of such functions.
Notice first that if g : (0, ∞) −→ [0, ∞) denotes any function such that
for all positive real numbers x and y, then the function f given by
−g(x) if x > 0
f (x) = (7)
0 if x ≤ 0
automatically satisfies (1). Indeed, we have f (x) ≤ 0 and hence also f (f (x)) = 0 for all real numbers x.
So (1) reduces to (5); moreover, this inequality is nontrivial only if x and y are positive. In this last
case it is provided by (6).
Now it is not hard to come up with a nonzero function g obeying (6). E.g. g(z) = Cez (where C is
a positive constant) fits since the inequality ey > y holds for all (positive) real numbers y. One may
also consider the function g(z) = ez − 1; in this case, we even have that f is continuous.
23
A7 Algebra – solutions 52nd IMO 2011
A7
√
Let a, b, and c be positive real numbers satisfying min(a+b, b+c, c+a) > 2 and a2 +b2 +c2 = 3.
Prove that
a b c 3
2
+ 2
+ 2
≥ . (1)
(b + c − a) (c + a − b) (a + b − c) (abc)2
Throughout both solutions, we denote the sums of the form f (a, b, c) + f (b, c, a) + f (c, a, b)
P
by f (a, b, c).
√
Solution 1. The condition b + c > 2 implies b2 + c2 > 1, so a2 = 3 − (b2 + c2 ) < 2, i.e.
√
a < 2 < b + c. Hence we have b + c − a > 0, and also c + a − b > 0 and a + b − c > 0 for
similar reasons.
We will use the variant of Hölder’s inequality
xp+1
1 xp+1
1 xp+1
n (x1 + x2 + . . . + xn )p+1
+ + . . . + ≥ ,
y1p y1p ynp (y1 + y2 + . . . + yn )p
which holds for all positive real numbers p, x1 , x2 , . . . , xn , y1, y2 , . . . , yn . Applying it to the
left-hand side of (1) with p = 2 and n = 3, we get
X a X (a2 )3 (a2 + b2 + c2 )3 27
= ≥ P 2 = P 2 . (2)
(b + c − a)2 a5 (b + c − a)2 a5/2 (b + c − a) a5/2 (b + c − a)
To estimate the denominator of the right-hand part, we use an instance of Schur’s inequality,
namely
X
a3/2 (a − b)(a − c) ≥ 0,
Moreover, by the inequality between the arithmetic mean and the fourth power mean we also
have √
√ √ !4
a+ b+ c a2 + b2 + c2
≤ = 1,
3 3
√ √ √
i.e., a + b + c ≤ 3. Hence, (2) yields
X a 27 3
2
≥ √ √ √ 2 ≥ 2 2 2 ,
(b + c − a) abc( a + b + c) abc
24
52nd IMO 2011 Algebra – solutions A7
Comment. In this solution, one may also start from the following version of Hölder’s inequality
n
! n
! n
! n
!3
X X X X
a3i b3i c3i ≥ ai bi ci
i=1 i=1 i=1 i=1
applied as
X a X X
2
· a3 (b + c − a) · a2 (b + c − a) ≥ 27.
(b + c − a)
After doing that, one only needs the slightly better known instances
X X
a3 (b + c − a) ≤ (a + b + c)abc and a2 (b + c − a) ≤ 3abc
of Schur’s Inequality.
a5 + b5 + c5 ≥ 3, (3)
which is weaker than the given one. Due to the symmetry we may assume that a ≥ b ≥ c.
X a3 b2 c2 X
≥ a5 ,
(b + c − a)2
Note that the signs of the expressions (yz)2 −(x(y + z − x))2 and yz−x(y+z−x) = (x−y)(x−z)
are the same for every positive x, y, z satisfying the triangle inequality. So the terms in (4)
corresponding to a and c are nonnegative, and hence it is sufficient to prove that the sum of
the terms corresponding to a and b is nonnegative. Equivalently, we need the relation
a3 b3
(a − b)(a − c)(bc + a(b + c − a)) ≥ (a − b)(b − c)(ac + b(a + c − b)).
(b + c − a)2 (a + c − b)2
Obviously, we have
a3 ≥ b3 ≥ 0, 0 < b + c − a ≤ a + c − b, and a − c ≥ b − c ≥ 0,
ab + ac + bc − a2 ab + ac + bc − b2
≥ .
b+c−a c+a−b
25
A7 Algebra – solutions 52nd IMO 2011
or
(a − b)(2ab − a2 − b2 + ac + bc) ≥ 0.
26
52nd IMO 2011 Combinatorics – solutions C1
C1
Let n > 0 be an integer. We are given a balance and n weights of weight 20 , 21 , . . . , 2n−1 . In a
sequence of n moves we place all weights on the balance. In the first move we choose a weight
and put it on the left pan. In each of the following moves we choose one of the remaining
weights and we add it either to the left or to the right pan. Compute the number of ways in
which we can perform these n moves in such a way that the right pan is never heavier than the
left pan.
Answer. The number f (n) of ways of placing the n weights is equal to the product of all odd
positive integers less than or equal to 2n − 1, i.e. f (n) = (2n − 1)!! = 1 · 3 · 5 · . . . · (2n − 1).
Firstly, note that after the first move the left pan is always at least 1 heavier than the right
one. Hence, any valid way of placing the n weights on the scale gives rise, by not considering
weight 1, to a valid way of placing the weights 2, 22 , . . . , 2n−1 .
If we divide the weight of each weight by 2, the answer does not change. So these n − 1 weights
can be placed on the scale in f (n − 1) valid ways. Now we look at weight 1. If it is put on
the scale in the first move, then it has to be placed on the left side, otherwise it can be placed
either on the left or on the right side, because after the first move the difference between the
weights on the left pan and the weights on the right pan is at least 2. Hence, there are exactly
2n − 1 different ways of inserting weight 1 in each of the f (n − 1) valid sequences for the n − 1
weights in order to get a valid sequence for the n weights. This proves the claim.
Since f (1) = 1, by induction we obtain for all positive integers n
Comment 1. The word “compute” in the statement of the problem is probably too vague. An
alternative but more artificial question might ask for the smallest n for which the number of valid
ways is divisible by 2011. In this case the answer would be 1006.
Comment 2. It is useful to remark that the answer is the same for any set of weights where each weight
is heavier than the sum of the lighter ones. Indeed, in such cases the given condition is equivalent to
asking that during the process the heaviest weight on the balance is always on the left pan.
Comment 3. Instead of considering the lightest weight, one may also consider the last weight put on
the balance. If this weight is 2n−1 then it should be put on the left pan. Otherwise it may be put on
27
C1 Combinatorics – solutions 52nd IMO 2011
any pan; the inequality would not be violated since at this moment the heaviest weight is already put
onto the left pan. In view of the previous comment, in each of these 2n − 1 cases the number of ways
to place the previous weights is exactly f (n − 1), which yields (1).
Solution 2. We present a different way of obtaining (1). Set f (0) = 1. Firstly, we find a
recurrent formula for f (n).
Assume n ≥ 1. Suppose that weight 2n−1 is placed on the balance in the i-th move with
1 ≤ i ≤ n. This weight has to be put on the left pan. For the previous moves we have n−1 i−1
choices of the weights and from Comment 2 there are f (i − 1) valid ways of placing them on
the balance. For later moves there is no restriction on the way in which the weights are to be
put on the pans. Therefore, all (n − i)!2n−i ways are possible. This gives
n n
X n−1 n−i
X (n − 1)!f (i − 1)2n−i
f (n) = f (i − 1)(n − i)!2 = . (2)
i=1
i−1 i=1
(i − 1)!
n−1
X (n − 2)!f (i − 1)2n−1−i
f (n − 1) = .
i=1
(i − 1)!
n−1
X (n − 2)!f (i − 1)2n−1−i
f (n) = 2(n − 1) + f (n − 1)
i=1
(i − 1)!
= (2n − 2)f (n − 1) + f (n − 1) = (2n − 1)f (n − 1),
QED.
Comment. There exist different ways of obtaining the formula (2). Here we show one of them.
Suppose that in the first move we use weight 2n−i+1 . Then the lighter n − i weights may be put
on the balance at any moment and on either pan. This gives 2n−i · (n − 1)!/(i − 1)! choices for the
moves (moments and choices of pan) with the lighter weights. The remaining i − 1 moves give a valid
sequence for the i − 1 heavier weights and this is the only requirement for these moves, so there are
f (i − 1) such sequences. Summing over all i = 1, 2, . . . , n we again come to (2).
28
52nd IMO 2011 Combinatorics – solutions C2
C2
Suppose that 1000 students are standing in a circle. Prove that there exists an integer k with
100 ≤ k ≤ 300 such that in this circle there exists a contiguous group of 2k students, for which
the first half contains the same number of girls as the second half.
Solution. Number the students consecutively from 1 to 1000. Let ai = 1 if the ith student
is a girl, and ai = 0 otherwise. We expand this notion for all integers i by setting ai+1000 =
ai−1000 = ai . Next, let
Sk (i) = ai + ai+1 + · · · + ai+k−1 .
There exist an integer k with 100 ≤ k ≤ 300 and an index i such that Sk (i) = Sk (i + k).
Assume now that this statement is false. Choose an index i such that S100 (i) attains the maximal
possible value. In particular, we have S100 (i−100)−S100 (i) < 0 and S100 (i) − S100 (i + 100) > 0,
for if we had an equality, then the statement would hold. This means that the function S(j) −
S(j + 100) changes sign somewhere on the segment [i − 100, i], so there exists some index j ∈
[i − 100, i − 1] such that
Subtracting the first inequality from the second one, we get aj+100 − aj ≥ aj+200 − aj+100 + 2, so
aj = 0, aj+100 = 1, aj+200 = 0.
Substituting this into the inequalities of (1), we also obtain S99 (j+1) ≤ S99 (j+101) ≤ S99 (j+1),
which implies
S99 (j + 1) = S99 (j + 101). (2)
Now let k and ℓ be the least positive integers such that aj−k = 1 and aj+200+ℓ = 1. By
symmetry, we may assume that k ≥ ℓ. If k ≥ 200 then we have aj = aj−1 = · · · = aj−199 = 0,
so S100 (j−199) = S100 (j−99) = 0, which contradicts the initial assumption. Hence ℓ ≤ k ≤ 199.
Finally, we have
Comparing with (2) we get S100+ℓ (j − ℓ + 1) = S100+ℓ (j + 101) and 100 + ℓ ≤ 299, which again
contradicts our assumption.
Comment. It may be seen from the solution that the number 300 from the problem statement can be
29
C2 Combinatorics – solutions 52nd IMO 2011
replaced by 299. Here we consider some improvements of this result. Namely, we investigate which
interval can be put instead of [100, 300] in order to keep the problem statement valid.
1, 1, . . . , 1, 0, 0, . . . , 0, 1, 1, . . . , 1, 0, 0, . . . , 0, 1, 1, . . . , 1, 0, 0, . . . , 0
| {z } | {z } | {z } | {z } | {z } | {z }
167 167 167 167 167 165
and
1, 1, . . . , 1, 0, 0, . . . , 0, 1, 1, . . . , 1, 0, 0, . . . , 0
| {z } | {z } | {z } | {z }
249 251 249 251
show that the interval can be changed neither to [84, 248] nor to [126, 374].
On the other hand, we claim that this interval can be changed to [125, 250]. Note that this statement
is invariant under replacing all 1’s by 0’s and vice versa. Assume, to the contrary, that there is no
admissible k ∈ [125, 250]. The arguments from the solution easily yield the following lemma.
Lemma. Under our assumption, suppose that for some indices i < j we have S125 (i) ≤ S125 (i + 125)
but S125 (j) ≥ S125 (j +125). Then there exists some t ∈ [i, j −1] such that at = at−1 = · · · = at−125 = 0
and at+250 = at+251 = · · · = at+375 = 0.
Let us call a segment [i, j] of indices a crowd, if (a) ai = ai+1 = · · · = aj , but ai−1 6= ai 6= aj+1 , and (b)
j − i ≥ 125. Now, using the lemma, one can get in the same way as in the solution that there exists
some crowd. Take all the crowds in the circle, and enumerate them in cyclic order as A1 , . . . , Ad . We
also assume always that As+d = As−d = As .
Consider one of the crowds, say A1 . We have A1 = [i, i + t] with 125 ≤ t ≤ 248 (if t ≥ 249, then
ai = ai+1 = · · · = ai+249 and therefore S125 (i) = S125 (i + 125), which contradicts our assumption).
We may assume that ai = 1. Then we have S125 (i + t − 249) ≤ 125 = S125 (i + t − 124) and
S125 (i) = 125 ≥ S125 (i + 125), so by the lemma there exists some index j ∈ [i + t − 249, i − 1] such
that the segments [j − 125, j] and [j + 250, j + 375] are contained in some crowds.
Let us fix such j and denote the segment [j + 1, j + 249] by B1 . Clearly, A1 ⊆ B1 . Moreover, B1
cannot contain any crowd other than A1 since |B1 | = 249 < 2 · 126. Hence it is clear that j ∈ Ad and
j + 250 ∈ A2 . In particular, this means that the genders of Ad and A2 are different from that of A1 .
Performing this procedure for every crowd As , we find segments Bs = [js + 1, js + 249] such that
|Bs | = 249, As ⊆ Bs , and js ∈ As−1 , js + 250 ∈ As+1 . So, Bs covers the whole segment between As−1
and As+1 , hence the sets B1 , . . . , Bd cover some 1000 consecutive indices. This implies 249d ≥ 1000,
and d ≥ 5. Moreover, the gender of Ai is alternating, so d is even; therefore d ≥ 6.
Consider now three segments A1 = [i1 , i′1 ], B2 = [j2 + 1, j2 + 249], A3 = [i3 , i′3 ]. By construction, we
have [j2 − 125, j2 ] ⊆ A1 and [j2 + 250, j2 + 375] ⊆ A3 , whence i1 ≤ j2 − 125, i′3 ≥ j2 + 375. Therefore
i′3 − i1 ≥ 500. Analogously, if A4 = [i4 , i′4 ], A6 = [i6 , i′6 ] then i′6 − i4 ≥ 500. But from d ≥ 6 we get
i1 < i′3 < i4 < i′6 < i1 + 1000, so 1000 > (i′3 − i1 ) + (i′6 − i4 ) ≥ 500 + 500. This final contradiction
shows that our claim holds.
One may even show that the interval in the statement of the problem may be replaced by [125, 249]
(both these numbers cannot be improved due to the examples above). But a proof of this fact is a bit
messy, and we do not present it here.
30
52nd IMO 2011 Combinatorics – solutions C3
C3
Let S be a finite set of at least two points in the plane. Assume that no three points of S are
collinear. By a windmill we mean a process as follows. Start with a line ℓ going through a
point P ∈ S. Rotate ℓ clockwise around the pivot P until the line contains another point Q
of S. The point Q now takes over as the new pivot. This process continues indefinitely, with
the pivot always being a point from S.
Show that for a suitable P ∈ S and a suitable starting line ℓ containing P , the resulting
windmill will visit each point of S as a pivot infinitely often.
Solution. Give the rotating line an orientation and distinguish its sides as the oranje side and
the blue side. Notice that whenever the pivot changes from some point T to another point U,
after the change, T is on the same side as U was before. Therefore, the number of elements
of S on the oranje side and the number of those on the blue side remain the same throughout
the whole process (except for those moments when the line contains two points).
U U U
T T T
First consider the case that |S| = 2n + 1 is odd. We claim that through any point T ∈ S,
there is a line that has n points on each side. To see this, choose an oriented line through T
containing no other point of S and suppose that it has n + r points on its oranje side. If
r = 0 then we have established the claim, so we may assume that r 6= 0. As the line rotates
through 180◦ around T , the number of points of S on its oranje side changes by 1 whenever
the line passes through a point; after 180◦, the number of points on the oranje side is n − r.
Therefore there is an intermediate stage at which the oranje side, and thus also the blue side,
contains n points.
Now select the point P arbitrarily, and choose a line through P that has n points of S on each
side to be the initial state of the windmill. We will show that during a rotation over 180◦ ,
the line of the windmill visits each point of S as a pivot. To see this, select any point T of S
and select a line ℓ through T that separates S into equal halves. The point T is the unique
point of S through which a line in this direction can separate the points of S into equal halves
(parallel translation would disturb the balance). Therefore, when the windmill line is parallel
to ℓ, it must be ℓ itself, and so pass through T .
Next suppose that |S| = 2n. Similarly to the odd case, for every T ∈ S there is an oriented
31
C3 Combinatorics – solutions 52nd IMO 2011
line through T with n − 1 points on its oranje side and n points on its blue side. Select such
an oriented line through an arbitrary P to be the initial state of the windmill.
We will now show that during a rotation over 360◦ , the line of the windmill visits each point
of S as a pivot. To see this, select any point T of S and an oriented line ℓ through T that
separates S into two subsets with n − 1 points on its oranje and n points on its blue side.
Again, parallel translation would change the numbers of points on the two sides, so when the
windmill line is parallel to ℓ with the same orientation, the windmill line must pass through T .
32
52nd IMO 2011 Combinatorics – solutions C4
C4
Determine the greatest positive integer k that satisfies the following property: The set of positive
integers can be partitioned into k subsets A1 , A2 , . . . , Ak such that for all integers n ≥ 15 and
all i ∈ {1, 2, . . . , k} there exist two distinct elements of Ai whose sum is n.
Solution 1. There are various examples showing that k = 3 does indeed have the property
under consideration. E.g. one can take
To check that this partition fits, we notice first that the sums of two distinct elements of Ai
obviously represent all numbers n ≥ 1 + 12 = 13 for i = 1, all numbers n ≥ 4 + 11 = 15 for
i = 2, and all numbers n ≥ 7 + 10 = 17 for i = 3. So, we are left to find representations of the
numbers 15 and 16 as sums of two distinct elements of A3 . These are 15 = 7 + 8 and 16 = 7 + 9.
Let us now suppose that for some k ≥ 4 there exist sets A1 , A2 , . . . , Ak satisfying the given
property. Obviously, the sets A1 , A2 , A3 , A4 ∪ · · · ∪ Ak also satisfy the same property, so one
may assume k = 4.
Put Bi = Ai ∩ {1, 2, . . . , 23} for i = 1, 2, 3, 4. Now for any index i each of the ten numbers
15, 16, . . . , 24 can be written as sum of two distinct elements of Bi . Therefore this set needs
to contain at least five elements. As we also have |B1 | + |B2 | + |B3 | + |B4 | = 23, there has to
be some index j for which |Bj | = 5. Let Bj = {x1 , x2 , x3 , x4 , x5 }. Finally, now the sums of
two distinct elements of Aj representing the numbers 15, 16, . . . , 24 should be exactly all the
pairwise sums of the elements of Bj . Calculating the sum of these numbers in two different
ways, we reach
4(x1 + x2 + x3 + x4 + x5 ) = 15 + 16 + . . . + 24 = 195.
Thus the number 195 should be divisible by 4, which is false. This contradiction completes our
solution.
Comment. There are several variation of the proof that k should not exceed 3. E.g., one may consider
the sets Ci = Ai ∩ {1, 2, . . . , 19} for i = 1, 2, 3, 4. As in the previous solution one can show that for
some index j one has |Cj | = 4, and the six pairwise sums of the elements of Cj should represent all
numbers 15, 16, . . . , 20. Let Cj = {y1 , y2 , y3 , y4 } with y1 < y2 < y3 < y4 . It is not hard to deduce
33
C4 Combinatorics – solutions 52nd IMO 2011
By now we know that G is a graph with 4k edges, at least k components and at most one
circuit. Consequently, G must have at least 4k + k − 1 vertices. Thus 5k − 1 ≤ 18, and k ≤ 3.
34
52nd IMO 2011 Combinatorics – solutions C5
C5
Let m be a positive integer and consider a checkerboard consisting of m by m unit squares.
At the midpoints of some of these unit squares there is an ant. At time 0, each ant starts
moving with speed 1 parallel to some edge of the checkerboard. When two ants moving in
opposite directions meet, they both turn 90◦ clockwise and continue moving with speed 1.
When more than two ants meet, or when two ants moving in perpendicular directions meet,
the ants continue moving in the same direction as before they met. When an ant reaches one
of the edges of the checkerboard, it falls off and will not re-appear.
Considering all possible starting positions, determine the latest possible moment at which the
last ant falls off the checkerboard or prove that such a moment does not necessarily exist.
3m
Antswer. The latest possible moment for the last ant to fall off is 2
− 1.
Solution. For m = 1 the answer is clearly correct, so assume m > 1. In the sequel, the word
collision will be used to denote meeting of exactly two ants, moving in opposite directions.
If at the beginning we place an ant on the southwest corner square facing east and an ant on
the southeast corner square facing west, then they will meet in the middle of the bottom row
at time m−12
. After the collision, the ant that moves to the north will stay on the board for
another m − 12 time units and thus we have established an example in which the last ant falls
off at time m−1
2
+ m − 21 = 3m 2
− 1. So, we are left to prove that this is the latest possible
moment.
Consider any collision of two ants a and a′ . Let us change the rule for this collision, and enforce
these two ants to turn anticlockwise. Then the succeeding behavior of all the ants does not
change; the only difference is that a and a′ swap their positions. These arguments may be
applied to any collision separately, so we may assume that at any collision, either both ants
rotate clockwise or both of them rotate anticlockwise by our own choice.
For instance, we may assume that there are only two types of ants, depending on their initial
direction: NE-ants, which move only north or east, and SW-ants, moving only south and west.
Then we immediately obtain that all ants will have fallen off the board after 2m − 1 time
units. However, we can get a better bound by considering the last moment at which a given
ant collides with another ant.
Choose a coordinate system such that the corners of the checkerboard are (0, 0), (m, 0), (m, m)
and (0, m). At time t, there will be no NE-ants in the region {(x, y) : x + y < t + 1} and no
SW-ants in the region {(x, y) : x + y > 2m − t − 1}. So if two ants collide at (x, y) at time t,
we have
t + 1 ≤ x + y ≤ 2m − t − 1. (1)
35
C5 Combinatorics – solutions 52nd IMO 2011
Analogously, we may change the rules so that each ant would move either alternatingly north
and west, or alternatingly south and east. By doing so, we find that apart from (1) we also
have |x − y| ≤ m − t − 1 for each collision at point (x, y) and time t.
An ant can thus only collide with another ant at time t if it happens to be in the region B(t).
The following figure displays B(t) for t = 12 and t = 72 in the case m = 6:
Now suppose that an NE-ant has its last collision at time t and that it does so at the point (x, y)
(if the ant does not collide at all, it will fall off the board within m− 12 < 3m
2
−1 time units, so this
case can be ignored). Then we have (x, y) ∈ B(t) and thus x+y ≥ t+1 and x−y ≥ −(m−t−1).
So we get
(t + 1) − (m − t − 1) m
x≥ =t+1− .
2 2
By symmetry we also have y ≥ t + 1 − 2 , and hence min{x, y} ≥ t + 1 − m2 . After this collision,
m
the ant will move directly to an edge, which will take at most m − min{x, y} units of time. In
sum, the total amount of time the ant stays on the board is at most
m 3m
t + (m − min{x, y}) ≤ t + m − t + 1 − = − 1.
2 2
By symmetry, the same bound holds for SW-ants as well.
36
52nd IMO 2011 Combinatorics – solutions C6
C6
Let n be a positive integer and let W = . . . x−1 x0 x1 x2 . . . be an infinite periodic word consisting
of the letters a and b. Suppose that the minimal period N of W is greater than 2n .
A finite nonempty word U is said to appear in W if there exist indices k ≤ ℓ such that
U = xk xk+1 . . . xℓ . A finite word U is called ubiquitous if the four words Ua, Ub, aU, and bU
all appear in W . Prove that there are at least n ubiquitous finite nonempty words.
Solution. Throughout the solution, all the words are nonempty. For any word R of length m,
we call the number of indices i ∈ {1, 2, . . . , N} for which R coincides with the subword
xi+1 xi+2 . . . xi+m of W the multiplicity of R and denote it by µ(R). Thus a word R appears
in W if and only if µ(R) > 0. Since each occurrence of a word in W is both succeeded by either
the letter a or the letter b and similarly preceded by one of those two letters, we have
So we have µ(U0 ) ≤ 2 < µ(U1 ) ≤ 4 < . . . ≤ 2n−1 < µ(Un−1 ), which implies in particular that
the words U0 , U1 , . . . , Un−1 have to be distinct. As they have been proved to be ubiquitous as
well, the problem is solved.
Comment 1. There is an easy construction for obtaining ubiquitous words from appearing words
whose multiplicity is at least two. Starting with any such word U we may simply extend one of its
occurrences in W forwards and backwards as long as its multiplicity remains fixed, thus arriving at a
37
C6 Combinatorics – solutions 52nd IMO 2011
Comment 2. The bound n for the number of ubiquitous subwords in the problem statement is not
optimal, but it is close to an optimal one in the following sense. There is a universal constant C > 0
such that for each positive integer n there exists an infinite periodic word W whose minimal period is
greater than 2n but for which there exist fewer than Cn ubiquitous words.
38
52nd IMO 2011 Combinatorics – solutions C7
C7
On a square table of 2011 by 2011 cells we place a finite number of napkins that each cover
a square of 52 by 52 cells. In each cell we write the number of napkins covering it, and we
record the maximal number k of cells that all contain the same nonzero number. Considering
all possible napkin configurations, what is the largest value of k?
Answer. 20112 − (522 − 352 ) · 39 − 172 = 4044121 − 57392 = 3986729.
Solution 1. Let m = 39, then 2011 = 52m − 17. We begin with an example showing that
there can exist 3986729 cells carrying the same positive number.
To describe it, we number the columns from the left to the right and the rows from the bottom
to the top by 1, 2, . . . , 2011. We will denote each napkin by the coordinates of its lower-
left cell. There are four kinds of napkins: first, we take all napkins (52i + 36, 52j + 1) with
0 ≤ j ≤ i ≤ m − 2; second, we use all napkins (52i + 1, 52j + 36) with 0 ≤ i ≤ j ≤ m − 2;
third, we use all napkins (52i + 36, 52i + 36) with 0 ≤ i ≤ m − 2; and finally the napkin (1, 1).
Different groups of napkins are shown by different types of hatchings in the picture.
Now except for those squares that carry two or more different hatchings, all squares have the
number 1 written into them. The number of these exceptional cells is easily computed to be
(522 − 352 )m − 172 = 57392.
We are left to prove that 3986729 is an upper bound for the number of cells containing the same
number. Consider any configuration of napkins and any positive integer M. Suppose there are
g cells with a number different from M. Then it suffices to show g ≥ 57392. Throughout the
solution, a line will mean either a row or a column.
Consider any line ℓ. Let a1 , . . . , a52m−17 be the numbers written into its consecutive cells.
P
For i = 1, 2, . . . , 52, let si = t≡i (mod 52) at . Note that s1 , . . . , s35 have m terms each, while
s36 , . . . , s52 have m − 1 terms each. Every napkin intersecting ℓ contributes exactly 1 to each si ;
39
C7 Combinatorics – solutions 52nd IMO 2011
hence the number s of all those napkins satisfies s1 = · · · = s52 = s. Call the line ℓ rich if
s > (m − 1)M and poor otherwise.
Suppose now that ℓ is rich. Then in each of the sums s36 , . . . , s52 there exists a term greater
than M; consider all these terms and call the corresponding cells the rich bad cells for this line.
So, each rich line contains at least 17 cells that are bad for this line.
If, on the other hand, ℓ is poor, then certainly s < mM so in each of the sums s1 , . . . , s35 there
exists a term less than M; consider all these terms and call the corresponding cells the poor
bad cells for this line. So, each poor line contains at least 35 cells that are bad for this line.
Let us call all indices congruent to 1, 2, . . . , or 35 modulo 52 small, and all other indices,
i.e. those congruent to 36, 37, . . . , or 52 modulo 52, big. Recall that we have numbered the
columns from the left to the right and the rows from the bottom to the top using the numbers
1, 2, . . . , 52m − 17; we say that a line is big or small depending on whether its index is big or
small. By definition, all rich bad cells for the rows belong to the big columns, while the poor
ones belong to the small columns, and vice versa.
In each line, we put a strawberry on each cell that is bad for this line. In addition, for each
small rich line we put an extra strawberry on each of its (rich) bad cells. A cell gets the
strawberries from its row and its column independently.
Notice now that a cell with a strawberry on it contains a number different from M. If this cell
gets a strawberry by the extra rule, then it contains a number greater than M. Moreover, it
is either in a small row and in a big column, or vice versa. Suppose that it is in a small row,
then it is not bad for its column. So it has not more than two strawberries in this case. On
the other hand, if the extra rule is not applied to some cell, then it also has not more than two
strawberries. So, the total number N of strawberries is at most 2g.
We shall now estimate N in a different way. For each of the 2 · 35m small lines, we have
introduced at least 34 strawberries if it is rich and at least 35 strawberries if it is poor, so at
least 34 strawberries in any case. Similarly, for each of the 2 · 17(m − 1) big lines, we put at
least min(17, 35) = 17 strawberries. Summing over all lines we obtain
as desired.
Comment. The same reasoning applies also if we replace 52 by R and 2011 by Rm − H, where m, R,
and H are integers with m, R ≥ 1 and 0 ≤ H ≤ 31 R. More detailed information is provided after the
next solution.
Solution 2. We present a different proof of the estimate which is the hard part of the problem.
Let S = 35, H = 17, m = 39; so the table size is 2011 = Sm + H(m − 1), and the napkin size is
52 = S + H. Fix any positive integer M and call a cell vicious if it contains a number distinct
40
52nd IMO 2011 Combinatorics – solutions C7
from M. We will prove that there are at least H 2 (m − 1) + 2SHm vicious cells.
Firstly, we introduce some terminology. As in the previous solution, we number rows and
columns and we use the same notions of small and big indices and lines; so, an index is small if
it is congruent to one of the numbers 1, 2, . . . , S modulo (S + H). The numbers 1, 2, . . . , S + H
will be known as residues. For two residues i and j, we say that a cell is of type (i, j) if the
index of its row is congruent to i and the index of its column to j modulo (S + H). The number
of vicious cells of this type is denoted by vij .
Let s, s′ be two variables ranging over small residues and let h, h′ be two variables ranging over
big residues. A cell is said to be of class A, B, C, or D if its type is of shape (s, s′ ), (s, h), (h, s),
or (h, h′ ), respectively. The numbers of vicious cells belonging to these classes are denoted in
this order by a, b, c, and d. Observe that each cell belongs to exactly one class.
Claim 1. We have
a b+c
m≤ 2
+ . (1)
S 2SH
Proof. Consider an arbitrary small row r. Denote the numbers of vicious cells on r belonging
to the classes A and B by α and β, respectively. As in the previous solution, we obtain that
α ≥ S or β ≥ H. So in each case we have Sα + Hβ ≥ 1.
Performing this argument separately for each small row and adding up all the obtained inequal-
ities, we get Sa + Hb ≥ mS. Interchanging rows and columns we similarly get Sa + Hc ≥ mS.
Summing these inequalities and dividing by 2S we get what we have claimed.
Claim 2. Fix two small residue s, s′ and two big residues h, h′ . Then 2m−1 ≤ vss′ +vsh′ +vhh′ .
Proof. Each napkin covers exactly one cell of type (s, s′ ). Removing all napkins covering a
vicious cell of this type, we get another collection of napkins, which covers each cell of type
(s, s′ ) either 0 or M times depending on whether the cell is vicious or not. Hence (m2 − vss′ )M
napkins are left and throughout the proof of Claim 2 we will consider only these remaining
napkins. Now, using a red pen, write in each cell the number of napkins covering it. Notice
that a cell containing a red number greater than M is surely vicious.
We call two cells neighbors if they can be simultaneously covered by some napkin. So, each cell
of type (h, h′ ) has not more than four neighbors of type (s, s′), while each cell of type (s, h′ ) has
not more than two neighbors of each of the types (s, s′) and (h, h′ ). Therefore, each red number
at a cell of type (h, h′ ) does not exceed 4M, while each red number at a cell of type (s, h′ ) does
not exceed 2M.
Let x, y, and z be the numbers of cells of type (h, h′ ) whose red number belongs to (M, 2M],
(2M, 3M], and (3M, 4M], respectively. All these cells are vicious, hence x + y + z ≤ vhh′ . The
red numbers appearing in cells of type (h, h′ ) clearly sum up to (m2 − vss′ )M. Bounding each
of these numbers by a multiple of M we get
(m2 − vss′ )M ≤ (m − 1)2 − (x + y + z) M + 2xM + 3yM + 4zM,
41
C7 Combinatorics – solutions 52nd IMO 2011
i.e.
2m − 1 ≤ vss′ + x + 2y + 3z ≤ vss′ + vhh′ + y + 2z.
Claim 3. We have
a b+c d
2m − 1 ≤ + + . (2)
S 2 2SH H 2
Proof. Averaging the previous result over all S 2 H 2 possibilities for the quadruple (s, s′ , h, h′ ),
we get 2m − 1 ≤ Sa2 + SHb
+ Hd2 . Due to the symmetry between rows and columns, the same
estimate holds with b replaced by c. Averaging these two inequalities we arrive at our claim.
Now let us multiply (2) by H 2 , multiply (1) by (2SH − H 2 ) and add them; we get
2 2H 2 + 2SH − H 2 H 2 + 2SH − H 2 2H
H (2m−1)+(2SH −H )m ≤ a· +(b+c) +d = a· +b+c+d.
S2 2SH S
The left-hand side is exactly H 2 (m − 1) + 2SHm, while the right-hand side does not exceed
a + b + c + d since 2H ≤ S. Hence we come to the desired inequality.
Comment 1. Claim 2 is the key difference between the two solutions, because it allows to get rid of
the notions of rich and poor cells. However, one may prove it by the “strawberry method” as well.
It suffices to put a strawberry on each cell which is bad for an s-row, and a strawberry on each cell
which is bad for an h′ -column. Then each cell would contain not more than one strawberry.
Comment 2. Both solutions above work if the residue of the table size T modulo the napkin size R
is at least 32 R, or equivalently if T = Sm + H(m − 1) and R = S + H for some positive integers S, H,
m such that S ≥ 2H. Here we discuss all other possible combinations.
Case 1. If 2H ≥ S ≥ H/2, then the sharp bound for the number of vicious cells is mS 2 + (m − 1)H 2 ;
it can be obtained by the same methods as in any of the solutions. To obtain an example showing
that the bound is sharp, one may simply remove the napkins of the third kind from the example in
Solution 1 (with an obvious change in the numbers).
Case 2. If 2S ≤ H, the situation is more difficult. If (S + H)2 > 2H 2 , then the answer and the
example are the same as in the previous case; otherwise the answer is (2m − 1)S 2 + 2SH(m − 1), and
the example is provided simply by (m − 1)2 nonintersecting napkins.
42
52nd IMO 2011 Combinatorics – solutions C7
Now we sketch the proof of both estimates for Case 2. We introduce a more appropriate notation
based on that from Solution 2. Denote by a− and a+ the number of cells of class A that contain the
number which is strictly less than M and strictly greater than M , respectively. The numbers b± , c± ,
and d± are defined in a similar way. One may notice that the proofs of Claim 1 and Claims 2, 3 lead
in fact to the inequalities
b− + c− d+ a b+ + c+ d+
m−1≤ + 2 and 2m − 1 ≤ 2
+ + 2
2SH H S 2SH H
(to obtain the first one, one needs to look at the big lines instead of the small ones). Combining these
inequalities, one may obtain the desired estimates.
These estimates can also be proved in some different ways, e.g. without distinguishing rich and poor
cells.
43
G1 Geometry – solutions 52nd IMO 2011
G1
Let ABC be an acute triangle. Let ω be a circle whose center L lies on the side BC. Suppose
that ω is tangent to AB at B ′ and to AC at C ′ . Suppose also that the circumcenter O of the
triangle ABC lies on the shorter arc B ′ C ′ of ω. Prove that the circumcircle of ABC and ω
meet at two points.
Solution. The point B ′ , being the perpendicular foot of L, is an interior point of side AB.
Analogously, C ′ lies in the interior of AC. The point O is located inside the triangle AB ′ C ′ ,
hence ∠COB < ∠C ′ OB ′ .
A
α
C′
B′
B L C
O′
Let α = ∠CAB. The angles ∠CAB and ∠C ′ OB ′ are inscribed into the two circles with
centers O and L, respectively, so ∠COB = 2∠CAB = 2α and 2∠C ′ OB ′ = 360◦ − ∠C ′ LB ′ .
From the kite AB ′ LC ′ we have ∠C ′ LB ′ = 180◦ − ∠C ′ AB ′ = 180◦ − α. Combining these, we
get
′ ′ 360◦ − ∠C ′ LB ′ 360◦ − (180◦ − α) α
2α = ∠COB < ∠C OB = = = 90◦ + ,
2 2 2
so
α < 60◦ .
Let O ′ be the reflection of O in the line BC. In the quadrilateral ABO ′ C we have
so the point O ′ is outside the circle ABC. Hence, O and O ′ are two points of ω such that one
of them lies inside the circumcircle, while the other one is located outside. Therefore, the two
circles intersect.
44
52nd IMO 2011 Geometry – solutions G1
Comment. There are different ways of reducing the statement of the problem to the case α < 60◦ .
E.g., since the point O lies in the interior of the isosceles triangle AB ′ C ′ , we have OA < AB ′ . So,
if AB ′ ≤ 2LB ′ then OA < 2LO, which means that ω intersects the circumcircle of ABC. Hence the
only interesting case is AB ′ > 2LB ′ , and this condition implies ∠CAB = 2∠B ′ AL < 2 · 30◦ = 60◦ .
45
G2 Geometry – solutions 52nd IMO 2011
G2
Let A1 A2 A3 A4 be a non-cyclic quadrilateral. Let O1 and r1 be the circumcenter and the
circumradius of the triangle A2 A3 A4 . Define O2 , O3 , O4 and r2 , r3 , r4 in a similar way. Prove
that
1 1 1 1
+ + + = 0.
O1 A1 − r1 O2 A2 − r2 O3 A3 − r3 O4 A4 − r42
2 2 2 2 2 2 2
O1 A21 − r12 = A1 B1 · A1 A3 .
On the other hand, from the equality MB1 · MA3 = MA2 · MA4 we obtain MB1 = yw/z.
Hence, we have yw z−x
O1 A21 − r12 = − x (z − x) = (yw − xz).
z z
Substituting the analogous expressions into the sought sum we get
4
X 1 1 z w x y
2 2
= − + − = 0,
i=1
Oi Ai − ri yw − xz z−x w−y x−z y−w
as desired.
Comment. One might reformulate the problem by assuming that the quadrilateral A1 A2 A3 A4 is
convex. This should not really change the difficulty, but proofs that distinguish several cases may
become shorter.
Solution 2. Introduce a Cartesian coordinate system in the plane. Every circle has an equation
of the form p(x, y) = x2 + y 2 + l(x, y) = 0, where l(x, y) is a polynomial of degree at most 1.
For any point A = (xA , yA ) we have p(xA , yA ) = d2 − r 2 , where d is the distance from A to the
center of the circle and r is the radius of the circle.
For each i in {1, 2, 3, 4} let pi (x, y) = x2 + y 2 + li (x, y) = 0 be the equation of the circle with
center Oi and radius ri and let di be the distance from Ai to Oi . Consider the equation
4
X pi (x, y)
= 1. (1)
i=1
d2i − ri2
46
52nd IMO 2011 Geometry – solutions G2
Since the coordinates of the points A1 , A2 , A3 , and A4 satisfy (1) but these four points do not
lie on a circle or on an line, equation (1) defines neither a circle, nor a line. Hence, the equation
is an identity and the coefficient of the quadratic term x2 + y 2 also has to be zero, i.e.
4
X 1
= 0.
d2
i=1 i
− ri2
Comment. Using the determinant form of the equation of the circle through three given points, the
same solution can be formulated as follows.
where i + 1, i + 2, and i + 3 have to be read modulo 4 as integers in the set {1, 2, 3, 4}.
u v 1 1
1 1
u2 v2 1 1
Expanding = 0 along the third column, we get ∆1 − ∆2 + ∆3 − ∆4 = 0.
u3 v3 1 1
u4 v4 1 1
The circle through Ai+1 , Ai+2 , and Ai+3 is given by the equation
x2 + y 2 x y 1
2 2
1 ui+1 + vi+1 ui+1 vi+1 1
= 0. (2)
∆i u2i+2 + vi+2
2 ui+2 vi+2 1
2 2
ui+3 + vi+3 ui+3 vi+3 1
On the left-hand side, the coefficient of x2 + y 2 is equal to 1. Substituting (ui , vi ) for (x, y) in (2) we
obtain the power of point Ai with respect to the circle through Ai+1 , Ai+2 , and Ai+3 :
u2 + v 2 ui vi 1
i i
2 2
1 ui+1 + vi+1 ui+1 vi+1 1 ∆
d2i − ri2 = = (−1)i+1 .
∆i u2i+2 + vi+2
2 ui+2 vi+2 1 ∆i
2 2
ui+3 + vi+3 ui+3 vi+3 1
Thus, we have
4
X 1 ∆1 − ∆2 + ∆3 − ∆4
= = 0.
d2
i=1 i
2
− ri ∆
47
G3 Geometry – solutions 52nd IMO 2011
G3
Let ABCD be a convex quadrilateral whose sides AD and BC are not parallel. Suppose that the
circles with diameters AB and CD meet at points E and F inside the quadrilateral. Let ωE be
the circle through the feet of the perpendiculars from E to the lines AB, BC, and CD. Let ωF
be the circle through the feet of the perpendiculars from F to the lines CD, DA, and AB.
Prove that the midpoint of the segment EF lies on the line through the two intersection points
of ωE and ωF .
Solution. Denote by P , Q, R, and S the projections of E on the lines DA, AB, BC, and
CD respectively. The points P and Q lie on the circle with diameter AE, so ∠QP E = ∠QAE;
analogously, ∠QRE = ∠QBE. So ∠QP E + ∠QRE = ∠QAE + ∠QBE = 90◦ . By similar
reasons, we have ∠SP E + ∠SRE = 90◦ , hence we get ∠QP S + ∠QRS = 90◦ + 90◦ = 180◦ ,
and the quadrilateral P QRS is inscribed in ωE . Analogously, all four projections of F onto the
sides of ABCD lie on ωF .
Denote by K the meeting point of the lines AD and BC. Due to the arguments above, there
is no loss of generality in assuming that A lies on segment DK. Suppose that ∠CKD ≥ 90◦ ;
then the circle with diameter CD covers the whole quadrilateral ABCD, so the points E, F
cannot lie inside this quadrilateral. Hence our assumption is wrong. Therefore, the lines EP
and BC intersect at some point P ′, while the lines ER and AD intersect at some point R′ .
C
M′
S
P′ ωE
B
N E
Q
K P A M N′ R′ D
Figure 1
We claim that the points P ′ and R′ also belong to ωE . Since the points R, E, Q, B are
concyclic, ∠QRK = ∠QEB = 90◦ − ∠QBE = ∠QAE = ∠QP E. So ∠QRK = ∠QP P ′ , which
means that the point P ′ lies on ωE . Analogously, R′ also lies on ωE .
In the same manner, denote by M and N the projections of F on the lines AD and BC
48
52nd IMO 2011 Geometry – solutions G3
respectively, and let M ′ = F M ∩ BC, N ′ = F N ∩ AD. By the same arguments, we obtain that
the points M ′ and N ′ belong to ωF .
M′ ωF
P′
R g
E
N
V
ωE
U
F
K P M N′ R′
Figure 2
Now we concentrate on Figure 2, where all unnecessary details are removed. Let U = NN ′ ∩
P P ′ , V = MM ′ ∩ RR′ . Due to the right angles at N and P , the points N, N ′ , P , P ′ are
concyclic, so UN · UN ′ = UP · UP ′ which means that U belongs to the radical axis g of the
circles ωE and ωF . Analogously, V also belongs to g.
Finally, since EUF V is a parallelogram, the radical axis UV of ωE and ωF bisects EF .
49
G4 Geometry – solutions 52nd IMO 2011
G4
Let ABC be an acute triangle with circumcircle Ω. Let B0 be the midpoint of AC and let C0
be the midpoint of AB. Let D be the foot of the altitude from A, and let G be the centroid
of the triangle ABC. Let ω be a circle through B0 and C0 that is tangent to the circle Ω at a
point X 6= A. Prove that the points D, G, and X are collinear.
Solution 1. If AB = AC, then the statement is trivial. So without loss of generality we may
assume AB < AC. Denote the tangents to Ω at points A and X by a and x, respectively.
Let Ω1 be the circumcircle of triangle AB0 C0 . The circles Ω and Ω1 are homothetic with center
A, so they are tangent at A, and a is their radical axis. Now, the lines a, x, and B0 C0 are the
three radical axes of the circles Ω, Ω1 , and ω. Since a 6 kB0 C0 , these three lines are concurrent
at some point W .
The points A and D are symmetric with respect to the line B0 C0 ; hence W X = W A = W D.
This means that W is the center of the circumcircle γ of triangle ADX. Moreover, we have
∠W AO = ∠W XO = 90◦ , where O denotes the center of Ω. Hence ∠AW X + ∠AOX = 180◦ .
a
A T
Ω1
Ω
C0
W B0
G
O
B D A0 C
ω
X
γ
x
Denote by T the second intersection point of Ω and the line DX. Note that O belongs to Ω1 .
Using the circles γ and Ω, we find ∠DAT = ∠ADX −∠AT D = 21 (360◦ −∠AW X)− 21 ∠AOX =
180◦ − 12 (∠AW X + ∠AOX) = 90◦ . So, AD ⊥ AT , and hence AT k BC. Thus, AT CB is an
isosceles trapezoid inscribed in Ω.
Denote by A0 the midpoint of BC, and consider the image of AT CB under the homothety h
with center G and factor − 21 . We have h(A) = A0 , h(B) = B0 , and h(C) = C0 . From the
50
52nd IMO 2011 Geometry – solutions G4
symmetry about B0 C0 , we have ∠T CB = ∠CBA = ∠B0 C0 A = ∠DC0 B0 . Using AT k DA0 ,
we conclude h(T ) = D. Hence the points D, G, and T are collinear, and X lies on the same
line.
Solution 2. We define the points A0 , O, and W as in the previous solution and we concentrate
on the case AB < AC. Let Q be the perpendicular projection of A0 on B0 C0 .
W C0 Q B0
G
O
J B D A0 C
X
x
To complete the argument, we note that the homothety centered at G sending the triangle ABC
to the triangle A0 B0 C0 maps the altitude AD to the altitude A0 Q. Therefore it maps D to Q,
so the points D, G, and Q are collinear. Hence G lies on ℓ as well.
Comment. There are various other ways to prove the collinearity of Q, D, and X obtained in the
middle part of Solution 2. Introduce for instance the point J where the lines AW and BC intersect.
Then the four points A, D, X, and J lie at the same distance from W , so the quadrilateral ADXJ is
cyclic. In combination with the fact that AW XQ is cyclic as well, this implies
51
G5 Geometry – solutions 52nd IMO 2011
G5
Let ABC be a triangle with incenter I and circumcircle ω. Let D and E be the second
intersection points of ω with the lines AI and BI, respectively. The chord DE meets AC at a
point F , and BC at a point G. Let P be the intersection point of the line through F parallel to
AD and the line through G parallel to BE. Suppose that the tangents to ω at A and at B meet
at a point K. Prove that the three lines AE, BD, and KP are either parallel or concurrent.
Solution 1. Since
the quadrilateral AIF E is cyclic. Denote its circumcircle by ω1 . Similarly, the quadrilat-
eral BDGI is cyclic; denote its circumcircle by ω2 .
The line AE is the radical axis of ω and ω1 , and the line BD is the radical axis of ω and ω2 .
Let t be the radical axis of ω1 and ω2 . These three lines meet at the radical center of the three
circles, or they are parallel to each other. We will show that t is in fact the line P K.
Let L be the second intersection point of ω1 and ω2 , so t = IL. (If the two circles are tangent
to each other then L = I and t is the common tangent.)
ω1
A
ω
K′=K I F
L
P′=P
B G C
ω2
D
Let the line t meet the circumcircles of the triangles ABL and F GL at K ′ 6= L and P ′ 6= L,
respectively. Using oriented angles we have
∠(AB, BK ′ ) = ∠(AL, LK ′ ) = ∠(AL, LI) = ∠(AE, EI) = ∠(AE, EB) = ∠(AB, BK),
52
52nd IMO 2011 Geometry – solutions G5
so BK ′ k BK. Similarly we have AK ′ k AK, and therefore K ′ = K. Next, we have
∠(P ′F, F G) = ∠(P ′L, LG) = ∠(IL, LG) = ∠(ID, DG) = ∠(AD, DE) = ∠(P F, F G),
Solution 2. Let M be the intersection point of the tangents to ω at D and E, and let the
lines AE and BD meet at T ; if AE and BD are parallel, then let T be their common ideal
point. It is well-known that the points K and M lie on the line T I (as a consequence of
Pascal’s theorem, applied to the inscribed degenerate hexagons AADBBE and ADDBEE).
The lines AD and BE are the angle bisectors of the angles ∠CAB and ∠ABC, respectively, so
D and E are the midpoints of the arcs BC and CA of the circle ω, respectively. Hence, DM
is parallel to BC and EM is parallel to AC.
Apply Pascal’s theorem to the degenerate hexagon CADDEB. By the theorem, the points
CA ∩ DE = F , AD ∩ EB = I and the common ideal point of lines DM and BC are collinear,
therefore F I is parallel to BC and DM. Analogously, the line GI is parallel to AC and EM.
ω E
A
T
K
I F
H P
B G C
M
D
FG
Now consider the homothety with scale factor − ED which takes E to G and D to F . Since the
triangles EDM and GF I have parallel sides, the homothety takes M to I. Similarly, since the
triangles DEI and F GP have parallel sides, the homothety takes I to P . Hence, the points
M, I, P and the homothety center H must lie on the same line. Therefore, the point P also
lies on the line T KIM.
Comment. One may prove that IF k BC and IG k AC in a more elementary way. Since ∠ADE =
∠EDC and ∠DEB = ∠CED, the points I and C are symmetric about DE. Moreover, since the
arcs AE and EC are equal and the arcs CD and DB are equal, we have ∠CF G = ∠F GC, so the
triangle CF G is isosceles. Hence, the quadrilateral IF CG is a rhombus.
53
G6 Geometry – solutions 52nd IMO 2011
G6
Let ABC be a triangle with AB = AC, and let D be the midpoint of AC. The angle bisector
of ∠BAC intersects the circle through D, B, and C in a point E inside the triangle ABC.
The line BD intersects the circle through A, E, and B in two points B and F . The lines AF
and BE meet at a point I, and the lines CI and BD meet at a point K. Show that I is the
incenter of triangle KAB.
Solution 1. Let D ′ be the midpoint of the segment AB, and let M be the midpoint of BC.
By symmetry at line AM, the point D ′ has to lie on the circle BCD. Since the arcs D ′ E
and ED of that circle are equal, we have ∠ABI = ∠D ′BE = ∠EBD = IBK, so I lies on
the angle bisector of ∠ABK. For this reason it suffices to prove in the sequel that the ray AI
bisects the angle ∠BAK.
From
1 1
∠DF A = 180◦ − ∠BF A = 180◦ − ∠BEA = ∠MEB = ∠CEB = ∠CDB
2 2
we derive ∠DF A = ∠DAF so the triangle AF D is isosceles with AD = DF .
E
D′ I D
K
ω1
B M C
ω2
Applying Menelaus’s theorem to the triangle ADF with respect to the line CIK, and applying
the angle bisector theorem to the triangle ABF , we infer
AC DK F I DK BF DK BF DK BF
1= · · =2· · =2· · = ·
CD KF IA KF AB KF 2 · AD KF AD
and therefore
BD BF + F D BF KF DF AD
= = +1= +1= = .
AD AD AD DK DK DK
54
52nd IMO 2011 Geometry – solutions G6
It follows that the triangles ADK and BDA are similar, hence ∠DAK = ∠ABD. Then
shows that the point K is indeed lying on the angle bisector of ∠BAK.
Solution 2. It can be shown in the same way as in the first solution that I lies on the angle
bisector of ∠ABK. Here we restrict ourselves to proving that KI bisects ∠AKB.
A
I D
ω1 O3 K
F
B C
ω2
ω3
O1
Denote the circumcircle of triangle BCD and its center by ω1 and by O1 , respectively. Since
the quadrilateral ABF E is cyclic, we have ∠DF E = ∠BAE = ∠DAE. By the same reason,
we have ∠EAF = ∠EBF = ∠ABE = ∠AF E. Therefore ∠DAF = ∠DF A, and hence
DF = DA = DC. So triangle AF C is inscribed in a circle ω2 with center D.
Denote the circumcircle of triangle ABD by ω3 , and let its center be O3 . Since the arcs BE
and EC of circle ω1 are equal, and the triangles ADE and F DE are congruent, we have
∠AO1 B = 2∠BDE = ∠BDA, so O1 lies on ω3 . Hence ∠O3 O1 D = ∠O3 DO1 .
The line BD is the radical axis of ω1 and ω3 . Point C belongs to the radical axis of ω1 and ω2 ,
and I also belongs to it since AI ·IF = BI ·IE. Hence K = BD ∩CI is the radical center of ω1 ,
ω2 , and ω3 , and AK is the radical axis of ω2 and ω3 . Now, the radical axes AK, BK and IK are
perpendicular to the central lines O3 D, O3 O1 and O1 D, respectively. By ∠O3 O1 D = ∠O3 DO1 ,
we get that KI is the angle bisector of ∠AKB.
Solution 3. Again, let M be the midpoint of BC. As in the previous solutions, we can deduce
∠ABI = ∠IBK. We show that the point I lies on the angle bisector of ∠KAB.
Let G be the intersection point of the circles AF C and BCD, different from C. The lines
55
G6 Geometry – solutions 52nd IMO 2011
CG, AF , and BE are the radical axes of the three circles AGF C, CDB, and ABF E, so
I = AF ∩ BE is the radical center of the three circles and CG also passes through I.
A B′
E
G
D
I
K
B M C
The angle between line DE and the tangent to the circle BCD at E is equal to ∠EBD =
∠EAF = ∠ABE = ∠AF E. As the tangent at E is perpendicular to AM, the line DE is
perpendicular to AF . The triangle AF E is isosceles, so DE is the perpendicular bisector
of AF and thus AD = DF . Hence, the point D is the center of the circle AF C, and this circle
passes through M as well since ∠AMC = 90◦ .
Let B ′ be the reflection of B in the point D, so ABCB ′ is a parallelogram. Since DC = DG
we have ∠GCD = ∠DBC = ∠KB ′ A. Hence, the quadrilateral AKCB ′ is cyclic and thus
∠CAK = ∠CB ′ K = ∠ABD = 2∠MAI. Then
1 1 1
∠IAB = ∠MAB − ∠MAI = ∠CAB − ∠CAK = ∠KAB
2 2 2
and therefore AI is the angle bisector of ∠KAB.
56
52nd IMO 2011 Geometry – solutions G7
G7
Let ABCDEF be a convex hexagon all of whose sides are tangent to a circle ω with center O.
Suppose that the circumcircle of triangle ACE is concentric with ω. Let J be the foot of the
perpendicular from B to CD. Suppose that the perpendicular from B to DF intersects the
line EO at a point K. Let L be the foot of the perpendicular from K to DE. Prove that
DJ = DL.
Solution 1. Since ω and the circumcircle of triangle ACE are concentric, the tangents from A,
C, and E to ω have equal lengths; that means that AB = BC, CD = DE, and EF = F A.
Moreover, we have ∠BCD = ∠DEF = ∠F AB.
A
B
F
J L′
O B′
C E
M K′
B′′
ω P
Consider the rotation around point D mapping C to E; let B ′ and L′ be the images of the
points B and J, respectively, under this rotation. Then one has DJ = DL′ and B ′ L′ ⊥ DE;
moreover, the triangles B ′ ED and BCD are congruent. Since ∠DEO < 90◦ , the lines EO
and B ′ L′ intersect at some point K ′ . We intend to prove that K ′ B ⊥ DF ; this would imply
K = K ′ , therefore L = L′ , which proves the problem statement.
Analogously, consider the rotation around F mapping A to E; let B ′′ be the image of B under
this rotation. Then the triangles F AB and F EB ′′ are congruent. We have EB ′′ = AB = BC =
EB ′ and ∠F EB ′′ = ∠F AB = ∠BCD = ∠DEB ′ , so the points B ′ and B ′′ are symmetrical
with respect to the angle bisector EO of ∠DEF . So, from K ′ B ′ ⊥ DE we get K ′ B ′′ ⊥ EF .
From these two relations we obtain
K ′D2 − K ′ E 2 = B ′ D2 − B ′E 2 and K ′ E 2 − K ′ F 2 = B ′′ E 2 − B ′′ F 2 .
K ′ D 2 − K ′ F 2 = B ′ D 2 − B ′′ F 2 = BD 2 − BF 2 ,
57
G7 Geometry – solutions 52nd IMO 2011
Comment. There are several variations of this solution. For instance, let us consider the intersection
point M of the lines BJ and OC. Define the point K ′ as the reflection of M in the line DO. Then
one has
DK ′2 − DB 2 = DM 2 − DB 2 = CM 2 − CB 2 .
Next, consider the rotation around O which maps CM to EK ′ . Let P be the image of B under this
rotation; so P lies on ED. Then EF ⊥ K ′ P , so
CM 2 − CB 2 = EK ′2 − EP 2 = F K ′2 − F P 2 = F K ′2 − F B 2 ,
Solution 2. Let us denote the points of tangency of AB, BC, CD, DE, EF , and F A to ω
by R, S, T , U, V , and W , respectively. As in the previous solution, we mention that AR =
AW = CS = CT = EU = EV .
The reflection in the line BO maps R to S, therefore A to C and thus also W to T . Hence, both
lines RS and W T are perpendicular to OB, therefore they are parallel. On the other hand,
the lines UV and W T are not parallel, since otherwise the hexagon ABCDEF is symmetric
with respect to the line BO and the lines defining the point K coincide, which contradicts the
conditions of the problem. Therefore we can consider the intersection point Z of UV and W T .
R A
B W
F
J L
S V
O
C E
T K
U
Next, we recall a well-known fact that the points D, F , Z are collinear. Actually, D is the pole
of the line UT , F is the pole of V W , and Z = T W ∩ UV ; so all these points belong to the
polar line of T U ∩ V W .
58
52nd IMO 2011 Geometry – solutions G7
−−→
Now, we put O into the origin, and identify each point (say X) with the vector OX. So, from
now on all the products of points refer to the scalar products of the corresponding vectors.
Since OK ⊥ UZ and OB ⊥ T Z, we have K · (Z − U) = 0 = B · (Z − T ). Next, the
condition BK ⊥ DZ can be written as K · (D − Z) = B · (D − Z). Adding these two equalities
we get
K · (D − U) = B · (D − T ).
59
G8 Geometry – solutions 52nd IMO 2011
G8
Let ABC be an acute triangle with circumcircle ω. Let t be a tangent line to ω. Let ta , tb ,
and tc be the lines obtained by reflecting t in the lines BC, CA, and AB, respectively. Show
that the circumcircle of the triangle determined by the lines ta , tb , and tc is tangent to the
circle ω.
To avoid a large case distinction, we will use the notion of oriented angles. Namely, for two
lines ℓ and m, we denote by ∠(ℓ, m) the angle by which one may rotate ℓ anticlockwise to
obtain a line parallel to m. Thus, all oriented angles are considered modulo 180◦ .
K A
tc B′′
C ′′
B′
X ta
C ′=S
B C
I D
A′′ t
E
T
F
tb
A′
It follows that ta and B ′′ C ′′ are parallel. Similarly, tb k A′′ C ′′ and tc k A′′ B ′′ . Thus, either the
triangles A′ B ′ C ′ and A′′ B ′′ C ′′ are homothetic, or they are translates of each other. Now we
will prove that they are in fact homothetic, and that the center K of the homothety belongs
60
52nd IMO 2011 Geometry – solutions G8
to ω. It would then follow that their circumcircles are also homothetic with respect to K and
are therefore tangent at this point, as desired.
Claim 2. The point of intersection I of the lines BB ′ and CC ′ lies on the circle ω.
Proof. We consider the case that t is not parallel to the sides of ABC; the other cases may be
regarded as limit cases. Let D = t ∩ BC, E = t ∩ AC, and F = t ∩ AB.
Due to symmetry, the line DB is one of the angle bisectors of the lines B ′ D and F D; analogously,
the line F B is one of the angle bisectors of the lines B ′ F and DF . So B is either the incenter
or one of the excenters of the triangle B ′ DF . In any case we have ∠(BD, DF ) + ∠(DF, F B) +
∠(B ′ B, B ′ D) = 90◦ , so
∠(BI, CI) = ∠(B ′ B, B ′ C ′ ) + ∠(B ′ C ′ , C ′ C) = ∠(BC, AC) − ∠(BC, AB) = ∠(AB, AC),
Now we can complete the proof. Let K be the second intersection point of B ′ B ′′ and ω.
Applying Pascal’s theorem to hexagon KB ′′ CIBC ′′ we get that the points B ′ = KB ′′ ∩ IB
and X = B ′′ C ∩ BC ′′ are collinear with the intersection point S of CI and C ′′ K. So S =
CI ∩ B ′ X = C ′ , and the points C ′ , C ′′ , K are collinear. Thus K is the intersection point
of B ′ B ′′ and C ′ C ′′ which implies that K is the center of the homothety mapping A′ B ′ C ′
to A′′ B ′′ C ′′ , and it belongs to ω.
Solution 2. Define the points T , A′ , B ′ , and C ′ in the same way as in the previous solution.
Let X, Y , and Z be the symmetric images of T about the lines BC, CA, and AB, respectively.
Note that the projections of T on these lines form a Simson line of T with respect to ABC,
therefore the points X, Y , Z are also collinear. Moreover, we have X ∈ B ′ C ′ , Y ∈ C ′ A′ ,
Z ∈ A′ B ′ .
Denote α = ∠(t, T C) = ∠(BT, BC). Using the symmetry in the lines AC and BC, we get
Since ∠(XC, XC ′) = ∠(Y C, Y C ′ ), the points X, Y , C, C ′ lie on some circle ωc . Define the
circles ωa and ωb analogously. Let ω ′ be the circumcircle of triangle A′ B ′ C ′ .
61
G8 Geometry – solutions 52nd IMO 2011
Now, applying Miquel’s theorem to the four lines A′ B ′ , A′ C ′ , B ′ C ′ , and XY , we obtain that
the circles ω ′ , ωa , ωb , ωc intersect at some point K. We will show that K lies on ω, and that
the tangent lines to ω and ω ′ at this point coincide; this implies the problem statement.
Due to symmetry, we have XB = T B = ZB, so the point B is the midpoint of one of the
arcs XZ of circle ωb . Therefore ∠(KB, KX) = ∠(XZ, XB). Analogously, ∠(KX, KC) =
∠(XC, XY ). Adding these equalities and using the symmetry in the line BC we get
∠(KB, KC) = ∠(XZ, XB) + ∠(XC, XZ) = ∠(XC, XB) = ∠(T B, T C).
Therefore, K lies on ω.
Next, let k be the tangent line to ω at K. We have
K A
ωb
ω ω′
ωc
Z
B′ tc
X
C′
B C Y
t
T
ta tb
A′
Comment. There exist various solutions combining the ideas from the two solutions presented above.
For instance, one may define the point X as the reflection of T with respect to the line BC, and
then introduce the point K as the second intersection point of the circumcircles of BB ′ X and CC ′ X.
Using the fact that BB ′ and CC ′ are the bisectors of ∠(A′ B ′ , B ′ C ′ ) and ∠(A′ C ′ , B ′ C ′ ) one can show
successively that K ∈ ω, K ∈ ω ′ , and that the tangents to ω and ω ′ at K coincide.
62
52nd IMO 2011 Number Theory – solutions N1
N1
For any integer d > 0, let f (d) be the smallest positive integer that has exactly d positive
divisors (so for example we have f (1) = 1, f (5) = 16, and f (6) = 12). Prove that for every
integer k ≥ 0 the number f (2k ) divides f (2k+1).
Solution 1. For any positive integer n, let d(n) be the number of positive divisors of n. Let
Q
n = p pa(p) be the prime factorization of n where p ranges over the prime numbers, the integers
Q
a(p) are nonnegative and all but finitely many a(p) are zero. Then we have d(n) = p (a(p)+1).
Thus, d(n) is a power of 2 if and only if for every prime p there is a nonnegative integer b(p)
with a(p) = 2b(p) − 1 = 1 + 2 + 22 + · · · + 2b(p)−1 . We then have
Y b(p)−1
Y i
X
n= p2 , and d(n) = 2k with k = b(p).
p i=0 p
r
Let S be the set of all numbers of the form p2 with p prime and r a nonnegative integer. Then
we deduce that d(n) is a power of 2 if and only if n is the product of the elements of some finite
subset T of S that satisfies the following condition: for all t ∈ T and s ∈ S with s t we have
s ∈ T . Moreover, if d(n) = 2k then the corresponding set T has k elements.
Note that the set Tk consisting of the smallest k elements from S obviously satisfies the condition
above. Thus, given k, the smallest n with d(n) = 2k is the product of the elements of Tk . This n
is f (2k ). Since obviously Tk ⊂ Tk+1 , it follows that f (2k ) f (2k+1 ).
To see this, note first that ℓ divides f (2k ). With the first part of Solution 1 one can see that
the integer n = f (2k )m/ℓ also satisfies d(n) = 2k . By the definition of f (2k ) this implies that
n ≥ f (2k ) so m ≥ ℓ. Since p 6= q the inequality (1) follows.
Q
Let the prime factorization of f (2k+1) be given by f (2k+1) = p pr(p) with r(p) = 2s(p) − 1.
P P
Since we have p s(p) = k + 1 > k = p b(p) there is a prime p with s(p) > b(p). For any
prime q 6= p with b(q) > 0 we apply inequality (1) twice and get
s(q) s(p)−1 b(p) b(q)−1
q2 > p2 ≥ p2 > q2 ,
which implies s(q) ≥ b(q). It follows that s(q) ≥ b(q) for all primes q, so f (2k ) f (2k+1 ).
63
N2 Number Theory – solutions 52nd IMO 2011
N2
Consider a polynomial P (x) = (x + d1 )(x + d2 ) · . . . · (x + d9 ), where d1 , d2 , . . . , d9 are nine
distinct integers. Prove that there exists an integer N such that for all integers x ≥ N the
number P (x) is divisible by a prime number greater than 20.
Solution 1. Note that the statement of the problem is invariant under translations of x; hence
without loss of generality we may suppose that the numbers d1 , d2 , . . . , d9 are positive.
The key observation is that there are only eight primes below 20, while P (x) involves more
than eight factors.
We shall prove that N = d8 satisfies the desired property, where d = max{d1 , d2 , . . . , d9 }.
Suppose for the sake of contradiction that there is some integer x ≥ N such that P (x) is
composed of primes below 20 only. Then for every index i ∈ {1, 2, . . . , 9} the number x + di
can be expressed as product of powers of the first 8 primes.
Since x + di > x ≥ d8 there is some prime power fi > d that divides x + di . Invoking the
pigeonhole principle we see that there are two distinct indices i and j such that fi and fj are
powers of the same prime number. For reasons of symmetry, we may suppose that fi ≤ fj .
Now both of the numbers x + di and x + dj are divisible by fi and hence so is their difference
di − dj . But as
0 < |di − dj | ≤ max(di , dj ) ≤ d < fi ,
By the pigeonhole principle, there are a prime number p and two distinct indices i and j such
that pi = pj = p. Let pαi and pαj be the greatest powers of p dividing x + di and x + dj ,
respectively. Due to symmetry we may suppose αi ≤ αj . But now pαi divides di − dj and hence
also Di , which means that all occurrences of p in the numerator of the fraction (x + di )/Di
cancel out, contrary to the choice of p = pi . This contradiction proves our claim.
64
52nd IMO 2011 Number Theory – solutions N2
Solution 3. Given a nonzero integer N as well as a prime number p we write vp (N) for the
exponent with which p occurs in the prime factorization of |N|.
Evidently, if the statement of the problem were not true, then there would exist an infinite
sequence (xn ) of positive integers tending to infinity such that for each n ∈ Z+ the integer
P (xn ) is not divisible by any prime number > 20. Observe that the numbers −d1 , −d2 , . . . , −d9
do not appear in this sequence.
Now clearly there exists a prime p1 < 20 for which the sequence vp1 (xn + d1 ) is not bounded;
thinning out the sequence (xn ) if necessary we may even suppose that
vp1 (xn + d1 ) −→ ∞.
Repeating this argument eight more times we may similarly choose primes p2 , . . . , p9 < 20 and
suppose that our sequence (xn ) has been thinned out to such an extent that vpi (xn + di ) −→ ∞
holds for i = 2, . . . , 9 as well. In view of the pigeonhole principle, there are distinct indices i
and j as well as a prime p < 20 such that pi = pj = p. Setting k = vp (di − dj ) there now has to
be some n for which both vp (xn + di ) and vp (xn + dj ) are greater than k. But now the numbers
xn + di and xn + dj are divisible by pk+1 whilst their difference di − dj is not – a contradiction.
Comment. This problem is supposed to be a relatively easy one, so one might consider adding the
hypothesis that the numbers d1 , d2 , . . . , d9 be positive. Then certain merely technical issues are not
going to arise while the main ideas required to solve the problems remain the same.
65
N3 Number Theory – solutions 52nd IMO 2011
N3
Let n ≥ 1 be an odd integer. Determine all functions f from the set of integers to itself such
that for all integers x and y the difference f (x) − f (y) divides xn − y n .
Answer. All functions f of the form f (x) = εxd + c, where ε is in {1, −1}, the integer d is a
positive divisor of n, and c is an integer.
Solution. Obviously, all functions in the answer satisfy the condition of the problem. We will
show that there are no other functions satisfying that condition.
Let f be a function satisfying the given condition. For each integer n, the function g defined
by g(x) = f (x) + n also satisfies the same condition. Therefore, by subtracting f (0) from f (x)
we may assume that f (0) = 0.
For any prime p, the condition on f with (x, y) = (p, 0) states that f (p) divides pn . Since the
set of primes is infinite, there exist integers d and ε with 0 ≤ d ≤ n and ε ∈ {1, −1} such that
for infinitely many primes p we have f (p) = εpd . Denote the set of these primes by P . Since a
function g satisfies the given condition if and only if −g satisfies the same condition, we may
suppose ε = 1.
The case d = 0 is easily ruled out, because 0 does not divide any nonzero integer. Suppose
d ≥ 1 and write n as md + r, where m and r are integers such that m ≥ 1 and 0 ≤ r ≤ d − 1.
Let x be an arbitrary integer. For each prime p in P , the difference f (p) − f (x) divides pn − xn .
Using the equality f (p) = pd , we get
Hence pr f (x)m − xn has to be zero. This implies r = 0 and xn = (xd )m = f (x)m . Since m is
odd, we obtain f (x) = xd .
Comment. If n is an even positive integer, then the functions f of the form
xd + c for some integers,
f (x) =
−xd + c for the rest of integers,
where d is a positive divisor of n/2 and c is an integer, also satisfy the condition of the problem.
Together with the functions in the answer, they are all functions that satisfy the condition when n is
even.
66
52nd IMO 2011 Number Theory – solutions N4
N4
For each positive integer k, let t(k) be the largest odd divisor of k. Determine all positive
integers a for which there exists a positive integer n such that all the differences
are divisible by 4.
Answer. a = 1, 3, or 5.
Solution. A pair (a, n) satisfying the condition of the problem will be called a winning pair.
It is straightforward to check that the pairs (1, 1), (3, 1), and (5, 4) are winning pairs.
Now suppose that a is a positive integer not equal to 1, 3, and 5. We will show that there are
no winning pairs (a, n) by distinguishing three cases.
Case 1: a is even. In this case we have a = 2α d for some positive integer α and some odd d. Since
a ≥ 2α , for each positive integer n there exists an i ∈ {0, 1, . . . , a − 1} such that n + i = 2α−1 e,
where e is some odd integer. Then we have t(n + i) = t(2α−1 e) = e and
So we get t(n + i) − t(n + a + i) ≡ 2 (mod 4), and (a, n) is not a winning pair.
Case 2: a is odd and a > 8. For each positive integer n, there exists an i ∈ {0, 1, . . . , a − 5}
such that n + i = 2d for some odd d. We get
and
t(n + a + i) = n + a + i ≡ n + a + i + 4 = t(n + a + i + 4) (mod 4).
Therefore, the integers t(n + a + i) − t(n + i) and t(n + a + i + 4) − t(n + i + 4) cannot be both
divisible by 4, and therefore there are no winning pairs in this case.
Case 3: a = 7. For each positive integer n, there exists an i ∈ {0, 1, . . . , 6} such that n + i is
either of the form 8k + 3 or of the form 8k + 6, where k is a nonnegative integer. But we have
and
t(8k + 6) = 4k + 3 ≡ 3 6≡ 1 ≡ t(8k + 6 + 7) (mod 4).
67
N5 Number Theory – solutions 52nd IMO 2011
N5
Let f be a function from the set of integers to the set of positive integers. Suppose that for
any two integers m and n, the difference f (m) − f (n) is divisible by f (m − n). Prove that for
all integers m, n with f (m) ≤ f (n) the number f (n) is divisible by f (m).
Solution 1. Suppose that x and y are two integers with f (x) < f (y). We will show that
f (x) f (y). By taking m = x and n = y we see that
f (x − y) |f (x) − f (y)| = f (y) − f (x) > 0,
Solution 2. We split the solution into a sequence of claims; in each claim, the letters m and n
denote arbitrary integers.
Claim 1. f (n) f (mn).
Proof. Since trivially f (n) f (1 · n) and f (n) f ((k + 1)n) − f (kn) for all integers k, this is
easily seen by using induction on m in both directions.
Claim 2. f (n) f (0) and f (n) = f (−n).
Proof. The first part follows by plugging m = 0 into Claim 1. Using Claim 1 twice with
m = −1, we get f (n) f (−n) f (n), from which the second part follows.
From Claim 1, we get f (1) f (n) for all integers n, so f (1) is the minimal value attained by f .
Next, from Claim 2, the function f can attain only a finite number of values since all these
values divide f (0).
Now we prove the statement of the problem by induction on the number Nf of values attained
by f . In the base case Nf ≤ 2, we either have f (0) 6= f (1), in which case these two numbers
are the only values attained by f and the statement is clear, or we have f (0) = f (1), in which
case we have f (1) f (n) f (0) for all integers n, so f is constant and the statement is obvious
again.
For the induction step, assume that Nf ≥ 3, and let a be the least positive integer with
f (a) > f (1). Note that such a number exists due to the symmetry of f obtained in Claim 2.
68
52nd IMO 2011 Number Theory – solutions N5
Claim 3. f (n) 6= f (1) if and only if a n.
Proof. Since f (1) = · · · = f (a − 1) < f (a), the claim follows from the fact that
We return to the induction step. So let us take two arbitrary integers m and n with f (m) ≤ f (n).
If a 6 m, then we have f (m) = f (1) f (n). On the other hand, suppose that a m; then by
Claim 3 a n as well. Now define the function g(x) = f (ax). Clearly, g satisfies the condi-
tions of the problem, but Ng < Nf − 1, since g does not attain f (1). Hence, by the induction
hypothesis, f (m) = g(m/a) g(n/a) = f (n), as desired.
Comment. After the fact that f attains a finite number of values has been established, there are
several ways of finishing the solution. For instance, let f (0) = b1 > b2 > · · · > bk be all these values.
One may show (essentially in the same way as in Claim 3) that the set Si = {n : f (n) ≥ bi } consists
exactly of all numbers divisible by some integer ai ≥ 0. One obviously has ai ai−1 , which implies
f (ai ) f (ai−1 ) by Claim 1. So, bk bk−1 · · · b1 , thus proving the problem statement.
Moreover, now it is easy to describe all functions satisfying the conditions of the problem. Namely, all
these functions can be constructed as follows. Consider a sequence of nonnegative integers a1 , a2 , . . . , ak
and another sequence of positive integers b1 , b2 , . . . , bk such that |ak | = 1, ai 6= aj and bi 6= bj for all
1 ≤ i < j ≤ k, and ai ai−1 and bi bi−1 for all i = 2, . . . , k. Then one may introduce the function
f (n) = bi(n) , where i(n) = min{i : ai n}.
These are all the functions which satisfy the conditions of the problem.
69
N6 Number Theory – solutions 52nd IMO 2011
N6
Let P (x) and Q(x) be two polynomials with integer coefficients such that no nonconstant
polynomial with rational coefficients divides both P (x) and Q(x). Suppose that for every
positive integer n the integers P (n) and Q(n) are positive, and 2Q(n) − 1 divides 3P (n) − 1.
Prove that Q(x) is a constant polynomial.
Solution. First we show that there exists an integer d such that for all positive integers n we
have gcd P (n), Q(n) ≤ d.
Since P (x) and Q(x) are coprime (over the polynomials with rational coefficients), Euclid’s al-
gorithm provides some polynomials R0 (x), S0 (x) with rational coefficients such that P (x)R0 (x)−
Q(x)S0 (x) = 1. Multiplying by a suitable positive integer d, we obtain polynomials R(x) =
d · R0 (x) and S(x) = d · S0 (x) with integer coefficients for which P (x)R(x) − Q(x)S(x) = d.
Then we have gcd P (n), Q(n) ≤ d for any integer n.
To prove the problem statement, suppose that Q(x) is not constant. Then the sequence Q(n)
is not bounded and we can choose a positive integer m for which
and therefore
M 2Q(m+ax) − 1 3P (m+ax) − 1.
70
52nd IMO 2011 Number Theory – solutions N6
which contradicts (1).
k
X k
(−1)k−i P (n + ai) = p · k! · ak .
i
i=0
Finally, we get b gcd P (n), k!·p ·Q(n)k , which is bounded by the same arguments as in the beginning
of the solution. So 3b − 1 is bounded, and hence 2Q(n) − 1 is bounded as well.
71
N7 Number Theory – solutions 52nd IMO 2011
N7
Let p be an odd prime number. For every integer a, define the number
a a2 ap−1
Sa = + +···+ .
1 2 p−1
Solution 1. For rational numbers p1 /q1 and p2 /q2 with the denominators q1 , q2 not divisible
by p, we write p1 /q1 ≡ p2 /q2 (mod p) if the numerator p1 q2 − p2 q1 of their difference is divisible
by p.
We start with finding an explicit formula for the residue of Sa modulo p. Note first that for
every k = 1, . . . , p − 1 the number kp is divisible by p, and
1 p (p − 1)(p − 2) · · · (p − k + 1) (−1) · (−2) · · · (−k + 1) (−1)k−1
= ≡ = (mod p)
p k k! k! k
Therefore, we have
p−1 p−1
X (−a)k (−1)k−1 X 1 p
k
Sa = − ≡− (−a) · (mod p).
k=1
k k=1
p k
The number on the right-hand side is integer. Using the binomial formula we express it as
p−1 p !
X
k 1 p 1 X p (a − 1)p − ap + 1
− (−a) · =− −1 − (−a)p + (−a)k =
k=1
p k p k=0
k p
(a − 1)p − ap + 1
Sa ≡ (mod p).
p
72
52nd IMO 2011 Number Theory – solutions N7
Solution 2. One may solve the problem without finding an explicit formula for Sa . It is
enough to find the following property.
p
Note that k1 + p−k
1
= k(p−k) ≡ 0 (mod p) for all 1 ≤ k ≤ p − 1; hence the first sum vanishes
modulo p. For the second sum, we use the relation k1 kj = 1j k−1
j−1
to obtain
p−1 j p−1
X a X k−1
Sa+1 ≡ (mod p).
j=1
j k=1 j − 1
p−1
X k−1 p−1 (p − 1)(p − 2) . . . (p − j)
= = ≡ (−1)j (mod p)
k=1
j −1 j j!
we obtain
p−1 j
X a (−1)j
Sa+1 ≡ = S−a .
j=1
j!
X −2 · 2k X −4 · 2k
S3 − 3S2 ≡ S−2 − 3S2 = + (mod p). (1)
1≤k≤p−1
k 1≤k≤p−1
k
k is even k is odd
(p−1)/2 (p−1)/2 ℓ
X −2 · 22ℓ X 4
=− .
ℓ=1
2ℓ ℓ=1
ℓ
p−1
(here we set m = ℓ + 2
). Hence,
(p−1)/2 p−1
X 4ℓ X 4m
S3 − 3S2 ≡ − − = −S4 (mod p).
ℓ=1
ℓ m
m=(p+1)/2
73
N8 Number Theory – solutions 52nd IMO 2011
N8
Let k be a positive integer and set n = 2k + 1. Prove that n is a prime number if and only if
the following holds: there is a permutation a1 , . . . , an−1 of the numbers 1, 2, . . . , n − 1 and a
sequence of integers g1 , g2 , . . . , gn−1 such that n divides giai − ai+1 for every i ∈ {1, 2, . . . , n − 1},
where we set an = a1 .
Solution. Let N = {1, 2, . . . , n − 1}. For a, b ∈ N, we say that b follows a if there exists an
integer g such that b ≡ g a (mod n) and denote this property as a → b. This way we have a
directed graph with N as set of vertices. If a1 , . . . , an−1 is a permutation of 1, 2, . . . , n − 1 such
that a1 → a2 → . . . → an−1 → a1 then this is a Hamiltonian cycle in the graph.
Step I. First consider the case when n is composite. Let n = pα1 1 . . . pαs s be its prime factoriza-
tion. All primes pi are odd.
Suppose that αi > 1 for some i. For all integers a, g with a ≥ 2, we have g a 6≡ pi (mod p2i ),
because g a is either divisible by p2i or it is not divisible by pi . It follows that in any Hamiltonian
cycle pi comes immediately after 1. The same argument shows that 2pi also should come
immediately after 1, which is impossible. Hence, there is no Hamiltonian cycle in the graph.
Now suppose that n is square-free. We have n = p1 p2 . . . ps > 9 and s ≥ 2. Assume that there
exists a Hamiltonian cycle. There are n−1 2
even numbers in this cycle, and each number which
follows one of them should be a quadratic residue modulo n. So, there should be at least n−1 2
pi +1
nonzero quadratic residues modulo n. On the other hand, for each pi there exist exactly 2
quadratic residues modulo pi ; by the Chinese Remainder Theorem, the number of quadratic
residues modulo n is exactly p12+1 · p22+1 · . . . · ps2+1 , including 0. Then we have a contradiction
by s
p1 + 1 p2 + 1 ps + 1 2p1 2p2 2ps 2 4n n−1
· ·...· ≤ · ·...· = n≤ < .
2 2 2 3 3 3 3 9 2
This proves the “if”-part of the problem.
Step II. Now suppose that n is prime. For any a ∈ N, denote by ν2 (a) the exponent of 2 in
the prime factorization of a, and let µ(a) = max{t ∈ [0, k] | 2t → a}.
74
52nd IMO 2011 Number Theory – solutions N8
proved.
Ai = {a ∈ N | ν2 (a) = i},
Bi = {a ∈ N | µ(a) = i},
and Ci = {a ∈ N | µ(a) ≥ i} = Bi ∪ Bi+1 ∪ . . . ∪ Bk .
We claim that |Ai | = |Bi | for all 0 ≤ i ≤ k. Obviously we have |Ai | = 2k−i−1 for all i =
0, . . . , k − 1, and |Ak | = 1. Now we determine |Ci |. We have |C0 | = n − 1 and by Fermat’s
theorem we also have Ck = {1}, so |Ck | = 1. Next, notice that Ci+1 = {x2 mod n | x ∈ Ci }.
For every a ∈ N, the relation x2 ≡ a (mod n) has at most two solutions in N. Therefore we
have 2|Ci+1| ≤ |Ci |, with the equality achieved only if for every y ∈ Ci+1 , there exist distinct
elements x, x′ ∈ Ci such that x2 ≡ x′2 ≡ y (mod n) (this implies x + x′ = n). Now, since
2k |Ck | = |C0|, we obtain that this equality should be achieved in each step. Hence |Ci | = 2k−i
for 0 ≤ i ≤ k, and therefore |Bi | = 2k−i−1 for 0 ≤ i ≤ k − 1 and |Bk | = 1.
From the previous arguments we can see that for each z ∈ Ci (0 ≤ i < k) the equation x2 ≡ z 2
(mod n) has two solutions in Ci , so we have n − z ∈ Ci . Hence, for each i = 0, 1, . . . , k − 1,
exactly half of the elements of Ci are odd. The same statement is valid for Bi = Ci \ Ci+1
for 0 ≤ i ≤ k − 2. In particular, each such Bi contains an odd number. Note that Bk = {1}
also contains an odd number, and Bk−1 = {2k } since Ck−1 consists of the two square roots of 1
modulo n.
Step III. Now we construct a Hamiltonian cycle in the graph. First, for each i with 0 ≤ i ≤ k,
connect the elements of Ai to the elements of Bi by means of an arbitrary bijection. After
performing this for every i, we obtain a subgraph with all vertices having in-degree 1 and out-
degree 1, so the subgraph is a disjoint union of cycles. If there is a unique cycle, we are done.
Otherwise, we modify the subgraph in such a way that the previous property is preserved and
the number of cycles decreases; after a finite number of steps we arrive at a single cycle.
For every cycle C, let λ(C) = minc∈C ν2 (c). Consider a cycle C for which λ(C) is maximal. If
λ(C) = 0, then for any other cycle C ′ we have λ(C ′ ) = 0. Take two arbitrary vertices a ∈ C
and a′ ∈ C ′ such that ν2 (a) = ν2 (a′ ) = 0; let their direct successors be b and b′ , respectively.
Then we can unify C and C ′ to a single cycle by replacing the edges a → b and a′ → b′ by
a → b′ and a′ → b.
Now suppose that λ = λ(C) ≥ 1; let a ∈ C ∩ Aλ . If there exists some a′ ∈ Aλ \ C, then a′ lies
in another cycle C ′ and we can merge the two cycles in exactly the same way as above. So, the
only remaining case is Aλ ⊂ C. Since the edges from Aλ lead to Bλ , we get also Bλ ⊂ C. If
λ 6= k − 1 then Bλ contains an odd number; this contradicts the assumption λ(C) > 0. Finally,
if λ = k − 1, then C contains 2k−1 which is the only element of Ak−1 . Since Bk−1 = {2k } = Ak
and Bk = {1}, the cycle C contains the path 2k−1 → 2k → 1 and it contains an odd number
again. This completes the proof of the “only if”-part of the problem.
75
N8 Number Theory – solutions 52nd IMO 2011
Comment 1. The lemma and the fact |Ai | = |Bi | together show that for every edge a → b of the
Hamiltonian cycle, ν2 (a) = µ(b) must hold. After this observation, the Hamiltonian cycle can be built
in many ways. For instance, it is possible to select edges from Ai to Bi for i = k, k − 1, . . . , 1 in such
a way that they form disjoint paths; at the end all these paths will have odd endpoints. In the final
step, the paths can be closed to form a unique cycle.
Comment 2. Step II is an easy consequence of some basic facts about the multiplicative group modulo
the prime n = 2k + 1. The Lemma follows by noting that this group has order 2k , so the a-th powers
are exactly the 2ν2 (a) -th powers. Using the existence of a primitive root g modulo n one sees that the
map from {1, 2, . . . , n − 1} to itself that sends a to g a mod n is a bijection that sends Ai to Bi for each
i ∈ {0, . . . , k}.
76
Shortlisted Problems with Solutions
Contributing Countries
The Organizing Committee and the Problem Selection Committee of IMO 2012 thank the
following 40 countries for contributing 136 problem proposals:
Martı́n Avendaño
Carlos di Fiore
Géza Kós
Svetoslav Savchev
4
Algebra
A1. Find all the functions f : Z → Z such that
f (a)2 + f (b)2 + f (c)2 = 2f (a)f (b) + 2f (b)f (c) + 2f (c)f (a)
b) Does there exist a partition of Q into three non-empty subsets A, B, C such that the sets
A + B, B + C, C + A are disjoint?
A4. Let f and g be two nonzero polynomials with integer coefficients and deg f > deg g.
Suppose that for infinitely many primes p the polynomial pf + g has a rational root. Prove
that f has a rational root.
and f (−1) 6= 0.
A6. Let f : N → N be a function, and let f m be f applied m times. Suppose that for
every n ∈ N there exists a k ∈ N such that f 2k (n) = n + k, and let kn be the smallest such k.
Prove that the sequence k1 , k2 , . . . is unbounded.
A7. We say that a function f : Rk → R is a metapolynomial if, for some positive integers m
and n, it can be represented in the form
where Pi,j are multivariate polynomials. Prove that the product of two metapolynomials is also
a metapolynomial.
5
Combinatorics
C1. Several positive integers are written in a row. Iteratively, Alice chooses two adjacent
numbers x and y such that x > y and x is to the left of y, and replaces the pair (x, y) by either
(y + 1, x) or (x − 1, x). Prove that she can perform only finitely many such iterations.
C2. Let n ≥ 1 be an integer. What is the maximum number of disjoint pairs of elements of the
set {1, 2, . . . , n} such that the sums of the different pairs are different integers not exceeding n?
C3. In a 999 × 999 square table some cells are white and the remaining ones are red. Let T
be the number of triples (C1 , C2 , C3) of cells, the first two in the same row and the last two in
the same column, with C1 and C3 white and C2 red. Find the maximum value T can attain.
C4. Players A and B play a game with N ≥ 2012 coins and 2012 boxes arranged around a
circle. Initially A distributes the coins among the boxes so that there is at least 1 coin in each
box. Then the two of them make moves in the order B, A, B, A, . . . by the following rules:
• On every move of his B passes 1 coin from every box to an adjacent box.
• On every move of hers A chooses several coins that were not involved in B’s previous
move and are in different boxes. She passes every chosen coin to an adjacent box.
Player A’s goal is to ensure at least 1 coin in each box after every move of hers, regardless of
how B plays and how many moves are made. Find the least N that enables her to succeed.
C5. The columns and the rows of a 3n × 3n square board are numbered 1, 2, . . . , 3n. Every
square (x, y) with 1 ≤ x, y ≤ 3n is colored asparagus, byzantium or citrine according as the
modulo 3 remainder of x + y is 0, 1 or 2 respectively. One token colored asparagus, byzantium
or citrine is placed on each square, so that there are 3n2 tokens of each color.
Suppose that one can permute the tokens so that each token is moved to a distance of
at most d from its original position, each asparagus token replaces a byzantium token, each
byzantium token replaces a citrine token, and each citrine token replaces an asparagus token.
Prove that it is possible to permute the tokens so that each token is moved to a distance of at
most d + 2 from its original position, and each square contains a token with the same color as
the square.
C6. Let k and n be fixed positive integers. In the liar’s guessing game, Amy chooses integers
x and N with 1 ≤ x ≤ N. She tells Ben what N is, but not what x is. Ben may then repeatedly
ask Amy whether x ∈ S for arbitrary sets S of integers. Amy will always answer with yes or no,
but she might lie. The only restriction is that she can lie at most k times in a row. After he
has asked as many questions as he wants, Ben must specify a set of at most n positive integers.
If x is in this set he wins; otherwise, he loses. Prove that:
b) For sufficiently large k there exist n ≥ 1.99k such that Ben cannot guarantee a win.
C7. There are given 2500 points on a circle labeled 1, 2, . . . , 2500 in some order. Prove that
one can choose 100 pairwise disjoint chords joining some of these points so that the 100 sums
of the pairs of numbers at the endpoints of the chosen chords are equal.
6
Geometry
G1. In the triangle ABC the point J is the center of the excircle opposite to A. This excircle
is tangent to the side BC at M, and to the lines AB and AC at K and L respectively. The
lines LM and BJ meet at F , and the lines KM and CJ meet at G. Let S be the point of
intersection of the lines AF and BC, and let T be the point of intersection of the lines AG
and BC. Prove that M is the midpoint of ST .
G2. Let ABCD be a cyclic quadrilateral whose diagonals AC and BD meet at E. The
extensions of the sides AD and BC beyond A and B meet at F . Let G be the point such that
ECGD is a parallelogram, and let H be the image of E under reflection in AD. Prove that
D, H, F , G are concyclic.
G3. In an acute triangle ABC the points D, E and F are the feet of the altitudes through A,
B and C respectively. The incenters of the triangles AEF and BDF are I1 and I2 respectively;
the circumcenters of the triangles ACI1 and BCI2 are O1 and O2 respectively. Prove that I1 I2
and O1 O2 are parallel.
G4. Let ABC be a triangle with AB 6= AC and circumcenter O. The bisector of ∠BAC
intersects BC at D. Let E be the reflection of D with respect to the midpoint of BC. The lines
through D and E perpendicular to BC intersect the lines AO and AD at X and Y respectively.
Prove that the quadrilateral BXCY is cyclic.
G5. Let ABC be a triangle with ∠BCA = 90◦ , and let C0 be the foot of the altitude
from C. Choose a point X in the interior of the segment CC0 , and let K, L be the points on
the segments AX, BX for which BK = BC and AL = AC respectively. Denote by M the
intersection of AL and BK. Show that MK = ML.
G6. Let ABC be a triangle with circumcenter O and incenter I. The points D, E and F on
the sides BC, CA and AB respectively are such that BD + BF = CA and CD + CE = AB.
The circumcircles of the triangles BF D and CDE intersect at P 6= D. Prove that OP = OI.
G7. Let ABCD be a convex quadrilateral with non-parallel sides BC and AD. Assume
that there is a point E on the side BC such that the quadrilaterals ABED and AECD are
circumscribed. Prove that there is a point F on the side AD such that the quadrilaterals
ABCF and BCDF are circumscribed if and only if AB is parallel to CD.
G8. Let ABC be a triangle with circumcircle ω and ℓ a line without common points with ω.
Denote by P the foot of the perpendicular from the center of ω to ℓ. The side-lines BC, CA, AB
intersect ℓ at the points X, Y, Z different from P . Prove that the circumcircles of the triangles
AXP, BY P and CZP have a common point different from P or are mutually tangent at P .
7
Number Theory
N1. Call admissible a set A of integers that has the following property:
If x, y ∈ A (possibly x = y) then x2 + kxy + y 2 ∈ A for every integer k.
Determine all pairs m, n of nonzero integers such that the only admissible set containing both m
and n is the set of all integers.
N2. Find all triples (x, y, z) of positive integers such that x ≤ y ≤ z and
x3 (y 3 + z 3 ) = 2012(xyz + 2).
m m
N3. Determine all integers m ≥ 2 such that every n with 3
≤n≤ 2
divides the binomial
n
coefficient m−2n
.
N4. An integer a is called friendly if the equation (m2 + n)(n2 + m) = a(m − n)3 has a
solution over the positive integers.
a) Prove that there are at least 500 friendly integers in the set {1, 2, . . . , 2012}.
N7. Find all n ∈ N for which there exist nonnegative integers a1 , a2 , . . . , an such that
1 1 1 1 2 n
a
+ a
+ · · · + a
= a
+ a
+ · · · + = 1.
21 22 2n 31 32 3an
N8. Prove that for every prime p > 100 and every integer r there exist two integers a and b
such that p divides a2 + b5 − r.
8
Algebra
A1. Find all the functions f : Z → Z such that
f (a)2 + f (b)2 + f (c)2 = 2f (a)f (b) + 2f (b)f (c) + 2f (c)f (a)
f (0) = 0. (1)
The substitution b = −a and c = 0 gives ((f (a) − f (−a))2 = 0. Hence f is an even function:
Now set b = a and c = −2a to obtain 2f (a)2 + f (2a)2 = 2f (a)2 + 4f (a)f (2a). Hence
If f (r) = 0 for some r ≥ 1 then the substitution b = r and c = −a−r gives (f (a+r)−f (a))2 = 0.
So f is periodic with period r, i. e.
In particular, if f (1) = 0 then f is constant, thus f (a) = 0 for all a ∈ Z. This function clearly
satisfies the functional equation. For the rest of the analysis, we assume f (1) = k 6= 0.
By (3) we have f (2) = 0 or f (2) = 4k. If f (2) = 0 then f is periodic of period 2, thus
f (even) = 0 and f (odd) = k. This function is a solution for every k. We postpone the
verification; for the sequel assume f (2) = 4k 6= 0.
By (3) again, we have f (4) = 0 or f (4) = 16k. In the first case f is periodic of period 4, and
f (3) = f (−1) = f (1) = k, so we have f (4n) = 0, f (4n+1) = f (4n+3) = k, and f (4n+2) = 4k
for all n ∈ Z. This function is a solution too, which we justify later. For the rest of the analysis,
we assume f (4) = 16k 6= 0.
We show now that f (3) = 9k. In order to do so, we need two substitutions:
Therefore f (3) = 9k, as claimed. Now we prove inductively that the only remaining function is
f (x) = kx2 , x ∈ Z. We proved this for x = 0, 1, 2, 3, 4. Assume that n ≥ 4 and that f (x) = kx2
holds for all integers x ∈ [0, n]. Then the substitutions a = n, b = 1, c = −n − 1 and a = n − 1,
b = 2, c = −n − 1 lead respectively to
Since k(n − 1)2 6= k(n − 3)2 for n 6= 2, the only possibility is f (n + 1) = k(n + 1)2 . This
completes the induction, so f (x) = kx2 for all x ≥ 0. The same expression is valid for negative
values of x since f is even. To verify that f (x) = kx2 is actually a solution, we need to check
the identity a4 + b4 + (a + b)4 = 2a2 b2 + 2a2 (a + b)2 + 2b2 (a + b)2 , which follows directly by
expanding both sides.
9
Therefore the only possible solutions of the functional equation are the constant function
f1 (x) = 0 and the following functions:
0 x ≡ 0 (mod 4)
2 0 x even
f2 (x) = kx f3 (x) = f4 (x) = k x ≡ 1 (mod 2)
k x odd
4k x ≡ 2 (mod 4)
for any non-zero integer k. The verification that they are indeed solutions was done for the
first two. For f3 note that if a + b + c = 0 then either a, b, c are all even, in which case
f (a) = f (b) = f (c) = 0, or one of them is even and the other two are odd, so both sides of
the equation equal 2k 2 . For f4 we use similar parity considerations and the symmetry of the
equation, which reduces the verification to the triples (0, k, k), (4k, k, k), (0, 0, 0), (0, 4k, 4k).
They all satisfy the equation.
Comment. We used several times the same fact: For any a, b ∈ Z the functional equation is a
quadratic equation in f (a + b) whose coefficients depend on f (a) and f (b):
Its discriminant is 16f (a)f (b). Since this value has to be non-negative for any a, b ∈ Z, we conclude
that either f or −f is always non-negative. Also, if f is a solution of the functional equation, then
−f is also a solution. Therefore we can assume f (x) ≥ 0 for all x ∈ Z. Now, the two solutions of the
quadratic equation are
2 p 2
p p p
f (a + b) ∈ f (a) + f (b) , f (a) − f (b) for all a, b ∈ Z.
The computation of f (3) from f (1), f (2) and f (4) that we did above follows immediately by setting
(a, b) = (1, 2) and (a, b) = (1, −4). The inductive step, where f (n + 1) is derived from f (n), f (n − 1),
f (2) and f (1), follows immediately using (a, b) = (n, 1) and (a, b) = (n − 1, 2).
10
b) Does there exist a partition of Q into three non-empty subsets A, B, C such that the sets
A + B, B + C, C + A are disjoint?
Here X + Y denotes the set {x + y | x ∈ X, y ∈ Y }, for X, Y ⊆ Z and X, Y ⊆ Q.
b) The answer is no. Suppose that Q can be partitioned into non-empty subsets A, B, C as
stated. Note that for all a ∈ A, b ∈ B, c ∈ C one has
a + b − c ∈ C, b + c − a ∈ A, c + a − b ∈ B. (1)
a + a′ − b = a + (a′ + c − b) − c ∈ C.
B + C = A + A, C + A = B + B, A + B = C + C.
Solution 2. We prove that the example for Z from the first solution is unique, and then use
this fact to solve part b).
Let Z = A ∪ B ∪ C be a partition of Z with A, B, C 6= ∅ and A + B, B + C, C + A disjoint.
We need the relations (1) which clearly hold for Z. Fix two consecutive integers from different
sets, say b ∈ B and c = b + 1 ∈ C. For every a ∈ A we have, in view of (1), a − 1 = a + b − c ∈ C
and a + 1 = a + c − b ∈ B. So every a ∈ A is preceded by a number from C and followed by a
number from B.
In particular there are pairs of the form c, c + 1 with c ∈ C, c + 1 ∈ A. For such a pair and
any b ∈ B analogous reasoning shows that each b ∈ B is preceded by a number from A and
followed by a number from C. There are also pairs b, b − 1 with b ∈ B, b − 1 ∈ A. We use them
in a similar way to prove that each c ∈ C is preceded by a number from B and followed by a
number from A.
By putting the observations together we infer that A, B, C are the three congruence classes
modulo 3. Observe that all multiples of 3 are in the set of the partition that contains 0.
11
Now we turn to part b). Suppose that there is a partition of Q with the given properties.
Choose three rationals ri = pi /qi from the three sets A, B, C, i = 1, 2, 3, and set N = 3q1 q2 q3 .
Let S ⊂ Q be the set of fractions with denominators N (irreducible or not). It is obtained
through multiplication of every integer by the constant 1/N, hence closed under sums and
differences. Moreover, if we identify each k ∈ Z with k/N ∈ S then S is essentially the set Z
with respect to addition. The numbers ri belong to S because
3p1 q2 q3 3p2 q3 q1 3p3 q1 q2
r1 = , r2 = , r3 = .
N N N
The partition Q = A ∪ B ∪ C of Q induces a partition S = A′ ∪ B ′ ∪ C ′ of S, with A′ = A ∩ S,
B ′ = B ∩ S, C ′ = C ∩ S. Clearly A′ + B ′ , B ′ + C ′ , C ′ + A′ are disjoint, so this partition has
the properties we consider.
By the uniqueness of the example for Z the sets A′ , B ′ , C ′ are the congruence classes mod-
ulo 3, multiplied by 1/N. Also all multiples of 3/N are in the same set, A′ , B ′ or C ′ . This holds
for r1 , r2 , r3 in particular as they are all multiples of 3/N. However r1 , r2 , r3 are in different sets
A′ , B ′ , C ′ since they were chosen from different sets A, B, C. The contradiction ends the proof.
Comment. The uniqueness of the example for Z can also be deduced from the argument in the first
solution.
12
x2 x3 x1
Solution. The substitution a2 = , a3 = , . . . , an = transforms the original problem
x1 x2 xn−1
into the inequality
for all x1 , . . . , xn−1 > 0. To prove this, we use the AM-GM inequality for each factor of the
left-hand side as follows:
(x1 + x2 )2 ≥ 22 x1 x2
x2
3 2
(x2 + x3 )3 = 2 2
+ x3 ≥ 33 x22 x3
x3
4 3
(x3 + x4 )4 = 3 3
+ x4 ≥ 44 x33 x4
.. .. ..
. . .
xn−1
n
n−1 n−1
(xn−1 + x1 )n = (n − 1) n−1
+ x1 ≥ nn xn−1 x1 .
Multiplying these inequalities together gives (*), with inequality sign ≥ instead of >. However
for the equality to occur it is necessary that x1 = x2 , x2 = 2x3 , . . . , xn−1 = (n − 1)x1 , implying
x1 = (n − 1)!x1 . This is impossible since x1 > 0 and n ≥ 3. Therefore the inequality is strict.
Comment. One can avoid the substitution ai = xi /xi−1 . Apply the weighted AM-GM inequality to
each factor (1 + ak )k , with the same weights like above, to obtain
k
k 1 kk
(1 + ak ) = (k − 1) + ak ≥ ak .
k−1 (k − 1)k−1
(1 + a2 )2 (1 + a3 )3 · · · (1 + an )n ≥ nn a2 a3 · · · an = nn .
The same argument as in the proof above shows that the equality cannot be attained.
13
A4. Let f and g be two nonzero polynomials with integer coefficients and deg f > deg g.
Suppose that for infinitely many primes p the polynomial pf + g has a rational root. Prove
that f has a rational root.
Solution 1. Since deg f > deg g, we have |g(x)/f (x)| < 1 for sufficiently large x; more
precisely, there is a real number R such that |g(x)/f (x)| < 1 for all x with |x| > R. Then for
all such x and all primes p we have
pf (x) + g(x) ≥ f (x) p − |g(x)| > 0.
|f (x)|
Hence all real roots of the polynomials pf + g lie in the interval [−R, R].
Let f (x) = an xn + an−1 xn−1 + · · · + a0 and g(x) = bm xm + bm−1 xm−1 + · · · + b0 where
n > m, an 6= 0 and bm 6= 0. Upon replacing f (x) and g(x) by ann−1 f (x/an ) and ann−1 g(x/an )
respectively, we reduce the problem to the case an = 1. In other words one can assume that f
is monic. Then the leading coefficient of pf + g is p, and if r = u/v is a rational root of pf + g
with (u, v) = 1 and v > 0, then either v = 1 or v = p.
First consider the case when v = 1 infinitely many times. If v = 1 then |u| ≤ R, so there
are only finitely many possibilities for the integer u. Therefore there exist distinct primes p
and q for which we have the same value of u. Then the polynomials pf + g and qf + g share
this root, implying f (u) = g(u) = 0. So in this case f and g have an integer root in common.
Now suppose that v = p infinitely many times. By comparing the exponent of p in the
denominators of pf (u/p) and g(u/p) we get m = n − 1 and pf (u/p) + g(u/p) = 0 reduces to
an equation of the form
n n−1 n n−1 n−2 n−1
u + an−1 pu + . . . + a0 p + bn−1 u + bn−2 pu + . . . + b0 p = 0.
The equation above implies that un + bn−1 un−1 is divisible by p and hence, since (u, p) = 1,
we have u + bn−1 = pk with some integer k. On the other hand all roots of pf + g lie in the
interval [−R, R], so that
Therefore the integer k can attain only finitely many values. Hence there exists an integer k
such that the number pk−bpn−1 = k − bn−1
p
is a root of pf + g for infinitely many primes p. For
these primes we have
1 1 1
f k − bn−1 + g k − bn−1 = 0.
p p p
So the equation
f (k − bn−1 x) + xg (k − bn−1 x) = 0 (1)
has infinitely many solutions of the form x = 1/p. Since the left-hand side is a polynomial, this
implies that (1) is a polynomial identity, so it holds for all real x. In particular, by substituting
x = 0 in (1) we get f (k) = 0. Thus the integer k is a root of f .
In summary the monic polynomial f obtained after the initial reduction always has an
integer root. Therefore the original polynomial f has a rational root.
14
Solution 2. Analogously to the first solution, there exists a real number R such that the
complex roots of all polynomials of the form pf + g lie in the disk |z| ≤ R.
For each prime p such that pf + g has a rational root, by Gauss’ lemma pf + g is the
product of two integer polynomials, one with degree 1 and the other with degree deg f − 1.
Since p is a prime, the leading coefficient of one of these factors divides the leading coefficient
of f . Denote that factor by hp .
By narrowing the set of the primes used we can assume that all polynomials hp have the
same degree and the same leading coefficient. Their complex roots lie in the disk |z| ≤ R, hence
Vieta’s formulae imply that all coefficients of all polynomials hp form a bounded set. Since
these coefficients are integers, there are only finitely many possible polynomials hp . Hence there
is a polynomial h such that hp = h for infinitely many primes p.
Finally, if p and q are distinct primes with hp = hq = h then h divides (p − q)f . Since
deg h = 1 or deg h = deg f − 1, in both cases f has a rational root.
Comment. Clearly the polynomial h is a common factor of f and g. If deg h = 1 then f and g share a
rational root. Otherwise deg h = deg f − 1 forces deg g = deg f − 1 and g divides f over the rationals.
Solution 3. Like in the first solution, there is a real number R such that the real roots of all
polynomials of the form pf + g lie in the interval [−R, R].
Let p1 < p2 < · · · be an infinite sequence of primes so that for every index k the polynomial
pk f + g has a rational root rk . The sequence r1 , r2 , . . . is bounded, so it has a convergent
subsequence rk1 , rk2 , . . .. Now replace the sequences (p1 , p2 , . . . ) and (r1 , r2 , . . . ) by (pk1 , pk2 , . . .)
and (rk1 , rk2 , . . .); after this we can assume that the sequence r1 , r2 , . . . is convergent. Let
α = lim rk . We show that α is a rational root of f .
k→∞
Over the interval [−R, R], the polynomial g is bounded, |g(x)| ≤ M with some fixed M.
Therefore
pk f (rk ) + g(rk ) |g(rk )| M
|f (rk )| = f (rk ) − = ≤ → 0,
pk pk pk
and
f (α) = f lim rk = lim f (rk ) = 0.
k→∞ k→∞
So α is a root of f indeed.
Now let uk , vk be relative prime integers for which rk = uvkk . Let a be the leading coefficient
of f , let b = f (0) and c = g(0) be the constant terms of f and g, respectively. The leading
coefficient of the polynomial pk f + g is pk a, its constant term is pk b + c. So vk divides pk a and
uk divides pk b + c. Let pk b + c = uk ek (if pk b + c = uk = 0 then let ek = 1).
We prove that α is rational by using the following fact. Let (pn ) and (qn ) be sequences of
integers such that the sequence (pn /qn ) converges. If (pn ) or (qn ) is bounded then lim(pn /qn ) is
rational .
Case 1: There is an infinite subsequence (kn ) of indices such that vkn divides a. Then (vkn )
is bounded, so α = limn→∞ (ukn /vkn ) is rational.
Case 2: There is an infinite subsequence (kn ) of indices such that vkn does not divide a.
For such indices we have vkn = pkn dkn where dkn is a divisor of a. Then
u kn pk n b + c b c b
α = lim = lim = lim + lim = lim .
n→∞ vkn n→∞ pkn dkn ekn n→∞ dkn ekn n→∞ pkn dkn ekn n→∞ dkn ekn
Comment. There are functions that satisfy the given equation but vanish at −1, for instance the
constant function 0 and f (x) = x2 − 1, x ∈ R.
16
A6. Let f : N → N be a function, and let f m be f applied m times. Suppose that for
every n ∈ N there exists a k ∈ N such that f 2k (n) = n + k, and let kn be the smallest such k.
Prove that the sequence k1 , k2 , . . . is unbounded.
Observe that S is unbounded because for every number n in S there exists a k > 0 such
that f 2k (n) = n + k is in S. Clearly f maps S into itself; moreover f is injective on S. Indeed
if f i (1) = f j (1) with i 6= j then the values f m (1) start repeating periodically from some point
on, and S would be finite.
Define g : S → S by g(n) = f 2kn (n) = n + kn . We prove that g is injective too. Suppose
that g(a) = g(b) with a < b. Then a + ka = f 2ka (a) = f 2kb (b) = b + kb implies ka > kb . So,
since f is injective on S, we obtain
Hence
n − nt n − nt
f n (1) = f nt (1) + =t+ . (1)
2 2
Now we show that T is infinite. We argue by contradiction. Suppose that there are only
finitely many chains Ct1 , . . . , Ctr , starting at t1 < · · · < tr . Fix N. If f n (1) with 1 ≤ n ≤ N
is in Ct then f n (1) = t + n−n 2
t
≤ tr + N2 by (1). But then the N + 1 distinct natural numbers
1, f (1), . . . , f N (1) are all less than tr + N2 and hence N + 1 ≤ tr + N2 . This is a contradiction if
N is sufficiently large, and hence T is infinite.
To complete the argument, choose any k in N and consider the k + 1 chains starting at the
first k + 1 numbers in T . Let t be the greatest one among these numbers. Then each of the
chains in question contains a number not exceeding t, and at least one of them does not contain
any number among t + 1, . . . , t + k. So there is a number n in this chain such that g(n) − n > k,
i. e. kn > k. In conclusion k1 , k2, . . . is unbounded.
17
A7. We say that a function f : Rk → R is a metapolynomial if, for some positive integers m
and n, it can be represented in the form
where Pi,j are multivariate polynomials. Prove that the product of two metapolynomials is also
a metapolynomial.
Solution. We use the notation f (x) = f (x1 , . . . , xk ) for x = (x1 , . . . , xk ) and [m] = {1, 2, . . . , m}.
Observe that if a metapolynomial f (x) admits a representation like the one in the statement
for certain positive integers m and n, then they can be replaced by any m′ ≥ m and n′ ≥ n. For
instance, if we want to replace m by m + 1 then it is enough to define Pm+1,j (x) = Pm,j (x) and
note that repeating elements of a set do not change its maximum nor its minimum. So one can
assume that any two metapolynomials are defined with the same m and n. We reserve letters
P and Q for polynomials, so every function called P, Pi,j , Q, Qi,j , . . . is a polynomial function.
We start with a lemma that is useful to change expressions of the form min max fi,j to ones
of the form max min gi,j .
Lemma. Let {ai,j } be real numbers, for all i ∈ [m] and j ∈ [n]. Then
where the max in the right-hand side is over all vectors (j1 , . . . , jm ) with j1 , . . . , jm ∈ [n].
Proof. We can assume for all i that ai,n = max{ai,1 , . . . , ai,n } and am,n = min{a1,n , . . . , am,n }.
The left-hand side is = am,n and hence we need to prove the same for the right-hand side.
If (j1 , j2 , . . . , jm ) = (n, n, . . . , n) then min{a1,j1 , . . . , am,jm } = min{a1,n , . . . , am,n } = am,n which
implies that the right-hand side is ≥ am,n . It remains to prove the opposite inequality and
this is equivalent to min{a1,j1 , . . . , am,jm } ≤ am,n for all possible (j1 , j2 , . . . , jm ). This is true
because min{a1,j1 , . . . , am,jm } ≤ am,jm ≤ am,n .
We need to show that the family M of metapolynomials is closed under multiplication, but
it turns out easier to prove more: that it is also closed under addition, maxima and minima.
First we prove the assertions about the maxima and the minima. If f1 , . . . , fr are metapoly-
nomials, assume them defined with the same m and n. Then
1 r s
f = max{f1 , . . . , fr } = max{max min Pi,j , . . . , max min Pi,j }= max min Pi,j .
i∈[m] j∈[n] i∈[m] j∈[n] s∈[r],i∈[m] j∈[n]
It follows that f = max{f1 , . . . , fr } is a metapolynomial. The same argument works for the
minima, but first we have to replace min max by max min, and this is done via the lemma.
Another property we need is that if f = max min Pi,j is a metapolynomial then so is −f .
Indeed, −f = min(− min Pi,j ) = min max Pi,j .
To prove M is closed under addition let f = max min Pi,j and g = max min Qi,j . Then
f (x) + g(x) = max min Pi,j (x) + max min Qi,j (x)
i∈[m] j∈[n] i∈[m] j∈[n]
= max (min Pi1 ,j (x) + min Qi2 ,j (x)) = max min Pi1 ,j1 (x) + Qi2 ,j2 (x) ,
i1 ,i2 ∈[m] j∈[n] j∈[n] i1 ,i2 ∈[m] j1 ,j2 ∈[n]
to the maximum of the product of the sets. We need to deal with the fact that a < b and c < d
do not imply ac < bd. However this is true for a, b, c, d ≥ 0.
In view of this we decompose each function f (x) into its positive part f + (x) = max{f (x), 0}
and its negative part f − (x) = max{0, −f (x)}. Note that f = f + − f − and f + , f − ∈ M if
f ∈ M. The whole problem reduces to the claim that if f and g are metapolynomials with
f, g ≥ 0 then f g it is also a metapolynomial.
Assuming this claim, consider arbitrary f, g ∈ M. We have
f g = (f + − f − )(g + − g − ) = f + g + − f + g − − f − g + + f − g − ,
Hence it suffices to check that P + Q+ ∈ M for any pair of polynomials P and Q. This reduces
to the identity
with u replaced by P (x) and v replaced by Q(x). The formula is proved by a case-by-case
analysis. If u ≤ 0 or v ≤ 0 then both sides equal 0. In case u, v ≥ 0, the right-hand side is
clearly ≤ uv. To prove the opposite inequality we use that uv equals
min{uv, u, v} if 0 ≤ u, v ≤ 1,
min{uv, uv 2, u2v} if 1 ≤ u, v,
min{uv, u, u2v} if 0 ≤ v ≤ 1 ≤ u,
min{uv, uv 2, v} if 0 ≤ u ≤ 1 ≤ v.
Comment. The case k = 1 is simpler and can be solved by proving that a function f : R → R is a
metapolynomial if and only if it is a piecewise polinomial (and continuos) function.
It is enough to prove that all such functions are metapolynomials, and this easily reduces to the
following case. Given a polynomial P (x) with P (0) = 0, the function f defined by f (x) = P (x) for
x ≥ 0 and 0 otherwise is a metapolynomial. For this last claim, it suffices to prove that (x+ )n is a
metapolynomial, and this follows from the formula (x+ )n = max{0, min{xn−1 , xn }, min{xn , xn+1 }}.
19
Combinatorics
C1. Several positive integers are written in a row. Iteratively, Alice chooses two adjacent
numbers x and y such that x > y and x is to the left of y, and replaces the pair (x, y) by either
(y + 1, x) or (x − 1, x). Prove that she can perform only finitely many such iterations.
Solution 1. Note first that the allowed operation does not change the maximum M of the
initial sequence. Let a1 , a2 , . . . , an be the numbers obtained at some point of the process.
Consider the sum
S = a1 + 2a2 + · · · + nan .
We claim that S increases by a positive integer amount with every operation. Let the operation
replace the pair (ai , ai+1 ) by a pair (c, ai ), where ai > ai+1 and c = ai+1 +1 or c = ai −1. Then the
new and the old value of S differ by d = (ic+(i+1)ai )−(iai +(i+1)ai+1 ) = ai −ai+1 +i(c−ai+1 ).
The integer d is positive since ai − ai+1 ≥ 1 and c − ai+1 ≥ 0.
On the other hand S ≤ (1 + 2 + · · · + n)M as ai ≤ M for all i = 1, . . . , n. Since S increases
by at least 1 at each step and never exceeds the constant (1 + 2 + · · · + n)M, the process stops
after a finite number of iterations.
Solution 2. Like in the first solution note that the operations do not change the maximum M
of the initial sequence. Now consider the reverse lexicographical order for n-tuples of integers.
We say that (x1 , . . . , xn ) < (y1 , . . . , yn ) if xn < yn , or if xn = yn and xn−1 < yn−1 , or if xn = yn ,
xn−1 = yn−1 and xn−2 < yn−2 , etc. Each iteration creates a sequence that is greater than
the previous one with respect to this order, and no sequence occurs twice during the process.
On the other hand there are finitely many possible sequences because their terms are always
positive integers not exceeding M. Hence the process cannot continue forever.
Solution 3. Let the current numbers be a1 , a2 , . . . , an . Define the score si of ai as the number
of aj ’s that are less than ai . Call the sequence s1 , s2 , . . . , sn the score sequence of a1 , a2 , . . . , an .
Let us say that a sequence x1 , . . . , xn dominates a sequence y1 , . . . , yn if the first index i
with xi 6= yi is such that xi < yi . We show that after each operation the new score sequence
dominates the old one. Score sequences do not repeat, and there are finitely many possibilities
for them, no more than (n − 1)n . Hence the process will terminate.
Consider an operation that replaces (x, y) by (a, x), with a = y + 1 or a = x − 1. Suppose
that x was originally at position i. For each j < i the score sj does not increase with the
change because y ≤ a and x ≤ x. If sj decreases for some j < i then the new score sequence
dominates the old one. Assume that sj stays the same for all j < i and consider si . Since x > y
and y ≤ a ≤ x, we see that si decreases by at least 1. This concludes the proof.
Comment. All three proofs work if x and y are not necessarily adjacent, and if the pair (x, y) is
replaced by any pair (a, x), with a an integer satisfying
P y ≤ a ≤ x. There is nothing special about
the “weights” 1, 2, . . . , n in the definition of S =P ni=1 iai from the first solution. For any sequence
w1 < w2 < · · · < wn of positive integers, the sum ni=1 wi ai increases by at least 1 with each operation.
Consider the same problem, but letting Alice replace the pair (x, y) by (a, x), where a is any positive
integer less than x. The same conclusion holds in this version, i. e. the process stops eventually. The
solution using the reverse lexicographical order works without any change. The first solution would
require a special set of weights like wi = M i for i = 1, . . . , n.
Comment. The first and the second solutions provide upper bounds for the number of possible
operations, respectively of order M n2 and M n where M is the maximum of the original sequence.
The upper bound (n − 1)n in the third solution does not depend on M .
20
C2. Let n ≥ 1 be an integer. What is the maximum number of disjoint pairs of elements of the
set {1, 2, . . . , n} such that the sums of the different pairs are different integers not exceeding n?
Solution. Consider x such pairs in {1, 2, . . . , n}. The sum S of the 2x numbers in them is at
least 1+2+· · ·+2x since the pairs are disjoint. On the other hand S ≤ n+(n−1)+· · ·+(n−x+1)
because the sums of the pairs are different and do not exceed n. This gives the inequality
2x(2x + 1) x(x − 1)
≤ nx − ,
2 2
2n−1
which leads to x ≤ 2n−1
5
. Hence there are at most 5
pairs with the given properties.
2n−1
We show a construction
with exactly 5
pairs. First consider the case n = 5k + 3 with
2n−1
k ≥ 0, where 5
= 2k + 1. The pairs are displayed in the following table.
3k + 1 3k ··· 2k + 2 4k + 2 4k + 1 · · · 3k + 3 3k + 2
Pairs
2 4 ··· 2k 1 3 ··· 2k − 1 2k + 1
Sums 3k + 3 3k + 4 ··· 4k + 2 4k + 3 4k + 4 · · · 5k + 2 5k + 3
The 2k + 1 pairs involve all numbers from 1 to 4k + 2; their sums are all numbers from 3k + 3
2n−1 works for n = 5k + 4 and n = 5k + 5 with k ≥ 0. In these
to 5k + 3. The same construction
cases the required number 5
of pairs equals 2k + 1 again, and the numbers in the table
do not exceed 5k + 3. In the case n = 5k + 2 with k ≥ 0 one needs only 2k pairs. They can
be obtained by ignoring the last column of the table (thus removing 5k + 3). Finally, 2k pairs
are also needed for the case n = 5k + 1 with k ≥ 0. Now it suffices to ignore the last column
of the table and then subtract 1 from each number in the first row.
Comment. The construction above is not unique. For instance, the following table shows another
set of 2k + 1 pairs for the cases n = 5k + 3, n = 5k + 4, and n = 5k + 5.
The table for the case n = 5k + 2 would be the same, with the pair (k + 1, 4k + 2) removed. For the
case n = 5k + 1 remove the last column and subtract 2 from each number in the second row.
21
C3. In a 999 × 999 square table some cells are white and the remaining ones are red. Let T
be the number of triples (C1 , C2 , C3) of cells, the first two in the same row and the last two in
the same column, with C1 and C3 white and C2 red. Find the maximum value T can attain.
4
Solution. We prove that in an n × n square table there are at most 4n 27
such triples.
Let row i and column j contain ai and bj white cells respectively, and let R be the set of
red cells. For every red cell (i, j) there are ai bj admissible triples (C1 , C2 , C3 ) with C2 = (i, j),
therefore X
T = ai bj .
(i,j)∈R
This is because there are n − ai red cells in row i and n − bj red cells in column j. Now we
maximize the right-hand side.
By the AM-GM inequality we have
3
2 1 1 2n 4n3
(n − x)x = (2n − 2x) · x · x ≤ = ,
2 2 3 27
2n
with equality if and only if x = 3
. By putting everything together, we get
Comment. One can obtain a better preliminary estimate with the Cauchy-Schwarz inequality:
1 1 !1 n 1
2 2 n 2
X X X X 2 X
T = ai bj ≤ a2i · b2j = (n − ai )a2i · 2
(n − bj )bj .
(i,j)∈R (i,j)∈R (i,j)∈R i=1 j=1
C4. Players A and B play a game with N ≥ 2012 coins and 2012 boxes arranged around a
circle. Initially A distributes the coins among the boxes so that there is at least 1 coin in each
box. Then the two of them make moves in the order B, A, B, A, . . . by the following rules:
• On every move of his B passes 1 coin from every box to an adjacent box.
• On every move of hers A chooses several coins that were not involved in B’s previous
move and are in different boxes. She passes every chosen coin to an adjacent box.
Player A’s goal is to ensure at least 1 coin in each box after every move of hers, regardless of
how B plays and how many moves are made. Find the least N that enables her to succeed.
Solution. We argue for a general n ≥ 7 instead of 2012 and prove that the required minimum N
is 2n − 2. For n = 2012 this gives Nmin = 4022.
a) If N = 2n − 2 player A can achieve her goal. Let her start the game with a regular
distribution: n − 2 boxes with 2 coins and 2 boxes with 1 coin. Call the boxes of the two kinds
red and white respectively. We claim that on her first move A can achieve a regular distribution
again, regardless of B’s first move M. She acts according as the following situation S occurs
after M or not: The initial distribution contains a red box R with 2 white neighbors, and R
receives no coins from them on move M.
Suppose that S does not occur. Exactly one of the coins c1 and c2 in a given red box X
is involved in M, say c1 . If M passes c1 to the right neighbor of X, let A pass c2 to its left
neighbor, and vice versa. By doing so with all red boxes A performs a legal move M ′ . Thus
M and M ′ combined move the 2 coins of every red box in opposite directions. Hence after M
and M ′ are complete each neighbor of a red box X contains exactly 1 coin that was initially
in X. So each box with a red neighbor is non-empty after M ′ . If initially there is a box X
with 2 white neighbors (X is red and unique) then X receives a coin from at least one of them
on move M since S does not occur. Such a coin is not involved in M ′ , so X is also non-empty
after M ′ . Furthermore each box Y has given away its initial content after M and M ′ . A red
neighbor of Y adds 1 coin to it; a white neighbor adds at most 1 coin because it is not involved
in M ′ . Hence each box contains 1 or 2 coins after M ′ . Because N = 2n − 2, such a distribution
is regular.
Now let S occur after move M. Then A leaves untouched the exceptional red box R. With
all remaining red boxes she proceeds like in the previous case, thus making a legal move M ′′ .
Box R receives no coins from its neighbors on either move, so there is 1 coin in it after M ′′ .
Like above M and M ′′ combined pass exactly 1 coin from every red box different from R to
each of its neighbors. Every box except R has a red neighbor different from R, hence all boxes
are non-empty after M ′′ . Next, each box Y except R loses its initial content after M and M ′′ .
A red neighbor of Y adds at most 1 coin to it; a white neighbor also adds at most 1 coin as
it does not participate in M ′′ . Thus each box has 1 or 2 coins after M ′′ , and the obtained
distribution is regular.
Player A can apply the described strategy indefinitely, so N = 2n−2 enables her to succeed.
b) For N ≤ 2n − 3 player B can achieve an empty box after some move of A. Let α be a
set of ℓ consecutive boxes containing a total of N(α) coins. We call α an arc if ℓ ≤ n − 2 and
N(α) ≤ 2ℓ − 3. Note that ℓ ≥ 2 by the last condition. Moreover if both extremes of α are
non-empty boxes then N(α) ≥ 2, so that N(α) ≤ 2ℓ − 3 implies ℓ ≥ 3. Observe also that if
an extreme X of α has more than 1 coin then ignoring X yields a shorter arc. It follows that
every arc contains an arc whose extremes have at most 1 coin each.
Given a clockwise labeling 1, 2, . . . , n of the boxes, suppose that boxes 1, 2, . . . , ℓ form an
arc α, with ℓ ≤ n − 2 and N(α) ≤ 2ℓ − 3. Suppose also that all n ≥ 7 boxes are non-empty.
Then B can move so that an arc α′ with N(α′ ) < N(α) will appear after any response of A.
23
One may assume exactly 1 coin in boxes 1 and ℓ by a previous remark. Let B pass 1 coin
in counterclockwise direction from box 1 and box n, and in clockwise direction from each
remaining box. This leaves N(α) − 2 coins in the boxes of α. In addition, due to 3 ≤ ℓ ≤ n − 2,
box ℓ has exactly 1 coin c, the one received from box ℓ − 1.
Let player A’s next move M pass k ≤ 2 coins to boxes 1, 2, . . . , ℓ from the remaining ones.
Only boxes 1 and ℓ can receive such coins, at most 1 each. If k < 2 then after move M boxes
1, 2, . . . , ℓ form an arc α′ with N(α′ ) < N(α). If k = 2 then M adds a coin to box ℓ. Also
M does not move coin c from ℓ because c is involved in the previous move of B. In summary
boxes 1, 2, . . . , ℓ contain N(α) coins like before, so they form an arc. However there are 2 coins
now in the extreme ℓ of the arc. Ignore ℓ to obtain a shorter arc α′ with N(α′ ) < N(α).
Consider any initial distribution without empty boxes. Since N ≤ 2n − 3, there are at
least 3 boxes in it with exactly 1 coin. It follows from n ≥ 7 that some 2 of them are the
extremes of an arc α. Hence B can make the move described above, which leads to an arc α′
with N(α′ ) < N(α) after A’s response. If all boxes in the new distribution are non-empty he
can repeat the same, and so on. Because N(α) cannot decrease indefinitely, an empty box will
occur after some move of A.
24
C5. The columns and the rows of a 3n × 3n square board are numbered 1, 2, . . . , 3n. Every
square (x, y) with 1 ≤ x, y ≤ 3n is colored asparagus, byzantium or citrine according as the
modulo 3 remainder of x + y is 0, 1 or 2 respectively. One token colored asparagus, byzantium
or citrine is placed on each square, so that there are 3n2 tokens of each color.
Suppose that one can permute the tokens so that each token is moved to a distance of
at most d from its original position, each asparagus token replaces a byzantium token, each
byzantium token replaces a citrine token, and each citrine token replaces an asparagus token.
Prove that it is possible to permute the tokens so that each token is moved to a distance of at
most d + 2 from its original position, and each square contains a token with the same color as
the square.
Solution. Without loss of generality it suffices to prove that the A-tokens can be moved to
distinct A-squares in such a way that each A-token is moved to a distance at most d + 2 from
its original place. This means we need a perfect matching between the 3n2 A-squares and the
3n2 A-tokens such that the distance in each pair of the matching is at most d + 2.
To find the matching, we construct a bipartite graph. The A-squares will be the vertices in
one class of the graph; the vertices in the other class will be the A-tokens.
Split the board into 3 × 1 horizontal triminos; then each trimino contains exactly one A-
square. Take a permutation π of the tokens which moves A-tokens to B-tokens, B-tokens to
C-tokens, and C-tokens to A-tokens, in each case to a distance at most d. For each A-square S,
and for each A-token T , connect S and T by an edge if T , π(T ) or π −1 (T ) is on the trimino
containing S. We allow multiple edges; it is even possible that the same square and the same
token are connected with three edges. Obviously the lengths of the edges in the graph do not
exceed d + 2. By length of an edge we mean the distance between the A-square and the A-token
it connects.
Each A-token T is connected with the three A-squares whose triminos contain T , π(T )
and π −1 (T ). Therefore in the graph all tokens are of degree 3. We show that the same is true
for the A-squares. Let S be an arbitrary A-square, and let T1 , T2 , T3 be the three tokens on
the trimino containing S. For i = 1, 2, 3, if Ti is an A-token, then S is connected with Ti ; if Ti
is a B-token then S is connected with π −1 (Ti ); finally, if Ti is a C-token then S is connected
with π(Ti ). Hence in the graph the A-squares also are of degree 3.
Since the A-squares are of degree 3, from every set S of A-squares exactly 3|S| edges start.
These edges end in at least |S| tokens because the A-tokens also are of degree 3. Hence every
set S of A-squares has at least |S| neighbors among the A-tokens.
Therefore, by Hall’s marriage theorem, the graph contains a perfect matching between
the two vertex classes. So there is a perfect matching between the A-squares and A-tokens
with edges no longer than d + 2. It follows that the tokens can be permuted as specified in the
problem statement.
Comment 1. In the original problem proposal the board was infinite and there were only two colors.
Having n colors for some positive integer n was an option; we chose n = 3. Moreover, we changed
the board to a finite one to avoid dealing with infinite graphs (although Hall’s theorem works in the
infinite case as well).
With only two colors Hall’s theorem is not needed. In this case we split the board into 2 × 1
dominos, and in the resulting graph all vertices are of degree 2. The graph consists of disjoint cycles
with even length and infinite paths, so the existence of the matching is trivial.
Having more than three colors would make the problem statement more complicated, because we
need a matching between every two color classes of tokens. However, this would not mean a significant
increase in difficulty.
25
Comment 2. According to Wikipedia, the color asparagus (hexadecimal code #87A96B) is a tone
of green that is named after the vegetable. Crayola created this color in 1993 as one of the 16 to
be named in the Name The Color Contest. Byzantium (#702963) is a dark tone of purple. Its first
recorded use as a color name in English was in 1926. Citrine (#E4D00A) is variously described as
yellow, greenish-yellow, brownish-yellow or orange. The first known use of citrine as a color name in
English was in the 14th century.
26
C6. Let k and n be fixed positive integers. In the liar’s guessing game, Amy chooses integers
x and N with 1 ≤ x ≤ N. She tells Ben what N is, but not what x is. Ben may then repeatedly
ask Amy whether x ∈ S for arbitrary sets S of integers. Amy will always answer with yes or no,
but she might lie. The only restriction is that she can lie at most k times in a row. After he
has asked as many questions as he wants, Ben must specify a set of at most n positive integers.
If x is in this set he wins; otherwise, he loses. Prove that:
a) If n ≥ 2k then Ben can always win.
b) For sufficiently large k there exist n ≥ 1.99k such that Ben cannot guarantee a win.
Solution. Consider an answer A ∈ {yes, no} to a question of the kind “Is x in the set S?”
We say that A is inconsistent with a number i if A = yes and i 6∈ S, or if A = no and i ∈ S.
Observe that an answer inconsistent with the target number x is a lie.
a) Suppose that Ben has determined a set T of size m that contains x. This is true initially
with m = N and T = {1, 2, . . . , N}. For m > 2k we show how Ben can find a number y ∈ T
that is different from x. By performing this step repeatedly he can reduce T to be of size 2k ≤ n
and thus win.
Since only the size m > 2k of T is relevant, assume that T = {0, 1, . . . , 2k , . . . , m−1}. Ben
begins by asking repeatedly whether x is 2k . If Amy answers no k + 1 times in a row, one
of these answers is truthful, and so x 6= 2k . Otherwise Ben stops asking about 2k at the first
answer yes. He then asks, for each i = 1, . . . , k, if the binary representation of x has a 0 in
the ith digit. Regardless of what the k answers are, they are all inconsistent with a certain
number y ∈ {0, 1, . . . , 2k − 1}. The preceding answer yes about 2k is also inconsistent with y.
Hence y 6= x. Otherwise the last k + 1 answers are not truthful, which is impossible.
Either way, Ben finds a number in T that is different from x, and the claim is proven.
k+1
b) We prove that if 1 < λ < 2 and n = (2 − λ)λ − 1 then Ben cannot guarantee a win.
To complete the proof, then it suffices to take λ such that 1.99 < λ < 2 and k large enough so
that
n = (2 − λ)λk+1 − 1 ≥ 1.99k .
Consider the following strategy for Amy. First she chooses N = n+1 and x ∈ {1, 2, . . . , n+1}
arbitrarily. After every answer of hers Amy determines, for each i = 1, 2, . . . , n + 1, the
number mi of consecutive answers she has given by that point that are inconsistent with i. To
decide on her next answer, she then uses the quantity
n+1
X
φ= λmi .
i=1
No matter what Ben’s next question is, Amy chooses the answer which minimizes φ.
We claim that with this strategy φ will always stay less than λk+1 . Consequently no expo-
nent mi in φ will ever exceed k, hence Amy will never give more than k consecutive answers
inconsistent with some i. In particular this applies to the target number x, so she will never lie
more than k times in a row. Thus, given the claim, Amy’s strategy is legal. Since the strategy
does not depend on x in any way, Ben can make no deductions about x, and therefore he cannot
guarantee a win.
It remains to show that φ < λk+1 at all times.
Initially
each mi is 0, so this condition holds
k+1 k+1
in the beginning due to 1 < λ < 2 and n = (2 − λ)λ − 1. Suppose that φ < λ at some
point, and Ben has just asked if x ∈ S for some set S. According as Amy answers yes or no,
the new value of φ becomes
X X X X
φ1 = 1+ λmi +1 or φ2 = λmi +1 + 1.
i∈S i∈S
/ i∈S i∈S
/
27
Since Amy chooses the option minimizing φ, the new φ will equal min(φ1 , φ2 ). Now we have
!
1 1 X X 1
min(φ1 , φ2 ) ≤ (φ1 + φ2 ) = 1 + λmi +1 + λmi +1 + 1 = (λφ + n + 1).
2 2 i∈S 2
i∈S
/
Because φ < λk+1, the assumptions λ < 2 and n = (2 − λ)λk+1 − 1 lead to
1
min(φ1 , φ2 ) < (λk+2 + (2 − λ)λk+1 ) = λk+1 .
2
The claim follows, which completes the solution.
Comment. Given a fixed k, let f (k) denote the minimum value of n for which Ben can guarantee a
victory. The problem asks for a proof that for large k
1.99k ≤ f (k) ≤ 2k .
C7. There are given 2500 points on a circle labeled 1, 2, . . . , 2500 in some order. Prove that
one can choose 100 pairwise disjoint chords joining some of these points so that the 100 sums
of the pairs of numbers at the endpoints of the chosen chords are equal.
Proof. Induction on n = |G|. The base n = 1 is clear. For the inductive step choose a vertex v0
in G of minimum degree d. Delete v0 and all of its neighbors v1 , . . . , vd and also all edges with
endpoints v0 , v1 , . . . , vd . This gives a new graph G′ . By the inductive assumption G′ contains
an independent set S ′ of vertices such that |S ′ | ≥ f (G′ ). Since no vertex in S ′ is a neighbor
of v0 in G, the set S = S ′ ∪ {v0 } is independent in G.
Let d′v be the degree of a vertex v in G′ . Clearly d′v ≤ dv for every such vertex v, and also
dvi ≥ d for all i = 0, 1, . . . , d by the minimal choice of v0 . Therefore
X X 1 Xd
1 1 d+1
f (G′ ) = ≥ = f (G) − ≥ f (G) − = f (G) − 1.
d′v + 1 dv + 1 i=0
d vi
+ 1 d + 1
v∈G′ v∈G
′
′ ′
Hence |S| = |S | + 1 ≥ f (G ) + 1 ≥ f (G), and the induction is complete.
499
We pass on to our problem. For clarity denote n = 2 and draw all chords determined by
the given 2n points. Color each chord with one of the colors 3, 4, . . . , 4n − 1 according to the
sum of the numbers at its endpoints. Chords with a common endpoint have different colors.
For each color c consider the following graph Gc . Its vertices are the chords of color c, and two
chords are neighbors in Gc if they intersect. Let f (Gc ) have the same meaning as in the lemma
for all graphs Gc .
Every chord ℓ divides the circle into two arcs, and one of them contains m(ℓ) ≤ n − 1 given
points. (In particular m(ℓ) = 0 if ℓ joins two consecutive points.) For each i = 0, 1, . . . , n − 2
there are 2n chords ℓ with m(ℓ) = i. Such a chord has degree at most i in the respective graph.
Indeed let A1 , . . . , Ai be all points on either arc determined by a chord ℓ with m(ℓ) = i and
color c. Every Aj is an endpoint of at most 1 chord colored c, j = 1, . . . , i. Hence at most
i chords of color c intersect ℓ.
It follows that forPeach i = 0, 1, . . . , n − 2 the 2n chords ℓ with m(ℓ) = i contribute at
2n
least i+1 to the sum c f (Gc ). Summation over i = 0, 1, . . . , n − 2 gives
X n−1
X 1
f (Gc ) ≥ 2n .
c i=1
i
Because there are 4n − 3 colors in all, averaging yields a color c such that
n−1 n−1
2n X 1 1X1
f (Gc ) ≥ > .
4n − 3 i=1 i 2 i=1 i
Pn−1 1
By the lemma there are at least 12 i=1 pairwise disjoint chords of color c, i. e. with the same
i Pn−1 1
sum c of the pairs of numbers at their endpoints. It remains to show that 12 i=1 i
≥ 100 for
499
n = 2 . Indeed we have
n−1
X 2400 400 2k 400 k−1
1 X1 X X 1 X 2
> =1+ >1+ = 201 > 200.
i=1
i i=1
i k=1 k−1+1
i k=1
2k
i=2
This completes the solution.
29
Geometry
G1. In the triangle ABC the point J is the center of the excircle opposite to A. This excircle
is tangent to the side BC at M, and to the lines AB and AC at K and L respectively. The
lines LM and BJ meet at F , and the lines KM and CJ meet at G. Let S be the point of
intersection of the lines AF and BC, and let T be the point of intersection of the lines AG
and BC. Prove that M is the midpoint of ST .
Solution. Let α = ∠CAB, β = ∠ABC and γ = ∠BCA. The line AJ is the bisector of ∠CAB,
so ∠JAK = ∠JAL = α2 . By ∠AKJ = ∠ALJ = 90◦ the points K and L lie on the circle ω
with diameter AJ.
The triangle KBM is isosceles as BK and BM are tangents to the excircle. Since BJ is the
bisector of ∠KBM, we have ∠MBJ = 90◦ − β2 and ∠BMK = β2 . Likewise ∠MCJ = 90◦ − γ2
and ∠CML = γ2 . Also ∠BMF = ∠CML, therefore
◦ β γ α
∠LF J = ∠MBJ − ∠BMF = 90 − − = = ∠LAJ.
2 2 2
Hence F lies on the circle ω. (By the angle computation, F and A are on the same side of BC.)
Analogously, G also lies on ω. Since AJ is a diameter of ω, we obtain ∠AF J = ∠AGJ = 90◦ .
A
ω
α α
2 2
F G
α α
2 2
S β M γ T
B C
K
J
The lines AB and BC are symmetric with respect to the external bisector BF . Because
AF ⊥ BF and KM ⊥ BF , the segments SM and AK are symmetric with respect to BF ,
hence SM = AK. By symmetry T M = AL. Since AK and AL are equal as tangents to the
excircle, it follows that SM = T M, and the proof is complete.
Comment. After discovering the circle AF KJLG, there are many other ways to complete the solu-
tion. For instance, from the cyclic quadrilaterals JM F S and JM GT one can find ∠T SJ = ∠ST J = α2 .
Another possibility is to use the fact that the lines AS and GM are parallel (both are perpendicular
MS AG
to the external angle bisector BJ), so M T = GT = 1.
30
G2. Let ABCD be a cyclic quadrilateral whose diagonals AC and BD meet at E. The
extensions of the sides AD and BC beyond A and B meet at F . Let G be the point such that
ECGD is a parallelogram, and let H be the image of E under reflection in AD. Prove that
D, H, F , G are concyclic.
Solution. We show first that the triangles F DG and F BE are similar. Since ABCD is cyclic,
the triangles EAB and EDC are similar, as well as F AB and F CD. The parallelogram ECGD
yields GD = EC and ∠CDG = ∠DCE; also ∠DCE = ∠DCA = ∠DBA by inscribed angles.
Therefore
H A B
Comment. Points E and G are always in the half-plane determined by the line F D that contains
B and C, but H is always in the other half-plane. In particular, DHF G is cyclic if and only if
∠F HD + ∠F GD = 180◦ .
31
G3. In an acute triangle ABC the points D, E and F are the feet of the altitudes through A,
B and C respectively. The incenters of the triangles AEF and BDF are I1 and I2 respectively;
the circumcenters of the triangles ACI1 and BCI2 are O1 and O2 respectively. Prove that I1 I2
and O1 O2 are parallel.
I3 O2
E
I
I1
Q I2
A F B
In addition II1 · IA = II2 · IB implies that I has the same power with respect to the
circles (ACI1 ), (BCI2 ) and (ABI1 I2 ). Then CI is the radical axis of (ACI1 ) and (BCI2 ); in
particular CI is perpendicular to the line of centers O1 O2 .
Now it suffices to prove that CI ⊥ I1 I2 . Let CI meet I1 I2 at Q, then it is enough to check
that ∠II1 Q + ∠I1 IQ = 90◦ . Since ∠I1 IQ is external for the triangle ACI, we have
It remains to note that ∠II1 I2 = β2 from the cyclic quadrilateral ABI1 I2 , and ∠ACI = γ
2
,
∠CAI = α2 . Therefore ∠II1 Q + ∠I1 IQ = α2 + β2 + γ2 = 90◦ , completing the proof.
Comment. It follows from the first part of the solution that the common point I3 6= C of the
circles (ACI1 ) and (BCI2 ) is the incenter of the triangle CDE.
32
G4. Let ABC be a triangle with AB 6= AC and circumcenter O. The bisector of ∠BAC
intersects BC at D. Let E be the reflection of D with respect to the midpoint of BC. The lines
through D and E perpendicular to BC intersect the lines AO and AD at X and Y respectively.
Prove that the quadrilateral BXCY is cyclic.
Solution. The bisector of ∠BAC and the perpendicular bisector of BC meet at P , the midpoint
d (they are different lines as AB 6= AC). In particular OP is perpendicular
of the minor arc BC
to BC and intersects it at M, the midpoint of BC.
Denote by Y ′ the reflexion of Y with respect to OP . Since ∠BY C = ∠BY ′ C, it suffices to
prove that BXCY ′ is cyclic.
M
B E D C
Y Y′
We have
∠XAP = ∠OP A = ∠EY P.
The first equality holds because OA = OP , and the second one because EY and OP are both
perpendicular to BC and hence parallel. But {Y, Y ′ } and {E, D} are pairs of symmetric points
with respect to OP , it follows that ∠EY P = ∠DY ′ P and hence
The last equation implies that XAY ′ P is cyclic. By the powers of D with respect to the
circles (XAY ′ P ) and (ABP C) we obtain
XD · DY ′ = AD · DP = BD · DC.
G5. Let ABC be a triangle with ∠BCA = 90◦ , and let C0 be the foot of the altitude
from C. Choose a point X in the interior of the segment CC0 , and let K, L be the points on
the segments AX, BX for which BK = BC and AL = AC respectively. Denote by M the
intersection of AL and BK. Show that MK = ML.
Solution. Let C ′ be the reflection of C in the line AB, and let ω1 and ω2 be the circles
with centers A and B, passing through L and K respectively. Since AC ′ = AC = AL and
BC ′ = BC = BK, both ω1 and ω2 pass through C and C ′ . By ∠BCA = 90◦ , AC is tangent
to ω2 at C, and BC is tangent to ω1 at C. Let K1 6= K be the second intersection of AX and
ω2 , and let L1 6= L be the second intersection of BX and ω1 .
K1
L1 ω2
ω1
C
ω3
K L
M
A C0 B
C′
By the powers of X with respect to ω2 and ω1 ,
XK · XK1 = XC · XC ′ = XL · XL1 ,
AL2 = AC 2 = AK · AK1 ,
G6. Let ABC be a triangle with circumcenter O and incenter I. The points D, E and F on
the sides BC, CA and AB respectively are such that BD + BF = CA and CD + CE = AB.
The circumcircles of the triangles BF D and CDE intersect at P 6= D. Prove that OP = OI.
u
L
A u
v
Apply the lemma to ∠BAC = α and the circle ω = ωA , which intersects AI at A′ . This
gives 2AA′ cos α2 = AE + AF = BC; by symmetry analogous relations hold for BB ′ and CC ′ .
It follows that A′ , B ′ and C ′ are independent of the choice of D, E and F , as stated.
We use the lemma two more times with ∠BAC = α. Let ω be the circle with diameter AI.
Then X and Y are the tangency points of the incircle of ABC with AB and AC, and hence
AX = AY = 21 (AB + AC − BC). So the lemma yields 2AI cos α2 = AB + AC − BC. Next,
if ω is the circumcircle of ABC and AI intersects ω at M 6= A then {X, Y } = {B, C}, and so
2AM cos α2 = AB + AC by the lemma. To summarize,
2AA′ cos α2 = BC, 2AI cos α2 = AB + AC − BC, 2AM cos α2 = AB + AC. (*)
These equalities imply AA′ + AI = AM, hence the segments AM and IA′ have a common
midpoint. It follows that I and A′ are equidistant from the circumcenter O. By symmetry
OI = OA′ = OB ′ = OC ′, so I, A′ , B ′ , C ′ are on a circle centered at O.
To prove OP = OI, now it suffices to show that I, A′ , B ′ , C ′ and P are concyclic. Clearly
one can assume P 6= I, A′ , B ′ , C ′ .
We use oriented angles to avoid heavy case distinction. The oriented angle between the lines l
and m is denoted by ∠(l, m). We have ∠(l, m) = −∠(m, l) and ∠(l, m) + ∠(m, n) = ∠(l, n)
for arbitrary lines l, m and n. Four distinct non-collinear points U, V, X, Y are concyclic if and
only if ∠(UX, V X) = ∠(UY, V Y ).
35
ωA
B′
F E
I P
O
C′
A′
B D C
ωC
ωB
Suppose for the moment that A′ , B ′ , P, I are distinct and noncollinear; then it is enough to
check the equality ∠(A′ P, B ′ P ) = ∠(A′ I, B ′ I). Because A, F, P, A′ are on the circle ωA , we have
∠(A′ P, F P ) = ∠(A′ A, F A) = ∠(A′ I, AB). Likewise ∠(B ′ P, F P ) = ∠(B ′ I, AB). Therefore
Here we assumed that P 6= F . If P = F then P 6= D, E and the conclusion follows similarly (use
∠(A′ F, B ′ F ) = ∠(A′ F, EF ) + ∠(EF, DF ) + ∠(DF, B ′ F ) and inscribed angles in ωA , ωB , ωC ).
There is no loss of generality in assuming A′ , B ′ , P, I distinct and noncollinear. If ABC
is an equilateral triangle then the equalities (*) imply that A′ , B ′ , C ′, I, O and P coincide, so
OP = OI. Otherwise at most one of A′ , B ′ , C ′ coincides with I. If say C ′ = I then OI ⊥ CI
by the previous reasoning. It follows that A′ , B ′ 6= I and hence A′ 6= B ′ . Finally A′ , B ′ and I
are noncollinear because I, A′ , B ′ , C ′ are concyclic.
Comment. The proposer remarks that the locus γ of the points P is an arc of the circle (A′ B ′ C ′ I).
The reflection I ′ of I in O belongs to γ; it is obtained by choosing D, E and F to be the tangency
points of the three excircles with their respective sides. The rest of the circle (A′ B ′ C ′ I), except I,
can be included in γ by letting D, E and F vary on the extensions of the sides and assuming signed
lengths. For instance if B is between C and D then the length BD must be taken with a negative
sign. The incenter I corresponds to the limit case where D tends to infinity.
36
G7. Let ABCD be a convex quadrilateral with non-parallel sides BC and AD. Assume
that there is a point E on the side BC such that the quadrilaterals ABED and AECD are
circumscribed. Prove that there is a point F on the side AD such that the quadrilaterals
ABCF and BCDF are circumscribed if and only if AB is parallel to CD.
Solution. Let ω1 and ω2 be the incircles and O1 and O2 the incenters of the quadrilater-
als ABED and AECD respectively. A point F with the stated property exists only if ω1
and ω2 are also the incircles of the quadrilaterals ABCF and BCDF .
B
O1
C
O2
O D F2 F 1 A
Let the tangents from B to ω2 and from C to ω1 (other than BC) meet AD at F1 and F2
respectively. We need to prove that F1 = F2 if and only if AB k CD.
Lemma. The circles ω1 and ω2 with centers O1 and O2 are inscribed in an angle with vertex O.
The points P, S on one side of the angle and Q, R on the other side are such that ω1 is the
incircle of the triangle P QO, and ω2 is the excircle of the triangle RSO opposite to O. Denote
p = OO1 · OO2. Then exactly one of the following relations holds:
OP · OR < p < OQ · OS, OP · OR > p > OQ · OS, OP · OR = p = OQ · OS.
v Q
R
ϕ x
O ϕ y O2 O1
S
u P
Going back to the problem, apply the lemma to the quadruples {B, E, D, F1 }, {A, B, C, D}
and {A, E, C, F2}. Assuming OE · OF1 > p, we obtain
In these cases F1 6= F2 and OB · OD 6= OA · OC, so the lines AB and CD are not parallel.
There remains the case OE · OF1 = p. Here the lemma leads to OB · OD = p = OA · OC
and OE · OF1 = p = OE · OF2 . Therefore F1 = F2 and AB k CD.
Comment. The conclusion is also true if BC and AD are parallel. One can prove a limit case of
the lemma for the configuration shown in the figure below, where r1 and r2 are parallel rays starting
at O′ and O′′ , with O′ O′′ ⊥ r1 , r2 and O the midpoint of O′ O′′ . Two circles with centers O1 and O2
are inscribed in the strip between r1 and r2 . The lines P Q and RS are tangent to the circles, with
P, S on r1 , and Q, R on r2 , so that O, O1 are on the same side of P Q and O, O2 are on different sides
of RS. Denote s = OO1 + OO2 . Then exactly one of the following relations holds:
O ′′ R Q
r2
O
O2 O1
r1
O′ S P
Once this is established, the proof of the original statement for BC k AD is analogous to the one
in the intersecting case. One replaces products by sums of relevant segments.
38
G8. Let ABC be a triangle with circumcircle ω and ℓ a line without common points with ω.
Denote by P the foot of the perpendicular from the center of ω to ℓ. The side-lines BC, CA, AB
intersect ℓ at the points X, Y, Z different from P . Prove that the circumcircles of the triangles
AXP, BY P and CZP have a common point different from P or are mutually tangent at P .
Solution 1. Let ωA , ωB , ωC and ω be the circumcircles of triangles AXP, BY P, CZP and ABC
respectively. The strategy of the proof is to construct a point Q with the same power with
respect to the four circles. Then each of P and Q has the same power with respect to ωA , ωB , ωC
and hence the three circles are coaxial. In other words they have another common point P ′ or
the three of them are tangent at P .
We first give a description of the point Q. Let A′ 6= A be the second intersection of ω
and ωA ; define B ′ and C ′ analogously. We claim that AA′ , BB ′ and CC ′ have a common point.
Once this claim is established, the point just constructed will be on the radical axes of the
three pairs of circles {ω, ωA }, {ω, ωB }, {ω, ωC }. Hence it will have the same power with respect
to ω, ωA, ωB , ωC .
C′ ωC
A P′ ω
B′
O
Q
ωA
ωB
B
C A′
X ℓ Z′ Y P X′ Z Y′
We proceed to prove that AA′ , BB ′ and CC ′ intersect at one point. Let r be the circumra-
dius of triangle ABC. Define the points X ′ , Y ′ , Z ′ as the intersections of AA′ , BB ′ , CC ′ with ℓ.
Observe that X ′ , Y ′ , Z ′ do exist. If AA′ is parallel to ℓ then ωA is tangent to ℓ; hence X = P
which is a contradiction. Similarly, BB ′ and CC ′ are not parallel to ℓ.
From the powers of the point X ′ with respect to the circles ωA and ω we get
X ′ P · (X ′ P + P X) = X ′ P · X ′ X = X ′ A′ · X ′ A = X ′ O 2 − r 2 ,
hence
X ′ P · P X = X ′ O 2 − r 2 − X ′ P 2 = OP 2 − r 2 .
We argue analogously for the points Y ′ and Z ′ , obtaining
X ′ P · P X = Y ′ P · P Y = Z ′ P · P Z = OP 2 − r 2 = k 2 . (1)
In these computations all segments are regarded as directed segments. We keep the same
convention for the sequel.
We prove that the lines AA′ , BB ′ , CC ′ intersect at one point by Ceva’s theorem. To avoid
distracting remarks we interpret everything projectively, i. e. whenever two lines are parallel
they meet at a point on the line at infinity.
39
Let U, V, W be the intersections of AA′ , BB ′ , CC ′ with BC, CA, AB respectively. The idea
is that although it is difficult to calculate the ratio BU
CU
, it is easier to deal with the cross-ratio
BU BX
/
CU CX
because we can send it to the line ℓ. With this in mind we apply Menelaus’ theorem
to the triangle ABC and obtain BX · CY · BZ
CX AY
AZ
= 1. Hence Ceva’s ratio can be expressed as
BU CV AW BU BX CV CY AW AZ
· · = / · / · / .
CU AV BW CU CX AV AY BW BZ
A
ω
W
V
Q
B
U
C
X ℓ Z′ Y P X′ Z Y′
Project the line BC to ℓ from A. The cross-ratio between BC and UX equals the cross-ratio
between ZY and X ′ X. Repeating the same argument with the lines CA and AB gives
BU CV AW ZX ′ ZX XY ′ XY Y Z ′ Y Z
· · = / · / · /
CU AV BW Y X ′ Y X ZY ′ ZY XZ ′ XZ
and hence
BU CV AW ZX ′ XY ′ Y Z ′
· · = (−1) · · · .
CU AV BW Y X ′ ZY ′ XZ ′
The equations (1) reduce the problem to a straightforward computation on the line ℓ.
For instance, the transformation t 7→ −k 2 /t preserves cross-ratio and interchanges the points
X, Y, Z with the points X ′ , Y ′ , Z ′ . Then
BU CV AW ZX ′ ZZ ′ XY ′ XZ ′
· · = (−1) · / · / = −1.
CU AV BW Y X ′ Y Z ′ ZY ′ ZZ ′
We proved that Ceva’s ratio equals −1, so AA′ , BB ′ , CC ′ intersect at one point Q.
Comment 1. There is a nice projective argument to prove that AX ′ , BY ′ , CZ ′ intersect at one point.
Suppose that ℓ and ω intersect at a pair of complex conjugate points D and E. Consider a projective
transformation that takes D and E to [i; 1, 0] and [−i, 1, 0]. Then ℓ is the line at infinity, and ω is
a conic through the special points [i; 1, 0] and [−i, 1, 0], hence it is a circle. So one can assume that
AX, BY, CZ are parallel to BC, CA, AB. The involution on ℓ taking X, Y, Z to X ′ , Y ′ , Z ′ and leaving
D, E fixed is the involution changing each direction to its perpendicular one. Hence AX, BY, CZ are
also perpendicular to AX ′ , BY ′ , CZ ′ .
It follows from the above that AX ′ , BY ′ , CZ ′ intersect at the orthocenter of triangle ABC.
Comment 2. The restriction that the line ℓ does not intersect the circumcricle ω is unnecessary.
The proof above works in general. In case ℓ intersects ω at D and E point P is the midpoint of DE,
and some equations can be interpreted differently. For instance
X ′ P · X ′ X = X ′ A′ · X ′ A = X ′ D · X ′ E,
and hence the pairs X ′ X and DE are harmonic conjugates. This means that X ′ , Y ′ , Z ′ are the
harmonic conjugates of X, Y, Z with respect to the segment DE.
40
Solution 2. First we prove that there is an inversion in space that takes ℓ and ω to parallel
circles on a sphere. Let QR be the diameter of ω whose extension beyond Q passes through P .
Let Π be the plane carrying our objects. In space, choose a point O such that the line QO is
perpendicular to Π and ∠P OR = 90◦ , and apply an inversion with pole O (the radius of the
inversion does not matter). For any object T denote by T ′ the image of T under this inversion.
The inversion takes the plane Π to a sphere Π′ . The lines in Π are taken to circles through O,
and the circles in Π also are taken to circles on Π′ .
O
ℓ′ R′
P′ ω′
ℓ Π′
Q′
P Q R
ω
Π
Since the line ℓ and the circle ω are perpendicular to the plane OP Q, the circles ℓ′ and ω ′
also are perpendicular to this plane. Hence, the planes of the circles ℓ′ and ω ′ are parallel.
Now consider the circles A′ X ′ P ′, B ′ Y ′ P ′ and C ′ Z ′ P ′. We want to prove that either they
have a common point (on Π′ ), different from P ′ , or they are tangent to each other.
W C′
A1
B1 H
ω′
A′
B′
C1
Π′
′
Y
ℓ′
P′
X′ O
Z′
The point X ′ is the second intersection of the circles B ′ C ′ O and ℓ′ , other than O. Hence,
the lines OX ′ and B ′ C ′ are coplanar. Moreover, they lie in the parallel planes of ℓ′ and ω ′ .
Therefore, OX ′ and B ′ C ′ are parallel. Analogously, OY ′ and OZ ′ are parallel to A′ C ′ and A′ B ′ .
Let A1 be the second intersection of the circles A′ X ′ P ′ and ω ′, other than A′ . The segments
A′ A1 and P ′ X ′ are coplanar, and therefore parallel. Now we know that B ′ C ′ and A′ A1 are
parallel to OX ′ and X ′ P ′ respectively, but these two segments are perpendicular because OP ′
is a diameter in ℓ′ . We found that A′ A1 and B ′ C ′ are perpendicular, hence A′ A1 is the altitude
in the triangle A′ B ′ C ′ , starting from A.
Analogously, let B1 and C1 be the second intersections of ω ′ with the circles B ′ P ′ Y ′
and C ′ P ′ Z ′ , other than B ′ and C ′ respectively. Then B ′ B1 and C ′ C1 are the other two al-
titudes in the triangle A′ B ′ C ′ .
41
Let H be the orthocenter of the triangle A′ B ′ C ′ . Let W be the second intersection of the
line P ′ H with the sphere Π′ , other than P ′ . The point W lies on the sphere Π′ , in the plane
of the circle A′ P ′X ′ , so W lies on the circle A′ P ′ X ′ . Similarly, W lies on the circles B ′ P ′ Y ′
and C ′ P ′ Z ′ as well; indeed W is the second common point of the three circles.
If the line P ′ H is tangent to the sphere then W coincides with P ′ , and P ′ H is the common
tangent of the three circles.
42
Number Theory
N1. Call admissible a set A of integers that has the following property:
If x, y ∈ A (possibly x = y) then x2 + kxy + y 2 ∈ A for every integer k.
Determine all pairs m, n of nonzero integers such that the only admissible set containing both m
and n is the set of all integers.
Solution. A pair of integers m, n fulfills the condition if and only if gcd(m, n) = 1. Suppose
that gcd(m, n) = d > 1. The set
is admissible, because if d divides x and y then it divides x2 + kxy + y 2 for every integer k.
Also m, n ∈ A and A 6= Z.
Now let gcd(m, n) = 1, and let A be an admissible set containing m and n. We use the
following observations to prove that A = Z:
To justify (i) let y = x in the definition of an admissible set; to justify (ii) let k = 2.
Since gcd(m, n) = 1, we also have gcd(m2 , n2 ) = 1. Hence one can find integers a, b such
that am2 + bn2 = 1. It follows from (i) that am2 ∈ A and bn2 ∈ A. Now we deduce from (ii)
that 1 = (am2 + bn2 )2 ∈ A. But if 1 ∈ A then (i) implies k ∈ A for every integer k.
43
N2. Find all triples (x, y, z) of positive integers such that x ≤ y ≤ z and
x3 (y 3 + z 3 ) = 2012(xyz + 2).
Solution. First note that x divides 2012 · 2 = 23 · 503. If 503 | x then the right-hand side of the
equation is divisible by 5033 , and it follows that 5032 | xyz + 2. This is false as 503 | x. Hence
x = 2m with m ∈ {0, 1, 2, 3}. If m ≥ 2 then 26 | 2012(xyz + 2). However the highest powers
of 2 dividing 2012 and xyz + 2 = 2m yz + 2 are 22 and 21 respectively. So x = 1 or x = 2,
yielding the two equations
In both cases the prime 503 = 3 · 167 + 2 divides y 3 + z 3 . We claim that 503 | y + z. This
is clear if 503 | y, so let 503 ∤ y and 503 ∤ z. Then y 502 ≡ z 502 (mod 503) by Fermat’s little
theorem. On the other hand y 3 ≡ −z 3 (mod 503) implies y 3·167 ≡ −z 3·167 (mod 503), i. e.
y 501 ≡ −z 501 (mod 503). It follows that y ≡ −z (mod 503) as claimed.
Therefore y + z = 503k with k ≥ 1. In view of y 3 + z 3 = (y + z) (y − z)2 + yz the two
equations take the form
m m
N3. Determine all integers m ≥ 2 such that every n with 3
≤n≤ 2
divides the binomial
n
coefficient m−2n
.
• If m is odd then there exist an odd prime p and an integer k ≥ 1 with m = p(2k + 1).
Pick n = pk, then m3 ≤ n ≤ m2 by k ≥ 1. However
1 n 1 pk (pk − 1)(pk − 2) · · · (pk − (p − 1))
= =
n m − 2n pk p p!
is not an integer, because p divides the denominator but not the numerator.
45
N4. An integer a is called friendly if the equation (m2 + n)(n2 + m) = a(m − n)3 has a
solution over the positive integers.
a) Prove that there are at least 500 friendly integers in the set {1, 2, . . . , 2012}.
Hence 5, 9, . . . , 2009 are friendly and so {1, 2, . . . , 2012} contains at least 502 friendly numbers.
b) We show that a = 2 is not friendly. Consider the equation with a = 2 and rewrite its
left-hand side as a difference of squares:
1
(m2 + n + n2 + m)2 − (m2 + n − n2 − m)2 = 2(m − n)3 .
4
Since m2 + n − n2 − m = (m − n)(m + n − 1), we can further reformulate the equation as
(m2 + n + n2 + m)2 = (m − n)2 8(m − n) + (m + n − 1)2 .
It follows that 8(m − n) + (m + n − 1)2 is a perfect square. Clearly m > n, hence there is an
integer s ≥ 1 such that
Comment. A computer search shows that there are 561 friendly numbers in {1, 2, . . . , 2012}.
46
Solution 1. We are going to prove that f (x) = axm for some nonnegative integers a and
m. If f (x) is the zero polynomial we are done, so assume that f (x) has at least one positive
coefficient. In particular f (1) > 0.
Let p be a prime number. The condition is that f (n) ≡ 0 (mod p) implies
The idea is to construct a prime p and a positive integer n such that p − 1 divides n and p
divides f (n). In this case, for k large enough p − 1 divides rad(n)k . Hence if (p, n) = 1 then
k
nrad(n) ≡ 1 (mod p) by Fermat’s little theorem, so that
k
f (1) ≡ f (nrad(n) ) ≡ 0 (mod p). (2)
Suppose that f (x) = g(x)xm with g(0) 6= 0. Let t be a positive integer, p any prime factor
of g(−t) and n = (p−1)t. So p−1 divides n and f (n) = f ((p − 1)t) ≡ f (−t) ≡ 0 (mod p), hence
either (p, n) > 1 or (2) holds. If (p, (p−1)t) > 1 then p divides t and g(0) ≡ g(−t) ≡ 0 (mod p),
meaning that p divides g(0).
In conclusion we proved that each prime factor of g(−t) divides g(0)f (1) 6= 0, and thus the
set of prime factors of g(−t) when t ranges through the positive integers is finite. This is known
to imply that g(x) is a constant polynomial, and so f (x) = axm .
Solution 2. Let f (x) be a polynomial with integer coefficients (not necessarily nonnegative)
such that rad(f (n)) divides rad(f (nrad(n) )) for any nonnegative integer n. We give a complete
description of all polynomials with this property. More precisely, we claim that if f (x) is such
a polynomial and ξ is a root of f (x) then so is ξ d for every positive integer d.
Therefore each root of f (x) is zero or a root of unity. In particular, if a root of unity ξ is
a root of f (x) then 1 = ξ d is a root too (for some positive integer d). In the original problem
f (x) has nonnegative coefficients. Then either f (x) is the zero polynomial or f (1) > 0 and
ξ = 0 is the only possible root. In either case f (x) = axm with a and m nonnegative integers.
To prove the claim let ξ be a root of f (x), and let g(x) be an irreducible factor of f (x) such
that g(ξ) = 0. If 0 or 1 are roots of g(x) then either ξ = 0 or ξ = 1 (because g(x) is irreducible)
and we are done. So assume that g(0), g(1) 6= 0. By decomposing d as a product of prime
numbers, it is enough to consider the case d = p prime. We argue for p = 2. Since rad(2k ) = 2
for every k, we have
rad(f (2k )) | rad(f (22k )).
Now we prove that g(x) divides f (x2 ). Suppose that this is not the case. Then, since g(x)
is irreducible, there are integer-coefficient polynomials a(x), b(x) and an integer N such that
Each prime factor p of g(2k ) divides f (2k ), so by rad(f (2k ))|rad(f (22k )) it also divides f (22k ).
From the equation above with x = 2k it follows that p divides N.
47
In summary, each prime divisor of g(2k ) divides N, for all k ≥ 0. Let p1 , . . . , pn be the odd
primes dividing N, and suppose that
yielding
g(2k ) ≡ g(1) (mod pα1 1 +1 · · · pnαn +1 ).
It follows that for each i the maximal power of pi dividing g(2k ) and g(1) is the same, namely pαi i .
On the other hand, for large enough k, the maximal power of 2 dividing g(2k ) and g(0) 6= 0
is the same. From the above, for k divisible by ϕ(pα1 1 +1 · · · pαnn +1 ) and large enough, we obtain
that g(2k ) divides g(0) · g(1). This is impossible because g(0), g(1) 6= 0 are fixed and g(2k ) is
arbitrarily large.
In conclusion, g(x) divides f (x2 ). Recall that ξ is a root of f (x) such that g(ξ) = 0; then
f (ξ 2 ) = 0, i. e. ξ 2 is a root of f (x).
Likewise if ξ is a root of f (x) and p an arbitrary prime then ξ p is a root too. The argument
is completely analogous, in the proof above just replace 2 by p and “odd prime” by “prime
different from p.”
Comment. The claim in the second solution can be proved by varying n (mod p) in (1). For instance,
we obtain
f (nrad(n+pk) ) ≡ 0 (mod p)
for every positive integer k. One can prove that if (n, p) = 1 then rad(n + pk) runs through all residue
classes r (mod p − 1) with (r, p − 1) squarefree. Hence if f (n) ≡ 0 (mod p) then f (nr ) ≡ 0 (mod p)
for all integers r. This implies the claim by an argument leading to the identity (3).
48
n
N6. Let x and y be positive integers. If x2 − 1 is divisible by 2n y + 1 for every positive
integer n, prove that x = 1.
Solution. First we prove the following fact: For every positive integer y there exist infinitely
many primes p ≡ 3 (mod 4) such that p divides some number of the form 2n y + 1.
Clearly it is enough to consider the case y odd. Let
2y + 1 = pe11 · · · perr
be the prime factorization of 2y + 1. Suppose on the contrary that there are finitely many
primes pr+1 , . . . , pr+s ≡ 3 (mod 4) that divide some number of the form 2n y + 1 but do not
divide 2y + 1.
We want to find an n such that pei i ||2n y +1 for 1 ≤ i ≤ r and pi ∤ 2n y +1 for r +1 ≤ i ≤ r +s.
For this it suffices to take
because then
2n y + 1 ≡ 2y + 1 (mod pe11 +1 · · · perr +1 p1r+1 · · · p1r+s ).
The last congruence means that pe11 , . . . , perr divide exactly 2n y + 1 and no prime pr+1 , . . . , pr+s
divides 2n y + 1. It follows that the prime factorization of 2n y + 1 consists of the prime powers
pe11 , . . . , perr and powers of primes ≡ 1 (mod 4). Because y is odd, we obtain
n
Comment. For each x and each odd prime p the maximal power of p dividing x2 − 1 for some n is
bounded and hence the same must be true for the numbers 2n y + 1. We infer that p2 divides 2p−1 − 1
for each prime divisor p of 2n y + 1. However trying to reach a contradiction with this conclusion alone
seems hopeless, since it is not even known if there are infinitely many primes p without this property.
49
N7. Find all n ∈ N for which there exist nonnegative integers a1 , a2 , . . . , an such that
1 1 1 1 2 n
+ + · · · + = + + · · · + = 1.
2a1 2a2 2an 3a1 3a2 3an
Pn Such
Solution. numbers a1 , a2 , . . . , an exist if and only if n ≡ 1 (mod 4) or n ≡ 2 (mod 4).
k
Let k=1 3ak = 1 with a1 , a2 , . . . , an nonnegative integers. Then 1·x1 +2·x2 +· · ·+n·xn = 3a
with x1 , . . . , xn powers of 3 and a ≥ 0. The right-hand side is odd, and the left-hand side has
the same parity as 1 + 2 + · · ·+ n. Hence the latter sum is odd, which implies n ≡ 1, 2 (mod 4).
Now we prove the converse.
Call feasible a sequence b1 , b2 , . . . , bn if there are nonnegative integers a1 , a2 , . . . , an such that
1 1 1 b1 b2 bn
a
+ a2 + · · · + an = a1 + a2 + · · · + an = 1.
2 1 2 2 3 3 3
Let bk be a term of a feasible sequence b1 , b2 , . . . , bn with exponents a1 , a2 , . . . , an like above,
and let u, v be nonnegative integers with sum 3bk . Observe that
1 1 1 u v bk
+ = and + = .
2ak +1 2ak +1 2ak 3ak +1 3ak +1 3ak
It follows that the sequence b1 , . . . , bk−1 , u, v, bk+1, . . . , bn is feasible. The exponents ai are the
same for the unchanged terms bi , i 6= k; the new terms u, v have exponents ak + 1.
We state the conclusion in reverse. If two terms u, v of a sequence are replaced by one
term u+v3
and the obtained sequence is feasible, then the original sequence is feasible too.
Denote by αn the sequence 1, 2, . . . , n. To show that αn is feasible for n ≡ 1, 2 (mod 4), we
transform it by n − 1 replacements {u, v} 7→ u+v 3
to the one-term sequence α1 . The latter is
feasible, with a1 = 0. Note that if m and 2m are terms of a sequence then {m, 2m} 7→ m, so
2m can be ignored if necessary.
Let n ≥ 16. We prove that αn can be reduced to αn−12 by 12 operations. Write n = 12k + r
where k ≥ 1 and 0 ≤ r ≤ 11. If 0 ≤ r ≤ 5 then the last 12 terms of αn can be partitioned into
2 singletons {12k − 6}, {12k} and the following 5 pairs:
{12k − 6 − i, 12k − 6 + i}, i = 1, . . . , 5 − r; {12k − j, 12k + j}, j = 1, . . . , r.
(There is only one kind of pairs if r ∈ {0, 5}.) One can ignore 12k − 6 and 12k since αn
contains 6k − 3 and 6k. Furthermore the 5 operations {12k − 6 − i, 12k − 6 + i} 7→ 8k − 4 and
{12k − j, 12k + j} 7→ 8k remove the 10 terms in the pairs and bring in 5 new terms equal
to 8k − 4 or 8k. All of these can be ignored too as 4k − 2 and 4k are still present in the
sequence. Indeed 4k ≤ n − 12 is equivalent to 8k ≥ 12 − r, which is true for r ∈ {4, 5}. And if
r ∈ {0, 1, 2, 3} then n ≥ 16 implies k ≥ 2, so 8k ≥ 12 − r also holds. Thus αn reduces to αn−12 .
The case 6 ≤ r ≤ 11 is analogous. Consider the singletons {12k}, {12k + 6} and the 5 pairs
{12k − i, 12k + i}, i = 1, . . . , 11 − r; {12k + 6 − j, 12k + 6 + j}, j = 1, . . . , r − 6.
Ignore the singletons like before, then remove the pairs via operations {12k − i, 12k + i} 7→ 8k
and {12k + 6 − j, 12k + 6 + j} 7→ 8k + 4. The 5 newly-appeared terms 8k and 8k + 4 can be
ignored too since 4k + 2 ≤ n − 12 (this follows from k ≥ 1 and r ≥ 6). We obtain αn−12 again.
The problem reduces to 2 ≤ n ≤ 15. In fact n ∈ {2, 5, 6, 9, 10, 13, 14} by n ≡ 1, 2 (mod 4).
The cases n = 2, 6, 10, 14 reduce to n = 1, 5, 9, 13 respectively because the last even term of αn
can be ignored. For n = 5 apply {4, 5} 7→ 3, then {3, 3} 7→ 2, then ignore the 2 occurrences
of 2. For n = 9 ignore 6 first, then apply {5, 7} 7→ 4, {4, 8} 7→ 4, {3, 9} 7→ 4. Now ignore
the 3 occurrences of 4, then ignore 2. Finally n = 13 reduces to n = 10 by {11, 13} 7→ 8 and
ignoring 8 and 12. The proof is complete.
50
N8. Prove that for every prime p > 100 and every integer r there exist two integers a and b
such that p divides a2 + b5 − r.
Solution 1. Throughout the solution, all congruence relations are meant modulo p.
Fix p, and let P = {0, 1, . . . , p − 1} be
the set of residue classes modulo p. For every r ∈ P,
2 5
let Sr = (a, b) ∈ P × P : a + b ≡ r , and let sr = |Sr |. Our aim is to prove sr > 0 for
all r ∈ P.
We will use the well-known fact that for every residue class r ∈ P and every positive
integer k, there are at most k values x ∈ P such that xk ≡ r.
Lemma. Let N be the number of quadruples (a, b, c, d) ∈ P 4 for which a2 + b5 ≡ c2 + d5 . Then
X
N= s2r (a)
r∈P
and
N ≤ p(p2 + 4p − 4). (b)
Proof. (a) For each residue class r there exist exactly sr pairs (a, b) with a2 + b5 ≡ r and sr
pairs (c, d) with c2 + d5 ≡ r. So there are s2r quadruples with a2 + b5 ≡ c2 + d5 ≡ r. Taking the
sum over all r ∈ P, the statement follows.
(b) Choose an arbitrary pair (b, d) ∈ P and look for the possible values of a, c.
1. Suppose that b5 ≡ d5 , and let k be the number of such pairs (b, d). The value b can be
chosen in p different ways. For b ≡ 0 only d = 0 has this property; for the nonzero values of b
there are at most 5 possible values for d. So we have k ≤ 1 + 5(p − 1) = 5p − 4.
The values a and c must satisfy a2 ≡ c2 , so a ≡ ±c, and there are exactly 2p − 1 such
pairs (a, c).
2. Now suppose b5 6≡ d5 . In this case a and c must be distinct. By (a − c)(a + c) = d5 − b5 ,
the value of a − c uniquely determines a + c and thus a and c as well. Hence, there are p − 1
suitable pairs (a, c).
Thus, for each of the k pairs (b, d) with b5 ≡ d5 there are 2p − 1 pairs (a, c), and for each of
the other p2 − k pairs (b, d) there are p − 1 pairs (a, c). Hence,
To prove the statement of the problem, suppose that Sr = ∅ for some r ∈ P; obviously
r 6≡ 0. Let T = x10 : x ∈ P \ {0} be the set of nonzero 10th powers modulo p. Since each
residue class is the 10th power of at most 10 elements in P, we have |T | ≥ p−1
10
≥ 4 by p > 100.
10
For every t ∈ T , we have Str = ∅. Indeed, if (x, y) ∈ Str and t ≡ z then
Solution 2. If 5 ∤ p − 1, then all modulo p residue classes are complete fifth powers and the
statement is trivial. So assume that p = 10k + 1 where k ≥ 10. Let g be a primitive root
modulo p.
We will use the following facts:
(F1) If some residue class x is not quadratic then x(p−1)/2 ≡ −1 (mod p).
(F2) For every integer d, as a simple corollary of the summation formula for geometric pro-
gressions, (
2k−1
X 2k if 2k d
5di
g ≡ (mod p).
i=0
0 if 2k 6 | d
Suppose that, contrary to the statement, some modulo p residue class r cannot be expressed
as a2 + b5 . Of course r 6≡ 0 (mod p). By (F1) we have (r − b5 )(p−1)/2 = (r − b5 )5k ≡ −1 (mod p)
for all residue classes b.
For t = 1, 2 . . . , k − 1 consider the sums
2k−1
X 5k
S(t) = r − g 5i g 5ti .
i=0
Since 1 ≤ j + t < 6k, the number 2k divides j + t only for j = 2k − t and j = 4k − t. Hence,
t 5k 3k+t 5k k+t
0 ≡ S(t) ≡ (−1) r + r · 2k (mod p),
2k − t 4k − t
5k 2k 5k
r + ≡ 0 (mod p).
2k − t 4k − t
Taking this for t = 1, 2 and eliminating r, we get
5k 5k 2k 5k 5k 5k 2k 5k
0≡ r + − r +
2k − 2 2k − 1 4k − 1 2k − 1 2k − 2 4k − 2
5k 5k 5k 5k
= −
2k − 2 4k − 1 2k − 1 4k − 2
(5k)! 2
= (2k − 1)(k + 2) − (3k + 2)(4k − 1)
(2k − 1)!(3k + 2)!(4k − 1)!(k + 2)!
−(5k)!2 · 2k(5k + 1)
= (mod p).
(2k − 1)!(3k + 2)!(4k − 1)!(k + 2)!
But in the last expression none of the numbers is divisible by p = 10k + 1, a contradiction.
52
Comment 1. The argument in the second solution is valid whenever k ≥ 3, that is for all primes
p = 10k + 1 except p = 11. This is an exceptional case when the statement is not true; r = 7 cannot
be expressed as desired.
Comment 2. The statement is true in a more general setting: for every positive integer n, for all
sufficiently large p, each residue class modulo p can be expressed as a2 + bn . Choosing t = 3 would
allow using the Cauchy-Davenport theorem (together with some analysis on the case of equality).
In the literature more general results are known. For instance, the statement easily follows from
the Hasse-Weil bound.
Shortlisted Problems with Solutions
Contributing Countries
The Organizing Committee and the Problem Selection Committee of IMO 2013 thank the following
50 countries for contributing 149 problem proposals.
Problems
Algebra
A1. Let n be a positive integer and let a1 , . . . , an´1 be arbitrary real numbers. Define the
sequences u0 , . . . , un and v0 , . . . , vn inductively by u0 “ u1 “ v0 “ v1 “ 1, and
uk`1 “ uk ` ak uk´1 , vk`1 “ vk ` an´k vk´1 for k “ 1, . . . , n ´ 1.
Prove that un “ vn .
(France)
A2. Prove that in any set of 2000 distinct real numbers there exist two pairs a ą b and c ą d
with a ‰ c or b ‰ d, such that ˇ ˇ
ˇa ´ b ˇ 1
ˇ c ´ d ´ 1ˇ ă 100000 .
ˇ ˇ
(Lithuania)
A3. Let Qą0 be the set of positive rational numbers. Let f : Qą0 Ñ R be a function satisfying
the conditions
f pxqf pyq ě f pxyq and f px ` yq ě f pxq ` f pyq
for all x, y P Qą0 . Given that f paq “ a for some rational a ą 1, prove that f pxq “ x for all
x P Qą0 .
(Bulgaria)
A4. Let n be a positive integer, and consider a sequence a1 , a2 , . . . , an of positive integers.
Extend it periodically to an infinite sequence a1 , a2 , . . . by defining an`i “ ai for all i ě 1. If
a1 ď a2 ď ¨ ¨ ¨ ď an ď a1 ` n
and
aai ď n ` i ´ 1 for i “ 1, 2, . . . , n,
prove that
a1 ` ¨ ¨ ¨ ` an ď n2 .
(Germany)
A5. Let Zě0 be the set of all nonnegative integers. Find all the functions f : Zě0 Ñ Zě0
satisfying the relation
f pf pf pnqqq “ f pn ` 1q ` 1
for all n P Zě0 .
(Serbia)
A6. Let m ‰ 0 be an integer. Find all polynomials P pxq with real coefficients such that
px3 ´ mx2 ` 1qP px ` 1q ` px3 ` mx2 ` 1qP px ´ 1q “ 2px3 ´ mx ` 1qP pxq
for all real numbers x.
(Serbia)
4 IMO 2013 Colombia
Combinatorics
C1. Let n be a positive integer. Find the smallest integer k with the following property: Given
any real numbers a1 , . . . , ad such that a1 ` a2 ` ¨ ¨ ¨ ` ad “ n and 0 ď ai ď 1 for i “ 1, 2, . . . , d, it
is possible to partition these numbers into k groups (some of which may be empty) such that the
sum of the numbers in each group is at most 1.
(Poland)
C2. In the plane, 2013 red points and 2014 blue points are marked so that no three of the
marked points are collinear. One needs to draw k lines not passing through the marked points and
dividing the plane into several regions. The goal is to do it in such a way that no region contains
points of both colors.
Find the minimal value of k such that the goal is attainable for every possible configuration of
4027 points.
(Australia)
C3. A crazy physicist discovered a new kind of particle which he called an imon, after some of
them mysteriously appeared in his lab. Some pairs of imons in the lab can be entangled, and each
imon can participate in many entanglement relations. The physicist has found a way to perform
the following two kinds of operations with these particles, one operation at a time.
piq If some imon is entangled with an odd number of other imons in the lab, then the physicist
can destroy it.
piiq At any moment, he may double the whole family of imons in his lab by creating a copy I 1
of each imon I. During this procedure, the two copies I 1 and J 1 become entangled if and only if
the original imons I and J are entangled, and each copy I 1 becomes entangled with its original
imon I; no other entanglements occur or disappear at this moment.
Prove that the physicist may apply a sequence of such operations resulting in a family of imons,
no two of which are entangled.
(Japan)
C4. Let n be a positive integer, and let A be a subset of t1, . . . , nu. An A-partition of n into k
parts is a representation of n as a sum n “ a1 ` ¨ ¨ ¨ ` ak , where the parts a1 , . . . , ak belong to A
and are not necessarily distinct. The number of different parts in such a partition is the number
of (distinct) elements in the set ta1 , a2 , . . . , ak u.
We say that an A-partition of n into k parts is optimal if there is no A-partition
? of n into r
3
parts with r ă k. Prove that any optimal A-partition of n contains at most 6n different parts.
(Germany)
C5. Let r be a positive integer, and let a0 , a1 , . . . be an infinite sequence of real numbers.
Assume that for all nonnegative integers m and s there exists a positive integer n P rm ` 1, m ` rs
such that
am ` am`1 ` ¨ ¨ ¨ ` am`s “ an ` an`1 ` ¨ ¨ ¨ ` an`s .
Prove that the sequence is periodic, i. e. there exists some p ě 1 such that an`p “ an for all n ě 0.
(India)
Shortlisted problems 5
C6. In some country several pairs of cities are connected by direct two-way flights. It is possible
to go from any city to any other by a sequence of flights. The distance between two cities is defined
to be the least possible number of flights required to go from one of them to the other. It is known
that for any city there are at most 100 cities at distance exactly three from it. Prove that there is
no city such that more than 2550 other cities have distance exactly four from it.
(Russia)
C7. Let n ě 2 be an integer. Consider all circular arrangements of the numbers 0, 1, . . . , n; the
n ` 1 rotations of an arrangement are considered to be equal. A circular arrangement is called
beautiful if, for any four distinct numbers 0 ď a, b, c, d ď n with a ` c “ b ` d, the chord joining
numbers a and c does not intersect the chord joining numbers b and d.
Let M be the number of beautiful arrangements of 0, 1, . . . , n. Let N be the number of pairs
px, yq of positive integers such that x ` y ď n and gcdpx, yq “ 1. Prove that
M “ N ` 1.
(Russia)
C8. Players A and B play a paintful game on the real line. Player A has a pot of paint with
four units of black ink. A quantity p of this ink suffices to blacken a (closed) real interval of length
p. In every round, player A picks some positive integer m and provides 1{2m units of ink from the
pot. Player B then picks an integer k and blackens the interval from k{2m to pk ` 1q{2m (some
parts of this interval may have been blackened before). The goal of player A is to reach a situation
where the pot is empty and the interval r0, 1s is not completely blackened.
Decide whether there exists a strategy for player A to win in a finite number of moves.
(Austria)
6 IMO 2013 Colombia
Geometry
G1. Let ABC be an acute-angled triangle with orthocenter H, and let W be a point on
side BC. Denote by M and N the feet of the altitudes from B and C, respectively. Denote
by ω1 the circumcircle of BW N, and let X be the point on ω1 which is diametrically opposite
to W . Analogously, denote by ω2 the circumcircle of CW M, and let Y be the point on ω2 which
is diametrically opposite to W . Prove that X, Y and H are collinear.
(Thaliand)
G2. Let ω be the circumcircle of a triangle ABC. Denote by M and N the midpoints of the
sides AB and AC, respectively, and denote by T the midpoint of the arc BC of ω not containing A.
The circumcircles of the triangles AMT and ANT intersect the perpendicular bisectors of AC
and AB at points X and Y , respectively; assume that X and Y lie inside the triangle ABC. The
lines MN and XY intersect at K. Prove that KA “ KT .
(Iran)
G3. In a triangle ABC, let D and E be the feet of the angle bisectors of angles A and B,
respectively. A rhombus is inscribed into the quadrilateral AEDB (all vertices of the rhombus
lie on different sides of AEDB). Let ϕ be the non-obtuse angle of the rhombus. Prove that
ϕ ď maxt=BAC, =ABCu.
(Serbia)
G4. Let ABC be a triangle with =B ą =C. Let P and Q be two different points on line AC
such that =P BA “ =QBA “ =ACB and A is located between P and C. Suppose that there
exists an interior point D of segment BQ for which P D “ P B. Let the ray AD intersect the circle
ABC at R ‰ A. Prove that QB “ QR.
(Georgia)
G5. Let ABCDEF be a convex hexagon with AB “ DE, BC “ EF , CD “ F A, and
=A ´ =D “ =C ´ =F “ =E ´ =B. Prove that the diagonals AD, BE, and CF are concurrent.
(Ukraine)
G6. Let the excircle of the triangle ABC lying opposite to A touch its side BC at the point A1 .
Define the points B1 and C1 analogously. Suppose that the circumcentre of the triangle A1 B1 C1
lies on the circumcircle of the triangle ABC. Prove that the triangle ABC is right-angled.
(Russia)
Shortlisted problems 7
Number Theory
N1. Let Zą0 be the set of positive integers. Find all functions f : Zą0 Ñ Zą0 such that
m2 ` f pnq | mf pmq ` n
for all positive integers m and n.
(Malaysia)
N2. Prove that for any pair of positive integers k and n there exist k positive integers
m1 , m2 , . . . , mk such that
2k ´ 1
ˆ ˙ˆ ˙ ˆ ˙
1 1 1
1` “ 1` 1` ¨¨¨ 1 ` .
n m1 m2 mk
(Japan)
N3. Prove that there exist infinitely many positive integers n such that the largest prime divisor
of n4 ` n2 ` 1 is equal to the largest prime divisor of pn ` 1q4 ` pn ` 1q2 ` 1.
(Belgium)
N4. Determine whether there exists an infinite sequence of nonzero digits a1 , a2 , a3 , . . . and a
positive integer N such that for every integer k ą N, the number ak ak´1 . . . a1 is a perfect square.
(Iran)
N5. Fix an integer k ě 2. Two players, called Ana and Banana, play the following game of
numbers: Initially, some integer n ě k gets written on the blackboard. Then they take moves
in turn, with Ana beginning. A player making a move erases the number m just written on the
blackboard and replaces it by some number m1 with k ď m1 ă m that is coprime to m. The first
player who cannot move anymore loses.
An integer n ě k is called good if Banana has a winning strategy when the initial number is n,
and bad otherwise.
Consider two integers n, n1 ě k with the property that each prime number p ď k divides n if
and only if it divides n1 . Prove that either both n and n1 are good or both are bad.
(Italy)
N6. Determine all functions f : Q ÝÑ Z satisfying
ˆ ˙
f pxq ` a ´x ` a¯
f “f
b b
for all x P Q, a P Z, and b P Zą0 . (Here, Zą0 denotes the set of positive integers.)
(Israel)
N7. Let ν be an irrational positive number, and let m be a positive integer. A pair pa, bq of
positive integers is called good if
arbνs ´ btaνu “ m.
A good pair pa, bq is called excellent if neither of the pairs pa´b, bq and pa, b´aq is good. (As usual,
by txu and rxs we denote the integer numbers such that x ´ 1 ă txu ď x and x ď rxs ă x ` 1.)
Prove that the number of excellent pairs is equal to the sum of the positive divisors of m.
(U.S.A.)
8 IMO 2013 Colombia
Solutions
Algebra
A1. Let n be a positive integer and let a1 , . . . , an´1 be arbitrary real numbers. Define the
sequences u0 , . . . , un and v0 , . . . , vn inductively by u0 “ u1 “ v0 “ v1 “ 1, and
Prove that un “ vn .
(France)
Solution 1. We prove by induction on k that
ÿ
uk “ ai1 . . . ait . p1q
0ăi1 ă...ăit ăk,
ij`1 ´ij ě2
Note that we have one trivial summand equal to 1 (which corresponds to t “ 0 and the empty
sequence, whose product is 1).
For k “ 0, 1 the sum on the right-hand side only contains the empty product, so (1) holds due
to u0 “ u1 “ 1. For k ě 1, assuming the result is true for 0, 1, . . . , k, we have
ÿ ÿ
uk`1 “ ai1 . . . ait ` ai1 . . . ait ¨ ak
0ăi1 ă...ăit ăk, 0ăi1 ă...ăit ăk´1,
ij`1 ´ij ě2 ij`1 ´ij ě2
ÿ ÿ
“ ai1 . . . ait ` ai1 . . . ait
0ăi1 ă...ăit ăk`1, 0ăi1 ă...ăit ăk`1,
ij`1 ´ij ě2, ij`1 ´ij ě2,
kRti1 ,...,it u kPti1 ,...,it u
ÿ
“ ai1 . . . ait ,
0ăi1 ă...ăit ăk`1,
ij`1 ´ij ě2
as required.
Applying (1) to the sequence b1 , . . . , bn given by bk “ an´k for 1 ď k ď n, we get
ÿ ÿ
vk “ bi1 . . . bit “ ai1 . . . ait . p2q
0ăi1 ă...ăit ăk, nąi1 ą...ąit ąn´k,
ij`1 ´ij ě2 ij ´ij`1 ě2
so Pn is a polynomial in n ´ 1 variables for each n ě 1. Two easy inductive arguments show that
so we need to prove Pn px1 , . . . , xn´1 q “ Pn pxn´1 , . . . , x1 q for every positive integer n. The cases
n “ 1, 2 are trivial, and the cases n “ 3, 4 follow from P3 px, yq “ 1 ` x ` y and P4 px, y, zq “
1 ` x ` y ` z ` xz.
Now we proceed by induction, assuming that n ě 5 and the claim hold for all smaller cases.
Using F pa, bq as an abbreviation for P|a´b|`1 pxa , . . . , xb q (where the indices a, . . . , b can be either
in increasing or decreasing order),
F pn, 1q “ F pn, 2q ` x1 F pn, 3q “ F p2, nq ` x1 F p3, nq
“ pF p2, n ´ 1q ` xn F p2, n ´ 2qq ` x1 pF p3, n ´ 1q ` xn F p3, n ´ 2qq
“ pF pn ´ 1, 2q ` x1 F pn ´ 1, 3qq ` xn pF pn ´ 2, 2q ` x1 F pn ´ 2, 3qq
“ F pn ´ 1, 1q ` xn F pn ´ 2, 1q “ F p1, n ´ 1q ` xn F p1, n ´ 2q
“ F p1, nq,
as we wished to show.
Comment 1. These sequences are related to the Fibonacci sequence; when a1 “ ¨ ¨ ¨ “ an´1 “ 1, we
have uk “ vk “ Fk`1 , the pk ` 1qst Fibonacci number. Also, for every positive integer k, the polynomial
Pk px1 , . . . , xk´1 q from Solution 2 is the sum of Fk`1 monomials.
Let n be a positive integer and let a1 , . . . , an´1 be arbitrary real numbers. Define the sequences
u0 , . . . , un and v0 , . . . , vn inductively by u0 “ v0 “ 0, u1 “ v1 “ 1, and
Prove that un “ vn .
All three solutions above can be reformulated to prove this statement; one may prove
ÿ
u n “ vn “ ai1 . . . ait´1 for n ą 0
0“i0 ăi1 ă...ăit “n,
ij`1 ´ij is odd
or observe that
ˆ ˙ ˆ ˙ˆ ˙ ˙ ˆ
uk`1 ak 1 uk ak 1
“ and pvk`1 ; vk q “ pvk ; vk´1 q .
uk 1 0 uk´1 1 0
Here we have
uk`1 1
“ ak ` “ rak ; ak´1 , . . . , a1 s
uk 1
ak´1 `
1
ak´2 ` . . . `
a1
and
vk`1 1
“ an´k ` “ ran´k ; an´k`1 , . . . , an´1 s,
vk 1
an´k`1 `
1
an´k`2 ` . . . `
an´1
so this alternative statement is equivalent to the known fact that the continued fractions ran´1 ; an´2 , . . . , a1 s
and ra1 ; a2 , . . . , an´1 s have the same numerator.
Shortlisted problems – solutions 11
A2. Prove that in any set of 2000 distinct real numbers there exist two pairs a ą b and c ą d
with a ‰ c or b ‰ d, such that ˇ ˇ
ˇa ´ b ˇ 1
ˇ
ˇc ´ d ´ 1 ˇă
ˇ 100000 .
(Lithuania)
Solution. For any set S of n “ 2000 distinct real numbers, let D1 ď D2 ď ¨ ¨ ¨ ď Dm be the
distances between them, displayed with their multiplicities. Here m “ npn ´ 1q{2. By rescaling
the numbers, we may assume that the smallest distance D1 between two elements of S is D1 “ 1.
Let D1 “ 1 “ y ´ x for x, y P S. Evidently Dm “ v ´ u is the difference between the largest
element v and the smallest element u of S.
If Di`1 {Di ă 1 ` 10´5 for some i “ 1, 2, . . . , m ´ 1 then the required inequality holds, because
0 ď Di`1 {Di ´ 1 ă 10´5 . Otherwise, the reverse inequality
Di`1 1
ě1` 5
Di 10
holds for each i “ 1, 2, . . . , m ´ 1, and therefore
ˆ ˙m´1
Dm Dm D 3 D2 1
v ´ u “ Dm “ “ ¨¨¨ ¨ ě 1` 5 .
D1 Dm´1 D 2 D1 10
˘n´ 1 “ npn
`From 1m `n˘´ 1q{2 ´ 1 “ 1000 ¨ 1999 ´ 1 ą 19 ¨ 105 , together with the fact that for all n ě 1,
1 ` n ě 1 ` 1 ¨ n1 “ 2, we get
ˆ ˙19¨105 ˜ˆ ˙105 ¸19
1 1
1` 5 “ 1` 5 ě 219 “ 29 ¨ 210 ą 500 ¨ 1000 ą 2 ¨ 105 ,
10 10
and so v ´ u “ Dm ą 2 ¨ 105 .
Since the distance of x to at least one of the numbers u, v is at least pu ´ vq{2 ą 105 , we have
|x ´ z| ą 105 .
for some z P tu, vu. Since y ´ x “ 1, we have either z ą y ą x (if z “ v) or y ą x ą z (if z “ u).
If z ą y ą x, selecting a “ z, b “ y, c “ z and d “ x (so that b ‰ d), we obtain
ˇ ˇ ˇ ˇ ˇ ˇ
ˇa ´ b ˇ ˇz ´ y ˇ ˇx ´ y ˇ 1 ´5
ˇ c ´ d ´ 1ˇ “ ˇ z ´ x ´ 1ˇ “ ˇ z ´ x ˇ “ z ´ x ă 10 .
ˇ ˇ ˇ ˇ ˇ ˇ
1
Comment. As the solution shows, the numbers 2000 and 100000 appearing in the statement of the problem
may be replaced by any n P Zą0 and δ ą 0 satisfying
δp1 ` δqnpn´1q{2´1 ą 2.
12 IMO 2013 Colombia
A3. Let Qą0 be the set of positive rational numbers. Let f : Qą0 Ñ R be a function satisfying
the conditions
for all x, y P Qą0 . Given that f paq “ a for some rational a ą 1, prove that f pxq “ x for all
x P Qą0 .
(Bulgaria)
Solution. Denote by Zą0 the set of positive integers.
Plugging x “ 1, y “ a into (1) we get f p1q ě 1. Next, by an easy induction on n we get
from (2) that
f pnxq ě nf pxq for all n P Zą0 and x P Qą0 . (3)
In particular, we have
f pnq ě nf p1q ě n for all n P Zą0 . (4)
From (1) again we have f pm{nqf pnq ě f pmq, so f pqq ą 0 for all q P Qą0 .
Now, (2) implies that f is strictly increasing; this fact together with (4) yields
This yields
f pxq ě x for every x ą 1. (5)
(Indeed, if x ą y ą 1 then xn ´ y n “ px ´ yqpxn´1 ` xn´2 y ` ¨ ¨ ¨ ` y n q ą npx ´ yq, so for a large n
we have xn ´ 1 ą y n and thus f pxq ą y.)
Now, (1) and (5) give an “ f paqn ě f pan q ě an , so f pan q “ an . Now, for x ą 1 let us choose
n P Zą0 such that an ´ x ą 1. Then by (2) and (5) we get
and therefore f pxq “ x for x ą 1. Finally, for every x P Qą0 and every n P Zą0 , from (1) and (3)
we get
nf pxq “ f pnqf pxq ě f pnxq ě nf pxq,
which gives f pnxq “ nf pxq. Therefore f pm{nq “ f pmq{n “ m{n for all m, n P Zą0 .
Comment. The condition f paq “ a ą 1 is essential. Indeed, for b ě 1 the function f pxq “ bx2 satisfies (1)
and (2) for all x, y P Qą0 , and it has a unique fixed point 1{b ď 1.
Shortlisted problems – solutions 13
a1 ď a2 ď ¨ ¨ ¨ ď an ď a1 ` n (1)
and
aai ď n ` i ´ 1 for i “ 1, 2, . . . , n, (2)
prove that
a1 ` ¨ ¨ ¨ ` an ď n2 .
(Germany)
Solution 1. First, we claim that
ai ď n ` i ´ 1 for i “ 1, 2, . . . , n. (3)
Thus our assumption that ai ě n ` i implies the stronger statement that ai ě 2n ` 1, which by
a1 ` n ě an ě ai gives a1 ě n ` 1. The minimality of i then yields i “ 1, and (4) becomes
contradictory. This establishes our first claim.
In particular we now know that a1 ď n. If an ď n, then a1 ď ¨ ¨ ¨ ď ¨ ¨ ¨ an ď n and the desired
inequality holds trivially. Otherwise, consider the number t with 1 ď t ď n ´ 1 such that
a1 ď a2 ď . . . ď at ď n ă at`1 ď . . . ď an . (5)
Since 1 ď a1 ď n and aa1 ď n by (2), we have a1 ď t and hence an ď n ` t. Therefore if for each
positive integer i we let bi be the number of indices j P tt ` 1, . . . , nu satisfying aj ě n ` i, we have
b1 ě b2 ě . . . ě bt ě bt`1 “ 0.
at`1 ` . . . ` an ď npn ´ tq ` b1 ` . . . ` bt .
a1 ` a2 ` ¨ ¨ ¨ ` an ď npn ´ tq ` nt “ n2
as we wished to prove.
14 IMO 2013 Colombia
Solution 2. In the first quadrant of an infinite grid, consider the increasing “staircase” obtained
by shading in dark the bottom ai cells of the ith column for 1 ď i ď n. We will prove that there
are at most n2 dark cells.
To do it, consider the n ˆ n square S in the first quadrant with a vertex at the origin. Also
consider the n ˆ n square directly to the left of S. Starting from its lower left corner, shade in light
the leftmost aj cells of the jth row for 1 ď j ď n. Equivalently, the light shading is obtained by
reflecting the dark shading across the line x “ y and translating it n units to the left. The figure
below illustrates this construction for the sequence 6, 6, 6, 7, 7, 7, 8, 12, 12, 14.
a ai
ai
n+i−1 i
We claim that there is no cell in S which is both dark and light. Assume, contrariwise, that
there is such a cell in column i. Consider the highest dark cell in column i which is inside S. Since
it is above a light cell and inside S, it must be light as well. There are two cases:
Case 1. ai ď n
If ai ď n then this dark and light cell is pi, ai q, as highlighted in the figure. However, this is the
pn ` iq-th cell in row ai , and we only shaded aai ă n ` i light cells in that row, a contradiction.
Case 2. ai ě n ` 1
If ai ě n ` 1, this dark and light cell is pi, nq. This is the pn ` iq-th cell in row n and we shaded
an ď a1 ` n light cells in this row, so we must have i ď a1 . But a1 ď aa1 ď n by (1) and (2), so
i ď a1 implies ai ď aa1 ď n, contradicting our assumption.
We conclude that there are no cells in S which are both dark and light. It follows that the
number of shaded cells in S is at most n2 .
Finally, observe that if we had a light cell to the right of S, then by symmetry we would have
a dark cell above S, and then the cell pn, nq would be dark and light. It follows that the number
of light cells in S equals the number of dark cells outside of S, and therefore the number of shaded
cells in S equals a1 ` ¨ ¨ ¨ ` an . The desired result follows.
which allows us to construct the following visualization: Consider n equally spaced points on a
circle, sequentially labelled 1, 2, . . . , n pmod nq, so point k is also labelled n ` k. We draw arrows
from vertex i to vertices i ` 1, . . . , ci for 1 ď i ď n, keeping in mind that ci ě i by (6). Since
ci ď n ` i ´ 1 by (3), no arrow will be drawn twice, and there is no arrow from a vertex to itself.
The total number of arrows is
n n ˆ ˙
ÿ ÿ n`1
number of arrows “ pci ´ iq “ ci ´
i“1 i“1
2
Now we show that we never draw both arrows i Ñ j and j Ñ i for 1 ď i ă j ď n. Assume
contrariwise. This means, respectively, that
i ă j ď ci and j ă n ` i ď cj .
n ˆ ˙ ˆ ˙
ÿ n n`1
ci ď ` “ n2 .
i“1
2 2
Comment 1. We sketch an alternative proof by induction. Begin by verifying the initial case n “ 1 and
the simple cases when a1 “ 1, a1 “ n, or an ď n. Then, as in Solution 1, consider the index t such that
a1 ď ¨ ¨ ¨ ď at ď n ă at`1 ď ¨ ¨ ¨ ď an . Observe again that a1 ď t. Define the sequence d1 , . . . , dn´1 by
#
ai`1 ´ 1 if i ď t ´ 1
di “
ai`1 ´ 2 if i ě t
and extend it periodically modulo n ´ 1. One may verify that this sequence also satisfies the hypotheses
of the problem. The induction hypothesis then gives d1 ` ¨ ¨ ¨ ` dn´1 ď pn ´ 1q2 , which implies that
n
ÿ t
ÿ n
ÿ
ai “ a1 ` pdi´1 ` 1q ` pdi´1 ` 2q ď t ` pt ´ 1q ` 2pn ´ tq ` pn ´ 1q2 “ n2 .
i“1 i“2 i“t`1
Comment 2. One unusual feature of this problem is that there are many different sequences for which
equality holds. The discovery of such optimal sequences is not difficult, and it is useful in guiding the
steps of a proof.
In fact, Solution 2 gives a complete description of the optimal sequences. Start with any lattice path
P from the lower left to the upper right corner of the n ˆ n square S using only steps up and right, such
that the total number of steps along the left and top edges of S is at least n. Shade the cells of S below
P dark, and the cells of S above P light. Now reflect the light shape across the line x “ y and shift it
up n units, and shade it dark. As Solution 2 shows, the dark region will then correspond to an optimal
sequence, and every optimal sequence arises in this way.
16 IMO 2013 Colombia
A5. Let Zě0 be the set of all nonnegative integers. Find all the functions f : Zě0 Ñ Zě0
satisfying the relation
f pf pf pnqqq “ f pn ` 1q ` 1 p˚q
for all n P Zě0 .
(Serbia)
Answer. There are two such functions: f pnq “ n ` 1 for all n P Zě0 , and
$
&n ` 1,
’ n ” 0 pmod 4q or n ” 2 pmod 4q,
f pnq “ n ` 5, n ” 1 pmod 4q, for all n P Zě0 . (1)
’
n ´ 3, n ” 3 pmod 4q
%
Throughout all the solutions, we write hk pxq to abbreviate the kth iteration of function h, so h0 is
the identity function, and hk pxq “ loomoon
hp. . . hpxq . . . qq for k ě 1.
k times
thus
f 4 pnq ` 1 “ f 4 pn ` 1q. (2)
I. Let us denote by Ri the range of f i ; note that R0 “ Zě0 since f 0 is the identity function.
Obviously, R0 Ě R1 Ě . . . . Next, from (2) we get that if a P R4 then also a ` 1 P R4 . This implies
that Zě0 zR4 — and hence Zě0 zR1 — is finite. In particular, R1 is unbounded.
Assume that f pmq “ f pnq for some distinct m and n. Then from p˚q we obtain f pm ` 1q “
f pn ` 1q; by an easy induction we then get that f pm ` cq “ f pn ` cq for every c ě 0. So the
function f pkq is periodic with period |m ´ n| for k ě m, and thus R1 should be bounded, which is
false. So, f is injective.
II. Denote now Si “ Ri´1 zRi ; all these sets are finite for i ď 4. On the other hand, by the
injectivity we have n P Si ðñ f pnq P Si`1 . By the injectivity again, f implements a bijection
between Si and Si`1 , thus |S1 | “ |S2 | “ . . . ; denote this common cardinality by k. If 0 P R3 then
0 “ f pf pf pnqqq for some n, thus from p˚q we get f pn ` 1q “ ´1 which is impossible. Therefore
0 P R0 zR3 “ S1 Y S2 Y S3 , thus k ě 1.
Next, let us describe the elements b of R0 zR3 “ S1 Y S2 Y S3. We claim that each such element
satisfies at least one of three conditions piq b “ 0, piiq b “ f p0q ` 1, and piiiq b ´ 1 P S1 . Otherwise
b ´ 1 P Zě0 , and there exists some n ą 0 such that f pnq “ b ´ 1; but then f 3 pn ´ 1q “ f pnq ` 1 “ b,
so b P R3 .
This yields
3k “ |S1 Y S2 Y S3 | ď 1 ` 1 ` |S1 | “ k ` 2,
or k ď 1. Therefore k “ 1, and the inequality above comes to equality. So we have S1 “ tau,
S2 “ tf paqu, and S3 “ tf 2 paqu for some a P Zě0 , and each one of the three options piq, piiq,
and piiiq should be realized exactly once, which means that
III. From (3), we get a ` 1 P tf paq, f 2 paqu (the case a ` 1 “ a is impossible). If a ` 1 “ f 2 paq then
we have f pa ` 1q “ f 3 paq “ f pa ` 1q ` 1 which is absurd. Therefore
f paq “ a ` 1. (4)
Next, again from (3) we have 0 P ta, f 2 paqu. Let us consider these two cases separately.
Case 1. Assume that a “ 0, then f p0q “ f paq “ a ` 1 “ 1. Also from (3) we get f p1q “ f 2 paq “
f p0q ` 1 “ 2. Now, let us show that f pnq “ n ` 1 by induction on n; the base cases n ď 1 are
established. Next, if n ě 2 then the induction hypothesis implies
n ` 1 “ f pn ´ 1q ` 1 “ f 3 pn ´ 2q “ f 2 pn ´ 1q “ f pnq,
establishing the step. In this case we have obtained the first of two answers; checking that is
satisfies p˚q is straightforward.
Case 2. Assume now that f 2 paq “ 0; then by (3) we get a “ f p0q ` 1. By (4) we get f pa ` 1q “
f 2 paq “ 0, then f p0q “ f 3 paq “ f pa ` 1q ` 1 “ 1, hence a “ f p0q ` 1 “ 2 and f p2q “ 3 by (4). To
summarize,
f p0q “ 1, f p2q “ 3, f p3q “ 0.
Now let us prove by induction on m that (1) holds for all n “ 4k, 4k ` 2, 4k ` 3 with k ď m and
for all n “ 4k ` 1 with k ă m. The base case m “ 0 is established above. For the step, assume
that m ě 1. From p˚q we get f 3 p4m ´ 3q “ f p4m ´ 2q ` 1 “ 4m. Next, by (2) we have
Solution 2. I. For convenience, let us introduce the function gpnq “ f pnq ` 1. Substituting f pnq
instead of n into p˚q we obtain
f 4 pnq ` 1 “ f f pn ` 1q ` 1 ` 1 “ f 4 pn ` 1q.
` ˘
(6)
This relation implies that both f and g are injective: if, say, f pmq “ f pnq then m ` c “
f pmq “ f 4 pnq “ n ` c. Next, since gpnq ě 1 for every n, we have c “ g 2 p0q ě 1. Thus from (7)
4
Using (8), we get that for every complete residue system n1 , . . . , nc modulo c we also have
c
ÿ
S“ δpni q.
i“1
and similarly
c´1
ÿ 1 c´1 1 c´1
˘ ÿ ÿ` ˘ ÿ ÿ`
c2 “ g 2pnq ´ n “ g k`1 pnq ´ g k pnq “ δpg k pnqq ` 1 “ 2S ` 2c.
` ˘
n“0 k“0 n“0 k“0 n“0
A6. Let m ‰ 0 be an integer. Find all polynomials P pxq with real coefficients such that
px3 ´ mx2 ` 1qP px ` 1q ` px3 ` mx2 ` 1qP px ´ 1q “ 2px3 ´ mx ` 1qP pxq (1)
xpx3 ´ mx2 ` 1qP px ` 1q ` xpx3 ` mx2 ` 1qP px ´ 1q “ rpx ` 1q ` px ´ 1qs px3 ´ mx ` 1qP pxq.
where Qpxq “ xP px ` 1q ´ px ` 1qP pxq. If deg P ě 2 then deg Q “ deg P , so Qpxq has a finite
multiset of complex roots, which we denote RQ . Each root is taken with its multiplicity. Then the
multiset of complex roots of Qpx ´ 1q is RQ ` 1 “ tz ` 1 : z P RQ u.
20 IMO 2013 Colombia
Let tx1 , x2 , x3 u and ty1 , y2 , y3 u be the multisets of roots of the polynomials Apxq “ x3 ´ mx2 ` 1
and Bpxq “ x3 ` mx2 ` 1, respectively. From (2) we get the equality of multisets
For every r P RQ , since r ` 1 is in the set of the right hand side, we must have r ` 1 P RQ or
r ` 1 “ xi for some i. Similarly, since r is in the set of the left hand side, either r ´ 1 P RQ or
r “ yi for some i. This implies that, possibly after relabelling y1 , y2 , y3 , all the roots of (2) may
be partitioned into three chains of the form tyi , yi ` 1, . . . , yi ` ki “ xi u for i “ 1, 2, 3 and some
integers k1 , k2, k3 ě 0.
Now we analyze the roots of the polynomial Aa pxq “ x3 ` ax2 ` 1. Using calculus or elementary
methods, we find that the local extrema of Aa pxq occur at x “ 0 and x “ ´2a{3; their values are
Aa p0q “ 1 ą 0 and Aa p´2a{3q “ 1 ` 4a3 {27, which is positive for integers a ě ´1 and negative for
integers a ď ´2. So when a P Z, Aa has three real roots if a ď ´2 and one if a ě ´1.
Now, since yi ´ xi P Z for i “ 1, 2, 3, the cubics Am and A´m must have the same number of
real roots. The previous analysis then implies that m “ 1 or m “ ´1. Therefore the real root α of
A1 pxq “ x3 ` x2 ` 1 and the 3 2
˘ root1 β of A´1 pxq “ x ´ x ` 1 must differ by an integer. But this
` real
3
is impossible,
` 1 ˘ because A1 ´ 2 “ ´ 8 and A1 p´1q “ 1 so ´1.5 ă α ă ´1, while A´1 p´1q “ ´1
5
and A´1 ´ 2 “ 8 , so ´1 ă β ă ´0.5.
It follows that deg P ď 1. Then, as shown in Solution 1, we conclude that the solutions are
P pxq “ tx for all real numbers t.
Shortlisted problems – solutions 21
Combinatorics
C1. Let n be a positive integer. Find the smallest integer k with the following property: Given
any real numbers a1 , . . . , ad such that a1 ` a2 ` ¨ ¨ ¨ ` ad “ n and 0 ď ai ď 1 for i “ 1, 2, . . . , d, it
is possible to partition these numbers into k groups (some of which may be empty) such that the
sum of the numbers in each group is at most 1.
(Poland)
Answer. k “ 2n ´ 1.
Solution 1. If d “ 2n ´ 1 and a1 “ ¨ ¨ ¨ “ a2n´1 “ n{p2n ´ 1q, then each group in such a partition
can contain at most one number, since 2n{p2n ´ 1q ą 1. Therefore k ě 2n ´ 1. It remains to show
that a suitable partition into 2n ´ 1 groups always exists.
We proceed by induction on d. For d ď 2n ´ 1 the result is trivial. If d ě 2n, then since
we may find two numbers ai , ai`1 such that ai ` ai`1 ď 1. We “merge” these two numbers into
one new number ai ` ai`1 . By the induction hypothesis, a suitable partition exists for the d ´ 1
numbers a1 , . . . , ai´1 , ai ` ai`1 , ai`2 , . . . , ad . This induces a suitable partition for a1 , . . . , ad .
Solution 2. We will show that it is even possible to split the sequence a1 , . . . , ad into 2n ´ 1
contiguous groups so that the sum of the numbers in each groups does not exceed 1. Consider a
segment S of length n, and partition it into segments S1 , . . . , Sd of lengths a1 , . . . , ad , respectively,
as shown below. Consider a second partition of S into n equal parts by n ´ 1 “empty dots”.
a1 a2 a3 a4 a5 a6 a7 a8 a9 a10
Assume that the n ´ 1 empty dots are in segments Si1 , . . . , Sin´1 . (If a dot is on the boundary
of two segments, we choose the right segment). These n ´ 1 segments are distinct because they
have length at most 1. Consider the partition:
In the example above, this partition is ta1 , a2 u, ta3 u, ta4 , a5 u, ta6 u, H, ta7 u, ta8 , a9 , a10 u. We claim
that in this partition, the sum of the numbers in this group is at most 1.
For the sets tait u this is obvious since ait ď 1. For the sets tait ` 1, . . . , ait`1 ´1 u this follows
from the fact that the corresponding segments lie between two neighboring empty dots, or between
an endpoint of S and its nearest empty dot. Therefore the sum of their lengths cannot exceed 1.
Solution 3. First put all numbers greater than 21 in their own groups. Then, form the remaining
groups as follows: For each group, add new ai s one at a time until their sum exceeds 12 . Since the
last summand is at most 12 , this group has sum at most 1. Continue this procedure until we have
used all the ai s. Notice that the last group may have sum less than 21 . If the sum of the numbers
in the last two groups is less than or equal to 1, we merge them into one group. In the end we are
left with m groups. If m “ 1 we are done. Otherwise the first m ´ 2 have sums greater than 12 and
the last two have total sum greater than 1. Therefore n ą pm ´ 2q{2 ` 1 so m ď 2n ´ 1 as desired.
22 IMO 2013 Colombia
Comment 1. The original proposal asked for the minimal value of k when n “ 2.
Comment 2. More generally, one may ask the same question for real numbers between 0 and 1 whose
sum is a real number r. In this case the smallest value of k is k “ r2rs ´ 1, as Solution 3 shows.
Solutions 1 and 2 lead to the slightly weaker bound k ď 2rrs ´ 1. This is actually the optimal bound
for partitions into consecutive groups, which are the ones contemplated in these two solutions. To see
this, assume that r is not an integer and let c “ pr ` 1 ´ rrsq{p1 ` rrsq. One easily checks that 0 ă c ă 21
and rrsp2cq ` prrs ´ 1qp1 ´ cq “ r, so the sequence
2c, 1 ´ c, 2c, 1 ´ c, . . . , 1 ´ c, 2c
of 2rrs ´ 1 numbers satisfies the given conditions. For this sequence, the only suitable partition into
consecutive groups is the trivial partition, which requires 2rrs ´ 1 groups.
Shortlisted problems – solutions 23
C2. In the plane, 2013 red points and 2014 blue points are marked so that no three of the
marked points are collinear. One needs to draw k lines not passing through the marked points and
dividing the plane into several regions. The goal is to do it in such a way that no region contains
points of both colors.
Find the minimal value of k such that the goal is attainable for every possible configuration of
4027 points.
(Australia)
Answer. k “ 2013.
Solution 1. Firstly, let us present an example showing that k ě 2013. Mark 2013 red and 2013
blue points on some circle alternately, and mark one more blue point somewhere in the plane. The
circle is thus split into 4026 arcs, each arc having endpoints of different colors. Thus, if the goal is
reached, then each arc should intersect some of the drawn lines. Since any line contains at most
two points of the circle, one needs at least 4026{2 “ 2013 lines.
It remains to prove that one can reach the goal using 2013 lines. First of all, let us mention
that for every two points A and B having the same color, one can draw two lines separating these
points from all other ones. Namely, it suffices to take two lines parallel to AB and lying on different
sides of AB sufficiently close to it: the only two points between these lines will be A and B.
Now, let P be the convex hull of all marked points. Two cases are possible.
Case 1. Assume that P has a red vertex A. Then one may draw a line separating A from all the
other points, pair up the other 2012 red points into 1006 pairs, and separate each pair from the
other points by two lines. Thus, 2013 lines will be used.
Case 2. Assume now that all the vertices of P are blue. Consider any two consecutive vertices
of P , say A and B. One may separate these two points from the others by a line parallel to AB.
Then, as in the previous case, one pairs up all the other 2012 blue points into 1006 pairs, and
separates each pair from the other points by two lines. Again, 2013 lines will be used.
Comment 1. Instead of considering the convex hull, one may simply take a line containing two marked
points A and B such that all the other marked points are on one side of this line. If one of A and B is
red, then one may act as in Case 1; otherwise both are blue, and one may act as in Case 2.
Solution 2. Let us present a different proof of the fact that k “ 2013 suffices. In fact, we will
prove a more general statement:
If n points in the plane, no three of which are collinear, are colored in red and blue arbitrarily,
then it suffices to draw tn{2u lines to reach the goal.
We proceed by induction on n. If n ď 2 then the statement is obvious. Now assume that n ě 3,
and consider a line ℓ containing two marked points A and B such that all the other marked points
are on one side of ℓ; for instance, any line containing a side of the convex hull works.
Remove for a moment the points A and B. By the induction hypothesis, for the remaining
configuration it suffices to draw tn{2u ´ 1 lines to reach the goal. Now return the points A and B
back. Three cases are possible.
Case 1. If A and B have the same color, then one may draw a line parallel to ℓ and separating A
and B from the other points. Obviously, the obtained configuration of tn{2u lines works.
Case 2. If A and B have different colors, but they are separated by some drawn line, then again
the same line parallel to ℓ works.
24 IMO 2013 Colombia
Case 3. Finally, assume that A and B have different colors and lie in one of the regions defined by
the drawn lines. By the induction assumption, this region contains no other points of one of the
colors — without loss of generality, the only blue point it contains is A. Then it suffices to draw
a line separating A from all other points.
Thus the step of the induction is proved.
Comment 2. One may ask a more general question, replacing the numbers 2013 and 2014 by any
positive integers m and n, say with m ď n. Denote the answer for this problem by f pm, nq.
One may show along the lines of Solution 1 that m ď f pm, nq ď m ` 1; moreover, if m is even then
f pm, nq “ m. On the other hand, for every odd m there exists an N such that f pm, nq “ m for all
m ď n ď N , and f pm, nq “ m ` 1 for all n ą N .
Shortlisted problems – solutions 25
C3. A crazy physicist discovered a new kind of particle which he called an imon, after some of
them mysteriously appeared in his lab. Some pairs of imons in the lab can be entangled, and each
imon can participate in many entanglement relations. The physicist has found a way to perform
the following two kinds of operations with these particles, one operation at a time.
piq If some imon is entangled with an odd number of other imons in the lab, then the physicist
can destroy it.
piiq At any moment, he may double the whole family of imons in his lab by creating a copy I 1
of each imon I. During this procedure, the two copies I 1 and J 1 become entangled if and only if
the original imons I and J are entangled, and each copy I 1 becomes entangled with its original
imon I; no other entanglements occur or disappear at this moment.
Prove that the physicist may apply a sequence of such operations resulting in a family of imons,
no two of which are entangled.
(Japan)
Solution 1. Let us consider a graph with the imons as vertices, and two imons being connected
if and only if they are entangled. Recall that a proper coloring of a graph G is a coloring of its
vertices in several colors so that every two connected vertices have different colors.
Lemma. Assume that a graph G admits a proper coloring in n colors (n ą 1). Then one may
perform a sequence of operations resulting in a graph which admits a proper coloring in n ´ 1
colors.
Proof. Let us apply repeatedly operation piq to any appropriate vertices while it is possible. Since
the number of vertices decreases, this process finally results in a graph where all the degrees are
even. Surely this graph also admits a proper coloring in n colors 1, . . . , n; let us fix this coloring.
Now apply the operation piiq to this graph. A proper coloring of the resulting graph in n
colors still exists: one may preserve the colors of the original vertices and color the vertex I 1 in
a color k ` 1 pmod nq if the vertex I has color k. Then two connected original vertices still have
different colors, and so do their two connected copies. On the other hand, the vertices I and I 1
have different colors since n ą 1.
All the degrees of the vertices in the resulting graph are odd, so one may apply operation piq
to delete consecutively all the vertices of color n one by one; no two of them are connected by
an edge, so their degrees do not change during the process. Thus, we obtain a graph admitting a
proper coloring in n ´ 1 colors, as required. The lemma is proved. l
Now, assume that a graph G has n vertices; then it admits a proper coloring in n colors.
Applying repeatedly the lemma we finally obtain a graph admitting a proper coloring in one color,
that is — a graph with no edges, as required.
Otherwise, the operation is of type piiq, and one may apply it to G and then delete the vertex A1
(it will have degree 1).
Thus one may change the process for G˝ into a corresponding process for G step by step. l
In view of this lemma, if at some moment a graph contains some isolated vertex, then we may
simply delete it; let us call this operation piiiq.
II. Let V “ tA01 , . . . , A0n u be the vertices of the initial graph. Let us describe which graphs can
appear during our operations. Assume that operation piiq was applied m times. If these were
the only operations applied, then the resulting graph Gm n has the set of vertices which can be
enumerated as
Vnm “ tAji : 1 ď i ď n, 0 ď j ď 2m ´ 1u,
where A0i is the common “ancestor” of all the vertices Aji , and the binary expansion of j (adjoined
with some zeroes at the left to have m digits) “keeps the history” of this vertex: the dth digit from
the right is 0 if at the dth doubling the ancestor of Aji was in the original part, and this digit is 1
if it was in the copy.
Next, the two vertices Aji and Aℓk in Gm
n are connected with an edge exactly if either (1) j “ ℓ
and there was an edge between A0i and A0k (so these vertices appeared at the same application of
operation piiq); or (2) i “ k and the binary expansions of j and ℓ differ in exactly one digit (so
their ancestors became connected as a copy and the original vertex at some application of piiq).
Now, if some operations piq were applied during the process, then simply some vertices in Gm n
disappeared. So, in any case the resulting graph is some induced subgraph of Gm n.
III. Finally, we will show that from each (not necessarily induced) subgraph of Gm n one can obtain
a graph with no vertices by applying operations piq, piiq and piiiq. We proceed by induction on n;
the base case n “ 0 is trivial.
For the induction step, let us show how to apply several operations so as to obtain a graph
containing no vertices of the form Ajn for j P Z. We will do this in three steps.
Step 1. We apply repeatedly operation piq to any appropriate vertices while it is possible. In the
resulting graph, all vertices have even degrees.
Step 2. Apply operation piiq obtaining a subgraph of Gm`1 n with all degrees being odd. In this
j
graph, we delete one by one all the vertices An where the sum of the binary digits of j is even; it
is possible since there are no edges between such vertices, so all their degrees remain odd. After
that, we delete all isolated vertices.
Step 3. Finally, consider any remaining vertex Ajn (then the sum of digits of j is odd). If its
degree is odd, then we simply delete it. Otherwise, since Ajn is not isolated, we consider any vertex
adjacent to it. It has the form Ajk for some k ă n (otherwise it would have the form Aℓn , where ℓ
has an even digit sum; but any such vertex has already been deleted at Step 2). No neighbor of Ajk
was deleted at Steps 2 and 3, so it has an odd degree. Then we successively delete Ajk and Ajn .
Notice that this deletion does not affect the applicability of this step to other vertices, since
no two vertices Aji and Aℓk for different j, ℓ with odd digit sum are connected with an edge. Thus
we will delete all the remaining vertices of the form Ajn , obtaining a subgraph of Gm`1 n´1 . The
application of the induction hypothesis finishes the proof.
C4. Let n be a positive integer, and let A be a subset of t1, . . . , nu. An A-partition of n into k
parts is a representation of n as a sum n “ a1 ` ¨ ¨ ¨ ` ak , where the parts a1 , . . . , ak belong to A
and are not necessarily distinct. The number of different parts in such a partition is the number
of (distinct) elements in the set ta1 , a2 , . . . , ak u.
We say that an A-partition of n into k parts is optimal if there is no A-partition
? of n into r
parts with r ă k. Prove that any optimal A-partition of n contains at most 3 6n different parts.
(Germany)
Solution 1. If there are no A-partitions of n, the result is vacuously true. Otherwise, let kmin
be the minimum number of parts in an A-partition of n, and let n “ a1 ` ¨ ¨ ¨ ` akmin be an
optimal partition. Denote by s the number of different parts in this partition, so we can write
S “ ta1 , . . ?
. , akmin u “ tb1 , . . . , bs u for some pairwise different numbers b1 ă ¨ ¨ ¨ ă bs in A.
3
ř If s ą ř 6n, we will prove that there exist subsets X and Y of S such that |X| ă |Y | and
xPX x “ yPY y. Then, deleting the elements of Y from our partition and adding the elements of
X to it, we obtain an A-partition of n into less than kmin parts, which is the desired contradiction.
For each positive integer k ď s, we consider the k-element subset
k
S1,0 :“ tb1 , . . . , bk u
k
as well as the following k-element subsets Si,j of S:
k
(
Si,j :“ b1 , . . . , bk´i , bk´i`j`1 , bs´i`2 , . . . , bs , i “ 1, . . . , k, j “ 1, . . . , s ´ k.
Pictorially, if we represent the elements of S by a sequence of dots in increasing order, and represent
a subset of S by shading in the appropriate dots, we have:
k
Si,j “ looooomooooon ˝ ˝ ˝ ˝ ˝ ˝ ˝ ‚looooooomooooooon
˝ ˝ ˝ ˝ ‚ looooomooooon
‚ ‚ ‚ ‚ ‚ ‚ ‚ ˝looomooon ‚‚‚‚‚‚‚‚
k´i j s´k´j i´1
Solution 2. Assume, to the contrary, that the statement is false, and choose the minimum
number n for which it fails. So there exists a set A Ď t1, . . . , nu together with an optimal A-
partition n “ a1 ` ¨ ¨ ¨ ` akmin of n refuting our statement, where, of course, kmin is the minimum
number of parts in an A-partition of n. Again, ? we define S “ ta1 , . . . , akmin u “ tb1 , . . . , bs u with
b1 ă ¨ ¨ ¨ ă bs ; by our assumption we have s ą 3 6n ą 1. Without loss of generality we assume
that akmin “ bs . Let us distinguish two cases.
Case 1. bs ě sps´1q
2
` 1.
Consider the partition n ´ bs “ a1 ` ¨ ¨ ¨ ` akmin ´1 , which is clearly a minimum A-partition
3
of n ´ bs with at least s ´ 1 ě 1 different parts. Now, from n ă s6 we obtain
0 ă Σi,k ´ Σi,j “ bk ´ bj ă bs ,
so the indices i and i1 are distinct, and we may assume that i ą i1 . Next, we observe that
Σi,j ´ Σi1 ,j 1 “ pbi1 ´ bj 1 q ` bj ` bi1 `1 ` ¨ ¨ ¨ ` bi´1 and bi1 ď bj 1 imply
so 0 ď r ď i ´ i1 ´ 1.
Thus, we may remove the i terms of Σi,j in our A-partition, and replace them by the i1 terms
of Σi1 ,j 1 and r terms equal to bs , for a total of r ` i1 ă i terms. The result is an A-partition of n
into a smaller number of parts, a contradiction.
Comment. The original proposal also contained a second part, showing that the estimate appearing in
the problem has the correct order of magnitude:
?
For every positive integer n, there exist a set A and an optimal A-partition of n that contains t 3 2nu
different parts.
The Problem Selection Committee removed this statement from the problem, since it seems to be less
suitable for the? competiton; but for completeness we provide an outline of its proof here.
Let k “ t 3 2nu ´ 1. The statement is trivial for n ă 4, so we assume n ě 4 and hence k ě 1. Let
h “ t n´1 n
k u. Notice that h ě k ´ 1.
Now let A “ t1, . . . , hu, and set a1 “ h, a2 “ h ´ 1, . . . , ak “ h ´ k ` 1, and ak`1 “ n ´ pa1 ` ¨ ¨ ¨ ` ak q.
It is not difficult to prove that ak ą ak`1 ě 1, which shows that
n “ a1 ` . . . ` ak`1
C5. Let r be a positive integer, and let a0 , a1 , . . . be an infinite sequence of real numbers.
Assume that for all nonnegative integers m and s there exists a positive integer n P rm ` 1, m ` rs
such that
am ` am`1 ` ¨ ¨ ¨ ` am`s “ an ` an`1 ` ¨ ¨ ¨ ` an`s .
Prove that the sequence is periodic, i. e. there exists some p ě 1 such that an`p “ an for all n ě 0.
(India)
Solution. For every indices m ď n we will denote Spm, nq “ am ` am`1 ` ¨ ¨ ¨ ` an´1 ; thus
Spn, nq “ 0. Let us start with the following lemma.
Lemma. Let b0 , b1 , . . . be an infinite sequence. Assume that for every nonnegative integer m there
exists a nonnegative integer n P rm ` 1, m ` rs such that bm “ bn . Then for every indices k ď ℓ
there exists an index t P rℓ, ℓ ` r ´ 1s such that bt “ bk . Moreover, there are at most r distinct
numbers among the terms of pbi q.
Proof. To prove the first claim, let us notice that there exists an infinite sequence of indices
k1 “ k, k2 , k3 , . . . such that bk1 “ bk2 “ ¨ ¨ ¨ “ bk and ki ă ki`1 ď ki ` r for all i ě 1. This sequence
is unbounded from above, thus it hits each segment of the form rℓ, ℓ`r ´1s with ℓ ě k, as required.
To prove the second claim, assume, to the contrary, that there exist r ` 1 distinct numbers
bi1 , . . . , bir`1 . Let us apply the first claim to k “ i1 , . . . , ir`1 and ℓ “ maxti1 , . . . , ir`1 u; we obtain
that for every j P t1, . . . , r ` 1u there exists tj P rs, s ` r ´ 1s such that btj “ bij . Thus the segment
rs, s ` r ´ 1s should contain r ` 1 distinct integers, which is absurd. l
Setting s “ 0 in the problem condition, we see that the sequence pai q satisfies the condi-
tion of the lemma, thus it attains at most r distinct values. Denote by Ai the ordered r-tuple
pai , . . . , ai`r´1q; then among Ai ’s there are at most r r distinct tuples, so for every k ě 0 two of the
tuples Ak , Ak`1 , . . . , Ak`rr are identical. This means that there exists a positive integer p ď r r such
that the equality Ad “ Ad`p holds infinitely many times. Let D be the set of indices d satisfying
this relation.
Now we claim that D coincides with the set of all nonnegative integers. Since D is unbounded,
it suffices to show that d P D whenever d ` 1 P D. For that, denote bk “ Spk, p ` kq. The
sequence b0 , b1 , . . . satisfies the lemma conditions, so there exists an index t P rd ` 1, d ` rs such
that Spt, t ` pq “ Spd, d ` pq. This last relation rewrites as Spd, tq “ Spd ` p, t ` pq. Since
Ad`1 “ Ad`p`1 , we have Spd ` 1, tq “ Spd ` p ` 1, t ` pq, therefore we obtain
Comment 1. In the present proof, the upper bound for the minimal period length is r r . This bound is
not sharp; for instance, one may improve it to pr ´ 1qr for r ě 3..
On the other hand, this minimal length may happen to be greater than r. For instance, it is easy to
check that the sequence with period p3, ´3, 3, ´3, 3, ´1, ´1, ´1q satisfies the problem condition for r “ 7.
Comment 2. The conclusion remains true even if the problem condition only holds for every s ě N for
some positive integer N . To show that, one can act as follows. Firstly, the sums of the form Spi, i ` N q
attain at most r values, as well as the sums of the form Spi, i`N `1q. Thus the terms ai “ Spi, i ` N ` 1q´
Spi ` 1, i ` N ` 1q attain at most r 2 distinct values. Then, among the tuples Ak , Ak`N , . . . , Ak`r2r N two
30 IMO 2013 Colombia
are identical, so for some p ď r 2r the set D “ td : Ad “ Ad`N p u is infinite. The further arguments apply
almost literally, with p being replaced by N p.
After having proved that such a sequence is also necessarily periodic, one may reduce the bound for
the minimal period length to r r — essentially by verifying that the sequence satisfies the original version
of the condition.
Shortlisted problems – solutions 31
C6. In some country several pairs of cities are connected by direct two-way flights. It is possible
to go from any city to any other by a sequence of flights. The distance between two cities is defined
to be the least possible number of flights required to go from one of them to the other. It is known
that for any city there are at most 100 cities at distance exactly three from it. Prove that there is
no city such that more than 2550 other cities have distance exactly four from it.
(Russia)
Solution. Let us denote by dpa, bq the distance between the cities a and b, and by
Si paq “ tc : dpa, cq “ iu
Comment 1. The upper bound 2550 is sharp. This can be seen by means of various examples; one of
them is the “Roman Empire”: it has one capital, called “Rome”, that is connected to 51 semicapitals by
internally disjoint paths of length 3. Moreover, each of these semicapitals is connected to 50 rural cities
by direct flights.
Comment 2. Observe that, under the conditions of the problem, there exists no bound for the size
of S1 pxq or S2 pxq.
32 IMO 2013 Colombia
Comment Y 3. 2The
] numbers 100 and 2550 appearing in the statement of the problem may be replaced
pn`1q
by n and 4 for any positive integer n. Still more generally, one can also replace the pair p3, 4q of
distances under consideration by any pair pr, sq of positive integers satisfying r ă s ď 32 r.
To adapt the above proof to this situation, one takes A “ Ss´r pxq and defines the concept of substan-
tiality as before. Then one takes A˚ to be a minimal substantial subset of A, and for each y P A˚ one
fixes an element dy P Ss pxq which is only reachable from x by a path of length s by passing through y.
As before, it suffices to show that for distinct a, y P A˚ and a path y “ y0 ´ y1 ´ . . . ´ yr “ dy , at least
one of the cities y0 , . . . , yr´1 has distance r from a. This can be done as above; the relation s ď 32 r is
used here to show that dpa, Yy0 q ď r.]
2
Moreover, the estimate pn`1q 4 is also sharp for every positive integer n and every positive integers
r, s with r ă s ď 23 r. This may be shown by an example similar to that in the previous comment.
Shortlisted problems – solutions 33
C7. Let n ě 2 be an integer. Consider all circular arrangements of the numbers 0, 1, . . . , n; the
n ` 1 rotations of an arrangement are considered to be equal. A circular arrangement is called
beautiful if, for any four distinct numbers 0 ď a, b, c, d ď n with a ` c “ b ` d, the chord joining
numbers a and c does not intersect the chord joining numbers b and d.
Let M be the number of beautiful arrangements of 0, 1, . . . , n. Let N be the number of pairs
px, yq of positive integers such that x ` y ď n and gcdpx, yq “ 1. Prove that
M “ N ` 1.
(Russia)
Solution 1. Given a circular arrangement of r0, ns “ t0, 1, . . . , nu, we define a k-chord to be
a (possibly degenerate) chord whose (possibly equal) endpoints add up to k. We say that three
chords of a circle are aligned if one of them separates the other two. Say that m ě 3 chords
are aligned if any three of them are aligned. For instance, in Figure 1, A, B, and C are aligned,
while B, C, and D are not.
A
B A
D
C u v
0 C n
D
t E
n−t
Figure 1 Figure 2
Claim. In a beautiful arrangement, the k–chords are aligned for any integer k.
Proof. We proceed by induction. For n ď 3 the statement is trivial. Now let n ě 4, and proceed
by contradiction. Consider a beautiful arrangement S where the three k–chords A, B, C are not
aligned. If n is not among the endpoints of A, B, and C, then by deleting n from S we obtain
a beautiful arrangement Sztnu of r0, n ´ 1s, where A, B, and C are aligned by the induction
hypothesis. Similarly, if 0 is not among these endpoints, then deleting 0 and decreasing all the
numbers by 1 gives a beautiful arrangement Szt0u where A, B, and C are aligned. Therefore
both 0 and n are among the endpoints of these segments. If x and y are their respective partners,
we have n ě 0 ` x “ k “ n ` y ě n. Thus 0 and n are the endpoints of one of the chords; say it
is C.
Let D be the chord formed by the numbers u and v which are adjacent to 0 and n and on the
same side of C as A and B, as shown in Figure 2. Set t “ u ` v. If we had t “ n, the n–chords A,
B, and D would not be aligned in the beautiful arrangement Szt0, nu, contradicting the induction
hypothesis. If t ă n, then the t-chord from 0 to t cannot intersect D, so the chord C separates t
and D. The chord E from t to n ´ t does not intersect C, so t and n ´ t are on the same side of C.
But then the chords A, B, and E are not aligned in Szt0, nu, a contradiction. Finally, the case
t ą n is equivalent to the case t ă n via the beauty-preserving relabelling x ÞÑ n ´ x for 0 ď x ď n,
which sends t-chords to p2n ´ tq–chords. This proves the Claim.
Having established the Claim, we prove the desired result by induction. The case n “ 2 is
trivial. Now assume that n ě 3. Let S be a beautiful arrangement of r0, ns and delete n to obtain
34 IMO 2013 Colombia
the beautiful arrangement T of r0, n ´ 1s. The n–chords of T are aligned, and they contain every
point except 0. Say T is of Type 1 if 0 lies between two of these n–chords, and it is of Type 2
otherwise; i.e., if 0 is aligned with these n–chords. We will show that each Type 1 arrangement
of r0, n ´ 1s arises from a unique arrangement of r0, ns, and each Type 2 arrangement of r0, n ´ 1s
arises from exactly two beautiful arrangements of r0, ns.
If T is of Type 1, let 0 lie between chords A and B. Since the chord from 0 to n must be
aligned with A and B in S, n must be on the other arc between A and B. Therefore S can be
recovered uniquely from T . In the other direction, if T is of Type 1 and we insert n as above,
then we claim the resulting arrangement S is beautiful. For 0 ă k ă n, the k–chords of S are also
k–chords of T , so they are aligned. Finally, for n ă k ă 2n, notice that the n–chords of S are
parallel by construction, so there is an antisymmetry axis ℓ such that x is symmetric to n ´ x with
respect to ℓ for all x. If we had two k–chords which intersect, then their reflections across ℓ would
be two p2n ´ kq-chords which intersect, where 0 ă 2n ´ k ă n, a contradiction.
If T is of Type 2, there are two possible positions for n in S, on either side of 0. As above, we
check that both positions lead to beautiful arrangements of r0, ns.
Hence if we let Mn be the number of beautiful arrangements of r0, ns, and let Ln be the number
of beautiful arrangements of r0, n ´ 1s of Type 2, we have
It then remains to show that Ln´1 is the number of pairs px, yq of positive integers with x ` y “ n
and gcdpx, yq “ 1. Since n ě 3, this number equals ϕpnq “ #tx : 1 ď x ď n, gcdpx, nq “ 1u.
To prove this, consider a Type 2 beautiful arrangement of r0, n ´ 1s. Label the positions
0, . . . , n ´ 1 pmod nq clockwise around the circle, so that number 0 is in position 0. Let f piq be
the number in position i; note that f is a permutation of r0, n ´ 1s. Let a be the position such
that f paq “ n ´ 1.
Since the n–chords are aligned with 0, and every point is in an n–chord, these chords are all
parallel and
f piq ` f p´iq “ n for all i.
Similarly, since the pn ´ 1q–chords are aligned and every point is in an pn ´ 1q–chord, these chords
are also parallel and
f piq ` f pa ´ iq “ n ´ 1 for all i.
Recall that this is an equality modulo n. Since f is a permutation, we must have pa, nq “ 1. Hence
Ln´1 ď ϕpnq.
To prove equality, it remains to observe that the labeling (1) is beautiful. To see this, consider
four numbers w, x, y, z on the circle with w ` y “ x ` z. Their positions around the circle satisfy
p´awq ` p´ayq “ p´axq ` p´azq, which means that the chord from w to y and the chord from
x to z are parallel. Thus (1) is beautiful, and by construction it has Type 2. The desired result
follows.
Shortlisted problems – solutions 35
Solution 2. Notice that there are exactly N irreducible fractions f1 ă ¨ ¨ ¨ ă fN in p0, 1q whose
denominator is at most n, since the pair px, yq with x ` y ď n and px, yq “ 1 corresponds to the
fraction x{px ` yq. Write fi “ abii for 1 ď i ď N.
We begin by constructing N ` 1 beautiful arrangements. Take any α P p0, 1q which is not one
of the above N fractions. Consider a circle of perimeter 1. Successively mark points 0, 1, 2, . . . , n
where 0 is arbitrary, and the clockwise distance from i to i`1 is α. The point k will be at clockwise
distance tkαu from 0, where tru denotes the fractional part of r. Call such a circular arrangement
cyclic and denote it by Apαq. If the clockwise order of the points is the same in Apα1 q and Apα2 q,
we regard them as the same circular arrangement. Figure 3 shows the cyclic arrangement Ap3{5`ǫq
of r0, 13s where ǫ ą 0 is very small.
2 7 12
10
4
5
9
0
13
1
8
6
3 11
Figure 3
numbers congruent to ka´1 i modulo bi , listed clockwise in increasing order. It follows that the first
number after 0 in Apfi ` ǫq is bi , and the first number after 0 which is less than bi is a´1 i pmod bi q,
which uniquely determines ai . In this way we can recover fi from the cyclic arrangement. Note
also that Apfi ` ǫq is not the trivial arrangement where we list 0, 1, . . . , n in order clockwise. It
follows that the N ` 1 cyclic arrangements Apǫq, Apf1 ` ǫq, . . . , ApfN ` ǫq are distinct.
Let us record an observation which will be useful later:
if fi ă α ă fi`1 then 0 is immediately after bi`1 and before bi in Apαq. (2)
Indeed, we already observed that bi is the first number after 0 in Apfi ` ǫq “ Apαq. Similarly we
see that bi`1 is the last number before 0 in Apfi`1 ´ ǫq “ Apαq.
36 IMO 2013 Colombia
Finally, we show that any beautiful arrangement of r0, ns is cyclic by induction on n. For n ď 2
the result is clear. Now assume that all beautiful arrangements of r0, n ´ 1s are cyclic, and consider
a beautiful arrangement A of r0, ns. The subarrangement An´1 “ Aztnu of r0, n ´ 1s obtained by
deleting n is cyclic; say An´1 “ An´1 pαq.
Let α be between the consecutive fractions pq11 ă pq22 among the irreducible fractions of de-
nominator at most n ´ 1. There is at most one fraction ni in p pq11 , pq22 q, since ni ă n´1
i
ď i`1
n
for
0 ă i ď n ´ 1.
p1 p2
Case 1. There is no fraction with denominator n between q1
and q2
.
In this case the only cyclic arrangement extending An´1 pαq is An pαq. We know that A and
An pαq can only differ in the position of n. Assume n is immediately after x and before y in An pαq.
Since the neighbors of 0 are q1 and q2 by (2), we have x, y ě 1.
y x−1
n n−1
x y−1
Figure 4
In An pαq the chord from n´ 1 to x is parallel and adjacent to the chord from n to x´ 1, so n´ 1
is between x ´ 1 and x in clockwise order, as shown in Figure 4. Similarly, n ´ 1 is between y
and y ´ 1. Therefore x, y, x ´ 1, n ´ 1, and y ´ 1 occur in this order in An pαq and hence in A
(possibly with y “ x ´ 1 or x “ y ´ 1).
Now, A may only differ from An pαq in the location of n. In A, since the chord from n ´ 1
to x and the chord from n to x ´ 1 do not intersect, n is between x and n ´ 1. Similarly, n is
between n ´ 1 and y. Then n must be between x and y and A “ An pαq. Therefore A is cyclic as
desired.
p1 i p2
Case 2. There is exactly one i with q1
ă n
ă q2
.
In this case there exist two cyclic arrangements An pα1 q and An pα2 q of the numbers 0, . . . , n
extending An´1 pαq, where pq11 ă α1 ă ni and ni ă α2 ă pq22 . In An´1 pαq, 0 is the only number
between q2 and q1 by (2). For the same reason, n is between q2 and 0 in An pα1 q, and between 0
and q1 in An pα2 q.
Letting x “ q2 and y “ q1 , the argument of Case 1 tells us that n must be between x and y
in A. Therefore A must equal An pα1 q or An pα2 q, and therefore it is cyclic.
This concludes the proof that every beautiful arrangement is cyclic. It follows that there are
exactly N ` 1 beautiful arrangements of r0, ns as we wished to show.
Shortlisted problems – solutions 37
C8. Players A and B play a paintful game on the real line. Player A has a pot of paint with
four units of black ink. A quantity p of this ink suffices to blacken a (closed) real interval of length
p. In every round, player A picks some positive integer m and provides 1{2m units of ink from the
pot. Player B then picks an integer k and blackens the interval from k{2m to pk ` 1q{2m (some
parts of this interval may have been blackened before). The goal of player A is to reach a situation
where the pot is empty and the interval r0, 1s is not completely blackened.
Decide whether there exists a strategy for player A to win in a finite number of moves.
(Austria)
segment I0r . In total it gives at most 3pxr ` 1{2m q ă 3pxr ` αq “ 3xr`1 . Thus condition piq is also
verified in this case. The claim is proved.
Finally, we can perform the desired estimation. Consider any situation in the game, say after the
pr´1qst move; assume that the segment r0, 1s is not completely black. By piiq, in the segment rxr , 1s
player B has colored several segments of different lengths; all these lengths are negative powers
of 2 not exceeding 1 ´ xr ; thus the total amount of ink used for this interval is at most 2p1 ´ xr q.
Using piq, we obtain that the total amount of ink used is at most 3xr ` 2p1 ´ xr q ă 3. Thus the
pot is not empty, and therefore A never wins.
Comment 1. Notice that this strategy works even if the pot contains initially only 3 units of ink.
Comment 2. There exist other strategies for B allowing him to prevent emptying the pot before the
whole interval is colored. On the other hand, let us mention some idea which does not work.
Player B could try a strategy in which the set of blackened points in each round is an interval of
the type r0, xs. Such a strategy cannot work (even if there is more ink available). Indeed, under the
assumption that B uses such a strategy, let us prove by induction on s the following statement:
For any positive integer s, player A has a strategy picking only positive integers m ď s in which,
if player B ever paints a point x ě 1 ´ 1{2s then after some move, exactly the interval r0, 1 ´ 1{2s s is
blackened, and the amount of ink used up to this moment is at least s{2.
For the base case s “ 1, player A just picks m “ 1 in the first round. If for some positive integer k
player A has such a strategy, for s ` 1 he can first rescale his strategy to the interval r0, 1{2s (sending in
each round half of the amount of ink he would give by the original strategy). Thus, after some round, the
interval r0, 1{2 ´ 1{2s`1 s becomes blackened, and the amount of ink used is at least s{4. Now player A
picks m “ 1{2, and player B spends 1{2 unit of ink to blacken the interval r0, 1{2s. After that, player A
again rescales his strategy to the interval r1{2, 1s, and player B spends at least s{4 units of ink to blacken
the interval r1{2, 1 ´ 1{2s`1 s, so he spends in total at least s{4 ` 1{2 ` s{4 “ ps ` 1q{2 units of ink.
Comment 3. In order to avoid finiteness issues, the statement could be replaced by the following one:
Players A and B play a paintful game on the real numbers. Player A has a paint pot with
four units of black ink. A quantity p of this ink suffices to blacken a (closed) real interval of
length p. In the beginning of the game, player A chooses (and announces) a positive integer
N . In every round, player A picks some positive integer m ď N and provides 1{2m units
of ink from the pot. The player B picks an integer k and blackens the interval from k{2m
to pk ` 1q{2m (some parts of this interval may happen to be blackened before). The goal of
player A is to reach a situation where the pot is empty and the interval r0, 1s is not completely
blackened.
Decide whether there exists a strategy for player A to win.
However, the Problem Selection Committee believes that this version may turn out to be harder than the
original one.
Shortlisted problems – solutions 39
Geometry
G1. Let ABC be an acute-angled triangle with orthocenter H, and let W be a point on
side BC. Denote by M and N the feet of the altitudes from B and C, respectively. Denote
by ω1 the circumcircle of BW N, and let X be the point on ω1 which is diametrically opposite
to W . Analogously, denote by ω2 the circumcircle of CW M, and let Y be the point on ω2 which
is diametrically opposite to W . Prove that X, Y and H are collinear.
(Thaliand)
Solution. Let L be the foot of the altitude from A, and let Z be the second intersection point of
circles ω1 and ω2 , other than W . We show that X, Y , Z and H lie on the same line.
Due to =BNC “ =BMC “ 90˝ , the points B, C, N and M are concyclic; denote their circle
by ω3 . Observe that the line W Z is the radical axis of ω1 and ω2 ; similarly, BN is the radical axis
of ω1 and ω3 , and CM is the radical axis of ω2 and ω3 . Hence A “ BN X CM is the radical center
of the three circles, and therefore W Z passes through A.
Since W X and W Y are diameters in ω1 and ω2 , respectively, we have =W ZX “ =W ZY “ 90˝ ,
so the points X and Y lie on the line through Z, perpendicular to W Z.
A
ω2
Y
ω1
ω3
N Z
H
X
B L W C
The quadrilateral BLHN is cyclic, because it has two opposite right angles. From the power
of A with respect to the circles ω1 and BLHN we find AL ¨ AH “ AB ¨ AN “ AW ¨ AZ. If H lies
AZ AL
on the line AW then this implies H “ Z immediately. Otherwise, by AH “ AW the triangles AHZ
˝
and AW L are similar. Then =HZA “ =W LA “ 90 , so the point H also lies on the line XY Z.
Let P be the point on ω1 such that W P is parallel to CN , and let Q be the point on ω2 such
that W Q is parallel to BM . Prove that P , Q and H are collinear if and only if BW “ CW
or AW K BC.
The Problem Selection Committee considered the first part more suitable for the competition.
40 IMO 2013 Colombia
G2. Let ω be the circumcircle of a triangle ABC. Denote by M and N the midpoints of the
sides AB and AC, respectively, and denote by T the midpoint of the arc BC of ω not containing A.
The circumcircles of the triangles AMT and ANT intersect the perpendicular bisectors of AC
and AB at points X and Y , respectively; assume that X and Y lie inside the triangle ABC. The
lines MN and XY intersect at K. Prove that KA “ KT .
(Iran)
Solution 1. Let O be the center of ω, thus O “ MY X NX. Let ℓ be the perpendicular bisector
of AT (it also passes through O). Denote by r the operation of reflection about ℓ. Since AT is the
angle bisector of =BAC, the line rpABq is parallel to AC. Since OM K AB and ON K AC, this
means that the line rpOMq is parallel to the line ON and passes through O, so rpOMq “ ON.
Finally, the circumcircle γ of the triangle AMT is symmetric about ℓ, so rpγq “ γ. Thus the
point M maps to the common point of ON with the arc AMT of γ — that is, rpMq “ X.
Similarly, rpNq “ Y . Thus, we get rpMNq “ XY , and the common point K of MN nd XY
lies on ℓ. This means exactly that KA “ KT .
A
ω
γ
K M
N
O ℓ
X
Y
B C
Solution 2. Let L be the second common point of the line AC with the circumcircle γ of
the triangle AMT . From the cyclic quadrilaterals ABT C and AMT L we get =BT C “ 180˝ ´
=BAC “ =MT L, which implies =BT M “ =CT L. Since AT is an angle bisector in these
quadrilaterals, we have BT “ T C and MT “ T L. Thus the triangles BT M and CT L are
congruent, so CL “ BM “ AM.
Let X 1 be the common point of the line NX with the external bisector of =BAC; notice
that it lies outside the triangle ABC. Then we have =T AX 1 “ 90˝ and X 1 A “ X 1 C, so we
get =X 1 AM “ 90˝ ` =BAC{2 “ 180˝ ´ =X 1 AC “ 180˝ ´ =X 1 CA “ =X 1 CL. Thus the
triangles X 1 AM and X 1 CL are congruent, and therefore
=MX 1 L “ =AX 1 C ` p=CX 1 L ´ =AX 1 Mq “ =AX 1 C “ 180˝ ´ 2=X 1 AC “ =BAC “ =MAL.
This means that X 1 lies on γ.
Thus we have =T XN “ =T XX 1 “ =T AX 1 “ 90˝ , so T X k AC. Then =XT A “ =T AC “
=T AM, so the cyclic quadrilateral MAT X is an isosceles trapezoid. Similarly, NAT Y is an
isosceles trapezoid, so again the lines MN and XY are the reflections of each other about the
perpendicular bisector of AT . Thus K belongs to this perpendicular bisector.
Shortlisted problems – solutions 41
X′
ω
M
N
Y
B C
γ
Comment. There are several different ways of showing that the points X and M are symmetrical with
respect to ℓ. For instance, one can show that the quadrilaterals AM ON and T XOY are congruent. We
chose Solution 1 as a simple way of doing it. On the other hand, Solution 2 shows some other interesting
properties of the configuration.
Let us define Y 1 , analogously to X 1 , as the common point of M Y and the external bisector of =BAC.
One may easily see that in general the lines M N and X 1 Y 1 (which is the external bisector of =BAC)
do not intersect on the perpendicular bisector of AT . Thus, any solution should involve some argument
using the choice of the intersection points X and Y .
42 IMO 2013 Colombia
G3. In a triangle ABC, let D and E be the feet of the angle bisectors of angles A and B,
respectively. A rhombus is inscribed into the quadrilateral AEDB (all vertices of the rhombus
lie on different sides of AEDB). Let ϕ be the non-obtuse angle of the rhombus. Prove that
ϕ ď maxt=BAC, =ABCu.
(Serbia)
Solution 1. Let K, L, M, and N be the vertices of the rhombus lying on the sides AE, ED, DB,
and BA, respectively. Denote by dpX, Y Zq the distance from a point X to a line Y Z. Since D
and E are the feet of the bisectors, we have dpD, ABq “ dpD, ACq, dpE, ABq “ dpE, BCq, and
dpD, BCq “ dpE, ACq “ 0, which implies
dpD, ACq ` dpD, BCq “ dpD, ABq and dpE, ACq ` dpE, BCq “ dpE, ABq.
Since L lies on the segment DE and the relation dpX, ACq ` dpX, BCq “ dpX, ABq is linear in X
inside the triangle, these two relations imply
dpL, ACq ` dpL, BCq “ dpL, ABq. (1)
Denote the angles as in the figure below, and denote a “ KL. Then we have dpL, ACq “ a sin µ
and dpL, BCq “ a sin ν. Since KLMN is a parallelogram lying on one side of AB, we get
dpL, ABq “ dpL, ABq ` dpN, ABq “ dpK, ABq ` dpM, ABq “ apsin δ ` sin εq.
Thus the condition (1) reads
sin µ ` sin ν “ sin δ ` sin ε. (2)
C
E L D
µ
K
ψ
ν
ψ M
α δ ε β
A N B
If one of the angles α and β is non-acute, then the desired inequality is trivial. So we assume
that α, β ă π{2. It suffices to show then that ψ “ =NKL ď maxtα, βu.
Assume, to the contrary, that ψ ą maxtα, βu. Since µ ` ψ “ =CKN “ α ` δ, by our
assumption we obtain µ “ pα ´ ψq ` δ ă δ. Similarly, ν ă ε. Next, since KN k ML, we have
β “ δ ` ν, so δ ă β ă π{2. Similarly, ε ă π{2. Finally, by µ ă δ ă π{2 and ν ă ε ă π{2, we
obtain
sin µ ă sin δ and sin ν ă sin ε.
This contradicts (2).
Comment. One can see that the equality is achieved if α “ β for every rhombus inscribed into the
quadrilateral AEDB.
Shortlisted problems – solutions 43
G4. Let ABC be a triangle with =B ą =C. Let P and Q be two different points on line AC
such that =P BA “ =QBA “ =ACB and A is located between P and C. Suppose that there
exists an interior point D of segment BQ for which P D “ P B. Let the ray AD intersect the circle
ABC at R ‰ A. Prove that QB “ QR.
(Georgia)
Solution 1. Denote by ω the circumcircle of the triangle ABC, and let =ACB “ γ. Note
that the condition γ ă =CBA implies γ ă 90˝ . Since =P BA “ γ, the line P B is tangent
to ω, so P A ¨ P C “ P B 2 “ P D 2. By PP D
A
“ PP D
C
the triangles P AD and P DC are similar, and
=ADP “ =DCP .
Next, since =ABQ “ =ACB, the triangles ABC and AQB are also similar. Then =AQB “
=ABC “ =ARC, which means that the points D, R, C, and Q are concyclic. Therefore =DRQ “
=DCQ “ =ADP .
ω
B R
P A Q C
Figure 1
Solution 2. Again, denote by ω the circumcircle of the triangle ABC. Denote =ACB “ γ. Since
=P BA “ γ, the line P B is tangent to ω.
Let E be the second intersection point of BQ with ω. If V 1 is any point on the ray CE
beyond E, then =BEV 1 “ 180˝ ´ =BEC “ 180˝ ´ =BAC “ =P AB; together with =ABQ “
=P BA this shows firstly, that the rays BA and CE intersect at some point V , and secondly
that the triangle V EB is similar to the triangle P AB. Thus we have =BV E “ =BP A. Next,
=AEV “ =BEV ´ γ “ =P AB ´ =ABQ “ =AQB; so the triangles P BQ and V AE are also
similar.
Let P H be an altitude in the isosceles triangle P BD; then BH “ HD. Let G be the intersection
point of P H and AB. By the symmetry with respect to P H, we have =BDG “ =DBG “ γ “
=BEA; thus DG k AE and hence BG GA
“ BD
DE
. Thus the points G and D correspond to each other
in the similar triangles P AB and V EB, so =DV B “ =GP B “ 90˝ ´ =P BQ “ 90˝ ´ =V AE.
Thus V D K AE.
44 IMO 2013 Colombia
Let T be the common point of V D and AE, and let DS be an altitude in the triangle BDR.
The points S and T are the feet of corresponding altitudes in the similar triangles ADE and BDR,
so BS
SR
“ TATE . On the other hand, the points T and H are feet of corresponding altitudes in the
similar triangles V AE and P BQ, so TATE “ HQBH
. Thus BS
SR
“ TATE “ HQ
BH
, and the triangles BHS
and BQR are similar.
Finally, SH is a median in the right-angled triangle SBD; so BH “ HS, and hence BQ “ QR.
R
B S
H
G ω
D
P
A Q C
T
E
V
Figure 2
Solution 3. Denote by ω and O the circumcircle of the triangle ABC and its center, respectively.
From the condition =P BA “ =BCA we know that BP is tangent to ω.
Let E be the second point of intersection of ω and BD. Due to the isosceles triangle BDP ,
the tangent of ω at E is parallel to DP and consequently it intersects BP at some point L. Of
course, P D k LE. Let M be the midpoint of BE, and let H be the midpoint of BR. Notice that
=AEB “ =ACB “ =ABQ “ =ABE, so A lies on the perpendicular bisector of BE; thus the
points L, A, M, and O are collinear. Let ω1 be the circle with diameter BO. Let Q1 “ HO X BE;
since HO is the perpendicular bisector of BR, the statement of the problem is equivalent to
Q1 “ Q.
Consider the following sequence of projections (see Fig. 3).
1. Project the line BE to the line LB through the center A. (This maps Q to P .)
2. Project the line LB to BE in parallel direction with LE. (P ÞÑ D.)
3. Project the line BE to the circle ω through its point A. (D ÞÑ R.)
4. Scale ω by the ratio 21 from the point B to the circle ω1 . (R ÞÑ H.)
5. Project ω1 to the line BE through its point O. (H ÞÑ Q1 .)
We prove that the composition of these transforms, which maps the line BE to itself, is the
identity. To achieve this, it suffices to show three fixed points. An obvious fixed point is B which
is fixed by all the transformations above. Another fixed point is M, its path being M ÞÑ L ÞÑ
E ÞÑ E ÞÑ M ÞÑ M.
Shortlisted problems – solutions 45
ω ω
ω1
O O
B B
R H U
M
D Z
Q ′ X
P Q P
C Q A A
Y
E E
L L
Figure 3 Figure 4
In order to show a third fixed point, draw a line parallel with LE through A; let that line
intersect BE, LB and ω at X, Y and Z ‰ A, respectively (see Fig. 4). We show that X is a
fixed point. The images of X at the first three transformations are X ÞÑ Y ÞÑ X ÞÑ Z. From
=XBZ “ =EAZ “ =AEL “ =LBA “ =BZX we can see that the triangle XBZ is isosceles.
Let U be the midpoint of BZ; then the last two transformations do Z ÞÑ U ÞÑ X, and the point X
is fixed.
Comment. Verifying that the point E is fixed seems more natural at first, but it appears to be less
straightforward. Here we outline a possible proof.
Let the images of E at the first three transforms above be F , G and I. After comparing the angles
depicted in Fig. 5 (noticing that the quadrilateral AF BG is cyclic) we can observe that the tangent LE
of ω is parallel to BI. Then, similarly to the above reasons, the point E is also fixed.
Q F
A P
E
L
Figure 5
46 IMO 2013 Colombia
x
z
A
Q
P C
y
y
F R
z D
E x
O
M L
A O1 C
T
Y′
O3 O2 D
Y
F E X
′
Z
ÝÝÑ ÝÝÑ ÝÝÑ ÝÝÑ ÝÝÑ ÝÝÑ
This allows us to consider a triangle LMN with LM “ EF , MN “ AB, and NL “ CD. Let O
be the circumcenter of △LMN and consider the points O1 , O2 , O3 such that △AO1B, △CO2 D,
and △EO3 F are translations of △MON, △NOL, and △LOM, respectively. Since F O3 and AO1
are translations of MO, quadrilateral AF O3 O1 is a parallelogram and O3 O1 “ F A “ CD “ NL.
Similarly, O1 O2 “ LM and O2 O3 “ MN. Therefore △O1O2 O3 – △LMN. Moreover, by means
of the rotation R one may check that these triangles have the same orientation.
Let T be the circumcenter of △O1 O2 O3 . We claim that AD, BE, and CF meet at T . Let
us show that C, T , and F are collinear. Notice that CO2 “ O2 T “ T O3 “ O3 F since they are
all equal to the circumradius of △LMN. Therefore △T O3F and △CO2T are isosceles. Using
directed angles between rays again, we get
>pT F, T O3q “ >pF O3 , F T q and >pT O2 , T Cq “ >pCT, CO2 q. (2)
Also, T and O are the circumcenters of the congruent triangles △O1O2 O3 and △LMN so we have
>pT O3 , T O2q “ >pON, OMq. Since CO2 and F O3 are translations of NO and MO respectively,
this implies
>pT O3 , T O2 q “ >pCO2 , F O3 q. (3)
48 IMO 2013 Colombia
Solution 3. Place the hexagon on the complex plane, with A at the origin and vertices labelled
clockwise. Now A, B, C, D, E, F represent the corresponding complex numbers. Also consider
the complex numbers a, b, c, a1 , b1 , c1 given by B ´ A “ a, D ´ C “ b, F ´ E “ c, E ´ D “ a1 ,
A ´ F “ b1 , and C ´ B “ c1 . Let k “ |a|{|b|. From a{b1 “ ´kei=A and a1 {b “ ´kei=D we get that
pa1 {aqpb1 {bq “ e´iθ and similarly pb1 {bqpc1 {cq “ e´iθ and pc1 {cqpa1 {aq “ e´iθ . It follows that a1 “ ar,
b1 “ br, and c1 “ cr for a complex number r with |r| “ 1, as shown below.
a
B
a cr
0 A a + cr
C
c(r − 1)λ
b
br W
W
W
W
D
D
D a + b + cr = c(r − 1)
D
F E ar
−br c
−br − c
We have
0 “ a ` cr ` b ` ar ` c ` br “ pa ` b ` cqp1 ` rq.
If r “ ´1, then the hexagon is centrally symmetric and its diagonals intersect at its center of
symmetry. Otherwise
a ` b ` c “ 0.
Therefore
A “ 0, B “ a, C “ a ` cr, D “ cpr ´ 1q, E “ ´br ´ c, F “ ´br.
Now consider a point W on AD given by the complex number cpr ´ 1qλ, where λ is a real number
with 0 ă λ ă 1. Since D ‰ A, we have r ‰ 1, so we can define s “ 1{pr ´ 1q. From rr “ |r|2 “ 1
we get
r r 1
1`s“ “ “ “ ´s.
r´1 r ´ rr 1´r
Now,
W is on BE ðñ cpr ´ 1qλ ´ a k a ´ p´br ´ cq “ bpr ´ 1q ðñ cλ ´ as k b
ðñ ´aλ ´ bλ ´ as k b ðñ apλ ` sq k b.
One easily checks that r ‰ ˘1 implies that λ ` s ‰ 0 since s is not real. On the other hand,
W on CF ðñ cpr ´ 1qλ ` br k ´br ´ pa ` crq “ apr ´ 1q ðñ cλ ` bp1 ` sq k a
ðñ ´aλ ´ bλ ´ bs k a ðñ bpλ ` sq k a ðñ b k apλ ` sq,
where in the last step we use that pλ ` sqpλ ` sq “ |λ ` s|2 P Rą0 . We conclude that AD X BE “
CF X BE, and the desired result follows.
Shortlisted problems – solutions 49
G6. Let the excircle of the triangle ABC lying opposite to A touch its side BC at the point A1 .
Define the points B1 and C1 analogously. Suppose that the circumcentre of the triangle A1 B1 C1
lies on the circumcircle of the triangle ABC. Prove that the triangle ABC is right-angled.
(Russia)
Solution 1. Denote the circumcircles of the triangles ABC and A1 B1 C1 by Ω and Γ, respectively.
Denote the midpoint of the arc CB of Ω containing A by A0 , and define B0 as well as C0 analogously.
By our hypothesis the centre Q of Γ lies on Ω.
Lemma. One has A0 B1 “ A0 C1 . Moreover, the points A, A0 , B1 , and C1 are concyclic. Finally,
the points A and A0 lie on the same side of B1 C1 . Similar statements hold for B and C.
Proof. Let us consider the case A “ A0 first. Then the triangle ABC is isosceles at A, which
implies AB1 “ AC1 while the remaining assertions of the Lemma are obvious. So let us suppose
A ‰ A0 from now on.
By the definition of A0 , we have A0 B “ A0 C. It is also well known and easy to show that BC1 “
CB1 . Next, we have =C1 BA0 “ =ABA0 “ =ACA0 “ =B1 CA0 . Hence the triangles A0 BC1
and A0 CB1 are congruent. This implies A0 C1 “ A0 B1 , establishing the first part of the Lemma.
It also follows that =A0 C1 A “ =A0 B1 A, as these are exterior angles at the corresponding vertices
C1 and B1 of the congruent triangles A0 BC1 and A0 CB1 . For that reason the points A, A0 , B1 ,
and C1 are indeed the vertices of some cyclic quadrilateral two opposite sides of which are AA0
and B1 C1 . l
Now we turn to the solution. Evidently the points A1 , B1 , and C1 lie interior to some semicircle
arc of Γ, so the triangle A1 B1 C1 is obtuse-angled. Without loss of generality, we will assume that
its angle at B1 is obtuse. Thus Q and B1 lie on different sides of A1 C1 ; obviously, the same holds
for the points B and B1 . So, the points Q and B are on the same side of A1 C1 .
Notice that the perpendicular bisector of A1 C1 intersects Ω at two points lying on different
sides of A1 C1 . By the first statement from the Lemma, both points B0 and Q are among these
points of intersection; since they share the same side of A1 C1 , they coincide (see Figure 1).
B (= Q)
B00000(= Q)
B
A11111
A
C11111
C
A C
B1111
B
Ω
A0
C0
Figure 1
50 IMO 2013 Colombia
Now, by the first part of the Lemma again, the lines QA0 and QC0 are the perpendicular
bisectors of B1 C1 and A1 B1 , respectively. Thus
recalling that A0 and C0 are the midpoints of the arcs CB and BA, respectively.
On the other hand, by the second part of the Lemma we have
From the last two equalities, we get =ABC “ 90˝ , whereby the problem is solved.
Solution 2. Let Q again denote the centre of the circumcircle of the triangle A1 B1 C1 , that lies
on the circumcircle Ω of the triangle ABC. We first consider the case where Q coincides with one
of the vertices of ABC, say Q “ B. Then BC1 “ BA1 and consequently the triangle ABC is
isosceles at B. Moreover we have BC1 “ B1 C in any triangle, and hence BB1 “ BC1 “ B1 C;
similarly, BB1 “ B1 A. It follows that B1 is the centre of Ω and that the triangle ABC has a right
angle at B.
So from now on we may suppose Q R tA, B, Cu. We start with the following well known fact.
Lemma. Let XY Z and X 1 Y 1 Z 1 be two triangles with XY “ X 1 Y 1 and Y Z “ Y 1 Z 1 .
piq If XZ ‰ X 1 Z 1 and =Y ZX “ =Y 1 Z 1 X 1 , then =ZXY ` =Z 1 X 1 Y 1 “ 180˝ .
piiq If =Y ZX ` =X 1 Z 1 Y 1 “ 180˝ , then =ZXY “ =Y 1 X 1 Z 1 .
Proof. For both parts, we may move the triangle XY Z through the plane until Y “ Y 1 and Z “ Z 1 .
Possibly after reflecting one of the two triangles about Y Z, we may also suppose that X and X 1
lie on the same side of Y Z if we are in case piq and on different sides if we are in case piiq. In both
cases, the points X, Z, and X 1 are collinear due to the angle condition (see Fig. 2). Moreover we
have X ‰ X 1 , because in case piq we assumed XZ ‰ X 1 Z 1 and in case piiq these points even lie
on different sides of Y Z. Thus the triangle XX 1 Y is isosceles at Y . The claim now follows by
considering the equal angles at its base. l
Y =Y′ Y =Y′
X X′
X X′ Z = Z′ Z = Z′
Figure 2(i) Figure 2(ii)
Relabeling the vertices of the triangle ABC if necessary we may suppose that Q lies in the
interior of the arc AB of Ω not containing C. We will sometimes use tacitly that the six trian-
gles QBA1 , QA1 C, QCB1 , QB1 A, QC1 A, and QBC1 have the same orientation.
As Q cannot be the circumcentre of the triangle ABC, it is impossible that QA “ QB “ QC
and thus we may also suppose that QC ‰ QB. Now the above Lemma piq is applicable to the
triangles QB1 C and QC1 B, since QB1 “ QC1 and B1 C “ C1 B, while =B1 CQ “ =C1 BQ holds
as both angles appear over the same side of the chord QA in Ω (see Fig. 3). So we get
We claim that QC “ QA. To see this, let us assume for the sake of a contradiction that
QC ‰ QA. Then arguing similarly as before but now with the triangles QA1 C and QC1 A we get
Adding this equation to (1), we get =A1 QB1 ` =BQA “ 360˝ , which is absurd as both summands
lie in the interval p0˝ , 180˝q.
This proves QC “ QA; so the triangles QA1 C and QC1 A are congruent their sides being equal,
which in turn yields
Finally our Lemma piiq is applicable to the triangles QA1 B and QB1 A. Indeed we have QA1 “ QB1
and A1 B “ B1 A as usual, and the angle condition =A1 BQ ` =QAB1 “ 180˝ holds as A and B
lie on different sides of the chord QC in Ω. Consequently we have
i.e. =CQA ` =A1 QC1 “ 180˝ . In light of (2) this may be rewritten as 2=CQA “ 180˝ and as Q
lies on Ω this implies that the triangle ABC has a right angle at B.
Q
Q
B
A11111
A
C11111
C
A C
B11111
B
Figure 3
Comment 1. One may also check that Q is in the interior of Ω if and only if the triangle ABC is
acute-angled.
Comment 2. The original proposal asked to prove the converse statement as well: if the triangle ABC
is right-angled, then the point Q lies on its circumcircle. The Problem Selection Committee thinks that
the above simplified version is more suitable for the competition.
52 IMO 2013 Colombia
Number Theory
N1. Let Zą0 be the set of positive integers. Find all functions f : Zą0 Ñ Zą0 such that
m2 ` f pnq | mf pmq ` n
Answer. f pnq “ n.
Solution 1. Setting m “ n “ 2 tells us that 4 ` f p2q | 2f p2q ` 2. Since 2f p2q ` 2 ă 2p4 ` f p2qq, we
must have 2f p2q ` 2 “ 4 ` f p2q, so f p2q “ 2. Plugging in m “ 2 then tells us that 4 ` f pnq | 4 ` n,
which implies that f pnq ď n for all n.
Setting m “ n gives n2 ` f pnq | nf pnq ` n, so nf pnq ` n ě n2 ` f pnq which we rewrite as
pn ´ 1qpf pnq ´ nq ě 0. Therefore f pnq ě n for all n ě 2. This is trivially true for n “ 1 also.
It follows that f pnq “ n for all n. This function obviously satisfies the desired property.
Solution 2. Setting m “ f pnq we get f pnqpf pnq`1q | f pnqf pf pnqq`n. This implies that f pnq | n
for all n.
Now let m be any positive integer, and let p ą 2m2 be a prime number. Note that p ą mf pmq
also. Plugging in n “ p´mf pmq we learn that m2 `f pnq divides p. Since m2 `f pnq cannot equal 1,
it must equal p. Therefore p ´ m2 “ f pnq | n “ p ´ mf pmq. But p ´ mf pmq ă p ă 2pp ´ m2 q, so
we must have p ´ mf pmq “ p ´ m2 , i.e., f pmq “ m.
which is impossible for m ą |f pnq ´ n|. It follows that f is the identity function.
Shortlisted problems – solutions 53
N2. Prove that for any pair of positive integers k and n there exist k positive integers
m1 , m2 , . . . , mk such that
2k ´ 1
ˆ ˙ˆ ˙ ˆ ˙
1 1 1
1` “ 1` 1` ¨¨¨ 1 ` .
n m1 m2 mk
(Japan)
Solution 1. We proceed by induction on k. For k “ 1 the statement is trivial. Assuming we
have proved it for k “ j ´ 1, we now prove it for k “ j.
Case 1. n “ 2t ´ 1 for some positive integer t.
Observe that
2j´1 ´ 1
ˆ ˙ˆ ˙ ˆ ˙
1 1 1
1` “ 1` 1` ¨¨¨ 1 ` ,
t m1 m2 mj´1
2j ´ 1 2t ` 2j ´ 1 2t ` 2j ´ 2 2j´1 ´ 1
ˆ ˙ˆ ˙
1
1` “ ¨ “ 1` 1` ,
2t 2t ` 2j ´ 2 2t 2t ` 2j ´ 2 t
2j´1 ´ 1
ˆ ˙ˆ ˙ ˆ ˙
1 1 1
1` “ 1` 1` ¨¨¨ 1 ` .
t m1 m2 mj´1
2k ´ 1 n ` S1 ` Ts n ` S1 ` Ts n ` Ts
1` “ “ ¨
n n n ` Ts n
r s
ź n ` Sp ` Ts ź n ` Tq
“ ¨
p“1
n ` Sp`1 ` Ts q“1 n ` Tq´1
r ˆ s ˆ
2ap 2bq
ź ˙ ź ˙
“ 1` ¨ 1` ,
p“1
n ` Sp`1 ` Ts q“1
n ` Tq´1
so if we define
n ` Sp`1 ` Ts n ` Tq´1
mp “ for 1 ď p ď r and mr`q “ for 1 ď q ď s,
2ap 2bq
the desired equality holds. It remains to check that every mi is an integer. For 1 ď p ď r we have
N3. Prove that there exist infinitely many positive integers n such that the largest prime divisor
of n4 ` n2 ` 1 is equal to the largest prime divisor of pn ` 1q4 ` pn ` 1q2 ` 1.
(Belgium)
Solution. Let pn be the largest prime divisor of n4 ` n2 ` 1 and let qn be the largest prime divisor
of n2 ` n ` 1. Then pn “ qn2 , and from
it follows that pn “ maxtqn , qn´1 u for n ě 2. Keeping in mind that n2 ´ n ` 1 is odd, we have
Therefore qn ‰ qn´1 .
To prove the result, it suffices to show that the set
Comment. Once the factorization of n4 ` n2 ` 1 is found and the set S is introduced, the problem is
mainly about ruling out the case that
might hold for some k P Zą0 . In the above solution, this is done by observing qpk`1q2 “ maxpqk , qk`1 q.
Alternatively one may notice that (1) implies that qj`2 ´ qj ě 6 for j ě k ` 1, since every prime greater
than 3 is congruent to ´1 or 1 modulo 6. Then there is some integer C ě 0 such that qn ě 3n ´ C for
all n ě k.
Now let the integer t be sufficiently large (e.g. t “ maxtk ` 1, C ` 3u) and set p “ qt´1 ě 2t. Then
p | pt ´ 1q2 ` pt ´ 1q ` 1 implies that p | pp ´ tq2 ` pp ´ tq ` 1, so p and qp´t are prime divisors of
pp ´ tq2 ` pp ´ tq ` 1. But p ´ t ą t ´ 1 ě k, so qp´t ą qt´1 “ p and p ¨ qp´t ą p2 ą pp ´ tq2 ` pp ´ tq ` 1,
a contradiction.
56 IMO 2013 Colombia
N4. Determine whether there exists an infinite sequence of nonzero digits a1 , a2 , a3 , . . . and a
positive integer N such that for every integer k ą N, the number ak ak´1 . . . a1 is a perfect square.
(Iran)
Answer. No.
Solution. Assume that a1 , a2 , a3 , . . . is such a sequence. For each positive integer k, let yk “
ak ak´1 . . . a1 . By the assumption, for each k ą N there exists a positive integer xk such that
yk “ x2k .
I. For every n, let 5γn be the greatest power of 5 dividing xn . Let us show first that 2γn ě n for
every positive integer n ą N.
Assume, to the contrary, that there exists a positive integer n ą N such that 2γn ă n, which
yields
´ yn ¯
yn`1 “ an`1 an . . . a1 “ 10n an`1 ` an an´1 . . . a1 “ 10n an`1 ` yn “ 52γn 2n 5n´2γn an`1 ` 2γn .
5
Since 5 {| yn {52γn , we obtain γn`1 “ γn ă n ă n ` 1. By the same arguments we obtain that
γn “ γn`1 “ γn`2 “ . . . . Denote this common value by γ.
Now, for each k ě n we have
which implies 52k ă 4 ¨ 52γ ¨ 10k`1 , or p5{2qk ă 40 ¨ 52γ . The last inequality is clearly false for
sufficiently large values of k. This contradiction shows that 2γn ě n for all n ą N.
II. Consider now any integer k ą maxtN{2, 2u. Since 2γ2k`1 ě 2k ` 1 and 2γ2k`2 ě 2k ` 2,
we have γ2k`1 ě k ` 1 and γ2k`2 ě k ` 1. So, from y2k`2 “ a2k`2 ¨ 102k`1 ` y2k`1 we obtain
52k`2 | y2k`2 ´ y2k`1 “ a2k`2 ¨ 102k`1 and thus 5 | a2k`2 , which implies a2k`2 “ 5. Therefore,
px2k`2 ´ x2k`1 qpx2k`2 ` x2k`1 q “ x22k`2 ´ x22k`1 “ y2k`2 ´ y2k`1 “ 5 ¨ 102k`1 “ 22k`1 ¨ 52k`2 .
Setting Ak “ x2k`2 {5k`1 and Bk “ x2k`1 {5k`1 , which are integers, we obtain
Both Ak and Bk are odd, since otherwise y2k`2 or y2k`1 would be a multiple of 10 which is false
by a1 ‰ 0; so one of the numbers Ak ´ Bk and Ak ` Bk is not divisible by 4. Therefore (1) yields
Ak ´ Bk “ 2 and Ak ` Bk “ 22k , hence Ak “ 22k´1 ` 1 and thus
since k ě 2. This implies that y2k`2 ą 102k`2 which contradicts the fact that y2k`2 contains 2k ` 2
digits. The desired result follows.
Shortlisted problems – solutions 57
N5. Fix an integer k ě 2. Two players, called Ana and Banana, play the following game of
numbers: Initially, some integer n ě k gets written on the blackboard. Then they take moves
in turn, with Ana beginning. A player making a move erases the number m just written on the
blackboard and replaces it by some number m1 with k ď m1 ă m that is coprime to m. The first
player who cannot move anymore loses.
An integer n ě k is called good if Banana has a winning strategy when the initial number is n,
and bad otherwise.
Consider two integers n, n1 ě k with the property that each prime number p ď k divides n if
and only if it divides n1 . Prove that either both n and n1 are good or both are bad.
(Italy)
Solution 1. Let us first observe that the number appearing on the blackboard decreases after
every move; so the game necessarily ends after at most n steps, and consequently there always has
to be some player possessing a winning strategy. So if some n ě k is bad, then Ana has a winning
strategy in the game with starting number n.
More precisely, if n ě k is such that there is a good integer m with n ą m ě k and
gcdpm, nq “ 1, then n itself is bad, for Ana has the following winning strategy in the game with
initial number n: She proceeds by first playing m and then using Banana’s strategy for the game
with starting number m.
Otherwise, if some integer n ě k has the property that every integer m with n ą m ě k and
gcdpm, nq “ 1 is bad, then n is good. Indeed, if Ana can make a first move at all in the game with
initial number n, then she leaves it in a position where the first player has a winning strategy, so
that Banana can defeat her.
In particular, this implies that any two good numbers have a non–trivial common divisor. Also,
k itself is good.
For brevity, we say that n ÝÑ x is a move if n and x are two coprime integers with n ą x ě k.
Claim 1. If n is good and n1 is a multiple of n, then n1 is also good.
Proof. If n1 were bad, there would have to be some move n1 ÝÑ x, where x is good. As n1 is a
multiple of n this implies that the two good numbers n and x are coprime, which is absurd. l
Claim 2. If r and s denote two positive integers for which rs ě k is bad, then r 2 s is also bad.
Proof. Since rs is bad, there is a move rs ÝÑ x for some good x. Evidently x is coprime to r 2 s as
well, and hence the move r 2 s ÝÑ x shows that r 2 s is indeed bad. l
Claim 3. If p ą k is prime and n ě k is bad, then np is also bad.
Proof. Otherwise we choose a counterexample with n being as small as possible. In particular, np
is good. Since n is bad, there is a move n ÝÑ x for some good x. Now np ÝÑ x cannot be a
valid move, which tells us that x has to be divisible by p. So we can write x “ pr y, where r and y
denote some positive integers, the latter of which is not divisible by p.
Note that y “ 1 is impossible, for then we would have x “ pr and the move x ÝÑ k would
establish that x is bad. In view of this, there is a least power y α of y that is at least as large
as k. Since the numbers np and y α are coprime and the former is good, the latter has to be
x n
bad. Moreover, the minimality of α implies y α ă ky ă py “ pr´1 ă pr´1 . So pr´1 ¨ y α ă n and
consequently all the numbers y α, py α , . . . , pr ¨ y α “ pppr´1 ¨ y α q are bad due to the minimal choice
of n. But now by Claim 1 the divisor x of pr ¨ y α cannot be good, whereby we have reached a
contradiction that proves Claim 3. l
Shortlisted problems – solutions 59
We now deduce the statement of the problem from these three claims. To this end, we call two
integers a, b ě k similar if they are divisible by the same prime numbers not exceeding k. We are
to prove that if a and b are similar, then either both of them are good or both are bad. As in this
case the product ab is similar to both a and b, it suffices to show the following: if c ě k is similar
to some of its multiples d, then either both c and d are good or both are bad.
Assuming that this is not true in general, we choose a counterexample pc0 , d0 q with d0 being
as small as possible. By Claim 1, c0 is bad whilst d0 is good. Plainly d0 is strictly greater than c0
and hence the quotient dc00 has some prime factor p. Clearly p divides d0 . If p ď k, then p
divides c0 as well due to the similarity, and hence d0 is actually divisible by p2 . So dp0 is good by
the contrapositive of Claim 2. Since c0 | dp0 , the pair pc0 , dp0 q contradicts the supposed minimality
of d0 . This proves p ą k, but now we get the same contradiction using Claim 3 instead of Claim 2.
Thereby the problem is solved.
Solution 2. We use the same analysis of the game of numbers as in the first five paragraphs of
the first solution. Let us call a prime number p small in case p ď k and big otherwise. We again
call two integers similar if their sets of small prime factors coincide.
Claim 4. For each integer b ě k having some small prime factor, there exists an integer x
similar to it with b ě x ě k and having no big prime factors.
Proof. Unless b has a big prime factor we may simply choose x “ b. Now let p and q denote a
small and a big prime factor of b, respectively. Let a be the product of all small prime factors
of b. Further define n to be the least non–negative integer for which the number x “ pn a is at
least as large as k. It suffices to show that b ą x. This is clear in case n “ 0, so let us assume
n ą 0 from now on. Then we have x ă pk due to the minimality of n, p ď a because p divides a
by construction, and k ă q. Therefore x ă aq and, as the right hand side is a product of distinct
prime factors of b, this implies indeed x ă b. l
Let us now assume that there is a pair pa, bq of similar numbers such that a is bad and b is
good. Take such a pair with maxpa, bq being as small as possible. Since a is bad, there exists a
move a ÝÑ r for some good r. Since the numbers k and r are both good, they have a common
prime factor, which necessarily has to be small. Thus Claim 4 is applicable to r, which yields
an integer r 1 similar to r containing small prime factors only and satisfying r ě r 1 ě k. Since
maxpr, r 1 q “ r ă a ď maxpa, bq the number r 1 is also good. Now let p denote a common prime
factor of the good numbers r 1 and b. By our construction of r 1 , this prime is small and due to
the similarities it consequently divides a and r, contrary to a ÝÑ r being a move. Thereby the
problem is solved.
Comment 1. Having reached Claim 4 of Solution 2, there are various other ways to proceed. For
instance, one may directly obtain the following fact, which seems to be interesting in its own right:
Claim 5. Any two good numbers have a common small prime factor.
Proof. Otherwise there exists a pair pb, b1 q of good numbers with b1 ě b ě k all of whose common prime
factors are big. Choose such a pair with b1 being as small as possible. Since b and k are both good, there
has to be a common prime factor p of b and k. Evidently p is small and thus it cannot divide b1 , which in
turn tells us b1 ą b. Applying Claim 4 to b we get an integer x with b ě x ě k that is similar to b and has
no big prime divisors at all. By our assumption, b1 and x are coprime, and as b1 is good this implies that
x is bad. Consequently there has to be some move x ÝÑ b˚ such that b˚ is good. But now all the small
prime factors of b also appear in x and thus they cannot divide b˚ . Therefore the pair pb˚ , bq contradicts
the supposed minimality of b1 . l
60 IMO 2013 Colombia
From that point, it is easy to complete the solution: assume that there are two similar integers a and b
such that a is bad and b is good. Since a is bad, there is a move a ÝÑ b1 for some good b1 . By Claim 5,
there is a small prime p dividing b and b1 . Due to the similarity of a and b, the prime p has to divide a
as well, but this contradicts the fact that a ÝÑ b1 is a valid move. Thereby the problem is solved.
Comment 2. There are infinitely many good numbers, e.g. all multiples of k. The increasing sequence
b0 , b1 , . . . , of all good numbers may be constructed recursively as follows:
‚ Start with b0 “ k.
‚ If bn has just been defined for some n ě 0, then bn`1 is the smallest number b ą bn that is coprime
to none of b0 , . . . , bn .
This construction can be used to determine the set of good numbers for any specific k as explained in the
next comment. It is already clear that if k “ pα is a prime power, then a number b ě k is good if and
only if it is divisible by p.
Comment 3. Let P ą 1 denote the product of all small prime numbers. Then any two integers a, b ě k
that are congruent modulo P are similar. Thus the infinite word Wk “ pXk , Xk`1 , . . .q defined by
#
A if i is bad
Xi “
B if i is good
for all i ě k is periodic and the length of its period divides P . As the prime power example shows, the
true period can sometimes be much smaller than P . On the other hand, there are cases where the period
is rather large; e.g., if k “ 15, the sequence of good numbers begins with 15, 18, 20, 24, 30, 36, 40, 42, 45
and the period of W15 is 30.
Comment 4. The original proposal contained two questions about the game of numbers, namely paq to
show that if two numbers have the same prime factors then either both are good or both are bad, and pbq
to show that the word Wk introduced in the previous comment is indeed periodic. The Problem Selection
Committee thinks that the above version of the problem is somewhat easier, even though it demands to
prove a stronger result.
Shortlisted problems – solutions 61
Recall that 0 ď pa ´ 1q{2 ă b{2. Thus, in both cases ω “ 0 and ω “ 1, the left-hand part of (3)
equals ω either by the minimality of b, or by f pωq “ ω. A contradiction. ` ˘
Thus b has to be odd, so b “ 2k ` 1 for some k ě 1. Applying (1) to 12 , k, b we get
ˆ ˙ ˆ ˙
ω`k 1
f “f “ ω. (4)
b 2
Since a and b are coprime, there exist integers r P t1, 2, . . . , bu and m such that ra ´ mb “ k ` ω.
Note that we actually have 1 ď r ă b, since the right hand side is not a multiple of b. If m
is negative, then we have ra ´ mb ą b ě k ` ω, which is absurd. Similarly, m ě r leads to
ra ´ mb ă br ´ br “ 0, which
` k`ω is likewise
˘ impossible; so we must have 0 ď m ď r ´ 1.
We finally substitute b , m, r into (1) and use (4) to learn
´ω ` m¯ ´a¯
f “f ‰ ω.
r b
But as above one may see that the left hand side has to equal ω due to the minimality of b. This
contradiction concludes our step B.
Step C. Now notice that if ω “ 0, then f pxq “ txu holds for all rational x with 0 ď x ă 1 and
hence by (2) this even holds for all rational numbers x. Similarly, if ω “ 1, then f pxq “ rxs holds
for all x P Q. Thereby the problem is solved.
Comment 1. An alternative treatment of Steps B and C from the second case, due to the proposer,
proceeds as follows. Let square brackets indicate the floor function in case ω “ 0 and the ceiling function
if ω “ 1. We are to prove that f pxq “ rxs holds for all x P Q, and because of Step A and (2) we already
know this in case 2x P Z. Applying (1) to p2x, 0, 2q we get
ˆ ˙
f p2xq
f pxq “ f ,
2
f p2n xq
„
f pxq “ for all px, nq P Q ˆ Zą0 . (6)
2n
Now suppose first that x is not an integer but can be written in the form pq with p P Z and q P Zą0 both
being odd. Let d denote the multiplicative order of 2 modulo q and let m be any large integer. Plugging
n “ dm into (6) and using (2) we get
Since x is not an integer, the square bracket function is continuous at x; hence as m tends to infinity the
above fomula gives f pxq “ rxs. To complete´the¯argument
” ı we just need to observe that if some y P Q
`y ˘ rys rys “y‰
satisfies f pyq “ rys, then (5) yields f 2 “ f 2 “ 2 “ 2 .
Shortlisted problems – solutions 63
Solution 2. Here we just give another argument for the second case of the above solution. Again
we use equation (2). It follows that the set S of all zeros of f contains for each x P Q exactly one
term from the infinite sequence . . . , x ´ 2, x ´ 1, x, x ` 1, x ` 2, . . . .
Next we claim that
p p
if pp, qq P Z ˆ Zą0 and q
P S, then q`1
P S holds as well. (7)
From (8) and (9) we get 0 P S Ď p´1, `1q and hence the real number α “ suppSq exists and
satisfies 0 ď α ď 1.
Let us assume that we actually had 0 ă α ă 1. Note that f pxq “ 0 if x P p0, αq X Q by (8),
and f pxq “ 1 if x P pα, 1q X Q by (9) and (2). Let K denote the unique positive integer satisfying
1`α
Kα ă 1 ď pK ` 1qα. The first of these two inequalities entails α ă K`1 , and thus there is a
` 1`α ˘
rational number x P α, K`1 . Setting y “ pK ` 1qx ´ 1 and substituting py, 1, K ` 1q into (1) we
learn ˆ ˙ ˆ ˙
f pyq ` 1 y`1
f “f “ f pxq.
K `1 K `1
Since α ă x ă 1 and 0 ă y ă α, this simplifies to
ˆ ˙
1
f “ 1.
K `1
1 1
But, as 0 ă K`1 ď α, this is only possible if α “ K`1 and f pαq “ 1. From this, however, we get
the contradiction
ˆ ˙ ˆ ˙ ˆ ˙
1 α`0 f pαq ` 0
0“f “f “f “ f pαq “ 1.
pK ` 1q2 K `1 K `1
Thus our assumption 0 ă α ă 1 has turned out to be wrong and it follows that α P t0, 1u. If
α “ 0, then we have S Ď p´1, 0s, whence S “ p´1, 0s X Q, which in turn yields f pxq “ rxs for all
x P Q due to (2). Similarly, α “ 1 entails S “ r0, 1q X Q and f pxq “ txu for all x P Q. Thereby
the solution is complete.
64 IMO 2013 Colombia
Comment 2. It seems that all solutions to this problems involve some case distinction separating the
constant solutions from the unbounded ones, though the “descriptions” of the cases may be different
depending on the work that has been done at the beginning of the solution. For instance, these two cases
can also be “f is periodic on the integers” and “f is not periodic on the integers”. The case leading to
the unbounded solutions appears to be the harder one.
In most approaches, the cases leading to the two functions x ÞÝÑ txu and x ÞÝÑ rxs can easily be
treated parallelly, but sometimes it may be useful to know that there is some symmetry in the problem
interchanging these two functions. Namely, if a function f : Q ÝÑ Z satisfies (1), then so does the
function g : Q ÝÑ Z defined by gpxq “ ´f p´xq for all x P Q. For that reason, we could have restricted
our attention to the case ω “ 0 in the first solution and, once α P t0, 1u had been obtained, to the case
α “ 0 in the second solution.
Shortlisted problems – solutions 65
N7. Let ν be an irrational positive number, and let m be a positive integer. A pair pa, bq of
positive integers is called good if
arbνs ´ btaνu “ m. p˚q
A good pair pa, bq is called excellent if neither of the pairs pa´b, bq and pa, b´aq is good. (As usual,
by txu and rxs we denote the integer numbers such that x ´ 1 ă txu ď x and x ď rxs ă x ` 1.)
Prove that the number of excellent pairs is equal to the sum of the positive divisors of m.
(U.S.A.)
Solution. For positive integers a and b, let us denote
We will deal with various values of m; thus it is convenient to say that a pair pa, bq is m-good or
m-excellent if the corresponding conditions are satisfied.
To start, let us investigate how the values f pa ` b, bq and f pa, b ` aq are related to f pa, bq. If
taνu ` tbνu ă 1, then we have tpa ` bqνu “ taνu ` tbνu and rpa ` bqνs “ raνs ` rbνs ´ 1, so
and
So, in both cases one of the numbers f pa ` b, aq and f pa, b ` aq is equal to f pa, bq while the other
is greater than f pa, bq by one of a and b. Thus, exactly one of the pairs pa ` b, bq and pa, b ` aq is
excellent (for an appropriate value of m).
Now let us say that the pairs pa ` b, bq and pa, b ` aq are the children of the pair pa, bq, while
this pair is their parent. Next, if a pair pc, dq can be obtained from pa, bq by several passings from a
parent to a child, we will say that pc, dq is a descendant of pa, bq, while pa, bq is an ancestor of pc, dq
(a pair is neither an ancestor nor a descendant of itself). Thus each pair pa, bq has two children,
it has a unique parent if a ‰ b, and no parents otherwise. Therefore, each pair of distinct positive
integers has a unique ancestor of the form pa, aq; our aim is now to find how many m-excellent
descendants each such pair has.
Notice now that if a pair pa, bq is m-excellent then minta, bu ď m. Indeed, if a “ b then
f pa, aq “ a “ m, so the statement is valid. Otherwise, the pair pa, bq is a child of some pair pa1 , b1 q. If
b “ b1 and a “ a1 `b1 , then we should have m “ f pa, bq “ f pa1 , b1 q`b1 , so b “ b1 “ m´f pa1 , b1 q ă m.
Similarly, if a “ a1 and b “ b1 ` a1 then a ă m.
Let us consider the set Sm of all pairs pa, bq such that f pa, bq ď m and minta, bu ď m. Then
all the ancestors of the elements in Sm are again in Sm , and each element in Sm either is of the
form pa, aq with a ď m, or has a unique ancestor of this form. From the arguments above we see
that all m-excellent pairs lie in Sm .
We claim now that the set Sm is finite. Indeed, assume, for instance, that it contains infinitely
many pairs pc, dq with d ą 2m. Such a pair is necessarily a child of pc, d ´ cq, and thus a descendant
of some pair pc, d1 q with m ă d1 ď 2m. Therefore, one of the pairs pa, bq P Sm with m ă b ď 2m
66 IMO 2013 Colombia
has infinitely many descendants in Sm , and all these descendants have the form pa, b ` kaq with k
a positive integer. Since f pa, b ` kaq does not decrease as k grows, it becomes constant for k ě k0 ,
where k0 is some positive integer. This means that taνu ` tpb ` kaqνu ă 1 for all k ě k0 . But this
yields 1 ą tpb ` kaqνu “ tpb ` k0 aqνu ` pk ´ k0 qtaνu for all k ą k0 , which is absurd.
Similarly, one can prove that Sm contains finitely many pairs pc, dq with c ą 2m, thus finitely
many elements at all.
We are now prepared for proving the following crucial lemma.
Lemma. Consider any pair pa, bq with f pa, bq ‰ m. Then the number gpa, bq of its m-excellent
descendants is equal to the number hpa, bq of ways to represent the number t “ m ´ f pa, bq as
t “ ka ` ℓb with k and ℓ being some nonnegative integers.
Proof. We proceed by induction on the number N of descendants of pa, bq in Sm . If N “ 0 then
clearly gpa, bq “ 0. Assume that hpa, bq ą 0; without loss of generality, we have a ď b. Then,
clearly, m ´ f pa, bq ě a, so f pa, b ` aq ď f pa, bq ` a ď m and a ď m, hence pa, b ` aq P Sm which
is impossible. Thus in the base case we have gpa, bq “ hpa, bq “ 0, as desired.
Now let N ą 0. Assume that f pa ` b, bq “ f pa, bq ` b and f pa, b ` aq “ f pa, bq (the other case
is similar). If f pa, bq ` b ‰ m, then by the induction hypothesis we have
Notice that both pairs pa ` b, bq and pa, b ` aq are descendants of pa, bq and thus each of them has
strictly less descendants in Sm than pa, bq does.
Next, each one of the hpa ` b, bq representations of m ´ f pa ` b, bq “ m ´ b ´ f pa, bq as the sum
k pa ` bq ` ℓ1 b provides the representation m ´ f pa, bq “ ka ` ℓb with k “ k 1 ă k 1 ` ℓ1 ` 1 “ ℓ.
1
Similarly, each one of the hpa, b ` aq representations of m ´ f pa, b ` aq “ m ´ f pa, bq as the sum
k 1 a ` ℓ1 pb ` aq provides the representation m ´ f pa, bq “ ka ` ℓb with k “ k 1 ` ℓ1 ě ℓ1 “ ℓ. This
correspondence is obviously bijective, so
as required.
Finally, if f pa, bq`b “ m then pa`b, bq is m-excellent, so gpa, bq “ 1`gpa, b`aq “ 1`hpa, b`aq
by the induction hypothesis. On the other hand, the number m ´ f pa, bq “ b has a representation
0 ¨ a ` 1 ¨ b and sometimes one more representation as ka ` 0 ¨ b; this last representation exists
simultaneously with the representation m ´ f pa, b ` aq “ ka ` 0 ¨ pb ` aq, so hpa, bq “ 1 ` hpa, b ` aq
as well. Thus in this case the step is also proved. l
Now it is easy to finish the solution. There exists a unique m-excellent pair of the form pa, aq,
and each other m-excellent pair pa, bq has a unique ancestor of the form px, xq with x ă m. By the
lemma, for every x ă m the number of its m-excellent descendants is hpx, xq, which is the number
of ways to represent m ´ f px, xq “ m ´ x as kx ` ℓx (with nonnegative integer k and ℓ). This
number is 0 if x {| m, and m{x otherwise. So the total number of excellent pairs is
ÿ m ÿ ÿ
1` “1` d“ d,
x|m, xăm
x d|m, dą1 d|m
as required.
Shortlisted problems – solutions 67
Comment. Let us present a sketch of an outline of a different solution. The plan is to check that the
number of excellent pairs does not depend on the (irrational) number ν, and to find this number for some
appropriate value of ν. For that, we first introduce some geometrical language. We deal only with the
excellent pairs pa, bq with a ‰ b.
Part I. Given an irrational positive ν, for every positive integer n we introduce two integral points Fν pnq “
pn, tnνuq and Cν pnq “ pn, rnνsq on the coordinate plane Oxy. Then p˚q reads as rOFν paqCν pbqs “ m{2;
here r¨s stands for the signed area. Next, we rewrite in these terms the condition on a pair pa, bq to be
excellent. Let ℓν , ℓ` ´
ν , and ℓν be the lines determined by the equations y “ νx, y “ νx ` 1, and y “ νx ´ 1,
respectively.
a). Firstly, we deal with all excellent pairs pa, bq with a ă b. Given some value of a, all the points C such
that rOFν paqCs “ m{2 lie on some line fν paq; if there exist any good pairs pa, bq at all, this line has to
contain at least one integral point, which happens exactly when gcdpa, taνuq | m.
Let Pν paq be the point of intersection of ℓ`ν and fν paq, and let pν paq be its abscissa; notice that pν paq
is irrational if it is nonzero. Now, if pa, bq is good, then the point Cν pbq lies on fν paq, which means that
the point of fν paq with abscissa b lies between ℓν and ℓ` ν and is integral. If in addition the pair pa, b ´ aq
is not good, then the point of fν paq with abscissa b ´ a lies above ℓ` ν (see Fig. 1). Thus, the pair pa, bq
with b ą a is excellent exactly when pν paq lies between b ´ a and b, and the point of fν paq with abscissa b
is integral (which means that this point is Cν pbq).
Notice now that, if pν paq ą a, then the number of excellent pairs of the form pa, bq (with b ą a) is
gcdpa, taνuq.
ℓν
ℓ+
ν
C (b)
Cννννν(b)
C
C (b)
(b) ℓν F (a)
Fννννν(a) ℓ−
ν
Pν (a) C
C
C (b)
Cννννν(b)
(b)
(b)
C
C
C (a)
Cννννν(a)
(a)
(a) Q
Q
Q (b)
Qννννν(b)
(b)
(b)
(a)
ffννννν(a)
Fννννν(b)
F
F (b)
(b)
Fννννν(a)
F
F (a)
(a)
b−a a b b a−b a
(b)
ccννννν(b)
Figure 1 Figure 2
b). Analogously, considering the pairs pa, bq with a ą b, we fix the value of b, introduce the line cν pbq
containing all the points F with rOF Cν pbqs “ m{2, assume that this line contains an integral point
(which means gcdpb, rbνsq | m), and denote the common point of cν pbq and ℓ´ ν by Qν pbq, its abscissa
being qν pbq. Similarly to the previous case, we obtain that the pair pa, bq is excellent exactly when qν paq
lies between a ´ b and a, and the point of cν pbq with abscissa a is integral (see Fig. 2). Again, if qν pbq ą b,
then the number of excellent pairs of the form pa, bq (with a ą b) is gcdpb, rbνsq.
Part II, sketchy. Having obtained such a description, one may check how the number of excellent pairs
changes as ν grows. (Having done that, one may find this number for
˘ one appropriate value of ν; for
1
`
instance, it is relatively easy to make this calculation for ν P 1, 1 ` m .)
Consider, for the initial value of ν, some excellent pair pa, tq with a ą t. As ν grows, this pair
eventually stops being excellent; this happens when the point Qν ptq passes through Fν paq. At the same
moment, the pair pa ` t, tq becomes excellent instead.
This process halts when the point Qν ptq eventually disappears, i.e. when ν passes through the ratio
of the coordinates of the point T “ Cν ptq. Hence, the point T afterwards is regarded as Fν ptq. Thus, all
the old excellent pairs of the form pa, tq with a ą t disappear; on the other hand, the same number of
excellent pairs with the first element being t just appear.
68 IMO 2013 Colombia
Similarly, if some pair pt, bq with t ă b is initially ν-excellent, then at some moment it stops being
excellent when Pν ptq passes through Cν pbq; at the same moment, the pair pt, b´ tq becomes excellent. This
process eventually stops when b ´ t ă t. At this moment, again the second element of the pair becomes
fixed, and the first one starts to increase.
These ideas can be made precise enough to show that the number of excellent pairs remains unchanged,
as required.
We should warn the reader that the rigorous elaboration of Part II is technically quite involved, mostly
by the reason that the set of moments when the collection of excellent pairs changes is infinite. Especially
much care should be applied to the limit points of this set, which are exactly the points when the line ℓν
passes through some point of the form Cν pbq.
The same ideas may be explained in an algebraic language instead of a geometrical one; the same
technicalities remain in this way as well.
Problems short list
with solutions
Shortlisted Problems with Solutions
Contributing Countries
The Organising Committee and the Problem Selection Committee of IMO 2014 thank the
following 43 countries for contributing 141 problem proposals.
Johan Meyer
Ilya I. Bogdanov
Géza Kós
Waldemar Pompe
Christian Reiher
Stephan Wagner
4 IMO 2014 South Africa
Problems
Algebra
A1. Let z0 ă z1 ă z2 ă ¨ ¨ ¨ be an infinite sequence of positive integers. Prove that there
exists a unique integer n ě 1 such that
z0 ` z1 ` ¨ ¨ ¨ ` zn
zn ă ď zn`1 .
n
(Austria)
A2. Define the function f : p0, 1q Ñ p0, 1q by
#
1
x` 2
if x ă 21 ,
f pxq “
x2 if x ě 21 .
Let a and b be two real numbers such that 0 ă a ă b ă 1. We define the sequences an and bn
by a0 “ a, b0 “ b, and an “ f pan´1 q, bn “ f pbn´1 q for n ą 0. Show that there exists a positive
integer n such that
pan ´ an´1 qpbn ´ bn´1 q ă 0.
(Denmark)
A3. For a sequence x1 , x2 , . . . , xn of real numbers, we define its price as
max |x1 ` ¨ ¨ ¨ ` xi |.
1ďiďn
Given n real numbers, Dave and George want to arrange them into a sequence with a
low price. Diligent Dave checks all possible ways and finds the minimum possible price D.
Greedy George, on the other hand, chooses x1 such that |x1 | is as small as possible; among
the remaining numbers, he chooses x2 such that |x1 ` x2 | is as small as possible, and so on.
Thus, in the ith step he chooses xi among the remaining numbers so as to minimise the value
of |x1 ` x2 ` ¨ ¨ ¨ ` xi |. In each step, if several numbers provide the same value, George chooses
one at random. Finally he gets a sequence with price G.
Find the least possible constant c such that for every positive integer n, for every collection
of n real numbers, and for every possible sequence that George might obtain, the resulting
values satisfy the inequality G ď cD.
(Georgia)
A4. Determine all functions f : Z Ñ Z satisfying
` ˘
f f pmq ` n ` f pmq “ f pnq ` f p3mq ` 2014
for all integers m and n.
(Netherlands)
Shortlisted problems 5
A5. Consider all polynomials P pxq with real coefficients that have the following property:
for any two real numbers x and y one has
for all n P Z.
(United Kingdom)
6 IMO 2014 South Africa
Combinatorics
C1. Let n points be given inside a rectangle R such that no two of them lie on a line parallel
to one of the sides of R. The rectangle R is to be dissected into smaller rectangles with sides
parallel to the sides of R in such a way that none of these rectangles contains any of the given
points in its interior. Prove that we have to dissect R into at least n ` 1 smaller rectangles.
(Serbia)
C2. We have 2 sheets of paper, with the number 1 written on each of them. We perform
m
the following operation. In every step we choose two distinct sheets; if the numbers on the two
sheets are a and b, then we erase these numbers and write the number a ` b on both sheets.
Prove that after m2m´1 steps, the sum of the numbers on all the sheets is at least 4m .
(Iran)
C3. Let n ě 2 be an integer. Consider an n ˆ n chessboard divided into n2 unit squares.
We call a configuration of n rooks on this board happy if every row and every column contains
exactly one rook. Find the greatest positive integer k such that for every happy configuration
of rooks, we can find a k ˆ k square without a rook on any of its k 2 unit squares.
(Croatia)
C4. Construct a tetromino by attaching two 2 ˆ 1 dominoes along their longer sides such
that the midpoint of the longer side of one domino is a corner of the other domino. This
construction yields two kinds of tetrominoes with opposite orientations. Let us call them S-
and Z-tetrominoes, respectively.
S-tetrominoes Z-tetrominoes
Assume that a lattice polygon P can be tiled with S-tetrominoes. Prove than no matter
how we tile P using only S- and Z-tetrominoes, we always use an even number of Z-tetrominoes.
(Hungary)
C5. Consider n ě 3 lines in the plane such that no two lines are parallel and no three have a
common point. These lines divide the plane into polygonal
Pa regions;
T let F be the set of regions
having finite area. Prove that it is possible to colour n{2 of the lines blue in such a way
that no region in F has a completely blue boundary. (For a real number x, rxs denotes the
least integer which is not smaller than x.)
(Austria)
Shortlisted problems 7
C6. We are given an infinite deck of cards, each with a real number on it. For every real
number x, there is exactly one card in the deck that has x written on it. Now two players draw
disjoint sets A and B of 100 cards each from this deck. We would like to define a rule that
declares one of them a winner. This rule should satisfy the following conditions:
1. The winner only depends on the relative order of the 200 cards: if the cards are laid down
in increasing order face down and we are told which card belongs to which player, but
not what numbers are written on them, we can still decide the winner.
2. If we write the elements of both sets in increasing order as A “ ta1 , a2 , . . . , a100 u and
B “ tb1 , b2 , . . . , b100 u, and ai ą bi for all i, then A beats B.
3. If three players draw three disjoint sets A, B, C from the deck, A beats B and B beats C,
then A also beats C.
How many ways are there to define such a rule? Here, we consider two rules as different if there
exist two sets A and B such that A beats B according to one rule, but B beats A according to
the other.
(Russia)
C7. Let M be a set of n ě 4 points in the plane, no three of which are collinear. Initially these
points are connected with n segments so that each point in M is the endpoint of exactly two
segments. Then, at each step, one may choose two segments AB and CD sharing a common
interior point and replace them by the segments AC and BD if none of them is present at this
moment. Prove that it is impossible to perform n3 {4 or more such moves.
(Russia)
C8. A card deck consists of 1024 cards. On each card, a set of distinct decimal digits is
written in such a way that no two of these sets coincide (thus, one of the cards is empty). Two
players alternately take cards from the deck, one card per turn. After the deck is empty, each
player checks if he can throw out one of his cards so that each of the ten digits occurs on an
even number of his remaining cards. If one player can do this but the other one cannot, the
one who can is the winner; otherwise a draw is declared.
Determine all possible first moves of the first player after which he has a winning strategy.
(Russia)
C9. There are n circles drawn on a piece of paper in such a way that any two circles
intersect in two points, and no three circles pass through the same point. Turbo the snail slides
along the circles in the following fashion. Initially he moves on one of the circles in clockwise
direction. Turbo always keeps sliding along the current circle until he reaches an intersection
with another circle. Then he continues his journey on this new circle and also changes the
direction of moving, i.e. from clockwise to anticlockwise or vice versa.
Suppose that Turbo’s path entirely covers all circles. Prove that n must be odd.
(India)
8 IMO 2014 South Africa
Geometry
G1. The points P and Q are chosen on the side BC of an acute-angled triangle ABC so
that =P AB “ =ACB and =QAC “ =CBA. The points M and N are taken on the rays AP
and AQ, respectively, so that AP “ P M and AQ “ QN. Prove that the lines BM and CN
intersect on the circumcircle of the triangle ABC.
(Georgia)
G2. Let ABC be a triangle. The points K, L, and M lie on the segments BC, CA, and AB,
respectively, such that the lines AK, BL, and CM intersect in a common point. Prove that it
is possible to choose two of the triangles ALM, BMK, and CKL whose inradii sum up to at
least the inradius of the triangle ABC.
(Estonia)
G3. Let Ω and O be the circumcircle and the circumcentre of an acute-angled triangle ABC
with AB ą BC. The angle bisector of =ABC intersects Ω at M ‰ B. Let Γ be the circle
with diameter BM. The angle bisectors of =AOB and =BOC intersect Γ at points P and Q,
respectively. The point R is chosen on the line P Q so that BR “ MR. Prove that BR k AC.
(Here we always assume that an angle bisector is a ray.)
(Russia)
G4. Consider a fixed circle Γ with three fixed points A, B, and C on it. Also, let us fix
a real number λ P p0, 1q. For a variable point P R tA, B, Cu on Γ, let M be the point on
the segment CP such that CM “ λ ¨ CP . Let Q be the second point of intersection of the
circumcircles of the triangles AMP and BMC. Prove that as P varies, the point Q lies on a
fixed circle.
(United Kingdom)
G5. Let ABCD be a convex quadrilateral with =B “ =D “ 90 . Point H is the foot of
˝
the perpendicular from A to BD. The points S and T are chosen on the sides AB and AD,
respectively, in such a way that H lies inside triangle SCT and
Prove that the circumcircle of triangle SHT is tangent to the line BD.
(Iran)
G6. Let ABC be a fixed acute-angled triangle. Consider some points E and F lying on
the sides AC and AB, respectively, and let M be the midpoint of EF . Let the perpendicular
bisector of EF intersect the line BC at K, and let the perpendicular bisector of MK intersect
the lines AC and AB at S and T , respectively. We call the pair pE, F q interesting, if the
quadrilateral KSAT is cyclic.
Suppose that the pairs pE1 , F1 q and pE2 , F2 q are interesting. Prove that
E1 E2 F1 F2
“ .
AB AC
(Iran)
G7. Let ABC be a triangle with circumcircle Ω and incentre I. Let the line passing through I
and perpendicular to CI intersect the segment BC and the arc BC (not containing A) of Ω at
points U and V , respectively. Let the line passing through U and parallel to AI intersect AV
at X, and let the line passing through V and parallel to AI intersect AB at Y . Let W and Z be
the midpoints of AX and BC, respectively. Prove that if the points I, X, and Y are collinear,
then the points I, W , and Z are also collinear.
(U.S.A.)
Shortlisted problems 9
Number Theory
N1. Let n ě 2 be an integer, and let An be the set
An “ t2n ´ 2k | k P Z, 0 ď k ă nu.
Determine the largest positive integer that cannot be written as the sum of one or more (not
necessarily distinct) elements of An .
(Serbia)
N2. Determine all pairs px, yq of positive integers such that
a3
7x2 ´ 13xy ` 7y 2 “ |x ´ y| ` 1 .
(U.S.A.)
N3. A coin is called a Cape Town coin if its value is 1{n for some positive integer n. Given
a collection of Cape Town coins of total value at most 99 ` 21 , prove that it is possible to split
this collection into at most 100 groups each of total value at most 1.
(Luxembourg)
N4. Let n ą 1 be a given integer. Prove that infinitely many terms of the sequence pak qkě1 ,
defined by Z k^
n
ak “ ,
k
are odd. (For a real number x, txu denotes the largest integer not exceeding x.)
(Hong Kong)
N5. Find all triples pp, x, yq consisting of a prime number p and two positive integers x and y
such that xp´1 ` y and x ` y p´1 are both powers of p.
(Belgium)
N6. Let a1 ă a2 ă ¨ ¨ ¨ ă an be pairwise coprime positive integers with a1 being prime
and a1 ě n ` 2. On the segment I “ r0, a1 a2 ¨ ¨ ¨ an s of the real line, mark all integers that are
divisible by at least one of the numbers a1 , . . . , an . These points split I into a number of smaller
segments. Prove that the sum of the squares of the lengths of these segments is divisible by a1 .
(Serbia)
N7. Let c ě 1 be an integer. Define a sequence of positive integers by a1 “ c and
an`1 “ a3n ´ 4c ¨ a2n ` 5c2 ¨ an ` c
for all n ě 1. Prove that for each integer n ě 2 there exists a prime number p dividing an but
none of the numbers a1 , . . . , an´1 .
(Austria)
N8. For every real number x, let }x} denote the distance between x and the nearest integer.
Prove that for every pair pa, bq of positive integers there exist an odd prime p and a positive
integer k satisfying › › › › › ›
› a › › b › ›a ` b›
› ›`› ›`›
› pk › › pk › › pk › “ 1.
›
(Hungary)
10 IMO 2014 South Africa
Solutions
Algebra
A1. Let z0 ă z1 ă z2 ă ¨ ¨ ¨ be an infinite sequence of positive integers. Prove that there
exists a unique integer n ě 1 such that
z0 ` z1 ` ¨ ¨ ¨ ` zn
zn ă ď zn`1 . p1q
n
(Austria)
Solution. For n “ 1, 2, . . . define
dn “ pz0 ` z1 ` ¨ ¨ ¨ ` zn q ´ nzn .
The sign of dn indicates whether the first inequality in (1) holds; i.e., it is satisfied if and only
if dn ą 0.
Notice that
so the second inequality in (1) is equivalent to dn`1 ď 0. Therefore, we have to prove that there
is a unique index n ě 1 that satisfies dn ą 0 ě dn`1 .
d1 “ pz0 ` z1 q ´ 1 ¨ z1 “ z0 ą 0.
From
` ˘ ` ˘
dn`1 ´ dn “ pz0 ` ¨ ¨ ¨ ` zn ` zn`1 q ´ pn ` 1qzn`1 ´ pz0 ` ¨ ¨ ¨ ` zn q ´ nzn “ npzn ´ zn`1 q ă 0
we can see that dn`1 ă dn and thus the sequence strictly decreases.
Hence, we have a decreasing sequence d1 ą d2 ą . . . of integers such that its first element d1
is positive. The sequence must drop below 0 at some point, and thus there is a unique index n,
that is the index of the last positive term, satisfying dn ą 0 ě dn`1 .
Comment. Omitting the assumption that z1 , z2 , . . . are integers allows the numbers dn to be all
positive. In such cases the desired n does not exist. This happens for example if zn “ 2 ´ 21n for all
integers n ě 0.
Shortlisted problems – solutions 11
Let a and b be two real numbers such that 0 ă a ă b ă 1. We define the sequences an and bn
by a0 “ a, b0 “ b, and an “ f pan´1 q, bn “ f pbn´1 q for n ą 0. Show that there exists a positive
integer n such that
pan ´ an´1 qpbn ´ bn´1 q ă 0.
(Denmark)
Solution. Note that
1
f pxq ´ x “ 2
ą0
1
if x ă 2
and
f pxq ´ x “ x2 ´ x ă 0
if x ě 21 . So if we consider p0, 1q as being divided into the two subintervals I1 “ p0, 21 q and
I2 “ r 21 , 1q, the inequality
` ˘` ˘
pan ´ an´1 qpbn ´ bn´1 q “ f pan´1 q ´ an´1 f pbn´1 q ´ bn´1 ă 0
If, on the other hand, ak and bk both lie in I2 , then minpak , bk q ě 21 and maxpak , bk q “
minpak , bk q ` dk ě 21 ` dk , which implies
ˇ ˇ ˇ ˇ
dk`1 “ |ak`1 ´ bk`1 | “ ˇa2k ´ b2k ˇ “ ˇpak ´ bk qpak ` bk qˇ ě |ak ´ bk | 21 ` 21 ` dk “ dk p1 ` dk q ě dk .
` ˘
This means that the difference dk is non-decreasing, and in particular dk ě d0 ą 0 for all k.
We can even say more. If ak and bk lie in I2 , then
dk`2 ě dk`1 ě dk p1 ` dk q ě dk p1 ` d0 q.
If ak and bk both lie in I1 , then ak`1 and bk`1 both lie in I2 , and so we have
d2m ě d0 p1 ` d0 qm .
For sufficiently large m, the right-hand side is greater than 1, but since a2m , b2m both lie in
p0, 1q, we must have d2m ă 1, a contradiction.
Thus there must be a positive integer n such that an´1 and bn´1 do not lie in the same
subinterval, which proves the desired statement.
12 IMO 2014 South Africa
Given n real numbers, Dave and George want to arrange them into a sequence with a
low price. Diligent Dave checks all possible ways and finds the minimum possible price D.
Greedy George, on the other hand, chooses x1 such that |x1 | is as small as possible; among
the remaining numbers, he chooses x2 such that |x1 ` x2 | is as small as possible, and so on.
Thus, in the ith step he chooses xi among the remaining numbers so as to minimise the value
of |x1 ` x2 ` ¨ ¨ ¨ ` xi |. In each step, if several numbers provide the same value, George chooses
one at random. Finally he gets a sequence with price G.
Find the least possible constant c such that for every positive integer n, for every collection
of n real numbers, and for every possible sequence that George might obtain, the resulting
values satisfy the inequality G ď cD.
(Georgia)
Answer. c “ 2.
Solution. If the initial numbers are 1, ´1, 2, and ´2, then Dave may arrange them as
1, ´2, 2, ´1, while George may get the sequence 1, ´1, 2, ´2, resulting in D “ 1 and G “ 2. So
we obtain c ě 2.
Therefore, it remains to prove that G ď 2D. Let x1 , x2 , . . . , xn be the numbers Dave and
George have at their disposal. Assume that Dave and George arrange them into sequences
d1 , d2, . . . , dn and g1 , g2 , . . . , gn , respectively. Put
We claim that
D ě S, (1)
M
Dě , and (2)
2
G ď N “ maxtM, Su. (3)
These inequalities yield the desired estimate, as G ď maxtM, Su ď maxtM, 2Su ď 2D.
The inequality (1) is a direct consequence of the definition of the price.
To prove (2), consider an index i with |di | “ M. Then we have
ˇ ˇ
M “ |di | “ ˇpd1 ` ¨ ¨ ¨ ` di q ´ pd1 ` ¨ ¨ ¨ ` di´1 qˇ ď |d1 ` ¨ ¨ ¨ ` di | ` |d1 ` ¨ ¨ ¨ ` di´1 | ď 2D,
as required.
It remains to establish (3). Put hi “ g1 ` g2 ` ¨ ¨ ¨ ` gi . We will prove by induction on
i that |hi | ď N. The base case i “ 1 holds, since |h1 | “ |g1 | ď M ď N. Notice also that
|hn | “ S ď N.
For the induction step, assume that |hi´1 | ď N. We distinguish two cases.
Case 1. Assume that no two of the numbers gi , gi`1 , . . . , gn have opposite signs.
Without loss of generality, we may assume that they are all nonnegative. Then one has
hi´1 ď hi ď ¨ ¨ ¨ ď hn , thus (
|hi | ď max |hi´1 |, |hn | ď N.
Case 2. Among the numbers gi , gi`1 , . . . , gn there are positive and negative ones.
Shortlisted problems – solutions 13
Then there exists some index j ě i such that hi´1 gj ď 0. By the definition of George’s
sequence we have
(
|hi | “ |hi´1 ` gi | ď |hi´1 ` gj | ď max |hi´1 |, |gj | ď N.
M M
Comment 1. One can establish the weaker inequalities D ě 2 and G ď D ` 2 from which the
result also follows.
Comment 2. One may ask a more specific question to find the maximal suitable c if the number n
is fixed. For n “ 1 or 2, the answer is c “ 1. For n “ 3, the answer is c “ 32 , and it is reached e.g.,
for the collection 1, 2, ´4. Finally, for n ě 4 the answer is c “ 2. In this case the arguments from the
solution above apply, and the answer is reached e.g., for the same collection 1, ´1, 2, ´2, augmented
by several zeroes.
14 IMO 2014 South Africa
` ˘
f f pmq ` n ` f pmq “ f pnq ` f p3mq ` 2014 (1)
Now if f p0q vanished, then gp0q “ 2C ą 0 would entail that f vanishes identically, contrary
to (1). Thus f p0q ‰ 0 and the previous equation yields gprq “ α f prq, where α “ fgp0q
p0q
is some
nonzero constant.
So the definition of g reveals f p3mq “ p1 ` αqf pmq ´ 2C, i.e.,
` ˘
f p3mq ´ β “ p1 ` αq f pmq ´ β (3)
2C
for all m P Z, where β “ α
. By induction on k this implies
for all n, t P Z.
Let us fix any positive integer k with d | p3k ´ 1q, which is possible, since gcdp3, dq “ 1.
E.g., by the Euler–Fermat theorem, we may take k “ ϕp|d|q. Now for each m P Z we get
So f is a linear function, say f pmq “ Am ` β for all m P Z with some constant A P Q. Plugging
this into (1) one obtains pA2 ´ 2Aqm ` pAβ ´ 2Cq “ 0 for all m, which is equivalent to the
conjunction of
A2 “ 2A and Aβ “ 2C . (6)
The first equation is equivalent to A P t0, 2u, and as C ‰ 0 the second one gives
This shows that f is indeed the function mentioned in the answer and as the numbers found
in (7) do indeed satisfy the equations (6) this function is indeed as desired.
Comment 1. One may see that α “ 2. A more pedestrian version of the above solution starts with
a direct proof of this fact, that can be obtained by substituting some special values into (1), e.g., as
follows.
Set D “ f p0q. Plugging m “ 0 into (1) and simplifying, we get
f pn ` Dq “ f pnq ` 2C (8)
Comment 2. It is natural to wonder what happens if one replaces the number 2014 appearing in
the statement of the problem by some arbitrary integer B.
If B is odd, there is no such function, as can be seen by using the same ideas as in the above
solution.
If B ‰ 0 is even, however, then the only such function is given by n ÞÝÑ 2n`B{2. In case 3 ∤ B this
was essentially proved above, but for the general case one more idea seems to be necessary. Writing
B “ 3ν ¨ k with some integers ν and k such that 3 ∤ k one can obtain f pnq “ 2n ` B{2 for all n that
are divisible by 3ν in the same manner as usual; then one may use the formula f p3nq “ 3f pnq ´ B to
establish the remaining cases.
16 IMO 2014 South Africa
Finally, in case B “ 0 there are more solutions than just the function n ÞÝÑ 2n. It can be shown
that all these other functions are periodic; to mention just one kind of example, for any even integers
r and s the function #
r if n is even,
f pnq “
s if n is odd,
also has the property under discussion.
Shortlisted problems – solutions 17
A5. Consider all polynomials P pxq with real coefficients that have the following property:
for any two real numbers x and y one has
|y 2 ´ P pxq| ď 2 |x| if and only if |x2 ´ P pyq| ď 2 |y| . (1)
2 x2 p|x| ´ Cq2
2 x2
|y ´ P pxq| “ y ` ` ` 2 |x| ě ` 2 |x| ě 2 |x| ,
C C C
where in the first estimate equality can only hold if |x| “ C, whilst in the second one it can
only hold if x “ 0. As these two conditions cannot be met at the same time, we have indeed
|y 2 ´ P pxq| ą 2 |x|.
To show that P p0q “ 1 is possible as well, we verify that the polynomial P pxq “ x2 ` 1
satisfies (1). Notice that for all real numbers x and y we have
|y 2 ´ P pxq| ď 2 |x| ðñ py 2 ´ x2 ´ 1q2 ď 4x2
ðñ 0 ď py 2 ´ px ´ 1q2 px ` 1q2 ´ y 2
` ˘` ˘
i.e.
1 ` 2b1{2
2 ´nq{2
xpn ă ,
an{2`1
which is surely absurd. Thus P is indeed a quadratic polynomial.
Fourth step: We prove that P pxq “ x2 ` 1.
In the light of our first three steps there are?two real numbers a ą 0 and b such that P pxq “
ax2 ` b. Now if x is large ? enough and y “ a x, the left part of (1) holds and the right part
2 2
reads |p1 ´ a qx ´ b| ď 2 a x. In view of the fact that a ą 0 this is only possible if a “ 1.
Finally, substituting y “ x ` 1 with x ą 0 into (1) we get
|2x ` 1 ´ b| ď 2x ðñ |2x ` 1 ` b| ď 2x ` 2 ,
i.e.,
b P r1, 4x ` 1s ðñ b P r´4x ´ 3, 1s
for all x ą 0. Choosing x large enough, we can achieve that at least one of these two statements
holds; then both hold, which is only possible if b “ 1, as desired.
Comment 1. There are some issues with this problem in that its most natural solutions seem to
use some basic facts from analysis, such as the continuity of polynomials or the intermediate value
theorem. Yet these facts are intuitively obvious and implicitly clear to the students competing at this
level of difficulty, so that the Problem Selection Committee still thinks that the problem is suitable
for the IMO.
Comment 2. It seems that most solutions will in the main case, where P p0q is nonnegative, contain
an argument that is somewhat asymptotic in nature showing that P is quadratic, and some part
narrowing that case down to P pxq “ x2 ` 1.
Shortlisted problems – solutions 19
Comment 3. It is also possible to skip the first step and start with the second step directly, but
then one has to work a bit harder to rule out the case P p0q “ 0. Let us sketch one possibility of doing
this: Take the auxiliary polynomial Qpxq such that P pxq “ xQpxq. Applying (1) to x “ 0 and an
arbitrary y ‰ 0 we get |Qpyq| ą 2. Hence we either have Qpzq ě 2 for all real z or Qpzq ď ´2 for all
real z. In particular there is some η P t´1, `1u such that P pηq ě 2 and P p´ηq ď ´2. Substituting
x “ ˘η into (1) we learn
Comment 4. Truly curious people may wonder about the set of all polynomials having property (1).
As explained in the solution above, P pxq “ x2 ` 1 is the only one with P p0q “ 1. On the other hand,
it is not hard to notice that for negative P p0q there are more possibilities than those mentioned above.
E.g., as remarked by the proposer, if a and b denote two positive real `numbers with ab˘ ą 1 and Q
denotes a polynomial attaining nonnegative values only, then P pxq “ ´ ax2 ` b ` Qpxq works.
More generally, it may be proved that if P pxq satisfies (1) and P p0q ă 0, then ´P pxq ą 2 |x| holds
for all x P R so that one just considers the equivalence of two false statements. One may generate all
such polynomials P by going through all combinations of a solution of the polynomial equation
x “ ApxqBpxq ` CpxqDpxq
for all n P Z.
(United Kingdom)
Solution 1.
Part I. Let us first check that each of the functions above really satisfies the given functional
equation. If f pnq “ n ` 1 for all n, then we have
If f pnq “ n ` 1 for n ą ´a and f pnq “ ´n ` 1 otherwise, then we have the same identity for
n ą ´a and
n2 ` 4f pnq “ n2 ´ 4n ` 4 “ p2 ´ nq2 “ f p1 ´ nq2 “ f pf pnqq2
otherwise. The same applies to the third solution (with a “ 0), where in addition one has
02 ` 4f p0q “ 0 “ f pf p0qq2.
Part II. It remains to prove that these are really the only functions that satisfy our func-
tional equation. We do so in three steps:
Step 1: We prove that f pnq “ n ` 1 for n ą 0.
Consider the sequence pak q given by ak “ f k p1q for k ě 0. Setting n “ ak in (1), we get
Of course, a0 “ 1 by definition. Since a22 “ 1 ` 4a1 is odd, a2 has to be odd as well, so we set
a2 “ 2r ` 1 for some r P Z. Then a1 “ r 2 ` r and consequently
Since 8r ` 4 ‰ 0, a23 ‰ pr 2 ` rq2 , so the difference between a23 and pr 2 ` rq2 is at least the
distance from pr 2 ` rq2 to the nearest even square (since 8r ` 4 and r 2 ` r are both even). This
implies that
ˇ ˇ
|8r ` 4| “ ˇa23 ´ pr 2 ` rq2 ˇ ě pr 2 ` rq2 ´ pr 2 ` r ´ 2q2 “ 4pr 2 ` r ´ 1q,
(for r “ 0 and r “ ´1, the estimate is trivial, but this does not matter). Therefore, we ave
4r 2 ď |8r ` 4| ´ 4r ` 4.
Shortlisted problems – solutions 21
If |r| ě 4, then
a contradiction. Thus |r| ă 4. Checking all possible remaining values of r, we find that
pr 2 ` rq2 ` 8r ` 4 is only a square in three cases: r “ ´3, r “ 0 and r “ 1. Let us now
distinguish these three cases:
and the sign needs to be chosen in such a way that a2k`1 ` 4ak`2 is again a square. This
yields a3 “ ´4, a4 “ ´3, a5 “ ´2, a6 “ ´1, a7 “ 0, a8 “ 1, a9 “ 2. At this point
we have reached a contradiction, since f p1q “ f pa0 q “ a1 “ 6 and at the same time
f p1q “ f pa8 q “ a9 “ 2.
The latter can only be a square if k “ 4 (since 1 and 9 are the only two squares whose
difference is 8). Then, however, a4 “ 5, a5 “ ´6 and a6 “ ˘1, so
but neither 32 nor 40 is a perfect square. Thus ak`1 “ k ` 2, which completes our
induction. This also means that f pnq “ f pan´1 q “ an “ n ` 1 for all n ě 1.
which gives us
˘2
n2 ď f p´nq2 “ ˘f pf p´nqq ` 1 ´ 4f pf p´nqq ď f pf p´nqq2 ` 6|f pf p´nqq| ` 1
`
ď pn ´ 3q2 ` 6pn ´ 3q ` 1 “ n2 ´ 8,
a contradiction.
• Thus, we are left with the case that f p´nq ą ´n. Now we argue as in the previous
case: if f p´nq ě 0, then f pf p´nqq “ f p´nq ` 1 by the first two steps, since f p0q “ 0
and f p´nq “ 0 would imply n “ 0 (as seen in Step 2) and is thus impossible. If
f p´nq ă 0, we can apply the induction hypothesis, so in any case we can infer that
f pf p´nqq “ ˘f p´nq ` 1. We obtain
˘2
p´nq2 ` 4f p´nq “ ˘f p´nq ` 1 ,
`
so either ˘2
n2 “ f p´nq2 ´ 2f p´nq ` 1 “ f p´nq ´ 1 ,
`
Since 1 and 9 are the only perfect squares whose difference is 8, we must have n “ 1,
which we have already considered.
Finally, suppose that f p´nq “ ´n ` 1 for some n ě 2. Then
f p´n ` 1q2 “ f pf p´nqq2 “ p´nq2 ` 4f p´nq “ pn ´ 2q2 ,
so f p´n` 1q “ ˘pn´ 2q. However, we already know that f p´n` 1q “ ´n` 2 or f p´n` 1q “ n,
so f p´n ` 1q “ ´n ` 2. l
Shortlisted problems – solutions 23
• If f pnq is not always equal to n ` 1, then there is a largest integer m (which cannot be
positive) for which this is not the case. In view of the lemma that we proved, we must
then have f pnq “ ´n ` 1 for any integer n ă m. If m “ ´a ă 0, we obtain f pnq “ ´n ` 1
for n ď ´a (and f pnq “ n ` 1 otherwise). If m “ 0, we have the additional possibility
that f p0q “ 0, f pnq “ ´n ` 1 for negative n and f pnq “ n ` 1 for positive n.
Solution 2. Let us provide an alternative proof for Part II, which also proceeds in several
steps.
Step 1. Let a be an arbitrary integer and b “ f paq. We first concentrate on the case where
|a| is sufficiently large.
From now on, we set D “ |f p0q|. Throughout Step 1, we will assume that a R t´D, 0, Du,
thus b ‰ 0.
2. From (1), noticing that f pf paqq and a have the same parity, we get
ˇ ˇ ˘2
0 ‰ 4|b| “ ˇf pf paqq2 ´ a2 ˇ ě a2 ´ |a| ´ 2 “ 4|a| ´ 4.
`
Hence we have
|b| “ |f paq| ě |a| ´ 1 for a R t´D, 0, Du. (3)
For the rest of Step 1, we also assume that |a| ě E “ maxtD ` 2, 10u. Then by (3) we
have |b| ě D ` 1 and thus |f pbq| ě D.
which implies ˘2 ˘2 ˘2
a2 ě |b| ´ 1 ´ 4|b| “ |b| ´ 3 ´ 8 ą |b| ´ 4
` ` `
We have shown that, with at most finitely many exceptions, f paq “ 1 ˘ a. Thus it will be
convenient for our second step to introduce the sets
( ( ` ˘
Z` “ a P Z : f paq “ a ` 1 , Z´ “ a P Z : f paq “ 1 ´ a , and Z0 “ Zz Z` Y Z´ .
24 IMO 2014 South Africa
4. Note that f pE `1q “ 1˘pE `1q. If f pE `1q “ E `2, then E `1 P Z` . Otherwise we have
f p1`Eq “ ´E; then the original equation (1) with n “ E `1 gives us pE ´1q2 “ f p´Eq2 ,
so f p´Eq “ ˘pE ´ 1q. By (4) this may happen only if f p´Eq “ 1 ´ E, so in this case
´E P Z` . In any case we find that Z` ‰ ∅.
5. Now take any a P Z` . We claim that every integer x ě a also lies in Z` . We proceed by
induction on x, the base case x “ a being covered by our assumption. For the induction
step, assume that f px ´ 1q “ x and plug n “ x ´ 1 into (1). We get f pxq2 “ px ` 1q2 , so
either f pxq “ x ` 1 or f pxq “ ´px ` 1q.
Assume that f pxq “ ´px ` 1q and x ‰ ´1, since otherwise we already have f pxq “ x ` 1.
Plugging n “ x into (1), we obtain f p´x ´ 1q2 “ px ´ 2q2 ´ 8, which may happen only if
x´2 “ ˘3 and f p´x´1q “ ˘1. Plugging n “ ´x´1 into (1), we get f p˘1q2 “ px`1q2 ˘4,
which in turn may happen only if x ` 1 P t´2, 0, 2u.
Thus x P t´1, 5u and at the same time x P t´3, ´1, 1u, which gives us x “ ´1. Since this
has already been excluded, we must have f pxq “ x ` 1, which completes our induction.
7. Assume that there exists some a P Z0 with b “ f paq R Z0 , so that f pbq “ 1 ˘ b. Then we
have a2 ` 4b “ p1 ˘ bq2 , so either a2 “ pb ´ 1q2 or a2 “ pb ´ 3q2 ´ 8. In the former case
we have b “ 1 ˘ a, which is impossible by our choice of a. So we get a2 “ pb ´ 3q2 ´ 8,
which implies f pbq “ 1 ´ b and |a| “ 1, |b ´ 3| “ 3.
If b “ 0, then we have f pbq “ 1, so b P Z` and therefore a0 ď 0; hence a “ ´1. But then
f paq “ 0 “ a ` 1, so a P Z` , which is impossible.
If b “ 6, then we have f p6q “ ´5, so f p´5q2 “ 16 and f p´5q P t´4, 4u. Then f pf p´5qq2 “
25 ` 4f p´5q P t9, 41u, so f p´5q “ ´4 and ´5 P Z` . This implies a0 ď ´5, which
contradicts our assumption that ˘1 “ a R Z` .
8. Thus we have shown that f pZ0 q Ď Z0 , and Z0 is finite. Take any element c P Z0 , and
consider the sequence defined by ci “ f i pcq. All elements of the sequence pci q lie in Z0 ,
hence it is bounded. Choose an index k for which |ck | is maximal, so that in particular
|ck`1 | ď |ck | and |ck`2| ď |ck |. Our functional equation (1) yields
Since ck and ck`2 have the same parity and |ck`2 | ď |ck |, this leaves us with three possi-
bilities: |ck`2 | “ |ck |, |ck`2 | “ |ck | ´ 2, and |ck | ´ 2 “ ˘2, ck`2 “ 0.
If |ck`2 | “ |ck | ´ 2, then f pck q “ ck`1 “ 1 ´ |ck |, which means that ck P Z´ or ck P Z` ,
and we reach a contradiction.
If |ck`2 | “ |ck |, then ck`1 “ 0, thus c2k`3 “ 4ck`2. So either ck`3 ‰ 0 or (by maximality
of |ck`2 | “ |ck |) ci “ 0 for all i. In the former case, we can repeat the entire argument
Shortlisted problems – solutions 25
with ck`2 in the place of ck . Now |ck`4 | “ |ck`2 | is not possible any more since ck`3 ‰ 0,
leaving us with the only possibility |ck | ´ 2 “ |ck`2 | ´ 2 “ ˘2.
Thus we know now that either all ci are equal to 0, or |ck | “ 4. If ck “ ˘4, then either
ck`1 “ 0 and |ck`2 | “ |ck | “ 4, or ck`2 “ 0 and ck`1 “ ´4. From this point onwards, all
elements of the sequence are either 0 or ˘4.
Let cr be the last element of the sequence that is not equal to 0 or ˘4 (if such an element
exists). Then cr`1 , cr`2 P t´4, 0, 4u, so
which gives us a contradiction. Thus all elements of the sequence are equal to 0 or ˘4,
and since the choice of c0 “ c was arbitrary, Z0 Ď t´4, 0, 4u.
Comment. All solutions known to the Problem Selection Committee are quite lengthy and technical,
as the two solutions presented here show. It is possible to make the problem easier by imposing
additional assumptions, such as f p0q ‰ 0 or f pnq ě 1 for all n ě 0, to remove some of the technicalities.
26 IMO 2014 South Africa
Combinatorics
C1. Let n points be given inside a rectangle R such that no two of them lie on a line parallel
to one of the sides of R. The rectangle R is to be dissected into smaller rectangles with sides
parallel to the sides of R in such a way that none of these rectangles contains any of the given
points in its interior. Prove that we have to dissect R into at least n ` 1 smaller rectangles.
(Serbia)
Solution 1. Let k be the number of rectangles in the dissection. The set of all points that
are corners of one of the rectangles can be divided into three disjoint subsets:
• A, which consists of the four corners of the original rectangle R, each of which is the
corner of exactly one of the smaller rectangles,
• B, which contains points where exactly two of the rectangles have a common corner
(T-junctions, see the figure below),
• C, which contains points where four of the rectangles have a common corner (crossings,
see the figure below).
We denote the number of points in B by b and the number of points in C by c. Since each
of the k rectangles has exactly four corners, we get
4k “ 4 ` 2b ` 4c.
It follows that 2b ď 4k ´ 4, so b ď 2k ´ 2.
Each of the n given points has to lie on a side of one of the smaller rectangles (but not
of the original rectangle R). If we extend this side as far as possible along borders between
rectangles, we obtain a line segment whose ends are T-junctions. Note that every point in B
can only be an endpoint of at most one such segment containing one of the given points, since
it is stated that no two of them lie on a common line parallel to the sides of R. This means
that
b ě 2n.
2k ´ 2 ě b ě 2n,
Solution 2. Let k denote the number of rectangles. In the following, we refer to the directions
of the sides of R as ‘horizontal’ and ‘vertical’ respectively. Our goal is to prove the inequality
k ě n ` 1 for fixed n. Equivalently, we can prove the inequality n ď k ´ 1 for each k, which
will be done by induction on k. For k “ 1, the statement is trivial.
Now assume that k ą 1. If none of the line segments that form the borders between the
rectangles is horizontal, then we have k ´ 1 vertical segments dividing R into k rectangles. On
each of them, there can only be one of the n points, so n ď k ´ 1, which is exactly what we
want to prove.
Otherwise, consider the lowest horizontal line h that contains one or more of these line
segments. Let R1 be the rectangle that results when everything that lies below h is removed
from R (see the example in the figure below).
The rectangles that lie entirely below h form blocks of rectangles separated by vertical line
segments. Suppose there are r blocks and ki rectangles in the ith block. The left and right
border of each block has to extend further upwards beyond h. Thus we can move any points
that lie on these borders upwards, so that they now lie inside R1 . This can be done without
violating the conditions, one only needs to make sure that they do not get to lie on a common
horizontal line with one of the other given points.
All other borders between rectangles in the ith block have to lie entirely below h. There are
ki ´ 1 such line segments, each of which can contain at most one of the given points. Finally,
there can be one point that lies on h. All other points have to lie in R1 (after moving some of
them as explained in the previous paragraph).
R′
C2. We have 2m sheets of paper, with the number 1 written on each of them. We perform
the following operation. In every step we choose two distinct sheets; if the numbers on the two
sheets are a and b, then we erase these numbers and write the number a ` b on both sheets.
Prove that after m2m´1 steps, the sum of the numbers on all the sheets is at least 4m .
(Iran)
Solution. Let Pk be the product of the numbers on the sheets after k steps.
Suppose that in the pk ` 1qth step the numbers a and b are replaced by a` b. In the product,
the number ab is replaced by pa ` bq2 , and the other factors do not change. Since pa ` bq2 ě 4ab,
we see that Pk`1 ě 4Pk . Starting with P0 “ 1, a straightforward induction yields
Pk ě 4 k
for all integers k ě 0; in particular
m´1 m
Pm¨2m´1 ě 4m¨2 “ p2m q2 ,
so by the AM–GM inequality, the sum of the numbers written on the sheets after m2m´1 steps
is at least
m
a
2m ¨ 2 Pm¨2m´1 ě 2m ¨ 2m “ 4m .
Comment 1. It is possible to achieve the sum 4m in m2m´1 steps. For example, starting from 2m
equal numbers on the sheets, in 2m´1 consecutive steps we can double all numbers. After m such
doubling rounds we have the number 2m on every sheet.
Comment 2. There are several versions of the solution above. E.g., one may try to assign to each
positive integer n a weight wn in such a way that the sum of the weights of the numbers written on
the sheets increases, say, by at least 2 in each step. For this purpose, one needs the inequality
2wa`b ě wa ` wb ` 2 (1)
to be satisfied for all positive integers a and b.
Starting from w1 “ 1 and trying to choose the weights as small as possible, one may find that
these weights can be defined as follows: For every positive integer n, one chooses k to be the maximal
integer such that n ě 2k , and puts
n ´ n¯
wn “ k ` k “ min d ` d . (2)
2 dPZě0 2
Now, in order to prove that these weights satisfy (1), one may take arbitrary positive integers a and b,
and choose an integer d ě 0 such that wa`b “ d ` a`b 2d
. Then one has
ˆ ˙
a`b ´ a ¯ b
2wa`b “ 2d ` 2 ¨ d “ pd ´ 1q ` d´1 ` pd ´ 1q ` d´1 ` 2 ě wa ` wb ` 2.
2 2 2
Since the initial sum of the weights was 2m , after m2m´1 steps the sum is at least pm ` 1q2m . To
finish the solution, one may notice that by (2) for every positive integer a one has
a
wa ď m ` m , i.e., a ě 2m p´m ` wa q. (3)
2
So the sum of the numbers a1 , a2 , . . . , a2m on the sheets can be estimated as
2m 2m 2m
ÿ ÿ ÿ
m m m m
ai ě 2 p´m ` wai q “ ´m2 ¨ 2 ` 2 wai ě ´m4m ` pm ` 1q4m “ 4m ,
i“1 i“1 i“1
as required.
For establishing the inequalities (1) and (3), one may also use the convexity argument, instead of
the second definition of wn in (2).
One may check that log2 n ď wn ď log2 n ` 1; thus, in some rough sense, this approach is obtained
by “taking the logarithm” of the solution above.
Shortlisted problems – solutions 29
Comment 3. An intuitive strategy to minimise the sum of numbers is that in every step we choose
the two smallest numbers. We may call this the greedy strategy. In the following paragraphs we prove
that the greedy strategy indeed provides the least possible sum of numbers.
Claim. Starting from any sequence x1 , . . . , xN of positive real numbers on N sheets, for any number
k of steps, the greedy strategy achieves the lowest possible sum of numbers.
Proof. We apply induction on k; for k “ 1 the statement is obvious. Let k ě 2, and assume that the
claim is true for smaller values.
` ˘
Every sequence of k steps can be encoded as S “ pi1 , j1 q, . . . , pik , jk q , where, for r “ 1, 2, . . . , k,
the numbers ir and jr are the indices of the two sheets that are chosen in the r th step. The resulting
final sum will be some linear combination of x1 , . . . , xN , say, c1 x1 ` ¨ ¨ ¨ ` cN xN with positive integers
c1 , . . . , cN that depend on S only. Call the numbers pc1 , . . . , cN q the characteristic vector of S.
` ˘
Choose a sequence S0 “ pi1 , j1 q, . . . , pik , jk q of steps that produces the minimal sum, starting
from x1 , . . . , xN , and let pc1 , . . . , cN q be the characteristic vector of S. We may assume that the sheets
are indexed in such an order that c1 ě c2 ě ¨ ¨ ¨ ě cN . If the sheets (and the numbers) are permuted by
a permutation π of the indices p1, 2, . . . , N q and then the same steps are performed, we can obtain the
řN
sum ct xπptq . By the rearrangement inequality, the smallest possible sum can be achieved when the
t“1
numbers px1 , . . . , xN q are in non-decreasing order. So we can assume that also x1 ď x2 ď ¨ ¨ ¨ ď xN .
Let ℓ be the largest index with c1 “ ¨ ¨ ¨ “ cℓ , and let the r th step be the first step for which cir “ c1
or cjr “ c1 . The role of ir and jr is symmetrical, so we can assume cir “ c1 and thus ir ď ℓ. We show
that cjr “ c1 and jr ď ℓ hold, too.
Before the r th step, on the ir th sheet we had the number xir . On the jr th sheet there was a linear
combination that contains the number xjr with a positive integer coefficient, and possibly some other
terms. In the r th step, the number xir joins that linear combination. From this point, each sheet
contains a linear combination of x1 , . . . , xN , with the coefficient of xjr being not smaller than the
coefficient of xir . This is preserved to the end of the procedure, so we have cjr ě cir . But cir “ c1 is
maximal among the coefficients, so we have cjr “ cir “ c1 and thus jr ď ℓ.
Either from cjr “ cir “ c1 or from the arguments in the previous paragraph we can see that none
of the ir th and the jr th sheets were used before step r. Therefore, the final linear combination of the
numbers does not change if the step pir , jr q is performed first: the sequence of steps
` ˘
S1 “ pir , jr q, pi1 , j1 q, . . . , pir´1 , jr´1 q, pir`1 , jr`1 q, . . . , piN , jN q
also produces the same minimal sum at the end. Therefore, we can replace S0 by S1 and we may
assume that r “ 1 and ci1 “ cj1 “ c1 .
As i1 ‰ j1 , we can see that ℓ ě 2 and c1 “ c2 “ ci1 “ cj1 . Let π be such a permutation of the
indices p1, 2, . . . , N q that exchanges 1, 2 with ir , jr and does not change the remaining indices. Let
` ˘
S2 “ pπpi1 q, πpj1 qq, . . . , pπpiN q, πpjN qq .
Since cπpiq “ ci for all indices i, this sequence of steps produces the same, minimal sum. Moreover, in
the first step we chose xπpi1 q “ x1 and xπpj1 q “ x2 , the two smallest numbers.
Hence, it is possible to achieve the optimal sum if we follow the greedy strategy in the first step.
By the induction hypothesis, following the greedy strategy in the remaining steps we achieve the
optimal sum.
30 IMO 2014 South Africa
r
6
r r r
5
4
3
r
2
r
1
0
0 1 2 3 4 5 6 7 8
Next, we show that each ℓ ˆ ℓ square A on the board contains a rook. Consider such a
square A, and consider ℓ consecutive rows the union of which contains A. Let the lowest of
these rows have number pℓ ` q with 0 ď p, q ď ℓ ´ 1 (notice that pℓ ` q ď ℓ2 ´ ℓ). Then the
rooks in this union are placed in the columns with numbers qℓ ` p, pq ` 1qℓ ` p, . . . , pℓ ´ 1qℓ ` p,
p ` 1, ℓ ` pp ` 1q, . . . , pq ´ 1qℓ ` p ` 1, or, putting these numbers in increasing order,
One readily checks that the first number in this list is at most ℓ ´ 1 (if p “ ℓ ´ 1, then q “ 0,
and the first listed number is qℓ ` p “ ℓ ´ 1), the last one is at least pℓ ´ 1qℓ, and the difference
between any two consecutive numbers is at most ℓ. Thus, one of the ℓ consecutive columns
intersecting A contains a number listed above, and the rook in this column is inside A, as
required. The construction for n “ ℓ2 is established.
Shortlisted problems – solutions 31
Comment. Part (i) allows several different proofs. E.g., in the last paragraph of the solution, it
suffices to deal only with the case n “ ℓ2 ` 1. Notice now that among the four corner squares, at
least one is empty. So the rooks in its row and in its column are distinct. Now, deleting this row and
column we obtain an ℓ2 ˆ ℓ2 square with ℓ2 ´ 1 rooks in it. This square can be partitioned into ℓ2
squares of size ℓ ˆ ℓ, so one of them is empty.
32 IMO 2014 South Africa
C4. Construct a tetromino by attaching two 2 ˆ 1 dominoes along their longer sides such
that the midpoint of the longer side of one domino is a corner of the other domino. This
construction yields two kinds of tetrominoes with opposite orientations. Let us call them S-
and Z-tetrominoes, respectively.
S-tetrominoes Z-tetrominoes
Assume that a lattice polygon P can be tiled with S-tetrominoes. Prove than no matter
how we tile P using only S- and Z-tetrominoes, we always use an even number of Z-tetrominoes.
(Hungary)
Solution 1. We may assume that polygon P is the union of some squares of an infinite
chessboard. Colour the squares of the chessboard with two colours as the figure below illustrates.
Observe that no matter how we tile P , any S-tetromino covers an even number of black
squares, whereas any Z-tetromino covers an odd number of them. As P can be tiled exclusively
by S-tetrominoes, it contains an even number of black squares. But if some S-tetrominoes and
some Z-tetrominoes cover an even number of black squares, then the number of Z-tetrominoes
must be even.
Comment. An alternative approach makes use of the following two colourings, which are perhaps
somewhat more natural:
Let s1 and s2 be the number of S-tetrominoes of the first and second type (as shown in the figure above)
respectively that are used in a tiling of P . Likewise, let z1 and z2 be the number of Z-tetrominoes of
the first and second type respectively. The first colouring shows that s1 ` z2 is invariant modulo 2, the
second colouring shows that s1 ` z1 is invariant modulo 2. Adding these two conditions, we find that
z1 ` z2 is invariant modulo 2, which is what we have to prove. Indeed, the sum of the two colourings
(regarding white as 0 and black as 1 and adding modulo 2) is the colouring shown in the solution.
Shortlisted problems – solutions 33
Solution 2. Let us assign coordinates to the squares of the infinite chessboard in such a way
that the squares of P have nonnegative coordinates only, and that the first coordinate increases
as one moves to the right, while the second coordinate increases as one moves upwards. Write
the integer 3i ¨ p´3qj into the square with coordinates pi, jq, as in the following figure:
..
.
..
81 .
..
27 81 .
9 27 81
3 9 27 81
1 3 9 27 81
The sum of the numbers written into four squares that can be covered by an S-tetromino
is either of the form
and thus divisible by 32. For this reason, the sum of the numbers written into the squares
of P , and thus also the sum of the numbers covered by Z-tetrominoes in the second covering,
is likewise divisible by 32. Now the sum of the entries of a Z-tetromino is either of the form
i.e., 16 times an odd number. Thus in order to obtain a total that is divisible by 32, an even
number of the latter kind of Z-tetrominoes needs to be used. Rotating everything by 90˝ , we
find that the number of Z-tetrominoes of the first kind is even as well. So we have even proven
slightly more than necessary.
Comment 1. In the second solution, 3 and ´3 can be replaced by other combinations as well.
For example, for any positive integer a ” 3 pmod 4q, we can write ai ¨ p´aqj into the square with
coordinates pi, jq and apply the same argument.
Comment 2. As the second solution shows, we even have the stronger result that the parity of the
number of each of the four types of tetrominoes in a tiling of P by S- and Z-tetrominoes is an invariant
of P . This also remains true if there is no tiling of P that uses only S-tetrominoes.
34 IMO 2014 South Africa
C5. Consider n ě 3 lines in the plane such that no two lines are parallel and no three have a
common point. These lines divide the plane into polygonal
Pa regions;
T let F be the set of regions
having finite area. Prove that it is possible to colour n{2 of the lines blue in such a way
that no region in F has a completely blue boundary. (For a real number x, rxs denotes the
least integer which is not smaller than x.)
(Austria)
Solution. Let L be the given set of lines. Choose a maximal (by inclusion) subset B Ď L such
that when we colour the lines Paof B Tblue, no region in F has a completely blue boundary. Let
|B| “ k. We claim that k ě n{2 .
Let us colour all the
`k˘lines of LzB red. Call a point blue if it is the intersection of two blue
lines. Then there are 2 blue points.
Now consider any red line ℓ. By the maximality of B, there exists at least one region A P F
whose only red side lies on ℓ. Since A has at least three sides, it must have at least one blue
vertex. Let us take one such vertex and associate it to ℓ.
Since each blue point belongs to four regions (some of which may be unbounded), ` ˘ it is
associated to at most four red lines. Thus the total number of red lines is at most 4 k2 . On
the other hand, this number is n ´ k, so
Comment 1. The constant factor in the estimate can be improved in different ways; we sketch
two of them below. On the other hand, the Problem Selection Committee is not aware of any results
?
showing that it is sometimes impossible to colour k lines satisfying the desired condition for k " n.
In this situation we find it more suitable to keep the original formulation of the problem.
Pa T
1. Firstly, we show that in the proof above one has in fact k “ |B| ě 2n{3 .
Let us make weighted associations as follows. Let a region A whose only red side lies on ℓ have
k vertices, so that k ´ 2 of them are blue. We associate each of these blue vertices to ℓ, and put the
1
weight k´2 on each such association. So the sum of the weights of all the associations is exactly n ´ k.
Now, one may check that among the four regions adjacent to a blue vertex v, at most two are trian-
gles. This means that the sum of the weights of all associations involving v is at most 1 ` 1 ` 21 ` 12 “ 3.
This leads to the estimate ˆ ˙
k
n´k ď3 ,
2
or
2n ď 3k2 ´ k ă 3k2 ,
Pa T
which yields k ě 2n{3 .
?
2. Next, we even show that k “ |B| ě r n s. For this, we specify the process of associating points
to red lines in one more different way.
Call a point red if it lies on a red line as well as on a blue line. Consider any red line ℓ, and take an
arbitrary region A P F whose only red side lies on ℓ. Let r 1 , r, b1 , . . . , bk be its vertices in clockwise
order with r 1 , r P ℓ; then the points r 1 , r are red, while all the points b1 , . . . , bk are blue. Let us
associate to ℓ the red point r and the blue point b1 . One may notice that to each pair of a red point r
and a blue point b, at most one red line can be associated, since there is at most one region A having
r and b as two clockwise consecutive vertices.
We claim now that at most two red lines are associated to each blue point b; this leads to the
desired bound ˆ ˙
k
n´k ď2 ðñ n ď k2 .
2
Shortlisted problems – solutions 35
Assume, to the contrary, that three red lines ℓ1 , ℓ2 , and ℓ3 are associated to the same blue point b.
Let r1 , r2 , and r3 respectively be the red points associated to these lines; all these points are distinct.
The point b defines four blue rays, and each point ri is the red point closest to b on one of these rays.
So we may assume that the points r2 and r3 lie on one blue line passing through b, while r1 lies on
the other one.
ℓ1
r1
r3
A b
r2
Now consider the region A used to associate r1 and b with ℓ1 . Three of its clockwise consecutive
vertices are r1 , b, and either r2 or r3 (say, r2 ). Since A has only one red side, it can only be the
triangle r1 br2 ; but then both ℓ1 and ℓ2 pass through r2 , as well as some blue line. This is impossible
by the problem assumptions.
Comment 2. The condition that the lines be non-parallel is essentially not used in the solution, nor
in the previous comment; thus it may be omitted.
36 IMO 2014 South Africa
C6. We are given an infinite deck of cards, each with a real number on it. For every real
number x, there is exactly one card in the deck that has x written on it. Now two players draw
disjoint sets A and B of 100 cards each from this deck. We would like to define a rule that
declares one of them a winner. This rule should satisfy the following conditions:
1. The winner only depends on the relative order of the 200 cards: if the cards are laid down
in increasing order face down and we are told which card belongs to which player, but
not what numbers are written on them, we can still decide the winner.
2. If we write the elements of both sets in increasing order as A “ ta1 , a2 , . . . , a100 u and
B “ tb1 , b2 , . . . , b100 u, and ai ą bi for all i, then A beats B.
3. If three players draw three disjoint sets A, B, C from the deck, A beats B and B beats C,
then A also beats C.
How many ways are there to define such a rule? Here, we consider two rules as different if there
exist two sets A and B such that A beats B according to one rule, but B beats A according to
the other.
(Russia)
Answer. 100.
Solution 1. We prove a more general statement for sets of cardinality n (the problem being
the special case n “ 100, then the answer is n). In the following, we write A ą B or B ă A for
“A beats B”.
Part I. Let us first define n different rules that satisfy the conditions. To this end, fix an
index k P t1, 2, . . . , nu. We write both A and B in increasing order as A “ ta1 , a2 , . . . , an u and
B “ tb1 , b2 , . . . , bn u and say that A beats B if and only if ak ą bk . This rule clearly satisfies all
three conditions, and the rules corresponding to different k are all different. Thus there are at
least n different rules.
Part II. Now we have to prove that there is no other way to define such a rule. Suppose
that our rule satisfies the conditions, and let k P t1, 2, . . . , nu be minimal with the property
that
Clearly, such a k exists, since this holds for k “ n by assumption. Now consider two disjoint sets
X “ tx1 , x2 , . . . , xn u and Y “ ty1 , y2 , . . . , yn u, both in increasing order (i.e., x1 ă x2 ă ¨ ¨ ¨ ă xn
and y1 ă y2 ă ¨ ¨ ¨ ă yn ). We claim that X ă Y if (and only if – this follows automatically)
xk ă yk .
To prove this statement, pick arbitrary real numbers ui , vi , wi R X Y Y such that
and
xk ă v1 ă v2 ă ¨ ¨ ¨ ă vk ă w1 ă w2 ă ¨ ¨ ¨ ă wn ă uk ă uk`1 ă ¨ ¨ ¨ ă un ă yk ,
and set
U “ tu1 , u2, . . . , un u, V “ tv1 , v2 , . . . , vn u, W “ tw1 , w2, . . . , wn u.
Then
• ui ă yi and xi ă vi for all i, so U ă Y and X ă V by the second condition.
Shortlisted problems – solutions 37
• The elements of U Y W are ordered in the same way as those of Ak´1 Y Bk´1 , and since
Ak´1 ą Bk´1 by our choice of k, we also have U ą W (if k “ 1, this is trivial).
• The elements of V Y W are ordered in the same way as those of Ak Y Bk , and since
Ak ă Bk by our choice of k, we also have V ă W .
It follows that
X ă V ă W ă U ă Y,
so X ă Y by the third condition, which is what we wanted to prove.
and ` ˘ ` ˘
t2i ´ 1 | 1 ď i ď n ´ 1u Y t2nu ă t2i | 1 ď i ď n ´ 1u Y t2n ´ 1u
holds.
Proof. Suppose that the first relation does not hold. Since our rule may only depend on the
relative order, we must also have
` ˘ ` ˘
t2u Y t3i ´ 2 | 2 ď i ď n ´ 1u Y t3n ´ 2u ą t1u Y t3i ´ 1 | 2 ď i ď n ´ 1u Y t3nu .
Likewise, if the second relation does not hold, then we must also have
` ˘ ` ˘
t1u Y t3i ´ 1 | 2 ď i ď n ´ 1u Y t3nu ą t3u Y t3i | 2 ď i ď n ´ 1u Y t3n ´ 1u .
the elements in pBx ´ εq Y pBy ` εq is the same as for the two sets t2u Y t2i ´ 1 | 2 ď i ď nu
and t1u Y t2i | 2 ď i ď nu, so that Bx ´ ε ă By ` ε. In either case, we obtain
A ă Bx ´ ε ă By ` ε ă A,
Comment. The problem asks for all possible partial orders on the set of n-element subsets of R such
that any two disjoint sets are comparable, the order relation only depends on the relative order of the
elements, and ta1 , a2 , . . . , an u ă tb1 , b2 , . . . , bn u whenever ai ă bi for all i.
As the proposer points out, one may also ask for all total orders on all n-element subsets of R
(dropping the condition of disjointness and requiring that ta1 , a2 , . . . , an u ĺ tb1 , b2 , . . . , bn u whenever
ai ď bi for all i). It turns out that the number of possibilities in this case is n!, and all possible total
orders are obtained in the following way. Fix a permutation σ P Sn . Let A “ ta1 , a2 , . . . , an u and
B “ tb1 , b2 , . . . , bn u be two subsets of R with a1 ă a2 ă ¨ ¨ ¨ ă an and b1 ă b2 ă ¨ ¨ ¨ ă bn . Then we say
that A ąσ B if and only if paσp1q , . . . , aσpnq q is lexicographically greater than pbσp1q , . . . , bσpnq q.
It seems, however, that this formulation adds rather more technicalities to the problem than
additional ideas.
Shortlisted problems – solutions 39
C7. Let M be a set of n ě 4 points in the plane, no three of which are collinear. Initially these
points are connected with n segments so that each point in M is the endpoint of exactly two
segments. Then, at each step, one may choose two segments AB and CD sharing a common
interior point and replace them by the segments AC and BD if none of them is present at this
moment. Prove that it is impossible to perform n3 {4 or more such moves.
(Russia)
Solution. A line is said to be red if it contains two points of M. As no three points of M are
collinear,
`n˘ n2 each red line determines a unique pair of points of M. Moreover, there are precisely
2
ă 2 red lines. By the value of a segment we mean the number of red lines intersecting it
in its interior, and the value of a set of segments is defined to be the sum of the values of its
elements. We will prove that piq the value of the initial set of segments is smaller than n3 {2
and that piiq each step decreases the value of the set of segments present by at least 2. Since
such a value can never be negative, these two assertions imply the statement of the problem.
To show piq we just need to observe that each segment has a value that is smaller than n2 {2.
Thus the combined value of the n initial segments is indeed below n ¨ n2 {2 “ n3 {2.
It remains to establish piiq. Suppose that at some moment we have two segments AB
and CD sharing an interior point S, and that at the next moment we have the two segments
AC and BD instead. Let XAB denote the set of red lines intersecting the segment AB in
its interior and let the sets XAC , XBD , and XCD be defined similarly. We are to prove that
|XAC | ` |XBD | ` 2 ď |XAB | ` |XCD |.
As a first step in this direction, we claim that
Indeed, if g is a red line intersecting, e.g. the segment AC in its interior, then it has to
intersect the triangle ACS once again, either in the interior of its side AS, or in the interior of
its side CS, or at S, meaning that it belongs to XAB or to XCD (see Figure 1). Moreover, the
red lines AB and CD contribute to XAB Y XCD but not to XAC Y XBD . Thereby (1) is proved.
h h
D D D
B B B
g S S S
C A C A C A
Indeed, a red line h appearing in XAC X XBD belongs, for similar reasons as above, also to
XAB X XCD . To make the argument precise, one may just distinguish the cases S P h (see
Figure 2) and S R h (see Figure 3). Thereby (2) is proved.
Adding (1) and (2) we obtain the desired conclusion, thus completing the solution of this
problem.
Shortlisted problems – solutions 41
Comment 1. There is a problem belonging to the folklore, in the solution of which one may use the
same kind of operation:
Given n red and n green points in the plane, prove that one may draw n nonintersecting segments
each of which connects a red point with a green point.
A standard approach to this problem consists in taking n arbitrary segments connecting the red
points with the green points, and to perform the same operation as in the above proposal whenever
an intersection occurs. Now each time one performs such a step, the total length of the segments that
are present decreases due to the triangle inequality. So, as there are only finitely many possibilities
for the set of segments present, the process must end at some stage.
In the above proposal, however, considering the sum of the Euclidean lengths of the segment that
are present does not seem to help much, for even though it shows that the process must necessarily
terminate after some finite number of steps, it does not seem to easily yield any upper bound on the
number of these steps that grows polynomially with n.
One may regard the concept of the value of a segment introduced in the above solution as an
appropriately discretised version of Euclidean length suitable for obtaining such a bound.
The Problem Selection Committee still believes the problem to be sufficiently original for the
competition.
Comment 2. There are some other essentially equivalent ways of presenting the same solution. E.g.,
put M “ tA1 , A2 , . . . , An u, denote the set of segments present at any moment by te1 , e2 , . . . , en u, and
called a triple pi, j, kq of indices with i ‰ j intersecting, if the line Ai Aj intersects the segment ek . It
may then be shown that the number S of intersecting triples satisfies 0 ď S ă n3 at the beginning
and decreases by at least 4 in each step.
Comment 3. It is not difficult to construct an example where cn2 moves are possible (for some
absolute constant c ą 0). It would be interesting to say more about the gap between cn2 and cn3 .
42 IMO 2014 South Africa
C8. A card deck consists of 1024 cards. On each card, a set of distinct decimal digits is
written in such a way that no two of these sets coincide (thus, one of the cards is empty). Two
players alternately take cards from the deck, one card per turn. After the deck is empty, each
player checks if he can throw out one of his cards so that each of the ten digits occurs on an
even number of his remaining cards. If one player can do this but the other one cannot, the
one who can is the winner; otherwise a draw is declared.
Determine all possible first moves of the first player after which he has a winning strategy.
(Russia)
Answer. All the moves except for taking the empty card.
Solution. Let us identify each card with the set of digits written on it. For any collection of
cards C1 , C2 , . . . , Ck denote by their sum the set C1 △ C2 △ ¨ ¨ ¨ △ Ck consisting of all elements
belonging to an odd number of the Ci ’s. Denote the first and the second player by F and S,
respectively.
Since each digit is written on exactly 512 cards, the sum of all the cards is ∅. Therefore,
at the end of the game the sum of all the cards of F will be the same as that of S; denote this
sum by C. Then the player who took C can throw it out and get the desired situation, while
the other one cannot. Thus, the player getting card C wins, and no draw is possible.
Now, given a nonempty card B, one can easily see that all the cards can be split into 512
pairs of the form pX, X △Bq because pX △Bq△B “ X. The following lemma shows a property
of such a partition that is important for the solution.
Lemma. Let B ‰ ∅ be some card. Let us choose 512 cards so that exactly one card is chosen
from every pair pX, X △ Bq. Then the sum of all chosen cards is either ∅ or B.
Proof. Let b be some element of B. Enumerate the pairs; let Xi be the card not containing b
in the ith pair, and let Yi be the other card in this pair. Then the sets Xi are exactly all the
sets not containing b, therefore each digit a ‰ b is written on exactly 256 of these cards, so
X1 △ X2 △ ¨ ¨ ¨ △ X512 “ ∅. Now, if we replace some summands in this sum by the other
elements from their pairs, we will simply add B several times to this sum, thus the sum will
either remain unchanged or change by B, as required. l
Now we consider two cases.
Case 1. Assume that F takes the card ∅ on his first move. In this case, we present a
winning strategy for S.
Let S take an arbitrary card A. Assume that F takes card B after that; then S takes A △ B.
Split all 1024 cards into 512 pairs of the form pX, X △Bq; we call two cards in one pair partners.
Then the four cards taken so far form two pairs p∅, Bq and pA, A △ Bq belonging to F and S,
respectively. On each of the subsequent moves, when F takes some card, S should take the
partner of this card in response.
Consider the situation at the end of the game. Let us for a moment replace card A belonging
to S by ∅. Then he would have one card from each pair; by our lemma, the sum of all these
cards would be either ∅ or B. Now, replacing ∅ back by A we get that the actual sum of the
cards of S is either A or A △ B, and he has both these cards. Thus S wins.
Case 2. Now assume that F takes some card A ‰ ∅ on his first move. Let us present a
winning strategy for F in this case.
Assume that S takes some card B ‰ ∅ on his first move; then F takes A △ B. Again, let
us split all the cards into pairs of the form pX, X △ Bq; then the cards which have not been
taken yet form several complete pairs and one extra element (card ∅ has not been taken while
its partner B has). Now, on each of the subsequent moves, if S takes some element from a
Shortlisted problems – solutions 43
complete pair, then F takes its partner. If S takes the extra element, then F takes an arbitrary
card Y , and the partner of Y becomes the new extra element.
Thus, on his last move S is forced to take the extra element. After that player F has cards
A and A △ B, player S has cards B and ∅, and F has exactly one element from every other
pair. Thus the situation is the same as in the previous case with roles reversed, and F wins.
Finally, if S takes ∅ on his first move then F denotes any card which has not been taken
yet by B and takes A △ B. After that, the same strategy as above is applicable.
Comment 1. If one wants to avoid the unusual question about the first move, one may change the
formulation as follows. (The difficulty of the problem would decrease somewhat.)
A card deck consists of 1023 cards; on each card, a nonempty set of distinct decimal digits is
written in such a way that no two of these sets coincide. Two players alternately take cards from
the deck, one card per turn. When the deck is empty, each player checks if he can throw out one of
his cards so that for each of the ten digits, he still holds an even number of cards with this digit. If
one player can do this but the other one cannot, the one who can is the winner; otherwise a draw is
declared.
Determine which of the players (if any) has a winning strategy.
The winner in this version is the first player. The analysis of the game from the first two paragraphs
of the previous solution applies to this version as well, except for the case C “ ∅ in which the result
is a draw. Then the strategy for S in Case 1 works for F in this version: the sum of all his cards at
the end is either A or A △ B, thus nonempty in both cases.
Comment 2. Notice that all the cards form a vector space over F2 , with △ the operation of addition.
Due to the automorphisms of this space, all possibilities for F’s first move except ∅ are equivalent.
The same holds for the response by S if F takes the card ∅ on his first move.
Comment 3. It is not that hard to show that in the initial game, F has a winning move, by the
idea of “strategy stealing”.
Namely, assume that S has a winning strategy. Let us take two card decks and start two games, in
which S will act by his strategy. In the first game, F takes an arbitrary card A1 ; assume that S takes
some B1 in response. Then F takes the card B1 at the second game; let the response by S be A2 .
Then F takes A2 in the first game and gets a response B2 , and so on.
This process stops at some moment when in the second game S takes Ai “ A1 . At this moment
the players hold the same sets of cards in both games, but with roles reversed. Now, if some cards
remain in the decks, F takes an arbitrary card from the first deck starting a similar cycle.
At the end of the game, player F’s cards in the first game are exactly player S’s cards in the second
game, and vice versa. Thus in one of the games F will win, which is impossible by our assumption.
One may notice that the strategy in Case 2 is constructed exactly in this way from the strategy
in Case 1. This is possible since every response by S wins if F takes the card ∅ on his first move.
44 IMO 2014 South Africa
C9. There are n circles drawn on a piece of paper in such a way that any two circles
intersect in two points, and no three circles pass through the same point. Turbo the snail slides
along the circles in the following fashion. Initially he moves on one of the circles in clockwise
direction. Turbo always keeps sliding along the current circle until he reaches an intersection
with another circle. Then he continues his journey on this new circle and also changes the
direction of moving, i.e. from clockwise to anticlockwise or vice versa.
Suppose that Turbo’s path entirely covers all circles. Prove that n must be odd.
(India)
Solution 1. Replace every cross (i.e. intersection of two circles) by two small circle arcs that
indicate the direction in which the snail should leave the cross (see Figure 1.1). Notice that
the placement of the small arcs does not depend on the direction of moving on the curves; no
matter which direction the snail is moving on the circle arcs, he will follow the same curves
(see Figure 1.2). In this way we have a set of curves, that are the possible paths of the snail.
Call these curves snail orbits or just orbits. Every snail orbit is a simple closed curve that has
no intersection with any other orbit.
anticlockwise anticlockwise
clockwise clockwise
anticlockwise anticlockwise
clockwise
clockwise
c d c d
b b
Figure 2
Consider what happens to the number of orbits when a cross is flipped. Denote by a, b, c,
and d the four arcs that meet at the cross such that a and b belong to the same circle. Before
the flipping a and b were connected to c and d, respectively, and after the flipping a and b are
connected to d and c, respectively.
The orbits passing through the cross are closed curves, so each of the arcs a, b, c, and d is
connected to another one by orbits outside the cross. We distinguish three cases.
Case 1: a is connected to b and c is connected to d by the orbits outside the cross (see
Figure 3.1).
Shortlisted problems – solutions 45
We show that this case is impossible. Remove the two small arcs at the cross, connect a
to b, and connect c to d at the cross. Let γ be the new closed curve containing a and b, and
let δ be the new curve that connects c and d. These two curves intersect at the cross. So one
of c and d is inside γ and the other one is outside γ. Then the two closed curves have to meet
at least one more time, but this is a contradiction, since no orbit can intersect itself.
a a a
γ a a
c d d d
c c c d c d
?!
b δ b b b b
change (±)
Figure 4 Figure 5
Double-count the total curvature of all orbits. Along every circle the total curvature is 2π.
At every cross, the two turnings make two changes with some angles having the same absolute
value but opposite signs, as depicted in Figure 5. So the changes in the direction at the crosses
cancel out. Hence, the total curvature is n ¨ 2π.
Now we have pP ´ Nq ¨ 2π “ n ¨ 2π, so P ´ N “ n. The number of (modified) orbits is
P ` N, that has a same parity as P ´ N “ n.
46 IMO 2014 South Africa
• Type-1 step: An arc of a circle is moved over an arc of another circle; such a step creates
or removes two intersections.
• Type-2 step: An arc of a circle is moved through the intersection of two other circles.
Type-1 Type-2
We assume that in every step only one circle is moved, and that this circle is moved over at
most one arc or intersection point of other circles.
We will show that the parity of the number of orbits does not change in any step. As every
circle becomes a separate orbit at the end of the procedure, this fact proves p˚q.
Consider what happens to the number of orbits when a Type-1 step is performed. The two
intersection points are created or removed in a small neighbourhood. Denote some points of the
two circles where they enter or leave this neighbourhood by a, b, c, and d in this order around
the neighbourhood; let a and b belong to one circle and let c and d belong to the other circle.
The two circle arcs may have the same or opposite orientations. Moreover, the four end-points
of the two arcs are connected by the other parts of the orbits. This can happen in two ways
without intersection: either a is connected to d and b is connected to c, or a is connected to b
and c is connected to d. Altogether we have four cases, as shown in Figure 7.
d d d d a d a d
a d a d
a a a a
b b b b
c b c b c
c c c b c b c
Figure 7
We can see that the number of orbits is changed by ´2 or `2 in the leftmost case when the
arcs have the same orientation, a is connected to d, and b is connected to c. In the other three
cases the number of orbits is not changed. Hence, Type-1 steps do not change the parity of the
number of orbits.
Now consider a Type-2 step. The three circles enclose a small, triangular region; by the
step, this triangle is replaced by another triangle. Again, the modification of the orbits is done
in some small neighbourhood; the structure does not change outside. Each side of the triangle
shaped region can be convex or concave; the number of concave sides can be 0, 1, 2 or 3, so
there are 4 possible arrangements of the orbits inside the neighbourhood, as shown in Figure 8.
Shortlisted problems – solutions 47
a f a f a f a f
b e b e b e b e
c d c d c d c d
b b b b
e e e e
c d c d c d c d
Figure 9
We can see that if only neighbouring points are connected, then the number of orbits is
changed by `2 or ´2. If two opposite points are connected (a and d in the figure), then the
orbits are re-arranged, but their number is unchanged. Hence, Type-2 steps also preserve the
parity. This completes the proof of p˚q.
Solution 3. Like in the previous solutions, we do not need all circle pairs to intersect but we
assume that the circles form a connected set. Denote by C and P the sets of circles and their
intersection points, respectively.
The circles divide the plane into several simply connected, bounded regions and one un-
bounded region. Denote the set of these regions by R. We say that an intersection point or
a region is odd or even if it is contained inside an odd or even number of circles, respectively.
Let Podd and Rodd be the sets of odd intersection points and odd regions, respectively.
Claim.
|Rodd | ´ |Podd | ” n pmod 2q. p1q
Proof. For each circle c P C, denote by Rc , Pc , and Xc the number of regions inside c, the
number of intersection points inside c, and the number of circles intersecting c, respectively.
The circles divide each other into several arcs; denote by Ac the number of such arcs inside c.
By double counting the regions and intersection points inside the circles we get
ÿ ÿ
|Rodd | ” Rc pmod 2q and |Podd | ” Pc pmod 2q.
cPC cPC
48 IMO 2014 South Africa
For each circle c, apply Euler’s polyhedron theorem to the (simply connected) regions in c.
There are 2Xc intersection points on c; they divide the circle into 2Xc arcs. The polyhedron
theorem yields pRc ` 1q ` pPc ` 2Xc q “ pAc ` 2Xc q ` 2, considering the exterior of c as a single
region. Therefore,
Rc ` Pc “ Ac ` 1. p2q
Moreover, we have four arcs starting from every interior points inside c and a single arc
starting into the interior from each intersection point on the circle. By double-counting the
end-points of the interior arcs we get 2Ac “ 4Pc ` 2Xc , so
Ac “ 2Pc ` Xc . p3q
Rc ´ Pc “ Xc ` 1. p4q
which yields ÿ
|Rodd | ´ |Podd | ” Xc ` n pmod 2q. p5q
cPC
ř
Notice that in Xc each intersecting circle pair is counted twice, i.e., for both circles in the
cPC
pair, so ÿ
Xc ” 0 pmod 2q,
cPC
Now insert the same small arcs at the intersections as in the first solution, and suppose that
there is a single snail orbit b.
First we show that the odd regions are inside the curve b, while the even regions are outside.
Take a region r P R and a point x in its interior, and draw a ray y, starting from x, that does
not pass through any intersection point of the circles and is neither tangent to any of the circles.
As is well-known, x is inside the curve b if and only if y intersects b an odd number of times
(see Figure 10). Notice that if an arbitrary circle c contains x in its interior, then c intersects y
at a single point; otherwise, if x is outside c, then c has 2 or 0 intersections with y. Therefore,
y intersects b an odd number of times if and only if x is contained in an odd number of circles,
so if and only if r is odd.
y
x
r
Figure 10
Now consider an intersection point p of two circles c1 and c2 and a small neighbourhood
around p. Suppose that p is contained inside k circles.
Shortlisted problems – solutions 49
We have four regions that meet at p. Let r1 be the region that lies outside both c1 and c2 ,
let r2 be the region that lies inside both c1 and c2 , and let r3 and r4 be the two remaining
regions, each lying inside exactly one of c1 and c2 . The region r1 is contained inside the same
k circles as p; the region r2 is contained also by c1 and c2 , so by k ` 2 circles in total; each of
the regions r3 and r4 is contained inside k ` 1 circles. After the small arcs have been inserted
at p, the regions r1 and r2 get connected, and the regions r3 and r4 remain separated at p (see
Figure 11). If p is an odd point, then r1 and r2 are odd, so two odd regions are connected at p.
Otherwise, if p is even, then we have two even regions connected at p.
r1
c1 c2
p
r3 r4
r2
Figure 11 Figure 12
Consider the system of odd regions and their connections at the odd points as a graph.
In this graph the odd regions are the vertices, and each odd point establishes an edge that
connects two vertices (see Figure 12). As b is a single closed curve, this graph is connected and
contains no cycle, so the graph is a tree. Then the number of vertices must be by one greater
than the number of edges, so
|Rodd | ´ |Podd | “ 1. p9q
The relations (1) and (9) together prove that n must be odd.
Comment. For every odd n there exists at least one configuration of n circles with a single snail orbit.
˝
Figure 13 shows a possible configuration with 5 circles. In general, if a circle is rotated by k ¨ 360 n
(k “ 1, 2, . . . , n ´ 1q around an interior point other than the centre, the circle and its rotated copies
together provide a single snail orbit.
Figure 13
50 IMO 2014 South Africa
Geometry
G1. The points P and Q are chosen on the side BC of an acute-angled triangle ABC so
that =P AB “ =ACB and =QAC “ =CBA. The points M and N are taken on the rays AP
and AQ, respectively, so that AP “ P M and AQ “ QN. Prove that the lines BM and CN
intersect on the circumcircle of the triangle ABC.
(Georgia)
Solution 1. Denote by S the intersection point of the lines BM and CN. Let moreover
β “ =QAC “ =CBA and γ “ =P AB “ =ACB. From these equalities it follows that the
triangles ABP and CAQ are similar (see Figure 1). Therefore we obtain
BP BP AQ NQ
“ “ “ .
PM PA QC QC
Moreover,
=BP M “ β ` γ “ =CQN .
Hence the triangles BP M and NQC are similar. This gives =BMP “ =NCQ, so the trian-
gles BP M and BSC are also similar. Thus we get
=CSB “ =BP M “ β ` γ “ 180˝ ´ =BAC ,
which completes the solution.
A
A
γ β
β β +γ γ B C
B C Q P
Q P
L
S
S
K
N M N X M
Figure 1 Figure 2
Solution 2. As in the previous solution, denote by S the intersection point of the lines BM
and NC. Let moreover the circumcircle of the triangle ABC intersect the lines AP and AQ
again at K and L, respectively (see Figure 2).
Note that =LBC “ =LAC “ =CBA and similarly =KCB “ =KAB “ =BCA. It implies
that the lines BL and CK meet at a point X, being symmetric to the point A with respect
to the line BC. Since AP “ P M and AQ “ QN, it follows that X lies on the line MN.
Therefore, using Pascal’s theorem for the hexagon ALBSCK, we infer that S lies on the
circumcircle of the triangle ABC, which finishes the proof.
Comment. Both solutions can be modified to obtain a more general result, with the equalities
AP “ P M and AQ “ QN
replaced by
AP QN
“ .
PM AQ
Shortlisted problems – solutions 51
G2. Let ABC be a triangle. The points K, L, and M lie on the segments BC, CA, and AB,
respectively, such that the lines AK, BL, and CM intersect in a common point. Prove that it
is possible to choose two of the triangles ALM, BMK, and CKL whose inradii sum up to at
least the inradius of the triangle ABC.
(Estonia)
Solution. Denote
BK CL AM
a“ , b“ , c“ .
KC LA MB
By Ceva’s theorem, abc “ 1, so we may, without loss of generality, assume that a ě 1. Then at
least one of the numbers b or c is not greater than 1. Therefore at least one of the pairs pa, bq,
pb, cq has its first component not less than 1 and the second one not greater than 1. Without
loss of generality, assume that 1 ď a and b ď 1.
Therefore, we obtain bc ď 1 and 1 ď ca, or equivalently
AM LA MB BK
ď and ď .
MB CL AM KC
The first inequality implies that the line passing through M and parallel to BC intersects the
segment AL at a point X (see Figure 1). Therefore the inradius of the triangle ALM is not
less than the inradius r1 of triangle AMX.
Similarly, the line passing through M and parallel to AC intersects the segment BK at
a point Y , so the inradius of the triangle BMK is not less than the inradius r2 of the trian-
gle BMY . Thus, to complete our solution, it is enough to show that r1 ` r2 ě r, where r is
the inradius of the triangle ABC. We prove that in fact r1 ` r2 “ r.
C
L
K
X
Y
r
r1 r2
A M B
Figure 1
Since MX k BC, the dilation with centre A that takes M to B takes the incircle of the
triangle AMX to the incircle of the triangle ABC. Therefore
r1 AM r2 MB
“ , and similarly “ .
r AB r AB
Adding these equalities gives r1 ` r2 “ r, as required.
Comment. Alternatively, one can use Desargues’ theorem instead of Ceva’s theorem, as follows:
The lines AB, BC, CA dissect the plane into seven regions. One of them is bounded, and amongst
the other six, three are two-sided and three are three-sided. Now define the points P “ BC X LM ,
Q “ CA X M K, and R “ AB X KL (in the projective plane). By Desargues’ theorem, the points P ,
Q, R lie on a common line ℓ. This line intersects only unbounded regions. If we now assume (without
loss of generality) that P , Q and R lie on ℓ in that order, then one of the segments P Q or QR lies
inside a two-sided region. If, for example, this segment is P Q, then the triangles ALM and BM K
will satisfy the statement of the problem for the same reason.
52 IMO 2014 South Africa
G3. Let Ω and O be the circumcircle and the circumcentre of an acute-angled triangle ABC
with AB ą BC. The angle bisector of =ABC intersects Ω at M ‰ B. Let Γ be the circle
with diameter BM. The angle bisectors of =AOB and =BOC intersect Γ at points P and Q,
respectively. The point R is chosen on the line P Q so that BR “ MR. Prove that BR k AC.
(Here we always assume that an angle bisector is a ray.)
(Russia)
Solution. Let K be the midpoint of BM, i.e., the centre of Γ. Notice that AB ‰ BC implies
K ‰ O. Clearly, the lines OM and OK are the perpendicular bisectors of AC and BM,
respectively. Therefore, R is the intersection point of P Q and OK.
Let N be the second point of intersection of Γ with the line OM. Since BM is a diameter
of Γ, the lines BN and AC are both perpendicular to OM. Hence BN k AC, and it suffices to
prove that BN passes through R. Our plan for doing this is to interpret the lines BN, OK,
and P Q as the radical axes of three appropriate circles.
Let ω be the circle with diameter BO. Since =BNO “ =BKO “ 90˝ , the points N and K
lie on ω.
Next we show that the points O, K, P , and Q are concyclic. To this end, let D and E
be the midpoints of BC and AB, respectively. Clearly, D and E lie on the rays OQ and OP ,
respectively. By our assumptions about the triangle ABC, the points B, E, O, K, and D
lie in this order on ω. It follows that =EOR “ =EBK “ =KBD “ =KOD, so the line
KO externally bisects the angle P OQ. Since the point K is the centre of Γ, it also lies on
the perpendicular bisector of P Q. So K coincides with the midpoint of the arc P OQ of the
circumcircle γ of triangle P OQ.
Thus the lines OK, BN, and P Q are pairwise radical axes of the circles ω, γ, and Γ. Hence
they are concurrent at R, as required.
R N B γ
Q
E
E
D
O
ω
ω
K
A C
M
Shortlisted problems – solutions 53
G4. Consider a fixed circle Γ with three fixed points A, B, and C on it. Also, let us fix
a real number λ P p0, 1q. For a variable point P R tA, B, Cu on Γ, let M be the point on
the segment CP such that CM “ λ ¨ CP . Let Q be the second point of intersection of the
circumcircles of the triangles AMP and BMC. Prove that as P varies, the point Q lies on a
fixed circle.
(United Kingdom)
Solution 1. Throughout the solution, we denote by >pa, bq the directed angle between the
lines a and b.
Let D be the point on the segment AB such that BD “ λ ¨ BA. We will show that either
Q “ D, or >pDQ, QBq “ >pAB, BCq; this would mean that the point Q varies over the
constant circle through D tangent to BC at B, as required.
Denote the circumcircles of the triangles AMP and BMC by ωA and ωB , respectively. The
lines AP , BC, and MQ are pairwise radical axes of the circles Γ, ωA , and ωB , thus either they
are parallel, or they share a common point X.
Assume that these lines are parallel (see Figure 1). Then the segments AP , QM, and BC
have a common perpendicular bisector; the reflection in this bisector maps the segment CP
to BA, and maps M to Q. Therefore, in this case Q lies on AB, and BQ{AB “ CM{CP “
BD{AB; so we have Q “ D.
P
ωA
ωA
A
M
Y
A P D
Q
Q ωB
D=
D =Q
Q M
M
X
B
B C C
ωB
Figure 1 Figure 2
Now assume that the lines AP , QM, and BC are concurrent at some point X (see Figure 2).
Notice that the points A, B, Q, and X lie on a common circle Ω by Miquel’s theorem
applied to the triangle XP C. Let us denote by Y the symmetric image of X about the
perpendicular bisector of AB. Clearly, Y lies on Ω, and the triangles Y AB and △XBA are
congruent. Moreover, the triangle XP C is similar to the triangle XBA, so it is also similar to
the triangle Y AB.
Next, the points D and M correspond to each other in similar triangles Y AB and XP C,
since BD{BA “ CM{CP “ λ. Moreover, the triangles Y AB and XP C are equi-oriented, so
>pMX, XP q “ >pDY, Y Aq. On the other hand, since the points A, Q, X, and Y lie on Ω, we
have >pQY, Y Aq “ >pMX, XP q. Therefore, >pQY, Y Aq “ >pDY, Y Aq, so the points Y , D,
and Q are collinear.
Finally, we have >pDQ, QBq “ >pY Q, QBq “ >pY A, ABq “ >pAB, BXq “ >pAB, BCq,
as desired.
54 IMO 2014 South Africa
Comment. In the original proposal, λ was supposed to be an arbitrary real number distinct from 0
ÝÝÑ ÝÝ
Ñ
and 1, and the point M was defined by CM “ λ ¨ CP . The Problem Selection Committee decided to
add the restriction λ P p0, 1q in order to avoid a large case distinction.
Solution 2. As in the previous solution, we introduce the radical centre X “ AP X BC X MQ
of the circles ωA , ωB , and Γ. Next, we also notice that the points A, Q, B, and X lie on a
common circle Ω.
If the point P lies on the arc BAC of Γ, then the point X is outside Γ, thus the point Q
belongs to the ray XM, and therefore the points P , A, and Q lie on the same side of BC.
Otherwise, if P lies on the arc BC not containing A, then X lies inside Γ, so M and Q lie on
different sides of BC; thus again Q and A lie on the same side of BC. So, in each case the
points Q and A lie on the same side of BC.
P
ωA
A
M
Q
Q ωB
B
C
Figure 3
Comment. It is not hard to guess that the desired circle should be tangent to BC at B. Indeed, the
second paragraph of this solution shows that this circle lies on one side of BC; on the other hand, in
the limit case P “ B, the point Q also coincides with B.
Solution 3. Let us perform an inversion centred at C. Denote by X 1 the image of a point X
under this inversion.
The circle Γ maps to the line Γ1 passing through the constant points A1 and B 1 , and con-
taining the variable point P 1 . By the problem condition, the point M varies over the circle γ
which is the homothetic image of Γ with centre C and coefficient λ. Thus M 1 varies over the
constant line γ 1 k A1 B 1 which is the homothetic image of A1 B 1 with centre C and coefficient 1{λ,
and M “ γ 1 X CP 1 . Next, the circumcircles ωA and ωB of the triangles AMP and BMC map
to the circumcircle ωA1 of the triangle A1 M 1 P 1 and to the line B 1 M 1 , respectively; the point Q
thus maps to the second point of intersection of B 1 M 1 with ωA1 (see Figure 4).
′
ωB
ℓ
P′ A′′′′
A B′
′
′
ωA Q′′′′
Q
V
M′ J γ′
Figure 4
Let J be the (constant) common point of the lines γ 1 and CA1 , and let ℓ be the (constant)
line through J parallel to CB 1 . Let V be the common point of the lines ℓ and B 1 M 1 . Applying
Pappus’ theorem to the triples pC, J, A1 q and pV, B 1 , M 1 q we get that the points CB 1 X JV ,
JM 1 X A1 B 1 , and CM 1 X A1 V are collinear. The first two of these points are ideal, hence so is
the third, which means that CM 1 k A1 V .
Now we have >pQ1 A1 , A1 P 1 q “ >pQ1 M 1 , M 1 P 1q “ =pV M 1 , A1 V q, which means that the
triangles B 1 Q1 A1 and B 1 A1 V are similar, and pB 1 A1 q2 “ B 1 Q1 ¨ B 1 V . Thus Q1 is the image of V
under the second (fixed) inversion with centre B 1 and radius B 1 A1 . Since V varies over the
constant line ℓ, Q1 varies over some constant circle Θ. Thus, applying the first inversion back
we get that Q also varies over some fixed circle.
One should notice that this last circle is not a line; otherwise Θ would contain C, and thus
ℓ would contain the image of C under the second inversion. This is impossible, since CB 1 k ℓ.
56 IMO 2014 South Africa
G5. Let ABCD be a convex quadrilateral with =B “ =D “ 90˝ . Point H is the foot of
the perpendicular from A to BD. The points S and T are chosen on the sides AB and AD,
respectively, in such a way that H lies inside triangle SCT and
=SHC ´ =BSC “ 90˝ , =T HC ´ =DT C “ 90˝ .
Prove that the circumcircle of triangle SHT is tangent to the line BD.
(Iran)
Solution. Let the line passing through C and perpendicular to the line SC intersect the line AB
at Q (see Figure 1). Then
=SQC “ 90˝ ´ =BSC “ 180˝ ´ =SHC ,
which implies that the points C, H, S, and Q lie on a common circle. Moreover, since SQ is a
diameter of this circle, we infer that the circumcentre K of triangle SHC lies on the line AB.
Similarly, we prove that the circumcentre L of triangle CHT lies on the line AD.
A
D
K
H
H
B
C
L
Q
Figure 1
In order to prove that the circumcircle of triangle SHT is tangent to BD, it suffices to show
that the perpendicular bisectors of HS and HT intersect on the line AH. However, these two
perpendicular bisectors coincide with the angle bisectors of angles AKH and ALH. Therefore,
in order to complete the solution, it is enough (by the bisector theorem) to show that
AK AL
“ . p1q
KH LH
We present two proofs of this equality.
First proof. Let the lines KL and HC intersect at M (see Figure 2). Since KH “ KC
and LH “ LC, the points H and C are symmetric to each other with respect to the line KL.
Therefore M is the midpoint of HC. Denote by O the circumcentre of quadrilateral ABCD.
Then O is the midpoint of AC. Therefore we have OM k AH and hence OM K BD. This
together with the equality OB “ OD implies that OM is the perpendicular bisector of BD
and therefore BM “ DM.
Since CM K KL, the points B, C, M, and K lie on a common circle with diameter KC.
Similarly, the points L, C, M, and D lie on a circle with diameter LC. Thus, using the sine
law, we obtain
AK sin =ALK DM CK CK KH
“ “ ¨ “ “ ,
AL sin =AKL CL BM CL LH
Shortlisted problems – solutions 57
D
K
H
H D
B K
M B H
N
C
L C
L
Figure 2 Figure 3
Second proof. If the points A, H, and C are collinear, then AK “ AL and KH “ LH, so
the equality p1q follows. Assume therefore that the points A, H, and C do not lie in a line and
consider the circle ω passing through them (see Figure 3). Since the quadrilateral ABCD is
cyclic,
=BAC “ =BDC “ 90˝ ´ =ADH “ =HAD .
Let N ‰ A be the intersection point of the circle ω and the angle bisector of =CAH. Then
AN is also the angle bisector of =BAD. Since H and C are symmetric to each other with
respect to the line KL and HN “ NC, it follows that both N and the centre of ω lie on the
line KL. This means that the circle ω is an Apollonius circle of the points K and L. This
immediately yields p1q.
Comment. Either proof can be used to obtain the following generalised result:
Let ABCD be a convex quadrilateral and let H be a point in its interior with =BAC “ =DAH. The
points S and T are chosen on the sides AB and AD, respectively, in such a way that H lies inside
triangle SCT and
=SHC ´ =BSC “ 90˝ , =T HC ´ =DT C “ 90˝ .
Then the circumcentre of triangle SHT lies on the line AH (and moreover the circumcentre of trian-
gle SCT lies on AC).
58 IMO 2014 South Africa
G6. Let ABC be a fixed acute-angled triangle. Consider some points E and F lying on
the sides AC and AB, respectively, and let M be the midpoint of EF . Let the perpendicular
bisector of EF intersect the line BC at K, and let the perpendicular bisector of MK intersect
the lines AC and AB at S and T , respectively. We call the pair pE, F q interesting, if the
quadrilateral KSAT is cyclic.
Suppose that the pairs pE1 , F1 q and pE2 , F2 q are interesting. Prove that
E1 E2 F1 F2
“ .
AB AC
(Iran)
Solution 1. For any interesting pair pE, F q, we will say that the corresponding triangle EF K
is also interesting.
Let EF K be an interesting triangle. Firstly, we prove that =KEF “ =KF E “ =A, which
also means that the circumcircle ω1 of the triangle AEF is tangent to the lines KE and KF .
Denote by ω the circle passing through the points K, S, A, and T . Let the line AM intersect
the line ST and the circle ω (for the second time) at N and L, respectively (see Figure 1).
Since EF k T S and M is the midpoint of EF , N is the midpoint of ST . Moreover, since K
and M are symmetric to each other with respect to the line ST , we have =KNS “ =MNS “
=LNT . Thus the points K and L are symmetric to each other with respect to the perpendicular
bisector of ST . Therefore KL k ST .
Let G be the point symmetric to K with respect to N. Then G lies on the line EF , and we
may assume that it lies on the ray MF . One has
(if K “ L, then the angle KLA is understood to be the angle between AL and the tangent
to ω at L). This means that the points K, G, E, and S are concyclic. Now, since KSGT is a
parallelogram, we obtain =KEF “ =KSG “ 180˝ ´ =T KS “ =A. Since KE “ KF , we also
have =KF E “ =KEF “ =A.
After having proved this fact, one may finish the solution by different methods.
A
ω ω1
E A
E
M
F S
E1
G F2
B
B N
N E2
K C F1
Z1 Z2
T
L B K2 K1 C
Figure 1 Figure 2
First method. We have just proved that all interesting triangles are similar to each other.
This allows us to use the following lemma.
Shortlisted problems – solutions 59
Lemma. Let ABC be an arbitrary triangle. Choose two points E1 and E2 on the side AC, two
points F1 and F2 on the side AB, and two points K1 and K2 on the side BC, in a way that the
triangles E1 F1 K1 and E2 F2 K2 are similar. Then the six circumcircles of the triangles AEi Fi ,
BFi Ki , and CEi Ki (i “ 1, 2) meet at a common point Z. Moreover, Z is the centre of the
spiral similarity that takes the triangle E1 F1 K1 to the triangle E2 F2 K2 .
Proof. Firstly, notice that for each i “ 1, 2, the circumcircles of the triangles AEi Fi , BFi Ki ,
and CKi Ei have a common point Zi by Miquel’s theorem. Moreover, we have
This yields that the points Z1 and Z2 correspond to each other in similar triangles E1 F1 K1
and E2 F2 K2 . Thus, if they coincide, then this common point is indeed the desired centre of a
spiral similarity.
Finally, in order to show that Z1 “ Z2 , one may notice that >pAB, AZ1 q “ >pE1 F1 , E1 Z1 q “
>pE2 F2 , E2 Z2 q “ >pAB, AZ2 q (see Figure 2). Similarly, one has >pBC, BZ1 q “ >pBC, BZ2 q
and >pCA, CZ1 q “ >pCA, CZ2 q. This yields Z1 “ Z2 . l
Now, let P and Q be the feet of the perpendiculars from B and C onto AC and AB,
respectively, and let R be the midpoint of BC (see Figure 3). Then R is the circumcentre
of the cyclic quadrilateral BCP Q. Thus we obtain =AP Q “ =B and =RP C “ =C, which
yields =QP R “ =A. Similarly, we show that =P QR “ =A. Thus, all interesting triangles are
similar to the triangle P QR.
A A
F1
P
P
F2 E2
Z
E1 Q
Q Z
B R C
B K1 K2 R C
Figure 3 Figure 4
Denote now by Z the common point of the circumcircles of AP Q, BQR, and CP R. Let
E1 F1 K1 and E2 F2 K2 be two interesting triangles. By the lemma, Z is the centre of any
spiral similarity taking one of the triangles E1 F1 K1 , E2 F2 K2 , and P QR to some other of them.
Therefore the triangles ZE1 E2 and ZF1 F2 are similar, as well as the triangles ZE1 F1 and ZP Q.
Hence
E1 E2 ZE1 ZP
“ “ .
F1 F2 ZF1 ZQ
Moreover, the equalities =AZQ “ =AP Q “ =ABC “ 180˝ ´ =QZR show that the point Z
lies on the line AR (see Figure 4). Therefore the triangles AZP and ACR are similar, as well
as the triangles AZQ and ABR. This yields
ZP ZP RB AZ AB AB
“ ¨ “ ¨ “ ,
ZQ RC ZQ AC AZ AC
Second method. Now we will start from the fact that ω1 is tangent to the lines KE and KF
(see Figure 5). We prove that if pE, F q is an interesting pair, then
AE AF
` “ 2 cos =A. (1)
AB AC
Let Y be the intersection point of the segments BE and CF . The points B, K, and C are
collinear, hence applying Pascal’s theorem to the degenerated hexagon AF F Y EE, we infer
that Y lies on the circle ω1 .
Denote by Z the second intersection point of the circumcircle of the triangle BF Y with
the line BC (see Figure 6). By Miquel’s theorem, the points C, Z, Y , and E are concyclic.
Therefore we obtain
BF ¨ AB ` CE ¨ AC “ BY ¨ BE ` CY ¨ CF “ BZ ¨ BC ` CZ ¨ BC “ BC 2 .
On the other hand, BC 2 “ AB 2 ` AC 2 ´ 2AB ¨ AC cos =A, by the cosine law. Hence
E
ω1 A
M
S E
ω1
F Y
F
Y
B
K C
B Z C
T
Figure 5 Figure 6
Third method. Again, we make use of the fact that all interesting triangles are similar (and
equi-oriented). Let us put the picture onto a complex plane such that A is at the origin, and
identify each point with the corresponding complex number.
Let EF K be any interesting triangle. The equalities =KEF “ =KF E “ =A yield that the
ratio ν “ K´E
F ´E
is the same for all interesting triangles. This in turn means that the numbers E,
F , and K satisfy the linear equation
Now let us choose the points X and Y on the rays AB and AC, respectively, so that
=CXA “ =AY B “ =A “ =KEF (see Figure 7). Then each of the triangles AXC and Y AB
is similar to any interesting triangle, which also means that
for some real ρ, σ, and λ. In view of (3), the equation (2) now reads λB ` p1 ´ λqC “ K “
µE ` νF “ ρB ` σC, or
pλ ´ ρqB “ pσ ` λ ´ 1qC.
Since the nonzero complex numbers B and C have different arguments, the coefficients in the
brackets vanish, so ρ “ λ and σ “ 1 ´ λ. Therefore,
E F
` “ ρ ` σ “ 1. (4)
Y X
Now, if pE1 , F1 q and pE2 , F2 q are two distinct interesting pairs, one may apply (4) to both
pairs. Subtracting, we get
E1 ´ E2 F2 ´ F1 E1 ´ E2 Y B
“ , so “ “ .
Y X F2 ´ F1 X C
Taking absolute values provides the required result.
E
K
A
F B X
Figure 7
Comment 2. In order to prove that =KEF “ =KF E “ =A, one may also use the following
well-known fact:
Let AEF be a triangle with AE ‰ AF , and let K be the common point of the symmedian taken from A
and the perpendicular bisector of EF . Then the lines KE and KF are tangent to the circumcircle ω1
of the triangle AEF .
In this case, however, one needs to deal with the case AE “ AF separately.
62 IMO 2014 South Africa
AM
“ cos =A . (5)
AK
As in Solution 1, we introduce the circle ω passing through the points K, S, A, and T , together
with the points N and L at which the line AM intersect the line ST and the circle ω for the
second time, respectively. Let moreover O be the centre of ω (see Figures 8 and 9). As in
Solution 1, we note that N is the midpoint of ST and show that KL k ST , which implies
=F AM “ =EAK.
X A
ω
E
A
M S ω
F O
O
E
N M
B F S
K C N
T T
L B K =L C
Figure 8 Figure 9
Suppose now that K ‰ L (see Figure 8). Then KL k ST , and consequently the lines KM
and KL are perpendicular. It implies that the lines LO and KM meet at a point X lying on the
circle ω. Since the lines ON and XM are both perpendicular to the line ST , they are parallel
to each other, and hence =LON “ =LXK “ =MAK. On the other hand, =OLN “ =MKA,
so we infer that triangles NOL and MAK are similar. This yields
AM ON ON
“ “ “ cos =T ON “ cos =A .
AK OL OT
If, on the other hand, K “ L, then the points A, M, N, and K lie on a common line, and
this line is the perpendicular bisector of ST (see Figure 9). This implies that AK is a diameter
of ω, which yields AM “ 2OK ´ 2NK “ 2ON. So also in this case we obtain
AM 2ON
“ “ cos =T ON “ cos =A .
AK 2OT
Let P and Q be the feet of the perpendiculars from B and C onto AC and AB, respectively
(see Figure 10). We claim that the point M lies on the line P Q. Consider now the composition
of the dilatation with factor cos =A and centre A, and the reflection with respect to the angle
bisector of =BAC. This transformation is a similarity that takes B, C, and K to P , Q, and M,
respectively. Since K lies on the line BC, the point M lies on the line P Q.
Shortlisted problems – solutions 63
A
ω
E
E
P
P
M
Q S
F
B
K C
Figure 10
Suppose that E ‰ P . Then also F ‰ Q, and by Menelaus’ theorem, we obtain
AQ F M EP
¨ ¨ “ 1.
F Q EM AP
Using the similarity of the triangles AP Q and ABC, we infer that
EP AP AB EP FQ
“ “ , and hence “ .
FQ AQ AC AB AC
The last equality holds obviously also in case E “ P , because then F “ Q. Moreover, since
the line P Q intersects the segment EF , we infer that the point E lies on the segment AP if
and only if the point F lies outside of the segment AQ.
Let now pE1 , F1 q and pE2 , F2 q be two interesting pairs. Then we obtain
E1 P F1 Q E2 P F2 Q
“ and “ .
AB AC AB AC
If P lies between the points E1 and E2 , we add the equalities above, otherwise we subtract
them. In any case we obtain
E1 E2 F1 F2
“ ,
AB AC
which completes the solution.
64 IMO 2014 South Africa
G7. Let ABC be a triangle with circumcircle Ω and incentre I. Let the line passing through I
and perpendicular to CI intersect the segment BC and the arc BC (not containing A) of Ω at
points U and V , respectively. Let the line passing through U and parallel to AI intersect AV
at X, and let the line passing through V and parallel to AI intersect AB at Y . Let W and Z be
the midpoints of AX and BC, respectively. Prove that if the points I, X, and Y are collinear,
then the points I, W , and Z are also collinear.
(U.S.A.)
Solution 1. We start with some general observations. Set α “ =A{2, β “ =B{2, γ “ =C{2.
Then obviously α ` β ` γ “ 90˝ . Since =UIC “ 90˝ , we obtain =IUC “ α ` β. Therefore
=BIV “ =IUC ´ =IBC “ α “ =BAI “ =BY V , which implies that the points B, Y , I,
and V lie on a common circle (see Figure 1).
Assume now that the points I, X and Y are collinear. We prove that =Y IA “ 90˝ .
Let the line XU intersect AB at N. Since the lines AI, UX, and V Y are parallel, we get
NX YN VU XU
“ “ “ ,
AI YA VI AI
implying NX “ XU. Moreover, =BIU “ α “ =BNU. This implies that the quadrilat-
eral BUIN is cyclic, and since BI is the angle bisector of =UBN, we infer that NI “ UI.
Thus in the isosceles triangle NIU, the point X is the midpoint of the base NU. This gives
=IXN “ 90˝ , i.e., =Y IA “ 90˝ .
A
x α
N
T
α
I
X
Y α
β γ
B U x C
Figure 1
Let S be the midpoint of the segment V C. Let moreover T be the intersection point of the
lines AX and SI, and set x “ =BAV “ =BCV . Since =CIA “ 90˝ ` β and SI “ SC, we
obtain
which implies that T I “ T A. Therefore, since =XIA “ 90˝ , the point T is the midpoint
of AX, i.e., T “ W .
To complete our solution, it remains to show that the intersection point of the lines IS
and BC coincide with the midpoint of the segment BC. But since S is the midpoint of the
segment V C, it suffices to show that the lines BV and IS are parallel.
Shortlisted problems – solutions 65
Solution 2. As in Solution 1, we first prove that the points B, Y , I, V lie on a common circle
and =Y IA “ 90˝ . The remaining part of the solution is based on the following lemma, which
holds true for any triangle ABC, not necessarily with the property that I, X, Y are collinear.
Lemma. Let ABC be the triangle inscribed in a circle Γ and let I be its incentre. Assume
that the line passing through I and perpendicular to the line AI intersects the side AB at the
point Y . Let the circumcircle of the triangle BY I intersect the circle Γ for the second time
at V , and let the excircle of the triangle ABC opposite to the vertex A be tangent to the
side BC at E. Then
=BAV “ =CAE .
?
Proof. Let ρ be the composition of the inversion with centre A and radius AB ¨ AC, and the
symmetry with respect to AI. Clearly, ρ interchanges B and C.
Let J be the excentre of the triangle ABC opposite to A (see Figure 2). Then we have
=JAC “ =BAI and =JCA “ 90˝ ` γ “ =BIA, so the triangles ACJ and AIB are similar,
and therefore AB ¨ AC “ AI ¨ AJ. This means that ρ interchanges I and J. Moreover, since
Y lies on AB and =AIY “ 90˝ , the point Y 1 “ ρpY q lies on AC, and =JY 1 A “ 90˝ . Thus ρ
maps the circumcircle γ of the triangle BY I to a circle γ 1 with diameter JC.
Finally, since V lies on both Γ and γ, the point V 1 “ ρpV q lies on the line ρpΓq “ AB as
well as on γ 1 , which in turn means that V 1 “ E. This implies the desired result. l
F
A W
I K
X
X ω1
Y
I
Y γ
E D
C B C
B U Z E
Y′
V
ω2
γ′
V
J
Figure 2 Figure 3
Comment 1. The properties =Y IA “ 90˝ and V A “ V C can be established in various ways. The
main difficulty of the problem seems to find out how to use these properties in connection to the points
W and Z.
In Solution 2 this principal part is more or less covered by the lemma, for which we have presented
a direct proof. On the other hand, this lemma appears to be a combination of two well-known facts;
let us formulate them in terms of the lemma statement.
Let the line IY intersect AC at P (see Figure 4). The first fact states that the circumcircle ω of
the triangle V Y P is tangent to the segments AB and AC, as well as to the circle Γ. The second fact
states that for such a circle, the angles BAV and CAE are equal.
The awareness of this lemma may help a lot in solving this problem; so the Jury might also consider
a variation of the proposed problem, for which the lemma does not seem to be useful; see Comment 3.
N W Q
A
I ω1
X′′′′
X
Y X
P
P D
B C
U Z E
Y I
ω2
B E C
ω V V′
ω2
V
Figure 4 Figure 5
Comment 2. The proposed problem stated the equivalence: the point I lies on the line XY if and
only if I lies on the line W Z. Here we sketch the proof of the “if” part (see Figure 5).
As in Solution 2, let BC touch the circles ω1 and ω2 at D and E, respectively. Since IZ k AE and W
lies on IZ, the line DX is also parallel to AE. Therefore, the triangles XU P and AIQ are similar.
Moreover, the line DX is symmetric to AE with respect to I, so IP “ IQ, where P “ U V X XD and
Q “ U V X AE. Thus we obtain
UV UX UP UP
“ “ “ .
VI IA IQ IP
So the pairs IU and P V are harmonic conjugates, and since =U DI “ 90˝ , we get =V DB “ =BDX “
=BEA. Therefore the point V 1 symmetric to V with respect to the perpendicular bisector of BC lies
on the line AE. So we obtain =BAV “ =CAE.
Shortlisted problems – solutions 67
The rest can be obtained by simply reversing the arguments in Solution 2. The points B, V , I, and Y
are concyclic. The lemma implies that =Y IA “ 90˝ . Moreover, the points B, U , I, and N , where
N “ U X X AB, lie on a common circle, so IN “ IU . Since IY K U N , the point X 1 “ IY X U N is
the midpoint of U N . But in the trapezoid AY V I, the line XU is parallel to the sides AI and Y V , so
N X “ U X 1 . This yields X “ X 1 .
The reasoning presented in Solution 1 can also be reversed, but it requires a lot of technicalities.
Therefore the Problem Selection Committee proposes to consider only the “only if” part of the original
proposal, which is still challenging enough.
Comment 3. The Jury might also consider the following variation of the proposed problem.
Let ABC be a triangle with circumcircle Ω and incentre I. Let the line through I perpendicular to CI
intersect the segment BC and the arc BC (not containing A) of Ω at U and V , respectively. Let the
line through U parallel to AI intersect AV at X. Prove that if the lines XI and AI are perpendicular,
then the midpoint of the segment AC lies on the line XI (see Figure 6).
α
N
α
M
I
X
A Y
α
β γ
B C
U
I
X
B C
U
V
Figure 6 Figure 7
Since the solution contains the arguments used above, we only sketch it.
Let N “ XU X AB (see Figure 7). Then =BN U “ =BAI “ =BIU , so the points B, U , I, and N lie
on a common circle. Therefore IU “ IN , and since IX K N U , it follows that N X “ XU .
Now set Y “ XI X AB. The equality N X “ XU implies that
VX XU NX YX
“ “ “ ,
VA AI AI YI
and therefore Y V k AI. Hence =BY V “ =BAI “ =BIV , so the points B, V , I, Y are concyclic.
Next we have IY K Y V , so =IBV “ 90˝ . This implies that BV is the external angle bisector of the
angle ABC, which gives =V AC “ =V CA.
So in order to show that M “ XI X AC is the midpoint of AC, it suffices to prove that =V M C “ 90˝ .
But this follows immediately from the observation that the points V , C, M , and I are concyclic, as
=M IV “ =Y BV “ 180˝ ´ =ACV .
The converse statement is also true, but its proof requires some technicalities as well.
68 IMO 2014 South Africa
Number Theory
N1. Let n ě 2 be an integer, and let An be the set
An “ t2n ´ 2k | k P Z, 0 ď k ă nu.
Determine the largest positive integer that cannot be written as the sum of one or more (not
necessarily distinct) elements of An .
(Serbia)
Answer. pn ´ 2q2n ` 1.
Solution 1.
Part I. First we show that every integer greater than pn ´ 2q2n ` 1 can be represented as
such a sum. This is achieved by induction on n.
For n “ 2, the set An consists of the two elements 2 and 3. Every positive integer m except
for 1 can be represented as the sum of elements of An in this case: as m “ 2 ` 2 ` ¨ ¨ ¨ ` 2 if m
is even, and as m “ 3 ` 2 ` 2 ` ¨ ¨ ¨ ` 2 if m is odd.
Now consider some n ą 2, and take an integer m ą pn´ 2q2n ` 1. If m is even, then consider
m pn ´ 2q2n ` 2
ě “ pn ´ 2q2n´1 ` 1 ą pn ´ 3q2n´1 ` 1.
2 2
By the induction hypothesis, there is a representation of the form
m
“ p2n´1 ´ 2k1 q ` p2n´1 ´ 2k2 q ` ¨ ¨ ¨ ` p2n´1 ´ 2kr q
2
for some ki with 0 ď ki ă n ´ 1. It follows that
where 0 ď k1 , k2 , . . . , kr ă n. Suppose first that two of the terms in the sum are the same, i.e.,
ki “ kj for some i ‰ j. If ki “ kj “ n ´ 1, then we can simply remove these two terms to get a
representation for
N ´ 2p2n ´ 2n´1 q “ N ´ 2n
Shortlisted problems – solutions 69
N ´ 2p2n ´ 2k q ` 2n ´ 2k`1 “ N ´ 2n .
This is a contradiction once again. Therefore, all ki have to be distinct, which means that
Thus we must have 2k1 ` 2k2 ` ¨ ¨ ¨ ` 2kr “ 2n ´ 1, which is only possible if each element of
t0, 1, . . . , n ´ 1u occurs as one of the ki . This gives us
a2n ` b “ m1 ` m2 ` ¨ ¨ ¨ ` mr .
b`ℓ
a´ℓ` ď a ´ ℓ ` b ` ℓ ´ 1 “ a ` b ´ 1 ă n ´ 1. l
2
Solution 3. Denote by Bn the set of all positive integers that can be written as a sum of
elements of An . In this solution, we explicitly describe all the numbers in Bn by an argument
similar to the first solution.
For a positive integer n, we denote by σ2 pnq the sum of its digits in the binary representation.
Notice that every positive integer m has a unique representation of the form m “ s2n ´ t with
some positive integer s and 0 ď t ď 2n ´ 1.
Lemma. For any two integers s ě 1 and 0 ď t ď 2n ´ 1, the number m “ s2n ´ t belongs to Bn
if and only if s ě σ2 ptq.
Proof. For t “ 0, the statement of the Lemma is obvious, since m “ 2s ¨ p2n ´ 2n´1 q.
Now suppose that t ě 1, and let
Assume now that there exist integers s and t with 1 ď s ă σ2 ptq and 0 ď t ď 2n ´ 1 such
that the number m “ s2n ´ t belongs to Bn . Among all such instances, choose the one for
which m is smallest, and let
d
ÿ
m“ p2n ´ 2ℓi q p0 ď ℓi ď n ´ 1q
i“1
be the corresponding representation. If all the ℓi ’s are distinct, then di“1 2ℓi ď j“0
řn´1 j
2 “ 2n ´1,
ř
řd ℓi
so one has s “ d and t “ i“1 2 , whence s “ d “ σ2 ptq; this is impossible. Therefore, two of
the ℓi ’s must be equal, say ℓd´1 “ ℓd . Then m ě 2p2n ´ 2ℓd q ě 2n , so s ě 2.
Now we claim that the number m1 “ m ´ 2n “ ps ´ 1q2n ´ t also belongs to Bn , which
contradicts the minimality assumption. Indeed, one has
so
d´2
ÿ
1
m “ p2n ´ 2ℓi q ` p2n ´ 2ℓd `1 q
i“1
is the desired representation of m1 (if ℓd “ n ´ 1, then the last summand is simply omitted).
This contradiction finishes the proof. l
By our lemma, the largest number M which does not belong to Bn must have the form
Comment. Alternatively one could ask to find all pairs px, yq of – not necessarily positive – integers
solving (1). The answer to that question is a bit nicer than the answer above: the set of solutions are
now described by
tx, yu “ tm3 ` m2 ´ 2m ´ 1, m3 ` 2m2 ´ m ´ 1u ,
where m varies through Z. This may be shown using essentially the same arguments as above. We
finally observe that the pair px, yq “ p1, 1q, that appears to be sporadic above, corresponds to m “ ´1.
72 IMO 2014 South Africa
N3. A coin is called a Cape Town coin if its value is 1{n for some positive integer n. Given
a collection of Cape Town coins of total value at most 99 ` 21 , prove that it is possible to split
this collection into at most 100 groups each of total value at most 1.
(Luxembourg)
Solution. We will show that for every positive integer N any collection of Cape Town coins
of total value at most N ´ 21 can be split into N groups each of total value at most 1. The
problem statement is a particular case for N “ 100.
We start with some preparations. If several given coins together have a total value also of
the form k1 for a positive integer k, then we may merge them into one new coin. Clearly, if the
resulting collection can be split in the required way then the initial collection can also be split.
After each such merging, the total number of coins decreases, thus at some moment we
come to a situation when no more merging is possible. At this moment, for every even k there
is at most one coin of value k1 (otherwise two such coins may be merged), and for every odd
k ą 1 there are at most k ´ 1 coins of value k1 (otherwise k such coins may also be merged).
Now, clearly, each coin of value 1 should form a single group; if there are d such coins then
we may remove them from the collection and replace N by N ´ d. So from now on we may
assume that there are no coins of value 1.
Finally, we may split all the coins in the following way. For each k “ 1, 2, . . . , N we put all
1 1
the coins of values 2k´1 and 2k into a group Gk ; the total value of Gk does not exceed
1 1
p2k ´ 2q ¨ ` ă 1.
2k ´ 1 2k
1
It remains to distribute the “small” coins of values which are less than 2N ; we will add them one
by one. In each step, take any remaining small coin. The total value of coins` in the˘ groups at
this moment is at most N ´ 21 , so there exists a group of total value at most N1 N ´ 12 “ 1 ´ 2N1
;
thus it is possible to put our small coin into this group. Acting so, we will finally distribute all
the coins.
Comment 1. The algorithm may be modified, at least the step where one distributes the coins of
1 1
values ě 2N . One different way is to put into Gk all the coins of values p2k´1q2s for all integer s ě 0.
One may easily see that their total value also does not exceed 1.
Comment 2. The original proposal also contained another part, suggesting to show that a required
splitting may be impossible if the total value of coins is at most 100. There are many examples of
such a collection, e.g. one may take 98 coins of value 1, one coin of value 21 , two coins of value 13 , and
four coins of value 51 .
The Problem Selection Committee thinks that this part is less suitable for the competition.
Shortlisted problems – solutions 73
N4. Let n ą 1 be a given integer. Prove that infinitely many terms of the sequence pak qkě1 ,
defined by
nk
Z ^
ak “ ,
k
are odd. (For a real number x, txu denotes the largest integer not exceeding x.)
(Hong Kong)
nm ´m
Solution 1. If n is odd, let k “ nm for m “ 1, 2, . . .. Then ak “ n , which is odd for
each m.
Henceforth, assume that n is even, say n “ 2t for some integer t ě 1. Then, for any m ě 2,
m m m
the integer n2 ´ 2m “ 2m p22 ´m ¨ t2 ´ 1q has an odd prime divisor p, since 2m ´ m ą 1. Then,
for k “ p ¨ 2m , we have
m
nk “ pn2 qp ” p2m qp “ p2p qm ” 2m ,
where the congruences are taken modulo p (recall that 2p ” 2 pmod pq, by Fermat’s little
nk
theorem). Also, from nk ´ 2m ă nk ă nk ` 2m pp ´ 1q, we see that the fraction lies strictly
k
nk ´ 2m nk ` 2m pp ´ 1q
between the consecutive integers and , which gives
p ¨ 2m p ¨ 2m
Z k^
n nk ´ 2m
“ .
k p ¨ 2m
n k
nk ´ 2m 2m
´1 nk
We finally observe that “ is an odd integer, since the integer ´ 1 is odd
p ¨ 2m p 2m
(recall that k ą m). Note that for different values of m, we get different values of k, due to the
different powers of 2 in the prime factorisation of k.
Comment. The case n even and n ą 2 can also be solved by recursively defining the sequence pki qiě1
by k1 “ 1 and ki`1 “ nki ´ 1 for i ě 1. Then pki q is strictly increasing and it follows (by induction
on i) that ki | nki ´ 1 for all i ě 1, so the ki are as desired.
The case n “ 2 can also be solved as follows: Let i ě 2. By Bertrand’s postulate, there exists a
i i
prime number p such that 22 ´1 ă p ¨ 2i ă 22 . This gives
i
p ¨ 2i ă 22 ă 2p ¨ 2i . (1)
74 IMO 2014 South Africa
i i
Also, we have that p ¨ 2i is a divisor of 2p¨2 ´ 22 , hence, using (1), we get that
i ^ i i i i
2p¨2 2p¨2 ´ 22 ` p ¨ 2i 2p¨2 ´i ´ 22 ´i ` p
Z
“ “ ,
p ¨ 2i p ¨ 2i p
Recall that there exists a with 1 ď a ă 2i such that ap ” ´1 pmod 2i q, so each ai satisfies
1 ď ai ă 2i . This implies that ai p ` 1 ă p ¨ 2i . Also, ai Ñ 8 as i Ñ 8, whence there are
infinitely many i such that ai ă ai`1 . From now on, we restrict ourselves only to these i.
i
Notice that p is a divisor of np ` 1, which, in turn, divides np¨2 ´Z1. It follows that p ¨ 2i is a
i i
np¨2 np¨2 ´ pai p ` 1q
^
p¨2i
divisor of n ´ pai p ` 1q, and we consequently see that the integer “
p ¨ 2i p ¨ 2i
i
is odd, since 2i`1 divides np¨2 , but not ai p ` 1.
Shortlisted problems – solutions 75
N5. Find all triples pp, x, yq consisting of a prime number p and two positive integers x and y
such that xp´1 ` y and x ` y p´1 are both powers of p.
(Belgium)
( (
Answer. pp, x, yq P p3, 2, 5q, p3, 5, 2q Y p2, n, 2k ´ nq | 0 ă n ă 2k .
Solution 1. For p “ 2, clearly all pairs of two positive integers x and y whose sum is a power
of 2 satisfy the condition. Thus we assume in the following that p ą 2, and we let a and b be
positive integers such that xp´1 ` y “ pa and x ` y p´1 “ pb . Assume further, without loss of
generality, that x ď y, so that pa “ xp´1 ` y ď x ` y p´1 “ pb , which means that a ď b (and
thus pa | pb ).
Now we have
pb “ y p´1 ` x “ ppa ´ xp´1 qp´1 ` x.
We take this equation modulo pa and take into account that p ´ 1 is even, which gives us
2
0 ” xpp´1q ` x pmod pa q.
2
If p | x, then pa | x, since xpp´1q ´1 ` 1 is not divisible by p in this case. However, this is
impossible, since x ď xp´1 ă pa . Thus we know that p ∤ x, which means that
2 ´1
pa | xpp´1q ` 1 “ xppp´2q ` 1.
2
By Fermat’s little theorem, xpp´1q ” 1 pmod pq, thus p divides x`1. Let pr be the highest
power of p that divides x ` 1. By the binomial theorem, we have
ppp´2q
ÿ ˆppp ´ 2q˙
ppp´2q
x “ p´1qppp´2q´k px ` 1qk .
k“0
k
Except for the terms corresponding to k “ 0, k “ 1 and k “ 2, all terms in the sum are clearly
divisible by p3r and thus by pr`2 . The remaining terms are
ppp ´ 2qpp2 ´ 2p ´ 1q
´ px ` 1q2 ,
2
which is divisible by p2r`1 and thus also by pr`2 ,
ppp ´ 2qpx ` 1q,
which is divisible by pr`1 , but not pr`2 by our choice of r, and the final term ´1 corresponding
to k “ 0. It follows that the highest power of p that divides xppp´2q ` 1 is pr`1 .
On the other hand, we already know that pa divides xppp´2q ` 1, which means that a ď r ` 1.
Moreover,
pr ď x ` 1 ď xp´1 ` y “ pa .
Hence we either have a “ r or a “ r ` 1.
If a “ r, then x “ y “ 1 needs to hold in the inequality above, which is impossible for
p ą 2. Thus a “ r ` 1. Now since pr ď x ` 1, we get
x2 ` x xp´1 ` y pa pa
x“ ď “ ď r “ p,
x`1 x`1 x`1 p
so we must have x “ p ´ 1 for p to divide x ` 1.
It follows that r “ 1 and a “ 2. If p ě 5, we obtain
pa “ xp´1 ` y ą pp ´ 1q4 “ pp2 ´ 2p ` 1q2 ą p3pq2 ą p2 “ pa ,
a contradiction. So the only case that remains is p “ 3, and indeed x “ 2 and y “ pa ´ xp´1 “ 5
satisfy the conditions.
76 IMO 2014 South Africa
Comment 1. In this solution, we are implicitly using a special case of the following lemma known
as “lifting the exponent”:
Lemma. Let n be a positive integer, let p be an odd prime, and let vp pmq denote the exponent of the
highest power of p that divides m.
If x and y are integers not divisible by p such that p | x ´ y, then we have
vp pxn ´ y n q “ vp px ´ yq ` vp pnq.
Likewise, if x and y are integers not divisible by p such that p | x ` y, then we have
vp pxn ` y n q “ vp px ` yq ` vp pnq.
Comment 2. There exist various ways of solving the problem involving the “lifting the exponent”
lemma. Let us sketch another one.
The cases x “ y and p | x are ruled out easily, so we assume that p ą 2, x ă y, and p ∤ x. In this
case we also have pa ă pb and p | x ` 1.
Now one has
y p ´ xp ” ypy p´1 ` xq ´ xpxp´1 ` yq ” 0 pmod pa q,
so by the lemma mentioned above one has pa´1 | y ´ x and hence y “ x ` tpa´1 for some positive
integer t. Thus one gets
The factors on the left-hand side are coprime. So if p | x, then xp´2 ` 1 | p ´ t, which is impossible
since x ă xp´2 ` 1. Therefore, p ∤ x, and thus x | p ´ t. Since p | x ` 1, the only remaining case is
x “ p ´ 1, t “ 1, and y “ pa´1 ` p ´ 1. Now the solution can be completed in the same way as before.
Solution 2. Again, we can focus on the case that p ą 2. If p | x, then also p | y. In this case,
let pk and pℓ be the highest powers of p that divide x and y respectively, and assume without
loss of generality that k ď ℓ. Then pk divides x ` y p´1 while pk`1 does not, but pk ă x ` y p´1,
which yields a contradiction. So x and y are not divisible by p. Fermat’s little theorem yields
0 ” xp´1 ` y ” 1 ` y pmod pq, so y ” ´1 pmod pq and for the same reason x ” ´1 pmod pq.
In particular, x, y ě p ´ 1 and thus xp´1 ` y ě 2pp ´ 1q ą p, so xp´1 ` y and y p´1 ` x are
both at least equal to p2 . Now we have
which simplifies to y p´2 ` y p´3x ` ¨ ¨ ¨ ` xp´2 ´ 1 ” ´p pmod p2 q. Thus the second factor in (1)
is divisible by p, but not p2 .
This means that pa´1 has to divide the other factor y ´ x. It follows that
Since x ” ´1 pmod pq, the last factor is xp´3 ´ xp´4 ` ¨ ¨ ¨ ` 1 ” p ´ 2 pmod pq and in particular
not divisible by p. We infer that pa´1 | x ` 1 and continue as in the first solution.
once again.
78 IMO 2014 South Africa
|X|2 modulo a1 .
ř
We are interested in computing
XPS
Note that the number A is marked, so in the definition of T the condition y ď A is enforced
without explicitly prescribing it.
Assign weights to the intervals` in ˘T , depending only on their lengths. The weight of an
arbitrary interval Y P T will be w |Y | , where
#
1 if k “ 1,
wpkq “
2 if k ě 2 .
ÿ |X|
` ˘ ÿ
p|X| ´ d ` 1q ¨ wpdq “ |X| ¨ 1 ` p|X| ´ 1q ` p|X| ´ 2q ` ¨ ¨ ¨ ` 1 ¨ 2 “ |X|2 .
` ˘
w |Y | “
Y PT , Y ĎX d“1
For every d “ 1, 2, . . . , a1 , we count how many intervals in T are of length d. Notice that
the multiples of a1 are all marked, so the lengths of the intervals in S and T cannot exceed a1 .
Let x be an arbitrary integer with 0 ď x ď A ´ 1 and consider the interval rx, x ` ds. Let r1 ,
. . . , rn be the remainders of x modulo a1 , . . . , an , respectively. Since a1 , . . . , an are pairwise
coprime, the number x is uniquely identified by the sequence pr1 , . . . , rn q, due to the Chinese
remainder theorem.
For every i “ 1, . . . , n, the property that the interval px, x`dq does not contain any multiple
of ai is equivalent with ri ` d ď ai , i.e. ri P t0, 1, . . . , ai ´ du, so there are ai ´ d ` 1 choices for
the number ri for each i. Therefore, the number of the remainder sequences pr1 , . . . , rn q that
satisfy rx, x ` ds P T is precisely pa1 ` 1 ´ dq ¨ ¨ ¨ pan ` 1 ´ dq. Denote this product by f pdq.
Shortlisted problems – solutions 79
Now we can group the last sum in (1) by length of the intervals. As we have seen, for every
d “ 1, . . . , a1 there are f pdq intervals Y P T with |Y | “ d. Therefore, (1) can be continued as
ÿ ÿ a1 a1
2
` ˘ ÿ ÿ
|X| “ w |Y | “ f pdq ¨ wpdq “ 2 f pdq ´ f p1q. (2)
XPS Y PT d“1 d“1
Having the formula (2), the solution can be finished using the following well-known fact:
Lemma. If p is a prime, F pxq is a polynomial with integer coefficients, and deg F ď p ´ 2, then
řp
F pxq is divisible by p.
x“1
Proof. Obviously, it is sufficient to prove the lemma for monomials of the form xk with k ď p´2.
Apply induction on k. If k “ 0 then F “ 1, and the statement is trivial.
Let 1 ď k ď p ´ 2, and assume that the lemma is proved for all lower degrees. Then
p p
˜ ˙ ¸
k ˆ
ÿ ÿ ÿ k ` 1
0 ” pk`1 “ xk`1 ´ px ´ 1qk`1 “ p´1qk´ℓ xℓ
` ˘
x“1 x“1 ℓ“0
ℓ
p k´1 ˆ ˙ÿ p p
k´ℓ k ` 1
ÿ ÿ ÿ
k ℓ
“ pk ` 1q x ` p´1q x ” pk ` 1q xk pmod pq.
x“1 ℓ“0
ℓ x“1 x“1
p
xk ” 0 pmod pq.
ř
Since 0 ă k ` 1 ă p, this proves l
x“1
a1
ř
In (2), by applying the lemma to the polynomial f and the prime a1 , we obtain that f pdq
d“1
Comment 1. With suitable sets of weights, the same method can be used to sum up other expressions
ř of3the segments. For example, wp1q “ 1 and wpkq “ 6pk ´ 1q for k ě 2 can be used to
on the lengths
compute |X| and to prove that this sum is divisible by a1 if a1 is a prime with a1 ě n ` 3. See
XPS
also Comment 2 after the second solution.
Solution 2. The conventions from the first paragraph of the first solution are still in force.
We shall prove the following more general statement:
Applying p‘q to d “ 1 and d “ 2 and using the equation x2 “ 2 x2 ` x1 , one easily gets
`˘ `˘
śn´1
the numbers p “ a1 ă a2 ă ¨ ¨ ¨ ă an and d are as above. Write A1 “ i“1 ai and A “ A1 an .
Mark the points on the real axis divisible by one of the numbers a1 , . . . , an´1 green and those
divisible by an red. The green points divide r0, A1 s into certain sub-intervals, say J1 , J2 , . . . ,
and Jℓ .
To translate intervals we use the notation ra, bs ` m “ ra ` m, b ` ms whenever a, b, m P Z.
For each i P t1, 2, . . . , ℓu let Fi be the family of intervals into which the red points partition
the intervals Ji , Ji ` A1 , . . . , and Ji ` pan ´ 1qA1 . We are to prove that
ℓ ÿ ˆ ˙
ÿ |X|
i“1 XPF
d
i
is divisible by p.
Let us fix any index i with 1 ď i ď ℓ for a while. Since the numbers A1 and an are coprime
by hypothesis, the numbers 0, A1 , . . . , pan ´ 1qA1 form a complete system of residues modulo an .
Moreover, we have |Ji | ď p ă an , as in particular all multiples of p are green. So each of the
intervals Ji , Ji ` A1 , . . . , and Ji ` pan ´ 1qA1 contains at most one red point. More precisely,
for each j P t1, . . . , |Ji | ´ 1u there is exactly one amongst those intervals containing a red point
splitting it into an interval of length j followed by an interval of length |Ji | ´ j, while the
remaining an ´ |Ji | ` 1 such intervals have no red points in their interiors. For these reasons
ÿ ˆ|X|˙ ˆˆ ˙
1
ˆ
|Ji | ´ 1
˙˙ ˆ ˙
|Ji |
“2 ` ¨¨¨` ` pan ´ |Ji | ` 1q
XPFi
d d d d
ˆ ˙ ˆ ˙ ˆ ˙
|Ji | |Ji | |Ji |
“2 ` pan ´ d ` 1q ´ pd ` 1q
d`1 d d`1
ˆ ˙ ˆ ˙
|Ji | |Ji |
“ p1 ´ dq ` pan ´ d ` 1q .
d`1 d
is divisible by p. By the induction hypothesis, however, it is even true that both summands
are divisible by p, for 1 ď d ă d ` 1 ď p ´ pn ´ 1q. This completes the proof of p‘q and hence
the solution of the problem.
Comment 2. `The ˘statement p‘q can also be proved by the method of the first solution, using the
weights wpxq “ x´2
d´2 .
Shortlisted problems – solutions 81
for all n ě 1. Prove that for each integer n ě 2 there exists a prime number p dividing an but
none of the numbers a1 , . . . , an´1 .
(Austria)
Solution. Let us define x0 “ 0 and xn “ an {c for all integers n ě 1. It is easy to see that the
sequence pxn q thus obtained obeys the recursive law
for all integers n ě 0. In particular, all of its terms are positive integers; notice that x1 “ 1
and x2 “ 2c2 ` 1. Since
holds for all integers n ě 0, it is also strictly increasing. Since xn`1 is by (1) coprime to c for
any n ě 0, it suffices to prove that for each n ě 2 there exists a prime number p dividing xn
but none of the numbers x1 , . . . , xn´1 . Let us begin by establishing three preliminary claims.
Claim 1. If i ” j pmod mq holds for some integers i, j ě 0 and m ě 1, then xi ” xj pmod xm q
holds as well.
Proof. Evidently, it suffices to show xi`m ” xi pmod xm q for all integers i ě 0 and m ě 1. For
this purpose we may argue for fixed m by induction on i using x0 “ 0 in the base case i “ 0.
Now, if we have xi`m ” xi pmod xm q for some integer i, then the recursive equation (1) yields
Claim 2. If the integers i, j ě 2 and m ě 1 satisfy i ” j pmod mq, then xi ” xj pmod x2m q
holds as well.
Proof. Again it suffices to prove xi`m ” xi pmod x2m q for all integers i ě 2 and m ě 1. As
above, we proceed for fixed m by induction on i. The induction step is again easy using (1),
but this time the base case i “ 2 requires some calculation. Set L “ 5c2 . By (1) we have
xm`1 ” Lxm ` 1 pmod x2m q, and hence
Now we direct our attention to the problem itself: let any integer n ě 2 be given. By Claim 3
there exists a prime number p appearing with a higher exponent in the prime factorisation of xn
than in the prime factorisation of x1 ¨ ¨ ¨ xn´2 . In particular, p | xn , and it suffices to prove that
p divides none of x1 , . . . , xn´1 .
Otherwise let k P t1, . . . , n ´ 1u be minimal such that p divides xk . Since xn´1 and xn are
coprime by (1) and x1 “ 1, we actually have 2 ď k ď n ´ 2. Write n “ qk ` r with some
integers q ě 0 and 0 ď r ă k. By Claim 1 we have xn ” xr pmod xk q, whence p | xr . Due to
the minimality of k this entails r “ 0, i.e. k | n.
Thus from Claim 2 we infer
xn ” xk pmod x2k q .
Now let α ě 1 be maximal with the property pα | xk . Then x2k is divisible by pα`1 and by our
choice of p so is xn . So by the previous congruence xk is a multiple of pα`1 as well, contrary to
our choice of α. This is the final contradiction concluding the solution.
84 IMO 2014 South Africa
N8. For every real number x, let }x} denote the distance between x and the nearest integer.
Prove that for every pair pa, bq of positive integers there exist an odd prime p and a positive
integer k satisfying › › › › › ›
› a › › b › ›a ` b›
› ›`› ›`›
› pk › › pk › › pk › “ 1. (1)
›
(Hungary)
1
ˇX ˇ
is an integer nearest to x, so }x} “ ˇ x ` 12 ´ xˇ. Thus we
X \ \
Solution. Notice first that x ` 2
have Z ^
1
x` “ x ˘ }x}. (2)
2
For every rational number r and every prime number p, denote by vp prq the exponent of p
in the prime factorisation of r. Recall the notation p2n ´ 1q!! for the product of all odd positive
integers not exceeding 2n ´ 1, i.e., p2n ´ 1q!! “ 1 ¨ 3 ¨ ¨ ¨ p2n ´ 1q.
Lemma. For every positive integer n and every odd prime p, we have
8 Z ^
` ˘ ÿ n 1
vp p2n ´ 1q!! “ ` .
k“1
pk 2
Proof. For every positive integer k, let us count the multiples of pk among the factors 1, 3, . . . ,
2n ´ 1. If ℓ is an arbitrary integer, the number p2ℓ ´ 1qpk is listed above if and only if
Z ^
k 1 n 1 n 1
0 ă p2ℓ ´ 1qp ď 2n ðñ ăℓď k ` ðñ 1 ď ℓ ď k ` .
2 p 2 p 2
Hence, the number of multiples of pk among the factors is precisely mk “ pnk ` 21 . Thus we
X \
obtain
n n vp p2i´1q 8 ÿmk 8 Z ^
` ˘ ÿ ÿ ÿ ÿ ÿ n 1
vp p2n ´ 1q!! “ vp p2i ´ 1q “ 1“ 1“ ` . l
i“1 i“1 k“1 k“1 ℓ“1 k“1
pk 2
Obviously, N ą 1, so there exists a prime p with vp pNq ą 0. Since N is a fraction of two odd
numbers, p is odd.
By our lemma,
8 ˆZ ^ Z ^ Z ^˙
ÿ a`b 1 a 1 b 1
0 ă vp pNq “ ` ´ k` ´ k` .
k“1
pk 2 p 2 p 2
Therefore, there exists some positive integer k such that the integer number
Z ^ Z ^ Z ^
a`b 1 a 1 b 1
dk “ ` ´ k` ´ k`
pk 2 p 2 p 2
Since }x} ă 21 for every rational x with odd denominator, the relation (3) can only be satisfied
if all three signs on the right-hand side are positive and dk “ 1. Thus we get
› › › › › ›
› a › › b › ›a ` b›
› ›`› ›`›
› pk › › pk › › pk › “ dk “ 1,
›
as required.
Comment 1. There are various choices for the number N in the solution. Here we sketch such a
version.
Let x and y be two rational numbers with odd denominators. It is easy to see that the condi-
tion }x} ` }y} ` }x ` y} “ 1 is satisfied if and only if
either txu ă 21 , tyu ă 12 , tx ` yu ą 12 , or txu ą 21 , tyu ą 12 , tx ` yu ă 12 ,
where txu denotes the fractional part of x.
In the context of our problem, the first condition seems easier to deal with. Also, one may notice
that
txu ă 12 ðñ κpxq “ 0 and txu ě 12 ðñ κpxq “ 1, (4)
where
κpxq “ t2xu ´ 2txu.
Now it is natural to consider the number
ˆ ˙
2a ` 2b
a`b
M “ ˆ ˙ˆ ˙ ,
2a 2b
a b
since ˜ ˆ ˆ ˙¸
8 ˙ ˆ ˙
ÿ 2pa ` bq 2a 2b
vp pM q “ κ k
´κ k
´κ .
k“1
p p pk
One may see that M ą 1, and that v2 pM q ď 0. Thus, there exist an odd prime p and a positive
integer k with ˆ ˙ ˆ ˙ ˆ ˙
2pa ` bq 2a 2b
κ k
´κ k
´κ ą 0.
p p pk
In view of (4), the last inequality yields
" * " * " *
a 1 b 1 a`b 1
k
ă , k
ă , and ą , (5)
p 2 p 2 pk 2
which is what we wanted to obtain.
Comment 2. Once one tries to prove the existence of suitable p and k satisfying (5), it seems somehow
natural to suppose that a ď b and to add the restriction pk ą a. In this case the inequalities (5) can
be rewritten as
2a ă pk , 2mpk ă 2b ă p2m ` 1qpk , and p2m ` 1qpk ă 2pa ` bq ă p2m ` 2qpk
for some positive integer m. This means exactly that one of the numbers 2a ` 1, 2a ` 3, . . . , 2a ` 2b ´ 1
is divisible by some number of the form pk which is greater than 2a.
Using more advanced techniques, one can show that such a number pk exists even with k “ 1.
This was shown in 2004 by Laishram and Shorey; the methods used for this proof are elementary
but still quite involved. In fact, their result generalises a theorem by Sylvester which states that
for every pair of integers pn, kq with n ě k ě 1, the product pn ` 1qpn ` 2q ¨ ¨ ¨ pn ` kq is divisible by
some prime p ą k. We would like to mention here that Sylvester’s theorem itself does not seem to
suffice for solving the problem.
Shortlisted Problems with Solutions
Contributing Countries
The Organizing Committee and the Problem Selection Committee of IMO 2015 thank the
following 53 countries for contributing 155 problem proposals:
Albania, Algeria, Armenia, Australia, Austria, Brazil, Bulgaria,
Canada, Costa Rica, Croatia, Cyprus, Denmark, El Salvador,
Estonia, Finland, France, Georgia, Germany, Greece, Hong Kong,
Hungary, India, Iran, Ireland, Israel, Italy, Japan, Kazakhstan,
Lithuania, Luxembourg, Montenegro, Morocco, Netherlands,
Pakistan, Poland, Romania, Russia, Saudi Arabia, Serbia,
Singapore, Slovakia, Slovenia, South Africa, South Korea, Sweden,
Turkey, Turkmenistan, Taiwan, Tanzania, Ukraine, United Kingdom,
U.S.A., Uzbekistan
Problems
Algebra
A1. Suppose that a sequence a1 , a2 , . . . of positive real numbers satisfies
kak
ak`1 ě
a2k ` pk ´ 1q
Combinatorics
C1. In Lineland there are n ě 1 towns, arranged along a road running from left to right.
Each town has a left bulldozer (put to the left of the town and facing left) and a right bulldozer
(put to the right of the town and facing right). The sizes of the 2n bulldozers are distinct.
Every time when a right and a left bulldozer confront each other, the larger bulldozer pushes
the smaller one off the road. On the other hand, the bulldozers are quite unprotected at their
rears; so, if a bulldozer reaches the rear-end of another one, the first one pushes the second one
off the road, regardless of their sizes.
Let A and B be two towns, with B being to the right of A. We say that town A can sweep
town B away if the right bulldozer of A can move over to B pushing off all bulldozers it meets.
Similarly, B can sweep A away if the left bulldozer of B can move to A pushing off all bulldozers
of all towns on its way.
Prove that there is exactly one town which cannot be swept away by any other one.
(Estonia)
C2. Let V be a finite set of points in the plane. We say that V is balanced if for any two
distinct points A, B P V, there exists a point C P V such that AC “ BC. We say that V is
center-free if for any distinct points A, B, C P V, there does not exist a point P P V such that
P A “ P B “ P C.
(a) Show that for all n ě 3, there exists a balanced set consisting of n points.
(b) For which n ě 3 does there exist a balanced, center-free set consisting of n points?
(Netherlands)
C3. For a finite set A of positive integers, we call a partition of A into two disjoint nonempty
subsets A1 and A2 good if the least common multiple of the elements in A1 is equal to the
greatest common divisor of the elements in A2 . Determine the minimum value of n such that
there exists a set of n positive integers with exactly 2015 good partitions.
(Ukraine)
C4. Let n be a positive integer. Two players A and B play a game in which they take turns
choosing positive integers k ď n. The rules of the game are:
piq A player cannot choose a number that has been chosen by either player on any previous
turn.
piiq A player cannot choose a number consecutive to any of those the player has already chosen
on any previous turn.
piiiq The game is a draw if all numbers have been chosen; otherwise the player who cannot
choose a number anymore loses the game.
The player A takes the first turn. Determine the outcome of the game, assuming that both
players use optimal strategies.
(Finland)
Shortlisted problems 5
C5. Consider an infinite sequence a1 , a2 , . . . of positive integers with ai ď 2015 for all i ě 1.
Suppose that for any two distinct indices i and j we have i ` ai ‰ j ` aj .
Prove that there exist two positive integers b and N such that
ˇ ˇ
ˇ ÿn ˇ
pa ´ bq ˇ ď 10072
ˇ ˇ
ˇ i
ˇi“m`1 ˇ
whenever n ą m ě N.
(Australia)
C6. Let S be a nonempty set of positive integers. We say that a positive integer n is clean if
it has a unique representation as a sum of an odd number of distinct elements from S. Prove
that there exist infinitely many positive integers that are not clean.
(U.S.A.)
C7. In a company of people some pairs are enemies. A group of people is called unsociable
if the number of members in the group is odd and at least 3, and it is possible to arrange all
its members around a round table so that every two neighbors are enemies. Given that there
are at most 2015 unsociable groups, prove that it is possible to partition the company into 11
parts so that no two enemies are in the same part.
(Russia)
6 IMO 2015 Thailand
Geometry
G1. Let ABC be an acute triangle with orthocenter H. Let G be the point such that the
quadrilateral ABGH is a parallelogram. Let I be the point on the line GH such that AC
bisects HI. Suppose that the line AC intersects the circumcircle of the triangle GCI at C
and J. Prove that IJ “ AH.
(Australia)
G2. Let ABC be a triangle inscribed into a circle Ω with center O. A circle Γ with center A
meets the side BC at points D and E such that D lies between B and E. Moreover, let F and
G be the common points of Γ and Ω. We assume that F lies on the arc AB of Ω not containing
C, and G lies on the arc AC of Ω not containing B. The circumcircles of the triangles BDF
and CEG meet the sides AB and AC again at K and L, respectively. Suppose that the lines
F K and GL are distinct and intersect at X. Prove that the points A, X, and O are collinear.
(Greece)
G3. Let ABC be a triangle with =C “ 900, and let H be the foot of the altitude from C.
A point D is chosen inside the triangle CBH so that CH bisects AD. Let P be the intersection
point of the lines BD and CH. Let ω be the semicircle with diameter BD that meets the
segment CB at an interior point. A line through P is tangent to ω at Q. Prove that the
lines CQ and AD meet on ω.
(Georgia)
G4. Let ABC be an acute triangle, and let M be the midpoint of AC. A circle ω passing
through B and M meets the sides AB and BC again at P and Q, respectively. Let T be
the point such that the quadrilateral BP T Q is a parallelogram. Suppose that T lies on the
circumcircle of the triangle ABC. Determine all possible values of BT {BM.
(Russia)
G5. Let ABC be a triangle with CA ‰ CB. Let D, F , and G be the midpoints of the
sides AB, AC, and BC, respectively. A circle Γ passing through C and tangent to AB at D
meets the segments AF and BG at H and I, respectively. The points H 1 and I 1 are symmetric
to H and I about F and G, respectively. The line H 1 I 1 meets CD and F G at Q and M,
respectively. The line CM meets Γ again at P . Prove that CQ “ QP .
(El Salvador)
G6. Let ABC be an acute triangle with AB ą AC, and let Γ be its circumcircle. Let H,
M, and F be the orthocenter of the triangle, the midpoint of BC, and the foot of the altitude
from A, respectively. Let Q and K be the two points on Γ that satisfy =AQH “ 900 and
=QKH “ 900 . Prove that the circumcircles of the triangles KQH and KF M are tangent to
each other.
(Ukraine)
G7. Let ABCD be a convex quadrilateral, and let P , Q, R, and S be points on the sides
AB, BC, CD, and DA, respectively. Let the line segments P R and QS meet at O. Suppose
that each of the quadrilaterals AP OS, BQOP , CROQ, and DSOR has an incircle. Prove that
the lines AC, P Q, and RS are either concurrent or parallel to each other.
(Bulgaria)
G8. A triangulation of a convex polygon Π is a partitioning of Π into triangles by diagonals
having no common points other than the vertices of the polygon. We say that a triangulation
is a Thaiangulation if all triangles in it have the same area.
Prove that any two different Thaiangulations of a convex polygon Π differ by exactly two
triangles. (In other words, prove that it is possible to replace one pair of triangles in the first
Thaiangulation with a different pair of triangles so as to obtain the second Thaiangulation.)
(Bulgaria)
Shortlisted problems 7
Number Theory
N1. Determine all positive integers M for which the sequence a0 , a1 , a2 , . . ., defined by
2M `1
a0 “ 2
and ak`1 “ ak tak u for k “ 0, 1, 2, . . ., contains at least one integer term.
(Luxembourg)
N2. Let a and b be positive integers such that a!b! is a multiple of a! ` b!. Prove that
3a ě 2b ` 2.
(United Kingdom)
N3. Let m and n be positive integers such that m ą n. Define xk “ pm ` kq{pn ` kq for k “
1, 2, . . . , n ` 1. Prove that if all the numbers x1 , x2 , . . . , xn`1 are integers, then x1 x2 ¨ ¨ ¨ xn`1 ´ 1
is divisible by an odd prime.
(Austria)
N4. Suppose that a0 , a1 , . . . and b0 , b1 , . . . are two sequences of positive integers satisfying
a0 , b0 ě 2 and
an`1 “ gcdpan , bn q ` 1, bn`1 “ lcmpan , bn q ´ 1
for all n ě 0. Prove that the sequence (an ) is eventually periodic; in other words, there exist
integers N ě 0 and t ą 0 such that an`t “ an for all n ě N.
(France)
N5. Determine all triples pa, b, cq of positive integers for which ab ´ c, bc ´ a, and ca ´ b are
powers of 2.
Explanation: A power of 2 is an integer of the form 2n , where n denotes some nonnegative
integer.
(Serbia)
N6. Let Zą0 denote the set of positive integers. Consider a function f : Zą0 Ñ Zą0 . For
any m, n P Zą0 we write f n pmq “ looomooon
f pf p. . . f pmq . . .qq. Suppose that f has the following two
n
properties:
f n pmq ´ m
piq If m, n P Zą0 , then P Zą0 ;
n
piiq The set Zą0 z tf pnq | n P Zą0 u is finite.
That is, ℧pnq is the number of prime factors of n greater than 10100 , counted with multiplicity.
Find all strictly increasing functions f : Z Ñ Z such that
` ˘
℧ f paq ´ f pbq ď ℧pa ´ bq for all integers a and b with a ą b.
(Brazil)
8 IMO 2015 Thailand
Solutions
Algebra
A1. Suppose that a sequence a1 , a2 , . . . of positive real numbers satisfies
kak
ak`1 ě (1)
a2k ` pk ´ 1q
for every positive integer k. Prove that a1 ` a2 ` ¨ ¨ ¨ ` an ě n for every n ě 2.
(Serbia)
Solution. From the constraint (1), it can be seen that
k a2k ` pk ´ 1q k´1
ď “ ak ` ,
ak`1 ak ak
and so
k´1
k
ak ě . ´
ak`1 ak
Summing up the above inequality for k “ 1, . . . , m, we obtain
ˆ ˙ ˆ ˙ ˆ ˙
1 0 2 1 m m´1 m
a1 ` a2 ` ¨ ¨ ¨ ` am ě ´ ` ´ ` ¨¨¨` ´ “ . (2)
a2 a1 a3 a2 am`1 am am`1
Now we prove the problem statement by induction on n. The case n “ 2 can be done by
applying (1) to k “ 1:
1
a1 ` a2 ě a1 ` ě 2.
a1
For the induction step, assume that the statement is true for some n ě 2. If an`1 ě 1, then
the induction hypothesis yields
` ˘
a1 ` ¨ ¨ ¨ ` an ` an`1 ě n ` 1. (3)
Comment 1. It can be seen easily that having equality in the statement requires a1 “ a2 “ 1 in the
base case n “ 2, and an`1 “ 1 in (3). So the equality a1 ` ¨ ¨ ¨ ` an “ n is possible only in the trivial
case a1 “ ¨ ¨ ¨ “ an “ 1.
Comment 2. After obtaining (2), there are many ways to complete the solution. We outline three
such possibilities.
• By applying the AM–GM inequality to the numbers a1 ` ¨ ¨ ¨ ` ak and kak`1 , we can conclude
a1 ` ¨ ¨ ¨ ` ak ` kak`1 ě 2k
Answer. There are two such functions, namely the constant function x ÞÑ ´1 and the successor
function x ÞÑ x ` 1.
Solution 1. It is immediately checked that both functions mentioned in the answer are as
desired.
Now let f denote any function satisfying (1) for ` all˘ x, y P Z. Substituting x “ 0 and
y “ f p0q into (1) we learn that the number z “ ´f f p0q satisfies f pzq “ ´1. So by plugging
y “ z into (1) we deduce that
` ˘
f px ` 1q “ f f pxq (2)
holds for all x P Z. Thereby (1) simplifies to
` ˘
f x ´ f pyq “ f px ` 1q ´ f pyq ´ 1 . (3)
We now work towards showing that f is linear by contemplating the difference f px`1q´f pxq
for any x P Z. By applying (3) with y “ x and (2) in this order, we obtain
` ˘ ` ˘
f px ` 1q ´ f pxq “ f x ´ f pxq ` 1 “ f f px ´ 1 ´ f pxqq ` 1 .
` ˘
Since (3) shows f x ´ 1 ´ f pxq “ f pxq ´ f pxq ´ 1 “ ´1, this simplifies to
f px ` 1q “ f pxq ` A ,
Comment. After (2) and (3) have been obtained, there are several other ways to combine them so as
to obtain linearity properties of f . For instance, using (2) thrice in a row and then (3) with x “ f pyq
one may deduce that
` ˘ ` ` ˘˘ ` ˘
f py ` 2q “ f f py ` 1q “ f f f pyq “ f f pyq ` 1 “ f pyq ` f p0q ` 1
holds for all y P Z. It follows that f behaves linearly on the even numbers and on the odd numbers
separately, and moreover that the slopes of these two linear functions coincide. From this point, one
may complete the solution with some straightforward case analysis.
A different approach using the equations (2) and (3) will be presented in Solution 2. To show
that it is also possible to start in a completely different way, we will also present a third solution that
avoids these equations entirely.
Shortlisted problems – solutions 11
Solution 2. We commence by deriving (2) and (3) as in the first solution. Now provided that f
is injective, (2) tells us that f is the successor function. Thus we may assume from now on that
f is not injective, i.e., that there are two integers a ą b with f paq “ f pbq. A straightforward
induction using (2) in the induction step reveals that we have f pa ` nq “ f pb ` nq for all
nonnegative integers n. Consequently, the sequence γn “ f pb ` nq is periodic and thus in
particular bounded, which means that the numbers
exist. ` ˘
Let us pick any integer y with f pyq “ ϕ and then an integer x ě a with f x ´ f pyq “ ϕ.
Due to the definition of ϕ and (3) we have
` ˘
ϕ ď f px ` 1q “ f x ´ f pyq ` f pyq ` 1 “ 2ϕ ` 1 ,
whence ϕ ě ´1. The same reasoning applied to ψ yields ψ ď ´1. Since ϕ ď ψ holds trivially,
it follows that ϕ “ ψ “ ´1, or in other words that we have f ptq “ ´1 for all integers t ě a.
Finally, if any integer y is given, we may find an integer x which is so large that x ` 1 ě a
and x ´ f pyq ě a hold. Due to (3) and the result from the previous paragraph we get
` ˘
f pyq “ f px ` 1q ´ f x ´ f pyq ´ 1 “ p´1q ´ p´1q ´ 1 “ ´1 .
f pz ` kq “ f pzq ` k (8)
holds for all integers z. In plain English, this means that on any residue class modulo k the
function f is linear with slope 1.
Now by (7) the set of all values
` attained
˘ by f is such a residue class. Hence, there exists an
absolute constant c such that f f pxq “ f pxq ` c holds for all x P Z. Thereby (1) simplifies to
` ˘
f x ´ f pyq “ f pxq ´ f pyq ` c ´ 1 . (9)
On the other hand, considering (1) modulo k we obtain d ” ´1 pmod kq because of (7). So
by (7) again, f attains the value ´1.
Thus we may apply (9) to some integer y with f pyq “ ´1, which gives f px ` 1q “ f pxq ` c.
So f is a linear function with slope c. Hence, (8) leads to c “ 1, wherefore there is an absolute
constant d1 with f pxq “ x`d1 for all x P Z. Using this for x “ 0 we obtain d1 “ d and finally (4)
discloses d “ 1, meaning that f is indeed the successor function.
Shortlisted problems – solutions 13
A3. Let n be a fixed positive integer. Find the maximum possible value of
ÿ
ps ´ r ´ nqxr xs ,
1ďrăsď2n
where the last equality follows from the fact that xr P t´1, 1u. Notice that for every r ă s, the
coefficient of xr xs in (1) is 2 for each i “ 1, . . . , r ´1, s, . . . , 2n,
ř and2this coefficient is ´2 for each
i “ r, . . . , s ´ 1. This implies that the coefficient of xr xs in 2n i“1 yi is 2p2n ´ s ` rq ´ 2ps ´ rq “
4pn ´ s ` rq. Therefore, summing (1) for i “ 1, 2, . . . , 2n yields
2n
ÿ ÿ
yi2 “ 4n2 ` 4pn ´ s ` rqxr xs “ 4n2 ´ 4Z. (2)
i“1 1ďrăsď2n
Comment 2. Several variations in setting up the auxiliary variables are possible. For instance, one
may let x2n`i “ ´xi and yi1 “ xi ` xi`1 ` ¨ ¨ ¨ ` xi`n´1 for any 1 ď i ď 2n. Similarly to Solution 1,
we obtain Y :“ y112 ` y212 ` ¨ ¨ ¨ ` y2n12 “ 2n2 ´ 2Z. Then, it suffices to show that Y ě 2n. If n is odd,
then each yi1 is odd, and so yi12 ě 1. If n is even, then each yi1 is even. We can check that at least one
of yi1 , yi`1
1 , y1 1 12 12 12 12
n`i , and yn`i`1 is nonzero, so that yi ` yi`1 ` yn`i ` yn`i`1 ě 4; summing these up for
i “ 1, 3, . . . , n ´ 1 yields Y ě 2n.
Therefore, we have
` ˘ pn ´ 1qnp2n ´ 1q
Z “ epA, Aq ´ epA, Bq ´ epB, Aq ` epB, Bq “ 2 epA, Aq ` epB, Bq ` . (5)
3
Thus, we need to maximize epA, Aq ` epB, Bq, where A and B form a partition of r2ns.
Due to the symmetry, we may assume that |A| “ n ´ p and |B| “ n ` p, where 0 ď p ď n.
From now on, we fix the value of p and find an upper bound for Z in terms of n and p.
Let a1 ă a2 ă ¨ ¨ ¨ ă an´p and b1 ă b2 ă ¨ ¨ ¨ ă bn`p list all elements of A and B, respectively.
Then
n´p ˆ ˙
ÿ ÿ n´p
epA, Aq “ paj ´ ai ´ nq “ p2i ´ 1 ´ n ` pqai ´ ¨n (6)
1ďiăjďn´p i“1
2
and similarly
n`p ˙ ˆ
ÿ n`p
epB, Bq “ p2i ´ 1 ´ n ´ pqbi ´ ¨ n. (7)
i“1
2
Thus, now it suffices to maximize the value of
n´p
ÿ n`p
ÿ
M“ p2i ´ 1 ´ n ` pqai ` p2i ´ 1 ´ n ´ pqbi . (8)
i“1 i“1
In order to get an upper bound, we will apply the rearrangement inequality to the se-
quence a1 , a2 , . . . , an´p , b1 , b2 , . . . , bn`p (which is a permutation of 1, 2, . . . , 2n), together with
the sequence of coefficients of these numbers in (8). The coefficients of ai form the sequence
n ´ p ´ 1, n ´ p ´ 3, . . . , 1 ´ n ` p ,
n ` p ´ 1, n ` p ´ 3, . . . , 1 ´ n ´ p .
Shortlisted problems – solutions 15
Finally, combining the information from (5), (6), (7), and (9), we obtain
ˆˆ ˙ ˆ ˙˙
pn ´ 1qnp2n ´ 1q n´p n`p
Zď ´ 2n `
3 2 2
p
ÿ n´p
ÿ
` 2 pn ` p ` 1 ´ 2iqp2n ` 1 ´ 2iq ` 2 pn ´ p ` 1 ´ 2iqp4n ´ 2p ` 3 ´ 4iq ,
i“1 i“1
A5. Let 2Z ` 1 denote the set of odd integers. Find all functions f : Z Ñ 2Z ` 1 satisfying
` ˘ ` ˘
f x ` f pxq ` y ` f x ´ f pxq ´ y “ f px ` yq ` f px ´ yq (1)
for every x, y P Z.
(U.S.A.)
Answer. Fix an odd positive integer d, an integer k, and odd integers ℓ0 , ℓ1 , . . . , ℓd´1 . Then
the function defined as
f pmd ` iq “ 2kmd ` ℓi d pm P Z, i “ 0, 1, . . . , d ´ 1q
satisfies the problem requirements, and these are all such functions.
Solution. Throughout the solution, all functions are assumed to map integers to integers.
For any function g and any nonzero integer t, define
∆M f pbq “ ∆M f p2x ´ b ´ Mq for any nonzero integer M such that f pxq | M. (3)
Lemma 2. Let g be a function. If t and s are nonzero integers such that ∆ts g “ 0 and
∆t ∆t g “ 0, then ∆t g “ 0.
Proof. Assume, without loss of generality, that s is positive. Let a be an arbitrary integer.
Since ∆t ∆t g “ 0, we have
` ˘
∆t gpaq “ ∆t gpa ` tq “ ¨ ¨ ¨ “ ∆t g a ` ps ´ 1qt .
The sum of these s equal numbers is ∆ts gpaq “ 0, so each of them is zero, as required. l
18 IMO 2015 Thailand
∆T {p ∆T {p ∆1 f “ 0.
This shows that all functions satisfying (1) are listed in the answer.
It remains to check that all such functions indeed satisfy (1). This is equivalent to check-
ing (2), which is true because for every integer x, the value of f pxq is divisible by d, so that
∆f pxq f is constant.
Comment. After obtaining Lemmas 1 and 2, it is possible to complete the steps in a different order.
Here we sketch an alternative approach.
For any function g and any nonzero integer t, we say that g is t-pseudo-periodic if ∆t ∆t g “ 0. In
this case, we call t a pseudo-period of g, and we say that g is pseudo-periodic.
Let us first prove a basic property: if a function g is pseudo-periodic, then its minimal positive
pseudo-period divides all its pseudo-periods. To establish this, it suffices to show that if t and s
are pseudo-periods of g with t ‰ s, then so is t ´ s. Indeed, suppose that ∆t ∆t g “ ∆s ∆s g “ 0.
Then ∆t ∆t ∆s g “ ∆ts ∆s g “ 0, so that ∆t ∆s g “ 0 by Lemma 2. Taking differences, we obtain
∆t ∆t´s g “ ∆s ∆t´s g “ 0, and thus ∆t´s ∆t´s g “ 0.
Now let f satisfy the problem condition. We will show that f is pseudo-periodic. When this is
done, we will let T 1 be the minimal pseudo-period of f , and show that T 1 divides 2f pxq for every
integer x, using arguments similar to Step 2 of the solution. Then we will come back to Step 1 by
showing that T 1 is also a quasi-period of f .
Shortlisted problems – solutions 19
First, Lemma 1 yields that ∆2py´xq ∆lcmpf pxq,f pyqq`f “ 0 for every distinct
˘ integers x and y. Hence
f is pseudo-periodic with pseudo-period Lx,y “ lcm 2py ´ xq, f pxq, f pyq .
We now show that T 1 | 2f pxq for every integer x. Suppose, to the contrary, that there exists an
integer u, a prime p, and a positive integer α such that pα | T 1 and pα ∤ 2f puq. Choose v as in Step 2 and
employ Lemma 1 to obtain ∆2f puq ∆lcmpf puq,f pvqq f “ 0. However, this implies that ∆T 1 {p ∆T 1 {p f “ 0, a
contradiction with the minimality of T 1 .
We now claim that ∆T 1 ∆2 f “ 0. Indeed, Lemma 1 implies that there exists an integer s such that
∆s ∆2 f “ 0. Hence ∆T 1 s ∆2 f “ ∆T 1 ∆T 1 ∆2 f “ 0, which allows us to conclude that ∆T 1 ∆2 f “ 0 by
Lemma 2. (The last two paragraphs are similar to Step 2 of the solution.)
Now, it is not difficult to finish the solution, though more work is needed to eliminate the factors
of 2 from the subscripts of ∆T 1 ∆2 f “ 0. Once this is done, we will obtain an odd quasi-period of f
that divides f pxq for all integers x. Then we can complete the solution as in Step 3.
20 IMO 2015 Thailand
A6. Let n be a fixed integer with n ě 2. We say that two polynomials P and Q with real
coefficients are block-similar if for each i P t1, 2, . . . , nu the sequences
Since Qpxq “ P px ´ 1q and P p0q “ P pkq “ P p2kq “ ¨ ¨ ¨ “ P pnkq, these polynomials are
block-similar (and distinct).
řm (b). For every polynomial F pxq and every nonnegative integer m, define ΣF pmq “
Part
F piq; in particular, ΣF p0q “ 0. It is well-known that for every nonnegative integer d the
i“1 ř
sum m d
i“1 i is a polynomial in m of degree d ` 1. Thus ΣF may also be regarded as a real
polynomial of degree deg F ` 1 (with the exception that if F “ 0, then ΣF “ 0 as well). This
allows us to consider the values of ΣF at all real points (where the initial definition does not
apply).
Assume for the sake of contradiction that there exist two distinct block-similar polynomials
P pxq and Qpxq of degree n. Then both polynomials ΣP ´Q pxq and ΣP 2 ´Q2 pxq have roots at the
points 0, k, 2k, . . . , nk. This motivates the following lemma, where we use the special polynomial
n
ź
T pxq “ px ´ ikq.
i“0
Lemma. Assume that F pxq is a nonzero polynomial such that 0, k, 2k, . . . , nk are among the
roots of the polynomial ΣF pxq. Then deg F ě n, and there exists a polynomial Gpxq such that
deg G “ deg F ´ n and F pxq “ T pxqGpxq ´ T px ´ 1qGpx ´ 1q.
Proof. If deg F ă n, then ΣF pxq has at least n ` 1 roots, while its degree is less than n ` 1.
Therefore, ΣF pxq “ 0 and hence F pxq “ 0, which is impossible. Thus deg F ě n.
The lemma condition yields that ΣF pxq “ T pxqGpxq for some polynomial Gpxq such that
deg G “ deg ΣF ´ pn ` 1q “ deg F ´ n.
Now, let us define F1 pxq “ T pxqGpxq ´ T px ´ 1qGpx ´ 1q. Then for every positive integer n
we have
n
ÿ ` ˘
ΣF1 pnq “ T pxqGpxq ´ T px ´ 1qGpx ´ 1q “ T pnqGpnq ´ T p0qGp0q “ T pnqGpnq “ ΣF pnq,
i“1
so the polynomial ΣF ´F1 pxq “ ΣF pxq ´ ΣF1 pxq has infinitely many roots. This means that this
polynomial is zero, which in turn yields F pxq “ F1 pxq, as required. l
Shortlisted problems – solutions 21
First, we apply the lemma to the nonzero polynomial R1 pxq “ P pxq´Qpxq.` Since the degree˘
of R1 pxq is at most n, we conclude that it is exactly n. Moreover, R1 pxq “ α ¨ T pxq ´ T px ´ 1q
for some nonzero constant α.
Our next aim is to prove that the polynomial Spxq “ P pxq ` Qpxq is constant. Assume the
contrary. Then, notice that the polynomial R2 pxq “ P pxq2 ´Qpxq2 “ R1 pxqSpxq is also nonzero
and satisfies the lemma condition. Since n ă deg R1 ` deg S “ deg R2 ď 2n, the lemma yields
since both T pxq and T px´1q are the products of linear polynomials, and their roots are distinct.
Thus R1 pxq | Gpxq ´ Gpx ´ 1q. However, this is impossible since Gpxq ´ Gpx ´ 1q is a nonzero
polynomial of degree less than n “ deg R1 .
Thus, our assumption` is wrong, ˘ and Spxq` is a constant polynomial, say Spxq “ β. Notice
that the polynomials 2P pxq ´ β {α and 2Qpxq ´ βq{α are also block-similar and distinct.
So we may replace the initial polynomials by these ones, thus obtaining two block-similar
polynomials P pxq and Qpxq with P pxq “ ´Qpxq “ T pxq ´ T px ´ 1q. It remains to show that
this is impossible.
For every i “ 1, 2 . . . , n, the values T pik ´ k ` 1q and T pik ´ 1q have the same sign. This
means that the values P pik ´ k ` 1q “ T pik ´ k ` 1q and P pikq “ ´T pik ´ 1q have opposite
signs, so P pxq has a root in each of the n segments rik ´ k ` 1, iks. Since deg P “ n, it must
have exactly one root in each of them.
Thus, the sequence P p1q, P p2q, . . . , P pkq should change sign exactly once. On the other
hand, since P pxq and ´P pxq are block-similar, this sequence must have as many positive terms
as negative ones. Since k “ 2ℓ ` 1 is odd, this shows that the middle term of the sequence
above must be zero, so P pℓ ` 1q “ 0, or T pℓ ` 1q “ T pℓq. However, this is not true since
n
ź n
ź
|T pℓ ` 1q| “ |ℓ ` 1| ¨ |ℓ| ¨ |ℓ ` 1 ´ ik| ă |ℓ| ¨ |ℓ ` 1| ¨ |ℓ ´ ik| “ |T pℓq| ,
i“2 i“2
where the strict inequality holds because n ě 2. We come to the final contradiction.
Comment 1. In the solution above, we used the fact that k ą 1 is odd. One can modify the
arguments of the last part in order to work for every (not necessarily odd) sufficiently large value of k;
namely, when k is even, one may show that the sequence P p1q, P p2q, . . . , P pkq has different numbers
of positive and negative terms.
On the other hand, the problem statement with k replaced by 2 is false, since the polynomials
P pxq “ T pxq ´ T px ´ 1q and Qpxq “ T px ´ 1q ´ T pxq are block-similar in this case, due to the fact that
P p2i ´ 1q “ ´P p2iq “ Qp2iq “ ´Qp2i ´ 1q “ T p2i ´ 1q for all i “ 1, 2, . . . , n. Thus, every complete
solution should use the relation k ą 2.
One may easily see that the condition n ě 2 is also substantial, since the polynomials x and
k ` 1 ´ x become block-similar if we set n “ 1.
It is easily seen from the solution that the result still holds if we assume that the polynomials have
degree at most n.
22 IMO 2015 Thailand
Since the sequences of values of P and Q in Zi are permutations of each other, we have
Rpp´ q “ P pp´ q ´ Qpp´ q ď 0 and Rpp` q “ P pp` q ´ Qpp` q ě 0. Since Rpxq is continuous, there
exists at least one root of Rpxq between p´ and p` — thus in Ii .
So, Rpxq has at least one root in each of the n disjoint segments Ii with i “ 1, 2, . . . , n.
Since Rpxq is nonzero and its degree does not exceed n, it should have exactly one root in each
of these segments, and all these roots are simple, as required.
Step 2. We prove that Spxq is constant.
We start with the following claim.
` ˘ ` ˘
Claim. For every i “ 1, 2, . . . , n, the sequence of values S pi ´ 1qk ` 1 , S pi ´ 1qk ` 2 , . . . ,
Spikq cannot be strictly increasing.
`
Proof. Fix any i P t1, 2, . . . , nu. Due to the symmetry, we may assume that P ikq ď Qpikq.
Choose now p´ and p` as in Step 1. If we had P pp` q “ P pp´ q, then P would be constant
on Zi , so all the elements of Zi would be the roots of Rpxq, which is not the case. In particular,
we have p` ‰ p´ . If p´ ą p` , then Spp´ q “ P pp´ q ` Qpp´ q ď Qpp` q ` P pp` q “ Spp` q, so our
claim holds.
We now show that the remaining case p´ ă p` is impossible. Assume first that P pp` q ą
Qpp` q. Then, like in Step 1, we have Rpp´ q ď 0, Rpp` q ą 0, and Rpikq ď 0, so Rpxq has a root
in each of the intervals rp´ , p` q and pp` , iks. This contradicts the result of Step 1.
We are left only with the case p´ ă p` and P pp` q “ Qpp` q (thus p` is the unique root of
Rpxq in Ii ). If p` “ ik, then the values of Rpxq on Zi z tiku are all of the same sign, which
is absurd since their sum is zero. Finally, if p´ ă p` ă ik, then Rpp´ q and Rpikq are both
negative. This means that Rpxq should have an even number of roots in rp´ , iks, counted with
multiplicity. This also contradicts the result of Step 1. l
` ˘
` In a similar ˘ way, one may prove that for every i “ 1, 2, . . . , n, the sequence S pi ´ 1qk ` 1 ,
S pi ´ 1qk ` 2 , . . . , Spikq cannot be strictly decreasing. This means that the polynomial
∆Spxq “ Spxq ´ Spx ´ 1q attains at least one nonnegative (value, as well as at least one non-
positive value, on the set Zi (and even on Zi z pi ´ 1qk ` 1 ); so ∆S has a root in Ii .
Thus ∆S has at least n roots; however, its degree is less than n, so ∆S should be identically
zero. This shows that Spxq is a constant, say Spxq ” β.
Step 3. Notice that the polynomials P pxq ´ β{2 and Qpxq ´ β{2 are also block-similar and
distinct. So we may replace the initial polynomials by these ones, thus reaching P pxq “ ´Qpxq.
Then Rpxq “ 2P pxq, so P pxq has exactly one root in each of the segments Ii , i “ 1, 2, . . . , n.
On the other hand, P pxq and ´P pxq should attain the same number of positive values on Zi .
Since k is odd, this means that Zi contains exactly one root of P pxq; moreover, this root should
be at the center of Zi , because P pxq has the same number of positive and negative values on Zi .
Thus we have found all n roots of P pxq, so
n
ź
P pxq “ c px ´ ik ` ℓq for some c P R z t0u,
i“1
Shortlisted problems – solutions 23
so P p1q ‰ ´P ptq for all t P Z1 . This shows that P pxq is not block-similar to ´P pxq. The final
contradiction.
Comment 2. One may merge Steps 1 and 2 in the following manner. As above, we set Rpxq “
P pxq ´ Qpxq and Spxq “ P pxq ` Qpxq.
We aim to prove that the polynomial Spxq “ 2P pxq ´ Rpxq “ 2Qpxq ` Rpxq is constant. Since the
degrees of Rpxq and Spxq do not exceed n, it suffices to show that the total number of roots of Rpxq
and ∆Spxq “ Spxq ´ Spx ´ 1q is at least 2n. For this purpose, we prove the following claim.
Claim. For every i “ 1, 2, . . . , n, either each of R and ∆S has a root in Ii , or R has at least two roots
in Ii .
Proof. Fix any i P t1, 2, . . . , nu. Let r P Zi be a point such that |Rprq| “ maxxPZi |Rpxq|; we may
assume that Rprq ą 0. Next, let p´ , q ` P Ii be some points such that P pp´ q “ minxPZi P pxq and
Qpq ` q “ maxxPZi Qpxq. Notice that P pp´ q ď Qprq ă P prq and Qpq ` q ě P prq ą Qprq, so r is different
from p´ and q ` .
Without loss of generality, we may assume that p´ ă r. Then we have Rpp´ q “ P pp´ q ´ Qpp´ q ď
0 ă Rprq, so Rpxq has a root in rp´ , rq. If q ` ą r, then, similarly, Rpq ` q ď 0 ă Rprq, and Rpxq also
has a root in pr, q ` s; so Rpxq has two roots in Ii , as required.
In the remaining case we have q ` ă r; it suffices now to show that in this case ∆S has a root in Ii .
Since P pp´ q ď Qprq and |Rpp´ q| ď Rprq, we have Spp´ q “ 2P pp´ q ´ Rpp´ q ď 2Qprq ` Rprq “ Sprq.
Similarly, we get Spq ` q “ 2Qpq ` q ` Rpq ` q ě 2P prq ´ Rprq “ Sprq. Therefore, the sequence of values
of S on Zi is neither strictly increasing nor strictly decreasing, which shows that ∆S has a root
in Ii . l
` ˘
Comment 3. After finding the relation P pxq ´ Qpxq “ α T pxq ´ T px ´ 1q from Solution 1, one
may also follow the approach presented in Solution 2. Knowledge of the difference of polynomials
may simplify some steps; e.g., it is clear now that P pxq ´ Qpxq has exactly one root in each of the
segments Ii .
24 IMO 2015 Thailand
Combinatorics
C1. In Lineland there are n ě 1 towns, arranged along a road running from left to right.
Each town has a left bulldozer (put to the left of the town and facing left) and a right bulldozer
(put to the right of the town and facing right). The sizes of the 2n bulldozers are distinct.
Every time when a right and a left bulldozer confront each other, the larger bulldozer pushes
the smaller one off the road. On the other hand, the bulldozers are quite unprotected at their
rears; so, if a bulldozer reaches the rear-end of another one, the first one pushes the second one
off the road, regardless of their sizes.
Let A and B be two towns, with B being to the right of A. We say that town A can sweep
town B away if the right bulldozer of A can move over to B pushing off all bulldozers it meets.
Similarly, B can sweep A away if the left bulldozer of B can move to A pushing off all bulldozers
of all towns on its way.
Prove that there is exactly one town which cannot be swept away by any other one.
(Estonia)
Solution 1. Let T1 , T2 , . . . , Tn be the towns enumerated from left to right. Observe first that,
if town Ti can sweep away town Tj , then Ti also can sweep away every town located between Ti
and Tj .
We prove the problem statement by strong induction on n. The base case n “ 1 is trivial.
For the induction step, we first observe that the left bulldozer in T1 and the right bulldozer
in Tn are completely useless, so we may forget them forever. Among the other 2n´2 bulldozers,
we choose the largest one. Without loss of generality, it is the right bulldozer of some town Tk
with k ă n.
Surely, with this large bulldozer Tk can sweep away all the towns to the right of it. Moreover,
none of these towns can sweep Tk away; so they also cannot sweep away any town to the left
of Tk . Thus, if we remove the towns Tk`1 , Tk`2, . . . , Tn , none of the remaining towns would
change its status of being (un)sweepable away by the others.
Applying the induction hypothesis to the remaining towns, we find a unique town among
T1 , T2 , . . . , Tk which cannot be swept away. By the above reasons, it is also the unique such
town in the initial situation. Thus the induction step is established.
Solution 2. We start with the same enumeration and the same observation as in Solution 1.
We also denote by ℓi and ri the sizes of the left and the right bulldozers belonging to Ti ,
respectively. One may easily see that no two towns Ti and Tj with i ă j can sweep each other
away, for this would yield ri ą ℓj ą ri .
Clearly, there is no town which can sweep Tn away from the right. Then we may choose the
leftmost town Tk which cannot be swept away from the right. One can observe now that no
town Ti with i ą k may sweep away some town Tj with j ă k, for otherwise Ti would be able
to sweep Tk away as well.
Now we prove two claims, showing together that Tk is the unique town which cannot be
swept away, and thus establishing the problem statement.
Claim 1. Tk also cannot be swept away from the left.
Proof. Let Tm be some town to the left of Tk . By the choice of Tk , town Tm can be swept
away from the right by some town Tp with p ą m. As we have already observed, p cannot be
greater than k. On the other hand, Tm cannot sweep Tp away, so a fortiori it cannot sweep Tk
away. l
Claim 2. Any town Tm with m ‰ k can be swept away by some other town.
Shortlisted problems – solutions 25
Proof. If m ă k, then Tm can be swept away from the right due to the choice of Tk . In the
remaining case we have m ą k.
Let Tp be a town among Tk , Tk`1, . . . , Tm´1 having the largest right bulldozer. We claim
that Tp can sweep Tm away. If this is not the case, then rp ă ℓq for some q with p ă q ď m. But
this means that ℓq is greater than all the numbers ri with k ď i ď m ´ 1, so Tq can sweep Tk
away. This contradicts the choice of Tk . l
Comment 1. One may employ the same ideas within the inductive approach. Here we sketch such
a solution.
Assume that the problem statement holds for the collection of towns T1 , T2 , . . . , Tn´1 , so that there
is a unique town Ti among them which cannot be swept away by any other of them. Thus we need
to prove that in the full collection T1 , T2 , . . . , Tn , exactly one of the towns Ti and Tn cannot be swept
away.
If Tn cannot sweep Ti away, then it remains to prove that Tn can be swept away by some other
town. This can be established as in the second paragraph of the proof of Claim 2.
If Tn can sweep Ti away, then it remains to show that Tn cannot be swept away by any other town.
Since Tn can sweep Ti away, it also can sweep all the towns Ti , Ti`1 , . . . , Tn´1 away, so Tn cannot be
swept away by any of those. On the other hand, none of the remaining towns T1 , T2 , . . . , Ti´1 can
sweep Ti away, so that they cannot sweep Tn away as well.
Comment 2. Here we sketch yet another inductive approach. Assume that n ą 1. Firstly, we find a
town which can be swept away by each of its neighbors (each town has two neighbors, except for the
bordering ones each of which has one); we call such town a loser. Such a town exists, because there
are n ´ 1 pairs of neighboring towns, and in each of them there is only one which can sweep the other
away; so there exists a town which is a winner in none of these pairs.
Notice that a loser can be swept away, but it cannot sweep any other town away (due to its
neighbors’ protection). Now we remove a loser, and suggest its left bulldozer to its right neighbor (if
it exists), and its right bulldozer to a left one (if it exists). Surely, a town accepts a suggestion if a
suggested bulldozer is larger than the town’s one of the same orientation.
Notice that suggested bulldozers are useless in attack (by the definition of a loser), but may serve
for defensive purposes. Moreover, each suggested bulldozer’s protection works for the same pairs of
remaining towns as before the removal.
By the induction hypothesis, the new configuration contains exactly one town which cannot be
swept away. The arguments above show that the initial one also satisfies this property.
Solution 3. We separately prove that piq there exists a town which cannot be swept away,
and that piiq there is at most one such town. We also make use of the two observations from
the previous solutions.
To prove piq, assume contrariwise that every town can be swept away. Let t1 be the leftmost
town; next, for every k “ 1, 2, . . . we inductively choose tk`1 to be some town which can sweep
tk away. Now we claim that for every k “ 1, 2, . . . , the town tk`1 is to the right of tk ; this leads
to the contradiction, since the number of towns is finite.
Induction on k. The base case k “ 1 is clear due to the choice of t1 . Assume now that for
all j with 1 ď j ă k, the town tj`1 is to the right of tj . Suppose that tk`1 is situated to the left
of tk ; then it lies between tj and tj`1 (possibly coinciding with tj ) for some j ă k. Therefore,
tk`1 can be swept away by tj`1 , which shows that it cannot sweep tj`1 away — so tk`1 also
cannot sweep tk away. This contradiction proves the induction step.
To prove piiq, we also argue indirectly and choose two towns A and B neither of which can
be swept away, with A being to the left of B. Consider the largest bulldozer b between them
(taking into consideration the right bulldozer of A and the left bulldozer of B). Without loss
of generality, b is a left bulldozer; then it is situated in some town to the right of A, and this
town may sweep A away since nothing prevents it from doing that. A contradiction.
26 IMO 2015 Thailand
Comment 3. The Problem Selection Committee decided to reformulate this problem. The original
formulation was as follows.
Let n be a positive integer. There are n cards in a deck, enumerated from bottom to top with
numbers 1, 2, . . . , n. For each i “ 1, 2, . . . , n, an even number ai is printed on the lower side and an
odd number bi is printed on the upper side of the ith card. We say that the ith card opens the j th card,
if i ă j and bi ă ak for every k “ i ` 1, i ` 2, . . . , j. Similarly, we say that the ith card closes the
j th card, if i ą j and ai ă bk for every k “ i ´ 1, i ´ 2, . . . , j. Prove that the deck contains exactly one
card which is neither opened nor closed by any other card.
Shortlisted problems – solutions 27
C2. Let V be a finite set of points in the plane. We say that V is balanced if for any two
distinct points A, B P V, there exists a point C P V such that AC “ BC. We say that V is
center-free if for any distinct points A, B, C P V, there does not exist a point P P V such that
P A “ P B “ P C.
(a) Show that for all n ě 3, there exists a balanced set consisting of n points.
(b) For which n ě 3 does there exist a balanced, center-free set consisting of n points?
(Netherlands)
A3 B 2 B1
O A2
A1
B3
O
O
A1 A9
E
A2 A8 C
A3 A7
A4 A5 A6
D
Figure 1 Figure 2
Comment (a ). There are many ways to construct an example by placing equilateral triangles in a
circle. Here we present one general method.
Let O be the center of a circle and let A1 , B1 , . . . , Ak , Bk be distinct points on the circle such
that the triangle OAi Bi is equilateral for each i. Then V “ tO, A1 , B1 , . . . , Ak , Bk u is balanced. To
construct a set of even cardinality, put extra points C, D, E on the circle such that triangles OCD
and ODE are equilateral (see Figure 2). Then V “ tO, A1 , B1 , . . . , Ak , Bk , C, D, Eu is balanced.
Part (b). We now show that there exists a balanced, center-free set containing n points for
all odd n ě 3, and that one does not exist for any even n ě 3.
If n is odd, then let V be the set of vertices of a regular n-gon. We have shown in part (a)
that V is balanced. We claim that V is also center-free. Indeed, if P is a point such that
28 IMO 2015 Thailand
Comment (b). We can rephrase the argument in graph theoretic terms as follows. Let V be a
balanced, center-free set consisting of n points. For any pair of distinct vertices A, B P V and for
any C P V such that AC “ BC, draw directed edges A Ñ C and B Ñ C. Then all pairs of vertices
generate altogether at least npn´1q directed edges; since the set is center-free, these edges are distinct.
So we must obtain a graph in which any two vertices are connected in both directions. Now, each
vertex has exactly n ´ 1 incoming edges, which means that n ´ 1 is even. Hence n is odd.
Shortlisted problems – solutions 29
C3. For a finite set A of positive integers, we call a partition of A into two disjoint nonempty
subsets A1 and A2 good if the least common multiple of the elements in A1 is equal to the
greatest common divisor of the elements in A2 . Determine the minimum value of n such that
there exists a set of n positive integers with exactly 2015 good partitions.
(Ukraine)
Answer. 3024.
Solution. Let A “ ta1 , a2 , . . . , an u, where a1 ă a2 ă ¨ ¨ ¨ ă an . For a finite nonempty set B
of positive integers, denote by lcm B and gcd B the least common multiple and the greatest
common divisor of the elements in B, respectively.
Consider any good partition pA1 , A2 q of A. By definition, lcm A1 “ d “ gcd A2 for some
positive integer d. For any ai P A1 and aj P A2 , we have ai ď d ď aj . Therefore, we have
A1 “ ta1 , a2 , . . . , ak u and A2 “ tak`1 , ak`2 , . . . , an u for some k with 1 ď k ă n. Hence, each
good partition is determined by an element ak , where 1 ď k ă n. We call such ak partitioning.
It is convenient now to define ℓk “ lcmpa1 , a2 , . . . , ak q and gk “ gcdpak`1 , ak`2 , . . . , an q for
1 ď k ď n ´ 1. So ak is partitioning exactly when ℓk “ gk .
We proceed by proving some properties of partitioning elements, using the following claim.
Claim. If ak´1 and ak are partitioning where 2 ď k ď n ´ 1, then gk´1 “ gk “ ak .
Proof. Assume that ak´1 and ak are partitioning. Since ℓk´1 “ gk´1 , we have ℓk´1 | ak .
Therefore, gk “ ℓk “ lcmpℓk´1 , ak q “ ak , and gk´1 “ gcdpak , gk q “ ak , as desired. l
Property 1. For every k “ 2, 3, . . . , n ´ 2, at least one of ak´1 , ak , and ak`1 is not partitioning.
Proof. Suppose, to the contrary, that all three numbers ak´1 , ak , and ak`1 are partitioning. The
claim yields that ak`1 “ gk “ ak , a contradiction. l
Property 2. The elements a1 and a2 cannot be simultaneously partitioning. Also, an´2 and
an´1 cannot be simultaneously partitioning
Proof. Assume that a1 and a2 are partitioning. By the claim, it follows that a2 “ g1 “ ℓ1 “
lcmpa1 q “ a1 , a contradiction.
Similarly, assume that an´2 and an´1 are partitioning. The claim yields that an´1 “ gn´1 “
gcdpan q “ an , a contradiction. l
Now let A be an n-element set with exactly 2015 good partitions. Clearly, we have
n ě 5. Using Property 2, we find that there is at most one partitioning
X n´5 \ element in each
of ta1 , a2 u and tan´2 , an´1 u. By Property 1, there are at least 3 non-partitioning elements
\ P 2pn´2q T
in ta3 , a4 , . . . , an´3 u. Therefore, there are at most pn ´ 1q ´ 2 ´ n´5
X
3
“ 3
partitioning
P 2pn´2q T
elements in A. Thus, 3
ě 2015, which implies that n ě 3024.
Finally, we show that there exists a set of 3024 positive integers with exactly 2015 parti-
tioning elements. Indeed, in the set A “ t2 ¨ 6i , 3 ¨ 6i , 6i`1 | 0 ď i ď 1007u, each element of the
form 3 ¨ 6i or 6i , except 61008 , is partitioning.
Therefore, the minimum possible value of n is 3024.
Comment. Here we will work out P theT general case when 2015 is replaced by an arbitrary positive
integer m. Note that the bound 2pn´2q 3 ě m obtained in the solution is, in fact, true for any positive
integers m and n. Using this bound, one can find that n ě 3m
P T
` 1.
P23m T
To show that the bound is sharp, one constructs a set of 2 ` 1 elements with exactly m good
partitions. Indeed, the minimum is attained on the set t6i , 2 ¨ 6i , 3 ¨ 6i | 0 ď i ď t ´ 1u Y t6t u for every
even m “ 2t, and t2 ¨ 6i , 3 ¨ 6i , 6i`1 | 0 ď i ď t ´ 1u for every odd m “ 2t ´ 1.
30 IMO 2015 Thailand
C4. Let n be a positive integer. Two players A and B play a game in which they take turns
choosing positive integers k ď n. The rules of the game are:
piq A player cannot choose a number that has been chosen by either player on any previous
turn.
piiq A player cannot choose a number consecutive to any of those the player has already chosen
on any previous turn.
piiiq The game is a draw if all numbers have been chosen; otherwise the player who cannot
choose a number anymore loses the game.
The player A takes the first turn. Determine the outcome of the game, assuming that both
players use optimal strategies.
(Finland)
Comment 2. One may also employ symmetry when n is odd. In particular, B could use a mirror
strategy. However, additional ideas are required to modify the strategy after A picks n`1
2 .
32 IMO 2015 Thailand
C5. Consider an infinite sequence a1 , a2 , . . . of positive integers with ai ď 2015 for all i ě 1.
Suppose that for any two distinct indices i and j we have i ` ai ‰ j ` aj .
Prove that there exist two positive integers b and N such that
ˇ ˇ
ˇ ÿn ˇ
pai ´ bqˇ ď 10072
ˇ ˇ
ˇ
ˇi“m`1 ˇ
whenever n ą m ě N.
(Australia)
Solution 1. We visualize the set of positive integers as a sequence of points. For each n we
draw an arrow emerging from n that points to n ` an ; so the length of this arrow is an . Due to
the condition that m ` am ‰ n ` an for m ‰ n, each positive integer receives at most one arrow.
There are some positive integers, such as 1, that receive no arrows; these will be referred to as
starting points in the sequel. When one starts at any of the starting points and keeps following
the arrows, one is led to an infinite path, called its ray, that visits a strictly increasing sequence
of positive integers. Since the length of any arrow is at most 2015, such a ray, say with starting
point s, meets every interval of the form rn, n ` 2014s with n ě s at least once.
Suppose for the sake of contradiction that there would be at least 2016 starting points.
Then we could take an integer n that is larger than the first 2016 starting points. But now the
interval rn, n ` 2014s must be met by at least 2016 rays in distinct points, which is absurd. We
have thereby shown that the number b of starting points satisfies 1 ď b ď 2015. Let N denote
any integer that is larger than all starting points. We contend that b and N are as required.
To see this, let any two integers m and n with n ą m ě N be given. The sum ni“m`1 ai
ř
gives the total length of the arrows emerging from m ` 1, . . . , n. Taken together, these arrows
form b subpaths of our rays, some of which may be empty. Now on each ray we look at
the first number that is larger than m; let x1 , . . . , xb denote these numbers, and let y1 , . . . , yb
enumerate in corresponding order the numbers defined similarly with respect to n. Then the
list of differences y1 ´ x1 , . . . , yb ´ xb consists of the lengths of these paths and possibly some
zeros corresponding to empty paths. Consequently, we obtain
n
ÿ b
ÿ
ai “ pyj ´ xj q ,
i“m`1 j“1
whence
n
ÿ b
ÿ b
ÿ
pai ´ bq “ pyj ´ nq ´ pxj ´ mq .
i“m`1 j“1 j“1
Now each of the b rays meets the interval rm ` 1, m ` 2015s at some point and thus x1 ´
m, . . . , xb ´ m are b distinct members of the set t1, 2, . . . , 2015u. Moreover, since m ` 1 is not a
starting point, it must belong to some ray; so 1 has to appear among these numbers, wherefore
b´1
ÿ b
ÿ b´1
ÿ
1` pj ` 1q ď pxj ´ mq ď 1 ` p2016 ´ b ` jq .
j“1 j“1 j“1
b´1
ÿ b
ÿ b´1
ÿ
1` pj ` 1q ď pyj ´ nq ď 1 ` p2016 ´ b ` jq .
j“1 j“1 j“1
Shortlisted problems – solutions 33
So altogether we get
ˇ ˇ
ˇ ÿn ˇ b´1
ˇ ÿ` ˘
pai ´ bqˇ ď p2016 ´ b ` jq ´ pj ` 1q “ pb ´ 1qp2015 ´ bq
ˇ
ˇ
ˇi“m`1 ˇ j“1
ˆ ˙2
pb ´ 1q ` p2015 ´ bq
ď “ 10072 ,
2
as desired.
n ` 1 ď sn ď n ` 2015
for all n P Zą0 . The members of the sequence s1 , s2 , . . . are distinct. We shall investigate the
set
M “ Zą0 z ts1 , s2 , . . .u .
where on the left-hand side we have a disjoint union containing altogether n ` 2016 elements.
But the set on the right-hand side has only n ` 2015 elements. This contradiction proves our
claim. l
Now we work towards proving that the positive integers b “ |M| and N “ maxpMq are as
required. Recall that we have just shown b ď 2015.
Let us consider any integer r ě N. As in the proof of the above claim, we see that
Br “ M Y ts1 , . . . , sr u (1)
is a subset of r1, r ` 2015s X Z with precisely b ` r elements. Due to the definitions of M and N,
we also know r1, r ` 1s X Z Ď Br . It follows that there is a set Cr Ď t1, 2, . . . , 2014u with
|Cr | “ b ´ 1 and ` ˘ ˇ (
Br “ r1, r ` 1s X Z Y r ` 1 ` x ˇ x P Cr . (2)
ř
For any finite set of integers J we denote theřsum of its elements by J. Now the equations (1)
and (2) give rise to two ways of computing Br and the comparison of both methods leads to
ÿ r
ÿ r
ÿ ÿ
M` si “ i ` bpr ` 1q ` Cr ,
i“1 i“1
or in other words to
ÿ r
ÿ ÿ
M` pai ´ bq “ b ` Cr . (3)
i“1
After this preparation, we consider any two integers m and n with n ą m ě N. Plugging
r “ n and r “ m into (3) and subtracting the estimates that result, we deduce
n
ÿ ÿ ÿ
pai ´ bq “ Cn ´ Cm .
i“m`1
34 IMO 2015 Thailand
Since Cn and Cm are subsets of t1, 2, . . . , 2014u with |Cn | “ |Cm | “ b ´ 1, it is clear that the
absolute value of the right-hand side of the above inequality attains its largest possible value if
either Cm “ t1, 2, . . . , b ´ 1u and Cn “ t2016 ´ b, . . . , 2014u, or the other way around. In these
two cases we have ˇÿ ÿ ˇ
ˇ Cn ´ Cm ˇ “ pb ´ 1qp2015 ´ bq ,
ˇ ˇ
as desired.
Comment. The sets Cn may be visualized by means of the following process: Start with an empty
blackboard. For n ě 1, the following happens during the nth step. The number an gets written on
the blackboard, then all numbers currently on the blackboard are decreased by 1, and finally all zeros
that have arisen get swept away.
It is not hard to see that the numbers present on the blackboard after n steps are distinct and
form the set Cn . Moreover, it is possible to complete a solution based on this idea.
Shortlisted problems – solutions 35
C6. Let S be a nonempty set of positive integers. We say that a positive integer n is clean if
it has a unique representation as a sum of an odd number of distinct elements from S. Prove
that there exist infinitely many positive integers that are not clean.
(U.S.A.)
Solution 1. Define an odd (respectively, even) representation of n to be a representation of n
as a sum of an odd (respectively, even) number of distinct elements of S. Let Zą0 denote the
set of all positive integers.
Suppose, to the contrary, that there exist only finitely many positive integers that are not
clean. Therefore, there exists a positive integer N such that every integer n ą N has exactly
one odd representation.
Clearly, S is infinite. We now claim the following properties of odd and even representations.
Property 1. Any positive integer n has at most one odd and at most one even representation.
Proof. We first show that every integer n has at most one even representation. Since S is infinite,
there exists x P S such that x ą maxtn, Nu. Then, the number n ` x must be clean, and x does
not appear in any even representation of n. If n has more than one even representation, then
we obtain two distinct odd representations of n ` x by adding x to the even representations
of n, which is impossible. Therefore, n can have at most one even representation.
Similarly, there exist two distinct elements y, z P S such that y, z ą maxtn, Nu. If n has
more than one odd representation, then we obtain two distinct odd representations of n ` y ` z
by adding y and z to the odd representations of n. This is again a contradiction. l
Property 2. Fix s P S. Suppose that a number n ą N has no even representation. Then
n ` 2as has an even representation containing s for all integers a ě 1.
Proof. It is sufficient to prove the following statement: If n has no even representation without s,
then n`2s has an even representation containing s (and hence no even representation without s
by Property 1).
Notice that the odd representation of n ` s does not contain s; otherwise, we have an even
representation of n without s. Then, adding s to this odd representation of n ` s, we get that
n ` 2s has an even representation containing s, as desired. l
Property 3. Every sufficiently large integer has an even representation.
Proof. Fix any s P S, and let r be an arbitrary element in t1, 2, . . . , 2su. Then, Property 2
implies that the set Zr “ tr ` 2as : a ě 0u contains at most one number exceeding N with
no even representation. Therefore,ŤZr contains finitely many positive integers with no even
representation, and so does Zą0 “ 2s r“1 Zr . l
In view of Properties 1 and 3, we may assume that N is chosen such that every n ą N has
exactly one odd and exactly one even representation. In particular, each element s ą N of S
has an even representation.
Property 4. For any s, t P S with N ă s ă t, the even representation of t contains s.
Proof. Suppose the contrary. Then, s ` t has at least two odd representations: one obtained by
adding s to the even representation of t and one obtained by adding t to the even representation
of s. Since the latter does not contain s, these two odd representations of s ` t are distinct, a
contradiction. l
řn
Let s1 ă s2 ă ¨ ¨ ¨ be all the elements of S, and set σn “ i“1 si for each nonnegative
integer n. Fix an integer k such that sk ą N. Then, Property 4 implies that for every i ą k
the even representation of si contains all the numbers sk , sk`1, . . . , si´1 . Therefore,
Let j0 be an integer satisfying j0 ą k and σj0 ą 2σk´1 . Then (1) shows that, for every j ą j0 ,
sj`1 ě σj ´ σk´1 ą σj {2. (2)
Next, let p ą j0 be an index such that Rp “ miniąj0 Ri . Then,
sp`1 “ sk ` sk`1 ` ¨ ¨ ¨ ` sp ` Rp`1 “ psp ´ Rp q ` sp ` Rp`1 ě 2sp .
Therefore, there is no element of S larger than sp but smaller than 2sp . It follows that the
even representation τ of 2sp does not contain any element larger than sp . On the other hand,
inequality (2) yields 2sp ą s1 ` ¨ ¨ ¨ ` sp´1, so τ must contain a term larger than sp´1 . Thus,
it must contain sp . After removing sp from τ , we have that sp has an odd representation not
containing sp , which contradicts Property 1 since sp itself also forms an odd representation
of sp .
This easily yields that Zě0 z E is also finite. Since Zě0 z O is also finite, by Property 1, there
exists a positive integer N such that every integer n ą N has exactly one even and one odd
representation.
Step 3. We investigate the structures of En and On .
Suppose that z P E2n . Since z can be represented as an even sum using ts1 , s2 , . . . , s2n u, so
can its complement σ2n ´ z. Thus, we get E2n “ σ2n ´ E2n . Similarly, we have
E2n “ σ2n ´ E2n , O2n “ σ2n ´ O2n , E2n`1 “ σ2n`1 ´ O2n`1, O2n`1 “ σ2n`1 ´ E2n`1 . (4)
Proof. Clearly On , En Ď r0, σn s for every positive integer n. We now prove On , En Ě pN, σn ´Nq.
Taking n sufficiently large, we may assume that sn`1 ě 2n´1 `1´nm ą 21 p2n´1 ´1`nmq ě σn {2.
Therefore, the odd representation of every element of pN, σn {2s cannot contain a term larger
than sn . Thus, pN, σn {2s Ď On . Similarly, since sn`1 ` s1 ą σn {2, we also have pN, σn {2s Ď En .
Equations (4) then yield that, for sufficiently large n, the interval pN, σn ´ Nq is a subset of
both On and En , as desired. l
Step 4. We obtain a final contradiction.
Notice that 0 P Zě0 z O and 1 P Zě0 z E. Therefore, the sets Zě0 z O and Zě0 z E are
nonempty. Denote o “ maxpZě0 z Oq and e “ maxpZě0 z Eq. Observe also that e, o ď N.
Taking k sufficiently large, we may assume that σ2k ą 2N and that Claim 2 holds for
all n ě 2k. Due to (4) and Claim 2, we have that σ2k ´ e is the minimal number greater than N
which is not in E2k , i.e., σ2k ´ e “ s2k`1 ` s1 . Similarly,
Therefore, we have
C7. In a company of people some pairs are enemies. A group of people is called unsociable
if the number of members in the group is odd and at least 3, and it is possible to arrange all
its members around a round table so that every two neighbors are enemies. Given that there
are at most 2015 unsociable groups, prove that it is possible to partition the company into 11
parts so that no two enemies are in the same part.
(Russia)
Solution 1. Let G “ pV, Eq be a graph where V is the set of people in the company and E
is the set of the enemy pairs — the edges of the graph. In this language, partitioning into 11
disjoint enemy-free subsets means properly coloring the vertices of this graph with 11 colors.
We will prove the following more general statement.
Claim. Let G be a graph with chromatic number k ě 3. Then G contains at least 2k´1 ´ k
unsociable groups.
Recall that the chromatic number of G is the least k such that a proper coloring
V “ V1 \ ¨ ¨ ¨ \ Vk (1)
exists. In view of 211 ´ 12 ą 2015, the claim implies the problem statement.
Let G be a graph with chromatic number k. We say that a proper coloring (1) of G is
leximinimal, if the k-tuple p|V1 |, |V2 |, . . . , |Vk |q is lexicographically minimal; in other words, the
following conditions are satisfied: the number n1 “ |V1 | is minimal; the number n2 “ |V2 | is
minimal, subject to the previously chosen value of n1 ; . . . ; the number nk´1 “ |Vk´1 | is minimal,
subject to the previously chosen values of n1 , . . . , nk´2 .
The following lemma is the core of the proof.
Lemma 1. Suppose that G “ pV, Eq is a graph with odd chromatic number k ě 3, and let (1)
be one of its leximinimal colorings. Then G contains an odd cycle which visits all color classes
V1 , V2 , . . . , Vk .
Proof of Lemma 1. Let us call a cycle colorful if it visits all color classes.
Due to the definition of the chromatic number, V1 is nonempty. Choose an arbitrary vertex
v P V1 . We construct a colorful odd cycle that has only one vertex in V1 , and this vertex is v.
We draw a subgraph of G as follows. Place v in the center, and arrange the sets V2 , V3 , . . . , Vk
in counterclockwise circular order around it. For convenience, let Vk`1 “ V2 . We will draw
arrows to add direction to some edges of G, and mark the vertices these arrows point to. First
we draw arrows from v to all its neighbors in V2 , and mark all those neighbors. If some vertex
u P Vi with i P t2, 3, . . . , ku is already marked, we draw arrows from u to all its neighbors
in Vi`1 which are not marked yet, and we mark all of them. We proceed doing this as long as
it is possible. The process of marking is exemplified in Figure 1.
Notice that by the rules of our process, in the final state, marked vertices in Vi cannot have
unmarked neighbors in Vi`1 . Moreover, v is connected to all marked vertices by directed paths.
Now move each marked vertex to the next color class in circular order (see an example in
Figure 3). In view of the arguments above, the obtained coloring V1 \ W2 \ ¨ ¨ ¨ \ Wk is proper.
Notice that v has a neighbor w P W2 , because otherwise
` ˘ ` ˘
V1 z tvu \ W2 Y tvu \ W3 \ ¨ ¨ ¨ \ Wk
would be a proper coloring lexicographically smaller than (1). If w was unmarked, i.e., w was
an element of V2 , then it would be marked at the beginning of the process and thus moved
to V3 , which did not happen. Therefore, w is marked and w P Vk .
Shortlisted problems – solutions 39
V5 V4 W5 W4
w
w
v −→ v
w
w
V2 V3 W2 W3
Figure 1 Figure 2
Since w is marked, there exists a directed path from v to w. This path moves through the
sets V2 , . . . , Vk in circular order, so the number of edges in it is divisible by k ´ 1 and thus even.
Closing this path by the edge w Ñ v, we get a colorful odd cycle, as required. l
Proof of the claim. Let us choose a leximinimal coloring (1) of G. For every set C Ď t1, 2, . . . , ku
such that |C| is odd and greater than 1, we will provide an odd cycle visiting exactly those
color classes whose indices are listed in the set C. This property ensures that we have different
cycles for different choices of C, and it proves the claim because there are 2k´1 ´ k choices for
the set C. Ť
Let VC “ cPC Vc , and let GC be the induced subgraph of G on the vertex set VC . We
also have the induced coloring of VC with |C| colors; this coloring is of course proper. Notice
further that the induced coloring is leximinimal: if we had a lexicographically smaller coloring
pWc qcPC of GC , then these classes, together the original color classes Vi for i R C, would provide
a proper coloring which is lexicographically smaller than (1). Hence Lemma 1, applied to the
subgraph GC and its leximinimal coloring pVc qcPC , provides an odd cycle that visits exactly
those color classes that are listed in the set C. l
Solution 2. We provide a different proof of the claim from the previous solution.
We say that a graph is critical if deleting any vertex from the graph decreases the graph’s
chromatic number. Obviously every graph contains a critical induced subgraph with the same
chromatic number.
Lemma 2. Suppose that G “ pV, Eq is a critical graph with chromatic number k ě 3. Then
every vertex v of G is contained in at least 2k´2 ´ 1 unsociable groups.
Proof. For every set X Ď V , denote by npXq the number of neighbors of v in the set X.
Since G is critical, there exists a proper coloring of G z tvu with k ´ 1 colors, so there exists
a proper coloring V “ V1 \ V2 `\ ¨ ¨ ¨ \ Vk of G such that ˘ V1 “ tvu. Among such colorings,
take one for which the sequence npV2 q, npV3 q, . . . , npVk q is lexicographically minimal. Clearly,
npVi q ą 0 for every i “ 2, 3, . . . , k; otherwise V2 \ . . . \ Vi´1 \ pVi Y V1 q \ Vi`1 \ . . . Vk would
be a proper coloring of G with k ´ 1 colors.
We claim that for every C Ď t2, 3, . . . , ku with |C| ě 2 being even, G contains an unsociable
group so that the set of its members’ colors is precisely C Y t1u. Since the number of such
sets C is 2k´2 ´ 1, this proves the lemma. Denote the elements of C by c1 , . . . , c2ℓ in increasing
order. For brevity, let Ui “ Vci . Denote by Ni the set of neighbors of v in Ui .
40 IMO 2015 Thailand
Ui Ui+1
P Q
x Ni+1
Ni S
v
Figure 3
Next, we build a path through U1 , U2 , . . . , U2ℓ as follows. Let the starting point of the path
be an arbitrary vertex v1 in the set N1 . For i ď 2ℓ ´ 1, if the vertex vi P Ni is already defined,
connect vi to some vertex in Ni`1 in the subgraph induced by Ui Y Ui`1 , and add these edges to
the path. Denote the new endpoint of the path by vi`1 ; by the construction we have vi`1 P Ni`1
again, so the process can be continued. At the end we have a path that starts at v1 P N1 and
ends at some v2ℓ P N2ℓ . Moreover, all edges in this path connect vertices in neighboring classes:
if a vertex of the path lies in Ui , then the next vertex lies in Ui`1 or Ui´1 . Notice that the path
is not necessary simple, so take a minimal subpath of it. The minimal subpath is simple and
connects the same endpoints v1 and v2ℓ . The property that every edge steps to a neighboring
color class (i.e., from Ui to Ui`1 or Ui´1 ) is preserved. So the resulting path also visits all of
U1 , . . . , U2ℓ , and its length must be odd. Closing the path with the edges vv1 and v2ℓ v we obtain
the desired odd cycle (see Figure 4). l
U1 U2 U3 U2ℓ−1 U2ℓ
...
N1 N2 N3 N2ℓ−1 N2ℓ
v1 v2 v3 v2ℓ
v2ℓ−1
v
Figure 4
Now we prove the claim by induction on k ě 3. The base case k “ 3 holds by applying
Lemma 2 to a critical subgraph. For the induction step, let G0 be a critical k-chromatic sub-
graph of G, and let v be an arbitrary vertex of G0 . By Lemma 2, G0 has at least 2k´2 ´ 1
unsociable groups containing v. On the other hand, the graph G0 z tvu has chromatic num-
ber k ´ 1, so it contains at least 2k´2 ´ pk ´ 1q unsociable groups by the induction hypothesis.
Altogether, this gives 2k´2 ´ 1 ` 2k´2 ´ pk ´ 1q “ 2k´1 ´ k distinct unsociable groups in G0 (and
thus in G).
Shortlisted problems – solutions 41
Comment 1. The claim we proved is sharp. The complete graph with k vertices has chromatic
number k and contains exactly 2k´1 ´ k unsociable groups.
Comment 2. The proof of Lemma 2 works for odd values of |C| ě 3 as well. Hence, the second
solution shows the analogous statement that the number of even sized unsociable groups is at least
2k ´ 1 ´ k2 .
` ˘
42 IMO 2015 Thailand
Geometry
G1. Let ABC be an acute triangle with orthocenter H. Let G be the point such that the
quadrilateral ABGH is a parallelogram. Let I be the point on the line GH such that AC
bisects HI. Suppose that the line AC intersects the circumcircle of the triangle GCI at C
and J. Prove that IJ “ AH.
(Australia)
Solution 1. Since HG k AB and BG k AH, we have BG K BC and CH K GH. There-
fore, the quadrilateral BGCH is cyclic. Since H is the orthocenter of the triangle ABC, we
have =HAC “ 900 ´=ACB “ =CBH. Using that BGCH and CGJI are cyclic quadrilaterals,
we get
=CJI “ =CGH “ =CBH “ =HAC.
Let M be the intersection of AC and GH, and let D ‰ A be the point on the line AC such
that AH “ HD. Then =MJI “ =HAC “ =MDH.
Since =MJI “ =MDH, =IMJ “ =HMD, and IM “ MH, the triangles IMJ and
HMD are congruent, and thus IJ “ HD “ AH.
I M
G
H
A B
J
Comment. Instead of introducing the point D, one can complete the solution by using the law of
sines in the triangles IJM and AM H, yielding
IJ sin =IM J sin =AM H AH AH
“ “ “ “ .
IM sin =M JI sin =HAM MH IM
G2. Let ABC be a triangle inscribed into a circle Ω with center O. A circle Γ with center A
meets the side BC at points D and E such that D lies between B and E. Moreover, let F and
G be the common points of Γ and Ω. We assume that F lies on the arc AB of Ω not containing
C, and G lies on the arc AC of Ω not containing B. The circumcircles of the triangles BDF
and CEG meet the sides AB and AC again at K and L, respectively. Suppose that the lines
F K and GL are distinct and intersect at X. Prove that the points A, X, and O are collinear.
(Greece)
Solution 1. It suffices to prove that the lines F K and GL are symmetric about AO. Now
the segments AF and AG, being chords of Ω with the same length, are clearly symmetric with
respect to AO. Hence it is enough to show
Let us denote the circumcircles of BDF and CEG by ωB and ωC , respectively. To prove (1),
we start from
=KF A “ =DF G ` =GF A ´ =DF K .
In view of the circles ωB , Γ, and Ω, this may be rewritten as
Due to the circles ωC and Ω, we obtain =KF A “ =CLG ´ =CAG “ =AGL. Thereby the
problem is solved.
A
Ω
X L
L
G
G
K
K
Γ ωC
F
F
O
B C
D
D E
E
ωB
Figure 1
Moreover, because of the circle ωB we have =DF K “ =CBA. Altogether, this yields
` ˘
2 =KF A “ =DF A ` =BF D ` =EBF ´ ψ ´ 2=CBA ,
which simplifies to
2 =KF A “ =BF A ` ϕ ´ ψ ´ =CBA .
Now the quadrilateral AF BC is cyclic, so this entails 2 =KF A “ α ` ϕ ´ ψ.
Due to the “symmetry” between B and C alluded to above, this argument also shows that
2 =AGL “ α ` ϕ ´ ψ. This concludes the proof of (1).
A
Ω
Γ
K
K
F
F
ωB G
G
ϕ ϕ
ψ ψ
B C
DD EE
Figure 2
Comment 1. As the first solution shows, the assumption that A be the center of Γ may be weakened
to the following one: The center of Γ lies on the line OA. The second solution may be modified to
yield the same result.
Comment 2. It might be interesting to remark that =GDK “ 900 . To prove this, let G1 denote
the point on Γ diametrically opposite to G. Because of =KDF “ =KBF “ =AGF “ =G1 DF , the
points D, K, and G1 are collinear, which leads to the desired result. Notice that due to symmetry we
also have =LEF “ 900 .
Moreover, a standard argument shows that the triangles AGL and BGE are similar. By symmetry
again, also the triangles AF K and CDF are similar.
There are several ways to derive a solution from these facts. For instance, one may argue that
=KF A “ =BF A ´ =BF K “ =BF A ´ =EDG1 “ p1800 ´ =AGBq ´ p1800 ´ =G1 GEq
“ =AGE ´ =AGB “ =BGE “ =AGL .
Comment 3. The original proposal did not contain the point X in the assumption and asked instead
to prove that the lines F K, GL, and AO are concurrent. This differs from the version given above only
insofar as it also requires to show that these lines cannot be parallel. The Problem Selection Committee
removed this part from the problem intending to make it thus more suitable for the Olympiad.
For the sake of completeness, we would still like to sketch one possibility for proving F K ∦ AO here.
As the points K and O lie in the angular region =F AG, it suffices to check =KF A ` =F AO ă 1800 .
Multiplying by 2 and making use of the formulae from the second solution, we see that this is equivalent
to pα ` ϕ ´ ψq ` p1800 ´ 2ϕq ă 3600 , which in turn is an easy consequence of α ă 1800 .
46 IMO 2015 Thailand
G3. Let ABC be a triangle with =C “ 900, and let H be the foot of the altitude from C.
A point D is chosen inside the triangle CBH so that CH bisects AD. Let P be the intersection
point of the lines BD and CH. Let ω be the semicircle with diameter BD that meets the
segment CB at an interior point. A line through P is tangent to ω at Q. Prove that the
lines CQ and AD meet on ω.
(Georgia)
Solution 1. Let K be the projection of D onto AB; then AH “ HK (see Figure 1). Since
P H k DK, we have
PD HK AH
“ “ . (1)
PB HB HB
Let L be the projection of Q onto DB. Since P Q is tangent to ω and =DQB “ =BLQ “
900 , we have =P QD “ =QBP “ =DQL. Therefore, QD and QB are respectively the internal
and the external bisectors of =P QL. By the angle bisector theorem, we obtain
PD PQ PB
“ “ . (2)
DL QL BL
AH PD DL
The relations (1) and (2) yield“ “ . So, the spiral similarity τ centered at B
HB PB LB
and sending A to D maps H to L. Moreover, τ sends the semicircle with diameter AB passing
through C to ω. Due to CH K AB and QL K DB, it follows that τ pCq “ Q.
Hence, the triangles ABD and CBQ are similar, so =ADB “ =CQB. This means that the
lines AD and CQ meet at some point T , and this point satisfies =BDT “ =BQT . Therefore,
T lies on ω, as needed.
Γ
C C
Q Q′
T T
P ω P ω
D L D
A H K B A H K B
Figure 1 Figure 2
Comment 1. Since =BAD “ =BCQ, the point T lies also on the circumcircle of the triangle ABC.
Solution 2. Let Γ be the circumcircle of ABC, and let AD meet ω at T . Then =AT B “
=ACB “ 900 , so T lies on Γ as well. As in the previous solution, let K be the projection of D
onto AB; then AH “ HK (see Figure 2).
Our goal now is to prove that the points C, Q, and T are collinear. Let CT meet ω again
at Q1 . Then, it suffices to show that P Q1 is tangent to ω, or that =P Q1 D “ =Q1 BD.
Since the quadrilateral BDQ1 T is cyclic and the triangles AHC and KHC are congruent, we
have =Q1 BD “ =Q1 T D “ =CT A “ =CBA “ =ACH “ =HCK. Hence, the right triangles
1 HK Q1 D
CHK and BQ D are similar. This implies that “ , and thus HK ¨ BD “ CK ¨ Q1 D.
CK BD
PD HK
Notice that P H k DK; therefore, we have “ , and so P D ¨ BK “ HK ¨ BD.
BD BK
PD CK
Consequently, P D ¨ BK “ HK ¨ BD “ CK ¨ Q1 D, which yields 1 “ .
QD BK
Since =CKA “ =KAC “ =BDQ1 , the triangles CKB and P DQ1 are similar, so =P Q1 D “
=CBA “ =Q1 BD, as required.
Shortlisted problems – solutions 47
Comment 2. There exist several other ways to prove that P Q1 is tangent to ω. For instance, one
PD P Q1
may compute and in terms of AH and HB to verify that P Q12 “ P D ¨ P B, concluding that
PB PB
P Q1 is tangent to ω.
Another possible approach is the following. As in Solution 2, we introduce the points T and Q1
and mention that the triangles ABC and DBQ1 are similar (see Figure 3).
Let M be the midpoint of AD, and let L be the projection of Q1 onto AB. Construct E on the
line AB so that EP is parallel to AD. Projecting from P , we get pA, B; H, Eq “ pA, D; M, 8q “ ´1.
EA PD
Since “ , the point P is the image of E under the similarity transform mapping ABC
AB DB
to DBQ1 . Therefore, we have pD, B; L, P q “ pA, B; H, Eq “ ´1, which means that Q1 D and Q1 B are
respectively the internal and the external bisectors of =P Q1 L. This implies that P Q1 is tangent to ω,
as required.
C
Q′
T
P ω
M D L
E A H K B
Figure 3
Solution 3. Introduce the points T and Q1 as in the previous solution. Note that T lies on
the circumcircle of ABC. Here we present yet another proof that P Q1 is tangent to ω.
Let Ω be the circle completing the semicircle ω. Construct a point F symmetric to C with
respect to AB. Let S ‰ T be the second intersection point of F T and Ω (see Figure 4).
C
Q′
T
P D
A K B
H
S
Figure 4
Since AC “ AF , we have =DKC “ =HCK “ =CBA “ =CT A “ =DT S “ 1800 ´
=SKD. Thus, the points C, K, and S are collinear. Notice also that =Q1 KD “ =Q1 T D “
=HCK “ =KF H “ 1800 ´ =DKF . This implies that the points F, K, and Q1 are collinear.
Applying Pascal’s theorem to the degenerate hexagon KQ1 Q1 T SS, we get that the tan-
gents to Ω passing through Q1 and S intersect on CF . The relation =Q1 T D “ =DT S yields
that Q1 and S are symmetric with respect to BD. Therefore, the two tangents also intersect
on BD. Thus, the two tangents pass through P . Hence, P Q1 is tangent to ω, as needed.
48 IMO 2015 Thailand
G4. Let ABC be an acute triangle, and let M be the midpoint of AC. A circle ω passing
through B and M meets the sides AB and BC again at P and Q, respectively. Let T be
the point such that the quadrilateral BP T Q is a parallelogram. Suppose that T lies on the
circumcircle of the triangle ABC. Determine all possible values of BT {BM.
(Russia)
?
Answer. 2.
Solution 1. Let S be the center of the parallelogram BP T Q, and let B 1 ‰ B be the point on
the ray BM such that BM “ MB 1 (see Figure 1). It follows that ABCB 1 is a parallelogram.
Then, =ABB 1 “ =P QM and =BB 1 A “ =B 1 BC “ =MP Q, and so the triangles ABB 1 and
MQP are similar. It follows that AM and MS are corresponding medians in these triangles.
Hence,
=SMP “ =B 1 AM “ =BCA “ =BT A. (1)
Since =ACT “ =P BT and =T AC “ =T BC “ =BT P , the triangles T CA and P BT are
similar. Again, as T M and P S are corresponding medians in these triangles, we have
=MT A “ =T P S “ =BQP “ =BMP. (2)
Now we deal separately with two cases.
Case 1. S does not lie on BM. Since the configuration is symmetric between A and C, we
may assume that S and A lie on the same side with respect to the line BM.
Applying (1) and (2), we get
=BMS “ =BMP ´ =SMP “ =MT A ´ =BT A “ =MT B,
the triangles BSM and BMT are similar. We now have BM 2 “ BS ¨ BT “ BT 2 {2, so
and so ?
BT “ 2BM.
Case 2. S lies on BM. It follows from (2) that =BCA “ =MT A “ =BQP “ =BMP
(see Figure 2). Thus, P Q?k AC and P M k AT . Hence, BS{BM “ BP {BA “ BM{BT , so
BT 2 “ 2BM 2 and BT “ 2BM.
B
Q
S
S
P
P
A M
M C
S Q
P
T A C
M
B′ T
Figure 1 Figure 2
Shortlisted problems – solutions 49
Comment 1. Here is another way to show that the triangles BSM and BM T are similar. Denote
by Ω the circumcircle of the triangle ABC. Let R be the second point of intersection of ω and Ω, and
let τ be the spiral similarity centered at R mapping ω to Ω. Then, one may show that τ maps each
point X on ω to a point Y on Ω such that B, X, and Y are collinear (see Figure 3). If we let K and L
be the second points of intersection of BM with Ω and of BT with ω, respectively, then it follows that
the triangle M KT is the image of SM L under τ . We now obtain =BSM “ =T M B, which implies
the desired result.
B
Ω ω
Ω X ω P
Q
Y M
M
A C
P
S R X
Q
Q
M
M T
A C
L
Y
K T B′
Figure 3 Figure 4
BQ ¨ BC “ BJ ¨ BT. (3)
50 IMO 2015 Thailand
We also have =T JQ “ 1800 ´ =QCT “ =T AB and =QT J “ =ABT , and so the triangles
T JQ and BAT are similar. We now have T J{T Q “ BA{BT . Therefore,
T J ¨ BT “ T Q ¨ BA “ BP ¨ BA. (4)
BX ¨ BP ` BY ¨ BQ “ BM 2
and
BP ¨ BA ` BQ ¨ BC “ BT 2 .
?
Since BA “ 2BX and BC “ 2BY , we have BT 2 “ 2BM 2 , and so BT “ 2BM.
B
B
Y
X
Q Q
P J P
P
A C A M
M C
T T
Figure 5 Figure 6
Comment 2. Here we give another proof of the lemma using Ptolemy’s theorem. We readily have
T C ¨ BA ` T A ¨ BC “ AC ¨ BT.
G5. Let ABC be a triangle with CA ‰ CB. Let D, F , and G be the midpoints of the
sides AB, AC, and BC, respectively. A circle Γ passing through C and tangent to AB at D
meets the segments AF and BG at H and I, respectively. The points H 1 and I 1 are symmetric
to H and I about F and G, respectively. The line H 1 I 1 meets CD and F G at Q and M,
respectively. The line CM meets Γ again at P . Prove that CQ “ QP .
(El Salvador)
Solution 1. We may assume that CA ą CB. Observe that H 1 and I 1 lie inside the segments
CF and CG, respectively. Therefore, M lies outside △ABC (see Figure 1).
Due to the powers of points A and B with respect to the circle Γ, we have
CH 1 ¨ CA “ AH ¨ AC “ AD 2 “ BD 2 “ BI ¨ BC “ CI 1 ¨ CB.
R C C
R
ω
H′′′′
H
H
H H′′′′
H P
P
Q
Q Q
Q
II′′′′ S S
Γ II′′′′ S
F
F G Γ
G M F
F G
G M
II
II
H
H
A D B
A D B
Figure 1 Figure 2
Then, =RSD “ =RDA “ =DF G. Hence, the quadrilateral RSGF is cyclic (see Figure 2).
Therefore, MH 1 ¨ MI 1 “ MF ¨ MG “ MR ¨ MS “ MP ¨ MC. Thus, the quadrilateral CP I 1H 1
is also cyclic. Let ω be its circumcircle.
Notice that =H 1 CQ “ =SDC “ =SRC and =QCI 1 “ =CDR “ =CSR. Hence,
△CH 1 Q „ △RCQ and △CI 1 Q „ △SCQ, and therefore QH 1 ¨ QR “ QC 2 “ QI 1 ¨ QS.
We apply the inversion with center Q and radius QC. Observe that the points R, C, and S
are mapped to H 1 , C, and I 1 , respectively. Therefore, the circumcircle Γ of △RCS is mapped
to the circumcircle ω of △H 1CI 1 . Since P and C belong to both circles and the point C is
preserved by the inversion, we have that P is also mapped to itself. We then get QP 2 “ QC 2 .
Hence, QP “ QC.
Comment 1. The problem statement still holds when Γ intersects the sides CA and CB outside
segments AF and BG, respectively.
52 IMO 2015 Thailand
C
H′′′′
H
II′′′′
X′′′′
X F
F
G
G M
M
H
H II
X A D B Y
Figure 3
Let T be the midpoint of CD, and let O be the center of Γ. Let CM meet T Y at N. To
avoid confusion, we clean some superfluous details out of the picture (see Figure 4).
Let V “ MT X CY . Since MT k Y D and DT “ T C, we get CV “ V Y . Then Ceva’s
theorem applied to △CT Y with the point M yields
T Q CV Y N TQ Y N
1“ ¨ ¨ “ ¨ .
QC V Y NT QC NT
TQ TN
Therefore, QC “N Y
. So, NQ k CY , and thus NQ K OC.
Note that the points O, N, T , and Y are collinear. Therefore, CQ K ON. So, Q is the
orthocenter of △OCN, and hence OQ K CP . Thus, Q lies on the perpendicular bisector
of CP , and therefore CQ “ QP , as required.
Q
Q P
P
O
TT M
M V
ℓ
N
Γ
D Y
Figure 4
Shortlisted problems – solutions 53
Comment 2. The second part of Solution 2 provides a proof of the following more general statement,
which does not involve a specific choice of Q on CD.
Let Y C and Y D be two tangents to a circle Γ with center O (see Figure 4). Let ℓ be the midline
of △Y CD parallel to Y D. Let Q and M be two points on CD and ℓ, respectively, such that the
line QM passes through Y . Then OQ K CM .
54 IMO 2015 Thailand
G6. Let ABC be an acute triangle with AB ą AC, and let Γ be its circumcircle. Let H,
M, and F be the orthocenter of the triangle, the midpoint of BC, and the foot of the altitude
from A, respectively. Let Q and K be the two points on Γ that satisfy =AQH “ 900 and
=QKH “ 900 . Prove that the circumcircles of the triangles KQH and KF M are tangent to
each other.
(Ukraine)
Solution 1. Let A1 be the point diametrically opposite to A on Γ. Since =AQA1 “ 900 and
=AQH “ 900 , the points Q, H, and A1 are collinear. Similarly, if Q1 denotes the point on Γ
diametrically opposite to Q, then K, H, and Q1 are collinear. Let the line AHF intersect Γ
again at E; it is known that M is the midpoint of the segment HA1 and that F is the midpoint
of HE. Let J be the midpoint of HQ1 .
Consider any point T such that T K is tangent to the circle KQH at K with Q and T
lying on different sides of KH (see Figure 1). Then =HKT “ =HQK and we are to prove
that =MKT “ =CF K. Thus it remains to show that =HQK “ =CF K ` =HKM . Due
to =HQK “ 900 ´ =Q1 HA1 and =CF K “ 900 ´ =KF A, this means the same as =Q1 HA1 “
=KF A ´ =HKM . Now, since the triangles KHE and AHQ1 are similar with F and J being
the midpoints of corresponding sides, we have =KF A “ =HJA, and analogously one may
obtain =HKM “ =JQH. Thereby our task is reduced to verifying
A
Q A
Γ
Γ
K Q
H T O
J H
Q′ J
B M F C
Q′
A′ E A′
Figure 1 Figure 2
To avoid confusion, let us draw a new picture at this moment (see Figure 2). Owing to
=Q1 HA1 “ =JQH ` =HJQ and =HJA “ =QJA ` =HJQ, we just have to show that
2 =JQH “ =QJA. To this end, it suffices to remark that AQA1 Q1 is a rectangle and that J,
being defined to be the midpoint of HQ1 , has to lie on the mid parallel of QA1 and Q1 A.
Solution 2. We define the points A1 and E and prove that the ray MH passes through Q
in the same way as in the first solution. Notice that the points A1 and E can play analogous
roles to the points Q and K, respectively: point A1 is the second intersection of the line MH
with Γ, and E is the point on Γ with the property =HEA1 “ 900 (see Figure 3).
In the circles KQH and EA1 H, the line segments HQ and HA1 are diameters, respectively;
so, these circles have a common tangent t at H, perpendicular to MH. Let R be the radical
center of the circles ABC, KQH and EA1 H. Their pairwise radical axes are the lines QK,
A1 E and the line t; they all pass through R. Let S be the midpoint of HR; by =QKH “
Shortlisted problems – solutions 55
Γ Q
C S
B M F t
A′ E R
Figure 3
=HEA1 “ 900 , the quadrilateral HERK is cyclic and its circumcenter is S; hence we have
SK “ SE “ SH. The line BC, being the perpendicular bisector of HE, passes through S.
The circle HMF also is tangent to t at H; from the power of S with respect to the circle
HMF we have
SM ¨ SF “ SH 2 “ SK 2 .
So, the power of S with respect to the circles KQH and KF M is SK 2 . Therefore, the line
segment SK is tangent to both circles at K.
56 IMO 2015 Thailand
G7. Let ABCD be a convex quadrilateral, and let P , Q, R, and S be points on the sides
AB, BC, CD, and DA, respectively. Let the line segments P R and QS meet at O. Suppose
that each of the quadrilaterals AP OS, BQOP , CROQ, and DSOR has an incircle. Prove that
the lines AC, P Q, and RS are either concurrent or parallel to each other.
(Bulgaria)
Solution 1. Denote by γA , γB , γC , and γD the incircles of the quadrilaterals AP OS, BQOP ,
CROQ, and DSOR, respectively.
We start with proving that the quadrilateral ABCD also has an incircle which will be
referred to as Ω. Denote the points of tangency as in Figure 1. It is well-known that QQ1 “ OO1
(if BC k P R, this is obvious; otherwise, one may regard the two circles involved as the incircle
and an excircle of the triangle formed by the lines OQ, P R, and BC). Similarly, OO1 “ P P1 .
Hence we have QQ1 “ P P1 . The other equalities of segment lengths marked in Figure 1 can
be proved analogously. These equalities, together with AP1 “ AS1 and similar ones, yield
AB ` CD “ AD ` BC, as required.
B Q Q1
P γB C
P1
O1
A γA γC
O
O
S1
S γD
R
D
Figure 1
Next, let us draw the lines parallel to QS through P and R, and also draw the lines parallel
to P R through Q and S. These lines form a parallelogram; denote its vertices by A1 , B 1 , C 1 ,
and D 1 as shown in Figure 2.
Since the quadrilateral AP OS has an incircle, we have AP ´ AS “ OP ´ OS “ A1 S ´ A1 P .
It is well-known that in this case there also exists a circle ωA tangent to the four rays AP ,
AS, A1 P , and A1 S. It is worth mentioning here that in case when, say, the lines AB and A1 B 1
coincide, the circle ωA is just tangent to AB at P . We introduce the circles ωB , ωC , and ωD in
a similar manner.
Assume that the radii of the circles ωA and ωC are different. Let X be the center of the
homothety having a positive scale factor and mapping ωA to ωC .
Now, Monge’s theorem applied to the circles ωA , Ω, and ωC shows that the points A, C,
and X are collinear. Applying the same theorem to the circles ωA , ωB , and ωC , we see that
the points P , Q, and X are also collinear. Similarly, the points R, S, and X are collinear, as
required.
If the radii of ωA and ωC are equal but these circles do not coincide, then the degenerate
version of the same theorem yields that the three lines AC, P Q, and RS are parallel to the
line of centers of ωA and ωC .
Finally, we need to say a few words about the case when ωA and ωC coincide (and thus they
also coincide with Ω, ωB , and ωD ). It may be regarded as the limit case in the following manner.
Shortlisted problems – solutions 57
ωC B′
B
P
P
Q
Q
A
A
C
C
O
O X
A′′′′
A C′′′′
C
ωB ωA
Ω R
R
S
D′
DD
Figure 2
Let us fix the positions of A, P , O, and S (thus we also fix the circles ωA , γA , γB , and γD ). Now
we vary the circle γC inscribed into =QOR; for each of its positions, one may reconstruct the
lines BC and CD as the external common tangents to γB , γC and γC , γD different from P R
and QS, respectively. After such variation, the circle Ω changes, so the result obtained above
may be applied.
Solution 2. Applying Menelaus’ theorem to △ABC with the line P Q and to △ACD with
the line RS, we see that the line AC meets P Q and RS at the same point (possibly at infinity)
if and only if
AP BQ CR DS
¨ ¨ ¨ “ 1. (1)
P B QC RD SA
EF ¨ F G F M2
“ .
GH ¨ HE HM 2
P J
F
G Q
II K C
K
A O
O
M
R
L
H S
E D
Figure 3 Figure 4
F
G
N
E
Figure 5
Let N be the point such that △N HG „ △M EF and such that N and M lie on different sides
of the line GH, as shown in Figure 5. Then =GN H ` =HM G “ =F M E ` =HM G “ 1800 . So,
Shortlisted problems – solutions 59
R′
P ′′′′ =
P =PP
Q
Q
B′′′′
B
C′′′
C
A′′′′ =
A
A =
A =AA =AA O′′′′ =
O =OO
D′′′′
D
S′′′′ =
S =SS
Q′
Figure 6
Denote by h the homothety centered at O that maps the incircle of CROQ to the incircle
of AP OS. Let Q1 “ hpQq, C 1 “ hpCq, R1 “ hpRq, O 1 “ O, S 1 “ S, A1 “ A, and P 1 “ P .
Furthermore, define B 1 “ A1 P 1 X C 1 Q1 and D 1 “ A1 S 1 X C 1 R1 as shown in Figure 6. Then
AP ¨ OS A1 P 1 ¨ O 1S 1
“ 1 1 1 1
P O ¨ SA P O ¨S A
holds trivially. We also have
CR ¨ OQ C 1 R1 ¨ O 1 Q1
“ 1 1
RO ¨ QC R O ¨ Q1 C 1
by the similarity of the quadrilaterals CROQ and C 1 R1 O 1 Q1 .
60 IMO 2015 Thailand
Next, consider the circumscribed quadrilaterals BQOP and B 1 Q1 O 1P 1 whose incenters lie
on different sides of the quadrilaterals’ shared side line OP “ O 1 P 1. Observe that BQ k B 1 Q1
and that B 1 and Q1 lie on the lines BP and QO, respectively. It is now easy to see that the
two quadrilaterals satisfy the hypotheses of Lemma 2. Thus, we deduce
BQ ¨ OP B 1 Q1 ¨ O 1 P 1
“ 1 1 .
QO ¨ P B Q O ¨ P 1B 1
Similarly, we get
DS ¨ OR D 1 S 1 ¨ O 1 R1
“ 1 1 .
SO ¨ RD S O ¨ R1 D 1
Multiplying these four equations, we obtain
AP BQ CR DS A1 P 1 B 1 Q1 C 1 R1 D 1 S 1
¨ ¨ ¨ “ 1 1 ¨ 1 1 ¨ 1 1 ¨ 1 1. (3)
P B QC RD SA P B QC RD SA
A1 P 1 B 1 Q1 C 1 R1 D 1 S 1
¨ ¨ ¨ “ 1.
P 1 B 1 Q1 C 1 R1 D 1 S 1 A1
E
Z B′
D X
D X Y
C H
C
A′ C′
A
B B A T Z
Y
Figure 1 Figure 2 Figure 3
Lemma 2. Every triangle in a Thaiangulation T of Π contains a side of Π.
Proof. Let ABC be a triangle in T . Apply an affine transform such that ABC maps to an
equilateral triangle; let A1 B 1 C 1 be the image of this triangle, and Π1 be the image of Π. Clearly,
T maps into a Thaiangulation T 1 of Π1 .
Assume that none of the sides of △A1 B 1 C 1 is a side of Π1 . Then T 1 contains some other
triangles with these sides, say, A1 B 1 Z, C 1 A1 Y , and B 1 C 1 X; notice that A1 ZB 1 XC 1 Y is a convex
hexagon (see Figure 3). The sum of its external angles at X, Y , and Z is less than 3600 . So one
of these angles (say, at Z) is less than 1200 , hence =A1 ZB 1 ą 600 . Then Z lies on a circular arc
subtended by A1 B 1 and ? having angular measure less than 2400 ; consequently, the altitude ZH
1 1
of △A B Z is less than 3 A1 B 1 {2. Thus rA1 B 1 Zs ă rA1 B 1 C 1 s, and T 1 is not a Thaiangulation.
A contradiction. l
62 IMO 2015 Thailand
A A X
Y′
Y
YY
A
C
D D
B C B C X
X Z
Z
Z′
T1 T2 X ′
B
Figure 4 Figure 5
so T is not a Thaiangulation.
Shortlisted problems – solutions 63
Solution 2. We will make use of the preliminary observations from Solution 1, together with
Lemma 1.
Arguing indirectly, we choose a convex polygon Π with the least possible number of sides
such that some two Thaiangulations T1 and T2 of Π violate the statement (thus Π has at least
five sides). Assume that T1 and T2 share a diagonal d splitting Π into two smaller polygons Π1
and Π2 . Since the problem statement holds for any of them, the induced Thaiangulations of
each of Πi differ by two triangles forming a parallelogram (the Thaiangulations induced on Πi
by T1 and T2 may not coincide, otherwise T1 and T2 would differ by at most two triangles). But
both these parallelograms are contained in T1 ; this contradicts Lemma 1. Therefore, T1 and T2
share no diagonal. Hence they also share no triangle.
We consider two cases.
Case 1. Assume that some vertex B of Π is an endpoint of some diagonal in T1 , as well as an
endpoint of some diagonal in T2 .
Let A and C be the vertices of Π adjacent to B. Then T1 contains some triangles ABX
and BCY , while T2 contains some triangles ABX 1 and BCY 1 . Here, some of the points X,
X 1 , Y , and Y 1 may coincide; however, in view of our assumption together with the fact that T1
and T2 share no triangle, all four triangles ABX, BCY , ABX 1 , and BCY 1 are distinct.
Since rABXs “ rBCY s “ rABX 1 s “ rBCY 1 s, we have XX 1 k AB and Y Y 1 k BC. Now,
if X “ Y , then X 1 and Y 1 lie on different lines passing through X and are distinct from that
point, so that X 1 ‰ Y 1 . In this case, we may switch the two Thaiangulations. So, hereafter we
assume that X ‰ Y .
In the convex pentagon ABCY X we have either =BAX ` =AXY ą 1800 or =XY C `
=Y CB ą 1800 (or both); due to the symmetry, we may assume that the first inequality holds.
ÝÝÑ
Let r be the ray emerging from X and co-directed with AB; our inequality shows that r points
to the interior of the pentagon (and thus to the interior of Π). Therefore, the ray opposite to r
points outside Π, so X 1 lies on r; moreover, X 1 lies on the “arc” CY of Π not containing X.
So the segments XX 1 and Y B intersect (see Figure 6).
Let O be the intersection point of the rays r and BC. Since the triangles ABX 1 and BCY 1
have no common interior points, Y 1 must lie on the “arc” CX 1 which is situated inside the
triangle XBO. Therefore, the line Y Y 1 meets two sides of △XBO, none of which may be XB
(otherwise the diagonals XB and Y Y 1 would share a common point). Thus Y Y 1 intersects BO,
which contradicts Y Y 1 k BC.
X
Y
X′
r
YY ′′′ O
A
C
B
Figure 6
Case 2. In the remaining case, each vertex of Π is an endpoint of a diagonal in at most one
of T1 and T2 . On the other hand, a triangulation cannot contain two consecutive vertices with
no diagonals from each. Therefore, the vertices of Π alternatingly emerge diagonals in T1 and
in T2 . In particular, Π has an even number of sides.
64 IMO 2015 Thailand
Next, we may choose five consecutive vertices A, B, C, D, and E of Π in such a way that
In order to do this, it suffices to choose three consecutive vertices B, C, and D of Π such that
the sum of their external angles is at most 1800 . This is possible, since Π has at least six sides.
E
A
B X Y D
C
Figure 7
We may assume that T1 has no diagonals from B and D (and thus contains the trian-
gles ABC and CDE), while T2 has no diagonals from A, C, and E (and thus contains the
triangle BCD). Now, since rABCs “ rBCDs “ rCDEs, we have AD k BC and BE k CD
(see Figure 7). By (2) this yields that AD ą BC and BE ą CD. Let X “ AC X BD and
Y “ CE X BD; then the inequalities above imply that AX ą CX and EY ą CY .
Finally, T2 must also contain some triangle BDZ with Z ‰ C; then the ray CZ lies in
the angle ACE. Since rBCDs “ rBDZs, the diagonal BD bisects CZ. Together with the
inequalities above, this yields that Z lies inside the triangle ACE (but Z is distinct from A
and E), which is impossible. The final contradiction.
Comment 2. Case 2 may also be accomplished with the use of Lemma 2. Indeed, since each
triangulation of an n-gon contains n ´ 2 triangles neither of which may contain three sides of Π,
Lemma 2 yields that each Thaiangulation contains exactly two ears. But each vertex of Π is a vertex
of an ear either in T1 or in T2 , so Π cannot have more than four vertices.
Shortlisted problems – solutions 65
Number Theory
N1. Determine all positive integers M for which the sequence a0 , a1 , a2 , . . ., defined by
2M `1
a0 “ 2
and ak`1 “ ak tak u for k “ 0, 1, 2, . . ., contains at least one integer term.
(Luxembourg)
Hence
bk pbk ´ 1q pbk ´ 3qpbk ` 2q
bk`1 ´ 3 “ ´3“ (2)
2 2
for all k ě 0.
Suppose that b0 ´ 3 ą 0. Then equation (2) yields bk ´ 3 ą 0 for all k ě 0. For each k ě 0,
define ck to be the highest power of 2 that divides bk ´ 3. Since bk ´ 3 is even for all k ě 0, the
number ck is positive for every k ě 0.
Note that bk ` 2 is an odd integer. Therefore, from equation (2), we have that ck`1 “ ck ´ 1.
Thus, the sequence c0 , c1 , c2 , . . . of positive integers is strictly decreasing, a contradiction. So,
b0 ´ 3 ď 0, which implies M “ 1.
For M “ 1, we can check that the sequence is constant with ak “ 23 for all k ě 0. Therefore,
the answer is M ě 2.
Solution 2. We provide an alternative way to show M “ 1 once equation (1) has been
reached. We claim that bk ” 3 pmod 2m q for all k ě 0 and m ě 1. If this is true, then we
would have bk “ 3 for all k ě 0 and hence M “ 1.
To establish our claim, we proceed by induction on m. The base case bk ” 3 pmod 2q is
true for all k ě 0 since bk is odd. Now suppose that bk ” 3 pmod 2m q for all k ě 0. Hence
bk “ 2m dk ` 3 for some integer dk . We have
Comment. The reason the number 3 which appears in both solutions is important, is that it is a
nontrivial fixed point of the recurrence relation for bk .
66 IMO 2015 Thailand
N2. Let a and b be positive integers such that a!b! is a multiple of a! ` b!. Prove that
3a ě 2b ` 2.
(United Kingdom)
Solution 1. If a ą b, we immediately get 3a ě 2b ` 2. In the case a “ b, the required
inequality is equivalent to a ě 2, which can be checked easily since pa, bq “ p1, 1q does not
satisfy a! ` b! | a!b!. We now assume a ă b and denote c “ b ´ a. The required inequality
becomes a ě 2c ` 2.
b!
Suppose, to the contrary, that a ď 2c ` 1. Define M “ a! “ pa ` 1qpa ` 2q ¨ ¨ ¨ pa ` cq. Since
a! ` b! | a!b! implies 1 ` M | a!M, we obtain 1 ` M | a!. Note that we must have c ă a; otherwise
1 ` M ą a!, which is impossible. We observe that c! | M since M is a product of c consecutive
integers. Thus gcdp1 ` M, c!q “ 1, which implies
ˇ
ˇ a!
1 ` M ˇˇ “ pc ` 1qpc ` 2q ¨ ¨ ¨ a. (1)
c!
If a ď 2c, then a!
c!
is a product of a ´ c ď c integers not exceeding a whereas M is a product of
c integers exceeding a. Therefore, 1 ` M ą a! c!
, which is a contradiction.
It remains to exclude the case a “ 2c ` 1. Since a ` 1 “ 2pc ` 1q, we have c ` 1 | M. Hence,
we can deduce from (1) that 1 ` M | pc ` 2qpc ` 3q ¨ ¨ ¨ a. Now pc ` 2qpc ` 3q ¨ ¨ ¨ a is a product
of a ´ c ´ 1 “ c integers not exceeding a; thus it is smaller than 1 ` M. Again, we arrive at a
contradiction.
Comment 1. One may derive X a \ a weakerX a \version of (1) and finish the problem as follows. After
assuming a ď 2c ` 1, we have 2 ď c, so 2 ! | M . Therefore,
ˇ ´Y ]
ˇ a ¯ ´Y a ] ¯
1 ` M ˇˇ `1 ` 2 ¨ ¨ ¨ a.
2 2
Observe that a2 ` 1
`X \ ˘ `X a \ ˘ PaT
2 ` 2 ¨ ¨P¨ aT is a product of 2 integers not exceeding a. This leads to a
contradiction when a is even since a2 “ a2 ď c and M is` a product ˘ ` a`5 ˘of c integersX aexceeding a. ˇ
a`3 a`1 ˇ
\
When a is odd, we can further deduce that 1 ` M | 2 2 ¨ ¨ ¨ a since 2 ` 1 “ 2 a ` 1.
Now a`3
` ˘ ` a`5 ˘ a´1
2 2 ¨ ¨ ¨ a is a product of 2 ď c numbers not exceeding a, and we get a contradiction.
Comment 2. The original problem statement also asks to determine when the equality 3a “ 2b ` 2
holds. It can be checked that the answer is pa, bq “ p2, 2q, p4, 5q.
Shortlisted problems – solutions 67
N3. Let m and n be positive integers such that m ą n. Define xk “ pm ` kq{pn ` kq for k “
1, 2, . . . , n ` 1. Prove that if all the numbers x1 , x2 , . . . , xn`1 are integers, then x1 x2 ¨ ¨ ¨ xn`1 ´ 1
is divisible by an odd prime.
(Austria)
Solution. Assume that x1 , x2 , . . . , xn`1 are integers. Define the integers
m`k m´n
ak “ xk ´ 1 “ ´1“ ą0
n`k n`k
for k “ 1, 2, . . . , n ` 1.
Let P “ x1 x2 ¨ ¨ ¨ xn`1 ´ 1. We need to prove that P is divisible by an odd prime, or in
other words, that P is not a power of 2. To this end, we investigate the powers of 2 dividing
the numbers ak .
Let 2d be the largest power of 2 dividing m ´ n, and let 2c be the largest power of 2 not
exceeding 2n ` 1. Then 2n ` 1 ď 2c`1 ´ 1, and so n ` 1 ď 2c . We conclude that 2c is one of the
numbers n ` 1, n ` 2, . . . , 2n ` 1, and that it is the only multiple of 2c appearing among these
numbers. Let ℓ be such that n ` ℓ “ 2c . Since m´n n`ℓ
is an integer, we have d ě c. Therefore,
d´c`1 m´n d´c`1
2 ∤ aℓ “ n`ℓ , while 2 | ak for all k P t1, . . . , n ` 1u z tℓu.
d´c`1
Computing modulo 2 , we get
Therefore, 2d´c`1 ∤ P .
On the other hand, for any k P t1, . . . , n ` 1u z tℓu, we have 2d´c`1 | ak . So P ě ak ě 2d´c`1 ,
and it follows that P is not a power of 2.
Comment. Instead of attempting to show that P is not a power of 2, one may try to find an odd
factor of P (greater than 1) as follows:
From ak “ m´n n`k P Zą0 , we get that m ´ n is divisible by n ` 1, n ` 2, . . . , 2n ` 1, and thus
it is also divisible by their least common multiple L. So m ´ n “ qL for some positive integer q;
L
hence xk “ q ¨ n`k ` 1.
L
Then, since n ` 1 ď 2c “ n ` ℓ ď 2n ` 1 ď 2c`1 ´ 1, we have 2c | L, but 2c`1 ∤ L. So n`ℓ is odd,
L
while n`k is even for k ‰ ℓ. Computing modulo 2q yields
N4. Suppose that a0 , a1 , . . . and b0 , b1 , . . . are two sequences of positive integers satisfying
a0 , b0 ě 2 and
an`1 “ gcdpan , bn q ` 1, bn`1 “ lcmpan , bn q ´ 1
for all n ě 0. Prove that the sequence (an ) is eventually periodic; in other words, there exist
integers N ě 0 and t ą 0 such that an`t “ an for all n ě N.
(France)
Solution 1. Let sn “ an ` bn . Notice that if an | bn , then an`1 “ an ` 1, bn`1 “ bn ´ 1 and
sn`1 “ sn . So, an increases by 1 and sn does not change until the first index is reached with
an ∤ sn . Define
(
Wn “ m P Zą0 : m ě an and m ∤ sn and wn “ min Wn .
Let g “ gN . We have proved that the sequence pan q eventually repeats the following cycle:
g ` 1 ÞÑ g ` 2 ÞÑ . . . ÞÑ w ÞÑ g ` 1.
Hence, the pair pan , rn q uniquely determines the pair pan`1 , rn`1 q. Since there are finitely many
possible pairs, the sequence of pairs pan , rn q is eventually periodic; in particular, the sequence
pan q is eventually periodic.
Comment. We show that there are only four possibilities for g and w (as defined in Solution 1),
namely (
pw, gq P p2, 1q, p3, 1q, p4, 2q, p5, 1q . (2)
This means that the sequence pan q eventually repeats one of the following cycles:
Using the notation of Solution 1, for n ě N the sequence pan q has a cycle pg ` 1, g ` 2, . . . , wq
such that g “ gcdpw, sn q. By the observations in the proof of Claim 2, the numbers g ` 1, . . . , w ´ 1 all
divide sn ; so the number L “ lcmpg ` 1, g ` 2, . . . , w ´ 1q also divides sn . Moreover, g also divides w.
Now choose any n ě N such that an “ w. By (1), we have
wpsn ´ wq w w2 ´ g 2
sn`1 “ g ` “ sn ¨ ´ .
g g g
2 2
Since L divides both sn and sn`1 , it also divides the number T “ w ´g
g .
Suppose first that w ě 6, which yields g ` 1 ď w2 ` 1 ď w ´ 2. Then pw ´ 2qpw ´ 1q | L | T , so we
have either w2 ´ g2 ě 2pw ´ 1qpw ´ 2q, or g “ 1 and w2 ´ g2 “ pw ´ 1qpw ´ 2q. In the former case we
get pw ´ 1qpw ´ 5q ` pg 2 ´ 1q ď 0 which is false by our assumption. The latter equation rewrites as
3w “ 3, so w “ 1, which is also impossible.
Now we are left with the cases when w ď 5 and g | w. The case pw, gq “ p4, 1q violates the
2 2
condition L | w ´g
g ; all other such pairs are listed in (2).
In the table below, for each pair pw, gq, we provide possible sequences pan q and pbn q. That shows
that the cycles shown in (3) are indeed possible.
w “2 g “1 an “ 2 bn “ 2 ¨ 2n ` 1
w “3 g “1 pa2k , a2k`1 q “ p2, 3q pb2k , b2k`1 q “ p6 ¨ 3k ` 2, 6 ¨ 3k ` 1q
w “4 g “2 pa2k , a2k`1 q “ p3, 4q pb2k , b2k`1 q “ p12 ¨ 2k ` 3, 12 ¨ 2k ` 2q
w “5 g “1 pa4k , . . . , a4k`3 q “ p2, 3, 4, 5q pb4k , . . . , b4k`3 q “ p6 ¨ 5k ` 4, . . . , 6 ¨ 5k ` 1q
70 IMO 2015 Thailand
N5. Determine all triples pa, b, cq of positive integers for which ab ´ c, bc ´ a, and ca ´ b are
powers of 2.
Explanation: A power of 2 is an integer of the form 2n , where n denotes some nonnegative
integer.
(Serbia)
Answer. There are sixteen such triples, namely p2, 2, 2q, the three permutations of p2, 2, 3q,
and the six permutations of each of p2, 6, 11q and p3, 5, 7q.
Solution 1. It can easily be verified that these sixteen triples are as required. Now let pa, b, cq
be any triple with the desired property. If we would have a “ 1, then both b ´ c and c ´ b were
powers of 2, which is impossible since their sum is zero; because of symmetry, this argument
shows a, b, c ě 2.
Case 1. Among a, b, and c there are at least two equal numbers.
Without loss of generality we may suppose that a “ b. Then a2 ´ c and apc ´ 1q are powers
of 2. The latter tells us that actually a and c ´ 1 are powers of 2. So there are nonnegative
integers α and γ with a “ 2α and c “ 2γ ` 1. Since a2 ´ c “ 22α ´ 2γ ´ 1 is a power of 2 and
thus incongruent to ´1 modulo 4, we must have γ ď 1. Moreover, each of the terms 22α ´ 2
and 22α ´ 3 can only be a power of 2 if α “ 1. It follows that the triple pa, b, cq is either p2, 2, 2q
or p2, 2, 3q.
Case 2. The numbers a, b, and c are distinct.
Due to symmetry we may suppose that
2 ď a ă b ă c. (1)
We are to prove that the triple pa, b, cq is either p2, 6, 11q or p3, 5, 7q. By our hypothesis, there
exist three nonnegative integers α, β, and γ such that
bc ´ a “ 2α , (2)
ac ´ b “ 2β , (3)
and ab ´ c “ 2γ . (4)
Evidently we have
αąβ ąγ. (5)
Depending on how large a is, we divide the argument into two further cases.
Case 2.1. a “ 2.
We first prove that γ “ 0. Assume for the sake of contradiction that γ ą 0. Then c is even
by (4) and, similarly, b is even by (5) and (3). So the left-hand side of (2) is congruent to 2
modulo 4, which is only possible if bc “ 4. As this contradicts (1), we have thereby shown that
γ “ 0, i.e., that c “ 2b ´ 1.
Now (3) yields 3b ´ 2 “ 2β . Due to b ą 2 this is only possible if β ě 4. If β “ 4, then we
get b “ 6 and c “ 2 ¨ 6 ´ 1 “ 11, which is a solution. It remains to deal with the case β ě 5.
Now (2) implies
and by β ě 5 the right-hand side is not divisible by 32. Thus α ď 4 and we get a contradiction
to (5).
Shortlisted problems – solutions 71
Case 2.2. a ě 3.
Pick an integer ϑ P t´1, `1u such that c ´ ϑ is not divisible by 4. Now
is divisible by 2β and, consequently, b`aϑ is divisible by 2β´1 . On the other hand, 2β “ ac´b ą
pa ´ 1qc ě 2c implies in view of (1) that a and b are smaller than 2β´1 . All this is only possible
if ϑ “ 1 and a ` b “ 2β´1 . Now (3) yields
ac ´ b “ 2pa ` bq , (6)
Since a is odd, it is not possible that both factors b`a and b´a are divisible by 4. Consequently,
one of them has to be a multiple of 2β´1 . Hence one of the numbers 2pb ` aq and 2pb ´ aq is
divisible by 2β and in either case we have
ac ´ b “ 2β ď 2pa ` bq . (7)
This in turn yields pa ´ 1qb ă ac ´ b ă 4b and thus a “ 3 (recall that a is odd and larger
than 1). Substituting this back into (7) we learn c ď b ` 2. But due to the parity b ă c entails
that b ` 2 ď c holds as well. So we get c “ b ` 2 and from bc ´ a “ pb ´ 1qpb ` 3q being a power
of 2 it follows that b “ 5 and c “ 7.
Case 3. Among a, b, and c both parities occur.
Without loss of generality, we suppose that c is odd and that a ď b. We are to show that
pa, b, cq is either p2, 2, 3q or p2, 6, 11q. As at least one of a and b is even, the expression ab ´ c
is odd; since it is also a power of 2, we obtain
ab ´ c “ 1 . (8)
We may suppose a ă b from now on. As usual, we let α ą β denote the integers satisfying
2α “ bc ´ a and 2β “ ac ´ b . (9)
2α “ ab2 ´ pa ` bq , (10)
and 2β “ a2 b ´ pa ` bq . (11)
The addition of both equation yields 2α ` 2β “ pab ´ 2qpa ` bq. Now ab ´ 2 is even but not
divisible by 4, so the highest power of 2 dividing a ` b is 2β´1 . For this reason, the equations
(10) and (11) show that the highest powers of 2 dividing either of the numbers ab2 and a2 b is
likewise 2β´1 . Thus there is an integer τ ě 1 together with odd integers A, B, and C such that
a “ 2τ A, b “ 2τ B, a ` b “ 23τ C, and β “ 1 ` 3τ .
Notice that A ` B “ 22τ C ě 4C. Moreover, (11) entails A2 B ´ C “ 2. Thus 8 “
4A B ´ 4C ě 4A2 B ´ A ´ B ě A2 p3B ´ 1q. Since A and B are odd with A ă B, this is only
2
Comment. In both solutions, there are many alternative ways to proceed in each of its cases. Here
we present a different treatment of the part “a ă b” of Case 3 in Solution 2, assuming that (8) and (9)
have already been written down:
Put d “ gcdpa, bq and define the integers p and q by a “ dp and b “ dq; notice that p ă q and
gcdpp, qq “ 1. Now (8) implies c “ d2 pq ´ 1 and thus we have
2α “ dpd2 pq 2 ´ p ´ qq
and 2β “ dpd2 p2 q ´ p ´ qq . (12)
N6. Let Zą0 denote the set of positive integers. Consider a function f : Zą0 Ñ Zą0 . For
any m, n P Zą0 we write f n pmq “ looomooon
f pf p. . . f pmq . . .qq. Suppose that f has the following two
n
properties:
f n pmq ´ m
piq If m, n P Zą0 , then P Zą0 ;
n
piiq The set Zą0 z tf pnq | n P Zą0 u is finite.
Step 1. We commence by checking that f is injective. For this purpose, we consider any
m, k P Zą0 with f pmq “ f pkq. By piq, every positive integer n has the property that
Step 2. Our next goal is to prove that each row of the Table is an arithmetic progression.
Assume contrariwise that the number t of rows which are arithmetic progressions would satisfy
0 ď t ă k. By permuting the rows if necessary we may suppose that precisely the first t rows
are arithmetic progressions, say with steps T1 , . . . , Tt . Our plan is to find a further row that
is “not too sparse” in an asymptotic sense, and then to prove that such a row has to be an
arithmetic progression as well.
Let us write T “ lcmpT1 , T2 , . . . , Tt q and A “ maxta1 , a2 , . . . , at u if t ą 0; and T “ 1 and
A “ 0 if t “ 0. For every integer n ě A, the interval ∆n “ rn ` 1, n ` T s contains exactly T {Ti
74 IMO 2015 Thailand
elements of the ith row (1 ď i ď t). Therefore, the number of elements from the last pk ´ tq
rows of the Table contained in ∆n does not depend on n ě A. It is not possible that none
of these intervals ∆n contains an element from the k ´ t last rows, because infinitely many
numbers appear in these rows. It follows that for each n ě A the interval ∆n contains at least
one member from these rows. “
This yields that for every positive integer d, the interval A ` 1, A ` pd ` 1qpk ´ tqT s contains
at least pd ` 1qpk ´ tq elements from the last k ´ t rows; therefore, there exists an index x with
t ` 1 ď x ď k, possibly depending on d, such that our interval contains at least d ` 1 elements
from the xth row. In this situation we have
Finally, since there are finitely many possibilities for x, there exists an index x ě t ` 1 such
that the set
X “ d P Zą0 ˇ f d pax q ď A ` pd ` 1qpk ´ tqT
ˇ (
f d pax q ´ ax
βd “
d
is a positive integer not exceeding
A ` pd ` 1qpk ´ tqT Ad ` 2dpk ´ tqT
ď “ A ` 2pk ´ tqT .
d d
This leaves us with finitely many choices for βd , which means that there exists a number Tx
such that the set ˇ (
Y “ d P X ˇ βd “ Tx
is infinite. Notice that we have f d pax q “ ax ` d ¨ Tx for all d P Y .
Now we are prepared to prove that the numbers in the xth row form an arithmetic progres-
sion, thus coming to a contradiction with our assumption. Let us fix any ˇ jpositive integer j.
ˇ
Since the set Y is infinite, we can choose a number y P Y such that y ´ j ą ˇf pax q ´ pax ` jTx qˇ.
Notice that both numbers
f y pax q ´ f j pax q “ f y´j f j pax q ´ f j pax q and f y pax q ´ pax ` jTx q “ py ´ jqTx
` ˘
are divisible by y ´ j. Thus, the difference between these numbers is also divisible by y ´ j.
Since the absolute value of this difference is less than y ´ j, it has to vanish, so we get f j pax q “
ax ` j ¨ Tx .
Hence, it is indeed true that all rows of the Table are arithmetic progressions.
Step 3. Keeping the above notation in force, we denote the step of the ith row of the table by Ti .
Now we claim that we have f pnq ´ n “ f pn ` T q ´ pn ` T q for all n P Zą0 , where
T “ lcmpT1 , . . . , Tk q .
To see this, let any n P Zą0 be given and denote the index of the row in which it appears in
the Table by i. Then we have f j pnq “ n ` j ¨ Ti for all j P Zą0 , and thus indeed
Comment 1. There are some alternative ways to complete the second part once the index x
corresponding to a “dense row” is found. For instance, one may show that for some integer Tx˚ the set
is infinite, and then one may conclude with a similar divisibility argument.
Comment 2. It may be checked that, conversely, any way to fill out the Table with finitely many
arithmetic progressions so that each positive integer appears exactly once, gives rise to a function f
satisfying the two conditions mentioned in the problem. For example, we may arrange the positive
integers as follows:
2 4 6 8 10 ...
1 5 9 13 17 ...
3 7 11 15 19 ...
As this example shows, it is not true that the function n ÞÑ f pnq ´ n has to be constant.
76 IMO 2015 Thailand
Answer. k ě 2.
` ˘
Solution 1. For any function f : Zą0 Ñ Zą0 , let Gf pm, nq “ gcd f pmq ` n, f pnq ` m . Note
that a k-good function is also pk ` 1q-good for any positive integer k. Hence, it suffices to show
that there does not exist a 1-good function and that there exists a 2-good function.
We first show that there is no 1-good function. Suppose that there exists a function f such
that Gf pm, nq “ 1 for all m ‰ n. Now, if there are two distinct even numbers m and n such
that f pmq and f pnq are both even, then 2 | Gf pm, nq, a contradiction. A similar argument
holds if there are two distinct odd numbers m and n such that f pmq and f pnq are both odd.
Hence we can choose an even m and an odd n such that f pmq is odd and f pnq is even. This
also implies that 2 | Gf pm, nq, a contradiction.
We now construct a 2-good function. Define f pnq “ 2gpnq`1 ´ n ´ 1, where g is defined
recursively by gp1q “ 1 and gpn ` 1q “ p2gpnq`1 q!.
For any positive integers m ą n, set
We need to show that gcdpA, Bq ď 2. First, note that A ` B “ 2gpmq`1 ` 2gpnq`1 ´ 2 is not
divisible by 4, so that 4 ∤ gcdpA, Bq. Now we suppose that there is an odd prime p for which
p | gcdpA, Bq and derive a contradiction.
We first claim that 2gpm´1q`1 ě B. This is a rather weak bound; one way to prove it is as fol-
lows. Observe that gpk `1q ą gpkq and hence 2gpk`1q`1 ě 2gpkq`1 `1 for every positive integer k.
By repeatedly applying this inequality, we obtain 2gpm´1q`1 ě 2gpnq`1 ` pm ´ 1q ´ n “ B.
Now, since p | B, we have p ´ 1 ă B ď 2gpm´1q`1 , so that p ´ 1 | p2gpm´1q`1 q! “ gpmq.
Hence 2gpmq ” 1 pmod pq, which yields A ` B ” 2gpnq`1 pmod pq. However, since p | A ` B,
this implies that p “ 2, a contradiction.
(i ) The values of f pnq have already been defined for all n ď m, and p-useful numbers ap have
already been defined for all p ď m ` 2;
Comment 1. For any p P P, we may also define ap at step m for an arbitrary m ď p ´ 2. The
construction will work as long as we define a finite number of ap at each step.
Comment 2. When attempting to construct a 2-good function f recursively, the following way
seems natural. Start with setting f p1q “ 1. Next, for each integer m ą 1, introduce the set Xm like
in Solution 2 and define f pmq so as to satisfy
This construction
` might seem˘ to work. Indeed, consider a fixed p P P, and suppose that p
divides gcd f pnq ` m, f pmq ` n for some n ă m. Choose such m and n so that maxpm, nq is
minimal. ` Then p P Xm . We can check ˘ that p ă m, so that the construction implies that p di-
vides gcd f pnq ` pm ´ pq, f pm ´ pq ` n . Since maxpn, m ´ pq ă maxpm, nq, this almost leads to a
contradiction—the only trouble is the possibility that n “ m ´ p. However, this flaw may happen to
be not so easy to fix.
We will present one possible way to repair this argument in the next comment.
Comment 3. There are many recursive constructions for a 2-good function f . Here we sketch one
general approach which may be specified in different ways. For convenience, we denote by Zp the set
of residues modulo p; all operations on elements of Zp are also performed modulo p.
The general structure is the same as in Solution 2, i.e. using the Chinese Remainder Theorem to
successively determine f pmq. But instead of designating a common “safe” residue ap for future steps,
we act as follows.
p1q p2q ppq
For every p P P, in some step of the process we define p subsets Bp , Bp , . . . , Bp Ă Zp . The
meaning of these sets is that
f pmq ` m should be congruent to some element in Bppiq whenever m ” i pmod pq for i P Zp . (1)
piq piq
Moreover, in every such subset we specify a safe element bp P Bp . The meaning now is that in
piq
future steps, it is safe to set f pmq ` m ” bp pmod pq whenever m ” i pmod pq. In view of (1), this
` piq
safety will follow from the condition that p ∤ gcd bp ` pj ´ iq, cpjq ´ pj ´ iq for all j P Zp and all
˘
pjq
cpjq P Bp . In turn, this condition can be rewritten as
´ bpiq pjq
p R Bp , where j ” i ´ bpiq
p pmod pq. (2)
78 IMO 2015 Thailand
piq piq
The construction in Solution 2 is equivalent to setting bp “ ´ap and Bp “ Zp z tap u for all i.
However, there are different, more technical specifications of our approach.
piq piq
One may view the (incomplete) construction in Comment 2 as defining Bp and bp at step p ´ 1
p0q p0q ( piq piq (
by setting Bp “ bp “ t0u and Bp “ bp “ tf piq ` i mod pu for every i “ 1, 2, . . . , p ´ 1.
However, this construction violates (2) as soon as some number of the form f piq ` i is divisible by
piq piq piq
some p with i ` 2 ď p P P, since then ´bp “ bp P Bp .
piq
Here is one possible way to repair this construction. For all p P P, we define the sets Bp and the
piq p1q p1q ( p´1q p0q p´1q ( p0q (
elements bp at step pp ´ 2q as follows. Set Bp “ bp “ t2u and Bp “ B p “ bp “ bp “
piq piq
t´1u. Next, for all i “ 2, . . . , p ´ 2, define Bp “ ti, f piq ` i mod pu and bp “ i. One may see that
these definitions agree with both (1) and (2).
Shortlisted problems – solutions 79
śk
N8. For every positive integer n with prime factorization n “ i“1 pαi i , define
ÿ
℧pnq “ αi .
i : pi ą10100
That is, ℧pnq is the number of prime factors of n greater than 10100 , counted with multiplicity.
Find all strictly increasing functions f : Z Ñ Z such that
` ˘
℧ f paq ´ f pbq ď ℧pa ´ bq for all integers a and b with a ą b. (1)
(Brazil)
which is impossible. l
Now we complete the induction step. By Claim 1, for every integer a each of the sequences
forms a complete residue system modulo k0 . This yields f paq ” f pa ` k0 q pmod k0 q. Thus,
f paq ” f pbq pmod k0 q whenever a ” b pmod k0 q.
Finally, if a ı b pmod k0 q then there exists an integer b1 such that b1 ” b pmod k0 q and
|a ´ b1 | ă k0 . Then f pbq ” f pb1 q ı f paq pmod k0 q. The induction step is proved.
Step 2. We prove that for some small integer a there exist infinitely many integers n such that
f pnq “ an. In other words, f is linear on some infinite set.
We start with the following general statement.
80 IMO 2015 Thailand
Claim 2. There exists a constant c such that f ptq ă ct for every positive integer t ą N.
Proof. Let d be the product of all small primes, and let α be a positive integer such that
2α ą f pNq. Then, for every p P S the numbers f p0q, f p1q, . . . , f pNq are distinct modulo pα .
Set P “ dα and c “ P ` f pNq.
Choose any integer t ą N. Due to the choice of α, for every p P S there exists at most one
nonnegative integer i ď N with pα | f ptq ´ f piq. Since |S| ă N, we` can choose˘ a nonnegative
integer j ď N such that pα ∤ f ptq ´ f pjq for `all p P S. Therefore,
˘ S f ptq ´ f pjq ă P .
On the other hand, Step 1 shows that L f ptq ´ f pjq “ Lpt ´ jq ď t ´ j. Since 0 ď j ď N,
this yields
` ˘ ` ˘ ` ˘
f ptq “ f pjq ` L f ptq ´ f pjq ¨ S f ptq ´ f pjq ă f pNq ` pt ´ jqP ď P ` f pNq t “ ct. l
` ˘
Now let T be the set of large primes. For every t P T , Step 1 implies L f ptq “ t, so the
ratio f ptq{t is an integer. Now Claim 2 leaves us with only finitely many choices for this ratio,
which means that there exists an infinite subset T 1 Ď T and a positive integer a such that
f ptq “ at for all t P T 1 , as required.
Since Lptq “ L f ptq “ LpaqLptq for all t P T 1 , we get Lpaq “ 1, so the number a is small.
` ˘
Comment 2. Step 2 is the main step of the solution. We sketch several different approaches allowing
to perform this step using statements which are weaker than Claim 2.
Approach 1. Let us again denote the ` product of all
˘ smalli primes by d. We focus on the values f pdi q,
i ě 0. In view of Step 1, we have L f pd q ´ f pd q “ Lpd ´ d q “ di´k ´ 1 for all i ą k ě 0.
i k k
Acting similarly to the beginning of the proof of Claim 2, one may choose a number α ě 0 such
that the residues of the numbers f pdi q, i “ 0, 1, . . . , N , are distinct `modulo pα for˘each p P S. Then,
for every i ą N , there exists an exponent k “ kpiq ď N such that S f pdi q ´ f pdk q ă P “ dα .
` Since there ˘are only finitely many options for kpiq, as well as for the corresponding numbers
S f pdi q ´ f pdk q , there exists an infinite set I of `exponents i ą N ˘ such that kpiq attains the same
value k0 for all i P I, and such that, moreover, S f pdi q ´ f pdk0 q attains the same value s0 for all
i P I. Therefore, for all such i we have
which means that f is linear on the infinite set tdi : i P Iu (although with rational coefficients).
Finally, one may implement the relation f pdi q ” f p1q pmod di ´ 1q in order to establish that in
fact f pdi q{di is a (small and fixed) integer for all i P I.
Shortlisted problems – solutions 81
for all k ą 3N .
Proof. Let k be an integer with k ą 3N . Set Π “ 3N
ś ` ˘
i“1 f pkq ´ f piq .
Notice that for every prime p P S, at most one of the numbers in the set
(
H “ f pkq ´ f piq : 1 ď i ď 3N
is divisible by a power of p which is greater than f p3N q; we say that such elements of H are bad.
Now, for each element h P H which is not bad we have Sphq ď f p3N qN , while the bad elements do
not exceed f pkq. Moreover, there are less than N bad elements in H. Therefore,
ź ` ˘3N 2 ` ˘N
SpΠq “ S phq ď f p3N q ¨ f pkq .
hPH
` ˘3N
This easily yields the lemma statement in view of the fact that LpΠqSpΠq “ Π ě µ f pkq for some
absolute constant µ. l
As a corollary of the lemma, one may get a weaker version of Claim 2 stating that there exists a
positive constant C such that f pkq ď Ck 3{2 for all k ą 3N . Indeed, from Step 1 we have
3N 3N
ź ź ˘2N
k3N ě
` ˘ `
Lpk ´ iq “ L f pkq ´ f piq ě c f pkq ,
i“1 i“1
Comment 3. In order to perform Step 3, it suffices to establish the equality f pnq “ an for any
infinite set of values of n. However, if this set has some good structure, then one may find easier ways
to complete this step.
For instance, after showing, as in Approach 2, that f pnq “ an for all n ě n0 with n ” 2 pmod N !q,
one may proceed as follows. Pick an arbitrary integer x and take any large prime p which is greater
than |f pxq ´ ax|. By the Chinese Remainder Theorem, there exists a positive integer n ą maxpx, n0 q
such that n ” 2 pmod N !q and n ” x pmod pq. By Step 1, we have f pxq ” f pnq “ an ” ax pmod pq.
Due to the choice of p, this is possible only if f pxq “ ax.
Shortlisted Problems with Solutions
57th International Mathematical Olympiad
Hong Kong, 2016
Note of Confidentiality
Contributing Countries
The Organising Committee and the Problem Selection Committee of IMO 2016 thank the
following 40 countries for contributing 121 problem proposals:
Front row from left: Yong-Gao Chen, Andy Liu, Tat Wing Leung (Chairman).
Back row from left: Yi-Jun Yao, Yun-Hao Fu, Yi-Jie He,
Zhongtao Wu, Heung Wing Joseph Lee, Chi Hong Chow,
Ka Ho Law, Tak Wing Ching.
Shortlisted problems 3
Problems
Algebra
A1. Let a, b and c be positive real numbers such that min {ab, bc, ca} > 1. Prove that
Ç å2
»
3
a+b+c
(a2 + 1)(b2 + 1)(c2 + 1) 6 + 1.
3
A2. Find the smallest real constant C such that for any positive real numbers a1 , a2 , a3 , a4
and a5 (not necessarily distinct), one can always choose distinct subscripts i, j, k and l such
that
a ak
i
aj
− 6 C.
al
A3. Find all integers n > 3 with the following property: for all real numbers a1 , a2 , . . . , an
and b1 , b2 , . . . , bn satisfying |ak | + |bk | = 1 for 1 6 k 6 n, there exist x1 , x2 , . . . , xn , each of
which is either −1 or 1, such that
n n
X X
xk ak + xk bk
6 1.
k=1 k=1
A4. Denote by R+ the set of all positive real numbers. Find all functions f : R+ → R+ such
that Ä ä
xf (x2 )f (f (y)) + f (yf (x)) = f (xy) f (f (x2 )) + f (f (y 2 ))
for all positive real numbers x and y.
A5.
a
(a) Prove that for every positive integer n, there exists a fraction where a and b are integers
√ √ √ b
satisfying 0 < b 6 n + 1 and n 6 ab 6 n + 1.
a
(b) Prove that there are infinitely many positive integers n such that there is no fraction
√ √ √ b
where a and b are integers satisfying 0 < b 6 n and n 6 ab 6 n + 1.
4 IMO 2016 Hong Kong
is written on the board. One tries to erase some linear factors from both sides so that each
side still has at least one factor, and the resulting equation has no real roots. Find the least
number of linear factors one needs to erase to achieve this.
A7. Denote by R the set of all real numbers. Find all functions f : R → R such that
f (0) 6= 0 and
f (x + y)2 = 2f (x)f (y) + max {f (x2 ) + f (y 2 ), f (x2 + y 2 )}
for all real numbers x and y.
A8. Determine the largest real number a such that for all n > 1 and for all real numbers
x0 , x1 , . . . , xn satisfying 0 = x0 < x1 < x2 < · · · < xn , we have
Ç å
1 1 1 2 3 n+1
+ + ··· + >a + + ··· + .
x1 − x0 x2 − x1 xn − xn−1 x1 x2 xn
Shortlisted problems 5
Combinatorics
C1. The leader of an IMO team chooses positive integers n and k with n > k, and announces
them to the deputy leader and a contestant. The leader then secretly tells the deputy leader
an n-digit binary string, and the deputy leader writes down all n-digit binary strings which
differ from the leader’s in exactly k positions. (For example, if n = 3 and k = 1, and if the
leader chooses 101, the deputy leader would write down 001, 111 and 100.) The contestant
is allowed to look at the strings written by the deputy leader and guess the leader’s string.
What is the minimum number of guesses (in terms of n and k) needed to guarantee the correct
answer?
C2. Find all positive integers n for which all positive divisors of n can be put into the cells
of a rectangular table under the following constraints:
C3. Let n be a positive integer relatively prime to 6. We paint the vertices of a regular
n-gon with three colours so that there is an odd number of vertices of each colour. Show that
there exists an isosceles triangle whose three vertices are of different colours.
C4. Find all positive integers n for which we can fill in the entries of an n × n table with
the following properties:
• in each row and each column, the letters I, M and O occur the same number of times;
and
• in any diagonal whose number of entries is a multiple of three, the letters I, M and O
occur the same number of times.
C5. Let n > 3 be a positive integer. Find the maximum number of diagonals of a regular
n-gon one can select, so that any two of them do not intersect in the interior or they are
perpendicular to each other.
6 IMO 2016 Hong Kong
C6. There are n > 3 islands in a city. Initially, the ferry company offers some routes between
some pairs of islands so that it is impossible to divide the islands into two groups such that
no two islands in different groups are connected by a ferry route.
After each year, the ferry company will close a ferry route between some two islands X
and Y . At the same time, in order to maintain its service, the company will open new routes
according to the following rule: for any island which is connected by a ferry route to exactly
one of X and Y , a new route between this island and the other of X and Y is added.
Suppose at any moment, if we partition all islands into two nonempty groups in any way,
then it is known that the ferry company will close a certain route connecting two islands from
the two groups after some years. Prove that after some years there will be an island which is
connected to all other islands by ferry routes.
C7. Let n > 2 be an integer. In the plane, there are n segments given in such a way that
any two segments have an intersection point in the interior, and no three segments intersect
at a single point. Jeff places a snail at one of the endpoints of each of the segments and claps
his hands n − 1 times. Each time when he claps his hands, all the snails move along their own
segments and stay at the next intersection points until the next clap. Since there are n − 1
intersection points on each segment, all snails will reach the furthest intersection points from
their starting points after n − 1 claps.
(a) Prove that if n is odd then Jeff can always place the snails so that no two of them ever
occupy the same intersection point.
(b) Prove that if n is even then there must be a moment when some two snails occupy the
same intersection point no matter how Jeff places the snails.
C8. Let n be a positive integer. Determine the smallest positive integer k with the following
property: it is possible to mark k cells on a 2n × 2n board so that there exists a unique
partition of the board into 1 × 2 and 2 × 1 dominoes, none of which contains two marked
cells.
Shortlisted problems 7
Geometry
G1. In a convex pentagon ABCDE, let F be a point on AC such that ∠F BC = 90◦ .
Suppose triangles ABF , ACD and ADE are similar isosceles triangles with
G2. Let ABC be a triangle with circumcircle Γ and incentre I. Let M be the midpoint of
side BC. Denote by D the foot of perpendicular from I to side BC. The line through I per-
pendicular to AI meets sides AB and AC at F and E respectively. Suppose the circumcircle
of triangle AEF intersects Γ at a point X other than A. Prove that lines XD and AM meet
on Γ.
G3. Let B = (−1, 0) and C = (1, 0) be fixed points on the coordinate plane. A nonempty,
bounded subset S of the plane is said to be nice if
(i) there is a point T in S such that for every point Q in S, the segment T Q lies entirely
in S; and
(ii) for any triangle P1 P2 P3 , there exists a unique point A in S and a permutation σ of the
indices {1, 2, 3} for which triangles ABC and Pσ(1) Pσ(2) Pσ(3) are similar.
Prove that there exist two distinct nice subsets S and S 0 of the set {(x, y) : x > 0, y > 0}
such that if A ∈ S and A0 ∈ S 0 are the unique choices of points in (ii), then the product
BA · BA0 is a constant independent of the triangle P1 P2 P3 .
G4. Let ABC be a triangle with AB = AC 6= BC and let I be its incentre. The line BI
meets AC at D, and the line through D perpendicular to AC meets AI at E. Prove that the
reflection of I in AC lies on the circumcircle of triangle BDE.
G5. Let D be the foot of perpendicular from A to the Euler line (the line passing through the
circumcentre and the orthocentre) of an acute scalene triangle ABC. A circle ω with centre
S passes through A and D, and it intersects sides AB and AC at X and Y respectively. Let
P be the foot of altitude from A to BC, and let M be the midpoint of BC. Prove that the
circumcentre of triangle XSY is equidistant from P and M .
8 IMO 2016 Hong Kong
G6. Let ABCD be a convex quadrilateral with ∠ABC = ∠ADC < 90◦ . The internal
angle bisectors of ∠ABC and ∠ADC meet AC at E and F respectively, and meet each
other at point P . Let M be the midpoint of AC and let ω be the circumcircle of triangle
BP D. Segments BM and DM intersect ω again at X and Y respectively. Denote by Q the
intersection point of lines XE and Y F . Prove that P Q ⊥ AC.
G7. Let I be the incentre of a non-equilateral triangle ABC, IA be the A-excentre, IA0 be
the reflection of IA in BC, and lA be the reflection of line AIA0 in AI. Define points IB , IB0
and line lB analogously. Let P be the intersection point of lA and lB .
(a) Prove that P lies on line OI where O is the circumcentre of triangle ABC.
(b) Let one of the tangents from P to the incircle of triangle ABC meet the circumcircle at
points X and Y . Show that ∠XIY = 120◦ .
G8. Let A1 , B1 and C1 be points on sides BC, CA and AB of an acute triangle ABC
respectively, such that AA1 , BB1 and CC1 are the internal angle bisectors of triangle ABC.
Let I be the incentre of triangle ABC, and H be the orthocentre of triangle A1 B1 C1 . Show
that
AH + BH + CH > AI + BI + CI.
Shortlisted problems 9
Number Theory
N1. For any positive integer k, denote the sum of digits of k in its decimal representation by
S(k). Find all polynomials P (x) with integer coefficients such that for any positive integer
n > 2016, the integer P (n) is positive and
N2. Let τ (n) be the number of positive divisors of n. Let τ1 (n) be the number of positive
divisors of n which have remainders 1 when divided by 3. Find all possible integral values of
the fraction ττ1(10n)
(10n)
.
N3. Define P (n) = n2 + n + 1. For any positive integers a and b, the set
{P (a), P (a + 1), P (a + 2), . . . , P (a + b)}
is said to be fragrant if none of its elements is relatively prime to the product of the other
elements. Determine the smallest size of a fragrant set.
N4. Let n, m, k and l be positive integers with n 6= 1 such that nk + mnl + 1 divides nk+l − 1.
Prove that
• m = 1 and l = 2k; or
nk−l −1
• l|k and m = nl −1
.
N5. Let a be a positive integer which is not a square number. Denote by A the set of all
positive integers k such that
x2 − a
k= 2 (1)
x − y2
√
for some integers x and y with x > a. Denote by B the set √ of all positive integers k such
that (1) is satisfied for some integers x and y with 0 6 x < a. Prove that A = B.
N6. Denote by N the set of all positive integers. Find all functions f : N → N such that
for all positive integers m and n, the integer f (m) + f (n) − mn is nonzero and divides
mf (m) + nf (n).
10 IMO 2016 Hong Kong
N7. Let n be an odd positive integer. In the Cartesian plane, a cyclic polygon P with area
S is chosen. All its vertices have integral coordinates, and all squares of its side lengths are
divisible by n. Prove that 2S is an integer divisible by n.
N8. Find all polynomials P (x) of odd degree d and with integer coefficients satisfying the
following property: for each positive integer n, there exist n positive integers x1 , x2 , . . . , xn
such that 21 < PP (x
(xi )
j)
< 2 and PP (x
(xi )
j)
is the d-th power of a rational number for every pair of
indices i and j with 1 6 i, j 6 n.
Shortlisted problems 11
Solutions
Algebra
A1. Let a, b and c be positive real numbers such that min {ab, bc, ca} > 1. Prove that
Ç å2
»
3
a+b+c
(a2 + 1)(b2 + 1)(c2 + 1) 6 + 1. (1)
3
a+b+c
Without loss of generality, assume a > b > c. This implies a > 1. Let d = 3
. Note
that
a(a + b + c) 1+1+1
ad = > = 1.
3 3
Then we can apply (2) to the pair (a, d) and the pair (b, c). We get
Ç å2 !2 Ç å2 !2
2 2 2 2 a+d b+c
(a + 1)(d + 1)(b + 1)(c + 1) 6 +1 +1 . (3)
2 2
Next, from
a+d b+c √ √
· > ad · bc > 1,
2 2
we can apply (2) again to the pair ( a+d
2
, b+c
2
). Together with (3), we have
Ç å2 !4
2 2 2 2 a+b+c+d
(a + 1)(d + 1)(b + 1)(c + 1) 6 +1 = (d2 + 1)4 .
4
Therefore, (a2 + 1)(b2 + 1)(c2 + 1) 6 (d2 + 1)3 , and (1) follows by taking cube root of both
sides.
12 IMO 2016 Hong Kong
Comment. After justifying the Claim, one may also obtain (1) by mixing variables. Indeed,
the function involved is clearly continuous, and hence it suffices to check that the condition
xy > 1 is preserved under each mixing step. This is true since whenever ab, bc, ca > 1, we
have
a+b a+b a+b 1+1
· > ab > 1 and ·c> = 1.
2 2 2 2
Solution 2. Let f (x) = ln (1 + x2 ). Then the inequality (1) to be shown is equivalent to
Ç å
f (a) + f (b) + f (c) a+b+c
6f ,
3 3
2(1 − x2 )
f 00 (x) = ,
(1 + x2 )2
we know that f is concave on [1, ∞). Then we can apply Jensen’s Theorem to get
A2. Find the smallest real constant C such that for any positive real numbers a1 , a2 , a3 , a4
and a5 (not necessarily distinct), one can always choose distinct subscripts i, j, k and l such
that
a ak
i
aj
− 6 C. (1)
al
Comment. The conclusion still holds if a1 , a2 , . . . , a5 are pairwise distinct, since in the con-
struction, we may replace the 2’s by real numbers sufficiently close to 2.
There are two possible simplifications for this problem:
1
(i) the answer C = 2
is given to the contestants; or
(ii) simply ask the contestants to prove the inequality (1) for C = 12 .
14 IMO 2016 Hong Kong
A3. Find all integers n > 3 with the following property: for all real numbers a1 , a2 , . . . , an
and b1 , b2 , . . . , bn satisfying |ak | + |bk | = 1 for 1 6 k 6 n, there exist x1 , x2 , . . . , xn , each of
which is either −1 or 1, such that
n n
X X
xk ak + xk bk
6 1. (1)
k=1 k=1
For any odd integer n > 3, we may assume without loss of generality bk > 0 for 1 6 k 6 n
(this can be done by flipping the pair (ak , bk ) to (−ak , −bk ) and xk to −xk if necessary) and
a1 > a2 > · · · > am > 0 > am+1 > · · · > an . We claim that the choice xk = (−1)k+1 for
1 6 k 6 n will work. Define
m
X n
X
s= x k ak and t = − x k ak .
k=1 k=m+1
Note that
s = (a1 − a2 ) + (a3 − a4 ) + · · · > 0
by the assumption a1 > a2 > · · · > am (when m is odd, there is a single term am at the end,
which is also positive). Next, we have
s = a1 − (a2 − a3 ) − (a4 − a5 ) − · · · 6 a1 6 1.
Similarly,
t = (−an + an−1 ) + (−an−2 + an−3 ) + · · · > 0
and
t = −an + (an−1 − an−2 ) + (an−3 − an−4 ) + · · · 6 −an 6 1.
From the condition, we have ak +bk = 1 for 1 6 k 6 m and −ak +bk = 1 for m+1 6 k 6 n.
It follows that nk=1 xk ak = s − t and nk=1 xk bk = 1 − s − t. Hence it remains to prove
P P
|s − t| + |1 − s − t| 6 1
under the constraint 0 6 s, t 6 1. By symmetry, we may assume s > t. If 1 − s − t > 0, then
we have
|s − t| + |1 − s − t| = s − t + 1 − s − t = 1 − 2t 6 1.
If 1 − s − t 6 0, then we have
|s − t| + |1 − s − t| = s − t − 1 + s + t = 2s − 1 6 1.
Hence, the inequality is true in both cases.
These show n can be any odd integer greater than or equal to 3.
Shortlisted problems 15
Solution 2. The even case can be handled in the same way as Solution 1. For the odd case,
we prove by induction on n.
Firstly, for n = 3, we may assume without loss of generality a1 > a2 > a3 > 0 and
b1 = a1 − 1 (if b1 = 1 − a1 , we may replace each bk by −bk ).
c − 1 6 b1 − b2 + b3 = a1 + a2 − a3 − 1 6 1 − c.
c − 1 6 −b1 + b2 + b3 = −a1 + a2 − a3 + 1 6 1 − c.
−c − 1 6 −b1 + b2 + b3 = −a1 + a2 − a3 + 1 6 1 + c.
c − 1 6 −b1 + b2 + b3 = −a1 − a2 + a3 + 1 6 1 − c.
−c − 1 6 −b1 + b2 + b3 = −a1 − a2 + a3 + 1 6 1 + c.
• Case 2. 0 > b0 > −a0 , in which case we take (x1 , x2 ) = (−1, 1).
We have | − a1 + a2 + a0 | + | − (a1 − 1) + (a2 − 1) + b0 | = | − d + a0 | + | − d + b0 |. If −d + a0 > 0,
this equals a0 − b0 = |a0 | + |b0 | 6 1. If −d + a0 < 0, this equals 2d − a0 − b0 6 2d 6 1.
A4. Denote by R+ the set of all positive real numbers. Find all functions f : R+ → R+ such
that Ä ä
xf (x2 )f (f (y)) + f (yf (x)) = f (xy) f (f (x2 )) + f (f (y 2 )) (1)
for all positive real numbers x and y.
1
Answer. f (x) = x
for any x ∈ R+ .
Solution 1. Taking x = y = 1 in (1), we get f (1)f (f (1)) + f (f (1)) = 2f (1)f (f (1)) and
hence f (1) = 1. Swapping x and y in (1) and comparing with (1) again, we find
xf (x2 )f (f (y)) + f (yf (x)) = yf (y 2 )f (f (x)) + f (xf (y)). (2)
Taking y = 1 in (2), we have xf (x2 ) + f (f (x)) = f (f (x)) + f (x), that is,
f (x)
f (x2 ) = . (3)
x
Take y = 1 in (1) and apply (3) to xf (x2 ). We get f (x) + f (f (x)) = f (x)(f (f (x2 )) + 1),
which implies
f (f (x))
f (f (x2 )) = . (4)
f (x)
For any x ∈ R+ , we find that
Ç å
2 f (f (x)) (4)
(3) (3) f (x)
f (f (x) ) = = f (f (x2 )) = f . (5)
f (x) x
It remains to show the following key step.
• Claim. The function f is injective.
Proof. Using (3) and (4), we rewrite (1) as
Ç å
f (f (x)) f (f (y))
f (x)f (f (y)) + f (yf (x)) = f (xy) + . (6)
f (x) f (y)
Take x = y in (6) and apply (3). This gives f (x)f (f (x)) + f (xf (x)) = 2 f (fx(x)) , which means
Ç å
2
f (xf (x)) = f (f (x)) − f (x) . (7)
x
Using (3), equation (2) can be rewritten as
f (x)f (f (y)) + f (yf (x)) = f (y)f (f (x)) + f (xf (y)). (8)
Suppose f (x) = f (y) for some x, y ∈ R+ . Then (8) implies
f (yf (y)) = f (yf (x)) = f (xf (y)) = f (xf (x)).
Using (7), this gives
Ç å Ç å
2 2
f (f (y)) − f (y) = f (f (x)) − f (x) .
y x
Noting f (x) = f (y), we find x = y. This establishes the injectivity.
18 IMO 2016 Hong Kong
By the Claim and (5), we get the only possible solution f (x) = x1 . It suffices to check that
this is a solution. Indeed, the left-hand side of (1) becomes
1 x y x
x· 2 ·y+ = + ,
x y x y
while the right-hand side becomes
1 2 x y
(x + y 2 ) = + .
xy y x
The two sides agree with each other.
Solution 2. Taking x = y = 1 in (1), we get f (1)f (f (1)) + f (f (1)) = 2f (1)f (f (1)) and
hence f (1) = 1. Putting x = 1 in (1), we have f (f (y)) + f (y) = f (y)(1 + f (f (y 2 ))) so that
f (f (y)) = f (y)f (f (y 2 )). (9)
Putting y = 1 in (1), we get xf (x2 ) + f (f (x)) = f (x)(f (f (x2 )) + 1). Using (9), this gives
xf (x2 ) = f (x). (10)
1
Replace y by x
in (1). Then we have
Ç Ç åå Ç å Ç Ç åå
2 1 f (x) 2 1
xf (x )f f +f = f (f (x )) + f f .
x x x2
The relation (10) shows f ( f (x)
x
) = f (f (x2 )). Also, using (9) with y = 1
x
and using (10) again,
the last equation reduces to Ç å
1
f (x)f = 1. (11)
x
Replace x by x1 and y by y1 in (1) and apply (11). We get
Ç å
1 1 1 1 1
2
+ = + .
xf (x )f (f (y)) f (yf (x)) f (xy) f (f (x )) f (f (y 2 ))
2
Clearing denominators, we can use (1) to simplify the numerators and obtain
f (xy)2 f (f (x2 ))f (f (y 2 )) = xf (x2 )f (f (y))f (yf (x)).
Using (9) and (10), this is the same as
f (xy)2 f (f (x)) = f (x)2 f (y)f (yf (x)). (12)
Substitute y = f (x) in (12) and apply (10) (with x replaced by f (x)). We have
f (xf (x))2 = f (x)f (f (x)). (13)
Taking y = x in (12), squaring both sides, and using (10) and (13), we find that
f (f (x)) = x4 f (x)3 . (14)
Finally, we combine (9), (10) and (14) to get
(14) (9) (14) (10)
y 4 f (y)3 = f (f (y)) = f (y)f (f (y 2 )) = f (y)y 8 f (y 2 )3 = y 5 f (y)4 ,
which implies f (y) = y1 . This is a solution by the checking in Solution 1.
Shortlisted problems 19
A5.
a
(a) Prove that for every positive integer n, there exists a fraction where a and b are integers
√ √ √ b
satisfying 0 < b 6 n + 1 and n 6 ab 6 n + 1.
a
(b) Prove that there are infinitely many positive integers n such that there is no fraction
√ √ √ b
where a and b are integers satisfying 0 < b 6 n and n 6 ab 6 n + 1.
Solution.
(a) Let r be the unique positive integer for which r2 6 n < (r + 1)2 . Write n = r2 + s. Then
we have 0 6 s 6 2r. We discuss in two cases according to the parity of s.
• Case 1. s is even.
s 2
Consider the number (r + 2r
) = r2 + s + ( 2rs )2 . We find that
2 s ã2 2
Å
n=r +s6r +s+ 6 r2 + s + 1 = n + 1.
2r
It follows that √ √
s
n6r+ 6 n + 1.
2r
s r2 +(s/2) √
Since s is even, we can choose the fraction r + 2r
= r
since r 6 n.
• Case 2. s is odd.
Consider the number (r + 1 − 2r+1−s
2(r+1)
)2 = (r + 1)2 − (2r + 1 − s) + ( 2r+1−s
2(r+1)
)2 . We find that
å2
2r + 1 − s
Ç
2 2 2
n = r + s = (r + 1) − (2r + 1 − s) 6 (r + 1) − (2r + 1 − s) +
2(r + 1)
2
6 (r + 1) − (2r + 1 − s) + 1 = n + 1.
It follows that
√ 2r + 1 − s √
n6r+1− 6 n + 1.
2(r + 1)
2r+1−s (r+1)2 −r+((s−1)/2)
Since s is odd, we can choose the fraction (r + 1) − 2(r+1)
= r+1
since
√
r + 1 6 n + 1.
√
(b) We show that for every positive integer r, there is no fraction ab with b 6 r2 + 1 such
√ √
that r2 + 1 6 ab 6 r2 + 2. Suppose on the contrary that such a fraction exists. Since
√
b 6 r2 + 1 < r + 1 and b is an integer, we have b 6 r. Hence,
This shows the square number a2 is strictly bounded between the two consecutive squares
(br)2 and (br + 1)2 , which is impossible. Hence, we have found infinitely many n = r2 + 1
for which there is no fraction of the desired form.
20 IMO 2016 Hong Kong
• Case 1. x = 1, 2, . . . , 2016.
In this case, one side of (1) is zero while the other side is not. This shows x cannot satisfy
(1).
• Case 2. 4k + 1 < x < 4k + 2 or 4k + 3 < x < 4k + 4 for some k = 0, 1, . . . , 503.
For j = 0, 1, . . . , 503 with j 6= k, the product (x − 4j − 1)(x − 4j − 4) is positive. For
j = k, the product (x − 4k − 1)(x − 4k − 4) is negative. This shows the left-hand side of (1)
is negative. On the other hand, each product (x − 4j − 2)(x − 4j − 3) on the right-hand side
of (1) is positive. This yields a contradiction.
• Case 3. x < 1 or x > 2016 or 4k < x < 4k + 1 for some k = 1, 2, . . . , 503.
The equation (1) can be rewritten as
503 503
(x − 4j − 1)(x − 4j − 4) Y
Ç å
Y 2
1= = 1− .
j=0 (x − 4j − 2)(x − 4j − 3) j=0 (x − 4j − 2)(x − 4j − 3)
Note that (x − 4j − 2)(x − 4j − 3) > 2 for 0 6 j 6 503 in this case. So each term in the
product lies strictly between 0 and 1, and the whole product must be less than 1, which is
impossible.
• Case 4. 4k + 2 < x < 4k + 3 for some k = 0, 1, . . . , 503.
This time we rewrite (1) as
503
x − 1 x − 2016 Y (x − 4j)(x − 4j − 1)
1= ·
x − 2 x − 2015 j=1 (x − 4j + 1)(x − 4j − 2)
503
x − 1 x − 2016 Y
Ç å
2
= · 1+ .
x − 2 x − 2015 j=1 (x − 4j + 1)(x − 4j − 2)
Clearly, x−1
x−2
and x−2016
x−2015
are both greater than 1. For the range of x in this case, each term
in the product is also greater than 1. Then the right-hand side must be greater than 1 and
hence a contradiction arises.
Shortlisted problems 21
From the four cases, we conclude that (1) has no real roots. Hence, the minimum number
of linear factors to be erased is 2016.
Comment. We discuss the general case when 2016 is replaced by a positive integer n. The
above solution works equally well when n is divisible by 4.
If n ≡ 2 (mod 4), one may leave l(x) = (x − 1)(x − 2) · · · (x − n2 ) on the left-hand side
and r(x) = (x − n2 − 1)(x − n2 − 2) · · · (x − n) on the right-hand side. One checks that for
x < n+12
, we have |l(x)| < |r(x)|, while for x > n+1
2
, we have |l(x)| > |r(x)|.
If n ≡ 3 (mod 4), one may leave l(x) = (x − 1)(x − 2) · · · (x − n+12
) on the left-hand side
and r(x) = (x − 2 )(x − 2 ) · · · (x − n) on the right-hand side. For x < 1 or n+1
n+3 x+5
2
< x < n+3
2
,
n+1 n+3
we have l(x) > 0 > r(x). For 1 < x < 2 , we have |l(x)| < |r(x)|. For x > 2 , we have
|l(x)| > |r(x)|.
If n ≡ 1 (mod 4), as the proposer mentioned, the situation is a bit more out of control.
Since the construction for n − 1 ≡ 0 (mod 4) works, the answer can be either n or n − 1. For
n = 5, we can leave the products (x − 1)(x − 2)(x − 3)(x − 4) and (x − 5). For n = 9, the
only example that works is l(x) = (x − 1)(x − 2)(x − 9) and r(x) = (x − 3)(x − 4) · · · (x − 8),
while there seems to be no such partition for n = 13.
22 IMO 2016 Hong Kong
A7. Denote by R the set of all real numbers. Find all functions f : R → R such that
f (0) 6= 0 and
f (x + y)2 = 2f (x)f (y) + max {f (x2 ) + f (y 2 ), f (x2 + y 2 )} (1)
for all real numbers x and y.
Answer.
• f (x) = −1 for any x ∈ R; or
• f (x) = x − 1 for any x ∈ R.
Solution 1. Taking x = y = 0 in (1), we get f (0)2 = 2f (0)2 +max {2f (0), f (0)}. If f (0) > 0,
then f (0)2 + 2f (0) = 0 gives no positive solution. If f (0) < 0, then f (0)2 + f (0) = 0 gives
f (0) = −1. Putting y = 0 in (1), we have f (x)2 = −2f (x) + f (x2 ), which is the same as
(f (x) + 1)2 = f (x2 ) + 1. Let g(x) = f (x) + 1. Then for any x ∈ R, we have
g(x2 ) = g(x)2 > 0. (2)
From (1), we find that f (x + y)2 > 2f (x)f (y) + f (x2 ) + f (y 2 ). In terms of g, this becomes
(g(x + y) − 1)2 > 2(g(x) − 1)(g(y) − 1) + g(x2 ) + g(y 2 ) − 2. Using (2), this means
(g(x + y) − 1)2 > (g(x) + g(y) − 1)2 − 1. (3)
Putting x = 1 in (2), we get g(1) = 0 or 1. The two cases are handled separately.
• Case 1. g(1) = 0, which is the same as f (1) = −1.
We put x = −1 and y = 0 in (1). This gives f (−1)2 = −2f (−1) − 1, which forces
f (−1) = −1. Next, we take x = −1 and y = 1 in (1) to get 1 = 2 + max {−2, f (2)}. This
clearly implies 1 = 2 + f (2) and hence f (2) = −1, that is, g(2) = 0. From (2), we can prove
n n n
inductively that g(22 ) = g(2)2 = 0 for any n ∈ N. Substitute y = 22 − x in (3). We obtain
n n
(g(x) + g(22 − x) − 1)2 6 (g(22 ) − 1)2 + 1 = 2.
n
For any fixed x > 0, we consider n to be sufficiently
√ large so that 22 − x > 0. From (2), this
2n
implies g(2 − x) > 0 so that g(x) 6 1 + 2. Using (2) again, we get
n n √
g(x)2 = g(x2 ) 6 1 + 2
for any n ∈ N. Therefore, |g(x)| 6 1 for any x > 0.
If there exists a ∈ R for which g(a) 6= 0, then for sufficiently large n we must have
1 1 1
g((a2 ) 2n ) = g(a2 ) 2n > 12 . By taking x = −y = −(a2 ) 2n in (1), we obtain
1 = 2f (x)f (−x) + max {2f (x2 ), f (2x2 )}
= 2(g(x) − 1)(g(−x) − 1) + max {2(g(x2 ) − 1), g(2x2 ) − 1}
Ç åÇ å
1 1 1
62 − − +0=
2 2 2
since |g(−x)| = |g(x)| ∈ ( 12 , 1] by (2) and the choice of x, and since g(z) 6 1 for z > 0. This
yields a contradiction and hence g(x) = 0 must hold for any x. This means f (x) = −1 for
any x ∈ R, which clearly satisfies (1).
Shortlisted problems 23
By induction on n, it is easy to prove that g(2n) > n + 1 for all n ∈ N. For any real
number a > 1, we choose a large n ∈ N and take k to be the positive integer such that
n
2k 6 a2 < 2k + 2. From (2) and (3), we have
n n n 1 n
(g(a)2 − 1)2 + 1 = (g(a2 ) − 1)2 + 1 > (g(2k) + g(a2 − 2k) − 1)2 > k 2 > (a2 − 2)2
4
n n
since g(a2 − 2k) > 0. For large n, this clearly implies g(a)2 > 1. Thus,
n n 1 n
(g(a)2 )2 > (g(a)2 − 1)2 + 1 > (a2 − 2)2 .
4
This yields
n 1 n
g(a)2 > (a2 − 2). (4)
2
Note that n å2n
a2
Ç
2 2
n = 1 + n 6 1 +
a2 − 2 a2 − 2 2n (a2n − 2)
by binomial expansion. This can be rewritten as
n 1 a
(a2 − 2) 2n > 2 .
1+ 2n (a2n −2)
This implies
n 1 » n
2a2 > (1 + 4a2n+1 + 1) > g(a)2 .
2
When n tends to infinity, this forces g(a) 6 a. Together with g(a) > a, we get g(a) = a for
all real numbers a > 1, that is, f (a) = a − 1 for all a > 1.
Finally, for any x ∈ R, we choose y sufficiently large in (1) so that y, x + y > 1. This gives
(x + y − 1)2 = 2f (x)(y − 1) + max {f (x2 ) + y 2 − 1, x2 + y 2 − 1}, which can be rewritten as
As the right-hand side is fixed, this can only hold for all large y when f (x) = x − 1. We now
check that this function satisfies (1). Indeed, we have
Solution 2. Taking x = y = 0 in (1), we get f (0)2 = 2f (0)2 +max {2f (0), f (0)}. If f (0) > 0,
then f (0)2 + 2f (0) = 0 gives no positive solution. If f (0) < 0, then f (0)2 + f (0) = 0 gives
f (0) = −1. Putting y = 0 in (1), we have
Replace x by −x in (5) and compare with (5) again. We get f (x)2 +2f (x) = f (−x)2 +2f (−x),
which implies
f (x) = f (−x) or f (x) + f (−x) = −2. (6)
Taking x = y and x = −y respectively in (1) and comparing the two equations obtained,
we have
f (2x)2 − 2f (x)2 = 1 − 2f (x)f (−x). (7)
Combining (6) and (7) to eliminate f (−x), we find that f (2x) can be ±1 (when f (x) = f (−x))
or ±(2f (x) + 1) (when f (x) + f (−x) = −2).
We prove the following.
• Claim. f (x) + f (−x) = −2 for any x ∈ R.
Proof. Suppose there exists a ∈ R such that f (a) + f (−a) 6= −2. Then f (a) = f (−a) 6= −1
and we may assume a > 0. We first show that f (a) 6= 1. Suppose f (a) = 1. Consider y = a
in (7). We get f (2a)2 = 1. Taking x = y = a in (1), we have 1 = 2 + max {2f (a2 ), f (2a2 )}.
From (5), f (a2 ) = 3 so that 1 > 2 + 6. This is impossible, and thus f (a) 6= 1.
As f (a) 6= ±1, we have f (a) = ±(2f ( a2 ) + 1). Similarly, f (−a) = ±(2f (− a2 ) + 1). These
two expressions are equal since f (a) = f (−a). If f ( a2 ) = f (− a2 ), then the above argument
works when we replace a by a2 . In particular, we have f (a)2 = f (2 · a2 )2 = 1, which is a
contradiction. Therefore, (6) forces f ( a2 ) + f (− a2 ) = −2. Then we get
Å
a Å ã
ã Å
a
Å ã ã
± 2f + 1 = ± −2f −3 .
2 2
Shortlisted problems 25
For any choices of the two signs, we either get a contradiction or f ( a2 ) = −1, in which case
f ( a2 ) = f (− a2 ) and hence f (a) = ±1 again. Therefore, there is no such real number a and the
Claim follows.
Replace x and y by −x and −y in (1) respectively and compare with (1). We get
Using the Claim, this simplifies to f (x+y) = f (x)+f (y)+1. In addition, (5) can be rewritten
as (f (x) + 1)2 = f (x2 ) + 1. Therefore, the function g defined by g(x) = f (x) + 1 satisfies
g(x + y) = g(x) + g(y) and g(x)2 = g(x2 ). The latter relation shows g(y) is nonnegative for
y > 0. For such a function satisfying the Cauchy Equation g(x + y) = g(x) + g(y), it must
be monotonic increasing and hence g(x) = cx for some constant c.
From (cx)2 = g(x)2 = g(x2 ) = cx2 , we get c = 0 or 1, which corresponds to the two
functions f (x) = −1 and f (x) = x − 1 respectively, both of which are solutions to (1) as
checked in Solution 1.
and
f (x) = f (−x) or f (x) + f (−x) = −2 (9)
for any x ∈ R. We shall show that one of the statements in (9) holds for all x ∈ R. Suppose
f (a) = f (−a) but f (a) + f (−a) 6= −2, while f (b) 6= f (−b) but f (b) + f (−b) = −2. Clearly,
a, b 6= 0 and f (a), f (b) 6= −1.
Taking y = a and y = −a in (1) respectively and comparing the two equations obtained,
we have f (x + a)2 = f (x − a)2 , that is, f (x + a) = ±f (x − a). This implies f (x + 2a) = ±f (x)
for all x ∈ R. Putting x = b and x = −2a − b respectively, we find f (2a + b) = ±f (b)
and f (−2a − b) = ±f (−b) = ±(−2 − f (b)). Since f (b) 6= −1, the term ±(−2 − f (b)) is
distinct from ±f (b) in any case. So f (2a + b) 6= f (−2a − b). From (9), we must have
f (2a + b) + f (−2a − b) = −2. Note that we also have f (b) + f (−b) = −2 where |f (b)|, |f (−b)|
are equal to |f (2a + b)|, |f (−2a − b)| respectively. The only possible case is f (2a + b) = f (b)
and f (−2a − b) = f (−b).
Applying the argument to −a instead of a and using induction, we have f (2ka + b) = f (b)
and f (2ka − b) = f (−b) for any integer k. Note that f (b) + f (−b) = −2 and f (b) 6= −1
imply one of √ f (b), f (−b) is less than −1. Without loss of generality, assume f (b) < −1. We
consider x = 2ka + b in (8) for sufficiently large k so that
yields a contradiction. Therefore, one of the statements in (9) must hold for all x ∈ R.
26 IMO 2016 Hong Kong
Note that the left-hand side is ±1 while the right-hand side is an even integer. This is a
contradiction. Therefore, f (x) = −1 for all x ∈ R, which is clearly a solution.
A8. Determine the largest real number a such that for all n > 1 and for all real numbers
x0 , x1 , . . . , xn satisfying 0 = x0 < x1 < x2 < · · · < xn , we have
Ç å
1 1 1 2 3 n+1
+ + ··· + >a + + ··· + . (1)
x1 − x0 x2 − x1 xn − xn−1 x1 x2 xn
(k − 1)2 32
Ç å
(xk−1 + (xk − xk−1 )) + > (k − 1 + 3)2 ,
xk−1 xk − xk−1
When n tends to infinity, the left-hand side tends to 1 while the right-hand side tends to
9
4
a. Therefore a has to be at most 49 .
Hence the largest value of a is 94 .
From (4), the inequality (1) holds for a = 49 . This is also the upper bound as can be
verified in the same way as Solution 1.
Shortlisted problems 29
Combinatorics
C1. The leader of an IMO team chooses positive integers n and k with n > k, and announces
them to the deputy leader and a contestant. The leader then secretly tells the deputy leader
an n-digit binary string, and the deputy leader writes down all n-digit binary strings which
differ from the leader’s in exactly k positions. (For example, if n = 3 and k = 1, and if the
leader chooses 101, the deputy leader would write down 001, 111 and 100.) The contestant
is allowed to look at the strings written by the deputy leader and guess the leader’s string.
What is the minimum number of guesses (in terms of n and k) needed to guarantee the correct
answer?
Answer. The minimum number of guesses is 2 if n = 2k and 1 if n 6= 2k.
Solution 1. Let X be the binary string chosen by the leader and let X 0 be the binary string
of length n every digit of which is different from that of X. The strings written by the deputy
leader are the same as those in the case when the leader’s string is X 0 and k is changed to
n − k. In view of this, we may assume k > n2 . Also, for the particular case k = n2 , this
argument shows that the strings X and X 0 cannot be distinguished, and hence in that case
the contestant has to guess at least twice.
It remains to show that the number of guesses claimed suffices. Consider any string Y
which differs from X in m digits where 0 < m < 2k. Without loss of generality, assume
the first m digits of X and Y are distinct. Let Z be the binary string obtained from X by
changing its first k digits. Then Z is written by the deputy leader. Note that Z differs from Y
by |m − k| digits where |m − k| < k since 0 < m < 2k. From this observation, the contestant
must know that Y is not the desired string.
As we have assumed k > n2 , when n < 2k, every string Y 6= X differs from X in fewer
than 2k digits. When n = 2k, every string except X and X 0 differs from X in fewer than 2k
digits. Hence, the answer is as claimed.
Solution 2. Firstly, assume n 6= 2k. Without loss of generality suppose the firstÄ digitä of
the leader’s string is 1. Then among the nk strings written by the deputy leader, n−1
Ä ä
k
will
Än−1ä
begin with 1 and k−1 will begin with 0. Since n 6= 2k, we have k + (k − 1) 6= n − 1 and
so n−1
Ä ä Än−1ä
k
6
= k−1
. Thus, by counting the number of strings written by the deputy leader that
start with 0 and 1, the contestant can tell the first digit of the leader’s string. The same can
be done on the other digits, so 1 guess suffices when n 6= 2k.
Secondly, for the case n = 2 and k = 1, the answer is clearly 2. For the remaining cases
where n = 2k > 2, the deputy leader would write down the same strings if the leader’s string
X is replaced by X 0 obtained by changing each digit of X. This shows at least 2 guesses
are needed. We shall show that 2 guesses suffice in this case. Suppose the first two digits of
the leader’s string are the same. Then among the strings written by the deputy leader, theä
prefices 01 and 10 will occur 2k−2
Ä ä Ä2k−2
k−1
times each, while the prefices 00 and 11 will occur k
times each. The two numbers are interchanged if the first two digits of the leader’s string
are different. Since 2k−2
Ä ä Ä2k−2ä
k−1
6
= k
, the contestant can tell whether the first two digits of the
leader’s string are the same or not. He can work out the relation of the first digit and the
30 IMO 2016 Hong Kong
other digits in the same way and reduce the leader’s string to only 2 possibilities. The proof
is complete.
Shortlisted problems 31
C2. Find all positive integers n for which all positive divisors of n can be put into the cells
of a rectangular table under the following constraints:
Answer. 1.
Solution 1. Suppose all positive divisors of n can be arranged into a rectangular table of
size k × l where the number of rows k does not exceed the number of columns l. Let the
sum of numbers in each column be s. Since n belongs to one of the columns, we have s > n,
where equality holds only when n = 1.
For j = 1, 2, . . . , l, let dj be the largest number in the j-th column. Without loss of
generality, assume d1 > d2 > · · · > dl . Since these are divisors of n, we have
n
dl 6 . (1)
l
As dl is the maximum entry of the l-th column, we must have
s n
dl > > . (2)
k k
The relations (1) and (2) combine to give nl > nk , that is, k > l. Together with k 6 l, we
conclude that k = l. Then all inequalities in (1) and (2) are equalities. In particular, s = n
and so n = 1, in which case the conditions are clearly satisfied.
Solution 2. Clearly n = 1 works. Then we assume n > 1 and let its prime factorization be
n = pr11 pr22 · · · prt t . Suppose the table has k rows and l columns with 1 < k 6 l. Note that kl is
the number of positive divisors of n and the sum of all entries is the sum of positive divisors
of n, which we denote by σ(n). Consider the column containing n. Since the column sum is
σ(n)
l
, we must have σ(n) l
> n. Therefore, we have
Ç å2
2 σ(n)
(r1 + 1)(r2 + 1) · · · (rt + 1) = kl 6 l <
n
Ç å2 Ç å2
1 1 1 1
= 1+ + · · · + r1 · · · 1 + + · · · + rt .
p1 p1 pt pt
This can be rewritten as
f (p1 , r1 )f (p2 , r2 ) · · · f (pt , rt ) < 1 (3)
where 2
1
r+1 (r + 1) 1 − p
f (p, r) = 2 = 2 .
1 1 1
1+ p
+ ··· + pr
1− pr+1
32 IMO 2016 Hong Kong
From these values and bounds, it is clear that (3) holds only when n = 2 or 4. In both cases,
it is easy to see that the conditions are not satisfied. Hence, the only possible n is 1.
Shortlisted problems 33
C3. Let n be a positive integer relatively prime to 6. We paint the vertices of a regular
n-gon with three colours so that there is an odd number of vertices of each colour. Show that
there exists an isosceles triangle whose three vertices are of different colours.
Solution. For k = 1, 2, 3, let ak be the number of isosceles triangles whose vertices contain
exactly k colours. Suppose on the contrary that a3 = 0. Let b, c, d be the number of vertices
of the three different colours respectively. We now count the number of pairs (4, E) where
4 is an isosceles triangle and E is a side of 4 whose endpoints are of different colours.
On the one hand, since we have assumed a3 = 0, each triangle in the pair must contain
exactly two colours, and hence each triangle contributes twice. Thus the number of pairs is
2a2 .
On the other hand, if we pick any two vertices A, B of distinct colours, then there are
three isosceles triangles having these as vertices, two when AB is not the base and one when
AB is the base since n is odd. Note that the three triangles are all distinct as (n, 3) = 1. In
this way, we count the number of pairs to be 3(bc + cd + db). However, note that 2a2 is even
while 3(bc + cd + db) is odd, as each of b, c, d is. This yields a contradiction and hence a3 > 1.
Comment. A slightly stronger version of this problem is to replace the condition (n, 6) = 1
by n being odd (where equilateral triangles are regarded as isosceles triangles). In that case,
the only difference in the proof is that by fixing any two vertices A, B, one can find exactly
one or three isosceles triangles having these as vertices. But since only parity is concerned in
the solution, the proof goes the same way.
The condition that there is an odd number of vertices of each colour is necessary, as can be
seen from the following example. Consider n = 25 and we label the vertices A0 , A1 , . . . , A24 .
Suppose colour 1 is used for A0 , colour 2 is used for A5 , A10 , A15 , A20 , while colour 3 is used
for the remaining vertices. Then any isosceles triangle having colours 1 and 2 must contain
A0 and one of A5 , A10 , A15 , A20 . Clearly, the third vertex must have index which is a multiple
of 5 so it is not of colour 3.
34 IMO 2016 Hong Kong
C4. Find all positive integers n for which we can fill in the entries of an n × n table with
the following properties:
• each entry can be one of I, M and O;
• in each row and each column, the letters I, M and O occur the same number of times;
and
• in any diagonal whose number of entries is a multiple of three, the letters I, M and O
occur the same number of times.
Answer. n can be any multiple of 9.
Solution. We first show that such a table exists when n is a multiple of 9. Consider the
following 9 × 9 table.
I I I M M M O O O
M M M O O O I I I
O O O I I I M M M
I I I M M M O O O
M M M O O O I I I (1)
O O O I I I M M M
I I I M M M O O O
M M M O O O I I I
O O O I I I M M M
It is a direct checking that the table (1) satisfies the requirements. For n = 9k where k is
a positive integer, we form an n × n table using k × k copies of (1). For each row and each
column of the table of size n, since there are three I’s, three M ’s and three O’s for any nine
consecutive entries, the numbers of I, M and O are equal. In addition, every diagonal of the
large table whose number of entries is divisible by 3 intersects each copy of (1) at a diagonal
with number of entries divisible by 3 (possibly zero). Therefore, every such diagonal also
contains the same number of I, M and O.
Next, consider any n × n table for which the requirements can be met. As the number of
entries of each row should be a multiple of 3, we let n = 3k where k is a positive integer. We
divide the whole table into k × k copies of 3 × 3 blocks. We call the entry at the centre of
such a 3 × 3 square a vital entry. We also call any row, column or diagonal that contains at
least one vital entry a vital line. We compute the number of pairs (l, c) where l is a vital line
and c is an entry belonging to l that contains the letter M . We let this number be N .
On the one hand, since each vital line contains the same number of I, M and O, it is
obvious that each vital row and each vital column contain k occurrences of M . For vital
diagonals in either direction, we count there are exactly
1 + 2 + · · · + (k − 1) + k + (k − 1) + · · · + 2 + 1 = k 2
On the other hand, there are 3k 2 occurrences of M in the whole table. Note that each
entry belongs to exactly 1 or 4 vital lines. Therefore, N must be congruent to 3k 2 mod 3.
From the double counting, we get 4k 2 ≡ 3k 2 (mod 3), which forces k to be a multiple of
3. Therefore, n has to be a multiple of 9 and the proof is complete.
36 IMO 2016 Hong Kong
C5. Let n > 3 be a positive integer. Find the maximum number of diagonals of a regular
n-gon one can select, so that any two of them do not intersect in the interior or they are
perpendicular to each other.
• Case 1. n is odd.
We first claim that no pair of diagonals is perpendicular. Suppose A, B, C, D are vertices
where AB and CD are perpendicular, and let E be the vertex lying on the perpendicular
bisector of AB. Let E 0 be the opposite point of E on the circumcircle of the regular polygon.
Since EC = E 0 D and C, D, E are vertices of the regular polygon, E 0 should also belong to
the polygon. This contradicts the fact that a regular polygon with an odd number of vertices
does not contain opposite points on the circumcircle.
E
C
A B
D
E0
Therefore in the odd case we can only select diagonals which do not intersect. In the
maximal case these diagonals should divide the regular n-gon into n − 2 triangles, so we can
select at most n − 3 diagonals. This can be done, for example, by selecting all diagonals
emanated from a particular vertex.
• Case 2. n is even.
If there is no intersection, then the proof in the odd case works. Suppose there are two
perpendicular diagonals selected. We consider the set S of all selected diagonals parallel to
one of them which intersect with some selected diagonals. Suppose S contains k diagonals
and the number of distinct endpoints of the k diagonals is l.
Firstly, consider the longest diagonal in one of the two directions in S. No other diagonal
in S can start from either endpoint of that diagonal, since otherwise it has to meet another
longer diagonal in S. The same holds true for the other direction. Ignoring these two longest
diagonals and their four endpoints, the remaining k − 2 diagonals share l − 4 endpoints where
each endpoint can belong to at most two diagonals. This gives 2(l − 4) > 2(k − 2), so that
k 6 l − 2.
Shortlisted problems 37
d2
d d2
d1 d d1
Consider a group of consecutive vertices of the regular n-gon so that each of the two
outermost vertices is an endpoint of a diagonal in S, while the interior points are not. There
are l such groups. We label these groups P1 , P2 , . . . , Pl in this order. We claim that each
selected diagonal outside S must connect vertices of the same group Pi . Consider any diagonal
d joining vertices from distinct groups Pi and Pj . Let d1 and d2 be two diagonals in S each
having one of the outermost points of Pi as endpoint. Then d must meet either d1 , d2 or a
diagonal in S which is perpendicular to both d1 and d2 . In any case d should belong to S by
definition, which is a contradiction.
Within the same group Pi , there are no perpendicular diagonals since the vertices belong
to the same side of a diameter of the circumcircle. Hence there can be at most |Pi |−2 selected
diagonals within Pi , including the one joining the two outermost points of Pi when |Pi | > 2.
Therefore, the maximum number of diagonals selected is
l
X l
X
(|Pi | − 2) + k = |Pi | − 2l + k = (n + l) − 2l + k = n − l + k 6 n − 2.
i=1 i=1
This upper bound can be attained as follows. We take any vertex A and let A0 be the
vertex for which AA0 is a diameter of the circumcircle. If we select all diagonals emanated
from A together with the diagonal d0 joining the two neighbouring vertices of A0 , then the
only pair of diagonals that meet each other is AA0 and d0 , which are perpendicular to each
other. In total we can take n − 2 diagonals.
A0
d0
A
Solution 2. The constructions and the odd case are the same as Solution 1. Instead of
dealing separately with the case where n is even, we shall prove by induction more generally
that we can select at most n − 2 diagonals for any cyclic n-gon with circumcircle Γ.
38 IMO 2016 Hong Kong
The base case n = 3 is trivial since there is no diagonal at all. Suppose the upper bound
holds for any cyclic polygon with fewer than n sides. For a cyclic n-gon, if there is a selected
diagonal which does not intersect any other selected diagonal, then this diagonal divides the
n-gon into an m-gon and an l-gon (with m + l = n + 2) so that each selected diagonal belongs
to one of them. Without loss of generality, we may assume the m-gon lies on the same side
of a diameter of Γ. Then no two selected diagonals of the m-gon can intersect, and hence we
can select at most m − 3 diagonals. Also, we can apply the inductive hypothesis to the l-gon.
This shows the maximum number of selected diagonals is (m − 3) + (l − 2) + 1 = n − 2.
It remains to consider the case when all selected diagonals meet at least one other selected
diagonal. Consider a pair of selected perpendicular diagonals d1 , d2 . They divide the circum-
ference of Γ into four arcs, each of which lies on the same side of a diameter of Γ. If there
are two selected diagonals intersecting each other and neither is parallel to d1 or d2 , then
their endpoints must belong to the same arc determined by d1 , d2 , and hence they cannot be
perpendicular. This violates the condition, and hence all selected diagonals must have the
same direction as one of d1 , d2 .
d2
d1
Take the longest selected diagonal in one of the two directions. We argue as in Solution
1 that its endpoints do not belong to any other selected diagonal. The same holds true for
the longest diagonal in the other direction. Apart from these four endpoints, each of the
remaining n − 4 vertices can belong to at most two selected diagonals. Thus we can select at
most 12 (2(n − 4) + 4) = n − 2 diagonals. Then the proof follows by induction.
Shortlisted problems 39
C6. There are n > 3 islands in a city. Initially, the ferry company offers some routes between
some pairs of islands so that it is impossible to divide the islands into two groups such that
no two islands in different groups are connected by a ferry route.
After each year, the ferry company will close a ferry route between some two islands X
and Y . At the same time, in order to maintain its service, the company will open new routes
according to the following rule: for any island which is connected by a ferry route to exactly
one of X and Y , a new route between this island and the other of X and Y is added.
Suppose at any moment, if we partition all islands into two nonempty groups in any way,
then it is known that the ferry company will close a certain route connecting two islands from
the two groups after some years. Prove that after some years there will be an island which is
connected to all other islands by ferry routes.
Solution. Initially, we pick any pair of islands A and B which are connected by a ferry route
and put A in set A and B in set B. From the condition, without loss of generality there must
be another island which is connected to A. We put such an island C in set B. We say that
two sets of islands form a network if each island in one set is connected to each island in the
other set.
Next, we shall included all islands to A ∪ B one by one. Suppose we have two sets A and
B which form a network where 3 6 |A ∪ B| < n. This relation no longer holds only when a
ferry route between islands A ∈ A and B ∈ B is closed. In that case, we define A0 = {A, B},
and B 0 = (A ∪ B) − {A, B}. Note that B 0 is nonempty. Consider any island C ∈ A − {A}.
From the relation of A and B, we know that C is connected to B. If C was not connected to
A before the route between A and B closes, then there will be a route added between C and
A afterwards. Hence, C must now be connected to both A and B. The same holds true for
any island in B − {B}. Therefore, A0 and B 0 form a network, and A0 ∪ B 0 = A ∪ B. Hence
these islands can always be partitioned into sets A and B which form a network.
As |A ∪ B| < n, there are some islands which are not included in A ∪ B. From the
condition, after some years there must be a ferry route between an island A in A ∪ B and an
island D outside A ∪ B which closes. Without loss of generality assume A ∈ A. Then each
island in B must then be connected to D, no matter it was or not before. Hence, we can
put D in set A so that the new sets A and B still form a network and the size of A ∪ B is
increased by 1. The same process can be done to increase the size of A ∪ B. Eventually, all
islands are included in this way so we may now assume |A ∪ B| = n.
Suppose a ferry route between A ∈ A and B ∈ B is closed after some years. We put A
and B in set A0 and all remaining islands in set B 0 . Then A0 and B 0 form a network. This
relation no longer holds only when a route between A, without loss of generality, and C ∈ B 0
is closed. Since this must eventually occur, at that time island B will be connected to all
other islands and the result follows.
40 IMO 2016 Hong Kong
C7. Let n > 2 be an integer. In the plane, there are n segments given in such a way that
any two segments have an intersection point in the interior, and no three segments intersect
at a single point. Jeff places a snail at one of the endpoints of each of the segments and claps
his hands n − 1 times. Each time when he claps his hands, all the snails move along their own
segments and stay at the next intersection points until the next clap. Since there are n − 1
intersection points on each segment, all snails will reach the furthest intersection points from
their starting points after n − 1 claps.
(a) Prove that if n is odd then Jeff can always place the snails so that no two of them ever
occupy the same intersection point.
(b) Prove that if n is even then there must be a moment when some two snails occupy the
same intersection point no matter how Jeff places the snails.
Solution. We consider a big disk which contains all the segments. We extend each segment
to a line li so that each of them cuts the disk at two distinct points Ai , Bi .
(a) For odd n, we travel along the circumference of the disk and mark each of the points Ai
or Bi ‘in’ and ‘out’ alternately. Since every pair of lines intersect in the disk, there are
exactly n − 1 points between Ai and Bi for any fixed 1 6 i 6 n. As n is odd, this means
one of Ai and Bi is marked ‘in’ and the other is marked ‘out’. Then Jeff can put a snail
on the endpoint of each segment which is closer to the ‘in’ side of the corresponding line.
We claim that the snails on li and lj do not meet for any pairs i, j, hence proving part
(a).
Ai Ai Aj
Aj
P P
Without loss of generality, we may assume the snails start at Ai and Aj respectively.
Let li intersect lj at P . Note that there is an odd number of points between arc Ai Aj .
Each of these points belongs to a line lk . Such a line lk must intersect exactly one of
Shortlisted problems 41
the segments Ai P and Aj P , making an odd number of intersections. For the other lines,
they may intersect both segments Ai P and Aj P , or meet none of them. Therefore, the
total number of intersection points on segments Ai P and Aj P (not counting P ) is odd.
However, if the snails arrive at P at the same time, then there should be the same number
of intersections on Ai P and Aj P , which gives an even number of intersections. This is a
contradiction so the snails do not meet each other.
(b) For even n, we consider any way that Jeff places the snails and mark each of the points
Ai or Bi ‘in’ and ‘out’ according to the directions travelled by the snails. In this case
there must be two neighbouring points Ai and Aj both of which are marked ‘in’. Let
P be the intersection of the segments Ai Bi and Aj Bj . Then any other segment meeting
one of the segments Ai P and Aj P must also meet the other one, and so the number of
intersections on Ai P and Aj P are the same. This shows the snails starting from Ai and
Aj will meet at P .
Comment. The conclusions do not hold for pseudosegments, as can be seen from the follow-
ing examples.
42 IMO 2016 Hong Kong
C8. Let n be a positive integer. Determine the smallest positive integer k with the following
property: it is possible to mark k cells on a 2n × 2n board so that there exists a unique
partition of the board into 1 × 2 and 2 × 1 dominoes, none of which contains two marked
cells.
Answer. 2n.
Solution. We first construct an example of marking 2n cells satisfying the requirement.
Label the rows and columns 1, 2, . . . , 2n and label the cell in the i-th row and the j-th column
(i, j).
For i = 1, 2, . . . , n, we mark the cells (i, i) and (i, i + 1). We claim that the required
partition exists and is unique. The two diagonals of the board divide the board into four
regions. Note that the domino covering cell (1, 1) must be vertical. This in turn shows that
each domino covering (2, 2), (3, 3), . . . , (n, n) is vertical. By induction, the dominoes in the
left region are all vertical. By rotational symmetry, the dominoes in the bottom region are
horizontal, and so on. This shows that the partition exists and is unique.
It remains to show that this value of k is the smallest possible. Assume that only k < 2n
cells are marked, and there exists a partition P satisfying the requirement. It suffices to show
there exists another desirable partition distinct from P . Let d be the main diagonal of the
board.
Construct the following graph with edges of two colours. Its vertices are the cells of the
board. Connect two vertices with a red edge if they belong to the same domino of P . Connect
two vertices with a blue edge if their reflections in d are connected by a red edge. It is possible
that two vertices are connected by edges of both colours. Clearly, each vertex has both red
and blue degrees equal to 1. Thus the graph splits into cycles where the colours of edges in
each cycle alternate (a cycle may have length 2).
Consider any cell c lying on the diagonal d. Its two edges are symmetrical with respect
to d. Thus they connect c to different cells. This shows c belongs to a cycle C(c) of length at
least 4. Consider a part of this cycle c0 , c1 , . . . , cm where c0 = c and m is the least positive
integer such that cm lies on d. Clearly, cm is distinct from c. From the construction, the path
symmetrical to this with respect to d also lies in the graph, and so these paths together form
C(c). Hence, C(c) contains exactly two cells from d. Then all 2n cells in d belong to n cycles
C1 , C2 , . . . , Cn , each has length at least 4.
By the Pigeonhole Principle, there exists a cycle Ci containing at most one of the k marked
cells. We modify P as follows. We remove all dominoes containing the vertices of Ci , which
Shortlisted problems 43
correspond to the red edges of Ci . Then we put the dominoes corresponding to the blue edges
of Ci . Since Ci has at least 4 vertices, the resultant partition P 0 is different from P . Clearly,
no domino in P 0 contains two marked cells as Ci contains at most one marked cell. This
shows the partition is not unique and hence k cannot be less than 2n.
44 IMO 2016 Hong Kong
Geometry
G1. In a convex pentagon ABCDE, let F be a point on AC such that ∠F BC = 90◦ .
Suppose triangles ABF , ACD and ADE are similar isosceles triangles with
D C
E M
A B
Shortlisted problems 45
Solution 2. From ∠CAD = ∠EDA, we have AC//ED. Together with AC//EX, we know
that E, D, X are collinear. Denote the common angle in (1) by θ. From 4ABF ∼ 4ACD,
we get AB
AC
AF
= AD so that 4ABC ∼ 4AF D. This yields ∠AF D = ∠ABC = 90◦ + θ and
hence ∠F DC = 90◦ , implying that BCDF is cyclic. Let Γ1 be its circumcircle.
AB AF
Next, from 4ABF ∼ 4ADE, we have AD = AE so that 4ABD ∼ 4AF E. Therefore,
∠AF E = ∠ABD = θ + ∠F BD = θ + ∠F CD = 2θ = 180◦ − ∠BF A.
This implies B, F, E are collinear. Note that F is the incentre of triangle DAB. Point E
lies on the internal angle bisector of ∠DBA and lies on the perpendicular bisector of AD. It
follows that E lies on the circumcircle Γ2 of triangle ABD, and EA = EF = ED.
Also, since CF is a diameter of Γ1 and M is the midpoint of CF , M is the centre of Γ1 and
hence ∠AM D = 2θ = ∠ABD. This shows M lies on Γ2 . Next, ∠M DX = ∠M AE = ∠DXM
since AM XE is a parallelogram. Hence M D = M X and X lies on Γ1 .
D C
E M
A B
Comment. In Solution 2, both the Radical Axis Theorem and Desargues’ Theorem could
imply DB, M E, F X are parallel. However, this is impossible as can be seen from the config-
uration. For example, it is obvious that DB and M E meet each other.
Solution 3. Let the common angle in (1) be θ. From 4ABF ∼ 4ACD, we have AB AC
AF
= AD
so that 4ABC ∼ 4AF D. Then ∠ADF = ∠ACB = 90◦ − 2θ = 90◦ − ∠BAD and hence
DF ⊥ AB. As F A = F B, this implies 4DAB is isosceles with DA = DB. Then F is the
incentre of 4DAB.
Next, from ∠AED = 180◦ − 2θ = 180◦ − ∠DBA, points A, B, D, E are concyclic. Since
we also have EA = ED, this shows E, F, B are collinear and EA = EF = ED.
D C
P
E M
A B
Note that C lies on the internal angle bisector of ∠BAD and lies on the external angle
bisector of ∠DBA. It follows that it is the A-excentre of triangle DAB. As M is the midpoint
of CF , M lies on the circumcircle of triangle DAB and it is the centre of the circle passing
through D, F, B, C. By symmetry, DEF M is a rhombus. Then the midpoints of AX, EM
and DF coincide, and it follows that DAF X is a parallelogram.
Let P be the intersection of BD and EM , and Q be the intersection of AD and BE. From
∠BAC = ∠DCA, we know that DC, AB, EM are parallel. Thus we have DP PB
= CM
MA
. This is
AE
further equal to BE since CM = DM = DE = AE and M A = BE. From 4AEQ ∼ 4BEA,
we find that
DP AE AQ QF
= = =
PB BE BA FB
by the Angle Bisector Theorem. This implies QD//F P and hence F, P, X are collinear, as
desired.
Shortlisted problems 47
G2. Let ABC be a triangle with circumcircle Γ and incentre I. Let M be the midpoint of
side BC. Denote by D the foot of perpendicular from I to side BC. The line through I per-
pendicular to AI meets sides AB and AC at F and E respectively. Suppose the circumcircle
of triangle AEF intersects Γ at a point X other than A. Prove that lines XD and AM meet
on Γ.
• Claim. For any cyclic quadrilateral P QRS whose diagonals meet at T , we have
QT P Q · QR
= .
TS P S · SR
1
QT [P QR] P Q · QR sin ∠P QR P Q · QR
= = 21 = .
TS [P SR] 2
P S · SR sin ∠P SR P S · SR
BM AB·BY
Applying the Claim to ABY C and XBY C respectively, we have 1 = MC
= AC·CY
and
BD0
D0 C
= XB·BY
XC·CY
. These combine to give
BD0 XB BY XB AC
0
= · = · . (1)
CD XC CY XC AB
Next, we use directed angles to find that ]XBF = ]XBA = ]XCA = ]XCE and
]XF B = ]XF A = ]XEA = ]XEC. This shows triangles XBF and XCE are directly
similar. In particular, we have
XB BF
= . (2)
XC CE
In the following, we give two ways to continue the proof.
FB BI EC IC
= and = . (3)
IB BC IC BC
48 IMO 2016 Hong Kong
X B1 C1
C2
B2
E
F I
B D M C
Next, construct a line parallel to BC and tangent to the incircle. Suppose it meets sides
AB and AC at B1 and C1 respectively. Let the incircle touch AB and AC at B2 and C2
respectively. By homothety, the line B1 I is parallel to the external angle bisector of ∠ABC,
and hence ∠B1 IB = 90◦ . Since ∠BB2 I = 90◦ , we get BB2 · BB1 = BI 2 , and similarly
CC2 · CC1 = CI 2 . Hence,
BD0 XB AC BF AC BI 2 AC BD
0
= · = · = 2
· =
CD XC AB CE AB CI AB CD
so that D0 = D. The result then follows.
Solution 2. Let ωA be the A-mixtilinear incircle of triangle ABC. From the properties of
mixtilinear incircles, ωA touches sides AB and AC at F and E respectively. Suppose ωA
is tangent to Γ at T . Let AM meet Γ again at Y , and let D1 , T1 be the reflections of D
and T with respect to the perpendicular bisector of BC respectively. It is well-known that
∠BAT = ∠D1 AC so that A, D1 , T1 are collinear.
X
S
E
F I
P
R
D
B C
M D1
T T1
We then show that X, M, T1 are collinear. Let R be the radical centre of ωA , Γ and the
circumcircle of triangle AEF . Then R lies on AX, EF and the tangent at T to Γ. Let AT
meet ωA again at S and meet EF at P . Obviously, SF T E is a harmonic quadrilateral.
Projecting from T , the pencil (R, P ; F, E) is harmonic. We further project the pencil onto
Γ from A, so that XBT C is a harmonic quadrilateral. As T T1 //BC, the projection from T1
onto BC maps T to a point at infinity, and hence maps X to the midpoint of BC, which is
M . This shows X, M, T1 are collinear.
We have two ways to finish the proof.
• Method 1. Note that both AY and XT1 are chords of Γ passing through the midpoint M
of the chord BC. By the Butterfly Theorem, XY and AT1 cut BC at a pair of symmetric
points with respect to M , and hence X, D, Y are collinear. The proof is thus complete.
50 IMO 2016 Hong Kong
• Method 2. Here, we finish the proof without using the Butterfly Theorem. As DT T1 D1
is an isosceles trapezoid, we have
G3. Let B = (−1, 0) and C = (1, 0) be fixed points on the coordinate plane. A nonempty,
bounded subset S of the plane is said to be nice if
(i) there is a point T in S such that for every point Q in S, the segment T Q lies entirely
in S; and
(ii) for any triangle P1 P2 P3 , there exists a unique point A in S and a permutation σ of the
indices {1, 2, 3} for which triangles ABC and Pσ(1) Pσ(2) Pσ(3) are similar.
Prove that there exist two distinct nice subsets S and S 0 of the set {(x, y) : x > 0, y > 0}
such that if A ∈ S and A0 ∈ S 0 are the unique choices of points in (ii), then the product
BA · BA0 is a constant independent of the triangle P1 P2 P3 .
Solution. If in the similarity of 4ABC and 4Pσ(1) Pσ(2) Pσ(3) , BC corresponds to the longest
side of 4P1 P2 P3 , then we have BC > AB > AC. The condition BC > AB is equivalent to
(x + 1)2 + y 2 6 4, while AB > AC is trivially satisfied for any point in the first quadrant.
Then we first define
Note that S is the intersection of a disk and the first quadrant, so it is bounded and convex,
and we can choose any T ∈ S to meet condition (i). For any point A in S, the relation
BC > AB > AC always holds. Therefore, the point A in (ii) is uniquely determined, while
its existence is guaranteed by the above construction.
T0
S0
S
x
B O C
Next, if in the similarity of 4A0 BC and 4Pσ(1) Pσ(2) Pσ(3) , BC corresponds to the second
longest side of 4P1 P2 P3 , then we have A0 B > BC > A0 C. The two inequalities are equivalent
to (x + 1)2 + y 2 > 4 and (x − 1)2 + y 2 6 4 respectively. Then we define
The boundedness condition is satisfied while (ii) can be argued as in the previous case. For
(i), note that S 0 contains points inside the disk (x − 1)2 + y 2 6 4 and outside the disk
(x + 1)2 + y 2 > 4. This shows we can take T 0 = (1, 2) in (i), which is the topmost point of
the circle (x − 1)2 + y 2 = 4.
It remains to check that the product BA · BA0 is a constant. Suppose we are given a
triangle P1 P2 P3 with P1 P2 > P2 P3 > P3 P1 . By the similarity, we have
P2 P3 P 1 P2
BA = BC · and BA0 = BC · .
P1 P2 P 2 P3
Comment. The original version of this problem includes the condition that the interiors of
S and S 0 are disjoint. We remove this condition since it is hard to define the interior of a
point set rigorously.
Shortlisted problems 53
G4. Let ABC be a triangle with AB = AC 6= BC and let I be its incentre. The line BI
meets AC at D, and the line through D perpendicular to AC meets AI at E. Prove that the
reflection of I in AC lies on the circumcircle of triangle BDE.
Solution 1.
I0
J
C
D
F
A I
E
Let Γ be the circle with centre E passing through B and C. Since ED ⊥ AC, the point
F symmetric to C with respect to D lies on Γ. From ∠DCI = ∠ICB = ∠CBI, the line
DC is a tangent to the circumcircle of triangle IBC. Let J be the symmetric point of I with
respect to D. Using directed lengths, from
the point J also lies on Γ. Let I 0 be the reflection of I in AC. Since IJ and CF bisect each
other, CJF I is a parallelogram. From ∠F I 0 C = ∠CIF = ∠F JC, we find that I 0 lies on Γ.
This gives EI 0 = EB.
Note that AC is the internal angle bisector of ∠BDI 0 . This shows DE is the external
angle bisector of ∠BDI 0 as DE ⊥ AC. Together with EI 0 = EB, it is well-known that E
lies on the circumcircle of triangle BDI 0 .
Solution 2. Let I 0 be the reflection of I in AC and let S be the intersection of I 0 C and AI.
Using directed angles, we let θ = ]ACI = ]ICB = ]CBI. We have
Å
π ã
π
]I 0 SE = ]I 0 CA + ]CAI = θ + + 2θ = 3θ +
2 2
and
π π π π
]I 0 DE = ]I 0 DC + = ]CDI + = ]DCB + ]CBD + = 3θ + .
2 2 2 2
0
This shows I , D, E, S are concyclic.
Next, we find ]I 0 SB = 2]I 0 SE = 6θ and ]I 0 DB = 2]CDI = 6θ. Therefore, I 0 , D, B, S
are concyclic so that I 0 , D, E, B, S lie on the same circle. The result then follows.
54 IMO 2016 Hong Kong
I0
C
D
A S
E I
B
Comment. The point S constructed in Solution 2 may lie on the same side as A of BC.
Also, since S lies on the circumcircle of the non-degenerate triangle BDE, we implicitly know
that S is not an ideal point. Indeed, one can verify that I 0 C and AI are parallel if and only
if triangle ABC is equilateral.
Solution 3. Let I 0 be the reflection of I in AC, and let D0 be the second intersection of AI
and the circumcircle of triangle ABD. Since AD0 bisects ∠BAD, point D0 is the midpoint of
the arc BD and DD0 = BD0 = CD0 . Obviously, A, E, D0 lie on AI in this order.
I0
D C
A D0
E I
We find that ∠ED0 D = ∠AD0 D = ∠ABD = ∠IBC = ∠ICB. Next, since D0 is the
circumcentre of triangle BCD, we have ∠EDD0 = 90◦ − ∠D0 DC = ∠CBD = ∠IBC. The
two relations show that triangles ED0 D and ICB are similar. Therefore, we have
BC BC DD0 BD0
= = = .
CI 0 CI D0 E D0 E
Also, we get
∠BCI 0 = ∠BCA + ∠ACI 0 = ∠BCA + ∠ICA = ∠BCA + ∠DBC = ∠BDA = ∠BD0 E.
These show triangles BCI 0 and BD0 E are similar, and hence triangles BCD0 and BI 0 E are
similar. As BCD0 is isosceles, we obtain BE = I 0 E.
As DE is the external angle bisector of ∠BDI 0 and EI 0 = EB, we know that E lies on
the circumcircle of triangle BDI 0 .
Shortlisted problems 55
Solution 4. Let AI and BI meet the circumcircle of triangle ABC again at A0 and B 0
respectively, and let E 0 be the reflection of E in AC. From
∠ABC ∠BAC ∠ABC
∠B 0 AE 0 = ∠B 0 AD − ∠E 0 AD = − = 90◦ − ∠BAC −
2 2 2
= 90◦ − ∠B 0 DA = ∠B 0 DE 0 ,
points B 0 , A, D, E 0 are concyclic. Then
∠BAC
∠DB 0 E 0 = ∠DAE 0 = = ∠BAA0 = ∠DB 0 A0
2
and hence B 0 , E 0 , A0 are collinear. It is well-known that A0 B 0 is the perpendicular bisector of
CI, so that CE 0 = IE 0 . Let I 0 be the reflection of I in AC. This implies BE = CE = I 0 E.
As DE is the external angle bisector of ∠BDI 0 and EI 0 = EB, we know that E lies on the
circumcircle of triangle BDI 0 .
B0
E0 I 0
C
D
A A0
E I
Solution 5. Let F be the intersection of CI and AB. Clearly, F and D are symmetric with
respect to AI. Let O be the circumcentre of triangle BIF , and let I 0 be the reflection of I in
AC.
I0
C
D
A E I
O
F
B
56 IMO 2016 Hong Kong
From ∠BF O = 90◦ − ∠F IB = 12 ∠BAC = ∠BAI, we get EI//F O. Also, from the
relation ∠OIB = 90◦ − ∠BF I = 90◦ − ∠CDI = ∠I 0 ID, we know that O, I, I 0 are collinear.
Note that ED//OI since both are perpendicular to AC. Then ∠F EI = ∠DEI = ∠OIE.
Together with EI//F O, the quadrilateral EF OI is an isosceles trapezoid. Therefore, we find
that ∠DIE = ∠F IE = ∠OEI so OE//ID. Then DEOI is a parallelogram. Hence, we have
DI 0 = DI = EO, which shows DEOI 0 is an isosceles trapezoid. In addition, ED = OI = OB
and OE//BD imply EOBD is another isosceles trapezoid. In particular, both DEOI 0 and
EOBD are cyclic. This shows B, D, E, I 0 are concyclic.
Solution 6. Let I 0 be the reflection of I in AC. Denote by T and M the projections from I
to sides AB and BC respectively. Since BI is the perpendicular bisector of T M , we have
DT = DM. (1)
Since ∠ADE = ∠AT I = 90◦ and ∠DAE = ∠T AI, we have 4ADE ∼ 4AT I. This shows
AD 0 0
AE
= AT
AI
AT
= AI 0 . Together with ∠DAT = 2∠DAE = ∠EAI , this yields 4DAT ∼ 4EAI .
In particular, we have
DT AT AT
0
= 0
= . (2)
EI AI AI
Obviously, the right-angled triangles AM B and AT I are similar. Then we get
AM AT
= . (3)
AB AI
I0
C
E I
A M
T
B
Shortlisted problems 57
Comment. A stronger version of this problem is to ask the contestants to prove the reflection
of I in AC lies on the circumcircle of triangle BDE if and only if AB = AC. Some of the
above solutions can be modified to prove the converse statement to the original problem. For
example, we borrow some ideas from Solution 2 to establish the converse as follows.
I0
C
D
A S
E I
This means I 0 , C, S are collinear. From this we get ]BSE = ]ESI 0 = ]ESC and hence
AS bisects both ∠BAC and ∠BSC. Clearly, S and A are distinct points. It follows that
4BAS ∼ = 4CAS and thus AB = AC.
As in some of the above solutions, an obvious way to prove the stronger version is to
establish the following equivalence: BE = EI 0 if and only if AB = AC. In addition to the
ideas used in those solutions, one can use trigonometry to express the lengths of BE and EI 0
in terms of the side lengths of triangle ABC and to establish the equivalence.
58 IMO 2016 Hong Kong
G5. Let D be the foot of perpendicular from A to the Euler line (the line passing through the
circumcentre and the orthocentre) of an acute scalene triangle ABC. A circle ω with centre
S passes through A and D, and it intersects sides AB and AC at X and Y respectively. Let
P be the foot of altitude from A to BC, and let M be the midpoint of BC. Prove that the
circumcentre of triangle XSY is equidistant from P and M .
Solution 1. Denote the orthocentre and circumcentre of triangle ABC by H and O respec-
tively. Let Q be the midpoint of AH and N be the nine-point centre of triangle ABC. It is
known that Q lies on the nine-point circle of triangle ABC, N is the midpoint of QM and
that QM is parallel to AO.
Let the perpendicular from S to XY meet line QM at S 0 . Let E be the foot of altitude
from B to side AC. Since Q and S lie on the perpendicular bisector of AD, using directed
angles, we have
π ã Å
]SDQ = ]QAS = ]XAS − ]XAQ = − ]AY X − ]BAP = ]CBA − ]AY X
2
= (]CBA − ]ACB) − ]BCA − ]AY X = ]P EM − (]BCA + ]AY X)
π
= ]P QM − ∠(BC, XY ) = − ∠(S 0 Q, BC) − ∠(BC, XY ) = ]SS 0 Q.
2
This shows D, S 0 , S, Q are concyclic.
E
Q
S
X
Y
D H O
N S 0
B P M C
O1
Let the perpendicular from N to BC intersect line SS 0 at O1 . (Note that the two lines
coincide when S is the midpoint of AO, in which case the result is true since the circumcentre
of triangle XSY must lie on this line.) It suffices to show that O1 is the circumcentre of
triangle XSY since N lies on the perpendicular bisector of P M . From
since SQ//OD and QA//O1 N , we know that D, O1 , S 0 , N are concyclic. Therefore, we get
Next, we show that S and S 0 are symmetric with respect to XY . By the Sine Law, we
have
SS 0 SQ SQ SQ SA
0
= 0
= = = .
sin ∠SQS sin ∠SS Q sin ∠SDQ sin ∠SAQ sin ∠SQA
It follows that
which is twice the distance from S to XY . Note that S and C lie on the same side of the
perpendicular bisector of P M if and only if ∠SAC < ∠OAC if and only if ∠Y XA > ∠CBA.
This shows S and O1 lie on different sides of XY . As S 0 lies on ray SO1 , it follows that S
and S 0 cannot lie on the same side of XY . Therefore, S and S 0 are symmetric with respect
to XY .
Let d be the diameter of the circumcircle of triangle XSY . As SS 0 is twice the distance
2
from S to XY and SX = SY , we have SS 0 = 2 SX d
. It follows from (1) that d = 2SO1 . As
SO1 is the perpendicular bisector of XY , point O1 is the circumcentre of triangle XSY .
Solution 2. Denote the orthocentre and circumcentre of triangle ABC by H and O respec-
tively. Let O1 be the circumcentre of triangle XSY . Consider two other possible positions of
S. We name them S 0 and S 00 and define the analogous points X 0 , Y 0 , O10 , X 00 , Y 00 O100 accordingly.
Note that S, S 0 , S 00 lie on the perpendicular bisector of AD.
As XX 0 and Y Y 0 meet at A and the circumcircles of triangles AXY and AX 0 Y 0 meet at
D, there is a spiral similarity with centre D mapping XY to X 0 Y 0 . We find that
π π
]SXY = − ]Y AX = − ]Y 0 AX 0 = ]S 0 X 0 Y 0
2 2
and similarly ]SY X = ]S 0 Y 0 X 0 . This shows triangles SXY and S 0 X 0 Y 0 are directly similar.
Then the spiral similarity with centre D takes points S, X, Y, O1 to S 0 , X 0 , Y 0 , O10 . Similarly,
there is a spiral similarity with centre D mapping S, X, Y, O1 to S 00 , X 00 , Y 00 , O100 . From these,
we see that there is a spiral similarity taking the corresponding points S, S 0 , S 00 to points
O1 , O10 , O100 . In particular, O1 , O10 , O100 are collinear.
60 IMO 2016 Hong Kong
A
Y 00
S 00 S0 Y0
O100
X S
X0 H Y
X 00 D O
O10
B P M C
O1
It now suffices to show that O1 lies on the perpendicular bisector of P M for two special
cases.
Firstly, we take S to be the midpoint of AH. Then X and Y are the feet of altitudes from
C and B respectively. It is well-known that the circumcircle of triangle XSY is the nine-point
circle of triangle ABC. Then O1 is the nine-point centre and O1 P = O1 M . Indeed, P and
M also lies on the nine-point circle.
Secondly, we take S 0 to be the midpoint of AO. Then X 0 and Y 0 are the midpoints of
AB and AC respectively. Then X 0 Y 0 //BC. Clearly, S 0 lies on the perpendicular bisector
of P M . This shows the perpendicular bisectors of X 0 Y 0 and P M coincide. Hence, we must
have O10 P = O10 M .
A
A
S S0
Y
X0 Y0
O10
X O
H O1 O
H
B P M C B P M C
Shortlisted problems 61
G6. Let ABCD be a convex quadrilateral with ∠ABC = ∠ADC < 90◦ . The internal
angle bisectors of ∠ABC and ∠ADC meet AC at E and F respectively, and meet each
other at point P . Let M be the midpoint of AC and let ω be the circumcircle of triangle
BP D. Segments BM and DM intersect ω again at X and Y respectively. Denote by Q the
intersection point of lines XE and Y F . Prove that P Q ⊥ AC.
Solution 1.
B0
Y
S
Q
P
A E
F
X M
C
Z B
Let ω1 be the circumcircle of triangle ABC. We first prove that Y lies on ω1 . Let Y 0 be
the point on ray M D such that M Y 0 · M D = M A2 . Then triangles M AY 0 and M DA are
oppositely similar. Since M C 2 = M A2 = M Y 0 · M D, triangles M CY 0 and M DC are also
oppositely similar. Therefore, using directed angles, we have
]AY 0 C = ]AY 0 M + ]M Y 0 C = ]M AD + ]DCM = ]CDA = ]ABC
so that Y 0 lies on ω1 .
Let Z be the intersection point of lines BC and AD. Since ]P DZ = ]P BC = ]P BZ,
point Z lies on ω. In addition, from ]Y 0 BZ = ]Y 0 BC = ]Y 0 AC = ]Y 0 AM = ]Y 0 DZ, we
also know that Y 0 lies on ω. Note that ∠ADC is acute implies M A 6= M D so M Y 0 6= M D.
Therefore, Y 0 is the second intersection of DM and ω. Then Y 0 = Y and hence Y lies on ω1 .
Next, by the Angle Bisector Theorem and the similar triangles, we have
FA AD AD CM YA YM YA
= = · = · = .
FC CD AM CD YM YC YC
Hence, F Y is the internal angle bisector of ∠AY C.
Let B 0 be the second intersection of the internal angle bisector of ∠CBA and ω1 . Then
B 0 is the midpoint of arc AC not containing B. Therefore, Y B 0 is the external angle bisector
of ∠AY C, so that B 0 Y ⊥ F Y .
62 IMO 2016 Hong Kong
Denote by l the line through P parallel to AC. Suppose l meets line B 0 Y at S. From
the point S lies on ω. Similarly, the line through X perpendicular to XE also passes through
the second intersection of l and ω, which is the point S. From QY ⊥ Y S and QX ⊥ XS,
point Q lies on ω and QS is a diameter of ω. Therefore, P Q ⊥ P S so that P Q ⊥ AC.
Solution 2. Denote by ω1 and ω2 the circumcircles of triangles ABC and ADC respectively.
Since ∠ABC = ∠ADC, we know that ω1 and ω2 are symmetric with respect to the midpoint
M of AC.
Firstly, we show that X lies on ω2 . Let X1 be the second intersection of ray M B and
ω2 and X 0 be its symmetric point with respect to M . Then X 0 lies on ω1 and X 0 AX1 C is a
parallelogram. Hence, we have
X0
M2 B0 M D0 M1
F
Y
Q P
E X
D
B
A
Also, we have
These yield ]DX1 B = ]DP B and hence X1 lies on ω. It follows that X1 = X and X lies
on ω2 . Similarly, Y lies on ω1 .
Shortlisted problems 63
Y0 N0 W 0 Z0
This implies
tan γ − tan α sin γ cos α − cos γ sin α sin α cos β − cos α sin β tan α − tan β
= = = .
tan γ + tan α sin γ cos α + cos γ sin α sin α cos β + cos α sin β tan α + tan β
Expanding and simplifying, we get the desired result tan2 α = tan β tan γ.
Proof.
B0
A0
E0
M0 F0
O0 F1
D0
C0
Let O0 be the centre of Γ and let Γ0 be the circle with centre M 0 passing through E 0 . Let
F1 be the inversion image of F 0 with respect to Γ. It is well-known that E 0 lies on the polar
of F 0 with respect to Γ. This shows E 0 F1 ⊥ O0 F 0 and hence F1 lies on Γ0 . It follows that the
inversion image of Γ0 with respect to Γ is Γ0 itself. This shows Γ0 is orthogonal to Γ, and thus
the power of M 0 with respect to Γ is the square of radius of Γ0 , which is (M 0 E 0 )2 .
We return to the main problem. Let Z be the intersection of lines AD and BC, and W
be the intersection of lines AB and CD. Since ]P DZ = ]P BC = ]P BZ, point Z lies on
ω. Similarly, W lies on ω. Applying Claim 2 to the cyclic quadrilateral ZBDW , we know
that the power of M with respect to ω is M A2 . Hence, M X · M B = M A2 .
Suppose the line through B perpendicular to BE meets line AC at T . Then BE and
BT are the angle bisectors of ∠CBA. This shows (T, E; A, C) is harmonic. Thus, we have
M E · M T = M A2 = M X · M B. It follows that E, T, B, X are concyclic.
Shortlisted problems 65
Q
P
T
A
E
F
0 0 M
P ,Q
X
Z B C
The result is trivial for the special case AD = CD since P, Q lie on the perpendicular
bisector of AC in that case. Similarly, the case AB = CB is trivial. It remains to consider
the general cases where we can apply Claim 1 in the latter part of the proof.
Let the projections from P and Q to AC be P 0 and Q0 respectively. Then P Q ⊥ AC if
0 0
and only if P 0 = Q0 if and only if EP
FP0
= EQ
F Q0
in terms of directed lengths. Note that
G7. Let I be the incentre of a non-equilateral triangle ABC, IA be the A-excentre, IA0 be
the reflection of IA in BC, and lA be the reflection of line AIA0 in AI. Define points IB , IB0
and line lB analogously. Let P be the intersection point of lA and lB .
(a) Prove that P lies on line OI where O is the circumcentre of triangle ABC.
(b) Let one of the tangents from P to the incircle of triangle ABC meet the circumcircle at
points X and Y . Show that ∠XIY = 120◦ .
Solution 1.
(a) Let A0 be the reflection of A in BC and let M be the second intersection of line AI
and the circumcircle Γ of triangle ABC. As triangles ABA0 and AOC are isosceles with
∠ABA0 = 2∠ABC = ∠AOC, they are similar to each other. Also, triangles ABIA and
AIC are similar. Therefore we have
AA0 AA0 AB AC AI AI
= · = · = .
AIA AB AIA AO AC AO
Together with ∠A0 AIA = ∠IAO, we find that triangles AA0 IA and AIO are similar.
IA0
A Y
D
X T
O
I
P
B C
Z
M
A0
IA
Denote by P 0 the intersection of line AP and line OI. Using directed angles, we have
IA0 IB
A
P X T D
Y
I O
B C
IB0
A0
IA
√
Consider the inversion with centre A and radius AB · AC followed by the reflection
in AI. Then B and C are mapped to each other, and I and IA are mapped to each other.
68 IMO 2016 Hong Kong
(b) Denote by R and r the circumradius and inradius of triangle ABC. Note that by the
above transformation, we have 4AP O ∼ 4AA0 IA0 and 4AA0 IA ∼ 4AIO. Therefore, we
find that
AO AO AIA AO
P O = A0 IA0 · 0
= AIA · 0 = 0 · AO = · AO.
AIA A IA A IA IO
This shows P O · IO = R2 , and it follows that P and I are mapped to each other under
the inversion with respect to the circumcircle Γ of triangle ABC. Then P X · P Y , which
is the power of P with respect to Γ, equals P I · P O. This yields X, I, O, Y are concyclic.
Let T be the touching point of the incircle to XY , and let D be the midpoint of XY .
Then
PO PO R2 R2 R
OD = IT · =r· =r· 2 2
= r · = .
PI P O − IO R − IO 2Rr 2
◦ ◦
This shows ∠DOX = 60 and hence ∠XIY = ∠XOY = 120 .
Comment. A simplification of this problem is to ask part (a) only. Note that the question in
part (b) implicitly requires P to lie on OI, or otherwise the angle is not uniquely determined
as we can find another tangent from P to the incircle.
Shortlisted problems 69
G8. Let A1 , B1 and C1 be points on sides BC, CA and AB of an acute triangle ABC
respectively, such that AA1 , BB1 and CC1 are the internal angle bisectors of triangle ABC.
Let I be the incentre of triangle ABC, and H be the orthocentre of triangle A1 B1 C1 . Show
that
AH + BH + CH > AI + BI + CI.
Solution. Without loss of generality, assume α = ∠BAC 6 β = ∠CBA 6 γ = ∠ACB.
Denote by a, b, c the lengths of BC, CA, AB respectively. We first show that triangle A1 B1 C1
is acute.
Choose points D and E on side BC such that B1 D//AB and B1 E is the internal angle
bisector of ∠BB1 C. As ∠B1 DB = 180◦ − β is obtuse, we have BB1 > B1 D. Thus,
BE BB1 DB1 BA BA1
= > = = .
EC B1 C B1 C AC A1 C
Therefore, BE > BA1 and 21 ∠BB1 C = ∠BB1 E > ∠BB1 A1 . Similarly, 21 ∠BB1 A > ∠BB1 C1 .
It follows that
1
∠A1 B1 C1 = ∠BB1 A1 + ∠BB1 C1 < (∠BB1 C + ∠BB1 A) = 90◦
2
is acute. By symmetry, triangle A1 B1 C1 is acute.
Let BB1 meet A1 C1 at F . From α 6 γ, we get a 6 c, which implies
ca ac
BA1 = 6 = BC1
b+c a+b
and hence ∠BC1 A1 6 ∠BA1 C1 . As BF is the internal angle bisector of ∠A1 BC1 , this shows
∠B1 F C1 = ∠BF A1 6 90◦ . Hence, H lies on the same side of BB1 as C1 . This shows H lies
inside triangle BB1 C1 . Similarly, from α 6 β and β 6 γ, we know that H lies inside triangles
CC1 B1 and AA1 C1 .
B0 60◦
H0
I0
C1 B1
H
I
F
B A1 DE C
70 IMO 2016 Hong Kong
AI + BI + CI = I 0 I + B 0 I 0 + IC = B 0 I 0 + I 0 I + IC. (1)
Similarly,
AH + BH + CH = H 0 H + B 0 H 0 + HC = B 0 H 0 + H 0 H + HC. (2)
As ∠AII 0 = ∠AI 0 I = 60◦ , ∠AI 0 B 0 = ∠AIB > 120◦ and ∠AIC > 120◦ , the quadrilateral
B 0 I 0 IC is convex and lies on the same side of B 0 C as A.
Next, since H lies inside triangle ACC1 , H lies outside B 0 I 0 IC. Also, H lying inside
triangle ABI implies H 0 lies inside triangle AB 0 I 0 . This shows H 0 lies outside B 0 I 0 IC and
hence the convex quadrilateral B 0 I 0 IC is contained inside the quadrilateral B 0 H 0 HC. It
follows that the perimeter of B 0 I 0 IC cannot exceed the perimeter of B 0 H 0 HC. From (1) and
(2), we conclude that
AH + BH + CH > AI + BI + CI.
For the case ∠AIC < 120◦ , we can rotate B, I, H through 60◦ about C to B 0 , I 0 , H 0 so
that B 0 and A lie on different sides of BC. The proof is analogous to the previous case and
we still get the desired inequality.
Shortlisted problems 71
Number Theory
N1. For any positive integer k, denote the sum of digits of k in its decimal representation by
S(k). Find all polynomials P (x) with integer coefficients such that for any positive integer
n > 2016, the integer P (n) is positive and
Answer.
• P (x) = c where 1 6 c 6 9 is an integer; or
• P (x) = x.
Solution 1. We consider three cases according to the degree of P .
• Case 1. P (x) is a constant polynomial.
Let P (x) = c where c is an integer constant. Then (1) becomes S(c) = c. This holds if
and only if 1 6 c 6 9.
• Case 2. deg P = 1.
We have the following observation. For any positive integers m, n, we have
These give 5a 6 S(5a). As a > 1, this holds only when a = 1, in which case (1) reduces to
S(n + b) = S(n) + b for all n > 2016. Then we find that
If b > 0, we choose n such that n + 1 + b = 10k for some sufficiently large k. Note that all
the digits of n + b are 9’s, so that the left-hand side of (3) equals 1 − 9k. As n is a positive
integer less than 10k − 1, we have S(n) < 9k. Therefore, the right-hand side of (3) is at least
1 − (9k − 1) = 2 − 9k, which is a contradiction.
The case b < 0 can be handled similarly by considering n + 1 to be a large power of 10.
Therefore, we conclude that P (x) = x, in which case (1) is trivially satisfied.
72 IMO 2016 Hong Kong
Solution 2. Let P (x) = ad xd + ad−1 xd−1 + · · · + a0 . Clearly ad > 0. There exists an integer
m > 1 such that |ai | < 10m for all 0 6 i 6 d. Consider n = 9 × 10k for a sufficiently large
integer k in (1). If there exists an index 0 6 i 6 d − 1 such that ai < 0, then all digits of P (n)
in positions from 10ik+m+1 to 10(i+1)k−1 are all 9’s. Hence, we have S(P (n)) > 9(k − m − 1).
On the other hand, P (S(n)) = P (9) is a fixed constant. Therefore, (1) cannot hold for large
k. This shows ai > 0 for all 0 6 i 6 d − 1.
Hence, P (n) is an integer formed by the nonnegative integers ad × 9d , ad−1 × 9d−1 , . . . , a0
by inserting some zeros in between. This yields
As S(m) 6 m for any positive integer m, with equality when 1 6 m 6 9, this forces each
ai × 9i to be a positive integer between 1 and 9. In particular, this shows ai = 0 for i > 2
and hence d 6 1. Also, we have a1 6 1 and a0 6 9. If a1 = 1 and 1 6 a0 6 9, we take
n = 10k + (10 − a0 ) for sufficiently large k in (1). This yields a contradiction since
The zero polynomial is also rejected since P (n) is positive for large n. The remaining candi-
dates are P (x) = x or P (x) = a0 where 1 6 a0 6 9, all of which satisfy (1), and hence are
the only solutions.
Shortlisted problems 73
N2. Let τ (n) be the number of positive divisors of n. Let τ1 (n) be the number of positive
divisors of n which have remainders 1 when divided by 3. Find all possible integral values of
the fraction ττ1(10n)
(10n)
.
Solution. In this solution, we always use pi to denote primes congruent to 1 mod 3, and use
qj to denote primes congruent to 2 mod 3. When we express a positive integer m using its
prime factorization, we also include the special case m = 1 by allowing the exponents to be
zeros. We first compute τ1 (m) for a positive integer m.
• Claim. Let m = 3x pa11 pa22 · · · pas s q1b1 q2b2 · · · qtbt be the prime factorization of m. Then
s t
Y 1Y
τ1 (m) = (ai + 1) (bj + 1)
. (1)
i=1
2 j=1
Proof. To choose a divisor of m congruent to 1 mod 3, it cannot have the prime divisor 3,
while there is no restriction on choosing prime factors congruent to 1 mod 3. Also, we have
to choose an even number of prime factors (counted with multiplicity) congruent to 2 mod 3.
If tj=1 (bj + 1) is even, then we may assume without loss of generality b1 + 1 is even. We
Q
can choose the prime factors q2 , q3 , . . . , qt freely in tj=2 (bj + 1) ways. Then the parity of
Q
the number of q1 is uniquely determined, and hence there are 12 (b1 + 1) ways to choose the
exponent of q1 . Hence (1) is verified in this case.
If tj=1 (bj + 1) is odd, we use induction on t to count the number of choices. When
Q
t = 1, there are d b12+1 e choices for which the exponent is even and b b12+1 c choices for which
the exponent is odd. For the inductive step, we find that there are
¢ ú
1 t−1 1 t−1 t
• ü
bt + 1 bt + 1 1Y
Y Y
(bj + 1) ·
+ (bj + 1) ·
=
(bj + 1)
2 j=1 2 2 j=1 2 2 j=1
choices with an even number of prime factors and hence b 12 tj=1 (bj + 1)c choices with an odd
Q
Let n = 3x 2y 5z pa11 pa22 · · · pas s q1b1 q2b2 · · · qtbt . Using the well-known formula for computing the
divisor function, we get
s
Y t
Y
τ (10n) = (x + 1)(y + 2)(z + 2) (ai + 1) (bj + 1). (2)
i=1 j=1
Qt
If c = (y + 2)(z + 2) j=1 (bj + 1) is even, then (2) and (3) imply
τ (10n)
= 2(x + 1).
τ1 (10n)
For this to be an integer, we need c + 1 divides 2(x + 1) since c and c + 1 are relatively prime.
Let 2(x + 1) = k(c + 1). Then (4) reduces to
t
τ (10n) Y
= kc = k(y + 2)(z + 2) (bj + 1). (5)
τ1 (10n) j=1
Noting that y, z are odd, the integers y + 2 and z + 2 are at least 3. This shows the integer
in this case must be composite. On the other hand, for any odd composite number ab with
ab−1
a, b > 3, we may simply take n = 3 2 · 2a−2 · 5b−2 so that ττ1(10n)
(10n)
= ab from (5).
We conclude that the fraction can be any even integer or any odd composite number.
Equivalently, it can be 2 or any composite number.
Shortlisted problems 75
N3. Define P (n) = n2 + n + 1. For any positive integers a and b, the set
{P (a), P (a + 1), P (a + 2), . . . , P (a + b)}
is said to be fragrant if none of its elements is relatively prime to the product of the other
elements. Determine the smallest size of a fragrant set.
Answer. 6.
Solution. We have the following observations.
(i) (P (n), P (n + 1)) = 1 for any n.
We have (P (n), P (n + 1)) = (n2 + n + 1, n2 + 3n + 3) = (n2 + n + 1, 2n + 2). Noting
that n2 + n + 1 is odd and (n2 + n + 1, n + 1) = (1, n + 1) = 1, the claim follows.
(ii) (P (n), P (n + 2)) = 1 for n 6≡ 2 (mod 7) and (P (n), P (n + 2)) = 7 for n ≡ 2 (mod 7).
From (2n+7)P (n)−(2n−1)P (n+2) = 14 and the fact that P (n) is odd, (P (n), P (n+2))
must be a divisor of 7. The claim follows by checking n ≡ 0, 1, . . . , 6 (mod 7) directly.
(iii) (P (n), P (n + 3)) = 1 for n 6≡ 1 (mod 3) and 3|(P (n), P (n + 3)) for n ≡ 1 (mod 3).
From (n + 5)P (n) − (n − 1)P (n + 3) = 18 and the fact that P (n) is odd, (P (n), P (n + 3))
must be a divisor of 9. The claim follows by checking n ≡ 0, 1, 2 (mod 3) directly.
Suppose there exists a fragrant set with at most 5 elements. We may assume it contains
exactly 5 elements P (a), P (a + 1), . . . , P (a + 4) since the following argument also works with
fewer elements. Consider P (a + 2). From (i), it is relatively prime to P (a + 1) and P (a + 3).
Without loss of generality, assume (P (a), P (a + 2)) > 1. From (ii), we have a ≡ 2 (mod 7).
The same observation implies (P (a + 1), P (a + 3)) = 1. In order that the set is fragrant,
(P (a), P (a + 3)) and (P (a + 1), P (a + 4)) must both be greater than 1. From (iii), this holds
only when both a and a + 1 are congruent to 1 mod 3, which is a contradiction.
It now suffices to construct a fragrant set of size 6. By the Chinese Remainder Theorem,
we can take a positive integer a such that
For example, we may take a = 197. From (ii), both P (a + 1) and P (a + 3) are divisible
by 7. From (iii), both P (a + 2) and P (a + 5) are divisible by 3. One also checks from
19|P (7) = 57 and 19|P (11) = 133 that P (a) and P (a + 4) are divisible by 19. Therefore, the
set {P (a), P (a + 1), . . . , P (a + 5)} is fragrant.
Therefore, the smallest size of a fragrant set is 6.
Comment. “Fragrant Harbour” is the English translation of “Hong Kong”.
A stronger version of this problem is to show that there exists a fragrant set of size k for
any k > 6. We present a proof here.
For each even positive integer m which is not divisible by 3, since m2 + 3 ≡ 3 (mod 4),
we can find a prime pm ≡ 3 (mod 4) such that pm |m2 + 3. Clearly, pm > 3.
76 IMO 2016 Hong Kong
N4. Let n, m, k and l be positive integers with n 6= 1 such that nk + mnl + 1 divides nk+l − 1.
Prove that
• m = 1 and l = 2k; or
nk−l −1
• l|k and m = nl −1
.
• Case 1. l > k.
Since (nk + mnl + 1, n) = 1, (2) yields
• Case 2. l < k.
This time (2) yields
nk + mnl + 1|nk + nk−l + m.
In particular, we get nk + mnl + 1 6 nk + nk−l + m, which implies
nk−l − 1
m6 . (3)
nl − 1
On the other hand, from (1) we may let nk+l − 1 = (nk + mnl + 1)t for some positive
integer t. Obviously, t is less than nl , which means t 6 nl − 1 as it is an integer. Then we
have nk+l − 1 6 (nk + mnl + 1)(nl − 1), which is the same as
nk−l − 1
m> . (4)
nl − 1
k−l
Equations (3) and (4) combine to give m = nnl −1−1 . As this is an integer, we have l|k − l.
This means l|k and it corresponds to the second result.
78 IMO 2016 Hong Kong
• Case 1. l > k.
Then (2) yields
nk + mnl + 1|nl + mnl−k + 1.
Since 2(nk +mnl +1) > 2mnl +1 > nl +mnl−k +1, it follows that nk +mnl +1 = nl +mnl−k +1,
that is,
m(nl − nl−k ) = nl − nk .
If m > 2, then m(nl − nl−k ) > 2nl − 2nl−k > 2nl − nl > nl − nk gives a contradiction. Hence
m = 1 and l − k = k, which means m = 1 and l = 2k.
• Case 2. l < k.
Then (2) yields
nk + mnl + 1|nk + nk−l + m.
Since 2(nk + mnl + 1) > 2nk + m > nk + nk−l + m, it follows that nk + mnl + 1 = nk + nk−l + m.
k−l
This gives m = nnl −1−1 . Note that nl − 1|nk−l − 1 implies l|k − l and hence l|k. The proof is
thus complete.
Comment. Another version of this problem is as follows: let n, m, k and l be positive integers
with n 6= 1 such that k and l do not divide each other. Show that nk + mnl + 1 does not
divide nk+l − 1.
Shortlisted problems 79
N5. Let a be a positive integer which is not a square number. Denote by A the set of all
positive integers k such that
x2 − a
k= (1)
x2 − y 2
√
for some integers x and y with x > a. Denote by B the set √ of all positive integers k such
that (1) is satisfied for some integers x and y with 0 6 x < a. Prove that A = B.
• Claim. For fixed k, let x, y be integers satisfying (1). Then the numbers x1 , y1 defined by
(x − y)2 − 4a (x − y)2 − 4a
Ç å Ç å
1 1
x1 = x−y+ , y1 = x−y−
2 x+y 2 x+y
x2 − xy − 2a 2(x2 − a)
x1 = = −x + = −x + 2k(x − y),
x+y x+y
both x1 and y1 are integers. Let u = x + y and v = x − y. The relation (1) can be rewritten
as
u2 − (4k − 2)uv + (v 2 − 4a) = 0.
v 2 −4a
By Vieta’s Theorem, the number z = u
satisfies
Since x1 and y1 are defined so that v = x1 + y1 and z = x1 − y1 , we can reverse the process
and verify (1) for x1 , y1 .
(x − y)2 − 4a (x − y)2 − 4a
Ç å
1 1
x1 = x − y + , y1 = x−y− .
2 x+y 2 x+y
(x − y)2 − 4a 2a + x(y − x) √
Ç å
1 2a
x1 > − x−y+ = > > a.
2 x+y x+y x+y
√ show that A ⊂ B. Take any k ∈ A so that (1) is satisfied for some integers
Next, we shall
x, y with x > a. Again, we may assume y is positive. Among all such representations of k,
we choose the one with smallest x + y. Define
(x − y)2 − 4a (x − y)2 − 4a
Ç å
1 1
x1 = x − y + , y1 = x−y− .
2 x+y 2 x+y
√
By the Claim, x1 , y1 are integers satisfying (1). Since k > 1, we get x > y > a. Therefore,
4a 4a
we have y1 > x+y > 0 and x+y < x + y. It follows that
4a − (x − y)2
® ´
x1 + y1 6 max x − y, < x + y.
x+y
√
If x1 > √a, we get a contradiction due to the minimality of x + y. Therefore, we must have
0 6 x1 < a, which means k ∈ B so that A ⊂ B.
The two subset relations combine to give A = B.
Solution 2. The relation (1) is equivalent to
ky 2 − (k − 1)x2 = a. (2)
Motivated by Pell’s Equation, we prove the following, which is essentially the same as the
Claim in Solution 1.
• Claim. If (x0 , y0 ) is a solution to (2), then ((2k − 1)x0 ± 2ky0 , (2k − 1)y0 ± 2(k − 1)x0 ) is
also a solution to (2).
Proof. We check directly that
√
If (2) is satisfied for some 0 6 x < a and nonnegative integer y, then clearly (1) implies
y > x. Also, we have k > 1 since a is not a square number. By the Claim, consider another
solution to (2) defined by
It satisfies x1 > (2k − 1)x + 2k(x + 1) = (4k − 1)x + 2k > x. Then we can replace the old
solution by a new one which has √ a larger value in x. After a finite number of replacements,
we must get a solution with x > √ a. This shows B ⊂ A.
If (2) is satisfied for some x > a and nonnegative integer y, by the Claim we consider
another solution to (2) defined by
√ √ »
From (2), we get ky > k − 1x. This implies ky > k(k − 1)x > (k − 1)x and hence
(2k − 1)x − 2ky < x. On the other hand, the relation (1) implies x > y. Then it is clear that
(2k − 1)x − 2ky > −x. These combine to give x1 < x, which means we have found a solution
to (2) with x having a smaller
√ absolute value. After a finite number of steps, we shall obtain
a solution with 0 6 x < a. This shows A ⊂ B.
The desired result follows from B ⊂ A and A ⊂ B.
u2 + (2v − 4kv)u + v 2 − 4a = 0.
N6. Denote by N the set of all positive integers. Find all functions f : N → N such that
for all positive integers m and n, the integer f (m) + f (n) − mn is nonzero and divides
mf (m) + nf (n).
Answer. f (n) = n2 for any n ∈ N.
Solution. It is given that
f (m) + f (n) − mn|mf (m) + nf (n). (1)
Taking m = n = 1 in (1), we have 2f (1) − 1|2f (1). Then 2f (1) − 1|2f (1) − (2f (1) − 1) = 1
and hence f (1) = 1.
Let p > 7 be a prime. Taking m = p and n = 1 in (1), we have f (p) − p + 1|pf (p) + 1 and
hence
f (p) − p + 1|pf (p) + 1 − p(f (p) − p + 1) = p2 − p + 1.
If f (p) − p + 1 = p2 − p + 1, then f (p) = p2 . If f (p) − p + 1 6= p2 − p + 1, as p2 − p + 1 is an
odd positive integer, we have p2 − p + 1 > 3(f (p) − p + 1), that is,
1
f (p) 6 (p2 + 2p − 2). (2)
3
Taking m = n = p in (1), we have 2f (p) − p2 |2pf (p). This implies
2f (p) − p2 |2pf (p) − p(2f (p) − p2 ) = p3 .
By (2) and f (p) > 1, we get
2
−p2 < 2f (p) − p2 6 (p2 + 2p − 2) − p2 < −p
3
since p > 7. This contradicts the fact that 2f (p) − p2 is a factor of p3 . Thus we have proved
that f (p) = p2 for all primes p > 7.
Let n be a fixed positive integer. Choose a sufficiently large prime p. Consider m = p in
(1). We obtain
f (p) + f (n) − pn|pf (p) + nf (n) − n(f (p) + f (n) − pn) = pf (p) − nf (p) + pn2 .
As f (p) = p2 , this implies p2 −pn+f (n)|p(p2 −pn+n2 ). As p is sufficiently large and n is fixed,
p cannot divide f (n), and so (p, p2 − pn + f (n)) = 1. It follows that p2 − pn + f (n)|p2 − pn + n2
and hence
p2 − pn + f (n)|(p2 − pn + n2 ) − (p2 − pn + f (n)) = n2 − f (n).
Note that n2 − f (n) is fixed while p2 − pn + f (n) is chosen to be sufficiently large. Therefore,
we must have n2 − f (n) = 0 so that f (n) = n2 for any positive integer n.
Finally, we check that when f (n) = n2 for any positive integer n, then
f (m) + f (n) − mn = m2 + n2 − mn
and
mf (m) + nf (n) = m3 + n3 = (m + n)(m2 + n2 − mn).
The latter expression is divisible by the former for any positive integers m, n. This shows
f (n) = n2 is the only solution.
Shortlisted problems 83
N7. Let n be an odd positive integer. In the Cartesian plane, a cyclic polygon P with area
S is chosen. All its vertices have integral coordinates, and the squares of its side lengths are
all divisible by n. Prove that 2S is an integer divisible by n.
Solution. Let P = A1 A2 . . . Ak and let Ak+i = Ai for i > 1. By the Shoelace Formula, the
area of any convex polygon with integral coordinates is half an integer. Therefore, 2S is an
integer. We shall prove by induction on k > 3 that 2S is divisible by n. Clearly, it suffices to
consider n = pt where p is an odd prime and t > 1. √ √ √
For the base case k = 3, let the side lengths of P be na, nb, nc where a, b, c are
positive integers. By Heron’s Formula,
16S 2 = n2 (2ab + 2bc + 2ca − a2 − b2 − c2 ).
This shows 16S 2 is divisible by n2 . Since n is odd, 2S is divisible by n.
Assume k > 4. If the square of length of one of the diagonals is divisible by n, then
that diagonal divides P into two smaller polygons, to which the induction hypothesis applies.
Hence we may assume that none of the squares of diagonal lengths is divisible by n. As
usual, we denote by νp (r) the exponent of p in the prime decomposition of r. We claim the
following.
• Claim. νp (A1 A2m ) > νp (A1 A2m+1 ) for 2 6 m 6 k − 1.
Proof. The case m = 2 is obvious since νp (A1 A22 ) > pt > νp (A1 A23 ) by the condition and the
above assumption.
Suppose νp (A1 A22 ) > νp (A1 A23 ) > · · · > νp (A1 A2m ) where 3 6 m 6 k − 1. For the induction
step, we apply Ptolemy’s Theorem to the cyclic quadrilateral A1 Am−1 Am Am+1 to get
A1 Am+1 × Am−1 Am + A1 Am−1 × Am Am+1 = A1 Am × Am−1 Am+1 ,
which can be rewritten as
A1 A2m+1 × Am−1 A2m = A1 A2m−1 × Am A2m+1 + A1 A2m × Am−1 A2m+1
− 2A1 Am−1 × Am Am+1 × A1 Am × Am−1 Am+1 . (1)
From this, 2A1 Am−1 ×Am Am+1 ×A1 Am ×Am−1 Am+1 is an integer. We consider the component
of p of each term in (1). By the inductive hypothesis, we have νp (A1 A2m−1 ) > νp (A1 A2m ). Also,
we have νp (Am A2m+1 ) > pt > νp (Am−1 A2m+1 ). These give
νp (A1 A2m−1 × Am A2m+1 ) > νp (A1 A2m × Am−1 A2m+1 ). (2)
Next, we have νp (4A1 A2m−1 × Am A2m+1 × A1 A2m × Am−1 A2m+1 ) = νp (A1 A2m−1 × Am A2m+1 ) +
νp (A1 A2m × Am−1 A2m+1 ) > 2νp (A1 A2m × Am−1 A2m+1 ) from (2). This implies
νp (2A1 Am−1 × Am Am+1 × A1 Am × Am−1 Am+1 ) > νp (A1 A2m × Am−1 A2m+1 ). (3)
Combining (1), (2) and (3), we conclude that
νp (A1 A2m+1 × Am−1 A2m ) = νp (A1 A2m × Am−1 A2m+1 ).
By νp (Am−1 A2m ) > pt > νp (Am−1 A2m+1 ), we get νp (A1 A2m+1 ) < νp (A1 A2m ). The Claim follows
by induction.
84 IMO 2016 Hong Kong
pt > νp (A1 A23 ) > νp (A1 A24 ) > · · · > νp (A1 A2k ) > pt ,
N8. Find all polynomials P (x) of odd degree d and with integer coefficients satisfying the
following property: for each positive integer n, there exist n positive integers x1 , x2 , . . . , xn
such that 21 < PP (x
(xi )
j)
< 2 and PP (x
(xi )
j)
is the d-th power of a rational number for every pair of
indices i and j with 1 6 i, j 6 n.
Answer. P (x) = a(rx + s)d where a, r, s are integers with a 6= 0, r > 1 and (r, s) = 1.
Solution. Let P (x) = ad xd + ad−1 xd−1 + · · · + a0 . Consider the substitution y = dad x + ad−1 .
By defining Q(y) = P (x), we find that Q is a polynomial with rational coefficients without
the term y d−1 . Let Q(y) = bd y d + bd−2 y d−2 + bd−3 y d−3 + · · · + b0 and B = max06i6d {|bi |}
(where bd−1 = 0).
The condition shows that for each n > 1, there exist integers y1 , y2 , . . . , yn such that
1 Q(yi ) Q(yi )
2
< Q(yj )
< 2 and Q(y j)
is the d-th power of a rational number for 1 6 i, j 6 n. Since n
can be arbitrarily large, we may assume all xi ’s and hence yi ’s are integers larger than some
absolute constant in the following.
By Dirichlet’s Theorem, since d is odd, we can find a sufficiently large prime p such that
p ≡ 2 (mod d). In particular, we have (p − 1, d) = 1. For this fixed p, we choose n to be
sufficiently large. Then by the Pigeonhole Principle, there must be d+1 of y1 , y2 , . . . , yn which
are congruent mod p. Without loss of generality, assume yi ≡ yj (mod p) for 1 6 i, j 6 d + 1.
We shall establish the following.
Q(yi ) yid
• Claim. Q(y1 )
= y1d
for 2 6 i 6 d + 1.
Q(yi ) ld
Proof. Let Q(y1 )
= md
where (l, m) = 1 and l, m > 0. This can be rewritten in the expanded
form
d−2
bd (md yid ld y1d ) bj (md yij − ld y1j ).
X
− =− (1)
j=0
Let c be the common denominator of Q, so that cQ(k) is an integer for any integer k.
Note that c depends only on P and so we may assume (p, c) = 1. Then y1 ≡ yi (mod p)
implies cQ(y1 ) ≡ cQ(yi ) (mod p).
• Case 1. p|cQ(y1 ).
cQ(yi )
In this case, there is a cancellation of p in the numerator and denominator of cQ(y1 )
, so
that md 6 p−1 |cQ(y1 )|. Noting |Q(y1 )| < 2By1d as y1 is large, we get
1 1
m 6 p− d (2cB) d y1 . (2)
1 Q(yi )
For large y1 and yi , the relation 2
< Q(y1 )
< 2 implies
1 yd
< id < 3. (3)
3 y1
We also have
1 ld
< d < 2. (4)
2 m
86 IMO 2016 Hong Kong
Suppose on the contrary that myi − ly1 6= 0. Then the absolute value of the above expression
is at least |bd |md−1 yid−1 . On the other hand, the absolute value of the right-hand side of (1)
is at most
d−2
B(md yij + ld y1j ) 6 (d − 1)B(md yid−2 + ld y1d−2 )
X
j=0
6 (d − 1)B(7md yid−2 )
1 1
6 7(d − 1)B(p− d (2cB) d y1 )md−1 yid−2
1 1
6 21(d − 1)Bp− d (2cB) d md−1 yid−1
by using successively (3), (4), (2) and again (3). This shows
1 1
|bd |md−1 yid−1 6 21(d − 1)Bp− d (2cB) d md−1 yid−1 ,
which must be smaller than |bd |pmd−1 yid−1 for large p. Again this yields a contradiction and
hence myi − ly1 = 0.
Q(yi ) ld yid
In both cases, we find that Q(y1 )
= md
= y1d
.
From the Claim, the polynomial Q(y1 )y d − y1d Q(y) has roots y = y1 , y2 , . . . , yd+1 . Since
its degree is at most d, this must be the zero polynomial. Hence, Q(y) = bd y d . This implies
P (x) = ad (x + ada ) . Let ada
d−1 d
d
d−1
d
= rs with integers r, s where r > 1 and (r, s) = 1. Since P has
integer coefficients, we need rd |ad . Let ad = rd a. Then P (x) = a(rx + s)d . It is obvious that
such a polynomial satisfies the conditions.
Comment. In the proof, the use of prime and Dirichlet’s Theorem can be avoided. One can
easily show that each P (xi ) can be expressed in the form uvid where u, vi are integers and u
cannot be divisible by the d-th power of a prime (note that u depends only on P ). By fixing a
large integer q and by choosing a large n, we can apply the Pigeonhole Principle and assume
Shortlisted problems 87
x1 ≡ x2 ≡ · · · ≡ xd+1 (mod q) and v1 ≡ v2 ≡ · · · ≡ vd+1 (mod q). Then the remaining proof
is similar to Case 2 of the Solution.
Alternatively, we give another modification of the proof as follows.
We take a sufficiently large n and consider the corresponding positive integers y1 , y2 , . . . , yn .
Q(yi ) lid
For each 2 6 i 6 n, let Q(y 1 )
= m d.
i
Contributing Countries
The Organizing Committee and the Problem Sele
tion Committee of IMO 2017 thank the
following 51
ountries for
ontributing 150 problem proposals:
Problems
Algebra
A1. Let a1 , a2 , . . . , an , k , and M be positive integers su
h that
1 1 1
` ` ¨¨¨` “k and a1 a2 . . . an “ M.
a1 a2 an
If M ą 1, prove that the polynomial
• In the rst line, Gugu writes down every number of the form a ´ b, where a and b are two
(not ne
essarily distin
t) numbers on his napkin.
• In the se
ond line, Gugu writes down every number of the form qab, where a and b are
two (not ne
essarily distin
t) numbers from the rst line.
• In the third line, Gugu writes down every number of the form a2 ` b2 ´ c2 ´ d2 , where
a, b, c, d are four (not ne
essarily distin
t) numbers from the rst line.
Determine all values of q su
h that, regardless of the numbers on Gugu's napkin, every
number in the se
ond line is also a number in the third line.
(Austria)
A3. Let S be a nite set, and let A be the set of all fun
tions from S to S . Let f be an
element of A, and let T “ f pSq be the image of S under f . Suppose that f ˝ g ˝ f ‰ g ˝ f ˝ g
for every g in A with g ‰ f . Show that f pT q “ T .
(India)
A4. A sequen
e of real numbers a1 , a2 , . . . satises the relation
Prove that this sequen
e is bounded, i.e., there is a
onstant M su
h that |an | ď M for all
positive integers n.
(Russia)
Shortlisted problems 5
n´1
ÿ
yi yi`1 “ y1 y2 ` y2 y3 ` y3 y4 ` ¨ ¨ ¨ ` yn´1 yn ě ´1.
i“1
ÿ
xi xj ě K
1ďiăjďn
for all x, y P R.
(Albania)
A7. Let a0 , a1 , a2 , . . . be a sequen
e of integers and b0 , b1 , b2 , . . . be a sequen
e of positive
integers su
h that a0 “ 0, a1 “ 1, and
#
an bn ` an´1 , if bn´1 “ 1
an`1 “ for n “ 1, 2, . . ..
an bn ´ an´1 , if bn´1 ą 1
Prove that at least one of the two numbers a2017 and a2018 must be greater than or equal to 2017.
(Australia)
A8. f : R Ñ R satises the following
ondition:
Assume that a fun
tion
` ˘` ˘
For every x, y P R su
h that f pxq ` y f pyq ` x ą 0, we have f pxq ` y “ f pyq ` x.
Combinatori
s
C1. A re
tangle R with odd integer side lengths is divided into small re
tangles with integer
side lengths. Prove that there is at least one among the small re
tangles whose distan
es from
the four sides of R are either all odd or all even.
(Singapore)
C2. Let n be a positive integer. Dene a
hameleon to be any sequen
e of 3n letters, with
exa
tly n o
urren
es of ea
h of the letters a, b, and c. Dene a swap to be the transposition of
two adja
ent letters in a
hameleon. Prove that for any
hameleon X , there exists a
hameleon Y
2
su
h that X
annot be
hanged to Y using fewer than 3n {2 swaps.
(Australia)
C3. Sir Alex plays the following game on a row of 9
ells. Initially, all
ells are empty. In
ea
h move, Sir Alex is allowed to perform exa
tly one of the following two operations:
(1) Choose any number of the form 2j , where j is a non-negative integer, and put it into an
empty
ell.
(2) Choose two (not ne
essarily adja
ent)
ells with the same number in them; denote that
j j`1
number by 2 . Repla
e the number in one of the
ells with 2 and erase the number in
the other
ell.
At the end of the game, one
ell
ontains the number 2n , where n is a given positive integer,
while the other
ells are empty. Determine the maximum number of moves that Sir Alex
ould
have made, in terms of n.
(Thailand)
C4. Let N ě2 be an integer. NpN ` 1q so
er players, no two of the same height, stand
in a row in some order. Coa
h Ralph wants to remove NpN ´ 1q people from this row so that
in the remaining row of 2N players, no one stands between the two tallest ones, no one stands
between the third and the fourth tallest ones, . . . , and nally no one stands between the two
shortest ones. Show that this is always possible.
(Russia)
C5. A hunter and an invisible rabbit play a game in the Eu
lidean plane. The hunter's
starting point H0
oin
ides with the rabbit's starting point R0 . In the nth round of the game
(n ě 1), the following happens.
(1) First the invisible rabbit moves se
retly and unobserved from its
urrent point Rn´1 to
some new point Rn with Rn´1 Rn “ 1.
1
(2) The hunter has a tra
king devi
e (e.g. dog) that returns an approximate position Rn of
1
the rabbit, so that Rn Rn ď 1.
(3) The hunter then visibly moves from point Hn´1 to a new point Hn with Hn´1 Hn “ 1.
9
Is there a strategy for the hunter that guarantees that after 10 su
h rounds the distan
e
between the hunter and the rabbit is below 100?
(Austria)
Shortlisted problems 7
X ˚ Y “ X Y tfX pyq : y P Y u.
Let A a ą 0 positive
be a set of integers, and let B be a set of bą0 positive integers. Prove
that if A ˚ B “ B ˚ A, then
A ˚ pA ˚ ¨ ¨ ¨ ˚ pA ˚ pA ˚ Aqq . . . q “ looooooooooooooooooomooooooooooooooooooon
looooooooooooooooooomooooooooooooooooooon B ˚ pB ˚ ¨ ¨ ¨ ˚ pB ˚ pB ˚ Bqq . . . q .
A appears b times B appears a times
(U.S.A.)
C8. Let n be a given positive integer. In the Cartesian plane, ea
h latti
e point
with nonnegative
oordinates initially
ontains a buttery, and there are no other butter-
ies. The neighborhood of a latti
e point c
onsists of all latti
e points within the axis-aligned
p2n ` 1q ˆ p2n ` 1q square
entered at c, apart from c itself. We
all a buttery lonely,
rowded,
or
omfortable, depending on whether the number of butteries in its neighborhood N is re-
spe
tively less than, greater than, or equal to half of the number of latti
e points in N.
Every minute, all lonely butteries y away simultaneously. This pro
ess goes on for as
long as there are any lonely butteries. Assuming that the pro
ess eventually stops, determine
the number of
omfortable butteries at the nal state.
(Bulgaria)
8 IMO 2017, Rio de Janeiro
Geometry
G1. Let ABCDE be a
onvex pentagon su
h that AB “ BC “ CD , =EAB “ =BCD ,
and =EDC “ =CBA. Prove that the perpendi
ular line from E to BC and the line seg-
ments AC and BD are
on
urrent.
(Italy)
G2. Let R and S be distin
t points on
ir
le Ω, and let t denote the tangent line to Ω
at R. Point R1 is the ree
tion of R with respe
t to S . A point I is
hosen on the smaller ar
RS of Ω so that the
ir
um
ir
le Γ of triangle ISR1 interse
ts t at two dierent points. Denote
by A the
ommon point of Γ and t that is
losest to R. Line AI meets Ω again at J . Show
1
that JR is tangent to Γ.
(Luxembourg)
G3. LetO be the
ir
um
enter of an a
ute s
alene triangle ABC . Line OA interse
ts the
altitudes of ABC through B and C at P and Q, respe
tively. The altitudes meet at H . Prove
that the
ir
um
enter of triangle P QH lies on a median of triangle ABC .
(Ukraine)
G4. In triangle ABC , let ω be the ex
ir
le opposite A. Let D , E , and F be the points
where ω is tangent to lines BC , CA, and AB , respe
tively. The
ir
le AEF interse
ts line BC
at P and Q. Let M be the midpoint of AD . Prove that the
ir
le MP Q is tangent to ω .
(Denmark)
G5. ABCC1 B1 A1 be a
onvex hexagon su
h that AB “ BC , and suppose that the
Let
line segmentsAA1 , BB1 , and CC1 have the same perpendi
ular bise
tor. Let the diagonals
AC1 and A1 C meet at D , and denote by ω the
ir
le ABC . Let ω interse
t the
ir
le A1 BC1
again at E ‰ B . Prove that the lines BB1 and DE interse
t on ω .
(Ukraine)
G6. Let ně3 be an integer. Two regular n-gons A and B are given in the plane. Prove
that the verti
es of A that lie inside B or on its boundary are
onse
utive.
(That is, prove that there exists a line separating those verti
es of A that lie inside B or on
its boundary from the other verti
es of A.)
(Cze
h Republi
)
G7. ABCD has an ins
ribed
ir
le with
enter I . Let Ia , Ib , Ic ,
A
onvex quadrilateral
and Id be the in
enters of the triangles DAB , ABC , BCD , and CDA, respe
tively. Suppose
that the
ommon external tangents of the
ir
les AIb Id and CIb Id meet at X , and the
ommon
0
external tangents of the
ir
les BIa Ic and DIa Ic meet at Y . Prove that =XIY “ 90 .
(Kazakhstan)
G8. There are 2017 mutually external
ir
les drawn on a bla
kboard, su
h that no two
are tangent and no three share a
ommon tangent. A tangent segment is a line segment that
is a
ommon tangent to two
ir
les, starting at one tangent point and ending at the other one.
Lu
iano is drawing tangent segments on the bla
kboard, one at a time, so that no tangent
segment interse
ts any other
ir
les or previously drawn tangent segments. Lu
iano keeps
drawing tangent segments until no more
an be drawn. Find all possible numbers of tangent
segments when he stops drawing.
(Australia)
Shortlisted problems 9
Number Theory
N1. The sequen
e a0 , a1 , a2 , . . . of positive integers satises
#? ?
an , if an is an integer
an`1 “ for every n ě 0.
an ` 3, otherwise
Determine all values of a0 ą 1 for whi
h there is at least one number a su
h that an “ a for
innitely many values of n.
(South Afri
a)
N2. Let p ě 2 be a prime number. Eduardo and Fernando play the following game making
moves alternately: in ea
h move, the
urrent player
hooses an index i in the set t0, 1, . . . , p´1u
that was not
hosen before by either of the two players and then
hooses an element ai of the
set t0, 1, 2, 3, 4, 5, 6, 7, 8, 9u. Eduardo has the rst move. The game ends after all the indi
es
i P t0, 1, . . . , p ´ 1u have been
hosen. Then the following number is
omputed:
p´1
ÿ
p´1
M “ a0 ` 10 ¨ a1 ` ¨ ¨ ¨ ` 10 ¨ ap´1 “ aj ¨ 10j .
j“0
The goal of Eduardo is to make the number M divisible by p, and the goal of Fernando is to
prevent this.
Prove that Eduardo has a winning strategy.
(Moro
o)
N3. Determine all integers n ě 2 with the following property: for any integers a1 , a2 , . . . , an
whose sum is not divisible by n, there exists an index 1 ď i ď n su
h that none of the numbers
pp ` qqp`q pp ´ qqp´q ´ 1
pp ` qqp´q pp ´ qqp`q ´ 1
is an integer.
(Japan)
Shortlisted problems 11
N6. Find the smallest positive integer n, or show that no su
h n exists, with the following
property: there are innitely many distin
t n-tuples of positive rational numbers pa1 , a2 , . . . , an q
su
h that both
1 1 1
a1 ` a2 ` ¨ ¨ ¨ ` an and ` ` ¨¨¨`
a1 a2 an
are integers.
(Singapore)
N7. Say that an ordered pair px, yq of integers is an irredu
ible latti
e point if x and
y are relatively prime. For any nite set S of irredu
ible latti
e points, show that there is a
homogenous polynomial in two variables, f px, yq, with integer
oe
ients, of degree at least 1,
su
h that f px, yq “ 1 for ea
h px, yq in the set S .
Note: A homogenous polynomial of degree n is any nonzero polynomial of the form
(U.S.A.)
N8. Let p be an odd prime number and Zą0 be the set of positive integers. Suppose that
a fun
tion f : Zą0 ˆ Zą0 Ñ t0, 1u satises the following properties:
• f p1, 1q “ 0;
• f pa, bq ` f pb, aq “ 1 for any pair of relatively prime positive integers pa, bq not both equal
to 1;
• f pa ` b, bq “ f pa, bq for any pair of relatively prime positive integers pa, bq.
Prove that
p´1
ÿ a
f pn2 , pq ě 2p ´ 2.
n“1
(Italy)
12 IMO 2017, Rio de Janeiro
Solutions
Algebra
A1. Let a1 , a2 , . . . , an , k , and M be positive integers su
h that
1 1 1
` ` ¨¨¨` “k and a1 a2 . . . an “ M.
a1 a2 an
If M ą 1, prove that the polynomial
ai px ` 1q1{ai ď x ` ai , (1)
n
ź n
ź řn n
ź
ai px ` 1q1{ai ď px ` ai q ðñ Mpx ` 1q i“1 1{ai ´ px ` ai q ď 0 ðñ P pxq ď 0
i“1 i“1 i“1
with equality i ai “ 1 for all i P t1, 2, . . . , nu. But this implies M “ 1, whi
h is not possible.
`
Hen
e P pxq ă 0 for all x P R , and P has no positive roots.
Comment 1. Inequality (1)
an be obtained in several ways. For instan
e, we may also use the
binomial theorem: sin
e ai ě 1,
ˆ ˙ ai ˆ ˙ ˆ ˙j ˆ ˙ ˆ ˙
x ai ÿ ai x ai ai x
1` “ ě ` ¨ “ 1 ` x.
ai j“0
j ai 0 1 ai
Both proofs of (1) mimi
proofs to Bernoulli's inequality for a positive integer exponent ai ; we
an
use this inequality dire
tly: ˆ ˙
x ai x
1` ě 1 ` ai ¨ “ 1 ` x,
ai ai
and so ˆ ˙
x
x ` ai “ ai 1` ě ai p1 ` xq1{ai ,
ai
or its (reversed) formulation, with exponent 1{ai ď 1:
1 x ` ai
p1 ` xq1{ai ď 1 ` ¨x“ ùñ ai p1 ` xq1{ai ď x ` ai .
ai ai
Shortlisted problems solutions 13
Solution 2. We will prove that, in fa
t, all
oe
ients of the polynomial P pxq are non-positive,
and at least one of them is negative, whi
h implies that P pxq ă 0 for x ą 0.
Indeed, sin
e aj ě 1 for all j and aj ą 1 for some j (sin
e a1 a2 . . . an “ M ą 1), we have
k “ a11 ` a12 ` ¨ ¨ ¨ ` a1n ă n, so the
oe
ient of xn in P pxq is ´1 ă 0. Moreover, the
oe
ient
r
of x in P pxq is negative for k ă r ď n “ degpP q.
r
For 0 ď r ď k , the
oe
ient of x in P pxq is
ˆ ˙ ÿ ˆ ˙ ÿ
k k
M¨ ´ ai1 ai2 ¨ ¨ ¨ ain´r “ a1 a2 ¨ ¨ ¨ an ¨ ´ ai1 ai2 ¨ ¨ ¨ ain´r ,
r 1ďi ăi 㨨¨ăi ďn
r 1ďi ăi 㨨¨ăi ďn
1 2 n´r 1 2 n´r
whi
h is non-positive i ˆ ˙
k ÿ 1
ď . (2)
r a a ¨ ¨ ¨ ajr
ďn j1 j2
1ďj 1 ăj2 㨨¨ăjr
We will prove (2) by indu
tion on r . For r “ 0 it is an equality be
ause the
onstant term of
ř
P pxq is P p0q “ 0, and if r “ 1, (2) be
omes k “ ni“1 a1i . For r ą 1, if (2) is true for a given
r ă k , we have
ˆ ˙ ˆ ˙ ÿ
k k´r k k´r 1
“ ¨ ď ¨ ,
r`1 r`1 r r ` 1 1ďj ăj 㨨¨ăj ďn aj1 aj2 ¨ ¨ ¨ ajr
1 2 r
k´r ÿ 1 ÿ 1
¨ ď ,
r ` 1 1ďj aj 1 aj 2 ¨ ¨ ¨ aj r aj 1 aj 2 ¨ ¨ ¨ aj r aj r`1
1 ăj2 㨨¨ăjr ďn 1ďj 1 㨨¨ăjr ăjr`1 ďn
whi
h is equivalent to
ˆ ˙ ÿ ÿ
1 1 1 1 1
` `¨¨¨` ´r ď pr ` 1q .
a1 a2 an 1ďj1 ăj2 㨨¨ăjr
a a ¨ ¨ ¨ ajr
ďn j1 j2
a a ¨ ¨ ¨ ajr ajr`1
ďn j1 j2
1ďj 1 㨨¨ăjr ăjr`1
1 1
Sin
e there are r`1 ways to
hoose a fra
tion
aji
from
aj1 aj2 ¨¨¨ajr ajr`1
to fa
tor out, every
1
term
aj1 aj2 ¨¨¨ajr ajr`1
in the right hand side appears exa
tly r`1 times in the produ
t
ˆ ˙ ÿ
1 1 1 1
` ` ¨¨¨` .
a1 a2 an 1ďj1 ăj2 㨨¨ăjr
a a ¨ ¨ ¨ ajr
ďn j1 j2
1 1 1 r
` ` ¨¨¨` ´
a2j1 aj2¨ ¨ ¨ ajr 2
aj1 aj2 ¨ ¨ ¨ ajr 2
aj1 aj2 ¨ ¨ ¨ ajr aj1 aj2 ¨ ¨ ¨ ajr
ˆ ˙
1 1 1 1
“ ` ` ¨¨¨` ´r ,
aj1 aj2 ¨ ¨ ¨ ajr aj1 aj2 ajr
1
whi
h are all non-positive be
ause ai ě 1 ùñ ai
ď 1, i “ 1, 2, . . . , n.
Comment 2. The result is valid for any real numbers ai , i “ 1, 2, . . . , n with ai ě 1 and produ
t M
greater than 1. A variation of Solution 1, namely using weighted AMGM (or the Bernoulli inequality
for real exponents), a
tually proves that P pxq ă 0 for x ą ´1 and x ‰ 0.
14 IMO 2017, Rio de Janeiro
A2. Let q be a real number. Gugu has a napkin with ten distin
t real numbers written on
it, and he writes the following three lines of real numbers on the bla
kboard:
• In the rst line, Gugu writes down every number of the form a ´ b, where a and b are two
(not ne
essarily distin
t) numbers on his napkin.
• In the se
ond line, Gugu writes down every number of the form qab, where a and b are
two (not ne
essarily distin
t) numbers from the rst line.
• In the third line, Gugu writes down every number of the form a2 ` b2 ´ c2 ´ d2 , where
a, b, c, d are four (not ne
essarily distin
t) numbers from the rst line.
Determine all values of q su
h that, regardless of the numbers on Gugu's napkin, every
number in the se
ond line is also a number in the third line.
(Austria)
Answer: ´2, 0, 2.
Solution 1. Call a number q good if every number in the se
ond line appears in the third line
un
onditionally. We rst show that the numbers 0 and ˘2 are good. The third line ne
essarily
ontains 0, so 0 is good. For any two numbers a, b in the rst line, write a “ x´y and b “ u´v ,
where x, y, u, v are (not ne
essarily distin
t) numbers on the napkin. We may now write
whi
h shows that 2 is good. By negating both sides of the above equation, we also see that ´2
is good.
We now show that ´2, 0, and 2 are the only good numbers. Assume for sake of
ontradi
tion
that q is a good number, where q R t´2, 0, 2u. We now
onsider some parti
ular
hoi
es of
numbers on Gugu's napkin to arrive at a
ontradi
tion.
Assume that the napkin
ontains the integers 1, 2, . . . , 10. Then, the rst line
ontains
the integers ´9, ´8, . . . , 9. The se
ond line then
ontains q and 81q , so the third line must
also
ontain both of them. But the third line only
ontains integers, so q must be an integer.
Furthermore, the third line
ontains no number greater than
2
162 “ 9 ` 92 ´ 02 ´ 02 or less
than ´162, so we must have ´162 ď 81q ď 162. This shows that the only possibilities for q
are ˘1.
Now assume that q “ ˘1. Let the napkin
ontain 0, 1, 4, 8, 12, 16, 20, 24, 28, 32. The rst
line
ontains ˘1 and ˘4, so the se
ond line
ontains ˘4. However, for every number a in the
2
rst line, a ı 2 pmod 4q, so we may
on
lude that a ” 0, 1 pmod 8q. Consequently, every
number in the third line must be
ongruent to ´2, ´1, 0, 1, 2 pmod 8q; in parti
ular, ˘4
annot
be in the third line, whi
h is a
ontradi
tion.
Solution 2. Let q be a good number, as dened in the rst solution, and dene the polynomial
P px1 , . . . , x10 q as
ź ź` ˘
pxi ´ xj q qpx1 ´ x2 qpx3 ´ x4 q ´ pa1 ´ a2 q2 ´ pa3 ´ a4 q2 ` pa5 ´ a6 q2 ` pa7 ´ a8 q2 ,
iăj ai PS
A3. Let S be a nite set, and let A be the set of all fun
tions from S to S . Let f be an
element of A, and let T “ f pSq be the image of S under f . Suppose that f ˝ g ˝ f ‰ g ˝ f ˝ g
for every g in A with g ‰ f . Show that f pT q “ T .
(India)
Solution. For n ě 1, denote the n-th
omposition of f with itself by
def
f n “ flooooooomooooooon
˝ f ˝ ¨¨¨˝ f .
n times
Claim. If there exists ně3 su
h that f n`2 “ f 2n`1 , then the restri
tion f: T Ñ T of f to T
is a bije
tion.
(here id stands for the identity fun
tion). Hen
e, the restri
tion f : T Ñ T of f to T is bije
tive
n´2
with inverse given by f : T Ñ T. l
It remains to show that n as in the
laim exists. For that, dene
def
Sm “ f m pSq pSm is image of f mq
Clearly the image of f m`1 is
ontained in the image of f m, i.e., there is a des
ending
hain of
subsets of S
S Ě S1 Ě S2 Ě S3 Ě S4 Ě ¨ ¨ ¨ ,
whi
h must eventually stabilise sin
e S is nite, i.e., there is a kě1 su
h that
def
Sk “ Sk`1 “ Sk`2 “ Sk`3 “ ¨ ¨ ¨ “ S8 .
This
an be
learly done by
hoosing m large enough with m ” 3 pmod rq. For instan
e, we
may take n “ 2kr ` 1, so that
Prove that this sequen
e is bounded, i.e., there is a
onstant M su
h that |an | ď M for all
positive integers n.
(Russia)
Solution 1. Set D “ 2017. Denote
Clearly, the sequen
es pmn q and pMn q are nonde
reasing. We need to prove that both are
bounded.
´2Mn ď an ď mn ´ Mn ,
when e
Now, say that an index nąD is lu ky if mn ď 2Mn . Two ases are possible.
Case 1. Assume that there exists a lu
ky index n. In this
ase, (1) yields mn`1 ď 2Mn and
Mn ď Mn`1 ď Mn . Therefore, Mn`1 “ Mn and mn`1 ď 2Mn “ 2Mn`1 . So, the index n ` 1
is also lu
ky, and Mn`1 “ Mn . Applying the same arguments repeatedly, we obtain that all
indi
es k ą n are lu
ky (i.e., mk ď 2Mk for all these indi
es), and Mk “ Mn for all su
h indi
es.
Thus, all of the mk and Mk are bounded by 2Mn .
Case 2. Assume now that there is no lu
ky index, i.e., 2Mn ă mn for all n ą D. Then (1)
shows that for all n ą D we have mn ď mn`1 ď mn , so mn “ mD`1 for all n ą D . Sin
e
Mn ă mn {2 for all su
h indi
es, all of the mn and Mn are bounded by mD`1 .
Thus, in both
ases the sequen
es pmn q and pMn q are bounded, as desired.
Solution 2. As in the previous solution, let D “ 2017. If the sequen
e is bounded above, say,
by Q, then we have that an ě minta1 , . . . , aD , ´2Qu for all n, so the sequen
e is bounded. As-
sume for sake of
ontradi
tion that the sequen
e is not bounded above. Let ℓ “ minta1 , . . . , aD u,
and L “ maxta1 , . . . , aD u. Call an index n good if the following
riteria hold:
We rst show that there must be some good index n. By assumption, we may take an
index N su
h that aN ą maxtL, ´2ℓu. n minimally su
h that an “ maxta1 , a2 , . . . , aN u.
Choose
Now, the rst
ondition in (2) is satised be
ause of the minimality of n, and the se
ond and
third
onditions are satised be
ause an ě aN ą L, ´2ℓ, and L ě ai for every i su
h that
1 ď i ď D.
18 IMO 2017, Rio de Janeiro
an ` au ` av ď 0, (3)
whenever u ` v “ n.
u to maximize au over 1 ď u ď n ´ 1, and
We dene the index let v “ n ´ u. Then, we note
that au ě av by the maximality of au .
Assume rst that v ď D . Then, we have that
aN ` 2ℓ ď 0,
be
ause au ě av ě ℓ. But this
ontradi
ts our assumption that an ą ´2ℓ in the se
ond
riteria
of (2).
Now assume that v ą D. Then, there exist some indi
es w1 , w2 summing up to v su
h that
av ` aw1 ` aw2 “ 0.
an ` au ď aw1 ` aw2 .
Be
ause an ą au , we have that maxtaw1 , aw2 u ą au . But sin
e ea
h of the wi is less than v, this
ontradi
ts the maximality of au .
Clearly, the sequen
es pmn q and pMn q are nonde
reasing. We need to prove that both are bounded.
Consider an arbitrary n ą 2D ; our rst aim is to bound an in terms of mi and Mi . Set k “ tn{2u.
(i) Choose indi
es p, q , and r su
h that an “ ´pap ` aq ` ar q and p ` q ` r “ n. Without loss of
generality, p ě q ě r .
Assume that p ě k ` 1pą Dq; then p ą q ` r . Hen
e
Now, say that an index n ą 2D is lu
ky if mn ď 2Mtn{2u`1 ` Mtn{2u . Two
ases are possible.
Case 1. Assume that there exists a lu
ky index n; set k “ tn{2u. In this
ase, (4) yields mn`1 ď
2Mk`1 ` Mk and Mn ď Mn`1 ď Mn (the last relation holds, sin
e mn ´ Mk`1 ´ Mk ď p2Mk`1 `
Mk q ´ Mk`1 ´ Mk “ Mk`1 ď Mn ). Therefore, Mn`1 “ Mn and mn`1 ď 2Mk`1 ` Mk ; the last relation
shows that the index n ` 1 is also lu
ky.
Thus, all indi
es N ą n are lu
ky, and MN “ Mn ě mN {3, when
e all the mN and MN are
bounded by 3Mn .
Case 2. Conversely, assume that there is no lu
ky index, i.e., 2Mtn{2u`1 ` Mtn{2u ă mn for all n ą 2D.
Then (4) shows that for all n ą 2D we have mn ď mn`1 ď mn , i.e., mN “ m2D`1 for all N ą 2D .
Sin
e MN ă m2N `1 {3 for all su
h indi
es, all the mN and MN are bounded by m2D`1 .
Thus, in both
ases the sequen
es pmn q and pMn q are bounded, as desired. l
Version 2. Let a1 , a2 , . . . be a sequen
e of numbers that satises the relation
an “ ´ max pai1 ` ¨ ¨ ¨ ` aik q for all n ą 2017.
i1 `¨¨¨`ik “n
We rst show that there must be some good index n. By assumption, we may take an index N
su
h that aN ą maxtL, ´kℓu. Choose n minimally su
h that an “ maxta1 , a2 , . . . , aN u. Now, the rst
ondition is satised be
ause of the minimality of n, and the se
ond and third
onditions are satised
be
ause an ě aN ą L, ´kℓ, and L ě ai for every i su
h that 1 ď i ď D .
Let n be a good index. We derive a
ontradi
tion. We have that
whenever v1 ` ¨ ¨ ¨ ` vk “ n.
We dene the sequen
e of indi
es v1 , . . . , vk´1 to greedily maximize av1 , then av2 , and so forth,
sele
ting only from indi
es su
h that the equation v1 ` ¨ ¨ ¨ ` vk “ n
an be satised by positive integers
v1 , . . . , vk . More formally, we dene them indu
tively so that the following
riteria are satised by
the vi :
1. 1 ď vi ď n ´ pk ´ iq ´ pv1 ` ¨ ¨ ¨ ` vi´1 q.
2. avi is maximal among all
hoi
es of vi from the rst
riteria.
First of all, we note that for ea
h i, the rst
riteria is always satisable by some vi , be
ause we
are guaranteed that
vi´1 ď n ´ pk ´ pi ´ 1qq ´ pv1 ` ¨ ¨ ¨ ` vi´2 q,
whi
h implies
1 ď n ´ pk ´ iq ´ pv1 ` ¨ ¨ ¨ ` vi´1 q.
Se
ondly, the sum v1 ` ¨ ¨ ¨ ` vk´1 is at most n ´ 1. Dene vk “ n ´ pv1 ` ¨ ¨ ¨ ` vk´1 q. Then, (6)
is satised by the vi . We also note that avi ě avj for all i ă j ; otherwise, in the denition of vi , we
ould have sele
ted vj instead.
Assume rst that vk ď D . Then, from (6), we have that
an ` kℓ ď 0,
by using that av1 ě ¨ ¨ ¨ ě avk ě ℓ. But this
ontradi
ts our assumption that an ą ´kℓ in the se
ond
riteria of (5).
20 IMO 2017, Rio de Janeiro
Now assume that vk ą D , and then we must have some indi
es w1 , . . . , wk summing up to vk su
h
that
avk ` aw1 ` ¨ ¨ ¨ ` awk “ 0.
But
ombining this with (6), we have
Be
ause an ą av1 ě ¨ ¨ ¨ ě avk´1 , we have that maxtaw1 , . . . , awk u ą avk´1 . But sin
e ea
h of the wi
is less than vk , in the denition of the vk´1 we
ould have
hosen one of the wi instead, whi
h is a
ontradi
tion. l
Comment 2. It seems that ea
h sequen
e satisfying the
ondition in Version 2 is eventually periodi
,
at least when its terms are integers.
However, up to this moment, the Problem Sele
tion Committee is not aware of a proof for this fa
t
(even in the
ase k “ 2).
Shortlisted problems solutions 21
ÿ
xi xj ě K
1ďiăjďn
ÿ n ´ 1 pn ´ 1qpn ´ 2q 2
xi xj “ ´ ` t.
iăj
2 2
ř
Thus, as t approa
hes 0 from above, iăj xi xj gets arbitrarily
lose to ´pn ´ 1q{2.řThis shows
that we may not take K any larger than ´pn ´ 1q{2. It remains to show that iăj xi xj ě
´pn ´ 1q{2 for any Shiny
hoi
e of the xi .
From now onward, assume that px1 , . . . , xn q
n-tuple. Let the zi (1 ď i ď n) be
is a Shiny
some permutation of the xi to be
hosen later. The indi
es for zi will always be taken modulo n.
ř ř
We will rst split up the sum iăj xi xj “ iăj zi zj into tpn ´ 1q{2u expressions, ea
h of the
form y1 y2 ` ¨ ¨ ¨ ` yn´1 yn for some permutation yi of the zi , and some leftover terms. More
spe
i
ally, write
n´1 t n´1 u
ÿ ÿ ÿ ÿ 2 ÿ
zi zj “ zi zj “ zi zj ` L, (1)
iăj q“0 i`j”q pmod nq p“1 i`j”2p´1,2p pmod nq
iıj pmod nq iıj pmod nq
where L “ z1 z´1 ` z2 z´2 ` ¨ ¨ ¨ ` zpn´1q{2 z´pn´1q{2 if n is odd, and L “ z1 z´1 ` z1 z´2 ` z2 z´2 `
¨ ¨ ¨ ` zpn´2q{2 z´n{2 if n is even. We note that for ea
h p “ 1, 2, . . . , tpn ´ 1q{2u, there is some
permutation yi of the zi su
h that
ÿ n´1
ÿ
zi zj “ yk yk`1 ,
i`j”2p´1,2p pmod nq k“1
iıj pmod nq
be
ause we may
hoose y2i´1 “ zi`p´1 for 1 ď i ď pn ` 1q{2 and y2i “ zp´i for 1 ď i ď n{2.
We show (1) graphi
ally for n “ 6, 7 in the diagrams below. The edges of the graphs ea
h
represent a produ
t zi zj , and the dashed and dotted series of lines represents the sum of the
edges, whi
h is of the form y1 y2 ` ¨ ¨ ¨ ` yn´1 yn for some permutation yi of the zi pre
isely when
the series of lines is a Hamiltonian path. The lled edges represent the summands of L.
22 IMO 2017, Rio de Janeiro
Now, be ause the zi are Shiny, we have that (1) yields the following bound:
ÿ Z^
n´1
zi zj ě ´ ` L.
iăj
2
It remains to show that, for ea
h n, there exists some permutation zi of the xi su
h that Lě0
when n is odd, and L ě ´1{2 when n is even. We now split into
ases based on the parity of n
and provide
onstru
tions of the permutations zi .
Sin
e we have not made any assumptions yet about the xi , we may now assume without
loss of generality that
x1 ď x2 ď ¨ ¨ ¨ ď xk ď 0 ď xk`1 ď ¨ ¨ ¨ ď xn . (2)
Case 1: n is odd.
Without loss of generality, assume that k (from (2)) is even, be
ause we may negate all
the xi if k is odd. We then have x1 x2 , x3 x4 , . . . , xn´2 xn´1 ě 0 be
ause the fa
tors are of the
same sign. Let L “ x1 x2 ` x3 x4 ` ¨ ¨ ¨ ` xn´2 xn´1 ě 0. We
hoose our zi so that this denition
of L agrees with the sum of the leftover terms in (1). Relabel the xi as zi su
h that
Case 2: n is even.
Let L “ x1 x2 ` x2 x3 ` ¨ ¨ ¨ ` xn´1 xn . Assume without loss of generality k ‰ 1. Now, we have
ě x2 x3 ` ¨ ¨ ¨ ` xn´1 xn ` xn x1 ě ´1,
where the rst inequality holds be
ause the only negative term in L is xk xk`1 , the se
ond
inequality holds be
ause x1 ď xk ď 0 ď xk`1 ď xn , and the third inequality holds be
ause
the xi are assumed to be Shiny. We thus have that L ě ´1{2. We now
hoose a suitable zi
su
h that the denition of L mat
hes the leftover terms in (1).
Shortlisted problems solutions 23
Relabel the xi with zi in the following manner: x2i´1 “ z´i , x2i “ zi (again taking indi
es
modulo n). We have that
ÿ
L“ zi zj ,
i`j”0,´1 pmod nq
iıj pmod nq
as desired.
ř
Solution 2. iăj xi xj ě ´pn ´ 1q{2 for any Shiny n-tuple
We present another proof that
px1 , . . . , xn q. xi as in (2), and let ℓ “ n ´ k . Assume without loss
Assume an ordering of the
of generality that k ě ℓ. Also assume k ‰ n, (as otherwise, all of the xi are nonpositive, and
so the inequality is trivial). Dene the sets of indi
es S “ t1, 2, . . . , ku and T “ tk ` 1, . . . , nu.
Dene the following sums:
ÿ ÿ ÿ
K“ xi xj , M“ xi xj , and L“ xi xj
iăj iPS iăj
i,jPS jPT i,jPT
n´1
ÿ
f pφq “ xφpiq xφpi`1q .
i“1
We know that f pφq ě ´1 for every permutation φ with the above property. Averaging f pφq
over all φ gives
1 ÿ 2ℓ 2pk ´ ℓ ´ 1q
´1 ď f pφq “ M ` K,
k!ℓ! φ kℓ kpk ´ 1q
where the equality holds be
ause there are kℓ produ
ts in M , of whi
h 2ℓ are sele
ted for ea
h φ,
and there are kpk ´ 1q{2 produ
ts in K , of whi
h k ´ ℓ ´ 1 are sele
ted for ea
h φ. We now
have ˆ ˙
k k´ℓ´1 k ℓ
K `L`M ě K `L` ´ ´ K “´ ` K ` L.
2 k´1 2 k´1
Sin
e k ďn´1 and K, L ě 0, we get the desired inequality.
Case 2: k “ ℓ “ n{2.
We do a similar approa
h,
onsidering all φ : t1, 2, . . . , nu Ñ t1, 2, . . . , nu su
h that φ´1 pT q “
t2, 4, . . . , 2ℓu, and dening f the same way. Analogously to Case 1, we have
1 ÿ 2ℓ ´ 1
´1 ď f pφq “ M,
k!ℓ! φ kℓ
be ause there are kℓ produ ts in M, of whi h 2ℓ ´ 1 are sele ted for ea h φ. Now, we have that
n2 n´1
K `L`M ěM ě´ ě´ ,
4pn ´ 1q 2
for all x, y P R.
(Albania)
Answer: There are 3 solutions:
x ÞÑ 0 or x ÞÑ x ´ 1 or x ÞÑ 1 ´ x px P Rq.
Solution. An easy
he
k shows that all the 3 above mentioned fun
tions indeed satisfy the
original equation p˚q.
In order to show that these are the only solutions, rst observe that if f pxq is a solution
then ´f pxq is also a solution. Hen
e, without loss of generality we may (and will) assume that
f p0q ď 0 from now on. We have to show that either f is identi
ally zero or f pxq “ x ´ 1
(@x P R).
x
Observe that, for a xed x ‰ 1, we may
hoose yPR so that x ` y “ xy ðñ y “ x´1
,
and therefore from the original equation p˚q we have
´ ´ x ¯¯
f f pxq ¨ f “0 px ‰ 1q. (1)
x´1
In parti
ular, plugging in x “ 0 in (1), we
on
lude that f has at least one zero, namely pf p0qq2 :
` ˘
f pf p0qq2 “ 0. (2)
Case 2: f p0q ă 0.
We begin with the following
Claim 1.
f p1q “ 0, f paq “ 0 ùñ a “ 1, and f p0q “ ´1. (3)
Proof. We need to show that 1 is the unique zero of f. First, observe that f has at least one
zero a by (2); if a ‰ 1 then setting x “ a in (1) we get f p0q “ 0, a
ontradi
tion. Hen
e
from (2) we get pf p0qq2 “ 1. Sin
e we are assuming f p0q ă 0, we
on
lude that f p0q “ ´1. l
Setting y“1 in the original equation p˚q we get
f pf pxqf p1qq`f px`1q “ f pxq ðñ f p0q`f px`1q “ f pxq ðñ f px`1q “ f pxq`1 px P Rq.
f pa ` N ` 1q “ f pb ` Nq ` 1.
However, by Claim 1 we have f px0 q ‰ 0 and f py0 q ‰ 0 sin
e x0 ‰ 1 and y0 ‰ 1, a
ontradi
tion.
l
Now the end is near. For any t P R, plug in px, yq “ pt, ´tq in the original equation p˚q to
get
f pf ptqf p1 ´ tqq ` f p1q “ f ptp1 ´ tqq ðñ f pf ptqf p1 ´ tqq “ f ptp1 ´ tqq by (3)
as desired.
Comment. Other approa
hes are possible. For instan
e, after Claim 1, we may dene
def
gpxq “ f pxq ` 1.
Repla ing x ` 1 and y ` 1 in pla e of x and y in the original equation p˚q, we get
and therefore, using (4) (so that in parti ular gpxq “ f px ` 1q), we may rewrite p˚q as
We are now to show that gpxq “ x for all x P R under the assumption (Claim 1) that 0 is the unique
zero of g .
Claim 3. Let n P Z and x P R. Then
(a) gpx ` nq “ x ` n, and the
onditions gpxq “ n and x “ n are equivalent.
Proof. For part (a), just note that gpx ` nq “ x ` n is just a reformulation of (4). Then gpxq “ n ðñ
gpx ´ nq “ 0 ðñ x ´ n “ 0 sin
e 0 is the unique zero of g. For part (b), we may assume that x ‰ 0
sin
e the result is obvious when x “ 0. Plug in y “ n{x in p˚˚q and use part (a) to get
´ ´ n ¯¯ ´ n¯ ´ n¯ ´ ´ n ¯¯ ´n¯
g gpxqg `g x` “g n`x` ðñ g gpxqg “ n ðñ gpxqg “ n.
x x x x x
In other words, for x ‰ 0 we have
n
gpxq “ ` ˘.
g n{x
In parti
ular, for n “ 1, we get gp1{xq “ 1{gpxq, and therefore repla
ing x Ð nx in the last equation
we nally get
n
gpnxq “ ` ˘ “ ngpxq,
g 1{x
as required.
Claim 4. The fun
tion g is additive, i.e., gpa ` bq “ gpaq ` gpbq for all a, b P R.
Proof. Set x Ð ´x and y Ð ´y in p˚˚q; sin
e g is an odd fun
tion (by Claim 3(b) with n “ ´1), we
get
gpgpxqgpyqq ´ gpx ` yq “ ´gp´xy ` x ` yq.
Subtra
ting the last relation from p˚˚q we have
and sin
e by Claim 3(b) we have 2gpx ` yq “ gp2px ` yqq, we may rewrite the last equation as
#
α “ xy ` x ` y
gpα ` βq “ gpαq ` gpβq where
β “ ´xy ` x ` y.
In other words, we have additivity for all α, β P R for whi h there are real numbers x and y satisfying
α`β α´β
x`y “ and xy “ ,
2 2
by Claim 3(b). Can
elling n, we get the desired result. (Alternatively, setting either pα, βq “ pa, bq or
pα, βq “ p´a, ´bq will ensure that p α`β α´β
2 q ´ 4 ¨ 2 ě 0).
2 l
Now we may nish the solution. Set y “ 1 in p˚˚q, and use Claim 3 to get
Prove that at least one of the two numbers a2017 and a2018 must be greater than or equal to 2017.
(Australia)
Solution 1. The value of b0 is irrelevant sin
e a0 “ 0, so we may assume that b0 “ 1.
Lemma. We have an ě 1 for all n ě 1.
Proof. Let us suppose otherwise in order to obtain a
ontradi
tion. Let
Note that n ě 2. It follows that an´1 ě 1 and an´2 ě 0. Thus we
annot have an “
an´1 bn´1 ` an´2 , so we must have an “ an´1 bn´1 ´ an´2 . Sin
e an ď 0, we have an´1 ď an´2 .
Thus we have an´2 ě an´1 ě an .
Let
r be the smallest index with ar ě ar`1 ě ar`2 . (2)
Case 1: bn´1 “ 1.
Then an`1 “ an bn ` an´1 . By the indu
tive assumption one of an´1 , an is at least n´1 and
the other, by the lemma, is at least 1. Hen
e
Case 2: bn´1 ą 1.
Sin
e we dened b0 “ 1 there is an index r with 1ďr ďn´1 su
h that
an ą an´1 ą ¨ ¨ ¨ ą ar`1 ě r ` 1 ùñ an ě n.
Solution 2. We say that an index ną1 is bad bn´1 “ 1 and bn´2 ą 1; otherwise n is good.
if
The value of b0 is irrelevant to the denition of pan q sin
e a0 “ 0; so we assume that b0 ą 1.
Lemma 1. (a) an ě 1 for all n ą 0.
(b) If ną1 is good, then an ą an´1 .
Case 1: k is bad.
We have bk´1 “ 1, so ak`1 “ bk ak ` ak´1 ě ak ` ak´1 ą ak ě 1, as required.
Case 2: k is good.
We already have ak ą ak´1 ě 1 by the indu
tion hypothesis. We
onsider three easy
sub
ases.
m “ infti ą 0 : bn`i´1 ą 1u
(possibly m “ `8). We
laim that j “ mintm, 3u works. Again, we distinguish several
ases,
a
ording to the value of m; in ea
h of them we use Lemma 1 without referen
e.
Case 1: m “ 1, so bn ą 1.
Then an`1 ě 2an ` an´1 ě an´1 ` 2, as required.
Case 3: m ą 2, so bn “ bn`1 “ 1.
Then we su
essively get
as required. l
Lemmas 1(b) and 2 provide enough information to prove that maxtan , an`1 u ě n for all n
and, moreover, that an ě n often enough. Indeed, assume that we have found some n with
an´1 ě n´1. If n is good, then by Lemma 1(b) we have an ě n as well. If n is bad, then Lemma 2
yields maxtan`i , an`i`1 u ě an´1 ` i ` 1 ě n ` i for all 0 ď i ă j and an`j ě an´1 ` j ` 1 ě n ` j ;
so n ` j is the next index to start with.
30 IMO 2017, Rio de Janeiro
` ˘` ˘
For every x, y P R su
h that f pxq ` y f pyq ` x ą 0, we have f pxq ` y “ f pyq ` x.
If gpxq ‰ gpyq, then the number x ` y lies (non-stri
tly) between gpxq and gpyq. p˚q
Noti
e here that the fun
tion g1 pxq “ ´gp´xq also satises p˚q, sin
e
g1 pxq ‰ g1 pyq ùñ gp´xq ‰ gp´yq ùñ ´px ` yq lies between gp´xq and gp´yq
ùñ x ` y lies between g1 pxq and g1 pyq.
Proof. We start with the rst
laim of the lemma. Noti
e that X ´ x ă x, so the
onsidered
interval is nonempty.
Take anya P pX ´ x; xq with gpaq ‰ X (if it exists). If gpaq ă X , then p˚q yields gpaq ď
a ` x ď gpxq “ X , so a ď X ´ x whi
h is impossible. Thus, gpaq ą X and hen
e by p˚q we get
X ď a ` x ď gpaq.
Now, for any b P pX ´ x; xq with gpbq ‰ X we similarly get b ` x ď gpbq. Therefore, the
number a ` b (whi
h is smaller than ea
h of a ` x and b ` x)
annot lie between gpaq and gpbq,
whi
h by p˚q implies that gpaq “ gpbq. Hen
e g may attain only two values on pX ´ x; xs,
namely X and gpaq ą X .
To prove the se
ond
laim, noti
e that g1 p´xq “ ´X ă 2 ¨ p´xq, so g1 attains at most two
values on p´X ` x, ´xs, i.e., ´X and, possibly, some ´Y ą ´X . Passing ba
k to g , we get
what we need. l
Lemma 2. If X ă 2x, then g is
onstant on pX ´ x; xq. Similarly, if X ą 2x, then g is
onstant
on px; X ´ xq.
Proof. Again, it su
es to prove the rst
laim only. Assume, for the sake of
ontradi
tion,
that there exista, b P pX ´ x; xq with gpaq ‰ gpbq; by Lemma 1, we may assume that gpaq “ X
and Y “ gpbq ą X .
Noti
e that mintX ´ a, X ´ bu ą X ´ x, so there exists a u P pX ´ x; xq su
h that
u ă mintX ´ a, X ´ bu. By Lemma 1, we have either gpuq “ X or gpuq “ Y . In the former
ase, by p˚q we have X ď u ` b ď Y whi
h
ontradi
ts u ă X ´ b. In the se
ond
ase, by p˚q
we have X ď u ` a ď Y whi
h
ontradi
ts u ă X ´ a. Thus the lemma is proved. l
Shortlisted problems solutions 31
Lemma 3. If X ă 2x, then gpaq “ X for all a P pX ´ x; xq. Similarly, if X ą 2x, then gpaq “ X
for all a P px; X ´ xq.
Proof. Again, we only prove the rst
laim.
By Lemmas 1 and 2, this
laim may be violated only if g takes on a
onstant value Y ąX
on pX ´ x, xq. Choose any a, b P pX ´ x; xq with a ă b. By p˚q, we have
Y ě b ` x ě X. (2)
Solution 2. As in the previous solution, we pass to the fun
tion p˚q and noti
e
g satisfying
that we need to prove the
ondition (1). We will also make use of the fun
tion g1 .
If g is
onstant, then (1) is
learly satised. So, in the sequel we assume that g takes on at
least two dierent values. Now we
olle
t some information about the fun
tion g .
Assume now, for the sake of
ontradi
tion, that (1) is violated by some x ă y . By Claim 3,
rx; ys is a Diri
hlet interval. Set
r “ infta : pa; ys is a Diri hlet intervalu and s “ suptb : rx; bq is a Diri hlet intervalu.
1 1
The fun
tion attains at least three distin
t values on rr ; ys, namely gpr q, gpxq, and gpyq.
g
1 1
Claim 3 now yields gpr q ď gpyq; the equality is impossible by the
hoi
e of r , so in fa
t
gpr 1 q ă Y . Applying p˚q to the pairs pr 1 , yq and pt, xq we obtain r 1 ` y ď Y ď t ` x, when
e
r ´ ∆ ` y ă r 1 ` y ď t ` x ă r ` ∆ ` x, or y ´ x ă 2∆. This is a
ontradi
tion.
Thus, gptq “ X for all t P pr; r ` ∆q. Applying the same argument to g1 , we get gptq “ Y
for all t P ps ´ ∆; sq.
Finally,
hoose some s1 , s2 P ps ´ ∆; sq with s1 ă s2 and denote δ “ ps2 ´ s1 q{2. As before,
we
hoose r P pr ´ δ; rq with gpr 1q R tX, Y u and obtain gpr 1q ă Y . Choose any t P pr; r ` δq; by
1
the above arguments, we have gptq “ X and gps1 q “ gps2 q “ Y . As before, we apply p˚q to the
1 1
pairs pr , s2 q and pt, s1 q obtaining r ´ δ ` s2 ă r ` s2 ď Y ď t ` s1 ă r ` δ ` s1 , or s2 ´ s1 ă 2δ .
This is a nal
ontradi
tion.
Comment 1. The original submission dis
ussed the same fun
tions f , but the question was dier-
ent namely, the following one:
Prove that the equation f pxq “ 2017x has at most one solution, and the equation f pxq “ ´2017x
has at least one solution.
The Problem Sele
tion Committee de
ided that the question we are proposing is more natural,
sin
e it provides more natural information about the fun
tion g (whi
h is indeed the main
hara
ter
in this story). On the other hand, the new problem statement is strong enough in order to imply the
original one easily.
Namely, we will dedu
e from the new problem statement (along with the fa
ts used in the solutions)
that piq for every N ą 0 the equation gpxq “ ´N x has at most one solution, and piiq for every N ą 1
the equation gpxq “ N x has at least one solution.
Claim piq is now trivial. Indeed, g is proven to be non-de
reasing, so gpxq`N x is stri
tly in
reasing
and thus has at most one zero.
We pro
eed on
laim piiq. If gp0q “ 0, then the required root has been already found. Otherwise,
we may assume that gp0q ą 0 and denote c “ gp0q. We intend to prove that x “ c{N is the required
root. Indeed, by monotoni
ity we have gpc{N q ě gp0q “ c; if we had gpc{N q ą c, then p˚q would yield
c ď 0 ` c{N ď gpc{N q whi
h is false. Thus, gpxq “ c “ N x.
Comment 2. There are plenty of fun
tions g satisfying p˚q (and hen
e of fun
tions f satisfying
the problem
onditions). One simple example is g0 pxq “ 2x. Next, for any in
reasing sequen
e
A “ p. . . , a´1 , a0 , a1 , . . . q whi
h is unbounded in both dire
tions (i.e., for every N this sequen
e
ontains
terms greater than N , as well as terms smaller than ´N ), the fun
tion gA dened by
satises p˚q. Indeed, pi
k any x ă y with gpxq ‰ gpyq; this means that x P rai ; ai`1 q and y P raj ; aj`1 q
for some i ă j . Then we have gpxq “ ai ` ai`1 ď x ` y ă aj ` aj`1 “ gpyq, as required.
There also exist examples of the mixed behavior; e.g., for an arbitrary sequen
e A as above and an
arbitrary subset I Ď Z the fun
tion
#
g0 pxq, x P rai ; ai`1 q with i P I ;
gA,I pxq “
gA pxq, x P rai ; ai`1 q with i R I
It is easy to see that for dierent sets A and B the fun
tions gA and gB are also dierent (sin
e, e.g.,
for any a P A z B the fun
tion gB is
onstant in a small neighborhood of a, but the fun
tion gA is not).
One may
he
k, similarly to the arguments above, that ea
h su
h fun
tion satises p˚q.
Finally, one more modi
ation is possible. Namely, for any x P A one may redene gA pxq (whi
h
is 2x) to be any of the numbers
This really
hanges the value if x has some right (respe
tively, left) semi-neighborhood disjoint from A,
so there are at most
ountably many possible
hanges; all of them
an be performed independently.
With some eort, one may show that the
onstru
tion above provides all fun
tions g satisfying p˚q.
34 IMO 2017, Rio de Janeiro
Combinatori
s
C1. A re
tangle R with odd integer side lengths is divided into small re
tangles with integer
side lengths. Prove that there is at least one among the small re
tangles whose distan
es from
the four sides of R are either all odd or all even.
(Singapore)
Solution. Let the width and height of R be odd numbers a and b. Divide R into ab unit
squares and
olor them green and yellow in a
he
kered pattern. Sin
e the side lengths of a
and b are odd, the
orner squares of R will all have the same
olor, say green.
Call a re
tangle (either R or a small re
tangle) green if its
orners are all green;
all it
yellow if the
orners are all yellow, and
all it mixed if it has both green and yellow
orners. In
parti
ular, R is a green re
tangle.
The re
tangle R is green, so it
ontains more green unit squares than yellow unit squares.
Therefore, among the small re
tangles, at least one is green. Let S be su
h a small green
re
tangle, and let its distan
es from the sides of R be x, y , u and v , as shown in the pi
ture.
The top-left
orner of R and the top-left
orner of S have the same
olor, whi
h happen if and
only if x and u have the same parity. Similarly, the other three green
orners of S indi
ate that
x and v have the same parity, y and u have the same parity, i.e. x, y , u and v are all odd or all
even.
R
x
u S v
y
Shortlisted problems solutions 35
C2. Let n be a positive integer. Dene a
hameleon to be any sequen
e of 3n letters, with
exa
tly n o
urren
es of ea
h of the letters a, b, and c. Dene a swap to be the transposition of
two adja
ent letters in a
hameleon. Prove that for any
hameleon X , there exists a
hameleon Y
2
su
h that X
annot be
hanged to Y using fewer than 3n {2 swaps.
(Australia)
Solution 1. To start, noti
e that the swap of two identi
al letters does not
hange a
hameleon,
so we may assume there are no su
h swaps.
For any two
hameleons X and Y , dene their distan
e dpX, Y q to be the minimal number
of swaps needed to transform X into Y (or vi
e versa). Clearly, dpX, Y q ` dpY, Zq ě dpX, Zq
for any three
hameleons X , Y , and Z .
Lemma. Consider two
hameleons
P “ loomoon cc . . . c
bb . . . b loomoon
aa . . . a loomoon and Q “ loomoon
cc . . . c loomoon aa . . . a .
bb . . . b loomoon
n n n n n n
statement.
Comment 1. The problem may be reformulated in a graph language. Constru
t a graph G with the
hameleons as verti
es, two verti
es being
onne
ted with an edge if and only if these
hameleons dier
by a single swap. Then dpX, Y q is the usual distan
e between the verti
es X and Y in this graph.
Re
all that the radius of a
onne
ted graph G is dened as
So we need to prove that the radius of the
onstru
ted graph is at least 3n2 {2.
It is well-known that the radius of any
onne
ted graph is at least the half of its diameter (whi
h
is simply maxu,vPV dpu, vq). Exa
tly this fa
t has been used above in order to nish the solution.
Solution 2. We use the notion of distan
e from Solution 1, but provide a dierent lower
bound for it.
In any
hameleon X, we enumerate the positions in it from left to right by 1, 2, . . . , 3n.
Dene sc pXq as the sum of positions o
upied by c. The value of sc
hanges by at most 1 on
ea
h swap, but this fa
t alone does not su
e to solve the problem; so we need an improvement.
For every
hameleon X, Xc the sequen
e obtained from X by removing all n
denote by
letters c. Enumerate the positions in Xc from left to right by 1, 2, . . . , 2n, and dene sc,b pXq
as the sum of positions in Xc o
upied by b. (In other words, here we
onsider the positions of
the b's relatively to the a's only.) Finally, denote
1
Now
onsider any swap
hanging a
hameleon X to X . If no letter c is involved into this
1
swap, then sc pXq “ sc pX q; on the other hand, exa
tly one letter b
hanges its position in Xc , so
|sc,b pXq ´ sc,b pX 1 q| “ 1. If a letter c is involved into a swap, then Xc “ Xc1 , so sc,b pXq “ sc,b pX 1 q
1 1 1
and |sc pXq ´ sc pX q| “ 1. Thus, in all
ases we have d pX, X q “ 1.
1
As in the previous solution, this means that dpX, Y q ě d pX, Y q for any two
hameleons X
1 2
and Y . Now, for any
hameleon X we will indi
ate a
hameleon Y with d pX, Y q ě 3n {2, thus
nishing the solution.
npn`1q
The fun
tion sc attains all integer values from 1 ` ¨ ¨ ¨ ` n “ to p2n ` 1q ` ¨ ¨ ¨ ` 3n “
2
2 npn`1q 2 npn`1q
2n ` 2 . If sc pXq ď n ` 2 , then we put the letter c into the last n positions in Y ;
otherwise we put the letter c into the rst n positions in Y . In either
ase we already have
|sc pXq ´ sc pY q| ě n2 .
npn`1q npn`1q n2
Similarly, sc,b ranges from
2
2
to n `
2
. So, if sc,b pXq ď
2
` npn`1q
2
, then we put
the letter b into the last n positions in Y whi
h are still free; otherwise, we put the letter b into
the rst n su
h positions. The remaining positions are o
upied by a. In any
ase, we have
2 2 2
|sc,b pXq ´ sc,b pY q| ě n2 , thus d1 pX, Y q ě n2 ` n2 “ 3n2 , as desired.
Comment 2. The two solutions above used two lower bounds |f pXq ´ f pY q| and d1 pX, Y q for the
number dpX, Y q. One may see that these bounds are
losely related to ea
h other, as
npn ` 1q npn ` 1q
fa,c pXq ` fb,c pXq “ sc pXq ´ and fa,b pXq “ sc,b pXq ´ .
2 2
One
an see that, e.g., the bound d1 pX, Y q
ould as well be used in the proof of the lemma in Solution 1.
Let us des
ribe here an even sharper bound whi
h also
an be used in dierent versions of the
solutions above.
In ea
h
hameleon X , enumerate the o
urren
es of a from the left to the right as a1 , a2 , . . . , an .
Sin
e we got rid of swaps of identi
al letters, the relative order of these letters remains the same during
the swaps. Perform the same operation with the other letters, obtaining new letters b1 , . . . , bn and
c1 , . . . , cn . Denote by A the set of the 3n obtained letters.
Sin
e all 3n letters be
ame dierent, for any
hameleon X and any s P A we may dene the
position Ns pXq of s in X (thus 1 ď Ns pXq ď 3n). Now, for any two
hameleons X and Y we say that
a pair of letters ps, tq P Aˆ A is an pX, Y q-inversion if Ns pXq ă Nt pXq but Ns pY q ą Nt pY q, and dene
d˚ pX, Y q to be the number of pX, Y q-inversions. Then for any two
hameleons Y and Y 1 diering by a
single swap, we have |d˚ pX, Y q ´ d˚ pX, Y 1 q| “ 1. Sin
e d˚ pX, Xq “ 0, this yields dpX, Y q ě d˚ pX, Y q
for any pair of
hameleons X and Y . The bound d˚ may also be used in both Solution 1 and Solution 2.
Comment 3. In fa
t, one may prove that the distan
e d˚ dened in the previous
omment
oin
ides
with d. Indeed, if X ‰ Y , then there exist an pX, Y q-inversion ps, tq. One
an show that su
h s and t
may be
hosen to o
upy
onse
utive positions in Y . Clearly, s and t
orrespond to dierent letters
among ta, b, cu. So, swapping them in Y we get another
hameleon Y 1 with d˚ pX, Y 1 q “ d˚ pX, Y q ´ 1.
Pro
eeding in this manner, we may
hange Y to X in d˚ pX, Y q steps.
Using this fa
t, one
an show that the estimate in the problem statement is sharp for all n ě 2.
(For n “ 1 it is not sharp, sin
e any permutation of three letters
an be
hanged to an opposite one in
no less than three swaps.) We outline the proof below.
For any k ě 0, dene
larger than 3n {2 .
For any distin
t letters u, v P ta, b, cu and any two
hameleons X and Y , we dene d˚u,v pX, Y q as
the number of pX, Y q-inversions ps, tq su
h that s and t are instan
es of u and v (in any of the two
possible orders). Then d˚ pX, Y q “ d˚a,b pX, Y q ` d˚b,c pX, Y q ` d˚c,a pX, Y q.
Shortlisted problems solutions 37
We start with the
ase when n “ 2k is even; denote X “ X2k . We show that d˚a,b pX, Y q ď 2k 2
for any
hameleon Y ; this yields the required estimate. Pro
eed by the indu
tion on k with the trivial
base
ase k “ 0. To perform the indu
tion step, noti
e that d˚a,b pX, Y q is indeed the minimal number of
swaps needed to
hange Yc into Xc . One may show that moving a1 and a2k in Y onto the rst and the
last positions in Y , respe
tively, takes at most 2k swaps, and that subsequent moving b1 and b2k onto
the se
ond and the se
ond last positions takes at most 2k ´ 2 swaps. After performing that, one may
delete these letters from both Xc and Yc and apply the indu
tion hypothesis; so Xc
an be obtained
from Yc using at most 2pk ´ 1q2 ` 2k ` p2k ´ 2q “ 2k 2 swaps, as required.
If n “ 2k ` 3 is odd, the proof is similar but more te
hni
ally involved. Namely, we
laim that
d˚a,b pX2k`3 , Y q ď 2k2 ` 6k ` 5 for any
hameleon Y , and that the equality is a
hieved only if Yc “
bb . . . b aa . . . a. The proof pro
eeds by a similar indu
tion, with some
are taken of the base
ase, as
well as of extra
ting the equality
ase. Similar estimates hold for d˚b,c and d˚c,a . Summing three su
h
estimates, we obtain R 2V
˚ 2 3n
d pX2k`3 , Y q ď 3p2k ` 6k ` 5q “ ` 1,
2
whi
h is by 1 more than we need. But the equality
ould be a
hieved only if Yc “ bb . . . b aa . . . a
and, similarly, Yb “ aa . . . a cc . . . c and Ya “ cc . . . c bb . . . b. Sin
e these three equalities
annot hold
simultaneously, the proof is nished.
38 IMO 2017, Rio de Janeiro
C3. Sir Alex plays the following game on a row of 9
ells. Initially, all
ells are empty. In
ea
h move, Sir Alex is allowed to perform exa
tly one of the following two operations:
(1) Choose any number of the form 2j , where j is a non-negative integer, and put it into an
empty
ell.
(2) Choose two (not ne
essarily adja
ent)
ells with the same number in them; denote that
j j`1
number by 2 . Repla
e the number in one of the
ells with 2 and erase the number in
the other
ell.
At the end of the game, one
ell
ontains the number 2n , where n is a given positive integer,
while the other
ells are empty. Determine the maximum number of moves that Sir Alex
ould
have made, in terms of n.
(Thailand)
ř8 ` n˘
Answer: 2 j“0 j
´ 1.
Solution 1. We will solve a more general problem, repla
ing the row of 9
ells with a row of k
ells, where k is a positive integer. Denote by mpn, kq the maximum possible number of moves
Sir Alex
an make starting with a row of k empty
ells, and ending with one
ell
ontaining
n
the number 2 and all the other k ´ 1
ells empty. Call an operation of type (1) an , insertion
and an operation of type (2) a merge.
Only one move is possible when k “ 1, so we have mpn, 1q “ 1. From now on we
onsider
k ě 2, and we may assume Sir Alex's last move was a merge. Then, just before the last move,
n´1
there were exa
tly two
ells with the number 2 , and the other k ´ 2
ells were empty.
n´1
Paint one of those numbers 2 blue, and the other one red. Now tra
e ba
k Sir Alex's
moves, always painting the numbers blue or red following this rule: if a and b merge into c,
paint a and b with the same
olor as c. Noti
e that in this ba
kward pro
ess new numbers are
produ
ed only by reversing merges, sin
e reversing an insertion simply means deleting one of
the numbers. Therefore, all numbers appearing in the whole pro
ess will re
eive one of the two
olors.
Sir Alex's rst move is an insertion. Without loss of generality, assume this rst number
inserted is blue. Then, from this point on, until the last move, there is always at least one
ell
with a blue number.
Besides the last move, there is no move involving a blue and a red number, sin
e all merges
involves numbers with the same
olor, and insertions involve only one number. Call an insertion
of a blue number or merge of two blue numbers a blue move, and dene a red move analogously.
The whole sequen
e of blue moves
ould be repeated on another row of k
ells to produ
e
n´1
one
ell with the number 2 and all the others empty, so there are at most mpn ´ 1, kq blue
moves.
Now we look at the red moves. Sin
e every time we perform a red move there is at least
one
ell o
upied with a blue number, the whole sequen
e of red moves
ould be repeated on a
n´1
row of k ´ 1
ells to produ
e one
ell with the number 2 and all the others empty, so there
are at most mpn ´ 1, k ´ 1q red moves. This proves that
It follows that
forn ě 1 and k ě 2.
k “ 1 or n “ 0, we must insert 2n on our rst move and immediately get the nal
If
onguration, so mp0, kq “ 1 and mpn, 1q “ 1, for n ě 0 and k ě 1. These initial values,
together with the re
urren
e relation (1), determine mpn, kq uniquely.
n
˙ ÿ ˆn˙
ÿˆ
k´2
k´1
mpn ` 1, kq “ mpn, kq ` mpn, k ´ 1q ` 1 “ 2 ´1`2 ´1`1
j“0
j j“0
j
k´1
ÿ n ˆ ˙ k´1
ÿ ˆ ˙ k´1
ÿ ˆˆ ˙ ˆ ˙˙ ÿ ˆn ` 1˙
k´1
n n n
“2 `2 ´1“2 ` ´1“2 ´ 1,
j“0
j j“0
j´1 j“0
j j´1 j“0
j
Comment 1. After dedu
ing the re
urren
e relation (1), it may be
onvenient to homogenize the
re
urren
e relation by dening hpn, kq “ mpn, kq ` 1. We get the new relation
Comment 2. We
an use a generating fun
tion to nd the answer without guessing. We work with
ř hpn, 0q “ 0 so that (3) is valid for k “ 1 as well. Now
the homogenized re
urren
e relation (3). Dene
we set up the generating fun
tion f px, yq “ n,kě0 hpn, kqxn y k . Multiplying the re
urren
e relation (3)
by xn y k and summing over n, k ě 1, we get
ÿ ÿ ÿ
hpn, kqxn y k “ x hpn ´ 1, kqxn´1 y k ` xy hpn ´ 1, k ´ 1qxn´1 y k´1 .
n,kě1 n,kě1 n,kě1
Completing the missing terms leads to the following equation on f px, yq:
ÿ ÿ ÿ
f px, yq ´ hpn, 0qxn ´ hp0, kqy k “ xf px, yq ´ x hpn, 0qxn ` xyf px, yq.
ně0 kě1 ně0
and extra ting the oe ient of y k in this last expression we nally obtain the value for hpn, kq,
ÿˆ
k´1
n
˙
hpn, kq “ 2 .
j“0
j
ÿˆ
k´1
n
˙
mpn, kq “ 2 ´ 1.
j“0
j
The generating fun
tion approa
h also works if applied to the non-homogeneous re
urren
e rela-
tion (1), but the
omputations are less straightforward.
Solution 2. Dene merges and insertions as in Solution 1. After ea
h move made by Sir Alex
we
ompute the number N of empty
ells, and the sum S of all the numbers written in the
ells. Insertions always in
rease S by some power of 2, and in
rease N exa
tly by 1. Merges do
not
hange S and de
rease N exa
tly by 1. Sin
e the initial value of N is 0 and its nal value
is 1, the total number of insertions ex
eeds that of merges by exa
tly one. So, to maximize the
number of moves, we need to maximize the number of insertions.
We will need the following lemma.
Lemma. If the binary representation of a positive integer A has d nonzero digits, then A
annot
be represented as a sum of fewer than d powers of 2. Moreover, any representation of A as a
sum of d powers of 2 must
oin
ide with its binary representation.
Proof. Let s be the minimum number of summands in all possible representations of A as sum
of powers of 2. Suppose there is su
h a representation with s summands, where two of the
summands are equal to ea
h other. Then, repla
ing those two summands with the result of
their sum, we obtain a representation with fewer than s summands, whi
h is a
ontradi
tion.
We dedu
e that in any representation with s summands, the summands are all distin
t, so any
su
h representation must
oin
ide with the unique binary representation of A, and s “ d. l
Now we split the solution into a sequen
e of
laims.
Claim 1. After every move, the number S is the sum of at most k ´ 1 distin
t powers of 2.
Proof. If S is the sum of k (or more) distin
t powers of 2, the Lemma implies that the k
ells
are lled with these numbers. This is a
ontradi
tion sin
e no more merges or insertions
an
be made. l
n
Let Apn, k ´ 1q denote the set of all positive integers not ex
eeding 2 with at most k ´ 1
nonzero digits in its base 2 representation. Sin
e every insertion in
reases the value of S , by
Claim 1, the total number of insertions is at most |Apn, k ´ 1q|. We pro
eed to prove that it is
possible to a
hieve this number of insertions.
Proof. Suppose S “ aj . Performing all possible merges, we eventually get dierent powers of 2
in all nonempty
ells. After that, by Claim 1 there will be at least one empty
ell, in whi
h we
want to insert aj`1 ´ aj . aj`1 ´ aj is a power of 2.
It remains to show that
For this purpose, we noti
e that if aj has less than k ´ 1 nonzero digits in base 2 then
aj`1 “ aj ` 1. Otherwise, we have aj “ 2bk´1 ` ¨ ¨ ¨ ` 2b2 ` 2b1 with b1 ă b2 ă ¨ ¨ ¨ ă bk´1 . Then,
b
adding any number less than 2 1 to aj will result in a number with more than k ´ 1 nonzero
Shortlisted problems solutions 41
binary digits. On the other hand,aj ` 2b1 is a sum of k powers of 2, not all distin
t, so by the
b
Lemma it will be a sum of less then k distin
t powers of 2. This means that aj`1 ´ aj “ 2 1 ,
ompleting the proof. l
Claims 1 and 2 prove that the maximum number of insertions is |Apn, k ´ 1q|. We now
ompute this number.
ř ` ˘
Claim 3. |Apn, k ´ 1q| “ j“0
k´1 n
j
.
C4. Let N ě2 be an integer. NpN ` 1q so
er players, no two of the same height, stand
in a row in some order. Coa
h Ralph wants to remove NpN ´ 1q people from this row so that
in the remaining row of 2N players, no one stands between the two tallest ones, no one stands
between the third and the fourth tallest ones, . . . , and nally no one stands between the two
shortest ones. Show that this is always possible.
(Russia)
Solution 1. Split the row into N blo
ks with N `1
onse
utive people ea
h. We will show
how to remove N ´1 people from ea
h blo
k in order to satisfy the
oa
h's wish.
First,
onstru
t a pN ` 1q ˆ N matrix where xi,j is the height of the ith tallest person of
th
the j blo
kin other words, ea
h
olumn lists the heights within a single blo
k, sorted in
de
reasing order from top to bottom.
We will reorder this matrix by repeatedly swapping whole
olumns. First, by
olumn per-
mutation, make sure that x2,1 “ maxtx2,i : i “ 1, 2, . . . , Nu (the rst
olumn
ontains the
largest height of the se
ond row). With the rst
olumn xed, permute the other ones so that
x3,2 “ maxtx3,i : i “ 2, . . . , Nu (the se
ond
olumn
ontains the tallest person of the third row,
rst
olumn ex
luded). In short, at step k (k “ 1, 2, . . . , N ´ 1), we permute the
olumns from
k to N so that xk`1,k “ maxtxi,k : i “ k, k ` 1, . . . , Nu, and end up with an array like this:
ą
x2,1 ą x2,2 x2,3 ¨ ¨ ¨ x2,N ´1 x2,N
ą
ą
ą
x3,1 x3,2 ą x3,3 ¨ ¨ ¨ x3,N ´1 x3,N
ą
ą
. . . .. . .
.. .. .. . .. ..
ą
ą
everyone s
anned up to that point. On
e again we end up with two people who are next to
ea
h other in the remaining row and whose heights
annot be separated by anyone else who
remains (sin
e the rest of their group is gone). After pi
king these 2 pairs, we still have N ´2
groups with at least N ´1 people ea
h.
If we repeat the s
anning pro
ess a total of N times, it is easy to
he
k that we will end
up with 2 people from ea
h group, for a total of 2N people remaining. The height order is
guaranteed by the grouping, and the s
anning
onstru
tion from left to right guarantees that
ea
h pair from a group stand next to ea
h other in the nal row. We are done.
Solution 3. This is essentially the same as solution 1, but presented indu
tively. The essen
e
of the argument is the following lemma.
Lemma. Assume that we have N disjoint groups of at least N `1 people in ea
h, all people
have distin
t heights. Then one
an
hoose two people from ea
h group so that among the
hosen people, the two tallest ones are in one group, the third and the fourth tallest ones are
in one group, . . . , and the two shortest ones are in one group.
Comment 1. One
an identify ea
h person with a pair of indi
es pp, hq (p, h P t1, 2, . . . , N pN ` 1qu)
so that the pth personin the row (say, from left to right) is the hth tallest person in the group. Say
that pa, bq separates px1 , y1 q and px2 , y2 q whenever a is stri
tly between x1 and y1 , or b is stri
tly
between x2 and y2 . So the
oa
h wants to pi
k 2N people ppi , hi qpi “ 1, 2, . . . , 2N q su
h that no
hosen
person separates pp1 , h1 q from pp2 , h2 q, no
hosen person separates pp3 , h3 q and pp4 , h4 q, and so on.
This formulation reveals a duality between positions and heights. In that sense, solutions 1 and 2 are
dual of ea
h other.
C5. A hunter and an invisible rabbit play a game in the Eu
lidean plane. The hunter's
starting point H0
oin
ides with the rabbit's starting point R0 . In the nth round of the game
(n ě 1), the following happens.
(1) First the invisible rabbit moves se
retly and unobserved from its
urrent point Rn´1 to
some new point Rn with Rn´1 Rn “ 1.
1
(2) The hunter has a tra
king devi
e (e.g. dog) that returns an approximate position Rn of
1
the rabbit, so that Rn Rn ď 1.
(3) The hunter then visibly moves from point Hn´1 to a new point Hn with Hn´1 Hn “ 1.
9
Is there a strategy for the hunter that guarantees that after 10 su
h rounds the distan
e
between the hunter and the rabbit is below 100?
(Austria)
Answer: There is no su
h strategy for the hunter. The rabbit wins".
Solution. If the answer were yes", the hunter would have a strategy that would work", no
matter how the rabbit moved or where the radar pings Rn1 appeared. We will show the opposite:
with bad lu
k from the radar pings, there is no strategy for the hunter that guarantees that
9
the distan
e stays below 100 in 10 rounds.
So, let dn
be the distan
e between the hunter and the rabbit after n rounds. Of
ourse, if
9
dn ě 100 for any n ă 10 , the rabbit has won it just needs to move straight away from the
hunter, and the distan
e will be kept at or above 100 thereon.
We will now show that, while dn ă 100, whatever given strategy the hunter follows, the
2 1
rabbit has a way of in
reasing dn by at least every 200 rounds (as long as the radar pings are
2
2 4 4 6 9
lu
ky enough for the rabbit). This way, dn will rea
h 10 in less than 2 ¨ 10 ¨ 200 “ 4 ¨ 10 ă 10
rounds, and the rabbit wins.
Suppose the hunter is at Hn and the rabbit is at Rn . Suppose even that the rabbit reveals
its position at this moment to the hunter (this allows us to ignore all information from previous
radar pings). Let r be the line Hn Rn , and Y1 and Y2 be points whi
h are 1 unit away from r
and 200 units away from Rn , as in the gure below.
Y1
200 y 1
r dn 200 − dn ε
Hn Rn H ′ Z R′
200 y 1
Y2
The rabbit's plan is simply to
hoose one of the points Y1 or Y2 and hop 200 rounds straight
towards it. Sin
e all hops stay within 1 distan
e unit from r , it is possible that all radar pings
stay on r. In parti
ular, in this
ase, the hunter has no way of knowing whether the rabbit
hose Y1 or Y2 .
Looking at su
h pings, what is the hunter going to do? If the hunter's strategy tells him to
1
go 200rounds straight to the right, he ends up at point H in the gure. Note that the hunter
does not have a better alternative! Indeed, after these 200 rounds he will always end up at
1
a point to the left of H . If his strategy took him to a point above r , he would end up even
further from Y2 ; and if his strategy took him below r, he would end up even further from Y1 .
In other words, no matter what strategy the hunter follows, he
an never be sure his distan
e
def 1 1
to the rabbit will be less than y “ H Y1 “ H Y2 after these 200 rounds.
2 1
To estimate y , we take Z as the midpoint of segment Y1 Y2 , we take R as a point 200 units
1 1 1
to the right of Rn and we dene ε “ ZR (note that H R “ dn ). Then
Shortlisted problems solutions 45
Comment 1. Many dierent versions of the solution above
an be found by repla
ing 200 with some
other number N for the number of hops the rabbit takes between reveals. If this is done, we have:
a 1 1
ε“N´ N2 ´ 1 ą ? ą
N ` N2 ´ 1 2N
and
ε2 ` 1 “ 2N ε,
so, as long as N ą dn , we would nd
N ´ dn
y 2 “ d2n ` εp2N ´ 2dn q ą d2n ` .
N
For example, taking N “ 101 is already enoughthe squared distan
e in
reases by at least 101
1
every
101 rounds, and 101 ¨ 10 “ 1.0201 ¨ 10 ă 10 rounds are enough for the rabbit. If the statement is
2 4 8 9
Comment 2. The original statement asked whether the distan
e
ould be kept under 1010 in 10100
rounds.
46 IMO 2017, Rio de Janeiro
Solution 1. Call a nˆnˆ1 box an x-box, a y -box, or a z -box, a
ording to the dire
tion of
its short side. Let C be the number of
olors in a valid
onguration. We start with the upper
bound for C.
Let C1 , C2 , and C3 be the sets of
olors whi
h appear in the big
ube exa
tly on
e, exa
tly
twi
e, and at least thri
e, respe
tively. Let Mi be the set of unit
ubes whose
olors are in Ci ,
and denote ni “ |Mi |.
Consider any x-box X , and let Y and Z be a y- and a z -box
ontaining the same set of
olors asX does.
Claim. 4|X X M1 | ` |X X M2 | ď 3n ` 1.
Proof. We distinguish two
ases.
Case 1: X X M1 ‰ ∅.
A
ube from X X M1 should appear in all three boxes X , Y , and Z , so it should lie in
X X Y X Z. X X M1 “ X X Y X Z and |X X M1 | “ 1.
Thus
Consider now the
ubes in X X M2 . There are at most 2pn ´ 1q of them lying in X X Y or
X X Z (be
ause the
ube from X X Y X Z is in M1 ). Let a be some other
ube from X X M2 .
1
Re
all that there is just one other
ube a sharing a
olor with a. But both Y and Z should
1 1 1
ontain su
h
ube, so a P Y X Z (but a R X X Y X Z ). The map a ÞÑ a is
learly inje
tive,
so the number of
ubes a we are interested in does not ex
eed |pY X Zq z X| “ n ´ 1. Thus
|X XM2 | ď 2pn´1q`pn´1q “ 3pn´1q, and hen
e 4|X XM1 |`|X XM2 | ď 4`3pn´1q “ 3n`1.
Case 2: X X M1 “ ∅.
In this
ase, the same argument applies with several
hanges. Indeed,X X M2
ontains
at most 2n ´ 1
ubes from X X Y or X X Z . Any other
ube a in X X M2
orresponds to
some a P Y X Z (possibly with a1 P X ), so there are at most n of them. All this results
1
in
|X X M2 | ď p2n ´ 1q ` n “ 3n ´ 1, whi
h is even better than we need (by the assumptions of
our
ase). l
Summing up the inequalities from the Claim over all x-boxes X , we obtain
` ˘
3. 2 n3 triplets of the form Ti,j,k “ tpi, j, kq, pj, k, iq, pk, i, jqu (with 1ďiăj ăk ďn or
1 ď i ă k ă j ď n).
One may easily see that the ith boxes of ea
h orientation
ontain the same set of
olors, and
that
3npn ´ 1q npn ´ 1qpn ´ 2q npn ` 1qp2n ` 1q
n` ` “
2 3 6
olors are used, as required.
Solution 2. We will approa
h a new version of the original problem. In this new version, ea
h
ube may have a
olor, or be invisible (not both). Now we make sets of
olors for ea
h nˆnˆ1
box as before (where invisible" is not
onsidered a
olor) and group them by orientation, also
as before. Finally, we require that, for every non-empty set in any group, the same set must
appear in the other 2 groups. What is the maximum number of
olors present with these new
requirements?
Let us
all strange a big nˆnˆn
ube whose painting s
heme satises the new requirements,
and let D be the number of
olors in a strange
ube. Note that any
ube that satises the
original requirements is also strange, so maxpDq is an upper bound for the original answer.
Claim. D ď npn`1qp2n`1q
6
.
C7. For any nite sets X and Y of positive integers, denote by fX pkq the k th smallest
positive integer not in X, and let
X ˚ Y “ X Y tfX pyq : y P Y u.
Let A be a set ofa ą 0 positive integers, and let B be a set of bą0 positive integers. Prove
that if A ˚ B “ B ˚ A, then
A ˚ pA ˚ ¨ ¨ ¨ ˚ pA ˚ pA ˚ Aqq . . . q “ looooooooooooooooooomooooooooooooooooooon
looooooooooooooooooomooooooooooooooooooon B ˚ pB ˚ ¨ ¨ ¨ ˚ pB ˚ pB ˚ Bqq . . . q .
A appears b times B appears a times
(U.S.A.)
Solution 1. g : Zą0 Ñ Zą0 and any subset X Ă Zą0 , we dene gpXq “
For any fun
tion
tgpxq : x P Xu. We have that the image of fX is fX pZą0 q “ Zą0 z X . We now show a general
lemma about the operation ˚, with the goal of showing that ˚ is asso
iative.
Lemma 1. Let X and Y be nite sets of positive integers. The fun
tions fX˚Y and fX ˝ fY are
equal.
Proof. We have
fX˚Y pZą0 q “ Zą0 zpX ˚Y q “ pZą0 zXqzfX pY q “ fX pZą0 qzfX pY q “ fX pZą0 zY q “ fX pfY pZą0 qq.
Thus, the fun
tions fX˚Y and fX ˝ fY are stri
tly in
reasing fun
tions with the same range.
Be
ause a stri
tly fun
tion is uniquely dened by its range, we have fX˚Y “ fX ˝ fY . l
Lemma 1 implies that ˚ is asso
iative, in the sense that pA ˚ Bq ˚ C “ A ˚ pB ˚ Cq for any
nite sets A, B , and C of positive integers. We prove the asso
iativity by noting
Zą0 z ppA ˚ Bq ˚ Cq “ fpA˚Bq˚C pZą0 q “ fA˚B pfC pZą0 qq “ fA pfB pfC pZą0 qqq
X ˚k “ loooooooooooooooooooomoooooooooooooooooooon
X ˚ pX ˚ ¨ ¨ ¨ ˚ pX ˚ pX ˚ Xqq . . . q .
X appears k times
Our goal is then to show that A ˚ B “ B ˚ A implies A˚b “ B ˚a . We will do so via the following
general lemma.
Lemma 2. Suppose that X and Y are nite sets of positive integers satisfying X ˚Y “ Y ˚X
and |X| “ |Y |. Then, we must have X “ Y.
Proof. Assume that X and Y are not equal. Let s be the largest number in exa
tly one of
X and Y. Without loss of generality, say that s P X z Y . The number fX psq
ounts the sth
number not in X, whi
h implies that
ˇ ˇ
fX psq “ s ` ˇX X t1, 2, . . . , fX psquˇ. (1)
( (
fX psq ` 1, fX psq ` 2, . . . X X “ fX psq ` 1, fX psq ` 2, . . . X Y,
Comment 1. Taking A “ X ˚k and B “ X ˚l generates many non-trivial examples where A˚B “ B˚A.
There are also other examples not of this form. For example, if A “ t1, 2, 4u and B “ t1, 3u, then
A ˚ B “ t1, 2, 3, 4, 6u “ B ˚ A.
Solution 2. We will use Lemma 1 from Solution 1. Additionally, let X ˚k be dened as in
Solution 1. If X and Y are nite sets, then
where the rst equivalen
e is be
ause fX and fY are stri
tly in
reasing fun
tions, and the se
ond
equivalen
e is be
ause fX pZą0 q “ Zą0 z X and fY pZą0 q “ Zą0 z Y .
Denote g “ fA and h “ fB . The given relation A ˚ B “ B ˚ A is equivalent to fA˚B “ fB˚A
be
ause of (3), and by Lemma 1 of the rst solution, this is equivalent to g ˝h “ h˝g . Similarly,
˚b
the required relation A “ B ˚a is equivalent to g b “ ha . We will show that
g b pnq “ ha pnq (4)
To start, we
laim that (4) holds for all su
iently large n. Indeed, let p and q be the
maximal elements of A and B , respe
tively; we may assume that p ě q . Then, for every n ě p
b a
we have gpnq “ n ` a and hpnq “ n ` b, when
e g pnq “ n ` ab “ h pnq, as was
laimed.
b
In view of this
laim, if (4) is not identi
ally true, then there exists a maximal s with g psq ‰
ha psq. Without loss of generality, we may assume that gpsq ‰ s, for if we had gpsq “ hpsq “ s,
then s would satisfy (4). As g is in
reasing, we then have gpsq ą s, so (4) holds for n “ gpsq.
But then we have
Claim. For every i, there exists some mi and ci su
h that for all m ą mi , we have that sm,i “ t`mn´ci .
Furthermore, the ci do not depend on the
hoi
e of T .
First, we show that this
laim implies Lemma 2. We may
hoose T “ X and T “ Y . Then, there
is some m1 su
h that for all m ě m1 , we have
Be
ause u is the minimum element of X , v is the minimum element of Y , and u ď v , we have that
˜ ¸ ˜ ¸
8
ď 8
ď
1 ` 1 ˘
fX ˚m pXq Y X ˚m “ fpY ˚X ˚pm´1q q pXq Y Y ˚ X ˚pm ´1q “ tu, u ` 1, . . . u,
m“m1 m“m1
and in both the rst and se
ond expressions, the unions are of pairwise distin
t sets. By (5), we obtain
1 1 1 1
X ˚m “ Y ˚ X ˚pm ´1q . Now, be
ause X and Y
ommute, we get X ˚m “ X ˚pm ´1q ˚ Y , and so X “ Y .
We now prove the
laim.
Proof of the
laim. We indu
t downwards on i, rst proving the statement for i “ n, and so on.
Assume that m is
hosen so that all elements of Sm are greater than all elements of T (whi
h is
possible be
ause T is nite). For i “ n, we have that sm,n ą sk,n for every k ă m. Thus, all pm ´ 1qn
numbers of the form sk,u for k ă m and 1 ď u ď n are less than sm,n . We then have that sm,n is the
ppm ´ 1qn ` xn qth number not in T , whi
h is equal to t ` pm ´ 1qn ` xn. So we may
hoose cn “ xn ´ n,
whi
h does not depend on T , whi
h proves the base
ase for the indu
tion.
For i ă n, we have again that all elements sm,j for j ă i and sp,i for p ă m are less than sm,i ,
so sm,i is the ppm ´ 1qi ` xi qth element not in T or of the form sp,j for j ą i and p ă m. But by
the indu
tive hypothesis, ea
h of the sequen
es sp,j is eventually periodi
with period n, and thus the
sequen
e sm,i su
h must be as well. Sin
e ea
h of the sequen
es sp,j ´ t with j ą i eventually do not
depend on T , the sequen
e sm,i ´ t eventually does not depend on T either, so the indu
tive step is
omplete. This proves the
laim and thus Lemma 2. l
Shortlisted problems solutions 51
C8. Let n be a given positive integer. In the Cartesian plane, ea
h latti
e point with
nonnegative
oordinates initially
ontains a buttery, and there are no other butteries. The
neighborhood of a latti
e point c
onsists of all latti
e points within the axis-aligned p2n ` 1q ˆ
p2n ` 1q square
entered at c, apart from c itself. We
all a buttery lonely,
rowded, or
om-
fortable, depending on whether the number of butteries in its neighborhood N is respe
tively
less than, greater than, or equal to half of the number of latti
e points in N.
Every minute, all lonely butteries y away simultaneously. This pro
ess goes on for as
long as there are any lonely butteries. Assuming that the pro
ess eventually stops, determine
the number of
omfortable butteries at the nal state.
(Bulgaria)
2
Answer: n ` 1.
Solution. We always identify a buttery with the latti
e point it is situated at. For two points p
and q, we write pěq if ea
h
oordinate of p is at least the
orresponding
oordinate of q. Let
O be the origin, and let Q be the set of initially o
upied points, i.e., of all latti
e points with
nonnegative
oordinates. Let RH “ tpx, 0q : x ě 0u and RV “ tp0, yq : y ě 0u be the sets of
the latti
e points lying on the horizontal and verti
al boundary rays of Q. Denote by Npaq the
neighborhood of a latti
e point a.
1. Initial observations. We
all a set of latti
e points up-right
losed if its points stay in the
set after being shifted by any latti
e ve
tor pi, jq with i, j ě 0. Whenever the butteries form a
up-right
losed set S , we have |Nppq X S| ě |Npqq X S| for any two points p, q P S with p ě q .
So, sin
e Q is up-right
losed, the set of butteries at any moment also preserves this property.
We assume all forth
oming sets of latti
e points to be up-right
losed.
When speaking of some set S of latti
e points, we
all its points lonely,
omfortable, or
rowded with respe
t to this set (i.e., as if the butteries were exa
tly at all points of S ). We
all a set S Ă Q stable if it
ontains no lonely points. In what follows, we are interested only
in those stable sets whose
omplements in Q are nite, be
ause one
an easily see that only a
nite number of butteries
an y away on ea
h minute.
If the initial set Q of butteries
ontains some stable set S, then,
learly no buttery of
this set will y away. On the other hand, the set F of all butteries in the end of the pro
ess
is stable. This means that F is the largest (with respe
t to in
lusion) stable set within Q, and
we are about to des
ribe this set.
2. A des
ription of a nal set. The following notion will be useful. Let U “ t~u1 , ~u2, . . . , ~ud u
be a set of d pairwise non-parallel latti
e ve
tors, ea
h having a positive x- and a negative
y -
oordinate. Assume that they are numbered in in
reasing order a
ording to slope. We now
ÝÝÝÑ ui
dene a U -
urve to be the broken line p0 p1 . . . pd su
h that p0 P RV , pd P RH , and pi´1 pi “ ~
for all i “ 1, 2, . . . , m (see the Figure below to the left).
~u~u44444
pp00000
~u1
O
~u~u22222 ~u pp1111 d0
~u33333 Kn
−→
d1 D
D
~u1 pp2222 d2
~u2 ~v1 d3
−→
~u3 ~v~v33333
pp33333 ~v4
~u4 O pp4444 r1 r2 r3 r4 (k4 = 3)
Now, let Kn “ tpi, jq : 1 ď i ď n, ´n ď j ď ´1u. Consider all the rays emerging at O and
passing through a point from Kn ; number them as r1 , . . . , rm in in
reasing order a
ording to
ÝÝÑ
slope. Let Ai be the farthest from O latti
e point in ri X Kn , set ki “ |ri X Kn |, let ~ vi “ OAi ,
and nally denote V “ t~ vi : 1 ď i ď mu; see the Figure above to the right. We will
on
entrate
1
on the V -
urve d0 d1 . . . dm ; let D be the set of all latti
e points p su
h that p ě p for some (not
1
ne
essarily latti
e) point p on the V -
urve. In fa
t, we will show that D “ F .
Clearly, the V -
urve is symmetri
in the line y “ x. Denote by D the
onvex hull of D .
3. We prove that the set D
ontains all stable sets. Let S Ă Q be a stable set (re
all that
it is assumed to be up-right
losed and to have a nite
omplement in Q). Denote by S its
onvex hull;
learly, the verti
es of S are latti
e points. The boundary of S
onsists of two rays
(horizontal and verti
al ones) along with some V˚ -
urve for some set of latti
e ve
tors V˚ .
Claim 1. For every ~vi P V , there is a ~vi˚ P V˚
o-dire
ted with ~v with |~vi˚ | ě |~v |.
Proof. Let ℓ be the supporting line of S parallel to ~vi (i.e., ℓ
ontains some point of S , and
the set S lies on one side of ℓ). Take any point b P ℓ X S and
onsider Npbq. The line ℓ splits
the set Npbq z ℓ into two
ongruent parts, one having an empty interse
tion with S . Hen
e, in
order for b not to be lonely, at least half of the set ℓ X Npbq (whi
h
ontains 2ki points) should
lie in S . Thus, the boundary of S
ontains a segment ℓ X S with at least ki ` 1 latti
e points
(in
luding b) on it; this segment
orresponds to the required ve
tor ~ vi˚ P V˚ . l
~v1∗
~v~viiii
|| ~v2∗
b ~v1
∗∗
> k {{zz ~v~viiiii∗∗
> ~v3∗
Kn kiiii p ~v2
pooii ~v4∗
nntts
s }}
ℓ ~v3 p ′
∂S
p
~v4
∂D
Proof of Claim 1 Proof of Claim 2
Claim 3. In the set D, all latti
e points of the V -
urve are
omfortable.
Proof. Let p be any latti
e point of the V -
urve, belonging to some segment di di`1 . Draw the
lineℓ
ontaining this segment. Then ℓ X D
ontains exa
tly ki ` 1 latti
e points, all of whi
h lie
inNppq ex
ept for p. Thus, exa
tly half of the points in Nppq X ℓ lie in D . It remains to show
that all points of Nppq above ℓ lie in D (re
all that all the points below ℓ la
k this property).
Shortlisted problems solutions 53
Noti
e that ea
h ve
tor in V has one
oordinate greater than n{2; thus the neighborhood
of p
ontains parts of at most two segments of the V -
urve su
eeding di di`1 , as well as at most
two of those pre
eding it.
The angles formed by these
onse
utive segments are obtained from those formed by rj and
1
rj´1 (with i ´ 1 ď j ď i ` 2) by shifts; see the Figure below. All the points in Nppq above ℓ
1 1
whi
h
ould lie outside D lie in shifted angles between rj , rj`1 or rj , rj´1 . But those angles,
restri
ted to Nppq, have no latti
e points due to the above remark. The
laim is proved. l
′
ri−1
′
ri+2 di
ri+2
di+1
ri+1 p
Kn ddi+2
i+2
i+2
i+2
ri
ri−1
Proof of Claim 3
Claim 4. All the points of D whi
h are not on the boundary of D are
rowded.
Proof. Let p P D be su
h a point. If it is to the up-right of some point p1 on the
urve, then the
ÝÑ
laim is easy: the shift of p1 p is still in D , and Nppq
ontains at least one more
Npp1 q X D by
point of D either below or to the left of p. So, we may assume that p lies in a right triangle
onstru
ted on some hypothenuse di di`1 . Noti
e here that di , di`1 P Nppq.
Draw a line ℓ k di di`1 through p, and draw a verti
al line h through di ; see Figure below.
Let DL and DR be the parts of D lying to the left and to the right of h, respe
tively (points
of D X h lie in both parts).
ℓ
p p
ddiiiii −→
di
di+1
ddi+1
i+1
i+1
i+1
i+1
Proof of Claim 4
Comment 1. The assumption that the pro
ess eventually stops is unne
essary for the problem, as
one
an see that, in fa
t, the pro
ess stops for every n ě 1. Indeed, the proof of Claims 3 and 4 do not
rely essentially on this assumption, and they together yield that the set D is stable. So, only butteries
that are not in D may y away, and this takes only a nite time.
This assumption has been inserted into the problem statement in order to avoid several te
hni
al
details regarding niteness issues. It may also simplify several other arguments.
Comment 2. The des
ription of the nal set Fp“ Dq seems to be
ru
ial for the solution; the
Problem Sele
tion Committee is not aware of any solution that
ompletely avoids su
h a des
ription.
On the other hand, after the set D has been dened, the further steps may be performed in several
ways. For example, in order to prove that all butteries outside D will y away, one may argue as
follows. (Here we will also make use of the assumption that the pro
ess eventually stops.)
First of all, noti
e that the pro
ess
an be modied in the following manner: Ea
h minute, exa
tly
one of the lonely butteries ies away, until there are no more lonely butteries. The modied pro
ess
ne
essarily stops at the same state as the initial one. Indeed, one may observe, as in solution above,
that the (unique) largest stable set is still the nal set for the modied pro
ess.
Thus, in order to prove our
laim, it su
es to indi
ate an order in whi
h the butteries should y
away in the new pro
ess; if we are able to exhaust the whole set Q z D , we are done.
Let C0 “ d0 d1 . . . dm be the V -
urve. Take its
opy C and shift it downwards so that d0
omes to
some point below the origin O . Now we start moving C upwards
ontinuously, until it
omes ba
k to its
initial position C0 . At ea
h moment when C meets some latti
e points, we
onvin
e all the butteries at
those points to y away in a
ertain order. We will now show that we always have enough arguments
for butteries to do so, whi
h will nish our argument for the
laim..
Let C 1 “ d10 d11 . . . d1m be a position of C when it meets some butteries. We assume that all butteries
under this
urrent position of C were already
onvin
ed enough and ied away. Consider the lowest
buttery b on C 1 . Let d1i d1i`1 be the segment it lies on; we
hoose i so that b ‰ d1i`1 (this is possible
be
ause C as not yet rea
hed C0 ).
Draw a line ℓ
ontaining the segment d1i d1i`1 . Then all the butteries in N pbq are situated on or
above ℓ; moreover, those on ℓ all lie on the segment di di`1 . But this segment now
ontains at most ki
butteries (in
luding b), sin
e otherwise some buttery had to o
upy d1i`1 whi
h is impossible by the
hoi
e of b. Thus, b is lonely and hen
e may be
onvin
ed to y away.
After b has ied away, we swit
h to the lowest of the remaining butteries on C 1 , and so on.
Claims 3 and 4 also allow some dierent proofs whi
h are not presented here.
Shortlisted problems solutions 55
Geometry
G1. ABCDE be a
onvex pentagon su
h that AB “ BC “ CD , =EAB “ =BCD , and
Let
=EDC “ =CBA. Prove that the perpendi
ular line from E to BC and the line segments AC
and BD are
on
urrent.
(Italy)
Solution 1. =A, =B , =C , =D , and =E as internal
Throughout the solution, we refer to
angles of the pentagon ABCDE . Let the perpendi
ular bise
tors of AC and BD , whi
h pass
respe
tively through B and C , meet at point I . Then BD K CI and, similarly, AC K BI .
Hen
e AC and BD meet at the ortho
enter H of the triangle BIC , and IH K BC . It remains
to prove that E lies on the line IH or, equivalently, EI K BC .
Lines IB and IC bise
t =B and =C , respe
tively. Sin
e IA “ IC , IB “ ID , and AB “
BC “ CD , the triangles IAB , ICB and ICD are
ongruent. Hen
e =IAB “ =ICB “
=C{2 “ =A{2, so the line IA bise
ts =A. Similarly, the line ID bise
ts =D . Finally, the
line IE bise
ts =E be
ause I lies on all the other four internal bise
tors of the angles of the
pentagon.
The sum of the internal angles in a pentagon is 5400 , so
In quadrilateral ABIE ,
1 1
=BIE “ 3600 ´ =EAB ´ =ABI ´ =AEI “ 3600 ´ =A ´ =B ´ =E
2 2
0 1 0
“ 360 ´ =A ´ =B ´ p270 ´ =A ´ =Bq
2
1
0
“ 90 ` =B “ 900 ` =IBC,
2
whi
h means that EI K BC ,
ompleting the proof.
D
I
B T C
Solution 2. We present another proof of the fa
t that E lies on line IH . Sin
e all ve internal
bise
tors of ABCDE meet at I, this pentagon has an ins
ribed
ir
le with
enter I. Let this
ir
le tou
h side BC at T .
Applying Brian
hon's theorem to the (degenerate) hexagon ABT CDE we
on
lude that
AC , BD and ET are
on
urrent, so point E also lies on line IHT ,
ompleting the proof.
Shortlisted problems solutions 57
and, similarly, =P AB “ =QCD . Sin
e AB “ CD , the triangles P AB and QCD are
ongruent
with the same orientation. Moreover, P QE is isos
eles with EP “ EQ.
A
D
I
P B C Q
Comment. Even though all three solutions used the point I , there are solutions that do not need it.
We present an outline of su
h a solution: if J is the in
enter of △QCD (with P and Q as dened in
Solution 3), then a simple angle
hasing shows that triangles CJD and BHC are
ongruent. Then if
S is the proje
tion of J onto side CD and T is the orthogonal proje
tion of H onto side BC , one
an
verify that
CD ` DQ ´ QC P B ` BC ` QC PQ
QT “ QC ` CT “ QC ` DS “ QC ` “ “ ,
2 2 2
so T is the midpoint of P Q, and E , H and T all lie on the perpendi
ular bise
tor of P Q.
58 IMO 2017, Rio de Janeiro
ω
A
ω
A R
I
R
I
Ω S
J
Ω
S
J R′
R′ A′
Solution 1 Solution 2
G3. Let O be the
ir
um
enter of an a
ute s
alene triangle ABC . Line OA interse
ts the
altitudes of ABC through B and C at P and Q, respe
tively. The altitudes meet at H . Prove
that the
ir
um
enter of triangle P QH lies on a median of triangle ABC .
(Ukraine)
AB ă AC . We have =P QH “ 900 ´
Solution. Suppose, without loss of generality, that
0 1
=QAB “ 90 ´ =OAB “ 2 =AOB “ =ACB , and similarly =QP H “ =ABC . Thus triangles
ABC and HP Q are similar. Let Ω and ω be the
ir
um
ir
les of ABC and HP Q, respe
tively.
0
Sin
e =AHP “ 90 ´ =HAC “ =ACB “ =HQP , line AH is tangent to ω .
A
A
Ω
P
P ω
H
H T O
Q
Q
C
S B
B M
G4. In triangle ABC , let ω be the ex
ir
le opposite A. Let D , E , and F be the points
where ω is tangent to lines BC , CA, and AB , respe
tively. The
ir
le AEF interse
ts line BC
at P and Q. Let M be the midpoint of AD . Prove that the
ir
le MP Q is tangent to ω .
(Denmark)
Solution 1. Ω the
ir
le AEF P Q, and denote by γ the
ir
le P QM . Let the line
Denote by
AD meet ω T ‰ D . We will show that γ is tangent to ω at T .
again at
1
We rst prove that points P, Q, M, T are
on
y
li
. Let A be the
enter of ω . Sin
e
1 1 1
A E K AE and A F K AF , AA is a diameter in Ω. Let N be the midpoint of DT ; from
A1 D “ A1 T we
an see that =A1 NA “ 900 and therefore N also lies on the
ir
le Ω. Now, from
the power of D with respe
t to the
ir
les γ and Ω we get
DT
DP ¨ DQ “ DA ¨ DN “ 2DM ¨ “ DM ¨ DT,
2
so P, Q, M, T are
on
y
li
.
If EF k BC , then ABC is isos
eles and the problem is now immediate by symmetry.
Otherwise, let the tangent line to ω
T meet line BC at point R. The tangent line segments
at
1
RD and RT have the same length, so A R is the perpendi
ular bise
tor of DT ; sin
e ND “ NT ,
N lies on this perpendi
ular bise
tor.
1 2 1
In right triangle A RD , RD “ RN ¨RA “ RP ¨RQ, in whi
h the last equality was obtained
2
from the power of R with respe
t to Ω. Hen
e RT “ RP ¨ RQ, whi
h implies that RT is also
tangent to γ . Be
ause RT is a
ommon tangent to ω and γ , these two
ir
les are tangent at T .
γ M
P B D C Q R
ω
E
F N
′
A
Solution 3. We give an alternative proof that the
ir
les are tangent at the
ommon point T.
Again, we start from the fa
t that P, Q, M, T are
on
y
li
. Let point O be the midpoint of
1 1 1 1
diameter AA . Then MO is the midline of triangle ADA , so MO k A D . Sin
e A D K P Q,
MO is perpendi
ular to PQ as well.
Looking at
ir
le Ω, whi
h has
enter O , MO K P Q implies that MO is the perpendi
ular
bise
tor of the
hord Ŋ
P Q. Thus M is the midpoint of ar
P Q from γ , and the tangent line m
to γ at M is parallel to P Q.
Ω
m
γ M
P B O D C Q
ω E
F N
′
A
TD
Consider the homothety with
enter T
TM
. It takes D to M , and the line P Q
and ratio
to the line m. Sin
e the
ir
le that is tangent to a line at a given point and that goes through
another given point is unique, this homothety also takes ω (tangent to P Q and going through T )
to γ (tangent to m and going through T ). We
on
lude that ω and γ are tangent at T .
62 IMO 2017, Rio de Janeiro
G5. ABCC1 B1 A1 be a
onvex hexagon su
h that AB “ BC , and suppose that the line
Let
segments AA1 , BB1 , and CC1 have the same perpendi
ular bise
tor. Let the diagonals AC1
and A1 C meet at D , and denote by ω the
ir
le ABC . Let ω interse
t the
ir
le A1 BC1 again
at E ‰ B . Prove that the lines BB1 and DE interse
t on ω .
(Ukraine)
Solution 1. AA1 “ CC1 , then the hexagon is symmetri
about the line BB1 ; in par-
If
ti
ular the
ir
les ABC and A1 BC1 are tangent to ea
h other. So AA1 and CC1 must be
dierent. Sin
e the points A and A1
an be inter
hanged with C and C1 , respe
tively, we may
assume AA1 ă CC1 .
Let R be the radi
al
enter of the
ir
les AEBC and A1 EBC1 , and the
ir
um
ir
le of the
symmetri
trapezoid ACC1 A1 ; that is the
ommon point of the pairwise radi
al axes AC , A1 C1 ,
and BE . By the symmetry of AC and A1 C1 , the point R lies on the
ommon perpendi
ular
bise
tor of AA1 and CC1 , whi
h is the external bise
tor of =ADC .
Let F be the se
ond interse
tion of the line DR and the
ir
le ACD . From the power of
R with respe
t to the
ir
les ω and ACF D we have RB ¨ RE “ RA ¨ RC “ RD ¨ DF , so the
points B, E, D and F are
on
y
li
.
The line RDF is the external bise
tor of =ADC , so the point F bise
ts the ar
CDA Ŕ.
By AB “ BC , on
ir
le ω , the point B is the midpoint of ar
AEC Ő ; let M be the point
Ŋ of ω . Noti
e that the
diametri
ally opposite to B , that is the midpoint of the opposite ar
CA
points B , F and M lie on the perpendi
ular bise
tor of AC , so they are
ollinear.
A A1
E
D
B X B1
ω
C1
C
Finally, let X be the se
ond interse
tion point of ω and the line DE . Sin
e BM is a diameter
in ω, we have =BXM “ 900 . Moreover,
so MX and F D are parallel. Sin
e BX is perpendi
ular to MX and BB1 is perpendi
ular
to F D , this shows that X lies on line BB1 .
Shortlisted problems solutions 63
AS : CS “ AD : CD “ AR : CR “ AE : CE “ AS 1 : CS 1 ,
so indeed S “ S 1.
By =RDS “ =SER “ 900 the points R, S , D and E are
on
y
li
.
A A1
E
D
B S = S′ B1
X
ω C1
C
M
Now let the linesBB1 and DE meet at point X . Noti
e that =EXB “ =EDS be
ause both
BB1 and DS are perpendi
ular to the line DR, we have that =EDS “ =ERS in
ir
le SRDE ,
and =ERS “ =EMB be
ause SR K BM and ER K ME . Therefore, =EXB “ =EMB , so
indeed, the point X lies on ω .
64 IMO 2017, Rio de Janeiro
G6. Let ně3 be an integer. Two regular n-gons A and B are given in the plane. Prove
that the verti
es of A that lie inside B or on its boundary are
onse
utive.
(That is, prove that there exists a line separating those verti
es of A that lie inside B or on
its boundary from the other verti
es of A.)
(Cze
h Republi
)
Solution 1. In both solutions, by a polygon we always mean its interior together with its
boundary.
We start with nding a regular n-gon C whi
h piq is ins
ribed into B (that is, all verti
es
of C lie on the perimeter of B); and piiq is either a translation of A, or a homotheti
image of A
with a positive fa
tor.
C C2
A22222
A
BR A
OA
OB BL
C C
A
A3333
A C3
B BB
Constru
tion of C Case 1: Translation
Let BT and BB be the top and the bottom verti
es of B (if several verti
es are extremal, we
take the rightmost of them). They split the perimeter of B into the right part BR and the left
part BL (the verti
es BT and BB are assumed to lie in both parts); ea
h part forms a
onne
ted
subset of the perimeter of B . So the verti
es of C are also split into two parts CL Ă BL and
CR Ă BR , ea
h of whi
h
onsists of
onse
utive verti
es.
Now, all the points in BR (and hen
e in CR ) move out from B under t, sin
e they are
the rightmost points of B on the
orresponding horizontal lines. It remains to prove that the
verti
es of CL whi
h stay in B under t are
onse
utive.
Denote by h the homothety mapping C to A. We need now to prove that the verti
es of C
whi
h stay in B after applying h are
onse
utive. If X P B , the
laim is easy. Indeed, if k ă 1,
then the verti
es of A lie on the segments of the form XC (C being a vertex of C ) whi
h lie
in B . If k ą 1, then the verti
es of A lie on the extensions of su
h segments XC beyond C ,
and almost all these extensions lie outside B . The ex
eptions may o
ur only in
ase when X
lies on the boundary of B , and they may
ause one or two verti
es of A stay on the boundary
of B . But even in this
ase those verti
es are still
onse
utive.
So, from now on we assume that X R B .
Now, there are two verti
es BT and B su
h that B is
ontained in the angle =BT XBB ;
BB of
if there are several options, say, for BT , then we
hoose the farthest one from X if k ą 1, and the
nearest one if k ă 1. For the visualization purposes, we refer the plane to Cartesian
oordinates
ÝÝÝÝÑ
so that the y -axis is
o-dire
tional with BB BT , and X lies to the left of the line BT BB . Again,
the perimeter of B is split by BT and BB into the right part BR and the left part BL , and the
set of verti
es of C is split into two subsets CR Ă BR and CL Ă BL .
BT
A1
C11111
C
B
C
A
A22222
C22222
C
X BR
A3 C
C33333
C
X BB A
X
Case 2, X inside B Sub
ase 2.1: ką1
Sub
ase 2.1: k ą 1.
In this sub
ase, all points from BR (and hen
e from CR ) move out from B under h, be
ause
they are the farthest points of B on the
orresponding rays emanated from X. It remains to
prove that the verti
es of CL whi
h stay in B under h are
onse
utive.
Again, let C1 , C2 , C3 be three verti
es in CL su
h that C2 is between C1 and C3 , and hpC1 q
and hpC3 q lie in B . Let Ai “ hpCi q. Then the ray XC2
rosses the segment C1 C3 beyond C2 ,
so this ray
rosses A1 A3 beyond A2 ; this implies that A2 lies in the triangle A1 C2 A3 , whi
h is
ontained in B .
BT C3333
C
A3
C22222
C
A
A22222
X
A BR
A
A11111 C
BB C11111
C
Sub
ase 2.2: kă1
Sub
ase 2.2: k ă 1.
This
ase is
ompletely similar to the previous one. All points from BL (and hen
e from CL
move out from B under h, be
ause they are the nearest points of B on the
orresponding
66 IMO 2017, Rio de Janeiro
rays emanated from X . Assume that C1 , C2 , and C3 are three verti
es in CR su
h that C2
lies between C1 and C3 , and hpC1 q and hpC3 q lie in B ; let Ai “ hpCi q. Then A2 lies on
the segment XC2 , and the segments XA2 and A1 A3
ross ea
h other. Thus A2 lies in the
triangle A1 C2 A3 , whi
h is
ontained in B .
Comment 2. After the polygon C has been found, the rest of the solution uses only the
onvexity of
the polygons, instead of regularity. Thus, it proves a more general statement:
Assume that A, B , and C are three
onvex polygons in the plane su
h that C is ins
ribed into B ,
and A
an be obtained from it via either translation or positive homothety. Then the verti
es of A that
lie inside B or on its boundary are
onse
utive.
Solution 2. Let OA and OB be the
enters of A and B, respe
tively. Denote rns “ t1, 2, . . . , nu.
We start with introdu
ing appropriate enumerations and notations. Enumerate the sidelines
of B
lo
kwise as ℓ1 , ℓ2 , . . . , ℓn . Denote by Hi the half-plane of ℓi that
ontains B (Hi is assumed
Ñ
Ý ÝÝÝÑ
to
ontain ℓi ); by Bi the midpoint of the side belonging to ℓi ; and nally denote bi “ Bi OB .
(As usual, the numbering is
y
li
modulo n, so ℓn`i “ ℓi et
.)
ÝÝÝÑ
Now,
hoose a vertex A1 of A su
h that the ve
tor OA A1 points mostly outside H1 ;
ÝÝÝÑ Ñ Ý
stri
tly speaking, this means that the s
alar produ
t xOA A1 , b1 y is minimal. Starting from A1 ,
enumerate the verti
es of A
lo
kwise as A1 , A2 , . . . , An ; by the rotational symmetry, the
hoi
e
ÝÝÝÑ
of A1 yields that the ve
tor OA Ai points mostly outside Hi , i.e.,
ÝÝÝÑ Ñ Ý ÝÝÝÑ Ñ Ý
xOA Ai , bi y “ minxOA Aj , bi y. (1)
jPrns
ℓ2
ℓ3
B1 B2
−
→
b1
ℓ1 A1 A2
−
→
b2 B3
Bn −
→ A3
H11111
H b3
−
→
An bn
A OB
B OA
Enumerations and notations
We intend to reformulate the problem in more
ombinatorial terms, for whi
h purpose we
introdu
e the following notion. Say that a subset I Ď rns is
onne
ted if the elements of this
set are
onse
utive in the
y
li
order (in other words, if we join ea
h i with i ` 1 mod n by an
edge, this subset is
onne
ted in the usual graph sense). Clearly, the union of two
onne
ted
subsets sharing at least one element is
onne
ted too. Next, for any half-plane H the indi
es
of verti
es of, say, A that lie in H form a
onne
ted set.
M2 = {1, 2, 3}
A
A22222
A11111
A A33333
A
ℓ2 M3 = {3, 4}
A
A44444
M1 = {n, 1, 2}
ℓℓ33333
AAnnnnn
ℓ1 B
B A
A
A55555
A
The sets Mi
The right-hand part of the last inequality does not depend on j. Therefore, if some j lies in Mi ,
then by (1) so does i. l
In view of Property 2, it is useful to dene the set
Ý
ÝÝ Ñ Ñ Ý ÝÝÝÝÑ Ñ Ý ÝÝÝÑ Ñ Ý ÝÝÝÑ Ñ Ý
i P M 1 ðñ Ai P Mi ðñ xBi Ai , bi y ă 0 ðñ xOB OA , bi y ă xOB Bi , bi y ` xAi OA , bi y.
The right-hand part of the obtained inequality does not depend on i, due to the rotational
1 ÝÝÝÝÑ Ñ Ý
symmetry; denote its
onstant value by µ. Thus, i P M if and only if xOB OA , bi y ă µ. This
ondition is in turn equivalent to the fa
t that Bi lies in a
ertain (open) half-plane whose
boundary line is orthogonal to OB OA ; thus, it denes a
onne
ted set. l
Now we
an nish the solution. Sin
e M1 Ď M, we have
ď ď
M“ Mi “ M 1 Y Mi ,
iPrns iPrns
so M
an be obtained from M 1 by adding all the sets Mi one by one. All these sets are
1
onne
ted, and ea
h nonempty Mi
ontains an element of M (namely, i). Thus their union is
also
onne
ted.
Comment 3. Here we present a way in whi
h one
an
ome up with a solution like the one above.
Assume, for sake of simpli
ity, that OA lies inside B . Let us rst put onto the plane a very small
regular n-gon A1
entered at OA and aligned with A; all its verti
es lie inside B . Now we start blowing
it up, looking at the order in whi
h the verti
es leave B . To go out of B , a vertex should
ross a
ertain
side of B (whi
h is hard to des
ribe), or, equivalently, to
ross at least one sideline of B and this
event is easier to des
ribe. Indeed, the rst vertex of A1 to
ross ℓi is the vertex A1i (
orresponding to Ai
ÝÝÝÑ ÝÑ
in A); more generally, the verti
es A1j
ross ℓi in su
h an order that the s
alar produ
t xOA Aj , bi y does
not in
rease. For dierent indi
es i, these orders are just
y
li
shifts of ea
h other; and this provides
some intuition for the notions and
laims from Solution 2.
68 IMO 2017, Rio de Janeiro
G7. A
onvex quadrilateral ABCD has an ins
ribed
ir
le with
enter I . Let Ia , Ib , Ic ,
and Id be the in
enters of the triangles DAB , ABC , BCD , and CDA, respe
tively. Suppose
that the
ommon external tangents of the
ir
les AIb Id and CIb Id meet at X , and the
ommon
0
external tangents of the
ir
les BIa Ic and DIa Ic meet at Y . Prove that =XIY “ 90 .
(Kazakhstan)
Solution. Denote by ωa , ωb , ωc and ωd the
ir
les AIb Id , BIa Ic , CIb Id , and DIa Ic , let their
enters be Oa , Ob , Oc and Od , and let their radii be ra , rb , rc and rd , respe
tively.
AC ` AB ´ BC AC ` AD ´ CD
AT “ “ “ AT 1 .
2 2
This shows T “ T 1. As an immediate
onsequen
e, Ib Id K AC .
The se
ond statement
an be shown analogously. l
D D
Id ωa Id
I
Oa
T′ T
A T C A C
Ib Ib
B B
Claim 2. The points Oa , Ob , Oc and Od lie on the lines AI , BI , CI and DI , respe tively.
Proof. By symmetry it su
es to prove the
laim for Oa . (See the gure to the right above.)
Noti
e rst that the in
ir
les of triangles ABC and ACD
an be obtained from the in
ir
le of
the quadrilateral ABCD with homothety
enters B and D , respe
tively, and homothety fa
tors
less than 1, therefore the points Ib and Id lie on the line segments BI and DI , respe
tively.
As is well-known, in every triangle the altitude and the diameter of the
ir
um
ir
le starting
from the same vertex are symmetri
about the angle bise
tor. By Claim 1, in triangle AId Ib ,
the segment AT is the altitude starting from A. Sin
e the foot T lies inside the segment
Ib Id , the
ir
um
enter Oa of triangle AId Ib lies in the angle domain Ib AId in su
h a way that
=Ib AT “ =Oa AId . The points Ib and Id are the in
enters of triangles ABC and ACD , so the
lines AIb and AId bise
t the angles =BAC and =CAD , respe
tively. Then
ωc
ωa
Id
I
Oa W U Oc
X
A C
Ib
Y
From the similarity of the
ir
les ωa and ωc , from Oa Ib “ Oa Id “ Oa A “ ra and O c Ib “
O c Id “ O c C “ rc , and from AC k Oa Oc we
an see that
Oa X Oa U ra O a Ib O a Id Oa A Oa I
“ “ “ “ “ “ .
Oc X Oc U rc O c Ib O c Id Oc C Oc I
So the points X, U, Ib , Id , I lie on the Apollonius
ir
le of the points Oa , Oc with ratio ra : rc . In
this Apollonius
ir
le XU is a diameter, and the lines IU and IX are respe
tively the internal
and external bise
tors of =Oa IOc “ =AIC , a
ording to the angle bise
tor theorem. Moreover,
in the Apollonius
ir
le the diameter UX is the perpendi
ular bise
tor of Ib Id , so the lines IX
and IU are the internal and external bise
tors of =Ib IId “ =BID , respe
tively.
Repeating the same argument for the points B, D instead of A, C , we get that the line IY is
the internal bise
tor of =AIC and the external bise
tor of =BID . Therefore, the lines IX and
IY respe
tively are the internal and external bise
tors of =BID , so they are perpendi
ular.
G8. There are 2017 mutually external
ir
les drawn on a bla
kboard, su
h that no two are
tangent and no three share a
ommon tangent. A tangent segment is a line segment that is
a
ommon tangent to two
ir
les, starting at one tangent point and ending at the other one.
Lu
iano is drawing tangent segments on the bla
kboard, one at a time, so that no tangent
segment interse
ts any other
ir
les or previously drawn tangent segments. Lu
iano keeps
drawing tangent segments until no more
an be drawn. Find all possible numbers of tangent
segments when he stops drawing.
(Australia)
Answer: If there were n
ir
les, there would always be exa
tly 3pn ´ 1q segments; so the only
possible answer is 3 ¨ 2017 ´ 3 “ 6048.
Solution 1. First,
onsider a parti
ular arrangement of
ir
les C1 , C2 , . . . , Cn where all the
enters are aligned and ea
h Ci is e
lipsed from the other
ir
les by its neighbors for example,
2
taking Ci with
enter pi , 0q and radius i{2 works. Then the only tangent segments that
an
be drawn are between adja
ent
ir
les Ci and Ci`1 , and exa
tly three segments
an be drawn
for ea
h pair. So Lu
iano will draw exa
tly 3pn ´ 1q segments in this
ase.
C3 C4 C5
C1 C2
For the general
ase, start from a nal
onguration (that is, an arrangement of
ir
les
and segments in whi
h no further segments
an be drawn). The idea of the solution is to
ontinuously resize and move the
ir
les around the plane, one by one (in parti
ular, making
sure we never have 4
ir
les with a
ommon tangent line), and show that the number of segments
drawn remains
onstant as the pi
ture
hanges. This way, we
an redu
e any
ir
le/segment
onguration to the parti
ular one mentioned above, and the nal number of segments must
remain at 3n ´ 3.
Some preliminary
onsiderations: look at all possible tangent segments joining any two
ir
les. A segment that is tangent to a
ir
le A
an do so in two possible orientations it
may
ome out of A in
lo
kwise or
ounter
lo
kwise orientation. Two segments tou
hing the
same
ir
le with the same orientation will never interse
t ea
h other. Ea
h pair pA, Bq of
ir
les
has 4
hoi
es of tangent segments, whi
h
an be identied by their orientations for example,
pA`, B´q would be the segment whi
h
omes out of A in
lo
kwise orientation and
omes out of
B in
ounter
lo
kwise orientation. In total, we have 2npn ´ 1q possible segments, disregarding
interse
tions.
Now we pi
k a
ir
le C and start to
ontinuously move and resize it, maintaining all existing
tangent segments a
ording to their identi
ations, in
luding those involving C. We
an keep
our
hoi
e of tangent segments until the
onguration rea
hes a transition. We lose nothing if
we assume that C ε units away from any other
ir
le, where ε is a positive, xed
is kept at least
onstant; therefore at a transition either: (1) a
urrently drawn tangent segment t suddenly
be
omes obstru
ted; or (2) a
urrently absent tangent segment t suddenly be
omes unobstru
ted
and available.
Claim. A transition
an only o
ur when three
ir
les C1 , C2 , C3 are tangent to a
ommon line ℓ
ontaining t, in a way su
h that the three tangent segments lying on ℓ (joining the three
ir
les
pairwise) are not obstru
ted by any other
ir
les or tangent segments (other than C1 , C2 , C3 ).
Proof. Sin
e (2) is ee
tively the reverse of (1), it su
es to prove the
laim for (1). Suppose t
has suddenly be
ome obstru
ted, and let us
onsider two
ases.
Shortlisted problems solutions 71
t
t
t
Ø Ø
Then the new
ir
le be
omes the third
ir
le tangent to ℓ, and no other
ir
les or tangent
segments are obstru
ting t.
Case 2: t be
omes obstru
ted by another tangent segment t1
When two segments t and t1 rst interse
t ea
h other, they must do so at a vertex of one of
1
them. But if a vertex of t rst
rossed an interior point of t, the
ir
le asso
iated to this vertex
was already blo
king t (absurd), or is about to (we already took
are of this in
ase 1). So we
1
only have to analyze the possibility of t and t suddenly having a
ommon vertex. However,
if that happens, this vertex must belong to a single
ir
le (remember we are keeping dierent
ir
les at least ε units apart from ea
h other throughout the moving/resizing pro
ess), and
therefore they must have dierent orientations with respe
t to that
ir
le.
t′
t′ t′
t
t
t
Ø Ø
Thus, at the transition moment, both t and t1 are tangent to the same
ir
le at a
ommon
point, that is, they must be on the same line ℓ and hen
e
we again have three
ir
les simultane-
1
ously tangent to ℓ. Also no other
ir
les or tangent segments are obstru
ting t or t (otherwise,
they would have disappeared before this transition). l
Next, we fo
us on the maximality of a
onguration immediately before and after a tran-
sition, where three
ir
les share a
ommon tangent line ℓ. Let the three
ir
les be C1 , C2 , C3 ,
ordered by their tangent points. The only possibly ae
ted segments are the ones lying on
ℓ, namely t12 , t23 and t13 . Sin
e C2 is in the middle, t12 and t23 must have dierent orienta-
tions with respe
t to C2 . For C1 , t12 and t13 must have the same orientation, while for C3 , t13
and t23 must have the same orientation. The gure below summarizes the situation, showing
alternative positions for C1 (namely, C1 and C11 ) and for C3 (C3 and C31 ).
C2
C3
C1
t12 t23
C1′
C3′
72 IMO 2017, Rio de Janeiro
Now perturb the diagram slightly so the three
ir
les no longer have a
ommon tangent,
while preserving the denition of t12 , t23 and t13 a
ording to their identi
ations. First note
that no other
ir
les or tangent segments
an obstru
t any of these segments. Also re
all that
tangent segments joining the same
ir
le at the same orientation will never obstru
t ea
h other.
The availability of the tangent segments
an now be
he
ked using simple diagrams.
C2
C3
C1
t13
t12 t23
C1′
C3′
In this ase, t13 is not available, but both t12 and t23 are.
C2
C3
C1
t12 t23
t13
C1′
C3′
Now t13 is available, but t12 and t23 obstru
t ea
h other, so only one
an be drawn.
In any
ase, exa
tly 2 out of these 3 segments
an be drawn. Thus the maximal number of
segments remains
onstant as we move or resize the
ir
les, and we are done.
Solution 2. First note that all tangent segments lying on the boundary of the
onvex hull of
the
ir
les are always drawn sin
e they do not interse
t anything else. Now in the nal pi
ture,
aside from the n
ir
les, the bla
kboard is divided into regions. We
an
onsider the pi
ture
as a plane (multi-)graph G in whi
h the
ir
les are the verti
es and the tangent segments are
the edges. The idea of this solution is to nd a relation between the number of edges and the
number of regions in G; then, on
e we prove that G is
onne
ted, we
an use Euler's formula
to nish the problem.
The boundary of ea
h region
onsists of 1 or more (for now) simple
losed
urves, ea
h
made of ar
s and tangent segments. The segment and the ar
might meet smoothly (as in Si ,
i “ 1, 2, . . . , 6 in the gure below) or not (as in P1 , P2 , P3 , P4 ;
all su
h points sharp
orners of
the boundary). In other words, if a person walks along the border, her dire
tion would suddenly
turn an angle of π at a sharp
orner.
Shortlisted problems solutions 73
S4 P2
P3
S5 S3
S2
S6
S1
P4
P1
Claim 1. The outer boundary B1 of any internal region has at least 3 sharp
orners.
Proof. Let a person walk one lap along B1 in the
ounter
lo
kwise orientation. As she does
so, she will turn
lo
kwise as she moves along the
ir
le ar
s, and not turn at all when moving
along the lines. On the other hand, her total rotation after one lap is 2π in the
ounter
lo
kwise
dire
tion! Where
ould she be turning
ounter
lo
kwise? She
an only do so at sharp
orners,
and, even then, she turns only an angle of π there. But two sharp
orners are not enough, sin
e
at least one ar
must be presentso she must have gone through at least 3 sharp
orners. l
Claim 2. Ea
h internal region is simply
onne
ted, that is, has only one boundary
urve.
Proof. Suppose, by
ontradi
tion, that some region has an outer boundary B1 and inner boun-
daries B2 , B3 , . . . , Bm (m ě 2). P1 be one of the sharp
orners of B1 .
Let
Now
onsider a
ar starting at P1 and traveling
ounter
lo
kwise along B1 . It starts in
reverse, i.e., it is initially fa
ing the
orner P1 . Due to the tangent
onditions, the
ar may travel
in a way so that its orientation only
hanges when it is moving along an ar
. In parti
ular, this
means the
ar will sometimes travel forward. For example, if the
ar approa
hes a sharp
orner
when driving in reverse, it would
ontinue travel forward after the
orner, instead of making an
immediate half-turn. This way, the orientation of the
ar only
hanges in a
lo
kwise dire
tion
sin
e the
ar always travels
lo
kwise around ea
h ar
.
Now imagine there is a laser pointer at the front of the
ar, pointing dire
tly ahead. Initially,
the laser endpoint hits P1 , but, as soon as the
ar hits an ar
, the endpoint moves
lo
kwise
around B1 . In fa
t, the laser endpoint must move
ontinuously along B1 ! Indeed, if the
endpoint ever jumped (within B1 , or from B1 to one of the inner boundaries), at the moment
of the jump the interrupted laser would be a drawable tangent segment that Lu
iano missed
(see gure below for an example).
P2
P3
Car
Laser
P1
74 IMO 2017, Rio de Janeiro
Now, let P2 and P3 be the next two sharp
orners the
ar goes through, after P1 (the
previous lemma assures their existen
e). At P2 the
ar starts moving forward, and at P3 it will
start to move in reverse again. So, atP3 , the laser endpoint is at P3 itself. So while the
ar
moved
ounter
lo
kwise between P1
P3 , the laser endpoint moved
lo
kwise between P1
and
and P3 . That means the laser beam itself s
anned the whole region within B1 , and it should
have
rossed some of the inner boundaries. l
Claim 3. Ea
h region has exa
tly 3 sharp
orners.
Proof. Consider again the
ar of the previous
laim, with its laser still rmly atta
hed to its
front, traveling the same way as before and going through the same
onse
utive sharp
orners
P1 , P2 and P3 . As we have seen, as the
ar goes
ounter
lo
kwise from P1 to P3 , the laser
endpoint goes
lo
kwise from P1 to P3 , so together they
over the whole boundary. If there
were a fourth sharp
orner P4 , at some moment the laser endpoint would pass through it. But,
sin
e P4 is a sharp
orner, this means the
ar must be on the extension of a tangent segment
going through P4 . Sin
e the
ar is not on that segment itself (the
ar never goes through P4 ),
we would have 3
ir
les with a
ommon tangent line, whi
h is not allowed.
P3
Laser Car
P4 P2
P1
l
r be the number of internal regions, and s be the
We are now ready to nish the solution. Let
number of tangent segments. Sin
e ea
h tangent segment
ontributes exa
tly 2 sharp
orners
to the diagram, and ea
h region has exa
tly 3 sharp
orners, we must have 2s “ 3r . Sin
e the
graph
orresponding to the diagram is
onne
ted, we
an use Euler's formula n ´ s ` r “ 1 and
nd s “ 3n ´ 3 and r “ 2n ´ 2.
Shortlisted problems solutions 75
Number Theory
N1. The sequen
e a0 , a1 , a2 , . . . of positive integers satises
#? ?
an , if an is an integer
an`1 “ for every n ě 0.
an ` 3, otherwise
Determine all values of a0 ą 1 for whi
h there is at least one number a su
h that an “ a for
innitely many values of n.
(South Afri
a)
Answer: All positive multiples of 3.
Solution. Sin
e the value of an`1 only depends on the value of an , if an “ am for two dierent
indi
es n and m, then the sequen
e is eventually periodi
. So we look for the values of a0 for
whi
h the sequen
e is eventually periodi
.
Claim 1. If an ” ´1 pmod 3q, then, for all m ą n, am is not a perfe
t square. It follows that
the sequen
e is eventually stri
tly in
reasing, so it is not eventually periodi
.
N2. Let p ě 2 be a prime number. Eduardo and Fernando play the following game making
moves alternately: in ea
h move, the
urrent player
hooses an index i in the set t0, 1, . . . , p´1u
that was not
hosen before by either of the two players and then
hooses an element ai of the
set t0, 1, 2, 3, 4, 5, 6, 7, 8, 9u. Eduardo has the rst move. The game ends after all the indi
es
i P t0, 1, . . . , p ´ 1u have been
hosen. Then the following number is
omputed:
p´1
ÿ
p´1
M “ a0 ` 10 ¨ a1 ` ¨ ¨ ¨ ` 10 ¨ ap´1 “ aj ¨ 10j .
j“0
The goal of Eduardo is to make the number M divisible by p, and the goal of Fernando is to
prevent this.
Prove that Eduardo has a winning strategy.
(Moro
o)
Solution. We say that a player makes the move pi, ai q if he
hooses the index i and then the
element ai of the set t0, 1, 2, 3, 4, 5, 6, 7, 8, 9u in this move.
If p “ 2 or p “ 5 then Eduardo
hooses i “ 0 and a0 “ 0 in the rst move, and wins, sin
e,
independently of the next moves, M will be a multiple of 10.
Now assume that the prime number p does not belong to t2, 5u. Eduardo
hooses i “ p ´ 1
pp´1q{2 2
and ap´1 “ 0 in the rst move. By Fermat's Little Theorem, p10 q “ 10p´1 ” 1 pmod pq,
pp´1q{2 2 pp´1q{2 pp´1q{2
so p | p10 q ´ 1 “ p10 ` 1qp10 ´ 1q. Sin
e p is prime, either p | 10pp´1q{2 ` 1 or
p | 10pp´1q{2 ´ 1. Thus we have two
ases:
Case a: 10pp´1q{2 ” ´1 pmod pq
In this
ase, for ea
h move pi, ai q of Fernando, Eduardo immediately makes the move pj, aj q “
pi ` p´1
2
, ai q, if 0 ď i ď p´3
2
, or pj, aj q “ pi ´
p´1
2
, ai q, if p´1
2
ď i ď p ´ 2. We will have 10j ” ´10i
pmod pq, and so aj ¨ 10j “ ai ¨ 10j ” ´ai ¨ 10i pmod pq. Noti
e that this move by Eduardo
is always possible. Indeed, immediately before a move by Fernando, for any set of the type
tr, r ` pp ´ 1q{2u with 0 ď r ď pp ´ 3q{2, either no element of this set was
hosen as an index
by the players in the previous moves or else both elements of this set were
hosen as indi
es by
the players in the previous moves. Therefore, after ea
h of his moves, Eduardo always makes
k
the sum of the numbers ak ¨ 10
orresponding to the already
hosen pairs pk, ak q divisible by
p, and thus wins the game.
p´3
ÿ
2
N3. Determine all integers n ě 2 with the following property: for any integers a1 , a2 , . . . , an
whose sum is not divisible by n, there exists an index 1 ď i ď n su
h that none of the numbers
Solution. Let us rst show that, if n “ ab, with a, b ě 2 integers, then the property in the
statement of the problem does not hold. Indeed, in this
ase, let ak “ a for 1 ď k ď n ´ 1 and
an “ 0. The sum a1 ` a2 ` ¨ ¨ ¨ ` an “ a ¨ pn ´ 1q is not divisible by n. Let i with 1 ď i ď n be
an arbitrary index. Taking j “ b if 1 ď i ď n ´ b, and j “ b ` 1 if n ´ b ă i ď n, we have
It follows that the given example is indeed a ounterexample to the property of the statement.
Now let n be a prime number. Suppose by
ontradi
tion that the property in the statement
of the problem does not hold. Then there are integers a1 , a2 , . . . , an whose sum is not divisible
by n su
h that for ea
h i, 1 ď i ď n, there is j , 1 ď j ď n, for whi
h the number ai ` ai`1 `
¨ ¨ ¨ ` ai`j´1 is divisible by n. Noti
e that, in any su
h
ase, we should have 1 ď j ď n ´ 1,
sin
e a1 ` a2 ` ¨ ¨ ¨ ` an is not divisible by n. So we may
onstru
t re
ursively a nite sequen
e
of integers 0 “ i0 ă i1 ă i2 ă ¨ ¨ ¨ ă in with is`1 ´ is ď n ´ 1 for 0 ď s ď n ´ 1 su
h that, for
0 ď s ď n ´ 1,
ais `1 ` ais `2 ` ¨ ¨ ¨ ` ais`1 ” 0 pmod nq
(where we take indi
es modulo n). Indeed, for 0 ď s ă n, we apply the previous observation
to i “ is ` 1 is`1 “ is ` j .
in order to dene
In the sequen
e of n ` 1 indi
es i0 , i1 , i2 , . . . , in , by the pigeonhole prin
iple, we have two
distin
t elements whi
h are
ongruent modulo n. So there are indi
es r, s with 0 ď r ă s ď n
su
h that is ” ir pmod nq and
s´1
ÿ
air `1 ` air `2 ` ¨ ¨ ¨ ` ais “ paij `1 ` aij `2 ` ¨ ¨ ¨ ` aij`1 q ” 0 pmod nq.
j“r
Sin
e is ” ir pmod nq, we have is ´ ir “ k ¨ n for some positive integer k , and, sin
e ij`1 ´ ij ď
n´1 for0 ď j ď n ´ 1, we have is ´ ir ď pn ´ 1q ¨ n, so k ď n ´ 1. But in this
ase
N4. Call a rational number short if it has nitely many digits in its de
imal expansion.
For a positive integer m, m-tasti
if there exists a number
we say that a positive integer t is
t
10 ´ 1 10k ´ 1
c P t1, 2, 3, . . . , 2017u su
h that is short, and su
h that is not short for any
c¨m c¨m
1 ď k ă t. Let Spmq be the set of m-tasti
numbers. Consider Spmq for m “ 1, 2, . . .. What is
the maximum number of elements in Spmq?
(Turkey)
Answer: 807.
Solution. First noti
e that x P Q is short if and only if there are exponents a, b ě 0 su
h that
2a ¨ 5b ¨ x P Z. x is short, then x “ 10nk for some k and we
an take a “ b “ k ; on the
In fa
t, if
a b 2b ¨5a q
other hand, if 2 ¨ 5 ¨ x “ q P Z then x “ , so x is short.
10a`b
a b 10t ´1 t
If m “ 2 ¨ 5 ¨ s, with gcdps, 10q “ 1, then is short if and only if s divides 10 ´ 1. So
m
we may (and will) suppose without loss of generality that gcdpm, 10q “ 1. Dene
The m-tasti
numbers are then pre
isely the smallest exponents t ą 0 su
h that 10t ” 1
pmod cmq for some integer c P C, that is, the set of orders of 10 modulo cm. In other words,
Sin
e there are 4 ¨ 201 ` 3 “ 807 numbers c with 1 ď c ď 2017 and gcdpc, 10q “ 1, namely
those su
h that c ” 1, 3, 7, 9 pmod 10q,
P “ t1 ă p ď 2017 : p is prime, p ‰ 2, 5u
and
hoose a positive integer α su
h that every p P P divides 10α ´ 1 (e.g. α “ ϕpT q, T being
the produ
t of all primes in P ), and let m “ 10α ´ 1.
Claim. For every c P C , we have
ordcm p10q “ cα.
As an immediate
onsequen
e, this implies |Spmq| “ |C| “ 807, nishing the problem.
cm 10t ´ 1 ùñ m 10t ´ 1 ùñ α t.
so
Comment. The Lifting the Exponent Lemma states that, for any odd prime p, any integers a, b
oprime with p su
h that p a ´ b, and any positive integer exponent n,
νp pan ´ bn q “ νp pa ´ bq ` νp pnq,
and, for p “ 2,
ν2 pan ´ bn q “ ν2 pa2 ´ b2 q ` νp pnq ´ 1.
Both
laims
an be proved by indu
tion on n.
80 IMO 2017, Rio de Janeiro
N5. Find all pairs pp, qq of prime numbers with pąq for whi h the number
pp ` qqp`q pp ´ qqp´q ´ 1
pp ` qqp´q pp ´ qqp`q ´ 1
is an integer.
(Japan)
Answer: The only su
h pair is p3, 2q.
Solution. Let M “ pp ` qqp´q pp ´ qqp`q ´ 1, whi
h is relatively prime with both p`q and
p ´ q . Denote by pp ´ qq´1 the multipli
ative inverse of pp ´ qq modulo M .
By eliminating the term ´1 in the numerator,
Case 1: q ě 5.
Consider an arbitrary prime divisor
´ r ¯of M . Noti
e that M is odd, so r ě 3. By p2q, the
´1
multipli
ative order of pp ` qq ¨ pp ´ qq modulo r is a divisor of the exponent 2q in (2), so
it
an be 1, 2, q or 2q .
By Fermat's theorem, the order divides r ´ 1. So, if the order is q or 2q then r ” 1 pmod qq.
2 2
If the order is 1 or 2 then r | pp ` qq ´ pp ´ qq “ 4pq , so r “ p or r “ q . The
ase r “ p is not
possible, be
ause, by applying Fermat's theorem,
` ˘p
M “ pp ` qqp´q pp ´ qqp`q ´ 1 ” q p´q p´qqp`q ´ 1 “ q 2 ´ 1 ” q 2 ´ 1 “ pq ` 1qpq ´ 1q pmod pq
and the last fa
tors q´1 and q ` 1 are less than p and thus p ∤ M . Hen
e, all prime divisors
of M are either q or of the form kq ` 1; it follows that all positive divisors of M are
ongruent
to 0 or 1 modulo q .
Now noti
e that
´ p´q p`q
¯´ p´q p`q
¯
M “ pp ` qq 2 pp ´ qq 2 ´ 1 pp ` qq 2 pp ´ qq 2 ` 1
is the produ
t of two
onse
utive positive odd numbers; both should be
ongruent to 0 or 1
modulo q. But this is impossible by the assumption q ě 5. So, there is no solution in Case 1.
Case 2: q “ 2.
By p1q, we have M | pp ` qq2q ´ pp ´ qq2q “ pp ` 2q4 ´ pp ´ 2q4 , so
Case 3: q “ 3.
Similarly to Case 2, we have
˜ˆ ˙6 ˆ ˙6 ¸
p`3 p´3
M | pp ` qq2q ´ pp ´ qq2q “ 64 ¨ ´ .
2 2
ˆ ˙6 ˆ ˙6
p`3 p´3
M| ´
2 2
and
ˆ ˙6 ˆ ˙6 ˆ ˙6
p´3 p`3 p`3 p´3 p`3
pp ` 3q pp ´ 3q ´1“M ď ´ ď ´ 1,
2 2 2
64pp ` 3qp´9 pp ´ 3qp`3 ď 1.
Ifp ě 11 then the left-hand side is obviously greater than 1. If p “ 7 then the left-hand side is
64 ¨ 10´2 ¨ 410 ą 1. If p “ 5 then the left-hand side is 64 ¨ 8´4 ¨ 28 “ 22 ą 1. Therefore, there is
no solution in Case 3.
82 IMO 2017, Rio de Janeiro
N6. Find the smallest positive integer n, or show that no su
h n exists, with the following
property: there are innitely many distin
t n-tuples of positive rational numbers pa1 , a2 , . . . , an q
su
h that both
1 1 1
a1 ` a2 ` ¨ ¨ ¨ ` an and ` ` ¨¨¨`
a1 a2 an
are integers.
(Singapore)
Answer: n “ 3.
1
Solution 1. For n “ 1, a1 P Zą0 and
a1
P Zą0 if and only if a1 “ 1. Next we show that
(ii) There are innitely many triples px, y, zq P Q2ą0 su
h that x ` y ` z P Z and x1 ` y1 ` z1 P Z.
We will look for triples su
h that x ` y ` z “ 1, so we may write them in the form
ˆ ˙
a b c
px, y, zq “ , , with a, b, c P Zą0
a`b`c a`b`c a`b`c
We want these to satisfy
1 1 c b
` ` ` “ 3 ðñ b2 ` c2 ´ 3bc ` b ` c “ 0 p˚q
b c b c
To show that equation p˚q has innitely many solutions, we use Vieta jumping (also known
as root ipping ): starting with b “ 2, c “ 3, the following algorithm generates innitely
many solutions. Let c ě b, and view p˚q as a quadrati
equation in b for c xed:
p2, 3q, p3, 6q, p6, 14q, p14, 35q, p35, 90q, p90, 234q, p234, 611q, p611, 1598q, p1598, 4182q, . . .
Shortlisted problems solutions 83
Comment. Although not needed for solving this problem, we may also expli
itly solve the re
ursion
given by the Vieta jumping. Dene the sequen
e pxn q as follows:
is an integer pretty. Then good n-tuples are pretty, and if pb1 , . . . , bn q is pretty then
ˆ ˙
b1 b2 bn
, ,...,
b1 ` b2 ` ¨ ¨ ¨ ` bn b1 ` b2 ` ¨ ¨ ¨ ` bn b1 ` b2 ` ¨ ¨ ¨ ` bn
is good sin
e the sum of its
omponents is 1, and the sum of the re
ipro
als of its
omponents
equals f pb1 , . . . , bn q. We de
lare pretty n-tuples proportional to ea
h other equivalent sin
e they
are pre
isely those whi
h give rise to the same good n-tuple. Clearly, ea
h su
h equivalen
e
lass
ontains exa
tly one n-tuple of positive integers having no
ommon prime divisors. Call su
h
n-tuple a primitive pretty tuple. Our task is to nd innitely many primitive pretty n-tuples.
2
For n “ 1, there is
learly a single primitive 1-tuple. For n “ 2, we have f pa, bq “ pa`bq
ab
,
whi
h
an be integral (for
oprime a, b P Zą0 ) only if a “ b “ 1 (see for instan
e (i) in the rst
solution).
Now we
onstru
t innitely many primitive pretty triples for n “ 3. Fix b, c, k P Zą0 ; we
will try to nd su
ient
onditions for the existen
e of an a P Qą0 su
h that f pa, b, cq “ k .
Write σ “ b ` c, τ “ bc. From f pa, b, cq “ k , we have that a should satisfy the quadrati
equation
a2 ¨ σ ` a ¨ pσ 2 ´ pk ´ 1qτ q ` στ “ 0 (1)
ppk ` 1qτ ´ σ 2 q2 ´ M 2 “ 2k ¨ 2τ 2
pk ` 1qτ ´ σ 2 “ τ 2 ` k, M “ τ 2 ´ k.
σ2
then k“ τ ´1
`τ will be integral, and we nd rational solutions to (1), namely
σ b`c τ2 ´ τ bc ¨ pbc ´ 1q
a“ “ or a“ “
τ ´1 bc ´ 1 σ b`c
84 IMO 2017, Rio de Janeiro
We
an now nd innitely many pairs pb, cq satisfying (2) by Vieta jumping. For example,
if we impose
pb ` cq2 “ 5 ¨ pbc ´ 1q
then all pairs pb, cq “ pvi , vi`1 q satisfy the above
ondition, where
For pb, cq “ pvi , vi`1 q, one of the solutions to (1) will be a “ pb ` cq{pbc ´ 1q “ 5{pb ` cq “
5{pvi ` vi`1 q. Then the pretty triple pa, b, cq will be equivalent to the integral pretty triple
After possibly dividing by 5, we obtain innitely many primitive pretty triples, as required.
Comment. There are many other innite series of pb, cq “ pvi , vi`1 q with bc ´ 1 | pb ` cq2 . Some of
them are:
(the last two are in fa
t one sequen
e prolonged in two possible dire
tions).
Shortlisted problems solutions 85
N7. Say that an ordered pair of integers is an irredu
ible latti
e point if x and y
px, yq
are relatively prime. S of irredu
ible latti
e points, show that there is a
For any nite set
homogenous polynomial in two variables, f px, yq, with integer
oe
ients, of degree at least 1,
su
h that f px, yq “ 1 for ea
h px, yq in the set S .
Note: A homogenous polynomial of degree n is any nonzero polynomial of the form
Then ℓi pxj , yj q “ 0 if and only if j “ i, be
ause there is only one latti
e point on ea
h line
through the origin. Thus, gi pxj , yj q “ 0 for all j ‰ i. Dene ai “ gi pxi , yi q, and note that
ai ‰ 0.
Note that gi px, yq is a degree n´1 polynomial with the following two properties:
1. gi pxj , yj q “ 0 if j ‰ i.
2. gi pxi , yi q “ ai .
For any N ě n ´ 1, there also exists a polynomial of degree N with the same two proper-
ties. Spe
i
ally, let Ii px, yq be a degree 1 homogenous polynomial su
h that Ii pxi , yi q “ 1,
whi
h exists sin
e pxi , yi q is irredu
ible. Then Ii px, yqN ´pn´1q gi px, yq satises both of the above
properties and has degree N .
We may now redu
e the problem to the following
laim:
Claim: For ea
h positive integer a, there is a homogenous polynomial fa px, yq, with integer
oe
ients, of degree at least 1, su
h that fa px, yq ” 1 pmod aq for all relatively prime px, yq.
To see that this
laim solves the problem, take a
to be the least
ommon multiple of the
k
numbers ai (1 ď i ď n). Take fa given by the
laim,
hoose some power fa px, yq that has
degree at least n ´ 1, and subtra
t appropriate multiples of the gi
onstru
ted above to obtain
the desired polynomial.
We prove the
laim by fa
toring a. First, if a is a power of a prime pa “ pk q, then we may
hoose either:
Solution 2. As in the previous solution, label the irredu
ible latti
e points px1 , y1q, . . . , pxn , yn q
and assume without loss of generality that no two of the points are
ollinear with the origin.
We indu
t on n to
onstru
t a homogenous polynomial f px, yq su
h that f pxi , yi q “ 1 for all
1 ď i ď n.
If n “ 1: Sin
e x1 and y1 are relatively prime, there exist some integers c, d su
h that
cx1 ` dy1 “ 1. Then f px, yq “ cx ` dy is suitable.
If n ě 2: By the indu
tion hypothesis we already have a homogeneous polynomial gpx, yq
with gpx1 , y1 q “ . . . “ gpxn´1 , yn´1 q “ 1. Let j “ deg g ,
n´1
ź
gn px, yq “ pyk x ´ xk yq,
k“1
n´1
ź
p | an “ gn pxn , yn q “ pyk xn ´ xk yn q,
k“1
ykd ¨ gpxn , yn q “ gpyk xn , yk yn q ” gpxk yn , yk yn q “ ynd ¨ gpxk , yk q “ ynd pmod pq. p1.2q
If p ∤ xk xn , then take the pp ´ 1qst power of p1.1q; otherwise take the pp ´ 1qst power of p1.2q;
by Fermat's theorem, in both
ases we get
α´1 pp´1q
gpxn , yn qp ” 1 pmod pα q,
whi
h implies that the exponent K “ n ¨ ϕpan q, whi
h is a multiple of all pα´1 pp ´ 1q, is a
suitable
hoi
e. (The fa
tor n is added only so that Kěn and so L ą 0.)
Shortlisted problems solutions 87
Comment. It is possible to show that there is no
onstant C for whi
h, given any two irredu
ible
latti
e points, there is some homogenous polynomial f of degree at most C with integer
oe
ients
that takes the value 1 on the two points. Indeed, if one of the points is p1, 0q and the other is pa, bq,
the polynomial f px, yq “ a0 xn ` a1 xn´1 y ` ¨ ¨ ¨ ` an y n should satisfy a0 “ 1, and so an ” 1 pmod bq.
If a “ 3 and b “ 2k with k ě 3, then n ě 2k´2 . If we
hoose 2k´2 ą C , this gives a
ontradi
tion.
88 IMO 2017, Rio de Janeiro
N8. Let p be an odd prime number and Zą0 be the set of positive integers. Suppose that
a fun
tion f : Zą0 ˆ Zą0 Ñ t0, 1u satises the following properties:
• f p1, 1q “ 0;
• f pa, bq ` f pb, aq “ 1 for any pair of relatively prime positive integers pa, bq not both equal
to 1;
• f pa ` b, bq “ f pa, bq for any pair of relatively prime positive integers pa, bq.
Prove that
p´1
ÿ a
f pn2 , pq ě 2p ´ 2.
n“1
(Italy)
Solution 1. Denote by Athe set of all pairs of
oprime positive integers. Noti
e that for
2
every pa, bq P A there exists a pair pu, vq P Z with ua ` vb “ 1. Moreover, if pu0 , v0 q is one
su
h pair, then all su
h pairs are of the form pu, vq “ pu0 ` kb, v0 ´ kaq, where k P Z. So there
exists a unique su
h pair pu, vq with ´b{2 ă u ď b{2; we denote this pair by pu, vq “ gpa, bq.
Lemma. Let pa, bq P A and pu, vq “ gpa, bq. Then f pa, bq “ 1 ðñ u ą 0.
Proof. We indu
t on a ` b. The base
ase is a ` b “ 2. In this
ase, we have that a “ b “ 1,
gpa, bq “ gp1, 1q “ p0, 1q and f p1, 1q “ 0, so the
laim holds.
Assume now that a ` b ą 2, and so a ‰ b, sin
e a and b are
oprime. Two
ases are possible.
Case 1: a ą b.
gpa ´ b, bq “ pu, v ` uq, sin
e upa ´ bq ` pv ` uqb “ 1 and u P p´b{2, b{2s.
Noti
e that Thus
f pa, bq “ 1 ðñ f pa ´ b, bq “ 1 ðñ u ą 0 by the indu
tion hypothesis.
Case 2: a ă b. (Then,
learly, b ě 2.)
Now we estimate v. Sin
e vb “ 1 ´ ua, we have
ab ab 1`a 1 a 1 a a
1` ą vb ě 1 ´ , so ě ` ąvě ´ ą´ .
2 2 2 b 2 b 2 2
Thus 1 ` a ą 2v ą ´a, so a ě 2v ą ´a, hen
e a{2 ě v ą ´a{2, and thus gpb, aq “ pv, uq.
Observe that f pa, bq “ 1 ðñ f pb, aq “ 0 ðñ f pb ´ a, aq “ 0. We know from Case 1
that gpb ´ a, aq “ pv, u ` vq. We have f pb ´ a, aq “ 0 ðñ v ď 0 by the indu
tive hypothesis.
Then, sin
e b ą a ě 1 and ua ` vb “ 1, we have v ď 0 ðñ u ą 0, and we are done. l
The Lemma proves that, for all pa, bq P A, f pa, bq “ 1 if and only if the inverse of a
modulo b, taken in t1, 2, . . . , b ´ 1u, is at most b{2. Then, for any odd prime p and integer
n su
h that n ı 0 pmod pq, f pn2 , pq “ 1 i the inverse of n2 mod p is less than p{2. Sin
e
tn2 mod p : 1 ď n ď p ´ 1u “ tn´2 mod p : 1 ď n ď p ´ 1u, in
luding multipli
ities (two for
ea
h quadrati
residue in ea
h set), we
on
lude that the desired sum is twi
e the number of
quadrati
residues that are less than p{2, i.e.,
p´1 ˇ" *ˇ
ÿ ˇ p´1 p ˇˇ
f pn , pq “ 2 ˇˇ k : 1 ď k ď
2
and
2
k mod p ă . (1)
n“1
2 2 ˇ
a a
Sin
e the number of perfe
t squares in the interval r1, p{2q is t p{2u ą p{2 ´ 1, we
on
lude that
p´1
ÿ ˆc ˙
2 p a
f pn , pq ą 2 ´1 “ 2p ´ 2.
n“1
2
Shortlisted problems solutions 89
Solution 2. We provide a dierent proof for the Lemma. For this purpose, we use
ontinued
fra
tions to nd gpa, bq “ pu, vq expli
itly.
The fun
tion f is
ompletely determined on A by the following
Claim. Represent a{b as a
ontinued fra
tion; that is, let a0 be an integer and a1 , . . . , ak be
positive integers su
h that ak ě 2 and
a 1
“ a0 ` “ ra0 ; a1 , a2 , . . . , ak s.
b 1
a1 `
1
a2 `
1
¨¨¨`
ak
Then f pa, bq “ 0 ðñ k is even.
Proof. We indu
t on b. If b “ 1, then a{b “ ras and k “ 0. Then, for a ě 1, an easy indu
tion
shows that f pa, 1q “ f p1, 1q “ 0.
Now
onsider the
ase b ą 1. Perform the Eu
lidean division a “ qb ` r , with 0 ď r ă b.
We have r ‰ 0 be
ause gcdpa, bq “ 1. Hen
e
a b
f pa, bq “ f pr, bq “ 1 ´ f pb, rq, “ rq; a1 , . . . , ak s, and “ ra1 ; a2 , . . . , ak s.
b r
Then the number of terms in the
ontinued fra
tion representations of a{b and b{r dier by
one. Sin
e r ă b, the indu
tive hypothesis yields
f pb, rq “ 0 ðñ k ´ 1 is even,
and thus
f pa, bq “ 0 ðñ f pb, rq “ 1 ðñ k ´ 1 is odd ðñ k is even. l
Now we use the following well-known properties of
ontinued fra
tions to prove the Lemma:
Let pi and qi be
oprime positive integers with ra0 ; a1 , a2 , . . . , ai s “ pi {qi , with the notation
borrowed from the Claim. In parti
ular, a{b “ ra0 ; a1 , a2 , . . . , ak s “ pk {qk . Assume that k ą 0
and dene q´1 “ 0 if ne
essary. Then
b
b “ qk “ ak qk´1 ` qk´2 ě ak qk´1 ě 2qk´1 ùñ qk´1 ď ,
2
with stri
t inequality for k ą 1, and
Now we nish the proof of the Lemma. It is immediate for k “ 0. If k “ 1, then p´1qk´1 “ 1,
so
´b{2 ă 0 ď p´1qk´1qk´1 ď b{2.
If k ą 1, we have qk´1 ă b{2, so
Thus, for any k ą 0, we nd that gpa, bq “ pp´1qk´1 qk´1 , p´1qk pk´1q, and so
Comment 2. The
ase p ” 1 pmod 4q is, in fa
t, easier than the original problem. We have, in
general, for 1 ď a ď p ´ 1,
f pa, pq “ 1 ´ f pp, aq “ 1 ´ f pp ´ a, aq “ f pa, p ´ aq “ f pa ` pp ´ aq, p ´ aq “ f pp, p ´ aq “ 1 ´ f pp ´ a, pq.
If p ” 1 pmod 4q, then a is a quadrati
residue modulo p if and only if p ´ a is a quadrati
residue
modulo p. Therefore, denoting by rk (with 1 ď rk ď p ´ 1) the remainder of the division of k 2 by p,
we get
p´1
ÿ p´1
ÿ p´1
2 1 ÿ p´1
f pn , pq “ f prn , pq “ pf prn , pq ` f pp ´ rn , pqq “ .
n“1 n“1
2 n“1
2
ř
Comment 3. The estimate for the sum pn“1 f pn2 , pq
an be improved by rening the nal argument
in Solution 1. In fa
t, one
an prove that
p´1
ÿ p´1
f pn2 , pq ě .
n“1
16
By
ounting the number of perfe
t squares in the intervals rkp, pk ` 1{2qpq, we nd that
p´1 p´1
˜[dˆ ˙ _ Y ¸
ÿ ÿ 1 a ]
f pn2 , pq “ k` p ´ kp . (2)
n“1 k“0
2
Ea
h summand of (2) is non-negative. We now estimate the number of positive summands. Suppose
that a summand is zero, i.e., [ dˆ ˙ _ Y
1 a ]
k` p “ kp “: q.
2
Then both of the numbers kp and kp ` p{2 lie within the interval rq 2 , pq ` 1q2 q. Hen
e
p
ă pq ` 1q2 ´ q 2 ,
2
whi
h implies
p´1
qě .
4
?
Sin
e q ď kp, if the k th summand of (2) is zero, then
q2 pp ´ 1q2 p´2 p´1
kě ě ą ùñ k ě .
p 16p 16 16
So at least the rst r p´1 p´1
16 s summands (from k “ 0 to k “ r 16 s ´ 1) are positive, and the result
follows.
Comment 4. The bound
an be further improved by using dierent methods. In fa
t, we prove that
p´1
ÿ p´3
f pn2 , pq ě .
n“1
4
To that end, we use the Legendre symbol
$
ˆ ˙ ’ &0 if p a
a
“ 1 if a is a nonzero quadrati
residue mod p
p ’
%
´1 otherwise.
We start with the following Claim, whi
h tells us that there are not too many
onse
utive quadrati
residues or
onse
utive quadrati
non-residues.
Shortlisted problems solutions 91
řp´1 ` n ˘` n`1 ˘
Claim. n“1 p p “ ´1.
` n ˘` n`1 ˘ ` npn`1q ˘
Proof. We have p p “ p . For 1 ď n ď p ´ 1, we get that npn ` 1q ” n2 p1 ` n´1 q pmod pq,
` npn`1q ˘ ` 1`n´1 ˘
hen
e p “ p . Sin
e t1 ` n´1 mod p : 1 ď n ď p ´ 1u “ t0, 2, 3, . . . , p ´ 1 mod pu, we nd
ÿˆ
p´1
n
˙ˆ
n`1
˙ ÿˆ
p´1
1 ` n´1
˙ ÿˆ
p´1
n
˙
“ “ ´ 1 “ ´1,
n“1
p p n“1
p n“1
p
řp ` ˘
be
ause n
n“1 p “ 0. l
Observe that (1) be
omes
p´1
ÿ " ˆ ˙ *
p´1 r
2
f pn , pq “ 2 |S| , S “ r: 1 ď r ď and “1 .
n“1
2 p
We
onne
t S with the sum from the
laim by pairing quadrati
residues and quadrati
non-residues.
To that end, dene
" ˆ ˙ *
1 p´1 r
S “ r: 1 ď r ď and “ ´1
2 p
" ˆ ˙ *
p`1 r
T “ r: ď r ď p ´ 1 and “1
2 p
" ˆ ˙ *
1 p`1 r
T “ r: ď r ď p ´ 1 and “ ´1
2 p
Sin
e there are exa
tly pp ´ 1q{2 nonzero quadrati
residues modulo p, |S| ` |T | “ pp ´ 1q{2. Also
we obviously have |T | ` |T 1 | “ pp ´ 1q{2. Then |S| “` |T˘`1 |. ˘
For the sake of brevity, dene t “ |S| “ |T 1 |. If np n`1 p “ ´1, then exa
tly of one the numbers
`n˘ ` n`1 ˘
p and p is equal to 1, so
ˇ" ˆ ˙ˆ ˙ *ˇ
ˇ p ´ 3 n n ` 1 ˇ
ˇ n: 1 ď n ď
ˇ and “ ´1 ˇˇ ď |S| ` |S ´ 1| “ 2t.
2 p p
` ˘` ˘ ` ˘ ` ˘
On the other hand, if np n`1
p “ ´1, then exa
tly one of np and n`1 p is equal to ´1, and
ˇ" ˆ ˙ˆ ˙ *ˇ
ˇ p ` 1 n n ` 1 ˇ
ˇ n:
ˇ ď n ď p ´ 2 and “ ´1 ˇˇ ď |T 1 | ` |T 1 ´ 1| “ 2t.
2 p p
` pp´1q{2 ˘` pp`1q{2 ˘
Thus, taking into a
ount that the middle term p p may happen to be ´1,
ˇ" ˆ ˙ˆ ˙ *ˇ
ˇ n`1 ˇ
ˇ n : 1 ď n ď p ´ 2 and n “ ´1 ˇˇ ď 4t ` 1.
ˇ p p
and so
ÿˆ
p´1
n
˙ˆ
n`1
˙
´1 “ ě p ´ 4t ´ 3 ´ p4t ` 1q “ p ´ 8t ´ 4,
n“1
p p
whi
h implies 8t ě p ´ 3, and thus
p´1
ÿ p´3
f pn2 , pq “ 2t ě .
n“1
4
92 IMO 2017, Rio de Janeiro
The
ase p ” 1 pmod 4q was already mentioned, and it is the equality
ase. If p ” 3 pmod 4q,
then, by a theorem of Diri
hlet, we have
ˇ" ˆ ˙ *ˇ
ˇ ˇ
ˇ r : 1 ď r ď p ´ 1 and r “ 1 ˇ ą p ´ 1 ,
ˇ 2 p ˇ 4
Contributing Countries
The Organising Committee and the Problem Sele
tion Committee of IMO 2018 thank the
following 49
ountries for
ontributing 168 problem proposals:
Problems
Algebra
A1. Let Qą0 denote the set of all positive rational numbers. Determine all fun
tions
f : Qą0 Ñ Qą0 satisfying ` ˘
f x2 f pyq2 “ f pxq2 f pyq
for all x, y P Qą0 .
(Switzerland)
A2. Find all positive integers n ě 3 for whi
h there exist real numbers a1 , a2 , . . . , an ,
an`1 “ a1 , an`2 “ a2 su
h that
ai ai`1 ` 1 “ ai`2
for all i “ 1, 2, . . . , n.
(Slovakia)
A3. Given any set S of positive integers, show that at least one of the following two
assertions holds:
ř ř
(1) There exist distin
t nite subsets F and G of S su
h that xPF 1{x “ xPG 1{x ;
ř
(2) There exists a positive rational number r ă 1 su
h that xPF 1{x ‰ r for all nite subsets
F of S .
(Luxembourg)
A4. Let a0 , a1 , a2 , . . . be a sequen
e of real numbers su
h that a0 “ 0, a1 “ 1, and for
every n ě 2 there exists 1 ď k ď n satisfying
an´1 ` ¨ ¨ ¨ ` an´k
an “ .
k
Find the maximal possible value of a2018 ´ a2017 .
(Belgium)
A5. Determine all fun
tions f : p0, 8q Ñ R satisfying
ˆ ˙ ´y¯
1
x` f pyq “ f pxyq ` f
x x
for all x, y ą 0.
(South Korea)
A6. Let m, n ě 2 be integers. Let f px1 , . . . , xn q be a polynomial with real
oe
ients
su
h that
Yx ` . . . ` x ] (
1 n
f px1 , . . . , xn q “ for every x1 , . . . , xn P 0, 1, . . . , m ´ 1 .
m
Prove that the total degree of f is at least n.
(Brazil)
A7. Find the maximal value of
c c c c
3 a 3 b 3 c 3 d
S“ ` ` ` ,
b`7 c`7 d`7 a`7
where a, b, c, d are nonnegative real numbers whi
h satisfy a ` b ` c ` d “ 100.
(Taiwan)
4 Cluj-Napo
a Romania, 314 July 2018
Combinatori
s
C1. Let n ě 3 be an integer. Prove that there exists a set S of 2n positive integers
satisfying the following property: For every m “ 2, 3, . . . , n the set S
an be partitioned into
two subsets with equal sums of elements, with one of subsets of
ardinality m.
(I
eland)
C2. Queenie and Horst play a game on a 20 ˆ 20
hessboard. In the beginning the board
is empty. In every turn, Horst pla
es a bla
k knight on an empty square in su
h a way that his
new knight does not atta
k any previous knights. Then Queenie pla
es a white queen on an
empty square. The game gets nished when somebody
annot move.
Find the maximal positive K su
h that, regardless of the strategy of Queenie, Horst
an
put at least K knights on the board.
(Armenia)
C3. Let n be a given positive integer. Sisyphus performs a sequen
e of turns on a board
onsisting of n ` 1 squares in a row, numbered 0 to n from left to right. Initially, n stones
are put into square 0, and the other squares are empty. At every turn, Sisyphus
hooses any
nonempty square, say with k stones, takes one of those stones and moves it to the right by at
most k squares (the stone should stay within the board). Sisyphus' aim is to move all n stones
to square n.
Prove that Sisyphus
annot rea
h the aim in less than
QnU QnU QnU QnU
` ` ` ¨¨¨`
1 2 3 n
turns. (As usual, rxs stands for the least integer not smaller than x.)
(Netherlands)
C4. An anti-Pas
al pyramid is a nite set of numbers, pla
ed in a triangle-shaped array
so that the rst row of the array
ontains one number, the se
ond row
ontains two numbers,
the third row
ontains three numbers and so on; and, ex
ept for the numbers in the bottom
row, ea
h number equals the absolute value of the dieren
e of the two numbers below it. For
instan
e, the triangle below is an anti-Pas
al pyramid with four rows, in whi
h every integer
from 1 to 1 ` 2 ` 3 ` 4 “ 10 o
urs exa
tly on
e:
4
2 6
5 7 1
8 3 10 9 .
Is it possible to form an anti-Pas
al pyramid with 2018 rows, using every integer from 1 to
1 ` 2 ` ¨ ¨ ¨ ` 2018 exa
tly on
e?
(Iran)
C5. Let k be a positive integer. The organising
ommittee of a tennis tournament is to
s
hedule the mat
hes for 2k players so that every two players play on
e, ea
h day exa
tly one
mat
h is played, and ea
h player arrives to the tournament site the day of his rst mat
h, and
departs the day of his last mat
h. For every day a player is present on the tournament, the
ommittee has to pay 1
oin to the hotel. The organisers want to design the s
hedule so as to
minimise the total
ost of all players' stays. Determine this minimum
ost.
(Russia)
Shortlisted problems 5
C6. Let a and b be distin
t positive integers. The following innite pro
ess takes pla
e on
an initially empty board.
piq If there is at least a pair of equal numbers on the board, we
hoose su
h a pair and
in
rease one of its
omponents by a and the other by b.
Prove that, no matter how we make the
hoi
es in piq, operation piiq will be performed only
nitely many times.
(Serbia)
C7. Consider 2018 pairwise
rossing
ir
les no three of whi
h are
on
urrent. These
ir
les
subdivide the plane into regions bounded by
ir
ular edges that meet at verti
es. Noti
e that
there are an even number of verti
es on ea
h
ir
le. Given the
ir
le, alternately
olour the
verti
es on that
ir
le red and blue. In doing so for ea
h
ir
le, every vertex is
oloured twi
e
on
e for ea
h of the two
ir
les that
ross at that point. If the two
olourings agree at a vertex,
then it is assigned that
olour; otherwise, it be
omes yellow. Show that, if some
ir
le
ontains
at least 2061 yellow points, then the verti
es of some region are all yellow.
(India)
6 Cluj-Napo
a Romania, 314 July 2018
Geometry
G1. Let ABC be an a
ute-angled triangle with
ir
um
ir
le Γ. Let D and E be points on
the segments AB and AC , respe
tively, su
h that AD “ AE . The perpendi
ular bise
tors of
the segments BD and CE interse
t the small ar
s AB
Ŋ and ACŊ at points F and G respe
tively.
Prove that DE k F G.
(Gree
e)
G2. Let ABC be a triangle with AB “ AC , and let M be the midpoint of BC . Let P be
a point su
h that P B ă P C and P A is parallel to BC . Let X and Y be points on the lines
P B and P C , respe
tively, so that B lies on the segment P X , C lies on the segment P Y , and
=P XM “ =P Y M . Prove that the quadrilateral AP XY is
y
li
.
(Australia)
G3. A
ir
le ω of radius 1 is given. A
olle
tion T of triangles is
alled good, if the following
onditions hold:
Determine all positive real numbers t su
h that, for ea
h positive integer n, there exists a
good
olle
tion of n triangles, ea
h of perimeter greater than t.
(South Afri
a)
G4. A point T is
hosen inside a triangle ABC . Let A1 , B1 , and C1 be the ree
tions
of T in BC , CA, and AB , respe
tively. Let Ω be the
ir
um
ir
le of the triangle A1 B1 C1 .
The lines A1 T , B1 T , and C1 T meet Ω again at A2 , B2 , and C2 , respe
tively. Prove that the
lines AA2 , BB2 , and CC2 are
on
urrent on Ω.
(Mongolia)
G5. Let ABC be a triangle with
ir
um
ir
le ω and in
entre I . A line ℓ interse
ts the
lines AI , BI , and CI at points D , E , and F , respe
tively, distin
t from the points A, B , C ,
and I . The perpendi
ular bise
tors x, y , and z of the segments AD , BE , and CF , respe
tively
determine a triangle Θ. Show that the
ir
um
ir
le of the triangle Θ is tangent to ω .
(Denmark)
G6. A
onvex quadrilateral ABCD satises AB ¨ CD “ BC ¨ DA. A point X is
hosen
inside the quadrilateral so that =XAB “ =XCD and =XBC “ =XDA. Prove that =AXB `
=CXD “ 180˝.
(Poland)
G7. Let O be the
ir
um
entre, and Ω be the
ir
um
ir
le of an a
ute-angled triangle ABC .
Let P be an arbitrary point on Ω, distin
t from A, B , C , and their antipodes in Ω. Denote
the
ir
um
entres of the triangles AOP , BOP , and COP by OA , OB , and OC , respe
tively.
The lines ℓA , ℓB , and ℓC perpendi
ular to BC , CA, and AB pass through OA , OB , and OC ,
respe
tively. Prove that the
ir
um
ir
le of the triangle formed by ℓA , ℓB , and ℓC is tangent to
the line OP .
(Russia)
Shortlisted problems 7
Number Theory
N1. Determine all pairs pn, kq of distin
t positive integers su
h that there exists a positive
integer s for whi
h the numbers of divisors of sn and of sk are equal.
(Ukraine)
N2. Let n ą 1 be a positive integer. Ea
h
ell of an n ˆ n table
ontains an integer.
Suppose that the following
onditions are satised:
piiq The sum of numbers in any row, as well as the sum of numbers in any
olumn, is
ongruent
to n modulo n2 .
Let Ri be the produ
t of the numbers in the ith row, and Cj be the produ
t of the numbers in
the j th
olumn. Prove that the sums R1 ` ¨ ¨ ¨ ` Rn and C1 ` ¨ ¨ ¨ ` Cn are
ongruent modulo n4 .
(Indonesia)
N3. Dene the sequen
e a0 , a1 , a2 , . . . by an “ 2n ` 2tn{2u . Prove that there are innitely
many terms of the sequen
e whi
h
an be expressed as a sum of (two or more) distin
t terms
of the sequen
e, as well as innitely many of those whi
h
annot be expressed in su
h a way.
(Serbia)
N4. Let a1 , a2 , . . ., an , . . . be a sequen
e of positive integers su
h that
a1 a2 an´1 an
` ` ¨¨¨` `
a2 a3 an a1
is an integer for all n ě k , where k is some positive integer. Prove that there exists a positive
integer m su
h that an “ an`1 for all n ě m.
(Mongolia)
N5. Four positive integers x, y , z , and t satisfy the relations
xy ´ zt “ x ` y “ z ` t.
Solutions
Algebra
A1. Let Qą0 denote the set of all positive rational numbers. Determine all fun
tions
f : Qą0 Ñ Qą0 satisfying ` ˘
f x2 f pyq2 “ f pxq2 f pyq p˚q
for all x, y P Qą0 .
(Switzerland)
Answer: f pxq “ 1 for all x P Qą0 .
Solution. Take any a, b P Qą0 . By substituting x “ f paq, y “ b and x “ f pbq, y “ a into p˚q
we get ` ˘2 ` ˘ ` ˘2
f f paq f pbq “ f f paq2 f pbq2 “ f f pbq f paq,
whi
h yields ` ˘2 ` ˘2
f f paq f f pbq
“ for all a, b P Qą0 .
f paq f pbq
` ˘2
In other words, this shows that there exists a
onstant C P Qą0 su
h that f f paq “ Cf paq,
or ˜ ` ˘ ¸2
f f paq f paq
“ for all a P Qą0 . (1)
C C
ˆ ˙2 ˆ ˙4 ˆ ˙2n
f paq f 2 paq f 3 paq f n`1paq
“ “ “ ¨¨¨ “
C C C C
for all positive integer n. So, f paq{C is the 2n -th power of a rational number for all positive
integer n. This is impossible unless f paq{C “ 1, sin
e otherwise the exponent of some prime in
the prime de
omposition of f paq{C is not divisible by su
iently large powers of 2. Therefore,
f paq “ C for all a P Qą0 .
Finally, after substituting f ” C into p˚q we get C “ C 3 , when
e C “ 1. So f pxq ” 1 is the
unique fun
tion satisfying p˚q.
Comment 1. There are several variations of the solution above. For instan
e, one may start with
nding f p1q “ 1. To do this, let d “ f p1q. By substituting x “ y “ 1 and x “ d2 , y “ 1 into p˚q
2 3 6 2 2 7
we get f pd q “ d and f pd q “ f pd q ¨ d “ d . By substituting now x “ 1, y “ d we obtain
2
Comment 2.
?
There exist non
onstant fun
tions f : R` Ñ R` satisfying p˚q for all real x, y ą 0
e.g., f pxq “ x.
10 Cluj-Napo
a Romania, 314 July 2018
A2. Find all positive integers n ě 3 for whi
h there exist real numbers a1 , a2 , . . . , an ,
an`1 “ a1 , an`2 “ a2 su
h that
ai ai`1 ` 1 “ ai`2
for all i “ 1, 2, . . . , n.
(Slovakia)
Answer: n
an be any multiple of 3.
Solution 1. For the sake of
onvenien
e, extend the sequen
e a1 , . . . , an`2 to an innite
periodi
sequen
e with period n. (n is not ne
essarily the shortest period.)
If n is divisible by 3, then pa1 , a2 , . . .q “ p´1, ´1, 2, ´1, ´1, 2, . . .q is an obvious solution.
We will show that in every periodi
sequen
e satisfying the re
urren
e, ea
h positive term is
followed by two negative values, and after them the next number is positive again. From this,
it follows that n is divisible by 3.
If the sequen
e
ontains two
onse
utive positive numbers ai , ai`1 , then ai`2 “ ai ai`1 `1 ą 1,
so the next value is positive as well; by indu
tion, all numbers are positive and greater than 1.
But then ai`2 “ ai ai`1 ` 1 ě 1 ¨ ai`1 ` 1 ą ai`1 for every index i, whi
h is impossible: our
sequen
e is periodi
, so it
annot in
rease everywhere.
If the number 0 o
urs in the sequen
e, ai “ 0 for some index i, then it follows that
ai`1 “ ai´1 ai ` 1 and ai`2 “ ai ai`1 ` 1 are two
onse
utive positive elements in the sequen
es
and we get the same
ontradi
tion again.
Noti
e that after any two
onse
utive negative numbers the next one must be positive: if
ai ă 0 and ai`1 ă 0, then ai`2 “ a1 ai`1 ` 1 ą 1 ą 0. Hen
e, the positive and negative numbers
follow ea
h other in su
h a way that ea
h positive term is followed by one or two negative values
and then
omes the next positive term.
Consider the
ase when the positive and negative values alternate. So, if ai is a negative
value then ai`1 is positive, ai`2 is negative and ai`3 is positive again.
Noti
e that ai ai`1 ` 1 “ ai`2 ă 0 ă ai`3 “ ai`1 ai`2 ` 1; by ai`1 ą 0 we
on
lude ai ă ai`2 .
Hen
e, the negative values form an innite in
reasing subsequen
e, ai ă ai`2 ă ai`4 ă . . .,
whi
h is not possible, be
ause the sequen
e is periodi
.
The only
ase left is when there are
onse
utive negative numbers in the sequen
e. Suppose
that ai and ai`1 are negative; then ai`2 “ ai ai`1 ` 1 ą 1. The number ai`3 must be negative.
We show that ai`4 also must be negative.
Noti
e that ai`3 is negative and ai`4 “ ai`2 ai`3 ` 1 ă 1 ă ai ai`1 ` 1 “ ai`2 , so
ai`5 ´ ai`4 “ pai`3 ai`4 ` 1q ´ pai`2 ai`3 ` 1q “ ai`3 pai`4 ´ ai`2 q ą 0,
therefore ai`5 ą ai`4 . Sin
e at most one of ai`4 and ai`5
an be positive, that means that ai`4
must be negative.
Now ai`3 and ai`4 are negative and ai`5 is positive; so after two negative and a positive
terms, the next three terms repeat the same pattern. That
ompletes the solution.
Solution 2. We prove that the shortest period of the sequen
e must be 3. Then it follows
that n must be divisible by 3.
Noti
e that the equation x2 ` 1 “ x has no real root, so the numbers a1 , . . . , an
annot be
all equal, hen
e the shortest period of the sequen
e
annot be 1.
By applying the re
urren
e relation for i and i ` 1,
pai`2 ´ 1qai`2 “ ai ai`1 ai`2 “ ai pai`3 ´ 1q, so
a2i`2 ´ ai ai`3 “ ai`2 ´ ai .
Shortlisted problems solutions 11
That proves that ai “ ai`3 for every index i, so the sequen
e a1 , a2 , . . . is indeed periodi
with
period 3. The shortest period
annot be 1, so it must be 3; therefore, n is divisible by 3.
A3. Given any set S of positive integers, show that at least one of the following two
assertions holds:
ř ř
(1) There exist distin
t nite subsets F and G of S su
h that xPF 1{x “ xPG 1{x ;
ř
(2) There exists a positive rational number r ă 1 su
h that xPF 1{x ‰ r for all nite subsets
F of S .
(Luxembourg)
Solution 1. Argue indire
tly. Agree, as usual, that the empty sum is 0 to
onsider rationals
ř
in r0, 1q; adjoining 0
auses no harm, sin
e xPF 1{x “ 0 for no nonempty nite řsubset F of S .
For every rational r in r0, 1q, let Fr be the unique nite subset of S su
h that xPFr 1{x “ r .
The argument hinges on the lemma below.
Lemma. If x is a member of S and q and r are rationals in r0, 1q su
h that q ´ r “ 1{x, then x
is a member of Fq if and only if it is not one of Fr .
Proof. If x is a member of Fq , then
ÿ 1 ÿ 1 1 1 ÿ 1
“ ´ “q´ “r“ ,
yPFq rtxu
y yPF
y x x yPF
y
q r
Comment. The solution above
an be adapted to show that the problem statement still holds, if the
ondition r ă 1 in (2) is repla
ed with r ă δ , for an arbitrary positive δ . This yields that, if S does not
ř
satisfy (1), then there exist innitely many positive rational numbers r ă 1 su
h that xPF 1{x ‰ r
for all nite subsets F of S.
Solution 2. A nite S
learly satises (2), so let S be innite. If S fails both
onditions,
so does S r t1u. We may and will therefore assume that S
onsists of integers greater than 1.
Label the elements of S in
reasingly x1 ă x2 ă ¨ ¨ ¨ , where x1 ě 2.
We rst show that S satises (2) if xn`1 ě 2xn for all n. In this
ase, xn ě 2n´1 x1 for
all n, so
ÿ 1 ÿ 1 2
s“ ď n´1 x
“ .
ně1
xn ně1
2 1 x 1
ř
If x1 ě 3, or x1 “ 2 and xn`1 ą 2xn for some n, then xPF 1{x ă s ă 1 for every nite subset
F of S , so S satises (2); and if x1 “ 2 and xn`1 “ 2xř n for all n, that is, xn “ 2 for all n, then
n
every nite subset F of S
onsists of powers of 2, so xPF 1{x ‰ 1{3 and again S satises (2).
Finally, we deal with the
ase where
ř xn`1 ă 2xn for some n. Consider the positive rational
r “ 1{xn ´ 1{xn`1 ă 1{xn`1 . If r “ xPF 1{x for no nite subset F of S , then S satises (2).
Shortlisted problems solutions 13
ř
We now
ř assume that r “ xPF0 1{x for some nite subset F0 of S , and show that S satises (1).
Sin
e xPF0 1{x “ r ă 1{xn`1 , it follows that xn`1 is not a member of F0 , so
ÿ 1 ÿ 1 1 1 1
“ ` “r` “ .
xPF0 Ytxn`1 u
x xPF x xn`1 xn`1 xn
0
Consequently,
ř ř F “ F0 Y txn`1 u and G “ txn u are distin
t nite subsets of S su
h that
xPF 1{x “ xPG 1{x, and S satises (1).
14 Cluj-Napo
a Romania, 314 July 2018
In parti
ular, Spn, 0q “ 0 and Spn, 1q “ an´1 . In these terms, for every integer n ě 2 there
exists a positive integer k ď n su
h that an “ Spn, kq{k .
For every integer n ě 1 we dene
Spn, kq Spn, kq
Mn “ max , mn “ min , and ∆n “ Mn ´ mn ě 0.
1ďkďn k 1ďkďn k
By denition, an P rmn , Mn s for all n ě 2; on the other hand, an´1 “ Spn, 1q{1 P rmn , Mn s.
Therefore,
a2018 ´ a2017 ď M2018 ´ m2018 “ ∆2018 ,
and we are interested in an upper bound for ∆2018 .
Also by denition, for any 0 ă k ď n we have kmn ď Spn, kq ď kMn ; noti
e that these
inequalities are also valid for k “ 0.
Claim 1. For every n ą 2, we have ∆n ď n´1 n
∆n´1 .
Proof. Choose positive integers k, ℓ ď n su
h that Mn “ Spn, kq{k and mn “ Spn, ℓq{ℓ. We
have Spn, kq “ an´1 ` Spn ´ 1, k ´ 1q, so
Ba
k to the problem, if an “ 1 for all n ď 2017, then a2018 ď 1 and hen
e a2018 ´ a2017 ď 0.
Otherwise, let 2 ď q ď 2017 be the minimal index with aq ă 1. We have Spq, iq “ i for all
i “ 1, 2, . . . , q ´ 1, while Spq, qq “ q ´ 1. Therefore, aq ă 1 yields aq “ Spq, qq{q “ 1 ´ 1q .
Now we have Spq ` 1, iq “ i ´ 1q for i “ 1, 2, . . . , q , and Spq ` 1, q ` 1q “ q ´ 1q . This gives us
so ∆q`1 “ Mq`1 ´ mq`1 “ pq ´ 1q{q 2 . Denoting N “ 2017 ě q and using Claim 1 for
n “ q ` 2, q ` 3, . . . , N ` 1 we nally obtain
ˆ ˙ ˆ ˙
q´1 q`1 q`2 N 1 1 1 1 N ´1
∆N `1 ď 2 ¨ ¨ ¨¨¨ “ 1´ 2 ď 1´ 2 “ ,
q q`2 q`3 N `1 N `1 q N `1 N N2
as required.
Comment 1. One may
he
k that the maximal value of a2018 ´ a2017 is attained at the unique
sequen
e, whi
h is presented in the solution above.
Comment 2. An easier question would be to determine the maximal value of |a2018 ´ a2017 |. In this
1
version, the answer
2018 is a
hieved at
a2017 ` ¨ ¨ ¨ ` a0 1
a1 “ a2 “ ¨ ¨ ¨ “ a2017 “ 1, a2018 “ “1´ .
2018 2018
1
To prove that this value is optimal, it su
es to noti
e that ∆2 “ 2 and to apply Claim 1 obtaining
1 2 2017 1
|a2018 ´ a2017 | ď ∆2018 ď ¨ ¨¨¨ “ .
2 3 2018 2018
whi
h establishes the rst inequality in the Claim. The proof of the se
ond inequality is
similar. l
Claim 3. For every positive integers k ě n, we have mn ď ak ď Mn .
Proof. By Claim 2, we have rmk , Mk s Ď rmk´1 , Mk´1 s Ď ¨ ¨ ¨ Ď rmn , Mn s. Sin
e ak P rmk , Mk s,
the
laim follows. l
16 Cluj-Napo
a Romania, 314 July 2018
Claim 4. For every integer n ě 2, we have Mn “ Spn, n ´ 1q{pn ´ 1q and mn “ Spn, nq{n.
Proof. We use indu
tion on n. The base
ase n “ 2 is routine. To perform the indu
tion step,
we need to prove the inequalities
Spn ´ k, n ´ kq
pn ´ kqSpn, kq ě kSpn ´ k, n ´ kq ðñ Spn, kq ě k ¨ .
n´k
By the indu
tion hypothesis, we have Spn ´ k, n ´ kq{pn ´ kq “ mn´k . By Claim 3, we get
an´i ě mn´k for all i “ 1, 2, . . . , k . Summing these k inequalities we obtain
Spn ´ k, n ´ kq
Spn, kq ě kmn´k “ k ¨ ,
n´k
as required.
The se
ond inequality in (1) is proved similarly. Indeed, this inequality is equivalent to
SpN ` 1, Nq aN ` SpN, N ´ 1q
aN `1 ´ aN ď MN `1 ´ aN “ ´ aN “ ´ aN
N N
SpN, N ´ 1q N ´ 1
“ ´ ¨ aN .
N N
On the other hand, the same Claim yields
SpN, Nq SpN, N ´ 1q
aN ě mN “ “ .
N N
Noti
ing that ea
h term in SpN, N ´ 1q is at most 1, so SpN, N ´ 1q ď N ´ 1, we nally obtain
Comment 2. Both solutions above dis
uss the properties of an arbitrary sequen
e satisfying the
problem
onditions. Instead, one may investigate only an optimal sequen
e whi
h maximises the value
of a2018 ´ a2017 . Here we present an observation whi
h allows to simplify su
h investigation for
instan
e, the proofs of Claim 1 in Solution 1 and Claim 4 in Solution 2.
The sequen
e pan q is uniquely determined by
hoosing, for every n ě 2, a positive integer kpnq ď n
su
h that an “ Spn, kpnqq{kpnq. Take an arbitrary 2 ď n0 ď 2018, and assume that all su
h inte-
gers kpnq, for n ‰ n0 , are xed. Then, for every n, the value of an is a linear fun
tion in an0 (whose
possible values
onstitute some dis
rete subset of rmn0 , Mn0 s
ontaining both endpoints). Hen
e,
a2018 ´ a2017 is also a linear fun
tion in an0 , so it attains its maximal value at one of the endpoints of
the segment rmn0 , Mn0 s.
This shows that, while dealing with an optimal sequen
e, we may assume an P tmn , Mn u for all
`nMn
2 ď n ď 2018. Now one
an easily see that, if an “ mn , then mn`1 “ mn and Mn`1 ď mnn`1 ; similar
estimates hold in the
ase an “ Mn . This already establishes Claim 1, and simplies the indu
tive
proof of Claim 4, both applied to an optimal sequen
e.
18 Cluj-Napo
a Romania, 314 July 2018
for all x, y ą 0.
(South Korea)
C2
Answer: f pxq “ C1 x ` with arbitrary
onstants C1 and C2 .
x
Solution 1. Fix a real number a ą 1, and take a new variable t. For the values f ptq, f pt2 q,
f patq and f pa2 t2 q, the relation (1) provides a system of linear equations:
ˆ ˙
1
x “ y “ t: t` f ptq “ f pt2 q ` f p1q (2a)
ˆ t ˙
t t a
x “ , y “ at : ` f patq “ f pt2 q ` f pa2 q (2b)
a ˆ a t ˙ ˆ ˙
1 1
x “ a2 t, y “ t : a2 t ` 2 f ptq “ f pa2 t2 q ` f (2
)
ˆ a ˙t a2
1
x “ y “ at : at ` f patq “ f pa2 t2 q ` f p1q (2d)
at
In order to eliminate f pt2 q, take the dieren
e of (2a) and (2b); from (2
) and (2d) eliminate
f pa2 t2 q; then by taking a linear
ombination, eliminate f patq as well:
ˆ ˙ ˆ ˙
1 t a
t` f ptq ´ ` f patq “ f p1q ´ f pa2 q and
t a t
ˆ ˙ ˆ ˙
1 1
2
a t ` 2 f ptq ´ at ` f patq “ f p1{a2 q ´ f p1q, so
at at
˜ˆ ˙ˆ ˙ ˆ ˙ˆ ˙¸
1 1 t a 1
at ` t` ´ ` a2 t ` 2 f ptq
at t a t at
ˆ ˙ ˆ ˙
1 ` 2
˘ t a ` ˘
“ at ` f p1q ´ f pa q ´ ` f p1{a2 q ´ f p1q .
at a t
Noti
e that on the left-hand side, the
oe
ient of f ptq is nonzero and does not depend on t:
ˆ ˙ˆ ˙ ˆ ˙ˆ ˙ ˆ ˙
1 1 t a 2 1 1 3 1
at ` t` ´ ` a t` 2 “ a ` ´ a ` 3 ă 0.
at t a t at a a
C2 paq
f pan q “ C1 paq ¨ an ` for every a ‰ 1 and every integer n. p4q
an
The relation (4)
an be easily extended to rational values of n, so we may
onje
ture that C1
and C2 are
onstants, and when
e f ptq “ C1 t ` Ct2 . As it was seen in the previous solution,
su
h fun
tions indeed satisfy (1).
The equation (1) is linear in f ; so if some fun
tions f1 and f2 satisfy (1) and c1 , c2 are real
numbers, then c1 f1 pxq ` c2 f2 pxq is also a solution of (1). In order to make our formulas simpler,
dene
f0 pxq “ f pxq ´ f p1q ¨ x.
This fun
tion is another one satisfying (1) and the extra
onstraint f0 p1q “ 0. Repeating the
same argument on linear re
urren
es, we
an write f0 paq “ Kpaqan ` Lpaq
an
with some fun
tions
K and L. By substituting n “ 0, we
an see that Kpaq ` Lpaq “ f0 p1q “ 0 for every a. Hen
e,
ˆ ˙
n n 1
f0 pa q “ Kpaq a ´ n .
a
Now take two numbers a ą b ą 1 arbitrarily and substitute x “ pa{bqn and y “ pabqn in (1):
ˆ n ˙
a bn ` ˘ ` ˘ ` ˘
n
` n f0 pabqn “ f0 a2n ` f0 b2n , so
b a
ˆ n n
˙ ˆ ˙ ˆ ˙ ˆ ˙
a b 1 1 1
` Kpabq pabq ´n
“ Kpaq a ´ 2n ` Kpbq b ´ 2n , or equivalently
2n 2n
bn an pabqn a b
ˆ ˙ ˆ ˙ ˆ ˙
1 1 1 1
2n 2n 2n
Kpabq a ´ 2n ` b ´ 2n “ Kpaq a ´ 2n ` Kpbq b ´ 2n . 2n
(5)
a b a b
By dividing (5) by a2n and then taking limit with n Ñ `8 we get Kpabq “ Kpaq. Then (5)
redu
es to Kpaq “ Kpbq. Hen
e, Kpaq “ Kpbq for all a ą b ą 1.
Fix a ą 1. For every x ą 0 there is some b and an integer n su
h that 1 ă b ă a and x “ bn .
Then ˆ ˙ ˆ ˙
n n 1 1
f0 pxq “ f0 pb q “ Kpbq b ´ n “ Kpaq x ´ .
b x
Hen
e, we have f pxq “ f0 pxq ` f p1qx “ C1 x ` C2
x
with C1 “ Kpaq ` f p1q and C2 “ ´Kpaq.
Comment. After establishing (5), there are several variants of nishing the solution. For example,
instead of taking a limit, we
an obtain a system of linear equations for Kpaq, Kpbq and Kpabq by
substituting two positive integers n in (5), say n “1 and n “ 2. This approa
h leads to a similar
ending as in the rst solution.
` ˘
Optionally, we dene another fun
tion f1 pxq “ f0 pxq ´ C x ´ x1 and pres
ribe Kpcq “ 0 for
another xed c. Then we
an
hoose ab “ c and de
rease the number of terms in (5).
20 Cluj-Napo
a Romania, 314 July 2018
A6. Let m, n ě 2 be integers. Let f px1 , . . . , xn q be a polynomial with real
oe
ients su
h
that Yx ` . . . ` x ]
1 n (
f px1 , . . . , xn q “ for every x1 , . . . , xn P 0, 1, . . . , m ´ 1 .
m
Prove that the total degree of f is at least n.
(Brazil)
Solution. We transform the problem to a single variable question by the following
Lemma. Let a1 , . . . , an be nonnegative integers and let Gpxq be a nonzero polynomial with
deg G ď a1 ` . . . ` an . Suppose that some polynomial F px1 , . . . , xn q satises
Comment 1. In the lemma we have equality for the
hoi
e F px1 , . . . , xn q “ Gpx1 ` . . . ` xn q, so it
indeed transforms the problem to an equivalent single-variable question.
#
1 if x ” ´1 pmod mq
p∆gqpxq “ for x “ 0, 1, . . . , npm ´ 1q ´ 1.
0 otherwise
Hen
e, ∆g vanishes at all integers x with 0 ď x ă npm ´ 1q and x ı ´1 pmod mq. This leads to
pm´1q2 n
deg g ě m ` 1.
m is even then this lower bound
If
an be improved to npm ´ 1q. For 0 ď N ă npm ´ 1q, the
pN ` 1qst forward dieren
e at x “ 0 is
ÿN ˆ ˙ ÿ ˆ ˙
` N `1 ˘ N ´k N N ´k N
∆ gp0q “ p´1q p∆gqpkq “ p´1q . p˚q
k“0
k 0ďkďN
k
k”´1 pmod mq
Sin
e m is even, all signs in the last sum are equal; with N “ npm ´ 1q´ 1 this proves ∆npm´1q gp0q ‰ 0,
indi
ating that deg g ě npm ´ 1q.
However, there are innitely many
ases when all terms in p˚q
an
el out, for example if m is an
odd divisor of n ` 1. In su
h
ases, deg f
an be less than npm ´ 1q.
AlonFüredi bound. Let S1 , . . . , Sn be nonempty nite sets in a eld and suppose that
the polynomial P px1 , . . . , xn q vanishes at the points of the grid S1 ˆ . . . ˆ Sn , ex
ept for a
řn ` ˘
single point. Then deg P ě |Si | ´ 1 .
i“1
(A well-known appli
ation of the AlonFüredi bound was the former IMO problem 2007/6.
Sin
e then, this result be
ame popular among the students and is part of the IMO training
for many IMO teams.)
The proof of the lemma
an be repla
ed by an appli
ation of the AlonFüredi bound as follows. Let
(
d “ deg G, and let G0 be the unique polynomial su
h that G0 pxq “ Gpxq for x P 0, 1, . . . , d ´ 1 but
deg G0 ă d. The polynomials G0 and G are dierent be
ause they have dierent degrees, and they
attain the same values at 0, 1, . . . , d ´ 1; that enfor
es G0 pdq ‰ Gpdq.
Choose some nonnegative integers b1 , . . . , bn so that b1 ď a1 , . . . , bn ď an , and b1 ` . . . ` bn “ d,
and
onsider the polynomial
c
x ` t ` 14
Claim. The rst bra
ket above does not ex
eed
3
.
7
Proof. Sin
e
1 ` ˘
X 3 ` Y 3 ` 3XY Z ´ Z 3 “ pX ` Y ´ Zq pX ´ Y q2 ` pX ` Zq2 ` pY ` Zq2 ,
2
the inequality X ` Y ď Z is equivalent (when X, Y, Z ě 0) to X 3 ` Y 3 ` 3XY Z ď Z 3 .
Therefore, the
laim is equivalent to
d
x t 3 xtpx ` t ` 14q x ` t ` 14
` `3 ď .
t`7 x`7 7px ` 7qpt ` 7q 7
Noti
e that
d d
3 xtpx ` t ` 14q 3 tpx ` 7q xpt ` 7q 7px ` t ` 14q
3 “3 ¨ ¨
7px ` 7qpt ` 7q 7pt ` 7q 7px ` 7q pt ` 7qpx ` 7q
tpx ` 7q xpt ` 7q 7px ` t ` 14q
ď ` `
7pt ` 7q 7px ` 7q pt ` 7qpx ` 7q
?
3
Solution 2. We present a dierent proof for the estimate S ď 8{ 7.
Start by using Hölder's inequality:
˜ ? ¸3 ÿ ˙3 ˜ÿ ¸2 ÿ
ÿ? 6a¨ 6 a `? ˘3
ÿ ` ? ˘3
ÿ ˆ
1 ? 1
S3 “ ?
3 ď 6
a ¨ 6
a ¨ ?
3 “ a .
cyc b ` 7 cyc cyc cyc b ` 7 cyc cyc
b ` 7
Noti
e that
px ´ 1q2 px ´ 7q2 2 448
ě 0 ðñ x ´ 16x ` 71 ě
x2 ` 7 x2 ` 7
yields
?
ÿ 1 1 ÿ` ? ˘ 1 ´ ÿ ? ¯ 48 ´ 2 ř b
ď b ´ 16 b ` 71 “ 384 ´ 16 b “ .
b`7 448 448 56
Finally,
˜ř ? ř? ` ř ? ˘ ¸3
1 ´ÿ ? ¯2 ´ ÿ? ¯ 1 a` a ` 48 ´ 2 a 512
S3 ď a 48 ´ 2 a ď “
56 56 3 7
Comment. All the above works if we repla
e 7 and 100 with k ą 0 and 2pk2 ` 1q, respe
tively; in this
ase, the answer be
omes
c
3 pk ` 1q2
2 .
k
Even further, a linear substitution allows to extend the solutions to a version with 7 and 100 being
repla
ed with arbitrary positive real numbers p and q satisfying q ě 4p.
24 Cluj-Napo
a Romania, 314 July 2018
Combinatori
s
C1. Let n ě 3 be an integer. Prove that there exists a set S of 2n positive integers
satisfying the following property: For every m “ 2, 3, . . . , n the set S
an be partitioned into
two subsets with equal sums of elements, with one of subsets of
ardinality m.
(I
eland)
Solution. We show that one of possible examples is the set
" *
k k 3n ` 9
S “ t1 ¨ 3 , 2 ¨ 3 : k “ 1, 2, . . . , n ´ 1u Y 1, ´1 .
2
It is readily veried that all the numbers listed above are distin
t (noti
e that the last two are
not divisible by 3).
The sum of elements in S is
ˆ n ˙ n´1ÿ n´1
3 `9 k k 3n ` 9 ÿ k`1 3n ` 9 3n`1 ´ 9
Σ “ 1` ´1 ` p1 ¨ 3 ` 2 ¨ 3 q “ ` 3 “ ` “ 2 ¨ 3n .
2 k“1
2 k“1
2 2
Hen
e, in order to show that this set satises the problem requirements, it su
es to present,
for every m “ 2, 3, . . . , n, an m-element subset Am Ă S whose sum of elements equals 3n .
Su
h a subset is
Am “ t2 ¨ 3k : k “ n ´ m ` 1, n ´ m ` 2, . . . , n ´ 1u Y t1 ¨ 3n´m`1 u.
as required.
Comment. Let us present a more general
onstru
tion. Let s1 , s2 , . . . , s2n´1 be a sequen
e of pairwise
distin
t positive integers satisfying s2i`1 “ s2i ` s2i´1 for all i “ 2, 3, . . . , n ´ 1. Set s2n “ s1 ` s2 `
¨ ¨ ¨ ` s2n´4 .
Assume that s2n is distin
t from the other terms of the sequen
e. Then the set S “ ts1 , s2 , . . . , s2n u
satises the problem requirements. Indeed, the sum of its elements is
2n´4
ÿ
Σ“ si ` ps2n´3 ` s2n´2 q ` s2n´1 ` s2n “ s2n ` s2n´1 ` s2n´1 ` s2n “ 2s2n ` 2s2n´1 .
i“1
Therefore, we have
Σ
“ s2n ` s2n´1 “ s2n ` s2n´2 ` s2n´3 “ s2n ` s2n´2 ` s2n´4 ` s2n´5 “ . . . ,
2
whi
h shows that the required sets Am
an be
hosen as
The solution above is an instan
e of this general
onstru
tion. Another instan
e, for n ą 3, is the
set
tF1 , F2 , . . . , F2n´1 , F1 ` ¨ ¨ ¨ ` F2n´4 u,
where F1 “ 1, F2 “ 2, Fn`1 “ Fn ` Fn´1 is the usual Fibona
i sequen
e.
Shortlisted problems solutions 25
C2. Queenie and Horst play a game on a 20 ˆ 20
hessboard. In the beginning the board
is empty. In every turn, Horst pla
es a bla
k knight on an empty square in su
h a way that his
new knight does not atta
k any previous knights. Then Queenie pla
es a white queen on an
empty square. The game gets nished when somebody
annot move.
Find the maximal positive K su
h that, regardless of the strategy of Queenie, Horst
an
put at least K knights on the board.
(Armenia)
Answer: K “ 202 {4 “ 100. In
ase of a 4N ˆ 4M board, the answer is K “ 4NM .
Solution. We show two strategies, one for Horst to pla
e at least 100 knights, and another
strategy for Queenie that prevents Horst from putting more than 100 knights on the board.
A strategy for Horst: Put knights only on bla
k squares, until all bla
k squares get
o
upied.
Colour the squares of the board bla
k and white in the usual way, su
h that the white
and bla
k squares alternate, and let Horst put his knights on bla
k squares as long as it is
possible. Two knights on squares of the same
olour never atta
k ea
h other. The number of
bla
k squares is 202 {2 “ 200. The two players o
upy the squares in turn, so Horst will surely
nd empty bla
k squares in his rst 100 steps.
A strategy for Queenie: Group the squares into
y
les of length 4, and after ea
h step
of Horst, o
upy the opposite square in the same
y
le.
Consider the squares of the board as verti
es of a graph; let two squares be
onne
ted if
two knights on those squares would atta
k ea
h other. Noti
e that in a 4 ˆ 4 board the squares
an be grouped into 4
y
les of length 4, as shown in Figure 1. Divide the board into parts of
size 4 ˆ 4, and perform the same grouping in every part; this way we arrange the 400 squares
of the board into 100
y
les (Figure 2).
A C
B
Figure 1 Figure 2 Figure 3
The strategy of Queenie
an be as follows: Whenever Horst puts a new knight to a
ertain
square A, whi
h is part of some
y
le A ´ B ´ C ´ D ´ A, let Queenie put her queen on the
opposite square C in that
y
le (Figure 3). From this point, Horst
annot put any knight on
A or C be
ause those squares are already o
upied, neither on B or D be
ause those squares
are atta
ked by the knight standing on A. Hen
e, Horst
an put at most one knight on ea
h
y
le, that is at most 100 knights in total.
Comment 1. Queenie's strategy
an be pres
ribed by a simple rule: divide the board into 4ˆ4
parts; whenever Horst puts a knight in a part P, Queenie ree
ts that square about the
entre of P
and puts her queen on the ree
ted square.
Comment 2. The result remains the same if Queenie moves rst. In the rst turn, she may put
her rst queen arbitrarily. Later, if she has to put her next queen on a square that already
ontains a
queen, she may move arbitrarily again.
26 Cluj-Napo
a Romania, 314 July 2018
C3. Let n be a given positive integer. Sisyphus performs a sequen
e of turns on a board
onsisting of n ` 1 squares in a row, numbered 0 to n from left to right. Initially, n stones
are put into square 0, and the other squares are empty. At every turn, Sisyphus
hooses any
nonempty square, say with k stones, takes one of those stones and moves it to the right by at
most k squares (the stone should stay within the board). Sisyphus' aim is to move all n stones
to square n.
Prove that Sisyphus
annot rea
h the aim in less than
QnU QnU QnU QnU
` ` ` ¨¨¨`
1 2 3 n
turns. (As usual, rxs stands for the least integer not smaller than x.)
(Netherlands)
Solution. The stones are indistinguishable, and all have the same origin and the same nal
position. So, at any turn we
an pres
ribe whi
h stone from the
hosen square to move. We
do it in the following manner. Number the stones from 1 to n. At any turn, after
hoosing a
square, Sisyphus moves the stone with the largest number from this square.
This way, when stone k is moved from some square, that square
ontains not more than k
stones (sin
e all their numbers are at most k ). Therefore, stone k is moved by at most k squares
at ea
h turn. Sin
e the total shift of the stone is exa
tly n, at least rn{ks moves of stone k
should have been made, for every k “ 1, 2, . . . , n.
By summing up over all k “ 1, 2, . . . , n, we get the required estimate.
Comment. The original submission
ontained the se
ond part, asking for whi
h values of n the equality
an be a
hieved. The answer is n “ 1, 2, 3, 4, 5, 7. The Problem Sele
tion Committee
onsidered this
part to be less suitable for the
ompetition, due to te
hni
alities.
Shortlisted problems solutions 27
a1
a2 b2
a3 b3 T
..................
an-1 bn-1
T’ T’’
bn an
Figure 1 Figure 2
Consider now (Figure 2) the two `equilateral' subtriangles of T whose bottom rows
ontain
the numbers to the left, respe
tively right, of the pair an , bn . (One of these subtriangles may
very well be empty.) At least one of these subtriangles, say T 1 , has side length ℓ ě rpn ´ 2q{2s.
Sin
e T 1 obeys the anti-Pas
al rule, it
ontains ℓ pairwise distin
t positive integers a11 , a12 , . . . , a1ℓ ,
where a11 is at the apex, and a1k and b1k “ a11 ` a12 ` ¨ ¨ ¨` a1k are the two neighbours below b1k´1 for
ea
h k “ 2, 3 . . . , ℓ. Sin
e the ak all lie outside T 1 , and they form a permutation of 1, 2, . . . , n,
the a1k are all greater than n. Consequently,
ℓp2n ` ℓ ` 1q
b1ℓ ě pn ` 1q ` pn ` 2q ` ¨ ¨ ¨ ` pn ` ℓq “
ˆ ˙ 2
1 n´2 n´2 5npn ´ 2q
ě ¨ 2n ` `1 “ ,
2 2 2 8
whi
h is greater than 1 ` 2 ` ¨ ¨ ¨ ` n “ npn ` 1q{2 for n “ 2018. A
ontradi
tion.
Comment. The above estimate may be slightly improved by noti
ing that b1ℓ ‰ bn . This implies
npn ` 1q{2 “ bn ą b1ℓ ě rpn ´ 2q{2s p2n ` rpn ´ 2q{2s ` 1q {2, so nď7 if n is odd, and n ď 12 if n is
even. It seems that the largest anti-Pas
al pyramid whose entries are a permutation of the integers
from 1 to 1 ` 2 ` ¨¨¨ ` n has 5 rows.
28 Cluj-Napo
a Romania, 314 July 2018
Bounding the total
ost from below. To` ˘this end, estimate ei`1 ´ bi`1 ` 1. Before day
` i ˘ bi`1 ,
only i players were present, so at most 2 mat
hes
ould be played. Therefore, bi`1 ď
i
2
` 1.
`˘ ` ˘ `i˘
Similarly, at most 2i mat
hes
ould be played after day ei`1 , so ei ě 2k
2
´ 2 . Thus,
ˆ ˙ ˆ ˙
2k i
ei`1 ´ bi`1 ` 1 ě ´2 “ kp2k ´ 1q ´ ipi ´ 1q.
2 2
This lower bound
an be improved for i ą k : List the i players who arrived rst, and
the i players who departed last; at least 2i ´ 2k players appear in both lists. The mat
hes
between these players were
ounted twi
e, though the players in ea
h pair have played only
on
e. Therefore, if i ą k , then
ˆ ˙ ˆ ˙ ˆ ˙
2k i 2i ´ 2k
ei`1 ´ bi`1 ` 1 ě ´2 ` “ p2k ´ iq2 .
2 2 2
An optimal tournament, We now des
ribe a s
hedule in whi
h the lower bounds above are all
a
hieved simultaneously. Split players into two groups X and Y , ea
h of
ardinality k . Next,
partition the s
hedule into three parts. During the rst part, the players from X arrive one by
one, and ea
h newly arrived player immediately plays with everyone already present. During
the third part (after all players from X have already departed) the players from Y depart one
by one, ea
h playing with everyone still present just before departing.
In the middle part, everyone from X should play with everyone from Y . Let S1 , S2 , . . . , Sk
be the players in X , and let T1 , T2 , . . . , Tk be the players in Y . Let T1 , T2 , . . . , Tk arrive in
this order; after Tj arrives, he immediately plays with all the Si , i ą j . Afterwards, players Sk ,
Sk´1 , . . . , S1 depart in this order; ea
h Si plays with all the Tj , i ď j , just before his departure,
and Sk departs the day Tk arrives. For 0 ď s ď k ´ 1, the number of mat
hes played between
Tk´s 's arrival and Sk´s 's departure is
k´1
ÿ k´1
ÿ 1 1
pk ´ jq ` 1 ` pk ´ j ` 1q “ sps ` 1q ` 1 ` sps ` 3q “ ps ` 1q2 .
j“k´s j“k´s
2 2
Thus, if i ą k , then the number of mat
hes that have been played between Ti´k`1 's arrival,
whi
h is bi`1 , and Si´k`1 's departure, whi
h is ei`1 , is p2k ´iq2 ; that is, ei`1 ´bi`1 `1 “ p2k ´iq2 ,
showing the se
ond lower bound a
hieved for all i ą k .
Shortlisted problems solutions 29
The latter sum is fairly tra table and yields the stated result; we omit the details.
Comment. If the number of players is odd, say, 2k ´ 1, the required minimum is kpk ´ 1qp4k ´ 1q{2.
In this
ase, |X| “ k, |Y | “ k ´ 1, the argument goes along the same lines, but some additional
te
hni
alities are to be taken
are of.
Shortlisted problems solutions 31
C6. Let a and b be distin
t positive integers. The following innite pro
ess takes pla
e on
an initially empty board.
piq If there is at least a pair of equal numbers on the board, we
hoose su
h a pair and
in
rease one of its
omponents by a and the other by b.
Prove that, no matter how we make the
hoi
es in piq, operation piiq will be performed only
nitely many times.
(Serbia)
Solution 1. We may assume gcdpa, bq “ 1; otherwise we work in the same way with multiples
of d “ gcdpa, bq.
Suppose that after N moves of type piiq and some moves of type piq we have to add two
new zeros. For ea
h integer k , denote by f pkq the number of times that the number k appeared
on the board up to this moment. Then f p0q “ 2N and f pkq “ 0 for k ă 0. Sin
e the board
ontains at most one k ´ a, every se
ond o
urren
e of k ´ a on the board produ
ed, at some
moment, an o
urren
e of k ; the same stands for k ´ b. Therefore,
Z ^ Z ^
f pk ´ aq f pk ´ bq
f pkq “ ` , p1q
2 2
yielding
f pk ´ aq ` f pk ´ bq
f pkq ě ´ 1. p2q
2
Sin
e gcdpa, bq “ 1, every integer x ą ab ´ a ´ b is expressible in the form x “ sa ` tb, with
integer s, t ě 0.
We will prove by indu
tion on s ` t that if x “ sa ` bt, with s, t nonnegative integers, then
f p0q
f pxq ą ´ 2. p3q
2s`t
The base
ase s`t “ 0 is trivial. Assume now that p3q is true for s`t “ v . Then, if s`t “ v `1
and x “ sa ` tb, at least one of the numbers s and t say s is positive, hen
e by p2q,
` ˘ ˆ ˙
f ps ´ 1qa ` tb 1 f p0q f p0q
f pxq “ f psa ` tbq ě ´1ą s`t´1
´ 2 ´ 1 “ s`t ´ 2.
2 2 2 2
Assume now that we must perform moves of type piiq ad innitum. Take n “ ab ´ a ´ b and
suppose b ą a. Sin
e ea
h of the numbers n ` 1, n ` 2, . . . , n ` b
an be expressed in the form
sa ` tb, with 0 ď s ď b and 0 ď t ď a, after moves of type piiq have been performed 2a`b`1
times and we have to add a new pair of zeros, ea
h f pn ` kq, k “ 1, 2, . . . , b, is at least 2. In
this
ase p1q yields indu
tively f pn ` kq ě 2 for all k ě 1. But this is absurd: after a nite
number of moves, f
annot attain nonzero values at innitely many points.
Solution 2. We start by showing that the result of the pro
ess in the problem does not
depend on the way the operations are performed. For that purpose, it is
onvenient to modify
the pro
ess a bit.
Claim 1. Suppose that the board initially
ontains a nite number of nonnegative integers,
and one starts performing type piq moves only. Assume that one had applied k moves whi
h led
to a nal arrangement where no more type piq moves are possible. Then, if one starts from the
same initial arrangement, performing type piq moves in an arbitrary fashion, then the pro
ess
will ne
essarily stop at the same nal arrangement
Shortlisted problems solutions 33
Proof. Throughout this proof, all moves are supposed to be of type piq.
Indu
t on k ; the base
ase k “ 0 is trivial, sin
e no moves are possible. Assume now that
k ě 1. Fix some
anoni
al pro
ess,
onsisting of k moves M1 , M2 , . . . , Mk , and rea
hing the
nal arrangement A. Consider any sample pro
ess m1 , m2 , . . . starting with the same initial
arrangement and pro
eeding as long as possible;
learly, it
ontains at least one move. We need
to show that this pro
ess stops at A.
Let move m1
onsist in repla
ing two
opies of x with x ` a and x ` b. If move M1 does
the same, we may apply the indu
tion hypothesis to the arrangement appearing after m1 .
Otherwise, the
anoni
al pro
ess should still
ontain at least one move
onsisting in repla
ing
px, xq ÞÑ px ` a, x ` bq, be
ause the initial arrangement
ontains at least two
opies of x, while
the nal one
ontains at most one su
h.
Let Mi be the rst su
h move. Sin
e the
opies of x are indistinguishable and no other
opy
of x disappeared before Mi in the
anoni
al pro
ess, the moves in this pro
ess
an be permuted
as Mi , M1 , . . . , Mi´1 , Mi`1 , . . . , Mk , without ae
ting the nal arrangement. Now it su
es to
perform the move m1 “ Mi and apply the indu
tion hypothesis as above. l
Claim 2. Consider any pro
ess starting from the empty board, whi
h involved exa
tly n moves
of type piiq and led to a nal arrangement where all the numbers are distin
t. Assume that
one starts with the board
ontaining 2n zeroes (as if n moves of type piiq were made in the
beginning), applying type piq moves in an arbitrary way. Then this pro
ess will rea
h the same
nal arrangement.
Proof. Starting with the board with 2n zeros, one may indeed model the rst pro
ess mentioned
in the statement of the
laim, omitting the type piiq moves. This way, one rea
hes the same
nal arrangement. Now, Claim 1 yields that this nal arrangement will be obtained when
type piq moves are applied arbitrarily. l
Claim 2 allows now to reformulate the problem statement as follows: There exists an integer
n su
h that, starting from 2n zeroes, one may apply type piq moves indenitely.
In order to prove this, we start with an obvious indu
tion on s ` t “ k ě 1 to show that if
we start with 2s`t zeros, then we
an get simultaneously on the board, at some point, ea
h of
the numbers sa ` tb, with s ` t “ k .
Suppose now that a ă b. Then, an appropriate use of separate groups of zeros allows us to
get two
opies of ea
h of the numbers sa ` tb, with 1 ď s, t ď b.
Dene N “ ab´a´b, and noti
e that after representing ea
h of numbers N `k , 1 ď k ď b, in
the form sa`tb, 1 ď s, t ď b we
an get, using enough zeros, the numbers N `1, N `2, . . . , N `a
and the numbers N ` 1, N ` 2, . . . , N ` b.
From now on we
an perform only moves of type piq. Indeed, if n ě N , the o
urren
e of the
numbers n ` 1, n ` 2, . . . , n ` a and n ` 1, n ` 2, . . . , n ` b and the repla
ement pn ` 1, n ` 1q ÞÑ
pn ` b ` 1, n ` a ` 1q leads to the o
urren
e of the numbers n ` 2, n ` 3, . . . , n ` a ` 1 and
n ` 2, n ` 3, . . . , n ` b ` 1.
Comment. The proofs of Claims 1 and 2 may be extended in order to show that in fa
t the number
of moves in the
anoni
al pro
ess is the same as in an arbitrary sample one.
34 Cluj-Napo
a Romania, 314 July 2018
C7. Consider 2018 pairwise
rossing
ir
les no three of whi
h are
on
urrent. These
ir
les
subdivide the plane into regions bounded by
ir
ular edges that meet at verti
es. Noti
e that
there are an even number of verti
es on ea
h
ir
le. Given the
ir
le, alternately
olour the
verti
es on that
ir
le red and blue. In doing so for ea
h
ir
le, every vertex is
oloured twi
e
on
e for ea
h of the two
ir
les that
ross at that point. If the two
olourings agree at a vertex,
then it is assigned that
olour; otherwise, it be
omes yellow. Show that, if some
ir
le
ontains
at least 2061 yellow points, then the verti
es of some region are all yellow.
(India)
Solution 1. Letting n “ 2018, we will show that,? if every region has at least one non-yellow
vertex, then every
ir
le
ontains at most n ` t n ´ 2u ´ 2 yellow points. In the
ase at hand,
the latter equals 2018 ` 44 ´ 2 “ 2060,
ontradi
ting the hypothesis.
Consider the natural geometri
graph G asso
iated with the
onguration of n
ir
les. Fix
any
ir
le C in the
onguration, let k be the number of yellow points on C , and nd a suitable
lower bound for the total number of yellow verti
es of G in terms of k and n. It turns out that
k is even, and G has at least
ˆ ˙ ˆ ˙
k{2 n ´ k{2 ´ 1 k2
k`2 `2 “ ´ pn ´ 2qk ` pn ´ 2qpn ´ 1q p˚q
2 2 2
yellow verti
es. The proof hinges on the two lemmata below.
Lemma 1. Let two
ir
les in the
onguration
ross at x and y . Then x and y are either both
yellow or both non-yellow.
Proof. This is be
ause the numbers of interior verti
es on the four ar
s x and y determine on
the two
ir
les have like parities. l
In parti
ular, ea
h
ir
le in the
onguration
ontains an even number of yellow verti
es.
Lemma 2. If xŇy , yŇz , and zŇx are
ir
ular ar
s of three pairwise distin
t
ir
les in the
ongu-
ration, then the number of yellow verti
es in the set tx, y, zu is odd.
Proof. Let C1 , C2 , C3 be the three
ir
les under
onsideration. Assume, without loss of gen-
erality, that C2 and C3
ross at x, C3 and C1
ross at y , and C1 and C2
ross at z . Let k1 ,
k2 , k3 be the numbers of interior verti
es on the three
ir
ular ar
s under
onsideration. Sin
e
ea
h
ir
le in the
onguration, dierent from the Ci ,
rosses the
y
le x x at an even
Ňy Y yŇz Y zŇ
number of points (re
all that no three
ir
les are
on
urrent), and self-
rossings are
ounted
twi
e, the sum k1 ` k2 ` k3 is even.
Let Z1 be the
olour z gets from C1 and dene the other
olours similarly. By the pre
eding,
the number of bi
hromati
pairs in the list pZ1 , Y1 q, pX2 , Z2 q, pY3 , X3 q is odd. Sin
e the total
number of
olour
hanges in a
y
le Z1 Y1 Y3 X3 X2 Z2 Z1 is even, the number of bi
hromati
pairs in the list pX2 , X3 q, pY1 , Y3 q, pZ1 , Z2q is odd, and the lemma follows. l
We are now in a position to prove that p˚q bounds the total number of yellow verti
es from
below. Refer to Lemma 1 to infer that the k yellow verti
es on C pair o to form the pairs of
points where C is
rossed by k{2`
ir
les ˘ in the
onguration. By Lemma 2, these
ir
les
ross
pairwise to a
ount for another 2 k{2 2
yellow verti
es. Finally, the remaining n ´ k{2 ´ 1
ir
les
in the
onguration
ross C at non-yellow verti
es, by Lemma 1, and` Lemma ˘ 2 applies again
to show that these
ir
les
ross pairwise to a
ount for yet another 2 n´k{2´1
2
yellow verti
es.
Consequently, there are at least p˚q yellow verti
es.
Next, noti
e that G is a plane graph on npn ´ 1q degree 4 verti
es, having exa
tly 2npn ´ 1q
edges and exa
tly npn ´ 1q ` 2 fa
es (regions), the outer fa
e in
lusive (by Euler's formula for
planar graphs).
Lemma 3. Ea
h fa
e of G has equally many red and blue verti
es. In parti
ular, ea
h fa
e has
an even number of non-yellow verti
es.
Shortlisted problems solutions 35
Proof. Tra
e the boundary of a fa
e on
e in
ir
ular order, and
onsider the
olours ea
h vertex
is assigned in the
olouring of the two
ir
les that
ross at that vertex, to infer that
olours of
non-yellow verti
es alternate. l
Consequently, if ea
h region has at least one non-yellow vertex, then it has at least two su
h.
Sin
e ea
h vertex of G has degree 4,
onsideration of vertex-fa
e in
iden
es shows that G has
at least npn ´ 1q{2 ` 1 non-yellow verti
es, and hen
e at most npn ´ 1q{2 ´ 1 yellow verti
es. (In
fa
t, Lemma 3 shows that there are at least npn ´ 1q{4 ` 1{2 red, respe
tively blue, verti
es.)
Finally, re
all the lower bound p˚q for the total number of yellow verti
es?in G, to write
npn ´ 1q{2 ´ 1 ě k 2 {2 ´ pn ´ 2qk ` pn ´ 2qpn ´ 1q, and
on
lude that k ď n ` t n ´ 2u ´ 2, as
laimed in the rst paragraph.
Solution 2. The rst two lemmata in Solution 1 show that the
ir
les in the
onguration
split into two
lasses: Consider any
ir
le C along with all
ir
les that
ross C at yellow points
to form one
lass; the remaining
ir
les then form the other
lass. Lemma 2 shows that any pair
of
ir
les in the same
lass
ross at yellow points; otherwise, they
ross at non-yellow points.
Call the
ir
les from the two
lasses white and bla
k, respe
tively. Call a region yellow if
its verti
es are all yellow. Let w and b be the numbers of white and bla
k
ir
les, respe
tively;
learly, w ` b “ n. Assume that w ě b, and that there is no yellow region. Clearly, b ě 1,
otherwise ea
h region is yellow. The white
ir
les subdivide the plane into wpw ´ 1q ` 2 larger
regions
all them white. The white regions (or rather their boundaries) subdivide ea
h bla
k
ir
le into bla
k ar
s. Sin
e there are no yellow regions, ea
h white region
ontains at least one
bla
k ar
.
Consider any white region; let it
ontain t ě 1 bla
k ar
s. We
laim that the number of
points at whi
h these t ar
s
ross does not ex
eed t ´ 1. To prove this,
onsider a multigraph
whose verti
es are these bla
k ar
s, two verti
es being joined by an edge for ea
h point at whi
h
the
orresponding ar
s
ross. If this graph had more than t ´ 1 edges, it would
ontain a
y
le,
sin
e it has t verti
es; this
y
le would
orrespond to a
losed
ontour formed by bla
k sub-ar
s,
lying inside the region under
onsideration. This
ontour would, in turn, dene at least one
yellow region, whi
h is impossible.
Let řti be the number of bla
k ar
s `inside
˘ the ith white region. The total number of bla
k
ar
s is i ti “ 2wb, and they
ross at 2 2b “ bpb ´ 1q points. By the pre
eding,
w2 ÿ
´w`2 w2 ÿ
´w`2
bpb ´ 1q ď pti ´ 1q “ ti ´ pw 2 ´ w ` 2q “ 2wb ´ pw 2 ´ w ` 2q,
i“1 i“1
?
or, equivalently, pw ´ bq2`ď w `?b ´ 2 “˘n ´ 2, whi
h is the
ase if and only if w ´ b ?
ď t n ´ 2u.
Consequently, b ď w ď n ` t n ´ 2u {2, so there are at most 2pw ´ 1q ď n ` t n ´ 2u ´ 2
yellow verti
es on ea
h
ir
le a
ontradi
tion.
36 Cluj-Napo
a Romania, 314 July 2018
Geometry
G1. Let ABC be an a
ute-angled triangle with
ir
um
ir
le Γ. Let D and E be points on
the segments AB and AC , respe
tively, su
h that AD “ AE . The perpendi
ular bise
tors of
the segments BD and CE interse
t the small ar
s AB
Ŋ and AC
Ŋ at points F and G respe
tively.
Prove that DE k F G.
(Gree
e)
Solution 1. In the sequel, all the
onsidered ar
s are small ar
s.
Let P be the midpoint of the ar
BCŊ . Then AP is the bise
tor of =BAC , hen
e, in the
isos
eles triangle ADE , AP K DE . So, the statement of the problem is equivalent to AP K F G.
In order to prove this, let K be the se
ond interse
tion of Γ with F D . Then the triangle
F BD is isos
eles, therefore
=AKF “ =ABF “ =F DB “ =ADK,
yielding AK “ AD . In the same way, denoting by L the se
ond interse
tion of Γ with GE , we
get AL “ AE . This shows that AK “ AL.
A
K
L
E
D
G
F
B C
P
Now =F BD “ =F DB gives AF Ŋ “ BF
Ŋ ` AK
Ŋ “ BF Ň , hen
e BF
Ŋ ` AL Ň . In a similar
Ŋ “ LF
way, we get CG
Ŋ “ GKŊ . This yields
Ŋ`P
AF ŊG Ň ` LF
AL Ň `P Ŋ Ŋ
C ` CG Ŋ ` LB
KL Ŋ ` BCŊ ` CK
Ŋ
=pAP, F Gq “ “ “ “ 90˝ .
2 2 4
E T
A D Z
B
X
F
Shortlisted problems solutions 37
Solution 3. As in the rst solution, we prove that F G K AP , where P is the midpoint of the
small ar
BC
Ŋ.
Let O be the
ir
um
entre of the triangle ABC , and let M and N be the midpoints of the
small ar
s AB
Ŋ and ACŊ , respe
tively. Then OM and ON are the perpendi
ular bise
tors of AB
and AC , respe
tively.
A
M
O E d
d G
D
F
B C
P
G2. Let ABC be a triangle with AB “ AC , and let M be the midpoint of BC . Let P be
a point su
h that P B ă P C and P A is parallel to BC . Let X and Y be points on the lines
P B and P C , respe
tively, so that B lies on the segment P X , C lies on the segment P Y , and
=P XM “ =P Y M . Prove that the quadrilateral AP XY is
y
li
.
(Australia)
Solution. Sin
e AB “ AC , AM is the perpendi
ular bise
tor of BC , hen
e =P AM “
=AMC “ 90˝ .
P A
M C
B
X Z
Now let Z be the
ommon point of AM and the perpendi
ular through Y to P C (noti
e
that Z lies on to the ray AM beyond M ). We have =P AZ “ =P Y Z “ 90˝ . Thus the points
P , A, Y , and Z are
on
y
li
.
Sin
e =CMZ “ =CY Z “ 90˝ , the quadrilateral CY ZM is
y
li
, hen
e =CZM “
=CY M . By the
ondition in the statement, =CY M “ =BXM , and, by symmetry in ZM ,
=CZM “ =BZM . Therefore, =BXM “ =BZM . It follows that the points B , X , Z , and M
are
on
y
li
, hen
e =BXZ “ 180˝ ´ =BMZ “ 90˝ .
Finally, we have =P XZ “ =P Y Z “ =P AZ “ 90˝ , hen
e the ve points P, A, X, Y, Z are
on
y
li
. In parti
ular, the quadrilateral AP XY is
y
li
, as required.
Comment 1. Clearly, the key point Z from the solution above
an be introdu
ed in several dierent
ways, e.g., as the se
ond meeting point of the
ir
le CM Y and the line AM , or as the se
ond meeting
point of the
ir
les CM Y and BM X , et
.
For some of denitions of Z its lo
ation is not obvious. For instan
e, if Z is dened as a
ommon
point of AM and the perpendi
ular through X to P X , it is not
lear that Z lies on the ray AM
beyond M . To avoid su
h slippery details some more restri
tions on the
onstru
tion may be required.
Comment 2. Let us dis
uss a
onne
tion to the Miquel point of a
y
li
quadrilateral. Set X1 “
1
M X X P C , Y “ M Y X P B , and Q “ XY X X 1 Y 1 (see the gure below).
We
laim that BC k P Q. (One way of proving this is the following. Noti
e that the quadruple
of lines P X, P M, P Y, P Q is harmoni
, hen
e the quadruple B , M , C , P Q X BC of their interse
tion
points with BC is harmoni
. Sin
e M is the midpoint of BC , P Q X BC is an ideal point, i.e.,
P Q k BC .)
1 1
It follows from the given equality =P XM “ =P Y M that the quadrilateral XY X Y is
y
li
.
Note that A is the proje
tion of M onto P Q. By a known des
ription, A is the Miquel point for the
1 1 1 1
sidelines XY, XY , X Y, X Y . In parti
ular, the
ir
le P XY passes through A.
Shortlisted problems solutions 39
A
Y’
B
X’
M
X
C
Y
Comment 3. An alternative approa
h is the following. One
an note that the (oriented) lengths of
the segments CY and BX are both linear fun
tions of a parameter t “ cot =P XM . As t varies, the
interse
tion point S of the perpendi
ular bise
tors of P X and P Y tra
es a xed line, thus the family
of
ir
les P XY has a xed
ommon point (other than P ). By
he
king parti
ular
ases, one
an show
that this xed point is A.
G3. A
ir
le ω of radius 1 is given. A
olle
tion T of triangles is
alled good, if the following
onditions hold:
Determine all positive real numbers t su
h that, for ea
h positive integer n, there exists a
good
olle
tion of n triangles, ea
h of perimeter greater than t.
(South Afri
a)
Answer: t P p0, 4s.
Solution. First, we show how to
onstru
t a good
olle
tion of n triangles, ea
h of perimeter
greater than 4. This will show that all t ď 4 satisfy the required
onditions.
Constru
t indu
tively an pn ` 2q-gon BA1 A2 . . . An C ins
ribed in ω su
h that BC is a
diameter, and BA1 A2 , BA2 A3 , . . . , BAn´1 An , BAn C is a good
olle
tion of n triangles. For
n “ 1, take any triangle BA1 C ins
ribed in ω su
h that BC is a diameter; its perimeter is greater
than 2BC “ 4. To perform the indu
tive step, assume that the pn ` 2q-gon BA1 A2 . . . An C is
already
onstru
ted. Sin
e An B ` An C ` BC ą 4, one
an
hoose a point An`1 on the small
ar
CA
Őn ,
lose enough to C , so that An B ` An An`1 ` BAn`1 is still greater than 4. Thus ea
h
of these new triangles BAn An`1 and BAn`1 C has perimeter greater than 4, whi
h
ompletes
the indu
tion step.
A1
A2
A3
C B
We pro
eed by showing that no t ą 4 satises the
onditions of the problem. To this end,
we assume that there exists a good
olle
tion T of n triangles, ea
h of perimeter greater than t,
and then bound n from above.
Take ε ą 0 su
h that t “ 4 ` 2ε.
Claim. There exists a positive
onstant σ “ σpεq su
h that any triangle ∆ with perimeter
2s ě 4 ` 2ε, ins
ribed in ω , has area Sp∆q at least σ .
Proof. Let a, b, c be the side lengths of ∆. Sin
e ∆ is ins
ribed in ω , ea
h side has length at
most 2. Therefore, as ´ a ě p2 ` εq ´ 2 “ ε.aSimilarly, s ´ b ě ε and s ´ c ě ε. a By Heron's
formula, Sp∆q “ sps ´ aqps ´ bqps ´ cq ě p2 ` εqε . Thus we
an set σpεq “ p2 ` εqε3 .
3
l
Now we see that the total area S of all triangles from T is at least nσpεq. On the other
hand, S does not ex
eed the area of the disk bounded by ω . Thus nσpεq ď π , whi
h means
that n is bounded from above.
abc
Comment 1. One may prove the Claim using the formula S“ instead of Heron's formula.
4R
Comment 2. In the statement of the problem
ondition piq
ould be repla
ed by a weaker one: ea
h
triangle from T lies within ω. This does not ae
t the solution above, but redu
es the number of ways
to prove the Claim.
Shortlisted problems solutions 41
G4. A point T is
hosen inside a triangle ABC . Let A1 , B1 , and C1 be the ree
tions
of T in BC , CA, and AB , respe
tively. Let Ω be the
ir
um
ir
le of the triangle A1 B1 C1 .
The lines A1 T , B1 T , and C1 T meet Ω again at A2 , B2 , and C2 , respe
tively. Prove that the
lines AA2 , BB2 , and CC2 are
on
urrent on Ω.
(Mongolia)
Solution. By ?pℓ, nq we always mean the dire
ted angle of the lines ℓ and n, taken modulo 180˝ .
Let CC2 meet Ω again at K (as usual, if CC2 is tangent to Ω, we set T “ C2 ). We show
that the line BB2
ontains K ; similarly, AA2 will also pass through K . For this purpose, it
su
es to prove that
?pC2 C, C2 A1 q “ ?pB2 B, B2 A1 q. (1)
By the problem
ondition, CB and CA are the perpendi
ular bise
tors of T A1 and T B1 ,
respe
tively. Hen
e, C is the
ir
um
entre of the triangle A1 T B1 . Therefore,
Similarly, we get
?pBA1 , BCq “ ?pC1 A1 , C1 C2 q “ ?pB2 A1 , B2 C2 q. (3)
The two obtained relations yield that the triangles A1 BC and A1 B2 C2 are similar and
equioriented, hen
e
A1 B2 A1 C2
“ and ?pA1 B, A1 Cq “ ?pA1 B2 , A1 C2 q.
A1 B A1 C
A
A
Ω K
B1
TT
B22222
B
B
C
C2
A11111
A
Comment 2. After obtaining (2) and (3), one an nish the solution in dierent ways.
For instan
e, introdu
ing the point X “ BC XB2 C2 , one gets from these relations that the 4-tuples
pA1 , B, B2 , Xq and pA1 , C, C2 , Xq are both
y
li
. Therefore, K is the Miquel point of the lines BB2 ,
CC2 , BC , and B2 C2 ; this yields that the meeting point of BB2 and CC2 lies on Ω.
Yet another way is to show that the points A1 , B , C , and K are
on
y
li
, as
By symmetry, the se
ond point K1 of interse
tion of BB2 with Ω is also
on
y
li
to A1 , B , and C,
hen
e K
1 “ K.
C1 A2
A
A
Ω K
C′′′′′
C
C
C
B1
TT
B′′′′′
B
B
B
B22222
B
B A′
B X
X C
C2
A
A11111
Comment 3. The requirement that the
ommon point of the lines AA2 , BB2 , and CC2 should lie
on Ω may seem to make the problem easier, sin
e it suggests some approa
hes. On the other hand,
there are also dierent ways of showing that the lines AA2 , BB2 , and CC2 are just
on
urrent.
A2 T , B2 T , and C2 T are perpendi
ular to
In parti
ular, the problem
onditions yield that the lines
the
orresponding sides of the triangle ABC . One may show that the lines AT , BT , and CT are also
perpendi
ular to the
orresponding sides of the triangle A2 B2 C2 , i.e., the triangles ABC and A2 B2 C2
are orthologi
, and their orthology
entres
oin
ide. It is known that su
h triangles are also perspe
tive,
i.e. the lines AA2 , BB2 , and CC2 are
on
urrent (in proje
tive sense).
To show this mutual orthology, one may again apply angle
hasing, but there are also other methods.
Let A1 , B 1 , and C 1 be the proje
tions of T onto the sides of the triangle ABC . Then A2 T ¨ T A1 “
B2 T ¨ T B 1 “ C2 T ¨ T C 1 , sin
e all three produ
ts equal (minus) half the power of T with respe
t to Ω.
This means that A2 , B2 , and C2 are the poles of the sidelines of the triangle ABC with respe
t to
some
ir
le
entred at T and having pure imaginary radius (in other words, the ree
tions of A2 , B2 ,
and C2 in T are the poles of those sidelines with respe
t to some regular
ir
le
entred at T ). Hen
e,
dually, the verti
es of the triangle ABC are also the poles of the sidelines of the triangle A2 B2 C2 .
44 Cluj-Napo
a Romania, 314 July 2018
G5. Let ABC be a triangle with
ir
um
ir
le ω and in
entre I . A line ℓ interse
ts the
lines AI , BI , and CI at points D , E , and F , respe
tively, distin
t from the points A, B , C ,
and I . The perpendi
ular bise
tors x, y , and z of the segments AD , BE , and CF , respe
tively
determine a triangle Θ. Show that the
ir
um
ir
le of the triangle Θ is tangent to ω .
(Denmark)
l
Z0
Z I
x F w
D lc W
B C
E
Dc
lb y X0
z
Db
X
Proof. Noti
e that ?pℓb , ℓcq “ ?pℓb , ℓq ` ?pℓ, ℓc q “ 2?py, ℓq ` 2?pℓ, zq “ 2?py, zq. But y K BI
and z K CI implies ?py, zq “ ?pBI, ICq, so, sin
e 2?pBI, ICq “ ?pBA, ACq, we obtain
Now we prove that X , X0 , T are
ollinear. Denote by Db and Dc the ree
tions of the point
D in the lines y and z , respe
tively. Then Db lies on ℓb , Dc lies on ℓc , and
?pDb X, XDc q “ ?pDb X, DXq ` ?pDX, XDc q “ 2?py, DXq ` 2?pDX, zq “ 2?py, zq
“ ?pBA, ACq “ ?pBT, T Cq,
hen
e the quadrilateral XDb T Dc is
y
li
. Noti
e also that sin
e XDb “ XD “ XDc , the
points D, Db , Dc lie on a
ir
le with
entre X . Using in this
ir
le the diameter Dc Dc1 yields
?pDb Dc , Dc Xq “ 90˝ ` ?pDb Dc1 , Dc1 Xq “ 90˝ ` ?pDb D, DDc q. Therefore,
Comment 1. After proving Claim 1 one may pro
eed in another way. As it was shown, the ree
tions
of ℓ in the sidelines of XY Z T . Thus ℓ is the Steiner line of T with respe
t to ∆XY Z
are
on
urrent at
(that is the line
ontaining the ree
tions Ta , Tb , Tc of T in the sidelines of XY Z ). The properties of
the Steiner line imply that T lies on Ω, and ℓ passes through the ortho
entre H of the triangle XY Z .
T
A
Ha la lc
lb
Y
l
Tc
x I Ta
Z
w W
H F
B D
C
E
Hb Hc
y z
Tb
X
LetHa , Hb , and Hc be the ree
tions of the point H in the lines x, y , and z , respe
tively. Then
the triangle Ha Hb Hc is ins
ribed in Ω and homotheti
to ABC (by an easy angle
hasing). Sin
e
Ha P ℓa , Hb P ℓb , and Hc P ℓc , the triangles Ha Hb Hc and ABC form a required pair of triangles ∆ and
δ mentioned in the preamble.
Comment 2. The following observation shows how one may guess the des
ription of the tangen
y
point T from Solution 1.
Let us x a dire
tion and move the line ℓ parallel to this dire
tion with
onstant speed.
Then the points D , E , and F are moving with
onstant speeds along the lines AI , BI , and CI ,
respe
tively. In this
ase x, y , and z are moving with
onstant speeds, dening a family of homotheti
triangles XY Z with a
ommon
entre of homothety T . Noti
e that the triangle X0 Y0 Z0 belongs to
this family (for ℓ passing through I ). We may spe
ify the lo
ation of T
onsidering the degenerate
ase when x, y , and z are
on
urrent. In this degenerate
ase all the lines x, y , z , ℓ, ℓa , ℓb , ℓc have a
ommon point. Note that the lines ℓa , ℓb , ℓc remain
onstant as ℓ is moving (keeping its dire
tion).
Thus T should be the
ommon point of ℓa , ℓb , and ℓc , lying on ω .
46 Cluj-Napo
a Romania, 314 July 2018
Solution 2. As mentioned in the preamble, it is su
ient to prove that the
entre T of the
homothety taking XY Z to X0 Y0 Z0 belongs to ω . Thus, it su
es to prove that ?pT X0 , T Y0 q “
?pZ0 X0 , Z0 Y0 q, or, equivalently, ?pXX0 , Y Y0 q “ ?pZ0 X0 , Z0 Y0 q.
Re
all that Y Z and Y0 Z0 are the perpendi
ular bise
tors of AD and AI , respe
tively. Then,
ÝÑ
the ve
tor ÑÝ
x perpendi
ular to Y Z and shifting the line Y0 Z0 to Y Z is equal to 12 ID. Dene
ÝÑ Ý ÝÑ
the shifting ve
tors Ñ Ýy “ 21 IE , Ñ
z “ 12 IF similarly. Consider now the triangle UV W formed by
the perpendi
ulars to AI , BI , and CI through D , E , and F , respe
tively (see gure below).
This is another triangle whose sides are parallel to the
orresponding sides of XY Z .
Claim 2. Ý Ñ ÝÝÝÑ ÝÑ ÝÝÑ ÝÝÑ
IU “ 2X0 X , IV “ 2Y0 Y , IW “ 2Z0 Z .
ÝÝÑ
Proof. We prove one of the relations, the other proofs being similar. To prove the equality of two
ve
tors it su
es to proje
t them onto two non-parallel axes and
he
k that their proje
tions
are equal.
ÝÝÝÑ ÝÑ ÝÑ
The proje
tion of X0 X onto IB equals ~y , while the proje
tion of IU onto IB is IE “ 2~y .
ÝÑ ÝÑ ÝÝÝÑ
The proje
tions onto the other axis IC are ~z and IF “ 2~z. Then IU “ 2X0 X follows. l
Noti
e that the line ℓ is the Simson line of the point I with respe
t to the triangle UV W ;
thus U , V , W , and I are
on
y
li
. It follows from Claim 2 that ?pXX0 , Y Y0 q “ ?pIU, IV q “
?pW U, W V q “ ?pZ0 X0 , Z0 Y0 q, and we are done.
V
Y
l
Ib
u
x F
W Y0
w
C z
A
D w
T I
X0
Ia X
Z0 B U
y
v
Ic Z E W
Consider now the homothety or translation h1 that maps XY ZT to Ia Ib Ic S and the homo-
thety h2 with
entre I and fa
tor 12 . Furthermore, let h “ h2 ˝ h1 . The transform h
an be a
homothety or a translation, and
h pT q “ h2 ph1 pT qq “ h2 pSq “ T,
hen
e T is a xed point of h. So, h is a homothety with
entre T . Note that h2 maps the
ex
entres Ia , Ib , Ic to X0 , Y0 , Z0 dened in the preamble. Thus the
entre T of the homothety
taking XY Z to X0 Y0 Z0 belongs to Ω, and this
ompletes the proof.
48 Cluj-Napo
a Romania, 314 July 2018
B’
E
C
B
A
D
Lemma. Let P QR be a triangle, and let X be a point in the interior of the angle QP R su
h that
P X ¨ QR
=QP X “ =P RX . Then ă 1 if and only if X lies in the interior of the triangle P QR.
PQ ¨ PR
Proof. The lo
us of points X with =QP X “ =P RX lying inside the angle QP R is an ar
α
of the
ir
le γ through R tangent to P Q at P . Let γ interse
t the line QR again at Y (if γ
PQ ¨ PR
is tangent to QR, then set Y “ R). The similarity △QP Y „ △QRP yields P Y “ .
QR
Now it su
es to show that P X ă P Y if and only if X lies in the interior of the triangle P QR.
Let m be a line through Y parallel to P Q. Noti
e that the points Z of γ satisfying P Z ă P Y
are exa
tly those between the lines m and P Q.
Case 1: Y lies in the segment QR (see the left gure below).
In this
ase Y splits α into two ar
s P
Ŋ Y and Y
Ŋ R. The ar
P
Ŋ Y lies inside the triangle P QR,
and P Y lies between m and P Q, hen
e P X ă P Y for points X P P
Ŋ Ŋ Y . The other ar
Y ŊR
lies outside triangle P QR, and Y R is on the opposite side of m than P , hen
e P X ą P Y for
Ŋ
X PY ŊR.
Shortlisted problems solutions 49
Case 2: Y lies on the ray QR beyond R (see the right gure below).
In this
ase the whole ar
α lies inside triangle P QR, and between m and P Q, thus P X ă
P Y for all X P α. l
P P
R
X
Q
Y R
Q
X Y
Applying the Lemma (to △EAD with the point X , and to △ECD with the point X ),
XA ¨ DE XC ¨ DE
we obtain that exa
tly one of two expressions and is less than 1, whi
h
AD ¨ AE CD ¨ CE
ontradi
ts (˚).
Solution 2. The solution
onsists of two parts. In Part 1 we show that it su
es to prove
that
XB AB
“ p1q
XD CD
and
XA DA
“ . p2q
XC BC
In Part 2 we establish these equalities.
Part 1. Using the sine law and applying (1) we obtain
X X
A 7→
A′
C′
C
D
D′
Now we need the following Lemma.
Lemma. Assume that the
orresponding angles of
onvex quadrilaterals XY ZT and X 1 Y 1 Z 1 T 1
are equal, and that XY ¨ ZT “ Y Z ¨ T X and X 1 Y 1 ¨ Z 1 T 1 “ Y 1 Z 1 ¨ T 1 X 1 . Then the two
quadrilaterals are similar.
Proof. Take the quadrilateral XY Z1 T1 similar to X 1 Y 1 Z 1 T 1 and sharing the side XY with
XY ZT , su
h that Z1 and T1 lie on the rays Y Z and XT , respe
tively, and Z1 T1 k ZT . We
need to prove that Z1 “ Z and T1 “ T . Assume the
ontrary. Without loss of generality,
T X ą XT1 . Let segments XZ and Z1 T1 interse
t at U . We have
T1 X T1 X TX XY XY
ă “ “ ă ,
T1 Z1 T1 U ZT YZ Y Z1
thus T1 X ¨ Y Z1 ă T1 Z1 ¨ XY . A
ontradi
tion. l
X Y
Z1
T1 U
Z
T
It follows from the Lemma that the quadrilaterals ABCD and D 1 A1 B 1 C 1 are similar, hen
e
BC A1 B 1 AB XD ¨ XA AB XD
“ 1 1 “ ¨ “ ¨ ,
AB DA XA ¨ XB DA AD XB
and therefore
XB AB 2 AB 2 AB
“ “ “ .
XD BC ¨ AD AB ¨ CD CD
We obtain (1), as desired; (2) is proved similarly.
Comment. Part 1 is an easy one, while part 2 seems to be
ru
ial. On the other hand, after the
proof of the similarityD 1 A1 B 1 C 1 „ ABCD one may nish the solution in dierent ways, e.g., as
follows. The similarity taking D A B C to ABCD maps X to the point X isogonally
onjugate
1 1 1 1 1
1
of X with respe
t to ABCD (i.e. to the point X inside ABCD su
h that =BAX “ =DAX ,
1
1 1 1
=CBX “ =ABX , =DCX “ =BCX , =ADX “ =CDX ). It is known that the required equality
=AXB ` =CXD “ 180˝ is one of known
onditions on a point X inside ABCD equivalent to the
existen
e of its isogonal
onjugate.
Shortlisted problems solutions 51
L
L
LCC
C
C
C
ωC
Ω
C
C
O
O
OAA
A
A
A ωB
O
L
L
LAA
A
A
A O
O
OCC
C
C
C
B O
O
OBB
B
B
B
P
P
P
P
LB
Similarly, the points P , LA , A are
ollinear, and the points P , LB , B are also
ollinear.
Finally, the
omputation above also shows that
Comment 1. The proof of Claim 2 may be repla
ed by the following remark: sin
e P belongs to the
ir
les ωA and ωC , P is the Miquel point of the four lines ℓA , ℓB , ℓC , and OA OB OC .
Comment 2. Claims 2 and 3
an be proved in several dierent ways and, in parti
ular, in the reverse
order.
Claim 3 implies that the triangles ABC and LA LB LC are perspe
tive with perspe
tor P. Claim 2
an be derived from this observation using spiral similarity. Consider the
entre Q of the spiral similarity
that maps ABC to LA LB LC . From known spiral similarity properties, the points LA , LB , P, Q are
on
y
li
, and so are LA , LC , P, Q.
Comment 3. The nal
on
lusion
an also be proved it terms of spiral similarity: the spiral similarity
with
entre Q lo
ated on the
ir
le ABC maps the
ir
le ABC to the
ir
le P LA LB LC . Thus these
ir
les are orthogonal.
Reformulation. Let ω be the in
ir
le, and let I be the in
entre of a triangle ABC . Let P be
a point of ω ω with the sides of ABC ). The tangent to ω at P
(other than the points of
onta
t of
meets the lines AB , BC , and CA at A1 , B 1 , and C 1 , respe
tively. Line ℓA parallel to the internal
1
angle bise
tor of =BAC passes through A ; dene lines ℓB and ℓC similarly. Prove that the line IP is
tangent to the
ir
um
ir
le of the triangle formed by ℓA , ℓB , and ℓC .
Though this formulation is equivalent to the original one, it seems more
hallenging, sin
e the point
of
onta
t is now hidden.
54 Cluj-Napo
a Romania, 314 July 2018
Number Theory
N1. Determine all pairs pn, kq of distin
t positive integers su
h that there exists a positive
integer s for whi
h the numbers of divisors of sn and of sk are equal.
(Ukraine)
Answer: All pairs pn, kq su
h that n ∤ k and k ∤ n.
Solution. As usual, the number of divisors of a positive integer n is denoted by dpnq. If
ś ś
n “ i pαi i is the prime fa
torisation of n, then dpnq “ i pαi ` 1q.
We start by showing that one
annot nd any suitable number s if k | n or n | k (and
k ‰ n). Suppose that n | k , and
hoose any positive integer s. Then the set of divisors of sn is
a proper subset of that of sk , hen
e dpsnq ă dpskq. Therefore, the pair pn, kq does not satisfy
the problem requirements. The
ase k | n is similar.
Now assume that n ∤ k and k ∤ n. Let p1 , . . . , pt be all primes dividing nk , and
onsider the
prime fa
torisations
źt ź t
n“ pi and k “
αi
pβi i .
i“1 i“1
First of all, if αi “ βi for some i, then, regardless of the value of γi , the
orresponding fa
tor
in (1) equals 1 and does not ae
t the produ
t. So we may assume that there is no su
h index i.
For the other fa
tors in (1), the following lemma is useful.
Lemma. Let α ą β be nonnegative integers. Then, for every integer M ě β ` 1, there exists a
nonnegative integer γ su
h that
α`γ`1 1 M `1
“1` “ .
β`γ`1 M M
Proof.
α`γ`1 1 α´β 1
“1` ðñ “ ðñ γ “ Mpα ´ βq ´ pβ ` 1q ě 0. l
β`γ`1 M β`γ`1 M
Now we
an nish the solution. Without loss of generality, there exists an index u su
h that
αi ą βi for i “ 1, 2, . . . , u, and αi ă βi for i “ u ` 1, . . . , t. The
onditions n ∤ k and k ∤ n mean
that 1 ď u ď t ´ 1.
Choose an integer X greater than all the αi and βi . By the lemma, we
an dene the
numbers γi so as to satisfy
αi ` γ i ` 1 uX ` i
“ for i “ 1, 2, . . . , u, and
βi ` γi ` 1 uX ` i ´ 1
βu`i ` γu`i ` 1 pt ´ uqX ` i
“ for i “ 1, 2, . . . , t ´ u.
αu`i ` γu`i ` 1 pt ´ uqX ` i ´ 1
Shortlisted problems solutions 55
as required.
Comment. The lemma
an be used in various ways, in order to provide a suitable value of s. In
parti
ular, one may apply indu
tion on the number t of prime fa
tors, using identities like
n n2 n`1
“ 2 ¨ .
n´1 n ´1 n
56 Cluj-Napo
a Romania, 314 July 2018
piiq The sum of numbers in any row, as well as the sum of numbers in any
olumn, is
ongruent
to n modulo n2 .
Let Ri be the produ
t of the numbers in the ith row, and Cj be the produ
t of the numbers in
the j th
olumn. Prove that the sums R1 ` ¨ ¨ ¨ ` Rn and C1 ` ¨ ¨ ¨ ` Cn are
ongruent modulo n4 .
(Indonesia)
Solution 1. Let Ai,j be the entry in the ith row and the j th
olumn; let P be the produ
t of
all n2 entries. For
onvenien
e, denote ai,j “ Ai,j ´ 1 and ri “ Ri ´ 1. We show that
n
ÿ
Ri ” pn ´ 1q ` P pmod n4 q. (1)
i“1
Due to symmetry of the problem
onditions, the sum of all the Cj is also
ongruent to pn ´ 1q`P
modulo n4 , when
e the
on
lusion.
By
ondition piq, the number n divides ai,j for all i and j . So, every produ
t of at least two
of the ai,j is divisible by n2 , hen
e
n
ź n
ÿ ÿ n
ÿ n
ÿ
Ri “ p1`ai,j q “ 1` ai,j ` ai,j1 ai,j2 `¨ ¨ ¨ ” 1` ai,j ” 1´n` Ai,j pmod n2 q
j“1 j“1 1ďj1 ăj2 ďn j“1 j“1
when
e n n
ÿ ÿ
Ri “ n ` ri ” n ` pP ´ 1q pmod n4 q,
i“1 i“1
as desired.
Comment. The original version of the problem statement ontained also the ondition
piiiq The produ t of all the numbers in the table is ongruent to 1 modulo n4 .
where the last two sums are taken over all unordered pairs/triples of pairwise dierent pairs
pi, jq; su
h
onventions are applied throughout the solution.
Similarly,
n
ÿ n ź
ÿ n ÿÿ ÿ ÿ ÿ ÿ
Ri “ p1 ` ai,j q ” n ` ai,j ` ai,j1 ai,j2 ` ai,j1 ai,j2 ai,j3 pmod n4 q.
i“1 i“1 j“1 i j i j1 , j2 i j1 , j2 , j3
Therefore,
ÿ ÿ ÿ
P ` pn ´ 1q ´ Ri ” ai1 ,j1 ai2 ,j2 ` ai1 ,j1 ai2 ,j2 ai3 ,j3
i pi1 ,j1 q, pi2 ,j2 q pi1 ,j1 q, pi2 ,j2 q, pi3 ,j3 q
i1 ‰i2 i1 ‰i2 ‰i3 ‰i1
ÿ
` ai1 ,j1 ai2 ,j2 ai3 ,j3 pmod n4 q.
pi1 ,j1 q, pi2 ,j2 q, pi3 ,j3 q
i1 ‰i2 “i3
We show that in fa
t ea
h of the three sums appearing in the right-hand part of this
ongruen
e
is divisible by n4 ; this yields (1). Denote those three sums by Σ1 , Σ2 , and Σ3 in order of
appearan
e. Re
all that by
ondition piiq we have
ÿ
ai,j ” 0 pmod n2 q for all indi
es i.
j
sin
e ea
h of the two fa
tors is divisible by n2 . Summing over all pairs pi1 , i2 q we obtain n4 | Σ1 .
Similarly, for every three indi
es i1 ă i2 ă i3 we have
ÿÿÿ ˆÿ ˙ ˆÿ ˙ ˆÿ ˙
ai1 ,j1 ai2 ,j2 ai3 ,j3 “ ai1 ,j1 ¨ ai2 ,j2 ¨ ai3 ,j3
j1 j2 j3 j1 j2 j3
sin
e the three fa
tors are divisible by n, n, and n2 , respe
tively. Summing over all 4-tuples of
indi
es pi1 , i2 , j2 , j3 q we get n4 | Σ3 .
58 Cluj-Napo
a Romania, 314 July 2018
N3. Dene the sequen
e a0 , a1 , a2 , . . . by an “ 2n ` 2tn{2u . Prove that there are innitely
many terms of the sequen
e whi
h
an be expressed as a sum of (two or more) distin
t terms
of the sequen
e, as well as innitely many of those whi
h
annot be expressed in su
h a way.
(Serbia)
Solution 1. Call a nonnegative integer representable if it equals the sum of several (possibly 0
or 1) distin
t terms of the sequen
e. We say that two nonnegative integers b and c are equivalent
(written as b „ c) if they are either both representable or both non-representable.
One
an easily
ompute
Sn´1 :“ a0 ` ¨ ¨ ¨ ` an´1 “ 2n ` 2rn{2s ` 2tn{2u ´ 3.
Indeed, we have Sn ´ Sn´1 “ 2n ` 2tn{2u “ an so we
an use the indu
tion. In parti
ular,
S2k´1 “ 22k ` 2k`1 ´ 3.
Note that, if n ě 3, then 2rn{2s ě 22 ą 3, so
Sn´1 “ 2n ` 2rn{2s ` 2tn{2u ´ 3 ą 2n ` 2tn{2u “ an .
Also noti
e that Sn´1 ´ an “ 2rn{2s ´ 3 ă an .
The main tool of the solution is the following
laim.
Claim 1. Assume that b is a positive integer su
h that Sn´1 ´ an ă b ă an for some n ě 3.
Then b „ Sn´1 ´ b.
Proof. As seen above, we have Sn´1 ą an . Denote c “ Sn´1 ´ b; then Sn´1 ´ an ă c ă an , so
the roles of b and c are symmetri
al.
Assume that b is representable. The representation
annot
ontain ai with i ě n, sin
e
b ă an . So b is the sum of some subset of ta0 , a1 , . . . , an´1 u; then c is the sum of the
omplement.
The
onverse is obtained by swapping b and c. l
We also need the following version of this
laim.
Claim 2. For any n ě 3, the number an
an be represented as a sum of two or more distin
t
terms of the sequen
e if and only if Sn´1 ´ an “ 2rn{2s ´ 3 is representable.
Proof. Denote c “ Sn´1 ´ an ă an . If an satises the required
ondition, then it is the sum
of some subset of ta0 , a1 , . . . , an´1 u; then c is the sum of the
omplement. Conversely, if c is
representable, then its representation
onsists only of the numbers from ta0 , . . . , an´1 u, so an is
the sum of the
omplement. l
By Claim 2, in order to prove the problem statement, it su
es to nd innitely many
representable numbers of the form 2t ´ 3, as well as innitely many non-representable ones.
Claim 3. For every t ě 3, we have 2t ´ 3 „ 24t´6 ´ 3, and 24t´6 ´ 3 ą 2t ´ 3.
Proof. The inequality follows from t ě 3. In order to prove the equivalen
e, we apply Claim 1
twi
e in the following manner.
First, sin
e S2t´3 ´ a2t´2 “ 2t´1 ´ 3 ă 2t ´ 3 ă 22t´2 ` 2t´1 “ a2t´2 , by Claim 1 we have
2t ´ 3 „ S2t´3 ´ p2t ´ 3q “ 22t´2 .
Se
ond, sin
e S4t´7 ´ a4t´6 “ 22t´3 ´ 3 ă 22t´2 ă 24t´6 ` 22t´3 “ a4t´6 , by Claim 1 we have
2 2t´2
„ S4t´7 ´ 22t´2 “ 24t´6 ´ 3.
Therefore, 2t ´ 3 „ 22t´2 „ 24t´6 ´ 3, as required. l
Now it is easy to nd the required numbers. Indeed, the number 2 ´ 3 “ 5 “ a0 ` a1 is
3
Solution 2. We keep the notion of representability and the notation Sn from the previous
solution. We say that an index n is good if an writes as a sum of smaller terms from the
sequen
e a0 , a1 , . . .. Otherwise we say it is bad. We must prove that there are innitely many
good indi
es, as well as innitely many bad ones.
Lemma 1. If m ě 0 is an integer, then 4m is representable if and only if either of 2m ` 1 and
2m ` 2 is good.
Proof. The
ase m “ 0 is obvious, so we may assume that m ě 1. Let n “ 2m ` 1 or 2m ` 2.
Then n ě 3. We noti
e that
Sn´1 ă an´2 ` an .
The inequality writes as 2n ` 2rn{2s ` 2tn{2u ´ 3 ă 2n ` 2tn{2u ` 2n´2 ` 2tn{2u´1 , i.e. as 2rn{2s ă
2n´2 ` 2tn{2u´1 ` 3. If n ě 4, then n{2 ď n ´ 2, so rn{2s ď n ´ 2 and 2rn{2s ď 2n´2 . For n “ 3
the inequality veries separately.
If n is good, then an writes as an “ ai1 ` ¨ ¨ ¨ ` air , where r ě 2 and i1 ă ¨ ¨ ¨ ă ir ă n.
Then ir “ n ´ 1 and ir´1 “ n ´ 2, for if n ´ 1 or n ´ 2 is missing from the sequen
e i1 , . . . , ir ,
then ai1 ` ¨ ¨ ¨ ` air ď a0 ` ¨ ¨ ¨ ` an´3 ` an´1 “ Sn´1 ´ an´2 ă an . Thus, if n is good, then both
an ´ an´1 and an ´ an´1 ´ an´2 are representable.
We now
onsider the
ases n “ 2m ` 1 and n “ 2m ` 2 separately.
If n “ 2m ` 1, then an ´ an´1 “ a2m`1 ´ a2m “ p22m`1 ` 2m q ´ p22m ` 2m q “ 22m . So we
proved that, if 2m ` 1 is good, then 22m is representable. Conversely, if 22m is representable,
then 22m ă a2m , so 22m is a sum of some distin
t terms ai with i ă 2m. It follows that
a2m`1 “ a2m ` 22m writes as a2m plus a sum of some distin
t terms ai with i ă 2m. Hen
e
2m ` 1 is good.
If n “ 2m ` 2, then an ´ an´1 ´ an´2 “ a2m`2 ´ a2m`1 ´ a2m “ p22m`2 ` 2m`1 q ´ p22m`1 `
2m q ´ p22m ` 2m q “ 22m . So we proved that, if 2m ` 2 is good, then 22m is representable.
Conversely, if 22m is representable, then, as seen in the previous
ase, it writes as a sum of some
distin
t terms ai with i ă 2m. Hen
e a2m`2 “ a2m`1 ` a2m ` 22m writes as a2m`1 ` a2m plus a
sum of some distin
t terms ai with i ă 2m. Thus 2m ` 2 is good. l
Lemma 2. If k ě 2, then 24k´2 is representable if and only if 2k`1 is representable.
In parti
ular, if s ě 2, then 4s is representable if and only if 44s´3 is representable. Also,
4 4s´3
ą 4s .
Proof. We have 24k´2 ă a4k´2 , so in a representation of 24k´2 we
an have only terms ai with
i ď 4k ´ 3. Noti
e that
Hen
e, any representation of 24k´2 must
ontain all terms from a2k to a4k´3 . (If any of these
terms is missing, then the sum of the remaining ones is ď pa0 ` ¨ ¨ ¨ ` a4k´3 q ´ a2k ă 24k´2 .)
ř4k´3
Hen
e, if 2 4k´2
is representable, then 2 4k´2
´ i“2k ai is representable. But
4k´3
ÿ
24k´2 ´ ai “ 24k´2 ´ pS4k´3 ´ S2k´1 q “ 24k´2 ´ p24k´2 ` 22k ´ 3q ` p22k ` 2k`1 ´ 3q “ 2k`1 .
i“2k
Now 42 “ a2 `a3 is representable, whereas 46 “ 4096 is not. Indeed, note that 46 “ 212 ă a12 ,
so the only available terms for a representation are a0 , . . . , a11 , i.e., 2, 3, 6, 10, 20, 36, 72,
136, 272, 528, 1056, 2080. Their sum is S11 “ 4221, whi
h ex
eeds 4096 by 125. Then any
representation of 4096 must
ontain all the terms from a0 , . . . , a11 that are greater that 125,
i.e., 136, 272, 528, 1056, 2080. Their sum is 4072. Sin
e 4096 ´ 4072 “ 24 and 24 is
learly not
representable, 4096 is non-representable as well.
Starting with these values of m, by using Lemma 2, we
an obtain innitely many rep-
resentable powers of 4, as well as innitely many non-representable ones. By Lemma 1, this
solves our problem.
Shortlisted problems solutions 61
Let n ě k . Sin
e gcdpa1 {δn , an {δn , an`1 {δn q “ 1, it follows by (2) that gcdpa1 {δn , an {δn q “ 1.
Let dn “ gcdpa1 , an q. Then dn “ δn ¨ gcdpa1 {δn , an {δn q “ δn , so dn divides an`1 , and therefore
dn divides dn`1 .
Consequently, from some rank on, the dn form a nonde
reasing sequen
e of integers not
ex
eeding a1 , so dn “ d for all n ě ℓ, where ℓ is some positive integer.
Finally, sin
e gcdpa1 {d, an`1 {dq “ 1, it follows by (1) that an`1 {d divides an {d, so an ě an`1
for all n ě ℓ. The
on
lusion follows.
Solution 2. We use the same notation sn . This time, we explore the exponents of primes in
the prime fa
torizations of the an for n ě k .
To start, for every n ě k , we know that the number
an an`1 an
sn`1 ´ sn “ ` ´ p˚q
an`1 a1 a1
Comment. Given any positive odd integer m,
onsider the m-tuple p2, 22 , . . . , 2m´1 , 2m q. Appending
an innite string of 1's to this m-tuple yields an eventually
onstant sequen
e of integers satisfying
the
ondition in the statement, and shows that the rank from whi
h the sequen
e stabilises may be
arbitrarily large.
There are more sophisti
ated examples. The solution to part (b) of 10532, Amer. Math. Monthly,
Vol. 105 No. 8 (O
t. 1998), 775777 (available at https://www.jstor.org/stable/2589009), shows
that, for every integer m ě 5, there exists an m-tuple pa1 , a2 , . . . , am q of pairwise distin
t positive
integers su
h that gcdpa1 , a2 q “ gcdpa2 , a3 q “ ¨ ¨ ¨ “ gcdpam´1 , am q “ gcdpam , a1 q “ 1, and the sum
a1 {a2 ` a2 {a3 ` ¨ ¨ ¨ ` am´1 {am ` am {a1 is an integer. Letting am`k “ a1 , k “ 1, 2, . . . , extends su
h an
m-tuple to an eventually
onstant sequen
e of positive integers satisfying the
ondition in the statement
of the problem at hand.
Here is the example given by the proposers of 10532. Let b1 “ 2, let bk`1 “ 1 ` b1 ¨ ¨ ¨ bk “
1 ` bk pbk ´ 1q, k ě 1, and set Bm “ b1 ¨ ¨ ¨ bm´4 “ bm´3 ´ 1. The m-tuple pa1 , a2 , . . . , am q dened below
satises the required
onditions:
xy ´ zt “ x ` y “ z ` t. p˚q
s2 “ a2 ` b2 “ c2 ` d2 (1)
and
2s “ a2 ´ c2 “ d2 ´ b2 (2)
(the last equality in (2) follows from (1)). We readily get from (2) that a, d ą 0.
In the sequel we will use only the relations (1) and (2), along with the fa
t that a, d, s
are positive integers, while b and c are nonnegative integers, at most one of whi
h may be
zero. Sin
e both relations are symmetri
with respe
t to the simultaneous swappings a Ø d
and b Ø c, we assume, without loss of generality, that b ě c (and hen
e b ą 0). Therefore,
d2 “ 2s ` b2 ą c2 , when
e
c2 ` d 2 s2
d2 ą “ . (3)
2 2
On the other hand, sin
e d2 ´ b2 is even by (2), the numbers b and d have the same parity,
so 0 ă b ď d ´ 2. Therefore,
s
2s “ d2 ´ b2 ě d2 ´ pd ´ 2q2 “ 4pd ´ 1q, i.e., dď ` 1. (4)
2
Combining (3) and (4) we obtain
´s ¯2
2s2 ă 4d2 ď 4 `1 , or ps ´ 2q2 ă 8,
2
whi
h yields s ď 4.
Finally, an easy
he
k shows that ea
h number of the form s2 with 1 ď s ď 4 has a unique
representation as a sum of two squares, namely s2 “ s2 ` 02 . Thus, (1) along with a, d ą 0
imply b “ c “ 0, whi
h is impossible.
Solution 2. We start with a
omplete des
ription of all 4-tuples px, y, z, tq of positive integers
satisfying p˚q. As in the solution above, we noti
e that the numbers
2s “ xy ´ zt “ ps ` pqps ´ pq ´ ps ` qqps ´ qq “ q 2 ´ p2 ,
q 2 ´p2
Set now k “ q´p
2
, ℓ“ q`p
2
. Then we have s “ 2
“ 2kℓ and hen
e
x “ s ` p “ 2kℓ ´ k ` ℓ, y “ s ´ p “ 2kℓ ` k ´ ℓ,
(5)
z “ s ` q “ 2kℓ ` k ` ℓ, t “ s ´ q “ 2kℓ ´ k ´ ℓ.
Re
all here that ℓ ě k ą 0 and, moreover, pk, ℓq ‰ p1, 1q, sin
e otherwise t “ 0.
Assume now that both xy and zt are squares. Then xyzt is also a square. On the other
hand, we have
sin
e ℓ ě 2 and k ě 1. Thus pD ´ 1q2 ă xyzt ă D 2 , and xyzt
annot be a perfe
t square; a
ontradi
tion.
Comment. The rst part of Solution 2 shows that all 4-tuples of positive integers x ě y , z ě t
satisfying p˚q have the form (5), where ℓ ě k ą 0 and ℓ ě 2. The
onverse is also true: every pair
of positive integers ℓ ě k ą 0, ex
ept for the pair k “ ℓ “ 1, generates via (5) a 4-tuple of positive
integers satisfying p˚q.
66 Cluj-Napo
a Romania, 314 July 2018
N6. Let f : t1, 2, 3, . . .u Ñ t2, 3, . . .u be a fun
tion su
h that f pm ` nq | f pmq ` f pnq for all
pairs m, n of positive integers. Prove that there exists a positive integer c ą 1 whi
h divides
all values of f .
(Mexi
o)
Solution 1. For every positive integer m, dene Sm “ tn : m | f pnqu.
Lemma. If the set Sm is innite, then Sm “ td, 2d, 3d, . . .u “ d ¨ Zą0 for some positive integer d.
Proof. Let d “ min Sm ; the denition of Sm yields m | f pdq.
Whenever n P Sm and n ą d, we have m | f pnq | f pn ´ dq ` f pdq, so m | f pn ´ dq and
therefore n ´ d P Sm . Let r ď d be the least positive integer with n ” r pmod dq; repeating
the same step, we
an see that n ´ d, n ´ 2d, . . . , r P Sm . By the minimality of d, this shows
r “ d and therefore d | n.
Starting from an arbitrarily large element of Sm , the pro
ess above rea
hes all multiples
of d; so they all are elements of Sm . l
The solution for the problem will be split into two
ases.
Case 1: The fun
tion f is bounded.
Call a prime p frequent if the set Sp is innite, i.e., if p divides f pnq for innitely many
positive integers n; otherwise
all p sporadi
. Sin
e the fun
tion f is bounded, there are only
a nite number of primes that divide at least one f pnq; so altogether there are nitely many
numbers n su
h that f pnq has a sporadi
prime divisor. Let N be a positive integer, greater
than all those numbers n.
Let p1 , . . . , pk be the frequent primes. By the lemma we have Spi “ di ¨ Zą0 for some di .
Consider the number
n “ Nd1 d2 ¨ ¨ ¨ dk ` 1.
Due to n ą N , all prime divisors of f pnq are frequent primes. Let pi be any frequent prime
divisor of f pnq. Then n P Spi , and therefore di | n. But n ” 1 pmod di q, whi
h means di “ 1.
Hen
e Spi “ 1 ¨ Zą0 “ Zą0 and therefore pi is a
ommon divisor of all values f pnq.
Case 2: f is unbounded.
We prove that f p1q divides all f pnq.
Let a “ f p1q. Sin
e 1 P Sa , by the lemma it su
es to prove that Sa is an innite set.
` ˘
Call a positive integer p a peak if f ppq ą max f p1q, . . . , f pp ´ 1q . Sin
e f is not bounded,
there are innitely many peaks. Let 1 “ p1 ă p2 ă . . . be the sequen
e of all peaks, and let
hk “ f ppk q. Noti
e that for any peak pi and for any k ă pi , we have f ppi q | f pkq ` f ppi ´ kq ă
2f ppi q, hen
e
f pkq ` f ppi ´ kq “ f ppi q “ hi . p1q
By the pigeonhole prin
iple, among the numbers h1 , h2 , . . . there are innitely many that
are
ongruent modulo a. Let k0 ă k1 ă k2 ă . . . be an innite sequen
e of positive integers
su
h that hk0 ” hk1 ” . . . pmod aq. Noti
e that
Comment. As an extension of the solution above, it
an be proven that if f is not bounded then
f pnq “ an with a “ f p1q.
Take an arbitrary positive integer n; we will show that f pn ` 1q “ f pnq ` a. Then it follows by
indu
tion that f pnq “ an.
Shortlisted problems solutions 67
On the other hand, there exists a wide family of bounded fun
tions satisfying the required proper-
ties. Here we present a few examples:
# #
2c if n is even 2018c if n ď 2018
f pnq “ c; f pnq “ f pnq “
c if n is odd; c if n ą 2018.
` ˘
Solution 2. Let dn “ gcd f pnq, f p1q . From` dn`1 | f p1q ˘ and dn`1 | f pn ` 1q | f pnq ` f p1q,
we
an see that dn`1 | f pnq; then dn`1 | gcd f pnq, f p1q “ dn . So the sequen
e d1 , d2 , . . .
is nonin
reasing in the sense that every element ` is a divisor
˘ of the previous elements. Let
d “ minpd1 , d2 , . . .q “ gcdpd1 .d2 , . . .q “ gcd f p1q, f p2q, . . . ; we have to prove d ě 2.
For the sake of
ontradi
tion, suppose that the statement is wrong, so d “ 1; that means
there is some index n0 su
h that dn “ 1 for every n ě n0 , i.e., f pnq is
oprime with f p1q.
Claim 1. If 2k ě n0 then f p2k q ď 2k .
Proof. By the
ondition, f p2nq | 2f pnq; a trivial indu
tion yields f p2k q | 2k f p1q. If 2k ě n0 then
f p2k q is
oprime with f p1q, so f p2k q is a divisor of 2k . l
Claim 2. There is a
onstant C su
h that f pnq ă n ` C for every n.
Proof. Take the rst power of 2 whi
h is greater than or equal to n0 : let K “ 2k ě n0 . By
Claim 1, we have f pKq ď K . Noti
e that f pn ` Kq | f pnq ` f pKq implies f pn ` Kq ď
f pnq ` f pKq ď f pnq ` K . If n “ tK ` r for some t ě 0 and 1 ď r ď K , then we
on
lude
` ˘
f pnq ď K ` f pn ´ Kq ď 2K ` f pn ´ 2Kq ď . . . ď tK ` f prq ă n ` max f p1q, f p2q, . . . , f pKq ,
` ˘
so the
laim is true with C “ max f p1q, . . . , f pKq . l
` ˘
Claim 3. If a, b P Zą0 are
oprime then gcd f paq, f pbq | f p1q. In parti
ular, if a, b ě n0 are
oprime then f paq and f pbq are
oprime.
` ˘
Proof. Let d “ gcd f paq, f pbq . We
an repli
ate Eu
lid's algorithm. Formally, apply indu
tion
on a ` b. If a “ 1 or b “ 1 then we already have d | f p1q.
Without loss of generality, suppose` 1 ă a ă b. Then˘ d | f paq and d | f pbq | f paq ` f pb ´ aq,
so d | f pb´aq. Therefore d divides gcd f paq, f pb´aq whi
h is a divisor of f p1q by the indu
tion
hypothesis. l
Let p1 ă p2 ă . . . be the sequen
e of all prime numbers; for every k , let qk be the lowest
power of pk with qk ě n0 . (Noti
e that there are only nitely many positive integers with
qk ‰ pk .)
Take a positive integer N , and
onsider the numbers
f p1q, f pq1 q, f pq2 q, . . . , f pqN q.
Here we have N ` 1 numbers, ea
h being greater than 1, and they are pairwise
oprime by
Claim 3. Therefore, they have at least N ` 1 dierent prime divisors in total, and their greatest
prime divisor is at least pN `1 . Hen
e, maxpf p1q, f pq1q, . . . , f pqN qq ě pN `1 .
Choose N su
h that maxpq1 , . . . , qN q “ pN (this is a
hieved if N is su
iently large), and
pN `1 ´ pN ą C (that is possible, be
ause there are arbitrarily long gaps between the primes).
Then we establish a
ontradi
tion
pN `1 ď maxpf p1q, f pq1 q, . . . , f pqN qq ă maxp1 ` C, q1 ` C, . . . , qN ` Cq “ pN ` C ă pN `1
whi
h proves the statement.
68 Cluj-Napo
a Romania, 314 July 2018
Claim 3. For every 0 ď k ď n ´ 30, among the denominators bk`1 , bk`2 , . . . , bk`30 , at least
ϕp30q “ 8 are divisible by d.
Proof. By Claim 1, the 2-wrong, 3-wrong and 5-wrong indi
es
an be
overed by three arithmeti
progressions with dieren
es 2, 3 and 5. By a simple in
lusion-ex
lusion, p2´1q¨p3´1q¨p5´1q “ 8
indi
es are not
overed; by Claim 2, we have d | bi for every un
overed index i. l
( ´Y n ] ¯ ´n ¯ n´2
5n ě max bi : d | bi ě ¨8 ¨dą ´1 ¨8¨ ą 5n.
30 30 20
Comment 1. It is possible that all terms in (1) are equal, for example with ai “ 2i´ 1 and bi “ 4i´ 2
ai 1
we have
bi “ 2.
Comment 2. The bound 5n in the statement is far from sharp; the solution above
an be modied
3
to work for 9n. For large n, the bound 5n
an be repla
ed by n 2 ´ε .